Download as pdf or txt
Download as pdf or txt
You are on page 1of 508

D:\EG_Physics-12_(26-06-2022)\Open_Files\0_Prelims\0_Prelims

\ 26-Jul-2022 Ved_Goswami Proof-3 Reader’s Sign _______________________ Date __________

12
CBSE
2022-23

PHYSICS
(As per the Latest Syllabus issued by CBSE on April 21, 2022)

Pushpa Vati Tyagi


M.Sc (Physics)
B.Ed, M.Ed, PGT (Physics)
Ex: HOD, Physics Deptt.
Sanskriti School, Delhi

Full Marks Pvt Ltd


(Progressive Educational Publishers)
New Delhi-110002
D:\EG_Physics-12_(26-06-2022)\Open_Files\0_Prelims\0_Prelims
\ 26-Jul-2022 Ved_Goswami Proof-3 Reader’s Sign _______________________ Date __________

Published by:

9, Daryaganj, New Delhi-110002


Phone: 011- 40556600 (100 Lines)
Website: www.fullmarks.org
E-mail: info@fmpl.in

© Publishers

All rights reserved. No part of this publication may be reproduced or transmitted, in any form or by any means, without permission. Any person
who does any unauthorised act in relation to this publication may be liable to criminal prosecution and civil claims for damages.

Branches:
• Chennai • Guwahati

Marketing Offices:
• Ahmedabad • Bengaluru • Bhopal • Dehradun • Hyderabad • Jaipur • Jalandhar • Kochi • Kolkata
• Lucknow • Mumbai • Patna • Ranchi

NEW EDITION

“This book is meant for educational and learning purposes. The author(s) of the book has/have taken all reasonable care to ensure that the contents
of the book do not violate any existing copyright or other intellectual property rights of any person in any manner whatsoever. In the event the
author(s) has/have been unable to track any source and if any copyright has been inadvertently infringed, please notify the publisher in writing for
corrective action.”
D:\EG_Physics-12_(26-06-2022)\Open_Files\0_Prelims\0_Prelims
\ 26-Jul-2022 Ved_Goswami Proof-3 Reader’s Sign _______________________ Date __________

Preface

Physics-12 is based on the latest curriculum guidelines specified by the CBSE. It will
certainly prove to be a torch-bearer for those who toil hard to achieve their goal.

This All-in-one Question Bank has been developed keeping in mind all the requirement of the students for
Board Examinations preparations like learning, practicing, revising and assessing.
Salient Features of the Book:
●● Each chapter is designed in ‘Topic wise’ manner where each topic is briefly explained with sufficient
Examples and Exercise. Exercise which covers Objective Type Questions and all the possible variety
of Questions.
●● Answers with hints are provided separately after the exercise.
●● Importance of Each Topic and Frequently Asked Types of Questions provides an idea to the students on
which type they should focus more.
●● Assignment is provided at the end of each chapter.
●● Previous years’ Board Questions have been covered in every chapter.
●● 1 solved and 2 unsolved Sample Papers are given for self assessment.
●● Common Errors by the students are provided to make students aware what errors are usually done
unknowingly.
●● The book has been well prepared to build confidence in students.
Suggestions for further improvement of the book, pointing out printing errors/mistakes which might have
crept in spite of all efforts, will be thankfully received and incorporated in the next edition.
—Publisher

(iii)
D:\EG_Physics-12_(26-06-2022)\Open_Files\0_Prelims\0_Prelims
\ 26-Jul-2022 Ved_Goswami Proof-3 Reader’s Sign _______________________ Date __________

Syllabus
Time : 3 Hours  Max. Marks : 70
No. of Periods Marks
Unit–I Electrostatics
Chapter–1. Electric Charges and Fields
26
Chapter–2. Electrostatic Potential and Capacitance 16
Unit–II Current Electricity
18
Chapter–3. Current Electricity
Unit–III Magnetic Effects of Current and Magnetism
Chapter–4. Moving Charges and Magnetism 25
Chapter–5. Magnetism and Matter
17
Unit–IV Electromagnetic Induction and Alternating Currents
Chapter–6. Electromagnetic Induction 24
Chapter–7. Alternating Current
Unit–V Electromagnetic Waves
04
Chapter–8. Electromagnetic Waves

Unit–VI Optics 18
Chapter–9. Ray Optics and Optical Instruments 30
Chapter–10. Wave Optics
Unit–VII Dual Nature of Radiation and Matter
8
Chapter–11. Dual Nature of Radiation and Matter
Unit–VIII Atoms and Nuclei 12
Chapter–12. Atoms 15
Chapter–13. Nuclei
Unit–IX Electronic Devices
Chapter–14. Semiconductor Electronics: Materials, 10 7
Devices and Simple Circuits
Total 160 70

Unit I: Electrostatics 26 Periods


Chapter–1. Electric Charges and Fields
Electric charges, Conservation of charge, Coulomb’s law-force between two-point charges, forces between multiple
charges; superposition principle and continuous charge distribution.
Electric field, electric field due to a point charge, electric field lines, electric dipole, electric field due to a dipole,
torque on a dipole in uniform electric field.
Electric flux, statement of Gauss’s theorem and its applications to find field due to infinitely long straight wire,
uniformly charged infinite plane sheet and uniformly charged thin spherical shell (field inside and outside).
Chapter–2. Electrostatic Potential and Capacitance
Electric potential, potential difference, electric potential due to a point charge, a dipole and system of charges;
equipotential surfaces, electrical potential energy of a system of two-point charges and of electric dipole in an
electrostatic field.

(iv)
D:\EG_Physics-12_(26-06-2022)\Open_Files\0_Prelims\0_Prelims
\ 26-Jul-2022 Ved_Goswami Proof-3 Reader’s Sign _______________________ Date __________

Conductors and insulators, free charges and bound charges inside a conductor. Dielectrics and electric polarization,
capacitors and capacitance, combination of capacitors in series and in parallel, capacitance of a parallel plate
capacitor with and without dielectric medium between the plates, energy stored in a capacitor (no derivation,
formulae only).
Unit II: Current Electricity 18 Periods
Chapter–3. Current Electricity
Electric current, flow of electric charges in a metallic conductor, drift velocity, mobility and their relation with
electric current; Ohm’s law, V-I characteristics (linear and non-linear), electrical energy and power, electrical
resistivity and conductivity, temperature dependence of resistance, Internal resistance of a cell, potential difference
and emf of a cell, combination of cells in series and in parallel, Kirchhoff’s rules, Wheatstone bridge.
Unit III: Magnetic Effects of Current and Magnetism 25 Periods
Chapter–4. Moving Charges and Magnetism
Concept of magnetic field, Oersted’s experiment.
Biot - Savart law and its application to current carrying circular loop.
Ampere’s law and its applications to infinitely long straight wire. Straight solenoid (only qualitative treatment),
force on a moving charge in uniform magnetic and electric fields.
Force on a current-carrying conductor in a uniform magnetic field, force between two parallel current-carrying
conductors-definition of ampere, torque experienced by a current loop in uniform magnetic field; Current loop as a
magnetic dipole and its magnetic dipole moment, moving coil galvanometer-its current sensitivity and conversion
to ammeter and voltmeter.
Chapter–5. Magnetism and Matter
Bar magnet, bar magnet as an equivalent solenoid (qualitative treatment only), magnetic field intensity due to a
magnetic dipole (bar magnet) along its axis and perpendicular to its axis (qualitative treatment only), torque on a
magnetic dipole (bar magnet) in a uniform magnetic field (qualitative treatment only), magnetic field lines.
Magnetic properties of materials- Para-, dia- and ferro - magnetic substances with examples, Magnetization of
materials, effect of temperature on magnetic properties.
Unit IV: Electromagnetic Induction and Alternating Currents 24 Periods
Chapter–6. Electromagnetic Induction
Electromagnetic induction; Faraday’s laws, induced EMF and current; Lenz’s Law, Self and mutual
induction.
Chapter–7. Alternating Current
Alternating currents, peak and RMS value of alternating current/voltage; reactance and impedance; LCR
series circuit (phasors only), resonance, power in AC circuits, power factor, wattless current.
AC generator, Transformer.
Unit V: Electromagnetic waves 04 Periods
Chapter–8. Electromagnetic Waves
Basic idea of displacement current, Electromagnetic waves, their characteristics, their transverse nature
(qualitative idea only).
Electromagnetic spectrum (radio waves, microwaves, infrared, visible, ultraviolet, X-rays, gamma rays)
including elementary facts about their uses.
Unit VI: Optics 30 Periods
Chapter–9: Ray Optics and Optical Instruments
Ray Optics: Reflection of light, spherical mirrors, mirror formula, refraction of light, total internal reflection and
optical fibers, refraction at spherical surfaces, lenses, thin lens formula, lens maker’s formula, magnification, power
of a lens, combination of thin lenses in contact, refraction of light through a prism.

Optical instruments: Microscopes and astronomical telescopes (reflecting and refracting) and their magnifying
powers.

(v)
D:\EG_Physics-12_(26-06-2022)\Open_Files\0_Prelims\0_Prelims
\ 26-Jul-2022 Ved_Goswami Proof-3 Reader’s Sign _______________________ Date __________

Chapter–10. Wave Optics


Wave optics: Wave front and Huygen’s principle, reflection and refraction of plane wave at a plane surface using
wave fronts. Proof of laws of reflection and refraction using Huygen’s principle. Interference, Young’s double slit
experiment and expression for fringe width (No derivation final expression only), coherent sources and sustained
interference of light, diffraction due to a single slit, width of central maxima (qualitative treatment only).

Unit VII: Dual Nature of Radiation and Matter 08 Periods


Chapter–11. Dual Nature of Radiation and Matter
Dual nature of radiation, Photoelectric effect, Hertz and Lenard’s observations; Einstein’s photoelectric equation-
particle nature of light.
Experimental study of photoelectric effect
Matter waves-wave nature of particles, de-Broglie relation.

Unit VIII: Atoms and Nuclei 15 Periods


Chapter–12. Atoms
Alpha-particle scattering experiment; Rutherford’s model of atom; Bohr model of hydrogen atom, Expression
for radius of nth possible orbit, velocity and energy of electron in his orbit, of hydrogen line spectra (qualitative
treatment only).
Chapter–13. Nuclei
Composition and size of nucleus, nuclear force
Mass-energy relation, mass defect; binding energy per nucleon and its variation with mass number; nuclear fission,
nuclear fusion.

Unit IX: Electronic Devices 10 Periods


Chapter–14. Semiconductor Electronics: Materials, Devices and Simple Circuits
Energy bands in conductors, semiconductors and insulators (qualitative ideas only) Intrinsic and extrinsic
semiconductors- p and n type, p-n junction
Semiconductor diode - I-V characteristics in forward and reverse bias, application of junction diode -diode as a
rectifier.

(vi)
D:\EG_Physics-12_(26-06-2022)\Open_Files\0_Prelims\0_Prelims
\ 26-Jul-2022 Ved_Goswami Proof-3 Reader’s Sign _______________________ Date __________

Question Paper Design


Theory
Max. Marks : 70 Time : 3 Hours
Approximate
S.No. Typology of Questions Total Marks
Percentage
1. Remembering: Exhibit memory of previously learned material by recalling
facts, terms, basic concepts, and answers.
Understanding: Demonstrate understanding of facts and 27 38%
ideas by organizing, comparing, translating, interpreting, giving descriptions,
and stating main ideas.
2. Applying: Solve problems to new situations by applying acquired knowledge,
22 32%
facts, techniques and rules in a different way.
3. Analysing: Examine and break information into parts by identifying motives or
causes. Make inferences and find evidence to support generalizations
Evaluating: Present and defend opinions by making judgments about
information, validity of ideas, or quality of work based on a set of criteria. 21 30%
Creating: Compile information together in a different way by combining
elements in a new pattern or proposing
alternative solutions.
TOTAL 70 100%

Practical: 30 Marks

Note:
1. The above template is only a sample. Suitable internal variations may be made for generating similar templates
keeping the overall weightage to different form of questions and typology of questions same.
2. For more details kindly refer to Sample Question Paper of class XII for the year 2022-23 to be published by CBSE at
its website

(vii)
Some biomolecules of proteins behaves as an ideal
D:\EG_Physics-12_(26-06-2022)\Open_Files\0_Prelims\0_Prelims electric dipole. = (q × 2a) E sinq

\ 26-Jul-2022 Ved_Goswami Proof-3 (b) E Reader’s Sign _______________________
or, τ = pEsin q Date __________
  
In vector form, τ = p × E

2a (c) (i) given: E = 105 N / C
F= qE
B (+ q) 2a = 2cm
q
q = 60°
(– q) O q
τ = 8 3 Nm


A F= – qE C

  τ = pE sin q
E
8 3 = 2a × q × 105 sin 60°
3
OVERVIEW OF A CHAPTER
p (Dipole moment)
8 3 = 2 × 10 −2 × q × 105 ×
2
\ q = 8 × 10–3C

\ 25-Jul-2022 Ved_Goswami
Physics XII
D:\EG_Physics-12_(26-06-2022)\Open_Files\Ch-1\Ch-1
Proof-3 Reader’s Sign _______________________ Date __________


 (torque)
(ii) Potential energy, P.E = –pE cos q
= – 2a × q × E × cos 60°
1
Since E is uniform, net force on the dipole is: = – 8 × 10–3 × 2 × 10–2 ×105×
2
(qE – qE) = 0 \ P.E = –8J

Topic 3. Gauss’s Law and its Applications

Topics covered
1 Electric Charges and Fields • Electric flux: It is the measure of number of electric
field lines crossing that area, normally.
→ →
f=
s
→ →
∫ E ⋅ dS =
qenclosed
ε0
Mathematically df = E ⋅ dS
1.1 Electric Charges and Electric Fields 1.2 Electric Dipole • Gaussian surface: Any closed surface imagined
Electric flux is a scalar quantity. S.I. unit of electric around the charge distribution, so that Gauss theorem
1.3 Gauss’s Law and its Applications flux is Nm2C–1 or Vm. can be conveniently applied to find electric field due to
Gauss Law the given charge distribution.

C hapter map • Gauss theorem: The total flux through a closed


surface, enclosing a volume, in vacuum is,
1
times
• Electric field due to infinitely long straight charged wire
of linear charge density λ:
Combination of charges Charge (Coulomb) Distribution of charges the net charge, enclosed by the surface. ε0 λ
E= , where r is the perpendicular distance of

E 2πε 0 r
the observation point from the wire.

dS
n
 σ.ds ρ.dV
 ∑ qi Electric Field ∫ | r |2 × r or k ∫ | r |2 × r
E = k
E nt
E = k i =1 2 × r s V S
| ri | q
Spherical
Gaussian +++++++
 1 qQ  surface
F= r Field Intensity Electric Flux  

4πε0 r 2  d φ = E.ds
  E (N/C) (Nm2/C)
F = Eq ElEctric chargEs and FiElds 27

Gauss’s Theorem λ
 n Electric field due to E=
1 Q 2πε0 r
E=
4πε0 r 2
r ∑ qi charge distributions
i =1
φ =
closed ε0
surface σ
E=
2ε0

✎ All concepts are presented in points, which


Topic 1. Electric Charges and Electric Fields can be easily learnt and remember.
• Electrostatics: It is the branch of physics which deals (ii) By induction: It is the process of charging an
with the forces between the charges at rest, electric fields
and potentials due to the charges at rest.
uncharged body, by keeping a charged body near it,
without making a physical contact.
✎ Each concept is well explain by relevant
• Electric charge: It is an intrinsic property of elementary
In this process the charge produced on the body, is
opposite to the body, used to charge it.
diagrams, tables and illustrations for better
particles like electrons and protons because of which
they exert electrical force and are able to respond to
(iii) By conduction: It is also called charging by physical
contact. If an uncharged body in brought in, electrical
understanding.
electrical force. Electric charge is a scalar quantity.
contact, with the charged body, some electrons, are
✎ Each chapter is divided into topics and
There are two kind of charges (a) Positive charge,
(b) Negative charge. Like charges repel each other but
shared between the bodies. The uncharged body
acquire a charge similar to the charged body.
each concept is dealt separately.
unlike charges attract each other.
• Method of producing charge on a body
• Conductors: The substances through which electric
charges can flow easily are called conductors. They
(i) By ✎  Flowchart
friction: When representation
we rub two bodies together,
D:\EG_Physics-12_(26-06-2022)\Open_Files\Ch-8\Ch-8
there
is, transfer of electrons from one body to another,
of the chapter. contain large number of free electrons. e.g. silver is the
best conductor of electricity.
\ 26-Jul-2022 Ved_Goswami Proof-3 Reader’s Sign _______________________ Date __________
which causes the charging of bodies. By friction, both • Insulators: The substances that do not allow the charges
bodies acquire equal but opposite kind of charge. to flow through them are called insulators. e.g. plastic,
wood.
11

2. Produced by accelerated charged particles. place in space. Ratio of electric and magnetic fields is
3. Obey principle of superposition. always constant and is equal to speed of EM waves
4. Energy in EM waves is divided equally between in that medium.
electric and magnetic fields. i.e., E0 = c
1 B0
5. In free space, they travel with a velocity c =
µ0 ε0 7. E.M. waves transport linear momentum and energy
where m0 = absolute permeability. as they travel through space. Momentum
e0 = absolute permittivity of the medium. U
1
p= , U → energy carried
In material medium, speed is v = c
µε 8. E.M. waves are not deflected by electric or magnetic
where e = permittivity of that medium. field.
6. Variation in amplitudes of electric and magnetic fields 9. The electric field of an e.m. wave is responsible for
in EM waves takes place at the same time and at same its optical effects, because E0 >> B0.

EXERCISE 8.2
I. Objective Type Questions (1 Mark) For questions numbers 4 and 5, two statements are
1. Choose the correct answers from the given options given-one labelled Assertion (A) and the other labelled
(MCQs). Reason (R). Select the correct answer to these questions
(i) A linearly polarized electromagnetic wave given from the codes (a), (b), (c) and (d) as given below.
(a) Both A and R are true and R is the correct
as E = E0 i cos(kz – wt) is incident normally on a
explanation of A
perfectly reflecting infinite wall at z = a. Assuming
(b) Both A and R are true but R is NOT the correct
that the material of the wall is optically inactive,
the reflected wave will be given as explanation of A ✎ Exercise to each topic has been dealt
[NCERT Exemplar]
(c) A is true but R is false separately and important NCERT Textual
(d) A is false and R is also false
(a) E = –E i c os(kz – wt).
r 0 4. Assertion (A): Infrared radiations play an important and NCERT Exemplar Questions included,
(b) Er = E0 i cos(kz + wt). role in maintaining the average temperature of earth.
segregated into 1 Mark, 2 Marks, 3 Marks
Reason (R): Infrared radiations are sometimes
(c) E = –E i cos(kz + wt).
r 0
referred to as heat waves. and 5 Marks Questions. Also included HOTS
(d) Er = E0 i cos(kz – wt). 5. Assertion (A): When a charged particle moves in a
(ii) Light with an energy flux of 20 W/cm2 falls on a circular path, it produces electromagnetic waves.
Questions that test the mental ability of the
non-reflecting surface at normal incidence. If the
surface has an area of 30 cm2, the total momentum
Reason (R): Charge particle has acceleration. learner.
II. Very Short Answer Type Questions (1 Mark)
delivered (for complete absorption) during 30
1. Out of microwaves, ultraviolet rays and infrared rays, 
minutes is [NCERT Exemplar]
(a) 36 × 10–5 kg m/s. (b)36 × 10–4 kg m/s. which radiations will be most effective for emission
of electrons from a metallic surface?
(c) 108 × 104 kg m/s. (d)1.08 × 107 kg m/s.
2. What is the evidence that shows that sound is not an
2. Fill in the blanks.
e.m. wave?
(i) .................... are transverse in nature.
3. Give the ratio of velocities of light rays of wavelength
(ii) .................... is responsible for the optical effects
4000Å and 8000Å in vacuum.
due to an electromagnetic radiation and hence it
is also called as ....................... . 4. The wavelength of electromagnetic radiation is
doubled. What will happen to the energy of the
3. State True or False
photon?
(i) The speed of electromagnetic waves in a medium
5. What are the direction of electric and magnetic field
1
is . vectors relative to each other and relative to the
µ0 ε0 direction of propagation of electromagnetic waves?
(ii) Normal light is due to magnetic field oscillations. [AI 2012]

268 Physics–12

(viii)
D:\EG_Physics-12_(26-06-2022)\Open_Files\0_Prelims\0_Prelims
\ 26-Jul-2022 Ved_Goswami Proof-3 Reader’s Sign _______________________ Date __________

D:\EG_Physics-12_(26-06-2022)\Open_Files\Ch-1\Ch-1
\ 26-Jul-2022 Ved_Goswami Proof-3 Reader’s Sign _______________________ Date __________

D:\EG_Physics-12_(26-06-2022)\Open_Files\Ch-1\Ch-1
\ 26-Jul-2022 Ved_Goswami Proof-3 Reader’s Sign _______________________ Date __________

REVISION CHART

Relative permitivity is ratio of


Quantization of electric charge: force b/w two point charges placed
certain distance apart in vacuum to Insulators: The substances
IMPORTANT FORMULAE The charge on a body, is an integral
multiple of, a basic quantity of the force b/w the same two charges that do not allow the charges
charge, which is the charge on an placed some distance apart in the to flow through them are
Formulae Symbols Applications electron. given medium. called insulators.

1. q = ± Ne q = Charge, N = number of Charge Quantization of charges


particle
2. q1q2 q1, q2 are point charges, To find force between two point charges
F= k F = Force Electric charge
r2 It is an intrinsic property of elementary particles like electrons and protons because of which they exert electrical force and are
able to respond to electrical force. Electric charge is a scalar quantity.
3. F = qE E = Electric Field Relation between F and E
4. q 1 Electric field due to a point charge
E= k k= , r = distance
r2 4πε 0
e0 = Absolute permittivity Methods of producing charge on Additivity of electrical charges: Conservation of charge: The
an body by The total charge on a system is the net electric charge in an isolated
5.   dS = small area To find electric flux (i) friction algebraic sum of all the charges in system is always constant.
∫ E ⋅ dS = E ∫ dS cos θ
φE =  (ii) Induction different parts of the body.
(iii) Conduction
6. Σq f = Electric flux Gauss theorem
φE = Conductors: The substances through
Sq = Net Charge enclosed within the
ε0 which electric charges can flow easily
surface are called conductors. They contain
large number of free electrons. Electric field: Electric field
7. λ l = linear charge density Field intensity due to infinitely long 1 Coulomb: It is that amount of charge e.g. silver is the best conductor of intensity/strength is defined as the
E= straight uniformly charged wire that repels an equal and similar charge electricity. force experienced per unit +ve
2πε 0 R with a force of 9 × 109 N, when placed ε m F0 test charge (vanishingly small)
εr = =
in vacuum at a distance of 1 m from it. ε 0 Fm placed at that point.
8. q r = radius of Gaussian surface (outside Field intensity due to uniformly
(a) outside the shell: E = k 2
r the shell) charged spherical shell
(b) on the shell:
R = Radius of shell
q 2
E = k 2 As q = 4πR σ s = surface charge density
R D:\EG_Physics-12_(26-06-2022)\Open_Files\Ch-2\Ch-2 Coulomb’s Law of Electrostatic
E = σ / ε0 The force ofVed_Goswami
\ 25-Jul-2022
attraction/repulsion
Proof-3
between two stationary point charges is directly proportional to the product of the magnitude of
Reader’s Sign _______________________ Date __________

the charges and inversely proportional to the square of the distance between them.
(c) inside the shell: E = 0
9. σ E = Electric field Field intensity due to thin infinite plane
(a) E = e0 = electric permittivity sheet of charge
2ε 0
(b) E. field at the equator of a dipole Electric Field Lines: It is a curve, Properties of energy
electric stored
field lines:
U = Electrostatic in
1 p (c) E. field of the axis of a dipole 8. U =to1 which
the tangent
Q2 1
= at CVany
2 1
= point
QV (i) Continuous smooth curves Energy stored in a capacitor
capacitor
(b) E equator = 2 C 2 2
gives the direction of force acting on (ii) start from positive charge and end at a negative charge
4πε 0 r 3
a test charge placed at that point. (iii) they never intersect each other
q1q2 U = Potential Energy Potential energy of a system of two point
1 2p 9. U= W = k (iv) do not form closed loop
W = Work done charges
(c) E axis = r12 (v) they are always normal to the surface of conductor
4πε 0 r 3 (vi) they do not pass through a conductor
1 E = Electric field strength
10. U= ε E2 Energy density of a parallel plate capacitor
10. t = p × E = pE sin q p = dipole moment Electric dipole in a uniform electric field 2 0
t = torque 42 Physics–12
11.  
p = q × 2a 2a = dipole length Dipole moment in electric field ✎ Have the complete essence of the chapter;
11.
C=

C0
t
1 − 
t = thickness of slab
d = distance between the plates
C0 = capacitance
Capacitance of parallel plate capacitor with
conducting slab in between
 d

✎ Quite effective for a quick revision before


C=
ε0 A
K = dielectric constant Capacitance of parallel plate capacitor with
COMMON ERRORS 12.
exams.
 1 dielectric slab in between
d − t 1 − 
 K
S.No. Errors Corrections
1. Confusion between Electric field intensity & Electric Read the question properly before attempting.
✎ Important formulae, Symbols and applications
2.
Potential due to electric dipole
Method for finding the directions of Electric field. Direction of motion positive charges or decreasing
COMMON ERRORS

in the chapter are provided at one place for


S No. Errors Corrections
direction of electric potential
1. Student can’t differentiate b/w electric potential of a Electric potential of a dipole depends on distance and
3. Area Vector concept in Gauss Theorem. Area vector is always perpendicular to the surface. dipole and a single charge angle, while potential due to a single charge depends
recapitulation. 2.
only on distance.
Students get confused b/w the terms electric potential Electric potential is a scalar quantity, while potential
ElEctric chargEs and FiElds 41 and electric potential gradient. gradient is a vector quantity.
3. Student can’t differentiate b/w the graphs In graph
V q V q
A A A A

q V q V
and
V 1
Slope = =
q capacitance
D:\EG_Physics-12_(26-06-2022)\Open_Files\Ch-8\Ch-8
\ 26-Jul-2022 Ved_Goswami Proof-3 Reader’s Sign _______________________ Date __________ While in graph
q
Slope = = capacitance
V
4. Effect of dielectric in capacitors Proper polarity for dielectric slab
5. Variation of charge potential, energy, field, While connected p.d is constant and while disconnected
capacitance etc. on connected and disconnected with charge is constant.
battery condition.
IMPORTANCE OF EACH TOPIC AND FREQUENTLY ASKED TYPES OF QUESTIONS
6. Numeric Problems based on combination of Three ways of solving networks:
capacitors 1. Solve from one end of the circuit if possible.
☞ Important topIcs 2. re-draw the circuit in a simpler way
1. Questions related to Electromagnetic Spectrum. 3. Apply wheatstone’s principle if applicable
2. Questions on arranging of wavelength and frequency of emw in decreasing or increasing order.
3. Questions based on formula v = nl.
* Maximum weightage is of Electromagnetic Waves. ✎ Common
80 P –12
errors and corrections have beenhysics

1. Electromagnetic waves with wavelength:


(a) l1 are used to treat muscular strain. \ 26-Jul-2022 tagged to clear the confusion with cautions
D:\EG_Physics-12_(26-06-2022)\Open_Files\Ch-14\Ch-14
Ved_Goswami Proof-2 Reader’s Sign _______________________ Date __________

(b) l2 are used by a FM radio station for broadcasting.


(c) l3 are used to detect fracture in bones.
answers for productive learning.
(d) l4 are absorbed by the ozone layer of the atmosphere.
Identify and name the part of electromagnetic spectrum to which these radiation belong. Arrange these wavelengths
in decreasing order of magnitude. ASSIGNMENT
[Ans. (a) l1 → Infra red radiations, (b) l2 → VHF radiowaves, (c) l3 → X-rays, (d) l4 → Ultraviolet rays. The I. Objective Type Questions (1 Mark)
wavelengths in decreasing order of magnitude are: l2 > l1 > l4 > l3 ] 1. Multiple choice questions:
2. Identify the type of e.m. waves, whose method of production, is associated with (i) In depletion region of a diode
(a) a klystron value (a) there are no mobile charges
(b) vibrations of atoms and molecules
(b) equal number of holes and electrons exist making the region neutral
(c) decay of atomic nuclei.
(c) the recombination of holes and electrons has taken place
Also give the approximate range of wavelength of each of these e.m. waves.
[Ans. (a) Microwaves, l from 0.1 m to 1 mm, (b) Infrared radiation, l from 1 mm to 700 nm, (d) immobile charged ions exist
(c) Gamma rays, l < 10–3 nm] (ii) On recombination the energy is released in the form of
3. In an electromagnetic wave propagating along the x-direction, the magnetic field oscillates at a frequency of 3 × (a) 24 dB (b) 41 dB (c) 250 dB (d) 1250 dB
1010 Hz and has an amplitude of 10–7 tesla, acting along the y-direction. 2. Fill in the blanks:
(a) What is the wavelength of the wave? (i) In insulator, the valence electrons form a band that is ......................... .
(b) Write the expression representing the corresponding oscillating electric field. (ii) ........................ is called as junction in p-n junction diode.
[Ans. (a) l = 100 cm (b) Ez= 30 sin 2p (3 ×1010t – 100x)Vm–1 ]
4. A radio can tune to any station in the 7.5 MHz to 12 MHz band. What is the corresponding wavelength band? II. Very Short Answer Type Questions (1 Mark)
[Ans. 40 m – 25 m] 3. Why is the conductivity of n-type semi-conductor greater than that of the p-type semiconductor even when both
5. The amplitude of the magnetic field part of a harmonic electromagnetic wave in vacuum is B0 = 510 nT. Waht is of these have same level of doping?
the amplitude of the electric field part of the wave? [Ans. 153 NC–1] 4. Define the term ‘Doping’
6. Suppose that the electric field amplitude of an electromagnetic wave is E0 = 120 NC and that its frequency is v =
–1
III. Short Answer Type Questions-I (2 Marks)
50.0 MHz. (a) Determine, B0, w, k and l. (b) Find expressions for Ε and Β
5. Explain the working of ‘Forward Biasing’ of pn-Junction.
[Ans. (a) B0 = 4 × 10–7 T, w = 3.14 × 108 rad s–1, k = 1.05 m–1, l = 6.00 m,
 6. Give two differences between a half wave rectifier and a full wave rectifier. [CBSE 2022]
(b) Β = 4 × 10–7 sin (1.05x – 3.14 × 108t) j T]+
7. What is the cut-off frequency beyond which the ionosphere does not reflect electromagnetic radiations? IV. Short Answer Type Questions-II (3 Marks)
[Ans. Ionosphere cannot reflect electromagnetic radiations having frequency higher than 40 MHz] 7. Explain, with the help of a circuit diagram, the working of a p-n junction diode as a half wave rectifier.
8. What is the nature of the waves used in radar? What is their wavelength range? 8. (i) Draw V-I characteristics of a p-n Junction diode.
[Ans. Microwaves. These are electromagnetic waves of the wavelength range 10–3 m to 0.3 m.] (ii) Differentiate between the threshold voltage and the breakdown voltage for a diode.
(iii) Write the property of a junction diode which makes it suitable for rectification of ac voltage. [CBSE 2022]



✎ Repeatedly asked numerical problem types has


280 been
P given
–12 separately for better preparation.
hysics ✎ Chapterwise Assignment appended
for self evaluation.

(ix)

474 Physics–12
D:\EG_Physics-12_(26-06-2022)\Open_Files\0_Prelims\0_Prelims
\ 26-Jul-2022 Ved_Goswami Proof-3 Reader’s Sign _______________________ Date __________

CONTENTS

1. Electric Charges and Fields...................................................................................................................................... 11


2. Electrostatic Potential and Capacitance................................................................................................................... 46

3. Current Electricity.................................................................................................................................................... 85

4. Moving Charges and Magnetism........................................................................................................................... 126

5. Magnetism and Matter........................................................................................................................................... 164

6. Electromagnetic Induction..................................................................................................................................... 187

7. Alternating Current................................................................................................................................................ 225

8. Electromagnetic Waves.......................................................................................................................................... 263

9. Ray Optics ............................................................................................................................................................. 282

10. Wave Optics........................................................................................................................................................... 339

11. Dual Nature of Radiation and Matter..................................................................................................................... 376

12. Atoms..................................................................................................................................................................... 407

13. Nuclei..................................................................................................................................................................... 433

14. Semiconductor Devices......................................................................................................................................... 452

• Sample Paper-1 (Solved)........................................................................................................................................ 475

• Sample Paper-2 (Unsolved)................................................................................................................................... 495

• Sample Paper-3 (Unsolved)................................................................................................................................... 501

(x)
D:\EG_Physics-12_(26-06-2022)\Open_Files\Ch-1\Ch-1
\ 27-Jul-2022 Ved_Goswami Proof-4 Reader’s Sign _______________________ Date __________


Topics
C
overed
1 Electric Charges and Fields

1.1 Electric Charges and Electric Fields 1.2 Electric Dipole


1.3 Gauss’s Law and its Applications

C hapter map
Combination of charges Charge (Coulomb) Distribution of charges

n
 σ.ds ρ.dV
 ∑ qi Electric Field ∫ | r |2 × r or k ∫ | r |2 × r
E = k
E=k i =1
× r s V
| ri |2

 1 qQ 
F= r Field Intensity Electric Flux  

4πε0 r 2  d φ = E.ds
  E (N/C) (Nm2/C)
F = Eq

Gauss’s Theorem λ
 n Electric field due to E=
1 Q 2πε0 r
E=
4πε0 r 2
r ∑ qi charge distributions
i =1
φ =
closed ε0
surface σ
E=
2ε0

Topic 1. Electric Charges and Electric Fields


• Electrostatics: It is the branch of physics which deals (ii) By induction: It is the process of charging an

with the forces between the charges at rest, electric fields uncharged body, by keeping a charged body near it,
and potentials due to the charges at rest. without making a physical contact.
In this process the charge produced on the body, is
• Electric charge: It is an intrinsic property of elementary

opposite to the body, used to charge it.

particles like electrons and protons because of which
(iii) By conduction: It is also called charging by physical
they exert electrical force and are able to respond to
contact. If an uncharged body in brought in, electrical
electrical force. Electric charge is a scalar quantity.
contact, with the charged body, some electrons, are
There are two kind of charges (a) Positive charge, shared between the bodies. The uncharged body
(b) Negative charge. Like charges repel each other but acquire a charge similar to the charged body.
unlike charges attract each other. • Conductors: The substances through which electric

• Method of producing charge on a body charges can flow easily are called conductors. They

(i) By friction: When we rub two bodies together, there contain large number of free electrons. e.g. silver is the
is, transfer of electrons from one body to another, best conductor of electricity.
which causes the charging of bodies. By friction, both • Insulators: The substances that do not allow the charges

bodies acquire equal but opposite kind of charge. to flow through them are called insulators. e.g. plastic,
wood.
11
D:\EG_Physics-12_(26-06-2022)\Open_Files\Ch-1\Ch-1
\ 27-Jul-2022 Ved_Goswami Proof-4 Reader’s Sign _______________________ Date __________

•• Basic Properties of Electric Charges [q] = [AT]


(a) Quantization of electric charge: The charge on a K in Coulomb’s laws in also written as
body, is an integral multiple of, a basic quantity of 1
charge, which is the charge on an electron. K=
4πε m
(b) Additivity of electrical charges: The total charge
on a system is the algebraic sum of all the charges in where em = permittivity of the medium.
different parts of the body. •• Coulomb’s law in vector form: Consider 2 point
charges q1 and q2 separated by a distance r.
If a system contains, n charges, q1, q2, ..., qn then the
  
total charge of the system F12 r12 F21
q = q1 + q2 + ... + qn q1 q2

r21
(c) Conservation of charge: The net electric charge in
an isolated system is always constant. Force on q1 due to q2.
•• Coulomb’s law of electrostatic: The force of attraction/ → 1 q1 q2 
F12 = r 21
repulsion between two stationary point charges is 4πε 0 → 2
directly proportional to the product of the magnitude | r 12 |
of the charges and inversely proportional to the square Let magnitude of
of the distance between them. Force is along the line r12 = r = r21
joining the two charges
q1 q2 → 1 q1 q2 
r
F12 = r 21 ...(1)
A B 4πε 0 r 2
F ∝ q1q2 Force on q2 due to q1
1 → 1 q1 q2  1 q1 q2 
F∝ 2 r12 ...(2)
r F 21 = 2 r12 = 4πε 2
4πε 0 | r21 | 0 r
q q
F ∝ 1 22 
r But r12 = − r 21
q1 q2 Equation (2) becomes
F= k → −1 q1 q2 
r2 F 21 = r 21 ...(3)
The value of ‘k’ depends upon the medium chosen and 4πε 0 r 2
system of units taken. Comparing (1) and (3)
→ →
For vacuum, k = 9 × 109 Nm2/C2 F12 = – F 21
Note: Coulomb’s law is experimental law. It is applicable Coulomb’s law is in accordance with Newton’s 3rd law
for macroscopic and microscopic distance. of motion which states that every action has equal and
Electrostatic force is a conservative force. opposite reaction.
•• SI unit of charge: 1 Coulomb •• Permittivity is measure of effectiveness of the medium
q1 q2 in transmitting the electric field.
F= k Relative permittivity (er) [no unit]: Also called
r2
dielectric constant, (dielectric – insulator).
q1 = 1 C, q2 = 1 C, r = 1 m
It is ratio of permittivity of medium to the permittivity
F = k = 9 × 109 N
of vacuum
It is that amount of charge that repels an equal and ε
similar charge with a force of 9 × 109 N, when placed er = m
ε0
in vacuum at a distance of 1 m from it.
Relative permittivity in terms of force b/w charges: If
The CGS unit is ‘Stat Coulomb’ or ‘esu of charge’.
2 charges are placed certain distance apart in vacuum then
1C = 3 × 109 stat coulomb 1 q1 q2
Dimensional formula of charge F0 = ...(1)
4πε 0 r 2
q In given medium
I=
t 1 q1 q2
q = It Fm = ...(2)
4πε m r 2

12 Physics–12
D:\EG_Physics-12_(26-06-2022)\Open_Files\Ch-1\Ch-1
\ 27-Jul-2022 Ved_Goswami Proof-4 Reader’s Sign _______________________ Date __________

F0 4πε m ε m + + + + +
\ = = = er dq
Fm 4πε 0 ε 0 + + + + +
dA
Relative permittivity is ratio of force b/w two point + + + + +

charges placed certain distance apart in vacuum to the + + + + +


force b/w the same two charges placed same distance •• Volume Charge Density r: It is the charge per unit
apart in the given medium. dq
•• Forces between multiple charges: The force acting on volume of the body ρ = .
dV
a point charge due to multiple charges is given by the
+ + + + + + + + + + +
vector sum of all individual forces acting on the charges. + + + + + + + + + + +
+ + + + + + + + + + +
F1 + + + + + + + + + + +
+ + + + + + + + + + +
+ + + +dV+ + + + + + +
F14 + + + + + + + + + + +
+ + + + + + + + + + +
+ + + + + + + + + + +
+ + + + + dq + + + + + +
+ + + + + + + + + + +
q2 q1 + + + + + + + + + + +
+ + + + + + + + + + +
+ + + + + + + + + + +
q3 q4
Electric Field Intensity
•• Principle of superposition of forces: The total force
•• Electric field: The region/space around a charged body
acting on a given charge due to a number of charges
within which its influence can be felt is called electric
around it is the vector sum of the individual forces acting
field.
on it due to all the charges.
Electric field intensity/strength is defined as the force
q1
experienced per unit positive test charge (vanishingly

r13 small) placed at that point. It is denoted by E .

r1

r12



r23

q2 F →
r2 E = lim


q3 dq → 0 dq
r3


dq should be vanishingly small so that it does not alter


Consider the charges q1, q2, q3 and so on ... qn. The net the field which it is being used to measure.
force on q1 is given by →
→ → → → → E is a vector quantity SI unit → N/C.
F1 = + + F12+ ... + F13 F14 F1n

Note: The force between two charges is unaffected by the Expression for E due to point charge.
presence of other charges in its vicinity. →
Consider a point charge +Q placed at origin. To find E at
Continuous Charge Distribution point P let us place a test charge q0 at that point.
•• Linear Charge Density λ: It is the charge per unit length → 1 Q q0 
dq F = ⋅r
λ= . 4πε 0 r 2
dl
dl
+Q q0
+ + + + + + + + + + + O P
r


dq


•• Surface Charge Density σ: It is the amount of charge E = lim F = Q ⋅ r = 1 ⋅ Q ⋅ r
dq q0 →0 q0 4πε0 r 2 4πε0 r 2
per unit surface area σ = .
dA

Electric Charges and Fields 13


D:\EG_Physics-12_(26-06-2022)\Open_Files\Ch-1\Ch-1
\ 27-Jul-2022 Ved_Goswami Proof-4 Reader’s Sign _______________________ Date __________

– Tangent to an electric field line at any point gives



direction of E at that point.

– NO two lines of force can cross each other. (If they do,
E there will be two tangents at that point, two direction

of E not possible).

r
– They are always normal to surface of conductor
on which the charges are in equilibrium. (It not,

component of E parallel to surface will set up
a current which is not possible in equilibrium
condition).
E
– They have tendency to contract lengthwise which
explains attraction between unlike charges.
2 – They have tendency to expand laterally. This explains
1/r
repulsion between two like charges.
•• Electric Field Lines: It is a curve, the tangent to which

at any point gives the direction of force acting on a test – They give measure of strength of E (closer in strong
charge placed at that point. field, far apart about in weak field, parallel and equally
•• Properties of Electric Field Lines: →
– Continuous smooth curves without breaks. spaced in uniform E ).
– Start at +ve charge, end at –ve charge cannot form →
closed loops. If there is a single charge, they start/ – They do not pass through conductor since E inside
end at infinity. a conductor is 0.

EXERCISE 1.1
I. Objective Type Questions (1 Mark) (b) directed perpendicular to the plane but
1. Choose the correct answers from the given options towards the plane.
(MCQs). (c) directed radially away from the point charge.
(i) A point positive charge is brought near an isolated (d) directed radially towards the point charge.
conducting sphere in given figure. The electric (iii) The electric field at a point is[NCERT Exemplar]
field is best given by [NCERT Exemplar] (a) always continuous.
(b) continuous if there is no charge at that point.
(c) discontinuous only if there is a negative charge
at that point.
(d) discontinuous if there is a charge at that point.
(iv) A positive charge +Q is uniformly distributed
along a circular ring of radius R. A small test
charge q is placed at the centre of ring, (Fig.).
Then:[NCERT Exemplar]
+ +Q
(a) Fig. (i) (b) Fig. (ii)
(c) Fig. (iii) (d) Fig. (iv)
(ii) A point charge +q is placed at a distance ‘d’ from q
an isolated conducting plane. The field at a point R z
P on the other side of the plane is
[NCERT Exemplar]
(a) If q > 0 and is displaced away from the centre
(a) directed perpendicular to the plane and away in the plane of ring, it will be pushed back
from the plane. towards the centre.
14 Physics–12
D:\EG_Physics-12_(26-06-2022)\Open_Files\Ch-1\Ch-1
\ 27-Jul-2022 Ved_Goswami Proof-4 Reader’s Sign _______________________ Date __________

(b) If q < 0 and is displaced away from the centre + + + + + + +


in the plane of ring, it will never return to the + + + + + + +

centre and will continue moving till it hits the


ring. (c)
(c) If q < 0, it will perform S.H.M. for small
displacement along the axis. –













(d) q at the centre of the ring is in an unstable + + + + + + +
equilibrium within the plane of the ring for + + + + + + +
q > 0.
(d)
(v) A negatively charged object X is repelled by
another charged object Y. However an object Z
is attracted to object Y. Which of the following is – – – – – – –
the most possibility for the object Z? – – – – – – –

(CBSE 2022) (ix) Three charges q, –q and q0 are placed as shown


(a) positively charged only in figure. The magnitude of the net force on the
(b) negatively charged only  1 
charge q0 at point O is  k = 
(c) neutral or positively charged  (4π ∈0 ) 
(d) neutral or negatively charged (CBSE 2022)
(vi) In an experiment three microscopic latex spheres y
are sprayed into a chamber and became charged
with charges + 3e, + 5e and –3e respectively. All
the three spheres came in contact simultaneously –q
for a moment and got separated. Which one of the
a
following are possible values for the final charge q0 q
on the spheres? (CBSE 2022) x
O
(a) +5e, –4e, + 5e (b) +6e, + 6e, –7e a
(c) –4e, + 3.5e + 5.5e
(d) +5e, –8e, + 7e
2kqq0
(vii) An object has charge of 1 C and gains 5.0 × 1018 (a) 0 (b)
electrons. The net charge on the object becomes– a2
(CBSE 2022) 2kqq0 1 kqq0
(c) (d)
(a) – 0.80 C (b) + 0.80 C a2 2 a2
(c) + 1.80 C (d) + 0.20 C 2. Fill in the blanks.
(viii) Which of the diagrams correctly represents the (i) The force between two point charges .......... when
electric field between two charged plates if a dielectric constant of the medium in which they
neutral conductor is placed in between the plates? are held increases.
(CBSE 2022) (ii) Two identical charges are placed at a distance 2a
apart as shown in figure. The potential energy of
+ + + + + + +
+ + + + + + +
the positively charged particle placed at the mid
(a) point O will be .......... .
Y
q O q
– – – – – – –
– – – – – – – 2a
+
+
+
+
+
+
+ +
+
+
+
+
+ (iii) .......... is a central force in nature.
3. State True or False
(b) (i) When a dielectric medium is placed in a uniform
electric field, the dielectric constant of the medium
– – – – – – – is the ratio of the absolute permittivity of the
– – – – – – – medium to the absolute permittivity of free space.

Electric Charges and Fields 15


D:\EG_Physics-12_(26-06-2022)\Open_Files\Ch-1\Ch-1
\ 27-Jul-2022 Ved_Goswami Proof-4 Reader’s Sign _______________________ Date __________

(ii) The number of electric lines of force that radiate distance in water, how does the electric force between
outward from 1C charge is 1.13 × 1011. them change? [AI 2011]
4. Match the Columns 7. A force F is acting between two point charges Q1 and
Column-I Column-II Q2 when placed in vacuum. If a third charge Q3 is
(Method of (Process) placed close to q1, what happens to the force between
Producing Charge) Q1 and Q2?
(i) Friction (a) Keeping a charged body near 8. Why is it necessary that the field lines from a point
uncharged body charge placed in the vicinity of a conductor must be
(ii) Induction (b) Charging by physical contact normal to the conductor at every point?
(iii) Conduction (c) When we rub two bodies [Foreign 2009]
together. 9. Why does the electric field inside a di-electric decrease
For question number 5, two statements are given-one when it is placed in an external electric field?
labelled Assertion (A) and the other labelled Reason 10. A metal sphere is placed in a uniform electric field.
(R). Select the correct answer to these questions from Which path is followed by electric lines and why?
the codes (a), (b), (c) and (d) as given below. a
(a) Both A and R are true and R is the correct explanation b
of A c
(b) Both A and R are true but R is NOT the correct
explanation of A
(c) A is true but R is false d

(d) A is false and R is also false 11. Draw electric field lines to represent a uniform electric
5. Assertion (A): If a proton and an electron are field.
"
released in uniform electric field (E ) at t = 0, then at 12. Draw the pattern of electric field around a charge
a certain instant, later their kinetic energies will be (i) q > 0 (ii) q < 0.
equal.1 13. A small test charge is released at rest at a point in an
"
Reason (R): Electric force (F ) on a proton and an electrostatic field configuration. Will it travel along
"
electron a uniform electric field (E ), has the different the field line passing through that point?
magnitude. 14. Sketch the electric field lines due to two equal positive
II. Very Short Answer Type Questions (1 Mark) charges placed near each other.
1. Ordinary rubber is an insulator. But the special rubber 15. Sketch the electric field lines of a +ve point charge
tyres of aircrafts are made slightly conducting why placed near a negative charge of the same magnitude
is it necessary? [NCERT] OR sketch the electric field lines due to an electric
dipole.
2. Is the force acting between two point charges q1 and
16. Sketch the electric field lines for two point charges q1
q2 kept at some distance in air, attractive or repulsive
and q2 such that q1 > q2, separated by a small distance
when: (i) q1q2 > 0, (ii) q1q2 < 0? [CBSE 2007]
d.[AI 2019]
3. What does q1 + q2 = 0 signify in electrostatics? 
17. Figure shows a point charge +Q, located at a distance
4. A point charge (+q) is kept in the vicinity of uncharged R/2 from the centre of a spherical metal sheet. Draw
conducting plate. Sketch electric field lines between the electric field lines for the given system.
the charge and the plate. [Delhi 2014] [S.P. 2015-16]
5. Two large conducting spheres carrying charges Q1 and
Q2 are brought close to each other. Is the magnitude
of electrostatic force between them exactly given by O
R/2
O′+Q
Q1 Q2/4πε0r2, where r is the distance between their
centres?[NCERT]
6. Force between two point charges kept at a distance d
apart in air is F. If these charges are kept at the same 18. Explain, why two field lines never cross each other?

16 Physics–12
D:\EG_Physics-12_(26-06-2022)\Open_Files\Ch-1\Ch-1
\ 27-Jul-2022 Ved_Goswami Proof-4 Reader’s Sign _______________________ Date __________

19. What is the work done by the field of a nucleus in a a uniform electric field E is maintained between the
complete circular orbit of the electron? What if the plates. Show that the vertical deflection of the particle
orbit is elliptical? at the far edge of the plate is qEL2/2mvx2.[NCERT]
20. Draw a graph to show the variation of E with 29. A spherical conductor of radius 12 cm has a charge
perpendicular distance r from the line of charge. of 1.6 × 10–7 C distributed uniformly on its surface.
[Delhi 2018] What is the electric field (a) in side the sphere
21. Draw the pattern of electric field lines when a point (b) just outside the sphere (c) at a point 18 cm from
charge –q is kept near an uncharged conducting plate. the centre of the sphere? [NCERT]
[Delhi 2019] 30. Two point charges q1 and q2 of magnitude 10–8 C
and –10–8 C respectively, are placed 0.1 m apart.
III. Short Answer Type Questions-I (2 Marks) Calculate the electric fields at points A, B and C as
22. A comb run through one’s dry hair attracts small bits shown in the Fig. [NCERT]
of paper why? What happens if the hair is wet or if C
it is a rainy day? [NCERT]
23. You are given three uncharged spherical conductors
A, B and C, touching each other. A glass rod carrying
positive charge is brought near A as shown. What will
be the charges on spheres A, B and C? B (q1) (q2)
.05m .05m A .05m
+ A B C 31. Obtain the formula for the electric field due to a long
+
+ thin wire of uniform linear charge density l without
using Gauss theorem. [NCERT]
[NCERT Exemplar]
32. Four point charges Q, q, Q and q are placed at the corners
24. Two point charges + 4e and + e are fixed a distance ‘a’ of a square of side ‘a’ as shown in the figure.
apart. Where should a third point charge q be placed [Delhi 2018]
on the line joining the two charges so that, it may be
Q q
in equilibrium.
25. An oil drop of 12 excess electrons is held stationary
under a constant electric field of 2.55 × 104 N/C in
millikan’s oil drop experiment. The density of oil is
1.26 g/cm3. Estimate the radius of the drop. (g = 9.81 q Q
a
m/s2; e = 1.6 × 10–19 C).[NCERT]
26. Find the work done in bringing a charge q from Find the
perpendicular distance r1 to r2 (r2 > r1). [Delhi 2018] (a) resultant electric force on a charge Q, and
IV. Short Answer Type Questions-II (3 Marks) (b) potential energy of this system.
27. Consider three charges q1, q2 and q3 each equal to q 33. (a) Two electric field lines cannot cross each other.
placed at the vertices of an equilateral triangle of side Also, they cannot form closed loops. Give reasons.
l. What is the force on a charge Q (with the same sign (b) A particle of charge 2 µC and mass 1.6 g is moving
as q) placed at the centroid of the triangle? with a velocity 4 ^i ms–1. At t = 0 the particle enters
[NCERT Exemplar] →
in a region having an electric field E (in NC–1)
28. A particle of mass m and charge (–q) enters the region
= 80 ^i + 60 ^j . Find the velocity of the particle at
between the two charged plates initially moving along
t = 5 s. [Delhi 2020]
x-axis with speed vx. The length of the plate is L and

Answers 1.1
I. Objective Type Questions 2. (i) decreases (ii) minimum
1. (i) (a) (ii) (a) (iii) (b) and (d) (iii) Electrostatic force
(iv) (a) (b) (c) and (d) (v) (c) (vi) (b) 3. (i) False (ii) True
(vii) (d) (viii) (d) (ix) (c) 4. (i)- (c) (ii)-(a) (iii)-(b)
5. (d)

Electric Charges and Fields 17


D:\EG_Physics-12_(26-06-2022)\Open_Files\Ch-1\Ch-1
\ 27-Jul-2022 Ved_Goswami Proof-4 Reader’s Sign _______________________ Date __________

II. Very Short Answer Type Questions 10. path (d) is followed by electric field lines.
1. The tyres of aircraft may get, highly charged, due Reason: There will be no electric field inside a metal
to friction between the tyres and the air strip, during sphere and field lines are normal to the surface of the
landing. When the tyres are made slightly conducting, sphere.
they will lose the charge to the earth, otherwise too 11. E

much of charges accumulated may produce spark and
result in fire.
2. (i) If q1q2 > 0, then q1 and q2 are like charges, so there
is repulsion.
(ii) If q1q2 < 0, then q1 and q2 are unlike charges, then
12.
there is attraction.
3. The equation q1 + q2 = 0 signifies that q1 and q2 are equal
and opposite charges and the charges are algebraically
additive.
+
4. +q

5. When the charged spheres are placed close to each (i) q > 0
other, the charge distribution on them will become non
uniform. Therefore, coulomb’s law is not valid. So the
magnitude of electrostatic force between them is not
exactly given by Q1Q2/4π∈0r2.

6. As dielectric constant.
F
K = air
Fmedium
Fair F
So, Fwater =
= (ii) q < 0
K 80
[ K for water = 80] 13. Not necessarily, because the electric field lines gives
1 the direction of force experienced by the test charge,
So, the force in water is times the force in air.
80 i.e., the direction of acceleration and not the velocity.
7. By superposition principle, the force between two 14. N → neutral point.
charges is not affected by the presence of the third
charge. Hence the force between Q1 and Q2 remains
same i.e., F.
8. If the field lines are not normal, then the electric field
will have a component parallel to the surface of the
conductor, which will make the electrons move along
+ N +
the surface and the conductor will not be in equilibrium. q q
9. When a dielectric is placed inside a electric field, the
dielectric gets polarised. The induced electric field
(Ein) due to the polarisation is opposite to the applied
electric field (E0). Thus electric field inside a di-electric
decreases.
+
– p + p
– 15.
Ein

+q –q

E0

18 Physics–12
D:\EG_Physics-12_(26-06-2022)\Open_Files\Ch-1\Ch-1
\ 27-Jul-2022 Ved_Goswami Proof-4 Reader’s Sign _______________________ Date __________

the paper, the paper gets polarised, resulting in a net


16.
force of attraction. 1
If the hair is wet, or it is a rainy day, friction between
d hair and the comb reduces, so the comb does not get
+q1
N
+q2
charged. Therefore it will not attract bits of paper. 1
23. By induction negative charge will appear on the end of
A facing the charged rod C will be positively charged,
but B will be neutral. 2
24. + 4e F2 P F1 +e
17. A B
+q
x a–x
For charge + q to be in equilibrium, net force on it must
4eq
O′ R be zero. Force on +q due to charge +4e = F1 = .
O +Q′
OO′ = 4π ∈o x 2
2
Force on +q due to charge +e
eq
= F2 =  1
4π ∈o (a − x)2
As the two forces are in opposite direction.
So, F1 = F2
18. Two electric field lines, can never cross each other
for q to be in equilibrium
because, if they cross, then there will be two directions
of electric field, at the point of intersection, which is 4eq eq
2
=
not possible. 4π ∈o x 4π ∈o (a − x) 2
19. When the orbit is circular then the work done is zero. 4(a – x)2 = x2
As electrostatic force is conservative, even in the case ⇒ 2(a – x) = ± x
of elliptical orbit, the work done will be zero. 2a
x= or 2a 1
λ 3
E= 2a
20.   As the charge is placed between +4e and +e, so x =
2π ∈o r 3
is possible.
25. n = 12, E = 2.55 × 104 N/C, r = 1.26 g/cm3
So r = 1.26 × 103 kg/m3
As the drop is stationary so
Weight of the drop = Force due to electric field
mg = qE 1
4 3
pr rg = neE
3
3neE
1 r3 =
or E∝ 4πρg
r
The curve between E and r. 3 × 12 × 1.6 × 10−19 × 2.55 × 104
r3 =
21. 4 × 3.14 × 1.26 × 103 × 9.8
= 0.95 × 10–18 1
or –7
r = 9.8 × 10 m
26. Work done in bringing a charge q from perpendicular
distance r1 to r2.
  dW = F.dr
r2
W= ∫r
1
qE.dr 

22. When the comb runs through dry hair, due to friction,
r2 λ
W = q ∫r dr  1
it gets charged. when this comb now is placed close to 1 2π ∈o r

Electric Charges and Fields 19


D:\EG_Physics-12_(26-06-2022)\Open_Files\Ch-1\Ch-1
\ 27-Jul-2022 Ved_Goswami Proof-4 Reader’s Sign _______________________ Date __________

qλ r2 1 28. In the vertical direction


W=
2π ∈o ∫r 1 r
⋅ dr
uy= 0, Fy = qE
qλ qE
or W = [loge r ]rr12 So ay =  1
2π ∈o m
L

or W = [loge r2 − log e r1 ]
2π ∈o E e

qλ r 
or W = log e  2   1
2π ∈o  r1 
Let deflection be y
27. O is the centroid of the triangle.
In x direction: S = L, Speed = vx
l 3
AD = AB sin 60° = So t = distance = L 
2 speed vx
2 1
BO = CO = AO = AD As Sy = u y t + a y t 2
3 2
2 l 3 l 2
= =  1   qEL2
3 2 3 y = 0 + 1  qE  L =  1
2  m   vx  2mvx2
Force on Q due to charge q1 = F1
Qq
Case is similar to the motion of a horizontal projectile
F1 = along AO in a gravitational field.
4πε 0 (l / 3 ) 2
1
where y = gt 2  1
3Qq 2
=
along AO
4πε 0l 2 29. (a) Inside the conducting sphere, E = 0 1
Force on Q due to charge q2 = F2 (b) Just outside the sphere
q
F = 3Qq along BO
2 E= , r → radius of sphere
4πε 0l 2 4πε 0 r 2
Similarly force on Q due to the charge q3 = F3
9 × 109 × 1.6 × 10−7
3Qq = = 105 N/C 1
F3 = along CO 1 (0.12)2
4πε 0l 2
(c) At a distance of 18 cm from the centre
→ → →
| F1 | = | F2 | = | F3 | = F R = 0.18 cm,

q 9 × 109 × 1.6 × 10−7
Resultant of F2 and F3 E= =
4πε 0 R 2 (0.18) 2
= F22 + F32 + 2F2 F3cos 120°
= 4.4 × 104 N/C 1
1
30. Here, q1 = 10–8 C, q2 = –10–8 C
=  2F2 + 2F2  − =F 1  2
    At A: Electric field at A due to q1
 2 
This resultant is along OA. 9 ×109 × 10−8
EA1 = = 3.6 × 104 N/C
Thus the total force on Q is F – F = 0 1 (0.05) 2
A  KQ 
q1
 E = r 2  Towards right
Electric field at A due to q2
9 × 109 × 10−8
F E EA2 =
F3 (0.05) 2
F2
O = 3.6 × 104 N/C Towards right
60° F1 → → →
4 1
q2
q3
E A = E A1 + E A2 = 7.2 × 10 N/C
B D C
 Towards right
20 Physics–12
D:\EG_Physics-12_(26-06-2022)\Open_Files\Ch-1\Ch-1
\ 27-Jul-2022 Ved_Goswami Proof-4 Reader’s Sign _______________________ Date __________

EC1 →
C d E can be resolved into two rectangular components, (i)
EC2
dE cos q perpendicular to the wire (ii) dE sin q upward
0.1m
60° parallel to the wire.
The parallel component dE sin q will be cancelled by the
parallel component of the field due to a similar charge
0.1m

element dx of the wire in the other half. The dE cos q


B EB1 EB2 components will get added.
λdx cos θ
(q1) .05m .05m (q2)
E = ∫ dE cos θ = ∫ ...(1)
4πε 0 (r 2 + x 2 )
9 ×109 × 10−8
At B:
EB1 = In D OCP, x = r tan q
(0.05) 2
dx = r sec2 q dq
= 3.6 × 104N/C Towards left
r2 + x2 = r2 + r2 tan2 q
9 −8
9 ×10 × 10 = r2 (1 + tan2 q) = r2 sec2 q 1
EB2 =
(0.05) 2 So equation (1) becomes
= 4 × 103 N/C Towards right π/2
E= λr sec2 θ d θ cos θ

→ →
E B = E B1 + E B2

= 3.6 × 104 – 4 × 103 ∫ 4πε 0 r 2 sec2 θ
−π/2

= 3.2 × 104 N/C Towards left 1 π/2


λ cos θ d θ
9 × 109 × 10−8
= ∫ 4πε 0 r
At C: EC1 = =9× 103 N/C −π/2
(0.1)2
λ λ
as indicated in diagram = [sin θ]π− /π2/ 2 =  1
4πε 0 r 2πε 0 r
9 −8
9 × 10 × 10
EC2 = 32. FD = Force on charge Q at A due to charge q at D.
(0.1)2
FC = Force on charge Q at A due to charge Q at C.
= 9 × 103 N/C Towards q2 FB = Force on charge Q at A due to charge q at B
E C1 and E C2 can be resolved into rectangular
FC FD
components. Their vertical components will cancel
each other and horizontal components will get added. Q q
FB B
EC = EC1 cos 60° + EC2 cos 60° A
1 1
= 9 × 103 × + 9 × 103 ×
2 2
3
= 9 × 10 N/C Towards right 1
D C
31. Let AB be a long thin wire of uniform linear charge q Q
a
density l. Let the perpendicular distance of the point P
from the wire be r. → → 1 Qq
FB = FD =
Consider an element of length dx of the wire with centre 4πε 0 a 2
at O. Distance OC = x.
→ 1 Q2
dq = ldx FC =  1
4πε 0 (a 2 ) 2
Electric field intensity due to this element
dq λdx The net resultant forces is equal to resultant F, of the two
dE = 2
=  1 → →
4πε 0 (OP) 4πε 0 (r 2 + x 2 ) mutually perpendicular forces FB and FD .
A dE sin θ 1 Qq 1 Q2
dE
\ F= 2 +
r θ 4πε 0 a 2 4πε 0 2a 2
C θ dE cos θ
P
x 1 1  Q2 
or F=  2 Qq + 
O
B
4πε 0 a 2  2 

Electric Charges and Fields 21


D:\EG_Physics-12_(26-06-2022)\Open_Files\Ch-1\Ch-1
\ 27-Jul-2022 Ved_Goswami Proof-4 Reader’s Sign _______________________ Date __________

33. (a) Electric field lines do not form closed loops because
1 1  Q2  the direction of an electric field is from positive to
    F=  2 Qq +   1
4πε 0 a 2  2  negative charge. So one can regard a line of force
P.E. of a system of two point charges q1 and q2 separated starting from a positive charge and ending on a
by a distance a is negative charge. This indicates that electric field
lines do not form closed loops.
qq
U= k 1 2 (b) q = 2 × 10–6 C, m = 1.6 × 10–3 kg
a
In this question four point charges are at the four vertices E= (80)2 + (60)2 = 100 NC–1,
of the square. So there are six pairs of charges (Q, q) at A t = 5 sec.
and B, (q, Q) at B and C, (Q, q) at C and D, (q, Q) at D F = Eq
and A, (Q, Q) at A and C and (q, q) at B and D. Therefore ⇒ ma = Eq
total P.E. of this system 100 × 2 × 10 −6
Eq
1  Qq qQ Qq qQ Q 2 q2 
⇒ a= =
  =  + + + + +  m 1.6 × 10 −3
4πε 0  a a a a 2a 2a  = 125 × 10–3 = 0.125 m/s2
1  4Qq Q 2 q2  using v = u + at
  =  + +  1
4πε 0  a 2a 2a  v = 4 + 0.125 × 5 = 4.625 m/s

Topic 2. Electric Dipole


•• Electric Dipole: It is a pair of two equal and opposite 2 pr
charges separated by a small distance. E =
4πε 0 (r 2 − a 2 ) 2
A 2a
–q +q
B
•• Electric field intensity on equatorial line of an

AB (dipole length) = 2a, is distance b/w the two electric dipole:
→ kp 1
charges constituting the dipole. It is taken as +ve when | E | = (r 2 + a 2 )3 / 2 , k = 4πε
being measured from –ve to +ve i.e. It’s direction is 0

from –ve charge to +ve charge. → kp


→ → For r >>> a | E | =
•• Dipole moment ( p ) : p = Magnitude of the charge × r3
dipole length Electric field is in a direction opposite to the direction

→ →
of dipole moment ( p ) .
p = q × 2a
It is a vector quantity. E axial
Its direction is always from –ve to +ve. For r >> a, =2
E equatorial
SI unit = Cm [Coulomb – metre]
•• The Field of an Electric Dipole: Line joining the two Note: Electric field intensity due to an electric dipole
charges is the axis of the dipole. Perpendicular bisector varies inversely as cube of the distance of the
of the dipole length is equator of the dipole. point from the centre of the dipole.
1
i.e.,   E ∝
r3
Electric dipole in a uniform electric field:
A B
–q +q (a) Force experienced by the dipole is zero.
→ →
(b) Torque acting t = p × E
Potential energy of a dipole placed
  in a uniform
•• Electric field intensity on the axial line of an electric
electric field is (– pE cos θ) = − p ⋅ E .
dipole:

22 Physics–12
D:\EG_Physics-12_(26-06-2022)\Open_Files\Ch-1\Ch-1
\ 27-Jul-2022 Ved_Goswami Proof-4 Reader’s Sign _______________________ Date __________

EXERCISE 1.2
I. Objective Type Questions (1 Mark) of the charge is doubled, the force acting on the
1. Choose the correct answers from the given options charge will become
(MCQs). (a) 2F (b) F/2 (c) F/4 (d) F/8
(i) Figure here shows electric field lines in which an [All India 2020]

electric dipole p is placed as shown. Which of 2. Fill in the blanks.
the following statements is correct? (i) Electric dipole is a pair of two .......... and ..........
charges separated by a small distance.

(ii) p = magnitude of the charge × ..........
3. Match the Columns
Column-I   Column-II
(i) Torque (a) q × 2a
(a) The dipole will not experience any force.  p
(ii) Dipole moment ( p ) (b)
( )
3/ 2
(b) The dipole will experience a force towards 4πε 0 r + a 2
2
right.   
(c) The dipole will experience a force towards (iii) E on axial line (c) p × E
left.  2pr
(iv) E on equatorial line (d)
( )
2
(d) The dipole will experience a force upwards. 4πε 0 r 2 − a 2
(ii) An electric dipole of moment p is placed in the
position For questions numbers 4 two statements are given-one
 of stable equilibrium in a uniform electric labelled Assertion (A) and the other labelled Reason
field E . The couple required to rotate it through
an angle q the initial position is: (R). Select the correct answer to these questions from
the codes (a), (b), (c) and (d) as given below.
(a) – pE cos q (b) pE tan q
(a) Both A and R are true and R is the correct
(c) pE cos q (d) pE sin q explanation of A
(iii) Consider a region inside which there are various (b) Both A and R are true but R is NOT the correct
types of charges, but the total charge is zero. At explanation of A
points outside the region,
(c) A is true but R is false
(a) the electric field is necessarily zero.
(d) A is false and R is also false
(b) the electric field is due to the dipole moment
of the charge distribution only. 4. Assertion (A): In a non-uniform electric field, a
dipole will have translatory as well as rotatory motion.
1
(c) the dominant electric field is ∝ 3 , for large Reason (R): In a non-uniform electric field, a dipole
r
‘r’, where ‘r’ is the distance of point (outside) experiences a force as well as torque.
from an origin in this region. [CBSE S.P. 2020-21]
(d) the work done to move a charged particle II. Very Short Answer Type Questions (1 Mark)
along a closed path, away from the region, 
1. A dipole of dipole
 moment p , is present in a uniform
will be zero. electric field E . For what value of angle between
 
(iv) An electric dipole placed in a non-uniform electric p and E , the torque experienced by the dipole, is
field can experience [Delhi 2020] minimum? [Delhi 2009 C]
(a) a force but not a torque 2. What orientation of an electric dipole in a uniform
(b) a torque but not a force electric field corresponds to its (i) stable and (ii)
unstable equilibrium? [Delhi 2010, 2017]
(c) always a force and a torque
3. What is the angle between the directions of electric
(d) neither a force nor a torque field at any (i) axial point and (ii) equatorial point due
(v) A point charge is situated at an axial point of a to an electric dipole? [S.P. 2008]
small electric dipole at a large distance from it. 4. The electric field strength at a distance r on the
The charge experiences a force F. If the distance equatorial line of a dipole is E. If the distance of the
Electric Charges and Fields 23
D:\EG_Physics-12_(26-06-2022)\Open_Files\Ch-1\Ch-1
\ 27-Jul-2022 Ved_Goswami Proof-4 Reader’s Sign _______________________ Date __________

point from the dipole is doubled, how will the electric 10. Show that the force acting on an electric dipole placed
field intensity be affected? [Delhi 2009 C] in a uniform electric field in zero. Also obtain an
III. Short Answer Type Questions-I (2 Marks) expression for torque acting on this dipole.
5. An electric dipole of length 20 cm having charges 3 × 11. (a) An electric dipole is kept first to the left and then
10–3 C, placed at 30° with respect to a uniform electric to the right of a negatively charged infinite plane
sheet having a uniform surface charge density.
field, experience a torque of magnitude 6 3 Nm.
The arrows p1 and p2 show the directions of its
Calculate (i) magnitude of electric field (ii) potential
electric dipole moment in the two cases.
energy of electric dipole. [Delhi 2006]
6. Derive the expression for the torque acting on an
electric dipole, when it is held in a uniform electric
field. Identify the orientation of the dipole in the p1 p2
electric field, in which it attains a stable equilibrium.
[Delhi 2020]
IV. Short Answer Type Questions-II (3 Marks)
7. Derive an expression for the electric field intensity at Identify for each case, whether the dipole is
a point on the equatorial line of an electric dipole of in stable or unstable equilibrium. Justify each

dipole moment p and length 2a. What is the direction answer.
of this field? [AI 2016, 2017] (b) Next, the dipole is kept in a similar way (as
8. Obtain an expression for electric field intensity at a shown), near an infinitely long straight wire
point on the axis of an electric dipole. having uniform negative linear charge density.
9. Two small identical electrical dipoles AB and CD,  [CBSE S.P. 2018-19]
each of dipole moment ‘p’ are kept at an angle of
120° as shown in the figure. What is the resultant
dipole moment of this combination? If this system p1 p2

is subjected to electric field E directed along +X
direction, what will be the direction and magnitude
of the torque acting on this? [Delhi 2011] 12. (a) Define an ideal electric dipole. Give an example.
D
Y (b) Derive an expression for the torque experienced
–q A +q by an electric dipole in a uniform electric field.
What is net force acting on this dipole?
X
120°
X
(c) An electric dipole of length 2 cm is placed with
its axis making an angle of 60° with respect to
+q uniform electric field of 105 N/C. If it experiences
B –q
C a torque of 8 3 Nm, calculate the (i) magnitude
Y
of charge on the dipole and (ii) potential energy.

Answers 1.2
I. Objective Type Questions 2. (i) Stable equilibrium: Dipole moment should be in
1. (i) (c) (ii) (d) (iii) (c and d) the direction of electric field i.e. θ = 0°.
(iv) (c) (v) (d) (ii) Unstable equilibrium: Dipole moment should be
2. (i) equal; opposite (ii) dipole length antiparallel to electric field i.e., θ = 180°.
3. (i)- (c) (ii)-(a) (iii)-(d) (iv)-(b) 3. Angle between electric field at an axial point and at an
4. (a) Both A and R are true and R is the correct explanation equatorial point is 180°.
of A
1
II. Very Short Answer Type Questions 4. For a small dipole E ∝ . So electric field intensity
r3
→ →
1. Torque is minimum when angle between p and E is 1
will become E.
0° as τ = pE sin θ 8

24 Physics–12
D:\EG_Physics-12_(26-06-2022)\Open_Files\Ch-1\Ch-1
\ 27-Jul-2022 Ved_Goswami Proof-4 Reader’s Sign _______________________ Date __________

5. 2a = 20 cm = 0.2 m, q = 3 × 10–3 C →
E+q
θ = 30° E+qcosθ
P
τ = 6 3 Nm E–qcosθ θ

(i) τ = pE sin θ, where p = q × 2a


–3 E–q
6 3 = 3 × 10 × 0.2 × E sin 30°
1 r
6 3 = 0.6 × 10–3 × E
2
–3
6 3 = 0.3 × 10 E θ
A B
–q 2a O +q
6 3

⇒ E= 4
= 2 3 × 10 N/C 1
0.3 × 10−3
OP = r, AB = 2a, AP2 = BP2 = (r2 + a2)
(ii) Potential energy of dipole → 1 q kq
E −q = 4πε = 2 along PA
U = – pE cos θ 0 ( AP)
2
(r + a 2 )
U = –3 × 10–3 × 0.2 × 2 3 × 104 cos 30° → 1 q kq
E +q = 2
= 2 along BP
4πε 0 (BP) (r + a 2 )
= − 12 3 × 3 = – 18 J 1
2 → →
| E −q | = | E +q |  1
6. Consider a dipole with ±q charges separated by a

distance 2a. →
 The components (vertical) of E −q and E +q
Let it is placed in a uniform electric field E such that
are equal and opposite to each other so they cancel out.
the axis

of the dipole makes an angle q with the electric
The horizontal components will add up
field E . Then the forces on the two charges are
  →
F1 = +q E | E | = E–q cos θ + E+q cos θ (equal magnitudes)
 
and F2 = –q E 2kq
= (r 2 + a 2 ) ⋅ cos θ 1
+q E
B F1 = F + qE
a
cos θ = 1
(r 2 + a 2 ) 2
2a

base
[cos θ = ]
F2 = –qE A  hypotenuse
–q C
→ 2kq a
|E| = ×
The net force acting upon the dipole is zero. Since, the 2
(r + a ) 2 1
magnitudes of two forces are equal and separated by a (r 2 + a 2 ) 2
distance 2a, the torque on the dipole is given by
2a × q × k pk
Torque t = Force × ^ distance between the line of action = 1
= 3  1
of two forces 2 2
(r + a ) (r + 2
a2 ) 2 2
(r + a2 ) 2
\ t = (qE) × (BC) or t = qE 2a sin q 8. Consider an electric dipole consisting of charges –q and
Since dipole moment +q separated by a distance 2a.
p = q(2a) Let P be a point on axis of the dipole
\ t = pE sin q r
  
\ τ = p×E A B

→ –q O +q P
7. E at an equatorial point of a dipole. Consider a dipole 2a
of charges –q and +q separated by a distance 2a placed
in vacuum. Let P be a point on the equatorial axis of   AB = 2a, pt is from negative charge to positive charge,
dipole. O is centre.

Electric Charges and Fields 25


D:\EG_Physics-12_(26-06-2022)\Open_Files\Ch-1\Ch-1
\ 27-Jul-2022 Ved_Goswami Proof-4 Reader’s Sign _______________________ Date __________

→ Resultant will be alongwith bisector of the angle


E at P due to A. between p1 and p2 as p1 = p2. So the resultant will make
1 q an angle of 60° with Y axis or 30° with X axis as electric
( − p)

E −q =
4πε 0 (AP)2 field is along x axis and pr is at 30° to x axis.
1 q Torque τ = pE sin 30°
= ( − p) ...(1)
4πε 0 (r + a )2 pE
τ=  1
2
1 q 1 q
( p) 

E +q = (+ p) = Direction of torque is along negative Z-direction.
4πε 0 (BP) 2 4πε 0 (r − a)2
10. Dipole in uniform electric field: Consider a dipole of
...(2) 1 → →
dipole moment p placed in a uniform electric field E
→ → →
E net = E − q + E + q making an angle of q with it.
F on –q charge =qE ( − i )
1 q 1 q
= ⋅ (− p) + ( p ) F on +q charge =qE (+ i )  1
4πε0 (r + a) 2 4πε0 (r − a ) 2
→ → →
q   −1 1  F net = F − q + F + q = qE( −i ) + qE( + i ) = 0
= ( p)  2
+ 
4πε 0  (r + a) (r − a)2  No translatory motion.
q   − (r − a)2 + (r + a)2  B
= ( p)  2 2 
+q qE
4πε 0  (r + a) (r − a) 

2a
q   − r 2 − a 2 + 2ar + r 2 + a 2 + 2ar 

E

= ( p)   A
4πε 0  (r + a)2 (r − a)2  qE –q C
   1
q   4ar 
= ( p)  2 2
 1
4πε 0  (r + a) (r − a)  Torque on dipole placed in uniform


q   4ar 
= ( p)  2 E : The two forces acting on the charges are equal,
2 2
4πε 0  (r − a )  parallel, opposite and their lines of action are different.
q × 2a × 2r × p
They form a couple. This couple will rotate the dipole.
= Torque = Force × distance between forces
4πε 0 (r 2 − a 2 ) 2
→ →
= qE × BC = qE × AB sin θ
E net = 2 pr 2k p r  1 = qE × 2a sin θ = pE sin θ
=
4πε 0 (r 2 − a 2 ) 2 (r 2 − a 2 )2 → → →
and τ = p×E 1
Electric field is in the direction of dipole moment.
Direction of torque perpendicular to both p and E.
Special case: 11. (a) p1: stable equilibrium
If r >> a p2: unstable equilibrium ½


2p The electric field, on either side, is directed towards
Then E =
4πε 0 r 3 the negatively charged sheet and its magnitude
is independent of the distance of the field point
→ 1 from the sheet. For position p1, dipole moment and
E ∝
r3 electric field are parallel. For position p2, they are
9. Resultant dipole moment antiparallel. ½+ ½
1 (b) The dipole will not be in equilibrium in any of the
pr = (p12 + p22 + 2 p1 p2 cos θ) 2  1 two positions. ½
p1 = p2 = p, q = 120° The electric field due to an infinite straight charged
pr = (p2 + p2 + 2p2 cos 120°)1/2 wire is non-uniform (E ∝ 1/r). ½
1 Hence there will be a net non-zero force on the dipole
  1  2 in each case. ½
=  2 p 2 + 2 p 2  −   = p 1
  2  [CBSE Marking Scheme, 2018-19]

26 Physics–12
D:\EG_Physics-12_(26-06-2022)\Open_Files\Ch-1\Ch-1
\ 27-Jul-2022 Ved_Goswami Proof-4 Reader’s Sign _______________________ Date __________


12. (a) Dipole moment | P | = q.2a ( ) Due to couple, dipoleis rotated in anticlockwise
direction. Draw AC ⊥ E
where, q = magnitude of either charge \ ⊥r distance between the forces
2a = distance between charges = arm of couple = AC
If charge q gets larger, and the distance 2a gets
 As, τ = moment of the couple
smaller and smaller, keeping the product P = q.(2a)
= force × arm of couple
constant, we get what is called an ideal dipole or
point dipole. = f × AC = f × AB sinq
Some biomolecules of proteins behaves as an ideal = f × 2a sinq = (qE × 2a) sinq
electric dipole. = (q × 2a) E sinq

(b) E    or, t = pEsin q
  
In vector form, τ = p × E

2a (c) (i) given: E = 105 N / C
F= qE
B (+ q) 2a = 2cm
q
q = 60°
(– q) O q
t = 8 3 Nm


A F= – qE C

  t = pE sin q
E
       8 3 = 2a × q × 105 sin 60°

p (Dipole moment)
−2 5 3
       8 3 = 2 × 10 × q × 10 × 2
\ q = 8 × 10–3C
(ii) Potential energy, P.E = –pE cos q
= – 2a × q × E × cos 60°


 (torque)
1
Since E is uniform, net force on the dipole is: = – 8 × 10–3 × 2 × 10–2 ×105×
2
(qE – qE) = 0 \ P.E = –8J

Topic 3. Gauss’s Law and its Applications


•• Electric flux: It is the measure of number of electric → → qenclosed
field lines crossing that area, normally. φ= ∫ E ⋅ dS = ε0
→ → s
Mathematically df = E ⋅ dS
•• Gaussian surface: Any closed surface imagined
Electric flux is a scalar quantity. S.I. unit of electric around the charge distribution, so that Gauss theorem
flux is Nm2C–1 or Vm. can be conveniently applied to find electric field due to
Gauss Law the given charge distribution.
•• Gauss theorem: The total flux through a closed •• Electric field due to infinitely long straight charged wire
1 of linear charge density λ:
surface, enclosing a volume, in vacuum is, times
the net charge, enclosed by the surface. ε 0 λ
E = , where r is the perpendicular distance of

E 2πε 0 r
the observation point from the wire.

dS

E nt
S
q
Spherical
Gaussian +++++++
surface

Electric Charges and Fields 27


D:\EG_Physics-12_(26-06-2022)\Open_Files\Ch-1\Ch-1
\ 27-Jul-2022 Ved_Goswami Proof-4 Reader’s Sign _______________________ Date __________

Gaussian surface is a cylinder at the axis of which is 1 q


placed a line charge. Outside the shell, E = for r > R
4πε 0 r 2
According to Gauss theorem
q Inside the shell, E =0 for r < R
# E $ ds = ε0 ...(1) q
At the surface, E= , r=R
S

Let l be the linear charge density, 4πε 0 R 2


q
l = l ⇒ q = ll...(2) Let s be the surface charge density
q
LHS of Gauss theorem = s= ...(1)
4πR 2
= # E ds cos 0° = E # ds First Case: When reference point is outside the shell r
= E (2prl)...(3) > R. Gaussian surface is a sphere of radius r.
Use (2) and (3) in (1) + + +
+ +
λl + +
E (2prl) = ε + +
0
+ +
λ
E= +
r′ R
+ E
2πr ε 0 + +
nt
r
If l > 0, the electric field is outwards, and if l < 0 the + +
electric field is inwards. + +
+
1 + + + +
Variation of E with r: as E ∝ r
q
S
# E $ ds = ε0
q
E(4pr2) = ε0 ...(2)
E
Use (1) in (2)
E ^ 4π r 2 h =
4π R 2 σ
ε0
r R2 σ
E=  ...(3)
r2 ε0
•• Electric field due to an infinite plane sheet of charge of
surface charge density σ. E µ 12 
r
σ Second Case: When reference point is inside the shell
    E = r′ < R
2ε 0
Gaussian surface is a sphere of radius r < R.
+ +
+ +
E + E q
+ +
+
+
+ + # E $ ds = ε0 
A +
+ + + A S

+
But q=0
Let s be the surface charge density E=0
q Third Case: When the reference point is on the surface
s= A ⇒ q = sA...(1)
r = R.
Gauss Theorem
q put r = R in eqn (3)
# E $ ds = ε0 R2 σ σ
q E= = ε0
2EA = ε0 R2 ε0

σA •• Variation of E with r.
2EA = ε  [using (1)]
0

σ
E = 2ε s /e0
0

The electric field is independent of the distance from E Eµ 12


r
the sheet.
•• Electric field due to a spherical shell of surface charge E=0 r=R
r
density σ and radius R:

28 Physics–12
D:\EG_Physics-12_(26-06-2022)\Open_Files\Ch-1\Ch-1
\ 27-Jul-2022 Ved_Goswami Proof-4 Reader’s Sign _______________________ Date __________

EXERCISE 1.3
I. Objective Type Questions (1 Mark) (c) the number of flux lines entering the surface
1. Choose the correct answers from the given options must be equal to the number of flux lines
(MCQs). leaving it.
(i) The electric flux through the surface (d) all charges must necessarily be outside the
surface.
(iv) A charge q is placed at the point of intersection of
body diagonals of a cube. The electric flux passing
through any one of its face is
[CBSE S.P. 2019-20]
q 3q
(a) (b)
(a) in fig. (iv) is the largest. 6ε 0 ε0
(b) in fig. (iii) is the least. 6q q
(c) (d)
(c) in fig. (ii) is same as fig. (iii) but is smaller ε0 3ε 0
than fig. (iv). (v) The electric flux through a closed Gaussian
(d) is the same for all the figures. surface depends upon
(ii) Five charges q1, q2, q3, q4 and q5 are fixed at their (a) net charge enclosed and permittivity of the
positions as in the figure. S is a Gaussian surface. medium.
The Gauss’s law is given by (b) net charge enclosed, permittivity of the
medium and the size of the Gaussian surface.
(c) net charge enclosed only
(d) permittivity of the medium only.
(vi) If the electric flux through a closed surface is zero,
then we can infer [Delhi, All India 2020]
q (a) no net charge is enclosed by the surface.
∫ E.ds = ε0 (b) uniform electric field exists within the surface.
s
(c) electric potential varies from point to point
Which of the following statements is correct?
inside the surface.
(a) E on the L.H.S. of the above equation will (d) charge is present inside the surface.
have a contribution from q1, q5 and q3, while
(vii) The magnitude of electric field due to a point
q on the R.H.S. will have a contribution from
charge 2q, at distance r is E. Then the magnitude
q2 and q4 only.
of electric field due to a uniformly charged thin
(b) E on the L.H.S. of the above equation will spherical shell of radius R with total charge q at
have a contribution from all charges while q r
on the R.H.S. will have a contribution from a distance (r >> R) will be [CBSE 2022]
2
q2 and q4 only.
E
(c) E on the L.H.S. of the above equation will (a) (b) 0 (c) 2E (d) 4E
4
have a contribution from all charges while q
(viii) A square sheet of side ‘a’ is lying parallel to XY
on the R.H.S. will have a contribution from
plane at z = a. The electric field in the region is
q1, q3 and q5 only. 
E = cz 2 k . The electric flux through the sheet is
(d) Both E on the L.H.S. and q on the R.H.S. will
 [CBSE 2022]
have contributions from q2 and q4 only.
1 3 1 4
(iii) If ∫s E.ds = 0 over a surface, then (a) a4c (b) a c (c) a c (d) 0
3 3
(a) the electric field inside the surface and on it
is zero. 2. Fill in the blanks.
(b) the electric field inside the surface is (i) ............... is the measure of number of electric
necessarily uniform. field lines crossing that area, normally.

Electric Charges and Fields 29


D:\EG_Physics-12_(26-06-2022)\Open_Files\Ch-1\Ch-1
\ 27-Jul-2022 Ved_Goswami Proof-4 Reader’s Sign _______________________ Date __________

(ii) Electric flux is a ............... quantity. II. Very Short Answer Type Questions (1 Mark)
(iii) Electric field due to an infinite plane sheet of 1. Define the term ‘electric flux’.Write its S.I. unit.
charge = ............... . [Delhi 2018]
(iv) A point charge is placed at the centre of a hollow 2. Is electric flux a scalar or a vector?  [Delhi 2018]
conducting sphere of internal radius ‘r’ and outer 3. Name a physical quantity whose unit is Vm.
radius ‘2r’. The ratio of the surface charge density 4. Two charges of magnitude –2Q and +Q are located
of the inner surface to that of the outer surface will at points (a, 0) and (4a, 0) respectively. What is the
be ............... . [Delhi 2020] electric flux due to these charges through a sphere of
(v) Electric flux through a spherical surface shown in radius ‘3a’ with its centre at origin? [AI 2013]
the figure, is ............... . [Delhi 2020] 5. A charge q is placed at the centre of a cube of side l.
What is the electric flux passing through each face
• –q1 of the cube? [AI 2012]
• +q3 6. What is the electric flux through a cube of side 1 cm
• +q2 which encloses an electric dipole. [Delhi 2015]
7. The figure shows three charges +2q, – q and +3q.
3. State True or False Two charges +2q and –q are enclosed within a surface
(i) If electric flux through a closed surface is zero. ‘S’. What is the electric flux due to this configuration
It means that the net charge inside the surface is through the surface ‘S’? [Delhi 2010]
zero.
(ii) The position of the charge inside the enclosing 2q S
surface is changed in such a way that the total +3q –q
charge remains constant. Then the total normal
electric flux through the enclosing surface
decreases. 8. An arbitrary surface encloses a dipole. What is the
4. Match the Columns electric flux through this surface? [NCERT Exemplar]
Column-I    Column-II 9. If Coulomb’s law involved 1/r3 dependence (instead
dq of 1/r2), would Gauss’s law be still true?  [NCERT]
(i) Linear charge density (a)
dA III. Short Answer Type Questions-I (2 Marks)
dq 10. A spherical conducting shell of inner radius r1 and
(ii) Surface charge density (b) outer radius r2 has a charge Q. A charge q is placed
dl
  at the centre of the shell,
(iii) Volume charge density (c) E ⋅ d S
(a) What is the surface charge density on the (i) inner
dq surface, (ii) outer surface of the shell?
(iv) Electric flux (d)
dV (b) Write expression for the electric field at point
For question number 5 two statements are given-one x > r2 from the centre of the shell. [AI 2010]
labelled Assertion (A) and the other labelled Reason
11. Charges of magnitude 2Q and –Q one located at points
(R). Select the correct answer to these questions from
(a, 0, 0) and (4a, 0, 0). Find the ratio of the flux of
the codes (a), (b), (c) and (d) as given below.
electric field, due to these charges, through concentric
(a) Both A and R are true and R is the correct
spheres of radii 2a and 8a centred at the origin.
explanation of A
[S.P. 2011]
(b) Both A and R are true but R is NOT the correct
explanation of A 12. A charge q is placed at the open end of a cylinder of
length l and radius r as shown in the figure.
(c) A is true but R is false
Find the electric flux through this cylinder.
(d) A is false and R is also false l
5. Assertion (A): Using Gauss law, it is possible to find
q
the electric field at any point. r

Reason (R): Gauss law is applicable for any type of


13. Obtain Coulomb’s laws of electrostatics using Gauss
charge distribution.
theorem.
30 Physics–12
D:\EG_Physics-12_(26-06-2022)\Open_Files\Ch-1\Ch-1
\ 27-Jul-2022 Ved_Goswami Proof-4 Reader’s Sign _______________________ Date __________

14. The flux of electrostatic field through the closed σ 


spherical surface S′ is found to be four times that n, where n is a unit vector in the outward normal
2ε 0
through the closed spherical surface S. Find the
direction, s is the surface charge density near the hole.
magnitude of the charge Q. Given q 1 = 1 µC,
[NCERT]
q2 = –2µC, q3 = 9.84 µC.
21. (i) Obtain an expression for electric field at a point
Q due to an infinite line charge distribution using
q1 S S′ Gauss theorem.
1m
q3 (ii) Show the variation of electric field with distance
q2 r (infinitely long line charge).
22. Obtain an expression for electric field at points A, B
and C due to two oppositely charged thin, infinitely
15. A point charge q is at a distance of d/2 directly above long parallel plates.
the centre of a square of side d, as shown in the +s –s
figure. Use Gauss’ law to obtain the expression for • • •
the electric flux through the square. [Delhi 2018] A B C
q
23. A hollow cylindrical box of length 1 m and area of
cross-section 25 cm2 is placed in a three dimensional
d/2
co-ordinate system as shown in the figure. The electric

d field in the region is given by E = 50xi where E is in
NC–1 and x is in metre. Find
y
d
16. If the point charge is now moved to a distance ‘d’ from
the centre of the square and the side of the square is I II
doubled, explain how the electric flux will be affected. o
x

[Delhi 2018] 1m
z
17. Use Gauss’ law to derive the expression for the 1m

electric field (E ) due to a straight uniformly charged (i) net flux through the cylinder
infinite line of charge density l C/m. [Delhi 2018] (ii) charge enclosed by the cylinder. [Delhi 2013]
IV. Short Answer Type Questions-II  (3 Marks) 24. (a) Consider an arbitrary electric field configuration.
18. Given the components of an electric field as Ex = αx, A small test charge is placed at a null point of the
Ey = 0 and Ez = 0, where α is a dimensional constant. configuration. Show that the equilibrium of the
Calculate the flux through each face of the cube of test charge is necessarily unstable.
side ‘a’, as shown in the figure, and the effective (b) Verify this result for the simple configuration
charge inside the cube. [AI 2015] of two equal and like charges placed a certain
y distance apart. [NCERT]
a
25. Two large charged plane sheets of charge densities
n I II n s and –2s C/m2 are arranged vertically with a
separation of d between them. Deduce expressions
x
a a for the electric field at points (i) to the left of the
z
a first sheet, (ii) to the right of the second sheet, and
(iii) between the two sheets. [AI 2019]
19. Find the electric field intensity due to a uniformly
26. A spherical conducting shell of inner radius r1 and
charged spherical shell at a point (i) outside the shell
outer radius r2 has a charge Q.
and (ii) inside the shell. Plot the graph of electric field
with distance from the centre of the shell.[AI 2016] (a) A charge q is placed at the centre of the shell.
Find out the surface charge density on the inner
20. A hollow charged conductor has a tiny hole cut into
and outer surface of the shell.
its surface. Show that the electric field in the hole is

Electric Charges and Fields 31


D:\EG_Physics-12_(26-06-2022)\Open_Files\Ch-1\Ch-1
\ 27-Jul-2022 Ved_Goswami Proof-4 Reader’s Sign _______________________ Date __________

(b) Is the electric field inside a cavity (with no charge) (b) Two point charges of + 1 µC and +4 µC are kept 30
zero; independent of the fact whether the shell is cm apart. How far from the + 1 µC charge on the
spherical or not? Explain. [AI 2019] line joining the two charges, will the net electric
V. Long Answer Type Questions (5 Marks) field be zero? [AI 2020]
27. (a) Using Gauss’s law, derive expression for intensity OR
of electric field at any point near the infinitely long (a) Two point charges q1 and q2 are kept r distance

straight uniformly charged wire. apart in a uniform external electric field E . Find
(b) The electric field components in the following the amount of work done in assembling this
figure are Ex = αx, Ey = 0, Ez = 0; in which α = 400 system of charges.
N/C m. Calculate (i) the electric flux through the (b) A cube of side 20 cm is kept in a region as shown
cube, and (ii) the charge within the cube assume →
that a = 0.1m. 5 in the figure. An electric field E exists in the
y region such that the potential at a point is given by
V = 10x + 5, where V is in volt and x is in m.
a y
nL nR
x
a a
a
z
[CBSE S.P. 2019-20] x
O
28. (a) Using Gauss law, derive expression for electric
field due to a spherical shell of uniform charge
distribution s and radius R at a point lying at a z
distance x from the centre of shell, such that
Find the
(i) 0 < x < R and (ii) x > R →
(b) An electric field is uniform and acts along +x (i) electric field E , and
direction in the region of positive x. It is also (ii) total electric flux through the cube.
uniform with the same magnitude but acts in –x  [AI 2020]
direction in the region of negative x. The value 30. (a) State Gauss’s law in electrostatics. Show
of the field is E = 200 N/C for x > 0 and E = –200 that with help of suitable figure that outward
N/C for x < 0. A right circular cylinder of length flux due to a point charge Q, in vacuum
20 cm and radius 5 cm has its centre at the origin within gaussian surface, is independent of its size
and its axis along the x-axis so that one flat face and shape.  5
is at x = + 10 cm and the other is at x = – 10 cm. (b) In the figure there are three infinite long thin sheets
Find having surface charge density +2σ, –2σ and +σ
(i) The net outward flux through the cylinder. respectively. Give the magnitude and direction
(ii) The net charge present inside the cylinder. of electric field at a point to the left of sheet of
[AI 2020] charge density +2σ and to the right of sheet of
29. (a) Use Gauss’s law to show that due to a uniformly charge density +σ. [CBSE S.P. 2020-21]
charged spherical shell of radius R, the electric
2s –2s s
field at any point situated outside the shell at a
distance r from its centre is equal to the electric
field at the same point, when the entire charge on
the shell were concentrated at its centre. Also plot A B C D
the graph showing the variation of electric field
with r, for r ≤ R and r ≥ R.

Answers 1.3
I. Objective Type Questions (iv) (a) (v) (a) (vi) (a) (vii) (c)
1. (i) (d) (ii) (b) (iii) (c) and (d) (viii) (a)

32 Physics–12
D:\EG_Physics-12_(26-06-2022)\Open_Files\Ch-1\Ch-1
\ 27-Jul-2022 Ved_Goswami Proof-4 Reader’s Sign _______________________ Date __________

charge –q is induced on the inner surface and +q on


2. (i) Electric flux (ii) scalar (iii) σ the outer surface of the shell. So the total charge on
2ε 0
(iv) –4 (v) q2 – q1 / e0 the outer surface is Q + q.
3. (i) True (ii) False Surface charge density on inner surface
4. (i)- (b) (ii)- (a) (iii)- (d) (iv)- (c) q
= – .
5. (c) 4pr12
II. Very Short Answer Type Questions Surface charge density on outer surface
Q+q
1. Electric flux: It is the measure of number of electric =  1
4πr2 2
field lines crossing that area normally.
(ii) For all points at distance x > r2, it behaves, as if the
Mathematically the electric flux passing through an area charge is placed at its centre

dS is 1 (Q + q )
E(x) =  1
→ → 4πε 0 x 2
dφ = E ⋅ dS Q+q
SI unit of electric flux is Nm2C–1 or Vm.
r2
2. It is a scalar quantity.
–q
3. Electric flux.
q r1
4. As the charge placed at (4a, 0) will not be enclosed by
the sphere of radius 3a.
−2Q
\ φ =
ε0 11.

–2Q +Q
2Q –Q
(0,0) a 2a 3a 4a
o
q (a,0,0) 2a 3a (4a,0,0)
5. φcube =
ε0
A cube has 6 faces so flux through one face Flux through a sphere of radius 2a
q 2Q
φface = φ1 =  ½
6ε 0 ε0
6. Zero, because a dipole consists of two equal but Flux through the sphere of radius 8a
opposite charges. So net charge = 0. 2Q − Q Q
φ2 = =  ½
7. According to Gauss theorem ε0 ε0
q φ1 2Q × ε 0
φ = enclosed = = 2 1
ε0 φ2 ε0 × Q
2q − q q 12. Construct a cylinder of length l and radius r on the left
φ= =
ε0 ε0 side of the charge q such that charge q is enclosed at
the centre of a cylinder of length 2l and radius r.
8. According to Gauss theorem
According to Gauss theorem
→ → q
φ= ∫ E ⋅ dS = ε0 Total flux φ =
q
 1
S ε0
Total charge of a dipole = 0 So the flux through the given cylinder
So electric flux = 0. q
=  1
9. No, Gauss’s law will not be true, if coulomb’s law 2ε 0
1
involved 3 dependence instead of 1 dependence.
r r2 q
10. (i) Charge Q resides on the outer surface of the
conducting shell. Due to q placed at its centre, a
l l

Electric Charges and Fields 33


D:\EG_Physics-12_(26-06-2022)\Open_Files\Ch-1\Ch-1
\ 27-Jul-2022 Ved_Goswami Proof-4 Reader’s Sign _______________________ Date __________

13. Let q1, q2 be the 2 point charges separated by a distance → → → →


r in vacuum.     Df = E . ∆S or f= ∫ E .d S
Consider the Gaussian surface to be a sphere of radius
r and centred at q1.
     f = ∫ E d S cos θ  1

Consider a small area element ds Electric flux is a scalar quantity.


Imagine a cube of side d, enclosing a charge q. The
ABCD square surface is one of the six faces of the
→ imaginary cube. Total electric flux through the cube
O P E
r q
q1 q2 →      f =
ds ε0

→ → → →
φ= ∫ E⋅ ds = ∫ Eds cos 0° = E ∫ ds (1)
S S S
= E× ...(1) 4πr2 O
q
According to Gauss theorem d d/2 C
B
q d
φ = 1 ...(2) 1
ε0
From (1) and (2) A d D

q 1 q1 Electric flux through ABCD surface


E × 4πr2 = 1 ⇒ E =
ε0 4πε 0 r 2 q
F = qE = q2E [at Point P] =  1
6ε 0
1 q1 1 q1q2 16. The electric flux remains the same because it depends
F = q2 × 2
=  1
4πε 0 r 4πε 0 r 2 only upon the charge enclosed and not on the size of
14. φS′ = 4φS given the surface. 1+1
φS = q1 + q2 + q3 17. Electric field due to straight uniformly charged infinite
ε0 long wire of charge density l C/m.
q1 + q2 + q3 + Q
φS′ =  1
ε0

Q
q1 S S′
1m
q3
q2

According to question, we have


φS′ = 4φS
q1 + q2 + q3 + Q 4(q1 + q2 + q3 ) Consider an infinite long thin wire of charge density
= C/m.
ε0 ε0
Q = 3(q1 + q2 + q3) 1 Total electric field at point P is radial. As the wire is
infinite long, electric field depends upon distance of
= 3(1 – 2 + 9.84) µC = 26.52 µC
point P from the wire.
15. Electric flux in an electric field over an area is a measure
Let us consider the cylindrical Gaussian. Surface of
of number of electric field lines crossing the area.

radius r and length l with the infinitely long line of
Electric flux Df through an area element ∆S in an charge at its axis. Electric field at any point on the
→ curved surface of the cylindrical Gaussian surface.
electric field E is
34 Physics–12
D:\EG_Physics-12_(26-06-2022)\Open_Files\Ch-1\Ch-1
\ 27-Jul-2022 Ved_Goswami Proof-4 Reader’s Sign _______________________ Date __________

\ Electric flux over the curved surface.


Gaussian surface
→ → +
∫ E .ds = ∫ E ds cos 0° = ∫ E ds + +
Curved surface S
O R
Curved surface = E.2prl ds
→ + r + 
as the circular surface angle between the E and outward + P
E

n is 90°
→ →
Let P be a point at distance ‘r’ from centre of shell
= ∫ E.ds cos 90° = 0  r > R.

\ ∫ E .ds 1
Circular surface Consider the Gaussian surface to be a sphere centred
\ Total flux over the whole cylindrical Gaussian surface at O and passing through P.
→ →
f = E(2prl) =
q φ= ∫ E ⋅ dS = ∫S E ⋅ dS cos 0° = E ∫S dS
ε0 S

(According to Gauss Theorem) = E × 4πr2...(1)


λl According to Gauss theorem
E.2prl = q
ε0 φ= ...(2)
ε0
λ 1 where q is the charge on the shell
or E= ⇒ E∝  1
2πε 0 r r q
E × 4πr2 =
ε0
18. Given Ex = αx, Ey = 0, Ez = 0
→ → q q
As electric field has only x-component, therefore, E ⋅ ∆S E= 2
, σ= ⇒ q = Aσ
4πε 0 r A
= φ = 0 for each of the four faces of the cube ⊥ to y-axis
1 1
and z-axis. Flux is there only through face I and II. E= 2
× σA = × σ 4πR2
4πε 0 r 4πε 0 r 2
At the face I
x= a σR 2
=  1
ε0 r 2
So EI = aa 1
2 2 3 For point P lying inside the shell
(ii)
fI = EI(a ) cos 180° = – αaa = –αa
At the face II Consider the Gaussian surface to be a sphere of
radius OP = r and centred at O.
y
q = charge enclosed by Gaussian surface = 0
a According to Gauss theorem
I II n q → → → → q
n 1 f= = ∫ E ⋅ dS or ∫ E ⋅ dS =
x ε0 ε0
a a
a +
z + +

x = 2a + +
So EII = α(2a) r P
+ O +
fII = EIIa2 cos 0° = 2αaa2 = 2αa3
R
fnet = fI + fII = – αa3 + 2αa3 = αa3 1 + +

19. E due to uniformly charged thin spherical shell +
+
+

(i) For point P outside the shell


E 4πr2 = 0 as q = 0 1
Consider thin spherical shell of radius R with
E= 0
uniform surface charge density σ.

∴ There is no electric field inside the shell.
Electric Charges and Fields 35
D:\EG_Physics-12_(26-06-2022)\Open_Files\Ch-1\Ch-1
\ 27-Jul-2022 Ved_Goswami Proof-4 Reader’s Sign _______________________ Date __________


Show variation of E with distance from centre of ds1

charged shell of radius R. S1 


E
+
+
+
E +
l + r
   1

E
s3 P
+ ds3


+
R +
(Radius of distance from the
S2 +
centre of the shell → r

shell) E
20. If the tiny hole in the conductor was filled then the ds2
electric field intensity close to the surface of the
(ii) 
conductor according to Gauss theorem
→ → σ
∫ E ⋅ dS = ε0  1
E 1
σdS σ
E dS = or E =
ε0 ε0
σ → r
i.e., E =
n 1 22. Let charge densities be +σ and –σ
ε0
This field is the superposition of fields due to the filled In region A fields are equal and opposite.
up hole and the field due to the rest of the charged → → σ σ 
E = E1 + E 2 = ( −i ) + (i )
conductor. ε0 ε0
As there is no electric field inside a conductor so the
σ 
two fields inside the conductor are equal and opposite. = (i ) [–1 + 1] = 0 1
But outside the conductor electric field is not zero, so ε0
In region B
they must be in same direction.
→ → → σ(i ) σ(i ) σ 
1 σ  E = E1 + E2 = + = (i )
Electric field due to each part = E = n 1 2ε 0 2ε 0 ε 0
2 2 ε0
In region C
21. (i) Consider a thin infinitely long straight wire having → → → σ  σ
uniform linear charge density λ C/m. E = E1 + E2 = (i ) + ( − i )  1
Consider the Gaussian surface to be a cylinder of ε0 ε0
radius r and length l and with its axis along the
= σ (i ) [1 – 1] = 0
line charge. ε0
→ → → → → → → →
φ = ∫ E ⋅ ds = ∫ E ⋅ ds1 + ∫ E ⋅ ds 2 + ∫ E ⋅ ds3 (+) σ (–) σ
S

= ∫ E ds1 cos 90° + ∫ Eds2 cos 90° + ∫ Eds3 cos 0° E1 E1 E1

= 0 + 0 + E ∫ ds3 A B C
E2 E2 E2
φ = E ∫ ds3 = E × 2πrl 1
According to Gauss theorem, 1 2 1
q → →
23. (i) Flux through face I = 50x × 25 × cos 180° 10–4
flux φ = ∫ E ⋅ ds = enc.
S
ε0 [φ = ES cos q]
λl As electric field is along positive x axis and area
φ = E × 2πrl = , vector along negative x axis.
ε0
So q = 180°
λ = linear charge density
and x= 1m
λ 1
E= E∝  1 So φI = –ES = –1250 × 1 × 10–4
2πr ε 0 r = –0.125 Nm2C–1 ½
36 Physics–12
D:\EG_Physics-12_(26-06-2022)\Open_Files\Ch-1\Ch-1
\ 27-Jul-2022 Ved_Goswami Proof-4 Reader’s Sign _______________________ Date __________

y σ 2σ σ
In region I Enet = − =  1
2ε 0 2ε 0 ε 0
σ σ σ + 2σ 3σ
I II In region II Enet = − = =  1
x 2ε 0 ε 0 2ε 0 2ε 0
o
1m σ σ σ − 2σ σ
In region III Enet = − = =−  1
z 1m 2ε 0 ε 0 2ε 0 ε0
For face II, angle between electric field and area 26. (a) The charge + Q resides on the outer surface of the
vecter is 0°. shell. As the charge q is placed at the centre of the
φII = ES = 50x × 25 × 10–4 shell, a charge –q is induced on the inner surface and
x=2m a charge + q is induced on the outer surface of the
So φII = 50 × 2 × 25 × 10–4 = 0.25 Nm2 C–1 ½ shell. Thus, the total charge on the inner surface of
Flux through the curved surface is zero as electric the shell is – q and on the outer surface of the shell
field is ⊥ to area vector. is (Q+ q).
So φnet = φI + φII = 0.25 – 0.125 = 0.125 Nm2C–11 The surface charge density on the inner surface
q q
(ii) As φnet = or q = φnet × e0 s1 =  ½
ε0 4πr12
= 0.125 × 8.85 × 10–12 = 1.1 × 10–12 C 1 +q
+q
24. (a) Suppose that the test charge is in stable equilibrium –q
when it is displaced from the null point, in any O r1
direction, the restoring force must act on it to  ½
bring it back to the same position. Thus there is net q
Q
inward flux of electric field through closed surface r2
(imagined) around the null point. But according to
The surface charge density on the outer surface
Gauss theorem, net electric flux must be zero as no
charge is enclosed. Hence the equilibrium cannot be Q+q
s2 =  ½
stable. 1 4 πr22
(b) Consider two like charges placed at A and B (b) Yes. Inside the cavity without any charge net electric
Let P be the mid point of the line joining the charges. field is always zero according to Gauss law. The
EP = 0 electric field intensity inside a cavity is zero, even
A B if the shell is not spherical and has any irregular
+q P +q shape. Take a closed loop such that a part of it is
FA
inside the cavity along a field line and the rest is
Resultant force outside the conductor. The net work done by the
FB field in carrying a test charge over a closed loop is
1 zero because the field inside the conductor is zero.
So, the electric field inside a cavity with no charge
q
+q
A P B

If we displace the test charge towards say right i.e. is always zero whatever is the shape. 1½
towards B, the repulsive force of B will be more than 27. (a) cm–1 ds2
the repulsive force of A so the net force on the test
charge is towards A. So a restoring force tends to
bring the charge back to the original position. The + E
+
something will happen, if we displace it towards +
+
left. But when test charge is displaced normal to the +
line joining the charges, a net force acts on the test +
+ ds1 1
L r
charge, which will take it further away from the null + E
+
point. Hence equilibrium cannot be stable. 1 +
25. I II III E +
+
– E1 + + – – +E1 E +
+ + + E1 – –
+ E2 + + + E2 – – – E2 ds3
side  – 2

Electric Charges and Fields 37


D:\EG_Physics-12_(26-06-2022)\Open_Files\Ch-1\Ch-1
\ 27-Jul-2022 Ved_Goswami Proof-4 Reader’s Sign _______________________ Date __________

According the Gauss’s law – S1 and S2 are two Gaussian spheres respectively for
 
 1 points P1 (x < R) and P2 (x > R)
∫ E . ds = (q)  ½
(i) By Gauss’s Law,
ε0
 
  
  
 1 Net outward flux through S1
∫ Eds1 + ∫ Eds 2 + ∫ Eds3 = ε0 (λL)   q
Eds cos 0° + Eds cos 90° + Eds cos 90°
f=  ∫ ⋅ dA = ε 1 = 0
E
∫ 1 ∫ 2 ∫ 3 S1 0

λL where, q1 = charge enclosed by S1


=  ½ ⇒ E= 0 1
ε0
(ii) Net outward flux through S2
λL
E ∫ ds1 =   q
ε0 f=  ∫ ⋅ dA = ε 2
E
S2 0
λL λ
E × 2prL = ⇒ E= where, q2 = charge enclosed by S2
ε0 2πε 0 r
 λ 
=
(
σ 4πR 2 ) [ q = sA] 1
E = r 1
2πε 0 r ε0
y
⇒ ∫
E

dA =
σ 4πR 2 ( )
a S2 ε0
(b) (i) nL nR  ½
(
σ 4πR 2 )
∫ dA = 4px2 ⇒ E =
x


a
a
a S2 (4πx ) ε2
0
z
2
 Ex = ax = 400 x σR

⇒ E=  1
Ey = E z = 0 ε0 x2
Hence flux will exist only on left and right faces of
(b) → →
cube as Ex≠ 0 dA 3 dA
  → →
 E ⋅ a 2 ( n ) + E ⋅ a 2 n R
L 2 R
E E
  1
E L ⋅ a 2 ( n2 ) + E R ⋅ a 2 n R = ( qin ) = f ½ 2 1
ε0
– EL· a2 (n2) + a2ER = fNet (i) f = f1 + f2 + f3
fNet = –(400a)a2 + a2 (400 × 2a) = EA + EA + 0 = 2 EA = 2Epr2
= –400a3 + 800a3 = 400a3 = 2 × 200 × 3.14 × (5 × 10–2)2
= 400 × (.1)3 = 0.4 Nm2C–1 ½ = 31400 × 10–4 = 3.14 Nm2/C 1
1 (ii) Net charge q = fe0
(ii)  fnet =
ε0
( qin ) ⇒ qin = e0 ·fnet
q = 3.14 × 8.854 × 10–12
qin = 8.85 × 10–12 × 0.4 = 27.8 × 10–12 C 1
= 3.540×10–12C ½
29. (a) When point P lies outside the spherical shell. The
[CBSE Marking Scheme, 2019-20]
total charge q inside the Gaussian surface is the
28. (a) charge on the shell of radius R and area 4pR2.
s + +
+
\ q = 4pR2s
S1 →
+ x dA Flux through the Gaussian surface,
+→ fE = E × 4pr2
+ R P1 E
S2 q
+ x + By Gauss’s theorem, fE =
ε0
+ +

P2 dA q 1 q
\E × 4pr2 =
or E = ⋅ 2
→ ε0 4πε 0 r
E [For r > R] 1

38 Physics–12
D:\EG_Physics-12_(26-06-2022)\Open_Files\Ch-1\Ch-1
\ 27-Jul-2022 Ved_Goswami Proof-4 Reader’s Sign _______________________ Date __________

E → dV
E =–
Gaussian dx
surface

∴ E = –10 Vm–1 1
→ 
E O r E= (ii) fE = E · dS
R P
= –10 × 0.24
Spherical shell, = –2.4 Vm 1
charge density = σ
30. (a) Gauss’s law in electrostatics:

The surface integral
E
of electrostatic field E produced by any sources over

This field is the same as that produced by a charge any closed surface enclosing a volume V in vacuum
q placed at the centre O. Hence for points outside i.e., total electric flux (fE) over the closed surface S
the shell, the field due to a uniformly charged shell 1
in vacuum is times the total charge (Q) contained
is as if the entire charge of the shell is concentrated ε0
at its centre. inside S, i.e.,
  Q
• When point P lies on the spherical shell. The fE = ∫s E.ds =
Gaussian surface just encloses the charged spherical ε0
shell. Applying Gauss’s theorem,  
df = E. ds
q 1 q Kq
E × 4pR2 = or E = ⋅ 2 [For r > R] 1 Kq   
ε0 4πε 0 R = r. ds = 2 r . ds . d s
r 2 r
σ
or E= [ q = 4pR2s]  ds 
ε0 \ ∫s d φ = ∫s Kq  r 2  r . d s
E E=
 ds 
f= ∫s Kq  r 2  .1
q
( )
E
r2     1 as r . d s = r d s cos 00 = 1
E=0
O R r ds 4πr 2
= Kq ∫ = Kq. = Kq.4p
s r2 r2
+1µC P +4µC 1 q
f= . q 4π =
(b) 4πε0 ε0
xm (0.3 – x)m
Q
At point P, due to equilibrium, we have f=
ε0
1 1 × 10−6 1 4 × 10 −6  (i.e., independent of shape and size)
=  1
4πε 0 x 2 4πε 0 (0.3 − x )2 (b) +2s –2s +s

(0.3 – = x)2 4x2


(0.3 – x) = ±2x
On taking +ve sign, we get A B C D

0.3 = 3x ⇒ x = 0.1m = 10 cm 1


OR
(a) Do it yourself.

(b) Given side = 20 cm = 0.2 m 1 EA =
2σ 
2ε 0
−i + ( )
2σ 
2ε 0
i +
σ
2ε 0
−i () ( )
∴ Area of cube (dS) = 6(side)2

(i) V = 10x + 5
= 6(0.2)2 = 0.24 m2 =
σ
2ε 0
−i( )
dV 

dx
= 10 ED =
σ 
2ε 0
i −()
2σ 
2ε 0
i +
2σ 
2ε 0
()
i =
σ 
2ε 0
i () ()
Electric Charges and Fields 39
D:\EG_Physics-12_(26-06-2022)\Open_Files\Ch-1\Ch-1
\ 27-Jul-2022 Ved_Goswami Proof-4 Reader’s Sign _______________________ Date __________

Case Based Questions


I. Faraday Cage: A Faraday cage or Faraday shield Gaussian surface: Any closed surface imagined
is an enclosure made of a conducting material. The around the charge distribution, so that Gauss theorem
fields within a conductor cancel out with any external can be conveniently applied to find electric field due
fields, so the electric field within the enclosure is zero. to the given charge distribution.
These Faraday cages act as big hollow conductors Electric field due to infinitely long straight charged
you can put things in to shield them from electrical wire of linear charge density λ:
fields. Any electrical shocks the cage receives, pass λ
E = , where r is the perpendicular distance of
harmlessly around the outside of the cage. 2πε 0 r
the observation point from the wire.
Electric field due to an infinite plane sheet of charge
of surface charge density σ.
σ
   E =
2ε 0
1. S.I. unit of electric flux is
(a) N2 mC (b) NmC–2
1. Which of the following material can be used to (c) Nm2C–1 (d) Nm–2C
make a Faraday cage?
2. Electric flux is a
(a) Plastic (b) Glass (c) Copper (d) Wood
(a) constant quantity (b) vector quantity
2. Example of a real-world Faraday cage is
(a) car (b) plastic box (c) scalar quantity (d) None of these
(c) lightning rod (d) metal rod 3. Two charges of magnitude –2Q and +Q are located
3. What is the electrical force inside a Faraday cage at points (a, 0) and (4a, 0) respectively. What is the
when it is struck by lightning? electric flux due to these charges through a sphere
(a) The same as the lightning of radius ‘3a’ with its centre at origin?
(b) Half that of the lightning Q – 2Q
(a) (b)
(c) Zero ε0 ε0
(d) A quarter of the lightning 3Q – 3Q
(c) (d)
4. An isolated point charge +q is placed inside the ε0 ε0
Faraday cage. Its surface must have charge equal 4. A charge q is placed at the centre of a cube of side
to l. What is the electric flux passing through each
(a) Zero (b) + q (c) – q (d) + 2q face of the cube?
5. A point charge of 2C is placed at centre of Faraday q q q q
(a) (b) (c) (d)
cage in the shape of cube with surface of 9 cm edge. 5ε 0 9ε 0 6ε 0 ε0
The number of electric field lines passing through 5. The figure shows three charges +2q, – q and +3q.
the cube normally will be Two charges +2q and – q are enclosed within a
(a) 1.9105 Nm2/C entering the surface surface ‘S’. What is the electric flux due to this
(b) 1.9105 Nm2/C leaving the surface configuration through the surface ‘S’?
(c) 2.0105 Nm2/C leaving the surface
(d) 2.0105 Nm2/C entering the surface
Ans. 1. (c) 2. (a) 3. (c) 4. (a)
5. (b)
II. Gauss theorem: The total flux through a closed
1 5q 3q 2q q
surface, enclosing a volume, in vacuum is, times (a) (b) (c) (d)
the net charge, enclosed by the surface. ε 0 2ε 0 ε0 ε0 ε0
→ → qenclosed Ans. 1. (c) 2. (c) 3. (b) 4. (c)
  f = ∫ E . dS =
ε0 5. (d)
s

40 Physics–12
D:\EG_Physics-12_(26-06-2022)\Open_Files\Ch-1\Ch-1
\ 27-Jul-2022 Ved_Goswami Proof-4 Reader’s Sign _______________________ Date __________

IMPORTANT FORMULAE
Formulae Symbols Applications
1. q = ± Ne q = Charge, N = number of Charge Quantization of charges
particle
2. q1q2 q1, q2 are point charges, To find force between two point charges
F= k F = Force
r2
3. F = qE E = Electric Field Relation between F and E
4. q 1 Electric field due to a point charge
E= k k= , r = distance
r2 4πε 0
e0 = Absolute permittivity
5.   dS = small area To find electric flux
∫ E ⋅ dS = E ∫ dS cos θ
φE = 

6. Σq f = Electric flux Gauss theorem


φE = Sq = Net Charge enclosed within the
ε0
surface
7. λ l = linear charge density Field intensity due to infinitely long
E= straight uniformly charged wire
2πε 0 R

8. q r = radius of Gaussian surface (outside Field intensity due to uniformly


(a) outside the shell: E = k 2
r the shell) charged spherical shell
(b) on the shell:
R = Radius of shell
q
E = k 2 As q = 4πR 2σ s = surface charge density
R
E = σ / ε 0
(c) inside the shell: E = 0
9. σ E = Electric field Field intensity due to thin infinite plane
(a) E = e0 = electric permittivity sheet of charge
2ε 0
(b) E. field at the equator of a dipole
1 p
   E equator =
4πε 0 r 3
(c) E. field of the axis of a dipole
1 2p
    E axis =
4πε 0 r 3
10. t = p × E = pE sin q p = dipole moment Electric dipole in a uniform electric field
t = torque
11.   2a = dipole length Dipole moment in electric field
p = q × 2a

COMMON ERRORS
S.No. Errors Corrections
1. Confusion between Electric field intensity & Electric Read the question properly before attempting.
Potential due to electric dipole
2. Method for finding the directions of Electric field. Direction of motion positive charges or decreasing
direction of electric potential
3. Area Vector concept in Gauss Theorem. Area vector is always perpendicular to the surface.

Electric Charges and Fields 41


D:\EG_Physics-12_(26-06-2022)\Open_Files\Ch-1\Ch-1
\ 27-Jul-2022 Ved_Goswami Proof-4 Reader’s Sign _______________________ Date __________

REVISION CHART

Relative permitivity is ratio of


Quantization of electric charge: force b/w two point charges placed
The charge on a body, is an integral certain distance apart in vacuum to Insulators: The substances
multiple of, a basic quantity of the force b/w the same two charges that do not allow the charges
charge, which is the charge on an placed some distance apart in the to flow through them are
electron. given medium. called insulators.

Electric charge
It is an intrinsic property of elementary particles like electrons and protons because of which they exert electrical force and are
able to respond to electrical force. Electric charge is a scalar quantity.

Methods of producing charge on Additivity of electrical charges: Conservation of charge: The


an body by The total charge on a system is the net electric charge in an isolated
(i) friction algebraic sum of all the charges in system is always constant.
(ii) Induction different parts of the body.
(iii) Conduction

Conductors: The substances through


which electric charges can flow easily
are called conductors. They contain
large number of free electrons. Electric field: Electric field
1 Coulomb: It is that amount of charge e.g. silver is the best conductor of intensity/strength is defined as the
that repels an equal and similar charge electricity. force experienced per unit +ve
with a force of 9 × 109 N, when placed ε m F0 test charge (vanishingly small)
εr = =
in vacuum at a distance of 1 m from it. ε 0 Fm placed at that point.

Coulomb’s Law of Electrostatic


The force of attraction/repulsion between two stationary point charges is directly proportional to the product of the magnitude of
the charges and inversely proportional to the square of the distance between them.

Electric Field Lines: It is a curve, Properties of electric field lines:


the tangent to which at any point (i) Continuous smooth curves
gives the direction of force acting on (ii) start from positive charge and end at a negative charge
a test charge placed at that point. (iii) they never intersect each other
(iv) do not form closed loop
(v) they are always normal to the surface of conductor
(vi) they do not pass through a conductor

42 Physics–12
D:\EG_Physics-12_(26-06-2022)\Open_Files\Ch-1\Ch-1
\ 27-Jul-2022 Ved_Goswami Proof-4 Reader’s Sign _______________________ Date __________

REVISION CHART

Electric Dipole
It is a pair of two equal and opposite charges separated by a small distance.

→ Electric field due to a dipole: torque


Dipole moment ( p ) :   
(i) At an axial point (ii) on equator τ = p×E

p = Magnitude of the charge 2 pr p
Eaxis = Eeq.=
( )
2 3
× dipole length
→ →
4πε 0 r 2 − a 2
(
4πε 0 r 2 + a 2 ) 2

p =q× 2a for r >> a


for r >> a,
It is a vector quantity. Its direction p
2p Eeq. =
is always from Eaxis = 4πε 0 r 3
4πε 0 r 3
–ve to +ve. SI unit = C–m E
 [Coulomb – metre] axis = 2
E eq

Area vector: It is the vector


Electric field due to an infinite
Electric field due to an infinite thin associated with area element
straight line of charge σ of a closed surface, it is always
λ sheet of charge E =
E= 2ε 0 taken along outward drawn
2πε 0 r normal.

Electric flux
It is the measure of number of electric field lines crossing that area, normally.
→ →
Mathematically df = E ⋅ dS
Electric flux is a scalar quantity. S.I. unit of electric flux is Nm2C–1 or Vm.

Linear Charge Density λ: It is the Applications of Gauss theorem Gauss theorem: The total flux
dq Electric field due to an infinite thin through a closed surface, enclosing
charge per unit length λ = .
dl charge spherical shell of radius r a volume in vacuum is 1 times
(a) Inside the shell ε0
E = 0 the net charge, enclosed by the
(b) E = q 2 on the surface surface.
Surface Charge Density σ: It is the 4πε 0 r
→ → qenclosed
amount of charge per unit surface area (c) Out side the shell i.e. R > r, f= ∫ E ⋅ dS =
q ε0
dq s
σ= . E =
dA 4πε 0 R 2

Volume Charge Density r: It is the


charge per unit volume of the body
dq
ρ= .
dV

Electric Charges and Fields 43


D:\EG_Physics-12_(26-06-2022)\Open_Files\Ch-1\Ch-1
\ 27-Jul-2022 Ved_Goswami Proof-4 Reader’s Sign _______________________ Date __________

IMPORTANCE OF EACH TOPIC AND FREQUENTLY ASKED TYPES OF QUESTIONS

☞ Important Topics
1. Questions based on Electric field due to a dipole. 2. Questions based on Gauss theorem and its application.
3. Short questions on the dipole moment and torque.
* Maximum weightage is of Application of Gauss’s Law.

1. The figure shows three charges +2q, – q and +3q. Two charges +2q and –q are enclosed within a surface ‘S’. What
is the electric flux due to this configuration through the surface ‘S’?
q
[Ans. ]
ε0
2q S
+3q –q

2. An arbitrary surface encloses a dipole. What is the electric flux through this surface? [Ans. Zero]
3. If the radius of the Gaussian surface enclosing a charge q is halved, how does the electric flux through the Gaussian
surface change? [Ans. Unchanged]
4. Obtain Coulomb’s laws of electrostatics using Gauss theorem.
5. A point charge +10µC is at a distance of 5 cm directly above the centre of a square of side 10 cm as shown in the
figure. What is the magnitude of the electric flux through the square? [Ans. 1.88 × 105 Nm2/C]
10µC

5 cm
D C

A 10 cm B
6. Find the electric field intensity due to a uniformly charged spherical shell at a point (i) outside the shell and
σR 2
(ii) inside the shell. Plot the graph of electric field with distance from the centre of the shell. [Ans. (i) E =
(ii) E = 0] ε0 r2
7. A pendulum bob of mass 80 mg and carrying charge of 3 × 10–8 C is placed in an horizontal electric field. It comes
to equilibrium position at an angle of 37° with the vertical. Calculate the intensity of electric field. (g = 10 m/s2)
[Ans. 2 × 10–4 N/C]

44 Physics–12
D:\EG_Physics-12_(26-06-2022)\Open_Files\Ch-1\Ch-1
\ 27-Jul-2022 Ved_Goswami Proof-4 Reader’s Sign _______________________ Date __________

ASSIGNMENT
I. Objective Type Questions (1 Mark)
1. Multiple choice questions
(i) The electric field at a point is
(a) always continuous. (b) continuous if there is no charge at that point.
(c) discontinuous only if there is a negative charge at that point.
(d) discontinuous if there is a charge at that point.

(ii) An electric dipole of moment p is placed in the position of stable equilibrium in a uniform electric field E .
The couple required to rotate it through an angle q the initial position is:
(a) –pE cos q (b) pE tan q
(c) pE cos q (d) pE sin q
2. Fill in the blanks:
(i) The force between two point charges .......... when dielectric constant of the medium in which they are held
increases.

(ii) p = magnitude of the charge × ..........
II. Very Short Answer Type Questions (1 Mark)
3. State one basic property of electric charge.
4. Name the physical quantity whose unit is (i) V/m, (ii) Vm.
III. Short Answer Type Question-I (2 Marks)
5. Write one similarity and one dissimilarity between electrostatic force and gravitational forces.
IV. Short Answer Type Questions-II (3 Marks)
6. Two charges q and – 3q are placed fixed on x-axis separated by distance ‘d’. Where should a third charge 2q be
placed such that it will not experience any force?
7. Derive an expression for the electric field at any point on the equatorial line of an electric dipole.
8. State Gauss theorem in electrostatics. Using Gauss theorem, find out an expression for electric field at a point due
to an infinitely long line charge distribution of linear charge density l.
9. State and prove Gauss’s theorem. Also, deduce coulomb’s law from Gauss’s theorem.

Electric Charges and Fields 45


D:\EG_Physics-12_(26-06-2022)\Open_Files\Ch-2\Ch-2
\ 27-Jul-2022 Ved_Goswami Proof-4 Reader’s Sign _______________________ Date __________

Topics Covered
2 Electrostatic Potential and Capacitance

2.1 Electrostatic Potential and Electrostatic Potential Energy 2.2 Behaviour of Materials
2.3 Capacitors and Capacitance

C hapter map
Electrostatic Potential

Relation between E & V Equipotential Electrostatic Potential (V)


 −dV Surfaces
E= 1 q
dr V= for point charge
4πε0 r

Capacitance

Effect of introducing Parallel plate Dielectrics and polarisation


conducting slab of thickness t ε A
ε A C= 0
C= 0 d Combination
( − t)
d of capacitors Effect of introducing
dielectric slab
ε0 A
Energy stored C=
 t
Q2 CV2 QV Parallel Series  d − t + 
=
U = = k
2C 2 2 C= C1 + C2 + C3 1 1 1 1
= + +
C C1 C2 C3

Potential Energy of Energy density,


two point charges ε E2
u= 0
1 q1q2 2
U=
4πε0 r

Topic 1. Electrostatic Potential and Electrostatic Potential Energy


•• Electric potential difference: Electric potential Electric Potential
difference between two points in an electric field is •• Electric potential: Electric potential at a point in an
defined as the amount of work done, in moving a unit electric field defined as the amount of work done in
positive charge, from one point to another, without moving a unit positive charge from infinity to that point
accelerating it. without accelerating it.
S.I. unit of potential difference is volt and it is a scalar Electric potential is a scalar quantity. SI unit of electric
quantity. potential is volt.

46
D:\EG_Physics-12_(26-06-2022)\Open_Files\Ch-2\Ch-2
\ 27-Jul-2022 Ved_Goswami Proof-4 Reader’s Sign _______________________ Date __________

•• Electric potential (V) at a point due to a point charge q: •• Electrostatic potential energy of a system of charges:
q It is defined as the amount of work done, to bring the
V = charges, constituting the system, at their respective
4πε 0 r
location, from infinity, without accelerating them.
where r is the distance of that point from the charge.
Mathematically
Electrostatic Potential due to Point Charges (a) Potential energy of a system of two charges q1 and
•• Electric potential due to a group of point charges: q2
Electric potential at a point due to a group of point 1 q1q2
charges is equal to the algebraic sum of the electric U=
4πε 0 r12
potentials due to individual charges at that point.
r12 is the distance between q1 and q2
n
1  q1 q2 q  1 q (b) Potential energy of a system of three charges, q1, q2
V =  +
4πε 0  r1 r2
+ ... + n  =
rn  4πε 0
∑ ri and q3.
i =1 i
•• Potential gradient: The rate of change of potential with 1  q1q2 q2 q3 q3q1 
distance at a point is called potential gradient. U=  + + 
4πε 0  r12 r23 r31 
−dV →
E= q1
dr
Potential gradient is a vector quantity. Æ
Æ r31
Electric field is in the direction of decreasing potential. r12
SI unit of potential gradient is Vm–1.
•• Equipotential surface: The surface, at every point q2
Æ
q3
of which, the electric potential is same, is called r23
equipotential surface. •• Potential energy of a dipole is a uniform electric field:
(a) If an electric dipole is rotated from its initial
orientation making an angle qi with electric field to
+ Æ
E the final orientation making angle qf with the field
S S then
Equipotential surface
[due to point charge] U = pE (cos qi – cos qf)
Equipotential surface (due to uniform electric field) (b) Potential energy of an electric dipole placed in a
uniform electric field
•• Properties of equipotential surfaces:
→ →
(a) No work is done in moving a charge over an U = − p⋅ E
equipotential surface.
•• When a dipole is placed in a uniform electric field:
(b) Electric field is normal to the equipotential surface.
(c) No two equipotential surfaces can intersect each other. (a) For stable equilibrium, dipole moment should be
parallel to electric field.
(d) Equipotential surfaces are, closer in the region of
strong electric field and farther apart, in the region (b) For unstable equilibrium, dipole moment should be
of weak electric field. antiparallel to electric field.

EXERCISE 2.1
I. Objective Type Questions (1 Mark) (b) increases because the charge moves along the
1. Choose the correct answers from the given options electric field.
(MCQs). (c) decreases because the charge moves along the
(i) A positively charged particle is released from rest electric field.
in a uniform electric field. The electric potential (d) decreases because the charge moves opposite
energy of the charge to the electric field.
(a) remains a constant because the electric field (ii) Consider a uniform electric field in the z
is uniform. direction. The potential is a constant

Electrostatic Potential and Capacitance 47


D:\EG_Physics-12_(26-06-2022)\Open_Files\Ch-2\Ch-2
\ 27-Jul-2022 Ved_Goswami Proof-4 Reader’s Sign _______________________ Date __________

(a) in all space. (b) for any x for a given z. F F F F


(c) for any y for a given z. (a) (b) (c) (d)
7 5 3 2
(d) on the x-y plane for a given z. [CBSE 2022]
(iii) The electric potential of earth is taken to be zero (ix) Two charges 14 mC and –4 mC are placed at
because earth is a good (–12 cm, 0, 0) and (12 cm, 0, 0) in an external
(a) Insulator (b) conductor  B
(c) semiconductor (d) dielectric electric field E =  2  , where B = 1.2 × 106
r 
[CBSE S.P. 2019-20] N/(cm2) and r is in metres. The electrostatic
(iv) An electric dipole consisting of charges +q and –q potential energy of the configuration is
separated by a distance L is in  stable equilibrium (a) 97.9 J (b) 102.1 J
in a uniform electric field E . The electrostatic (c) 2.1 J (d) –97.9 J [CBSE 2022]
potential energy of the dipole is (x) Equipotentials at a large distance from a collection
(a) qLE (b) zero (c) –qLE (d) –2qEL of charges whose total sum is not zero are
[All India 2020] (a) spheres (b) planes
(v) The electric potential V at any point (x, y, z) is (c) ellipsoids (d) paraboloids
given by V = 3x2 where x is in metres and V in [CBSE 2022]
volts. the electric field at the point (1 m, 0, 2 m) (xi) Four charges – q, – q, + q and + q are placed at the
is corners of a square of side 2 L is shown in figure.
(a) 6 V/m along –x-axis The electric potential at point A midway between
(b) 6 V/m along +x-axis the two charges + q and + q is
(c) 1.5 V/m along –x-axis +q –q
(d) 1.5 V/m along +x-axis[CBSE 2022]
(vi) A potential difference of 200 V is maintained
across a conductor of resistance 100 W. The
number of electrons passing through it in 1s is A 2L
(a) 1.25 × 1019 (b) 2.5 × 1018
(c) 1.25 × 10 18 (d) 2.5 × 1016
[CBSE 2022] +q –q
(vii) A + 3.0 nC charge Q is initially at rest at a distance 1 2q  1 
of r1 = 10 cm from a + 5.0 nC charge q fixed at (a)  1− 
4π ∈0 L  5
the origin. The charge Q is moved away from q
to a new position at r2 = 15 cm. In this process 1 2q  1 
(b)  1+ 
work done by the field is 4π ∈0 L  5
(a) 1.29 × 10–5 J (b) 5.8 × 103 J
1 q  1 
(c) 8.64 × 106 J (d) 4.5 × 10–7 J (c)  1− 
[CBSE 2022] 4π ∈0 2L  5
(viii) Four objects W, X, Y and Z, each with charge +q (d) zero [CBSE 2022]
are held fixed at four points of a square of side 2. Fill in the blanks.
d as shown in the figure. Objects X and Z are (i) Two equipotential surfaces cannot intersect each
on the midpoints of the sides of the square. The other because .......... .
electrostatic force exerted by object W on object (ii) Electric potential V due to an electric dipole is
X is F. Then the magnitude of the force exerted related to the distance r of the observation point
by object W on Z is as .......... .
W X Y
(iii) A charge q is distributed over two spheres of
radii r1 and r2 such that their surface densities are
equal. Then is the ratio of their potential will be
d
....................... .
(iv) The physical quantity having SI unit NC–1 m is
Z
....................... . [All India 2020]

48 Physics–12
D:\EG_Physics-12_(26-06-2022)\Open_Files\Ch-2\Ch-2
\ 27-Jul-2022 Ved_Goswami Proof-4 Reader’s Sign _______________________ Date __________

3. Match the Columns Reason (R): Electric field strength depends on


Column-I Column-II variation of potential with x.
 
(i) Electric potential at a point (a) −p ⋅ E II. Very Short Answer Type Questions (1 Mark)
due to a point charge q 1. Name the physical quantity whose SI unit is JC–1. Is
q it a scalar or a vector quantity? [Delhi 2010]
(ii) Relation between potential (b) 4πε r 2. A hollow metal sphere of radius 5 cm is charged such
0
gradient and electric field. that the potential on its surface is 10 V. What is the
(iii) Potential energy of an (c) Potential potential at the centre of the sphere? [AI 2011]
electric dipole placed gradient 3. We know that electric field is discontinuous across the
 surface of a charged conductor. Is electric potential
in a uniform E
 also discontinuous there?
(iv) A vector quantity. (d) E = − dV 4. What is the geometrical shape of equipotential surface
dr
For questions numbers 4, 5, 6 and 7, two statements are due to a single isolated charge?
given-one labelled Assertion (A) and the other labelled 5. Depict the equipotential surface for a system of two
Reason (R). Select the correct answer to these questions identical positive point charges placed distance ‘d’
from the codes (a), (b), (c) and (d) as given below. apart. [Delhi 2010]
(a) Both A and R are true and R is the correct 6. Draw the equipotential surfaces due to an electric
explanation of A dipole.
(b) Both A and R are true but R is NOT the correct 7. Draw a equipotential surface for a system of two
explanation of A charges Q and –Q, separated by a distance ‘a’. Locate
the points where the potential due to the dipole is zero.
(c) A is true but R is false
Or
(d) A is false and R is also false
Depict equipotential surface due to an electric dipole.
4. Assertion (A): Electric field is always normal to
[Delhi 2017]
equipotential surfaces and along the direction of
decreasing order of potential. 8. Sketch the equipotential surface for a point charge.
Reason (R): Negative gradient of electric potential 9. Is electrostatic potential necessarily zero at a point
is electric field. [CBSE S.P. 2020-21] where electric field strength is zero? Give one
example for your answer.
5. Assertion (A): For the practical purposes, the earth
is used a reference at zero potential (0V) in electrical 10. Give an example where electrostatic potential is zero
circuits. but electric field intensity is non zero.
Reason (R): The electrical potential of a sphere of 11. Why do two equipotential surfaces not intersect each
radius r with charge q uniformly distributed on the other?
q 12. A hollow metal sphere of radius 5 cm is charged
surface is denoted by 4πε r .
0 such that potential on its surface is 10 V. What is the
6. Assertion (A): Electric potential (V) is a vector. potential at the centre of the sphere? [AI 2011]
Reason (R): In an electric field, a positive charge 13. Why is electrostatic potential constant throughout the
move from lower potential to higher potential. volume of the conductor and has the same value (as
7. Assertion (A): If electric potential varies along x-axis inside) in its surface? [Delhi 2012]
→ 14. A 500 µC charge is at the centre of a square of side
as shown in Fig. electric field strength ( E max ) will 10 cm. Find the work done in moving a charge of 10
have maximum magnitude at x = 0. µC between two diagonally opposite points of the
square . [Delhi 2008]
15. What is the work done in moving a test charge q
through a distance of 1 cm along the equatorial line
of an electric dipole? [AI 2009]
16. What is the amount of work done in moving a charge
around a circular arc of radius r at the centre of which
another point charge is located? [AI 2013 C]

Electrostatic Potential and Capacitance 49


D:\EG_Physics-12_(26-06-2022)\Open_Files\Ch-2\Ch-2
\ 27-Jul-2022 Ved_Goswami Proof-4 Reader’s Sign _______________________ Date __________

17. If a point charge is taken from A to C and then from what point (out of A and C) is the electric potential
C to B on a circle drawn with another point charge more and why? [Delhi 2012]
+q placed at its centre. Then along which path more
B (2, 3)
work will be done.

E

A
B
+q
(2, 0) C A (6, 0)
C

18. The figure shows the field lines of a positive point 25. Establish the relation between electric field and
charge. What will be the sign of the potential energy electric potential at a point. Draw the equipotential
difference of a small negative charge between the surface for an electric field pointing in +Z direction
points Q and P? Justify your answer. [AI 2015] with its magnitude increasing at constant rate along
–Z direction. [CBSE S.P. 2020-21]
26. Two charges 2 µC and – 2µC are placed at points A
•Q and B, 6 cm apart (a) identify an equipotential surface
•P of the system (b) what is the direction of electric field
+
at every point on this surface?
27. A charge of 8 mC is located at the origin. Calculate
the work done in taking a small charge of –2 × 10–9
C from a point P(0, 0, 3 cm) to Q (0, 4 cm, 0) via a
19. Write two important characteristics of equipotential point R(0, 6 cm, 9 cm).  [NCERT]
surfaces. [Delhi 2020] 28. N small conducting liquid droplets, each of radius
III. Short Answer Type Questions-I (2 Marks) r, are charged to a potential V each. These droplets
coalesce to form a single large drop without any
20. Two charged spherical conductors of radii R1 and R2 charge leakage. Find the potential drop.
when connected by a connecting wire acquire charges
[Delhi 2020]
q1 and q2 respectively. Find the ratio of their charge
densities in terms of their radii. [Delhi 2014] 29. The magnitude of electric field (in NC–1) in a region
varies with the distance r(in m) as E = 10r + 5. By how
21. Depict the equipotential surfaces due to
much does the electric potential increase in moving
(i) an electric dipole,
from point at r = 1 m to a point at r = 10 m.
(ii) two identical positive charges separated by a
[Delhi 2020]
distance. [AI 2015][CBSE S.P. 2019-20]
22. Explain why, for any charge configuration, the IV. Short Answer Type Questions-II (3 Marks)
equipotential surface through a point is normal to the 30. Obtain an expression for electric potential due to a
electric field at that point. point charge.
Draw a sketch of equipotential surfaces due to a single 31. Obtain an expression for electric potential due to an
charge (– q), depicting the electric field lines due to electric dipole at a point on its axis.
the charge. [AI 2016] [Delhi 2008, AI 2013 C]
23. Two uniformly charged parallel plates having charge 32. Obtain an expression for electric potential due to an
densities +σ and –σ are placed in the XZ plane at a electric dipole at a point whose position vector makes
distance ‘d’ apart (i) sketch the equipotential surface an angle q with dipole.
due to electric field between the plates. (ii) If a particle 33. (a) Show that the equipotential surfaces are closed
of mass m and charge –q, remains stationary between together in the regions of strong field and far apart
the plates, what is the magnitude and direction of this in the regions of weak field. Draw equipotential
field? surfaces for an electric dipole.
24. A test charge q is moved without acceleration from A (b) Concentric equipotential surfaces due to a charged
to C along the path from A to B and then from B to C body placed at the centre are shown. Identify the
in the electric field E as shown in the figure. Calculate polarity of the charge and draw the electric field
the (i) potential difference between A and C (ii) At lines due to it. [S.P. 2015-16]

50 Physics–12
D:\EG_Physics-12_(26-06-2022)\Open_Files\Ch-2\Ch-2
\ 27-Jul-2022 Ved_Goswami Proof-4 Reader’s Sign _______________________ Date __________

40V A
30V
q
20V
10V

–4q 2q
34. Define electrostatic potential at a point. Write its B l C
S.I. unit. Three point charges q1, q2 and q3 are kept
respectively at points A, B and C as shown in the 39. A particle, having a charge +5 mC, is initially at rest
figure. Derive the expression for the electrostatic at the point x = 30 cm on the x axis. The particle
potential energy of the system.[CBSE S.P. 2019-20] begins to move due to the presence of a charge Q that
A
is kept fixed at the origin. Find the kinetic energy of
the particle at the instant it has moved 15 cm from
its initial position if (a) Q = +15mC and (b) Q =
r12 r13
–15mC. [CBSE SP 2018-19]
40. (a) Draw the equipotential surfaces corresponding to
B C
r23 a uniform electric field in the z-direction.
35. Obtain an expression for the work done to dissociate (b) Derive an expression for the electric dipole
the system of three charges placed at the vertices of potential at any point along the axial line of an
an equilateral triangle of side ‘a’ as shown below. electric dipole. [Delhi 2019]
[AI 2016] 41. (i) Two point charges +Q1 and –Q2 are placed r
q distance apart. Obtain the expression for the
amount of work done to place a third charge Q3 at
the midpoint of the line joining the two charges.
a a (ii) At what distance from charge +Q1 on the line
joining the two charges (in terms of Q1, Q2 and
r) will this work done be zero. [Delhi 2020]
–4q a + 2q V. Long Answer Type Questions (5 Marks)
36. A small sphere of radius r1 and charge q1 is enclosed 42. (a) Define equipotential surface. Why is the electric
by a spherical shell of radius r2 and charge q2. Show field at any point on the equipotential surface
that if q1 is positive, charge will necessarily flow from directed normal to the surface?
sphere to shell (when two are connected by a wire) (b) Draw the equipotential surfaces for an electric
no matter what the charge q2 on the shell is. dipole. Why does the separation between
[NCERT] successive equipotential surfaces get wider as the
distance from the charges increases?
37. Two charges 5 × 10–8 C and – 3 × 10–8 C are placed
(c) For this dipole, draw a plot showing the variation
16 cm apart. At what points on the line joining the two
of potential V versus x, where x (x >> 2a), is the
charges is the electric potential zero. Take potential
distance from the point charge –q along the line
at infinity to be zero. [NCERT]
joining the two charges. [AI 2015]
38. (a) Three point charges q, –4q and 2q are placed at 43. (a) Deduce the expression for the potential energy of
the vertices of an equilateral triangle ABC of side →
‘l’ as shown in the figure. Obtain the expression a system of two charges q1 and q2 located at r1
for the magnitude of the resultant electric force →

acting on the charge q.  [Delhi 2018] and r2 respectively in an external electric field.
(b) Three point charges, + Q, + 2Q and – 3Q are
(b) Find out the amount of the work done to separate
placed at the vertices of an equilateral triangle
the charges at infinite distance.
ABC of side l. If these charges are displaced to
the mid-points A1, B1 and C1 respectively, find the

Electrostatic Potential and Capacitance 51


D:\EG_Physics-12_(26-06-2022)\Open_Files\Ch-2\Ch-2
\ 27-Jul-2022 Ved_Goswami Proof-4 Reader’s Sign _______________________ Date __________

amount of the work done in shifting the charges (b) Draw equipotential surfaces due to an isolated
to the new locations. [AI 2015] point charge (–q) and depict the electric field lines.
44. (a) Find the expression for the potential energy of (c) Three point charges +1 µC, –1 µC and +2 µC are
a system of two point charges q1 and q2 located initially infinite distance apart. Calculate the work
→ → done in assembling these charges at the vertices
at r1 and r2 respectively in an external electric of an equilateral triangle of side 10 cm.[AI 2020]
field. [CBSE S.P. 2020-21]

Answers 2.1
I. Objective Type Questions 8.
1. (i) (c) (ii) (b, c & d) (iii) (b)
(iv) (c) (v) (a) (vi) (a) (vii) (d)
(viii) (b) (ix) (c) (x) (a) (xi) (a) +
 q
2. (i) then there will be two values of V at that point,
which is not possible.
r
(ii) V ∝ r–2 (iii) 1
r2 9. No, potential may not be zero at a point where electric
(iv) electric potential
field intensity is zero.
3. (i)- (b) (ii)-(d) (iii)-(a) (iv)-(c)
For example, inside a charged shell E = 0. But potential
4. (a) Both A and R are true and R is the correct explanation
of A in not zero.
5. (a) 6. (d) 7. (b) 10. At any point on the equator of a dipole, potential is zero
but electric field is non zero.
II. Very Short Answer Type Questions
11. If they intersect, then at the point of intersection, there
1. Electric potential, it is a scalar quantity. will be two values of electric potential which is not
2. Potential at the centre possible.
= Potential at the surface. 12. Potential on the surface = 10 V
  = 10 V −dV
3. No, electric potential is continuous. As E= ,
dr
4. Equipotential surface is spherical in shape for q > 0 or Inside a hollow metal sphere E = 0
q < 0 at the centre. dV
So = 0    ⇒   V = constant
5. dr
−dV
13. As E=
+ + dr
But inside a conductor E = 0, and E has no tangential
component on the surface so
−dV
6. Equipotential surface =0 Or V = constant
dr
14. A D

r r
E

500 µC

r r

7. B C
+ – As the potential at two diagonally opposite points of a
square will have same value so the work done is zero.
15. As potential at any point on the equator of a dipole is

P is any point on equator, where potential is zero. zero. So work done is zero.

52 Physics–12
D:\EG_Physics-12_(26-06-2022)\Open_Files\Ch-2\Ch-2
\ 27-Jul-2022 Ved_Goswami Proof-4 Reader’s Sign _______________________ Date __________

16. Zero, because at all points of the circular arc, potential


will be same.
17. Work done from A to C = Work done from C to B = 0
18. Electric field is in the direction of decreasing potential
Electric field line
so VQ < VP, let the charge to be placed between Q and
P be –q.
–q
UQ – UP = –q(VQ – VP) = q(VP – VQ) Equipotential
surface
As VP > VQ. So UQ – UP is positive.
19. (i) P otential remains same at all the points on
equipotential surface.
(ii) N o work is required to move a charge on an As potential everywhere on the surface is constant so
equipotential surface as dW = qdV = 0 [Q dV = 0]
W = (VB – VA)q = (VA – VA)q = 0. → →
q E ⋅ dr = 0
20. When the spherical conductors are connected the charge
will flow from the one at higher potential to the one at qE dr cos θ = 0
lower potential, till they have common potential. Thus ∴ cos θ = cos 90°
q2 i.e. θ = 90° 1
q1
=  1 Equipotential surface for charge (–q) will be sphere.
4πε 0 R1 4πε 0 R 2
d
q1 R 23. (i) Equipotential surface is at a distance from either
  or = 1 ...(i) plate and parallel to XZ plane. 2
q2 R2 y
Ratio of surface charge density is
σ1 q1 4πR 22 q1 R 22
= × = × 2
σ2 4πR12 q2 q2 R1
x
R R2 R
= 1 × 22 = 2 [Using (i)] 1
R 2 R1 R1 z
21. Equipotential surface for (i) dipole Fig. (a) (ii) for two
equal charges Fig. (b). + + + + + +

d Equipotential    1
surface
+ –
1 – – – – – –

(a) (ii) Forces acting on –q are


(a) weight mg, vertically downward
(b) electric force qE, vertically upward
+ +
1 So qE = mg
mg
E=  1
(b) q
[CBSE Marking Scheme, 2019-20] Electric field should be in vertically downward
direction.
22. Equipotential surface is normal to electric field at that + + + +
point, because the work done to move a unit positive –q
charge from one point to another on an equipotential
– – – –
surface is zero. 1
24. (i) Potential difference does not depend upon the path
→ →
dW = F ⋅dr chosen.
− dV − (VC − VA ) VC − VA
= q→ →
E ⋅ dr
E = = =
dx 2−6 4

Electrostatic Potential and Capacitance 53


D:\EG_Physics-12_(26-06-2022)\Open_Files\Ch-2\Ch-2
\ 27-Jul-2022 Ved_Goswami Proof-4 Reader’s Sign _______________________ Date __________

VC – VA = 4E 27. q = –2 × 10–9 C, Q = 8 mC
So VC > VA 1 OP = 3 cm = 3 × 10–2 m
(ii) Electric field is in the direction of decreasing OQ = 4 cm = 4 × 10–2 m
potential. z

So potential at C > potential at A. 1



25. (a) Relation b/w V and E : Consider electric field due (0,0,3) P
to point charge +q located at O. Let P and Q be the
two points. At P, V is potential and at Q, V + dV is
Q
potential. O
y
PQ = dr 8 mC
(0,4,0)
+q V + dV V 
E
O Q P x

A test charge q0 is moved from P to Q without giving As electrostatic force is conservative so the work done
to move a charge is independent of the path.
it any acceleration.
→ →
OP = ri, OQ = rf
→ →
dW = F ⋅ dr = q0 E ⋅ dr WPQ = Uf – Ui = q(VQ – VP)
= q0E dr cos 180° = –q0E dr 1  Q Q 
= q −  1
Work done dW − q0 E dr  4πε 0 r f 4πε 0 ri 
dV = = =
Charge q0 q0
qQ  1 1 
dV = –Edr =  − 
4πε 0  r f ri 
E = −dV  1 = 9 × 109 × (– 2 × 10–9) × 8 × 10–3
dr
100 100 
E = –potential gradient, having Unit ×  −
 4 3 
 (= V/m)
Potential gradient is a vector quantity. 1
= +144 × 10–3 × 100 × = 1.2 J 1
y
– – – – 12
x 28. qnew = Nq; q = charge on each small conducting liquid
droplet
Equipotential surface Vol. of single large drop = N × Vol. of one small
(parallel to xy plane)
conducting liquid droplet
4 3  4 3
pR =  πr  ⇒ R = N1/3r
z
...(1)
+ + + + 3 3 
26. (a) Equipotential surface will be a plane normal to AB R = radius of single larger drop,
and passing through its mid point. Potential on this
surface will be zero. 1 r = radius of small conducting liquid droplets
(b) The direction of electric field at every point on this kq
 V= = potential on each small droplet
surface is along normal to the plane and opposite to r
dipole moment. \ V′ = potential on single large drop
P
E kQ k ( Nq )
= = 1  [using (1)]
R N 3r
2  kq 
2µC –2µC = N 3 
 r
A B
 1 2
⇒ V’ = N 3 V 
29. Point A be given at r = 1 m, point B be given at r =10
m.

54 Physics–12
D:\EG_Physics-12_(26-06-2022)\Open_Files\Ch-2\Ch-2
\ 27-Jul-2022 Ved_Goswami Proof-4 Reader’s Sign _______________________ Date __________

Potential at A is VA and potential at B is VB 31. Consider electric dipole of dipole moment p and length
→ 2a. Let P be a point on axis of dipole. 1
The relation between the electric field ( E ) and potential A O B P
difference (V) is given by the relation –q
a a
+q
r–a
B 
VB – VA = – ∫ E ⋅ d r
r+a
r
A
10 k (− q)
VB – VA = – ∫ (10r + 5) .dr V–q =
1 (r + a)
10
10r 2  k ( q)
= – + 5r  V+q =  1
 2 1 (r − a)
Vnet = V–q + V+q
 10 (10)2   10 (1)2 
= –  + 5 × 10 −  + 5 × 1  k ( − q) k (q)
 2   2   = +
(r + a) (r − a)
= – [550 – 10] = – 540 volt.
 −1 1 
30. Consider a point charge ‘Q’ placed at O. Suppose a = kq  + 
test charge q0 is at A. Distance OA = x, OP = r. Let us  (r + a) (r − a) 
move the test charge by small amount dx = AB.  − (r − a) + (r + a) 
(Dividing the distance into small part since force is = kq  
 (r 2 − a 2 ) 
variable).
dx  −r + a + r + a 
= kq  2 2 
 (r − a ) 
Q P B A
O q0 F
kq (2a ) k × q × 2a kp
r = 2 = 2 = 2  1
x (r − a ) (r − a ) (r − a 2 )
2 2

1 Qq0 Special case: If r >> a for a short dipole.


F=
4πε 0 x 2 kp
V= 2
work done against electrostatic force r
→ → 1
dW = F ⋅ dx = F dx cos 180° V∝
r2
= – F dx 1 32. Electric potential due to dipole at general point:
Total work done = W =∫ dW Consider an electric dipole of dipole moment p and
length 2a. O is centre of dipole OP = r.
r r
1 Qq0 P
= − ∫ Fdx = − ∫ ⋅ dx
∞ ∞
4πε 0 x 2
r1
r
− Qq0 1 r
=
4πε 0 ∫ x 2 ⋅ dx C
r2

r O θ
− Qq0 A
a θ
∫x
−2 B
= dx –q
a +q
4πε 0 ∞
r r
Qq0  x −2+1  Qq0 1 D
= −   = (−q) k (− q)
4πε 0  −2 + 1  ∞ 4πε 0 x ∞ V–q = k =
AP r1
Qq0  1 1  Qq0 1
=  −  = ⋅  1 kq kq
4πε 0  r ∞  4πε 0 r V+q = =  1
BP r2
Work done Qq0 Q OC
V= = q0 =  1 = cos θ
Charge 4πε 0 r 4πε 0 r OB

Electrostatic Potential and Capacitance 55


D:\EG_Physics-12_(26-06-2022)\Open_Files\Ch-2\Ch-2
\ 27-Jul-2022 Ved_Goswami Proof-4 Reader’s Sign _______________________ Date __________


⇒ OC = OB cos θ = a cos θ A
q1
BP = r2 ≈ CP = r – OC = r – a cos θ
OD = a cos θ
AP = r1 ≈ DP = r + OD = r + a cos θ r12 r13

1 1
Vnet = V–q + V+q = kq  −   1
 r2 r1  B C
r23
 1 1  q2 q3
= kq  −  W1 = 0, W2 = W
 r − a cos θ r + a cos θ 
W2 = work done = V1 × q2
 r + a cos θ − r + a cos θ 
= kq   V1 is potential at B produced by q1
 (r − a cos θ) (r + a cos θ)  1 q1
V1 =
 2a cos θ  4πε 0 r12
= kq   1
 r 2 − a 2 cos 2 θ  1 q1q2
W2 = V1 × q2 =  1
dV 4πε 0 r12
33.(a) As E= –
dr 1 q1q2
For the same change in potential i.e., same dV Total work = W1 + W2 =
4πε 0 r12
E∝ 1 when q3 is brought to point C work need to be done against
dr the force of repulsion of q1 and q2 both.
So in the region of high electric field, dr is small. If
E is more, dr is less. Thus equipotential surfaces are Potential at C due to
close together in the region of strong field and far apart 1 q2
q2 = V2 =
in the region of weak fields. 1 4πε 0 r23
(b) As electric field is in the direction of decreasing 1 q1q3 1 q2 q3
potential, so the direction of electric field lines is W3 = +
4πε 0 r13 4πε 0 r23
towards the centre of the circles. Charge is negative.
 1 The total work done to assemble the charges
Radially inward:   = W1 + W2 + W3
1 q1q2 1 q1q3 1 q2 q3
  = 0 + + +
4πε 0 r12 4πε 0 r13 4πε 0 r23
This work done to assemble the charges is stored as its
potential energy U.
     1
1  q1q2 q2 q3 q3 q1 
U=  + +  1
4πε 0  r12 r23 r13 
[CBSE Marking Scheme, 2019-20]
34. Electrostatic potential at a point: It is the amount of 35. Ui = Initial P.E.
work done to move a unit positive charge from infinity 1  q ( −4q ) 2q ( −4q ) 2qq 
to that point in the electric field without accelerating it. =  + +  1
4πε 0  a a a 
S.I. unit of electrostatic potential is volt. ½
Electrostatic potential energy: The amount of work 1  −10q 2 
=
done in assembling the charges at their respective 4πε 0  a 
positions/locations from infinity is electrostatic After dissociation each charge is infinite distance apart
potential energy. ½ so Uf = 0. 1
Expression: Suppose a point charge q1 is brought to a Work done in dissociating the given system
position. A no work done to bring charge q1 as there is 10q 2
( −10q 2 )
no field to work against. Charge q2 is brought to point = Uf – Ui = 0 – =  1
4πε 0 a 4πε 0 a
B at a distance ‘r’ from A.
56 Physics–12
D:\EG_Physics-12_(26-06-2022)\Open_Files\Ch-2\Ch-2
\ 27-Jul-2022 Ved_Goswami Proof-4 Reader’s Sign _______________________ Date __________

36. Potential of the sphere with radius r1 is V1. 8x = 0.8


x = 0.1 m = 10 cm 1
r2 Shell →
38. (a) Force F1 on charge q at A due to 2q charge is along
he
re CA.
Sp

\
→ 2q 2 …(1)
r1 F1 = K
l2
q1 q2

V11 → Potential of sphere due to charge q1


V12 → Potential of sphere due to charge q2
kq kq
V1 = V11 + V12 = 1 + 2  1
r1 r2
Similarly,
kq kq
V2 = V21 + V22 = 1 + 2  1
r2 r2
kq1 kq2 kq1 kq2
V1 – V2 = + − − Force F2 on charge q at A due to charge –4q at B is
r1 r2 r2 r2
along AC
= kq  1 − 1  = kq1 (r2 − r1 )  1 → 4q 2
1  F2 = K …(2)
r
 1 2r r r
1 2 l2
→ →
V1 – V2 is positive so long as q1 is +ve and +ve charge F1 and F2 makes an angle 120° with each other
always flows from higher potential to lower potential.
\ FNet = F2 + F2 + 2F F cos 120°  1
1 2 1 2
37. x 0.16–x
A P B 2
FNet =  F2  2  F2   −1
–8 –8   + F2 + 2   F2  
5×10 –3×10 2 2 2
q1 = 5 × 10–8 C, q2 = –3 × 10–8 C
F22 F2
Let the potential be zero, x distance away from A, say FNet = + F22 − 2
at point P. 4 2
Potential at P due to q1 1+ 4 − 2 3
FNet = F2 = F2
q1 9 × 10 × 5 × 10 9 −8 4 2
V1 = =  1
4πε 0 x x 3 4q 2
FNet = K 2
The potential at P due to q2 2 l
q2 1 3 4q 2
V2 = FNet = × × 2
4πε 0 (0.16 − x) 4π ∈0 2 l
9 × 109 × ( −3 × 10 −8 )
= 1 3 q2
0.16 − x FNet = ½
2π ∈0 l 2
V = V1 + V2 = 0    
(b) Potential energy of the system
9 × 109 × 5 × 10−8 9 × 109 × 3 × 10−8
V= − =0 Ui = UAB + UBC + UCA
x 0.16 − x
 −4q 2 −8q 2 +2q 2 
5 3 Ui = K  + + 
i.e., − =0  l l l 
x 0.16 − x 1
or Ui = [ −10q 2 ]
5(0.16 – x) = 3x 4π ∈0 l

Electrostatic Potential and Capacitance 57


D:\EG_Physics-12_(26-06-2022)\Open_Files\Ch-2\Ch-2
\ 27-Jul-2022 Ved_Goswami Proof-4 Reader’s Sign _______________________ Date __________

10q 2 41. (i) +Q1 –Q2


or Ui = −
 1
4π ∈0 l r/2 r/2
Work done to separate these charges at • distances ½
–Q3
W = U f – U i fi W = U• – U
(at ∞)
 10q 2 
= 0 – −
Work done = W13 +W32
 4π ∈ l  0
KQ1Q3 KQ3Q 2
10q 2 W= −  ½
fi   W =
 ½ r r
4π ∈0 l 2 2
39. From energy conservation, 2KQ1Q3 2KQ3Q 2
W= −
Ui + Ki = Uf + Kf r r
kQq/ri + 0 = kQq/rf + Kf ½
2KQ3 ( Q1 − Q 2 ) 2Q3 ( Q1 − Q 2 )
Kf = kQq (1/ri – 1/rf) ½ W= =
r 4πε 0 r
When Q is +15 mC, q will move 15 cm away
from it. Hence rf = 45 cm  ½
Kf = 9 × 109 × 15 × 10–6 × 5 × 10–6 (ii) Work done will be zero for the point where the
[1/(30 × 10–2) – 1/(45 × 10–2)] ½ electric potential is zero as W = V.q. Considering
a point M to be lying in between the two charges,
= 0.75 J½
where electric potential is zero.
When Q is –15 mC, q will move 15 cm towards it. Hence
rf = 15 cm +Q1 M –Q2
    ½
Kf = 9 × 109 × (–15 × 10–6) × 5 × 10–6 x r–x
[1/(30 × 10–2) – 1/(15 × 10–2)] ½ Let the distance of the point M from the charge +Q1
= 2.25 J ½ be x.
[CBSE Marking Scheme, 2018-19] KQ1 KQ 2
V= − =0
40. (a) x (r − x)
Q1 Q2

⇒ =
(r − x)
Equipotential
surface x
rQ1 – xQ1 – xQ2 = 0
rQ1
x= 1
     1

(Q1 + Q2 ) 

(b) 42. (a) Equipotential surface: It is the locus of all points
     having same potential.
Let P be an axial point at distance r from the centre →
of the dipole. Electric potential at point P is given E is always normal to the equipotential surface.
as → →
Work done = qdV = F ⋅ d r

V = V1 + V2 1
V1 and V2 are the potentials at point P due to charges → →
0 = q E ⋅ d r = qE dr cos θ
+q and –q respectively.
as q ≠ 0, E ≠ 0, dr ≠ 0.
\ V = 1  q + −q 
4πε 0  r − a r + a  Thus electric field is directed normal to the surface.
q 2a  1
=
4πε 0 r 2 − a 2 (b) Equipotential surface for an electric dipole.
1  p  Equipotential surfaces are closer in region of strong
= 1
4πε 0  r 2 − a 2  field and farther apart in region of weak field.

58 Physics–12
D:\EG_Physics-12_(26-06-2022)\Open_Files\Ch-2\Ch-2
\ 27-Jul-2022 Ved_Goswami Proof-4 Reader’s Sign _______________________ Date __________

− dV (b) Initial P.E. of the system


E=
dr A (+Q)

dV
|E| =
dr
A1 C1

+ –

B (+ 2Q) B1 C (–3Q)
For constant P.D.
1
E ∝ 1 ⇒ dr ∝  1 Ui =
1  qA qB qB qC qC qA 
+ +
dr E  
4πε 0  rAB rBC rCA 
(c) Variation of V vs x, where x is the distance from the
dipole on axial line and x >> a.  Q × 2Q 2Q( − 3Q) 
 l + 
1 p = 1 l
V=  1  
4πε 0 x 2 4πε 0  ( −3Q × Q) 
+
 l 

1
V =  2Q 2 − 6Q 2 − 3Q 2 
4πε 0l  

Distance from dipole −7Q 2


x =  ½
4πε 0l

43. (a) Let q1 and q2 be the two charges located at r1 and A1B1 = B1C1 = C1A1 =
l
→ → 2
r2 respectively, in an external field E .
 2Q × 2Q 2Q( − 3Q) × 2 
The work done to bring a charge q1 from infinity to +
→ 1  l l 

the position r1 due to the external field E is given Uf =  
4πε 0  ( −3Q) (Q)2 
by +
 l 
W1 = q1V1 ½
→ 1× 2 −14Q 2
Where V1 is the potential at position r . Similarly, = [−7Q 2 ] =  ½
1 4πε 0l 4πε 0l
the work done to bring charge q2 from infinity to

position r2 due to external field is −14Q 2  −7Q 2 
Work done = Uf – Ui = −
4πε 0l  4πε 0l 
W2 = q2V2 ½

Where V2 is the potential at position r2 . Q2 −7Q 2
=
4πε 0l
[ −14 + 7 ] =
4πε 0l
 ½

While bringing q2 to position r2 , work has to be
Hence proved
done against the field of q1.
44. (a) See solution (43a) 2
q1q2
So W3 =
4πε 0 r12 (b)
r12 = distance between q1 and q2. Equipotential
–q surfaces 1
So, total work done in assembling the system (concentric
U = W1 + W2 + W3 sphere)
q1q2
= q1V1 + q2V2 +  ½
4πε 0 r12

Electrostatic Potential and Capacitance 59


D:\EG_Physics-12_(26-06-2022)\Open_Files\Ch-2\Ch-2
\ 27-Jul-2022 Ved_Goswami Proof-4 Reader’s Sign _______________________ Date __________

(c) W = Energy of system = U12 + U13 + U23

q1 = + 1µC = k/r(q1q2 + q2q3 + q1q3) 1


= 9× 109/10 × 10–2[(+1)×(–1)
+ (+1)×(+2) + (–1)×(+2)] × 10–12
= 9 × 10–2(–1 + 2 –2)
q2 = – 1µC
q3 = + 2µC = –0.09 J. 1

Topic 2. Behaviour of Materials


Conductors and Insulators (ii) Non-polar molecules: In non-polar molecules, the
•• Conductors: These are the substances which allow large centres of positive and negative charges coincide.
scale physical movement of electric charges through The molecules have no permanent dipole moment.

them when an external electric field ( E ) is applied. For Examples of non-polar molecules are oxygen,
example: silver, copper, etc. hydrogen molecules.

•• Insulators: These are the substances which do not E0
allow physical movement of electric→charges through
them when an external electric field ( E ) is applied. For + + –+ –+
example: diamond, glass, wood, mica, etc.
+ +
+ –+
+
Free Charges and Bound Charges + –+
+
•• Free and bound charges:
+ –+
+
(i) In conductors, the electrons of the outer shells of 
the atoms are loosely bound to the nucleus. They E
get detached from the atoms and move almost freely •• Dielectric placed in an electric field:
inside the metal. The positive ions which consist
(i) When a non-polar di-electric is placed in an external
of nuclei and electrons of inner shells remain held
in their fixed positions. These immobile charges electric field, the positive and negative charges are
constitute the bound charges. displaced in opposite directions. The displacement
(ii) In insulators, the electrons are tightly bound to the stops when the external force on the charges of the
nuclei and cannot be detached from the atoms. molecules is balanced by the force due to internal
fields in the molecule. Thus the non-polar molecules
Dielectrics and Polarisation develops induced dipole moment. We say that the
•• Dielectrics: Dielectrics are non conducting substances. dielectric is polarised.
•• The molecules of a substance may be polar or non-polar (ii) When a dielectric with polar molecules placed in
(i) Polar molecules: Polar molecules are those molecules an electric field, the dipoles align themselves in the
in which the centres of positive and negative charges direction of applied electric field. Thus net dipole
are separated, even when there is no external field. moment is induced, the dielectric is polarised.
Polar molecules have permanent dipole moment. •• Dielectric constant: It is the ratio of strength of the
In the sample, the molecular dipoles are randomly applied electric field (E0) to the strength of the reduced
oriented so net dipole moments is zero. value of the electric field (E) on placing the dielectric
 between the plates of the capacitor.
E0
E0
    K = E
+ –+ –+
– +–
–+ –+
•• Polarisation density: It is the induced dipole moment
– developed per unit volume in a dielectric slab on placing
–+
it in an electric field.
+
+– –+

 •• Electric Susceptibility: Polarisation density ∝ resultant


Unpolarized E field
dielectric

60 Physics–12
D:\EG_Physics-12_(26-06-2022)\Open_Files\Ch-2\Ch-2
\ 27-Jul-2022 Ved_Goswami Proof-4 Reader’s Sign _______________________ Date __________

→ → →
i.e. P ∝ E P
Susceptibility χ = →
→ →
ε0 E
or  P = e0 χ E Electric susceptibility is the ratio of polarisation to e0
times the electric field.

EXERCISE 2.2
I. Objective Type Questions (1 Mark) (b) Both A and R are true but R is NOT the correct
1. Choose the correct answers from the given options explanation of A
(MCQs). (c) A is true but R is false
(i) Dielectric strength of a medium is 2 KV mm–1.
(d) A is false and R is also false
What is the maximum potential difference that
can be set up across a 50 mm specimen without 4. Assertion (A): In non-polar molecules, the centre of
puncturing it? positive and negative charges coincide.
(a) 10 V (b) 100 V (c) 1000 V (d) 10,000 V Reason (R): The molecules have no permanent
(ii) The dielectric strength of air at N.T.P. is 3 × 106 dispole moment.
V/m. Then the maximum charge that can be given
II. Very Short Answer Type Questions (1 Mark)
to a spherical conductor of radius 3 m is:
(a) 3 × 10–1 C (b) 3 × 102 C 1. Guess a possible reason why water has a much greater
(c) 3 × 10–3 C (d) 3 × 10–4 C dielectric constant (= 80) than say, mica (= 6).
2. Fill in the blanks. 2. What is meant by the term di-electric strength? Write
(i) Electric susceptibility is the ratio of polarisation its SI unit.
to .......... the electric field. 3. State the SI unit of the electric polarization vector
(ii) Electric field inside a conductor is .......... . P. [CBSE S.P. 2018-19]
3. Match the Columns
III. Short Answer Type Questions-I (2 Marks)
Column-I Column-II
4. Define the terms polarisation density and electric
(i) Non-polar molecules (a) Dielectric
susceptibility.
(ii) Polar molecules (b) No permanent dipole
moment 5. What happens to (i) potential difference between the
plates of a capacitor (ii) charge stored when the space
(iii) Ratio of polarisation (c) Net dipole moment
between the plates is filled with a di-electric of di-
to e0 times the electric is zero
electric constant k and the battery has been removed
field before inserting the di-electric.
(iv) Ratio of E0 to E (d) Electric susceptibility
IV. Short Answer Type Questions-II (3 Marks)
For questions numbers 4 two statements are given-one
labelled Assertion (A) and the other labelled Reason 6. Distinguish, with the help of a suitable diagram, the
(R). Select the correct answer to these questions from difference in the behaviour of a conductor and a
the codes (a), (b), (c) and (d) as given below. dielectric placed in an external electric field. How
(a) Both A and R are true and R is the correct does polarised dielectric modify the original external
explanation of A field?

Answers 2.2
I. Objective Type Questions II. Very Short Answer Type Questions
1. (i) (b) (ii) (c) 1. This is because, water molecules in its normal state has
2. (i) e0 times (ii) zero permanent dipole moment.
2. Dielectric strength: The maximum electric field that
3. (i)-(b) (ii)-(c) (iii)-(d) (iv)-(a) can exist in a dielectric without causing the breakdown
4. (a) of its insulating property is called dielectric strength.
Its unit is N/C or V/m
3. C/m2. [CBSE Marking Scheme, 2018-19]
Electrostatic Potential and Capacitance 61
D:\EG_Physics-12_(26-06-2022)\Open_Files\Ch-2\Ch-2
\ 27-Jul-2022 Ved_Goswami Proof-4 Reader’s Sign _______________________ Date __________

4. Polarisation density: The induced dipole moment +σ –σ


developed per unit volume of the dielectric when placed – +
in an external field is called polarisation density. – +

E=o
– +
It is denoted by σP
– +
p q × 2a q × d q – +
σP = = = =  1
V Ad Ad A – +

Electric susceptibility: The polarisation is directly


Behaviour of a di-electric in an external electric field:

→ When a polar dielectric is placed in an electric field,
proportional to the resultant field E in the dielectric.
the atomic dipoles align themselves in the direction

→ → of applied electric field and the polar dielectric gets
P ∝ E polarised. 1
→ →
P = e0 χ E –
Ein
+
+ –
Where χ = susceptibility. – + – +
→ – + – +
P – + – +
χ= →
 1
– + – +
ε0 E
– + – +
Electric susceptibility is the ratio of polarisation to e0 – + – +
times the electric field.
V E0
5. (i) P.D. will decrease V = 0  1
K
In case of non polar di-electric, atomic dipole moment
(ii) Charge stored will remains same. 1 is zero. When it is placed in an external electric field,
6. Behaviour of conductors in an external electric field: centre of mass of atomic positive and negative charges
→ move in opposite direction as unlike charges attract each
When conductor is placed in an electric field, E ext , other. As a result, the dielectric gets polarised, and the
the free charges in conductor begin to move opposite induced electric field is opposite to the external field.

to E ext . So, –ve charge is induced on left side of → → →
conductor and +ve on right side of conductor. The | E net | = | E 0 | − | E in |
→ →
process continues till E induced = E ext . Net E = 0. 1 Thus inside a di-electric, the electric field decreases.1

Topic 3. Capacitors and Capacitance


•• Capacitance: The electrical capacitance of a conductor on the conductor gets neutralized. Thus there is a limit
is the measure of it’s ability to hold electric charge. The on the capacitance of a conductor.
capacitance is defined as the charge required to increase •• Capacitance of a spherical conductor = 4pe0r, r is radius
the potential of the conductor by unit amount. of the sphere.
Capacitance of a conductor depends upon •• Method of increasing capacitance of a conductor:
(i) Size and shape of the conductor Capacitance of a conductor can be increased by placing
(ii) Nature of the surrounding medium. an uncharged conductor closed to it and connecting the
(iii) Presence of other conductors in its neighbourhood uncharged conductor to earth. The device so formed is
called a capacitor.
•• Small capacitance of isolated conductors: Isolated
conductors have a very small capacitance. Reason: •• Principle of a capacitor: Consider a positively charged
When a conductor holds a large amount of charge, it’s metal conductor A and place an uncharged conductor
potential becomes very high. If the associated electric B close to it, as shown in Figure. Due to induction, the
field becomes high enough, the atoms or molecules of the closer face of conductor B acquires negative charge and
surrounding medium get ionized and the charge placed its farther face acquires a positive charge. The negative

62 Physics–12
D:\EG_Physics-12_(26-06-2022)\Open_Files\Ch-2\Ch-2
\ 27-Jul-2022 Ved_Goswami Proof-4 Reader’s Sign _______________________ Date __________

charge on conductor B tends to reduce the potential on •• From eq. (1) we see that for large C, V is small for a
conductor A, while the positive charge on conductor given Q, this means a capacitor with large capacitance
B tends to increase the potential on A. As the negative can hold large amount of charge Q at a relatively small V.
charge of conductor plate B is closer to conductor A than S.I. unit of capacitance is Farad.
its positive charge, so the net effect is that the potential
of A is decreased by a small amount and hence its The capacitance of a conductor is said to be 1 Farad if
capacitance increases by a small amount. the charge of 1 Coulomb given to it raises its potential
by 1 Volt.
A B A B
+ + – + + + – In practice 1 Farad is a very big unit. The most common
– –
+
+
+
+ –
+
+
+
+
+
+ – units are its sub multiples.
+ + – + + + –
+ + –
– + + + –
– 1 microfarad = lµF = l0–6 F
+ + + + +
+ +

– + + +

– 1 nanofarad = 1nF = l0–9 F
+ + – + + + –
+ + –

+ + + –

1 picofarad = 1pF = l0–12 F
+ + + + +
– –
+
+
+
+ –
+
+
+
+
+
+ – •• Combination of capacitors: The capacitors can be
– –
+
+
+
+
– +
+
+
+
+
+
– combined suitably to get the required capacitance. Two
– –
+ + – + + + – simple possibilities of grouping of capacitors are
Principle of a capacitor
(1) Series grouping (2) Parallel grouping
Now if the positive face of plate B is earthed, its positive
(1) Series grouping of capacitors: The capacitors are
charge gets neutralised due to the flow of electrons from
said to be connected in series if we can navigate
the earth to the plate B. The negative charge on B is held from one point to another in the circuit only through
in position due to the positive charge on A. The negative one path. The figure shows capacitors C1, C2 and C3
charge on B reduces the potential of A considerably and connected in series
hence increases its capacitance by a large amount.
1 1 1 1
Hence we see that the capacitance of an isolated = + +
conductor is considerably increased when we place an CS C1 C2 C3
earthed connected conductor near it. Such a system of C1 C2 C3
two conductors is called a capacitor.
+Q –Q +Q –Q +Q –Q
•• Capacitor: Capacitor is an assembly of two conductors
separated by an insulating medium and it is used to
store electric energy by accumulating charge on the V
conductors.
Note:
The two conductors have equal and opposite charges
+Q and –Q there is a potential difference V between the (i) The charge on each capacitor is same.
conductors. (ii) The reciprocal of equivalent capacitance of a series
•• Capacitance: The magnitude of the charge ‘Q’ on the combination is equal to the sum of the reciprocals of
capacitor is directly proportional to potential difference the individual capacitances.
between the two conductors (iii) The equivalent capacitance is smaller than the
Q∝V smallest individual capacitance.
or Q = CV (iv) If n identical capacitors of capacitance C each are
where C is the constant of proportionality connected in series, then equivalent capacitance is
Q C
C=  ...(1) .
V n
The constant C is called the capacitance of the capacitor.
Capacitance is independent of Q or V. The capacitance (2) Parallel grouping of capacitors: Capacitors are said
C depends upon the geometrical configuration i.e. the to be connected in parallel between two points, if we
shape, size, separation and the nature of the material can move from one point to another along different
separating the conductors. paths.
“The capacitance is defined as the ratio of the charge The positive plates of all the capacitors are connected
stored on either plate to the potential difference across to one common point and the negative plates are
the plates.” connected to another common point.

Electrostatic Potential and Capacitance 63


D:\EG_Physics-12_(26-06-2022)\Open_Files\Ch-2\Ch-2
\ 27-Jul-2022 Ved_Goswami Proof-4 Reader’s Sign _______________________ Date __________

The potential difference across each capacitor is same and Q Aε 0


the charge gets divided depending upon the capacitance C= =
      V d
of the capacitors.
+Q1 –Q1
Thus the capacitance of a parallel plate capacitor depends
+ – (i) Area of the plates (C ∝ A)
+ – C1
+ – (ii) Distance between the plates (C ∝ 1/d)
+Q2
+
–Q2
– (iii) Permittivity of the medium between the plates
+ –
C
(C ∝ e0)
– 2
+

•• Capacitance of a parallel plate capacitor when the
+
+ – space between the plates is filled with dielectric
+Q3 –Q3

Consider a dielectric inserted between the plates fully
+
+ – C3 occupying the intervening region. The dielectric is
+ – polarized by the field and, the positive charge appears
V
on the face of the di-electric facing the negative plate of
the capacitor and vice versa.
(+) (–)
–QP +QP
(i) Figure shows three capacitors of capacitance C1, C2
and C3 connected in parallel
or CP = C1 + C2 + C3
For a parallel combination of n capacitors, we can
write
CP = C1 + C2 + ... Cn
Note: For parallel combination of capacitors
(i) The equivalent capacitance is equal to the sum of the
+σ –σ
individual capacitances.
The induced electric field due to the polarization of
(ii) The equivalent capacitance is larger than the largest
the dielectric is opposite to the original electric field
individual capacitance.
between the plates of the capacitor. So the net electric
(iii) The potential difference across each capacitor is same. field between the plates decreases
(iv) The charge on each capacitor is proportional to its Aε 0 K
capacitance. C=
     d
•• Energy stored in a Capacitor: A capacitor is a device The product e0K is called the permittivity of the medium
to store energy. The process of charging a capacitor and is denoted by e.
involves the transferring of charges from its one plate to
another. The work done in charging the capacitor is stored ε
    e = e0K. or K=
in the form of electrical potential energy. This energy is ε0
supplied by the battery at the expense of its chemical K is called the di-electric constant of the substance and
energy. This energy can be recovered by allowing the is a dimensionless ratio.
capacitor to discharge.
C
=K
1 Q2 1 1 C0
U= ⋅ = ⋅ CV 2 = QV     
   2 C 2 2
So di-electric constant can be defined as the ratio of
•• Capacitance of a parallel plate capacitor: A parallel the capacitance of a parallel plate capacitor with the
plate capacitor consists of two large plane parallel given medium as the medium of separation between
conducting plates separated by a small distance. Let us the plates to the capacitance of the same capacitor
first take the medium between the plates to be vacuum. with vacuum as the medium of separation of the
Let A be the area of each plate and d the separation plates.
between the plates. As     K > 1

64 Physics–12
D:\EG_Physics-12_(26-06-2022)\Open_Files\Ch-2\Ch-2
\ 27-Jul-2022 Ved_Goswami Proof-4 Reader’s Sign _______________________ Date __________

So capacitance of a capacitor increases from its vacuum Battery disconnected Battery kept connected
value when the space is filled with a dielectric. from the capacitor across the capacitor
•• Capacitance of a parallel plate capacitor with a
Q = Q0 (constant) Q = KQ0
dielectric slab of thickness t < d
The capacitance of the capacitor on introduction of V0
V= V = V0 (constant)
dielectric slab t < d K
ε0A
C= E0
t E= E = E0 (constant)
d −t + K
k
C = KC0 C = KC0
Note: Effect of dielectric on various parameters when
a slab of dielectric constant (K) fills the entire space U0
U= U = KU0
between the plates. K

EXERCISE 2.3
I. Objective Type Questions (1 Mark) and Q′2 as charges on C1 and C2 and V1 and V2
1. Choose the correct answers from the given options as voltage respectively]. Then
(MCQs).
(i) A capacitor of 4 mF is connected as shown in
circuit here. The internal resistance of the battery
is 0.5 W. The amount of charge on the capacitor
plates will be

(a) charge on C1 gets redistributed such that V1


= V2
(b) charge on C1 gets redistributed such that Q′1
= Q′2
(c) charge on C1 gets redistributed such that C1V1
+ C2V2 = C1E
(a) 0 (b) 4 mC (d) charge on C1 gets redistributed such that Q′1
(c) 16 mC (d) 8 mC + Q′2 = Q
(ii) A parallel plate capacitor is made of two dielectric (iv) A variable capacitor is connected to a 200 V
blocks in series. One of the blocks has thickness battery. If its capacitance is changed from 2 mF
d1 and dielectric constant k1 and the other has to X mF, the decrease in energy of the capacitor
thickness d2 and dielectric constant k2 as shown is 2 × 10–2 J. The value of X is [CBSE 2022]
in the figure. This arrangement can be thought as a (a) 1 mF (b) 2 mF
dielectric slab of thickness d(d1 + d2) and effective (c) 3 mF (d) 4 mF
dielectric constant k. The k is (v) A car battery is charged by a 12 V supply, and
energy stored in it is 7.20 × 105 J. The charge
passed through the battery is [CBSE 2022]
4
(a) 6.0 × 10 C (b) 5.8 × 10 J 3

(c) 8.64 × 10 J6 (d) 1.6 × 105 C


(a) k1d1 + k2d 2 (b) k1d1 + k2d 2 2. Fill in the blanks.
d1 + d 2 k1 + k2 (i) The capacity of an isolated conducting sphere of
k1k2(d1 + d 2) 2k1k2 radius R is proportional to .......... .
(c) (d) (ii) Two spherical conductors each of capacity C are
(k1d1 + k2d 2) k1 + k2
charged to potential V and – V. These are then
(iii) In the circuit shown in figure, initially key K1 is
connected by means of a fine wire. The loss of
closed and key K2 is open. Then K1 is opened
energy is .......... .
and K2 is closed (order is important). [Take Q′1

Electrostatic Potential and Capacitance 65


D:\EG_Physics-12_(26-06-2022)\Open_Files\Ch-2\Ch-2
\ 27-Jul-2022 Ved_Goswami Proof-4 Reader’s Sign _______________________ Date __________

(iii) A 2mF capacitor is charged to 100 volt and then Reason (R): The surface density of charge on the
its plates are connected by a concluding wire. The plate remains constant or unchanged.
heat produced is .......... . II. Very Short Answer Type Questions (1 Mark)
3. State True or False 1. Two isolated metal spheres A and B have radii R and
(i) Four capacitors each of 25 mF are connected as 2R respectively, and same charge q. Find which of
shown in figure. The voltmeter shows a dc of 200 the two spheres have greater capacitance.
V. The charge on each plate of capacitor is ± 2 ×
2. What meaning would you give to the capacitance of
10–2 C.
a single conductor?
V
3. Sketch graph to show how the charge given to a
capacitor of capacitance C varies with potential
difference.
4. Write a physical quantity whose SI unit is coulomb
volt–1. Is it a vector or a scalar quantity?
(ii) A number of capacitors, each of capacitance 1 mF 5. What is the basic purpose of using a capacitor?
and each one of which gets punctured if a potential 6. Can you place any amount of charge on a capacitor?
difference just exceeding 500 V is applied are Give reason for your answer.
provided. Then an arrangement suitable for giving 7. An uncharged insulated conductor A is brought near
a capacitor of capacitance 3 mF across which 2000 a charged insulated conductor B. What happens to
V may be applied requires at least 4 component the charge and potential of B?
capacitors.
8. What is the equivalent capacitance C of the five
(iii) Three capacitors of capacitances 3 mF, 9 mF and capacitors, connected as shown in the Fig?
18 mF are connected once in series and then in
C1 C2 C3 C4 C5
parallel. The ratio of equivalent capacitances CS/
CP will be 1 : 15.
4. Match the Columns
Column-I Column-II
(i) Capacitance of a spherical (a) Q
V
conductor
(ii) Capacitance of a parallel (b) K 9. The graph below shows the variation of the total
plate capacitor energy E stored in a capacitor against the value of the
(iii) Energy stored in a capacitor (c) 4pe0r capacitance C itself. Which of the two, the charge on
the capacitor or the potential used to charge it, is kept
(iv) Ratio of C to C0 1
(d) CV 2 constant for this graph? [S.P. 2008]
2
For questions numbers 5 two statements are given-one
labelled Assertion (A) and the other labelled Reason
(R). Select the correct answer to these questions from E

the codes (a), (b), (c) and (d) as given below.


(a) Both A and R are true and R is the correct
explanation of A
(b) Both A and R are true but R is NOT the correct C
explanation of A
10. Two isolated metal spheres A and B have radii R and
(c) A is true but R is false 2R respectively, and same charge q. Find which of the
(d) A is false and R is also false two spheres have greater energy density just outside
5. Assertion (A): A parallel plate capacitor is connected the surface of the spheres.
across battery through a key. A dielectric slab of III. Short Answer Type Questions-I (2 Marks)
dielectric constant K is introduced between the plates. 11. The graph shows the variation of voltage V across the
The energy which is stored becomes K times. plates of two capacitors A and B, versus amount of
66 Physics–12
D:\EG_Physics-12_(26-06-2022)\Open_Files\Ch-2\Ch-2
\ 27-Jul-2022 Ved_Goswami Proof-4 Reader’s Sign _______________________ Date __________

charge Q stored on them. Which of the two has higher 17. The space between the plates of a parallel plate
capacitance? Give reason. capacitor is completely filled in two ways. In
the first case, it is filled with a slab of dielectric
constant K. In the second case, it is filled with two
B slabs of equal dimensions but dielectric constants
K1 and K2 respectively as shown in the figure.
A
V The capacitance of the capacitor is same in the
two cases. Obtain the relationship between K, K1
and K2. [AI 2020]

l/2 K1 l/2
Q

12. Explain the principle of a capacitor. What is the K


l/2 K2 l/2
symbol used for representing a capacitor in a circuit
diagram? [Delhi 2005, 2014]
d d
13. A parallel plate capacitor with air in between the (Case 1) (Case 2)
plates has a capacitance of 8 pF. What will be the
capacitance if the distance between the plates be IV. Short Answer Type Questions-II (3 Marks)
reduced by half and the space between them is filled 18. Show that the force on each plate of a parallel plate
with a substance of di-electric constant K = 6? 1
capacitor has a magnitude equal to qE,
14. Find total energy stored in capacitors given in the 2
circuit[CBSE S.P. 2019-20] where q is the charge on the capacitor, E is the
2µF magnitude of electric field between the plates. Explain
1
the origin of factor . [NCERT]
2
1µF
6V 2µF 19. Given two parallel conducting plates of area A and
1µF charge densities +σ and –σ. A dielectric slab of
constant K and a conducting slab of thickness d each
are inserted in between them as shown.
2µF
+ –
15. Obtain the expression for the energy stored in a
capacitor connected across a dc battery. Hence define + –

energy density of the capacitor. [Delhi 2020] + –


Conductor
Dielectric

16. The space between the plates of a parallel plate + –

capacitor is completely filled in two ways. In + –


the first case, it is filled with a slab of dielectric –
+
constant K. In the second case, it is filled with two
slabs of equal thickness and dielectric constants +
d d d d d

K1 and K2 respectively as shown in the figure.


(i) Find the potential difference between the plates.
The capacitance of the capacitor is same in the
two cases. Obtain the relationship between K, K1 (ii) Plot E versus x graph, taking x = 0 at positive plate
and K2. [Delhi 2020] and x = 5d at negative plate. [S.P. 2015-16]
20. A slab of material of dielectric constant K, has the
same area as the plates of a parallel plate capacitor, but
has thickness d , where d is the separation between
2
K K1 K2 the plates, find the expression for the capacitance
when the slab is inserted between the plates.
[Delhi 2010, 2013]
d d/2 d/2
(Case 1) (Case 2) 21. Two dielectric slabs of di-electric constants K1 and
K2 are filled in between the two plates, each of area

Electrostatic Potential and Capacitance 67


D:\EG_Physics-12_(26-06-2022)\Open_Files\Ch-2\Ch-2
\ 27-Jul-2022 Ved_Goswami Proof-4 Reader’s Sign _______________________ Date __________

A, of the parallel plate capacitor as shown in the Fig. reduced by 120 V, the charge stored in it becomes
Find the net capacitance of the capacitor. 120 µC.
[Delhi 2005] Calculate:
A/2 A/2 (i) The potential V and the unknown capacitance C.
(ii) What will be the charge stored in the capacitor,
if the voltage applied had increased by 120 V?
K1 K2
[Delhi 2013]
26. The equivalent capacitance of the combination
between A and B is 4 µF.
22. Three circuits, each consisting of a switch ‘S’ and A B
two capacitors, are initially charged, as shown in the
figure. After the switch has been closed, in which 20F C

circuit will the charge on the left-hand capacitor (a) (i) Calculate capacitance of capacitor C.
increase, (b) decrease and (c) remain same? Give (ii) Calculate charge on each capacitor if a 12 V
reasons. [AI 2015] battery is connected across A and B.
S S (iii) What will be the potential drop across each
capacitor? [Delhi 2009]
+ + 6Q + + 3Q
6Q 2C C 3Q C C 27. Net capacitance of three identical capacitors in
series is 1 µF. What will be their net capacitance in
(a) (b) parallel? Find the ratio of energy stored in the two
S configurations if they are both connected to the same
source. [Delhi 2011]
+ +
3C C 28. Two parallel plate capacitors X and Y have the same
3Q
6Q
area of plates and same separation between them. X
(c) has air between the plates and Y contains a di-electric
medium of er = 4.
23. The two plates of a parallel plate capacitor are 4 mm
apart. A slab of dielectric constant 3 and thickness 3
mm is introduced between the plates with its faces x Y

parallel to them. The distance between the plates as so


adjusted that the capacitance of the capacitor becomes
2
rd of its original value. What is the new distance 12 V
3 Calculate (i) Capacitance of X and Y if equivalent
between the plates? [AI 2008]
capacitance of the combination is 4 µF.
24. The figure below shows two identical capacitors
(ii) Potential difference between the plates of X and
C1 and C2, each of 1 µF capacitance , connected to
a battery of 6 V. Initially, switch S is closed. After Y.
sometime, S is left open and a di-electric slab of (iii) What is the ratio of electrostatic energy stored is
dielectric constant K = 3 is inserted to fill completely X and Y?  [Delhi 2009]
the space between the plates of two capacitors. How 29. A network of four 10 µF capacitors is connected to a
will the (i) charge (ii) potential difference between 500 V supply as shown in the figure. Determine the
the plates of the capacitors be affected after the slabs [AI 2013]
are inserted? [AI 2011] + –
+ –
s
+ –
C2
++ +
1F –– –
6V C1 C1 C3
1F – – –
C2 + + +
C4
+ –
25. A capacitor of unknown capacitance is connected + –

across a battery of V volts. The charge stored in it
+

is 360 µC. When potential across the capacitor is +


500 V

68 Physics–12
D:\EG_Physics-12_(26-06-2022)\Open_Files\Ch-2\Ch-2
\ 27-Jul-2022 Ved_Goswami Proof-4 Reader’s Sign _______________________ Date __________

(i) equivalent capacitance of the network and 6µF 12µF

(ii) charge on each capacitor.


2V
30. Calculate the potential difference and the energy V2

stored in the capacitor C2 in the circuit shown in the


figure. Given potential at A is 90 V, C1 = 20 µF, C2
= 30 µF and C3 = 15 µF. [AI 2015]
V

A 35. In a network, four capacitors C1, C2, C3 and C4 are


C1 C2 C3
connected as shown in the figure.
31. Find the equivalent capacitance of the network shown
in the figure, when each capacitor is of 1 µF. When the
ends X and Y are connected to a 6 V battery, find out
(i) the charge and (ii) the energy stored in the network.
[AI 2015]
C1

C5 C2 (i) Calculate the net capacitance in the circuit.


X Y (ii) If the charge on the capacitor C 1 is 6 mC,
C3 (a) calculate the charge on the capacitors C3 and
C4
C4, and (b) net energy stored in the capacitors C3
and C4 connected in series. [AI 2019]
32. Three identical capacitors C1, C2 and C3 of capacitance 36. In the figure given below, find the
6 µF each are connected to a 12 V battery as shown
(i) equivalent capacitance of the network between
in the figure. Find
points A and B.
(i) charge on each capacitor Given C1 = C5 = 4 µF, C2 = C3 = C4 = 2 µF
(ii) equivalent capacitance of the network (ii) maximum charge supplied by the battery, and
(iii) energy stored in the network of capacitors (iii) total energy stored in the network. [AI 2020]
C1 C1 C2 C3 C4 C5
A B

C3
12V

+ –
C2
5V
33. Find the total energy stored in the capacitors in the
V. Long Answer Type Questions (5 Marks)
given network:
37. (i) Derive the expression for the energy stored in
A
2F(C1)
a parallel plate capacitor. Hence obtain the
expression for the energy density of the electric
C5
field.
6V C3 2F(C2) (ii) A fully charged parallel plate capacitor is connected
1F across an uncharged identical capacitor. Show
that the energy stored in the combination is less
D than that stored initially in the single capacitor.
2F(C4)
[AI 2015]
34. Two capacitors of capacitance 6 µF and 12 µF are 38. (i) Describe briefly the process of transferring the
connected in series with a battery. The voltage across charge between the two plates of a parallel plate
6 µF capacitor is 2 V. Compute the total battery capacitor when connected to a battery. Derive an
voltage. expression for the energy stored in a capacitor.

Electrostatic Potential and Capacitance 69


D:\EG_Physics-12_(26-06-2022)\Open_Files\Ch-2\Ch-2
\ 27-Jul-2022 Ved_Goswami Proof-4 Reader’s Sign _______________________ Date __________

(ii) A parallel plate capacitor is charged by a battery ratio of the energy stored in the combination to
to a potential difference V. It is disconnected from the initial energy on the single capacitor.
battery and then connected to another uncharged [Delhi 2019]
capacitor of the same capacitance. Calculate the

Answers 2.3
I. Objective Type Questions 2
1 1  σ
1. (i) (d) (ii) (c) (iii) (a & d) (iv) (d) (v) (d) 10. Energy density u = ε0 E 2 = ε0  
2. (i) R 2 2
(ii) 2CV (iii) 0.001 J 2 2  ε0 
3. (i) False (ii) False (iii) True 1 σ2 1 Q2
4. (i)-(c) (ii)-(a) (iii)-(b) (iv)-(d) = =
2 ε 0 2ε 0 A 2
5. (c)
II. Very Short Answer Type Questions 1
So, u∝
1. As C = 4πe0R A2
For sphere A, CA = 4πe0R uA > uB
For sphere B, CB = 4πe02R = 2CA Q
\ Clearly CB > CA 11. As Q = CV or C =  1
V
2. A single conductor is a capacitor with second conductor
From the graph we see that for the same value of Q, A
at infinity.
has lower value of V than B, as VA < VB so CA > CB.
3. Graph will be a straight line as Q ∝ V.
Thus A has higher capacitance. 1
12. Principle of capacitor: Consider charged metal plate A
of area a and negligible thickness and uncharged metal
Q
plate B placed close to it. 1
–ve charged comes on left side and +ve on right side
of B due to induction. –ve charge is going to decrease
potential of A while +ve is going to increase it. But –ve
V
charge is more effective, being closer to A. Net effect
is the potential of A decreasing by small amount, V′ =
Slope of the line = C
V – dV. So the capacity to hold charge has increased.
4. Capacitance, it is a scalar quantity.
Then, B is earthed and its +ve charge vanishes, but –ve
5. Basic purpose of using a capacitor is to store charge
charge remain in place due to +ve of A. Potential of A
and electric energy.
decrease by large amount and now even more charge
6. No,
can be given to it. We can say that capacitance of an
Reason: As the charge given to a capacitor increases,
insulated conductor increases when placed near earth
the potential difference between its plates also increases.
connected conductor.
A stage will come when the electric field between the
plates will exceed the di-electric strength of the medium
and the medium will get ionised. Thus no more charge
can be placed on it.
7. Charge is unchanged and potential of B will decrease.
8. C3, As the combinations of C1, C2 and C4, C5 have been
shorted.
  1
1 1 Q2 1
9. As U = CV2 = = QV Symbol for capacitor:
2 2 C 2
1
If Q is constant, then U ∝
C (Fixed capacitor) (Variable capacitor)
So we get a graph of the type given. Hence Q is constant.
70 Physics–12
D:\EG_Physics-12_(26-06-2022)\Open_Files\Ch-2\Ch-2
\ 27-Jul-2022 Ved_Goswami Proof-4 Reader’s Sign _______________________ Date __________

13. With air in between the plates transferring each instalment of charge. Let at any instant
Aε 0 charge on two plates are ±q
C0 = = 8 pF 1 q
d P.D. between the plates V =
C
d The amount of work done in supplying an additional
When d′ = and a dielectric is introduce
2 charge dq to the capacitor is
Aε 0 K Aε 0 × 6 × 2 dW = V·dq
C′ = = q
d′ d dW = × dq 1
= 12C0 = 12 × 8 pF = 96 pF 1 C
14. Total work done in giving a charge Q to the capacitor
Q
C2 = 2µF q 1 Q2
W = ∫ dq =
0
C 2 C
1µF = C4 This work is stored in the form of PE
6V 2µF = C3
C1 = 1µF 1 Q2 1 1
\    U = = = CV 2 QV  ½
2 C 2 2
Energy stored per unit volume
C5 = 2µF Energy stored in the capacitor
C2 and C3 are in series =
Volume between the space
1 1 1 in parallelplate capacitor
= + = 1 ⇒ C´ = 1µF ½
C′ 2 2 1 1 ε0 A 2
C´ and C4 are in parallel CV 2 V
u= 2 = 2 d
C´´ and C4 are in parallel A×d A×d
C´´ = 1 + 1 = 2µF But V=E×d
1 ε0 A 2 2
C´´ and C5 are in series E d 1
u= 2 d
2
1 1 1 or   u = 2 e0E  ½
= + = 1 ⇒ C´´´ = 1µF ½ A×d
C′′′ 2 2 Here E is the strength of electric field in the space
C´´´ and C1 are in parallel between the plates of the capacitor
Ceq = 1 + 1 = 2µF ½ SI unit of energy density u is J m–3
Energy stored 16. Let initial capacitance of the capacitor be C1
1 1 ε0 A

\ C1 = K
U = CV2 = × 2 × 10–6 × 62 ½ d
2 2
When dielectrics of dielectric constant K1 and K2 is
= 36 × 10–6 J d
filled of thickness each then the arrangement will
[CBSE Marking Scheme, 2019-20] 2
15. Let initially there is no charge on the plates. Let behave as the two capacitors are connected in series
positive charge be transferred from plate 2 to plate 1 then
in very small instalments of dq each till conductor 1 1 1 1
= +  1
gets charged +Q. By charge conservation, conductor 2 C2 ε0A ε0 A
K1 K2
would acquire charge –Q. d d
2 2
+Q + – –Q
– 1  1 1  K  K 2 + K1 
+ =  +  =  
+ – ε0 A  2K1 2K 2  2C1  K1K 2 
+ –
+ – d
+ – C  2K1K 2 
+ – or C2 = 1   1
+ – K  K1 +K 2 
+ – Given C1 = C2
1 + – 2
2K1K 2
At every stage of charging conductor 1 is at a higher So, K=
potential than conductor 2. Hence work is done in K1 + K 2

Electrostatic Potential and Capacitance 71


D:\EG_Physics-12_(26-06-2022)\Open_Files\Ch-2\Ch-2
\ 27-Jul-2022 Ved_Goswami Proof-4 Reader’s Sign _______________________ Date __________

Kε0 A (ii) Graph


17. C1 =  ...(1)
d
C2 = parallel combination of two capacitors
( 2) + K ε (A 2)
E0
K1ε0 A 2 0
Ei 1
=  1
d d E0
k
ε0 A
= ( K1 + K 2 )  ...(2)
2d o Distance
d 2d 3d 4d 5d between
 Given that C1 = C2 plates x →

So, on equating (1) and (2), we get 20. Before inserting the dielectric slab, let the electric field
between the plates is E0 and the potential difference be
K1 + K 2
K=  1 V0.
2 So V0 = E0d ...(1)
18. Suppose we increase the separation of the capacitor
When the di-electric is inserted, the di-electric gets
plates by a small distance ∆x against the force F.
polarised and the induced electric field is opposite to
Work done by the external agent = F ∆x. the applied electric field. Let the electric field inside the
Let u be the energy stored per unit volume then increase di-electric be E.
in potential energy of the capacitor. + –P +P –

= u × increase in vol.
= uA ∆x 1
F ∆x = uA ∆x
1     1
F = uA = e0E2A
E0
or E
2
1 1 σ 
= (e0E) AE =  ε 0  AE
2 2  ε0 
1
qE as σA = q
= 1 E0
2 So E=
k
Reason for factor 1 : d d
2 P.D. between the plates = V = E0 +E
2 2
As in between the plates electric field is E but just
outside the plates electric field is zero. So average value V = E0 d + E 0 d = E 0 d  k + 1 
E 2 k 2 2  k 
contributes to the force. 1
2 σ d  k + 1
V=   1
19. (i) Net potential ε0 2  k 
E σ
V = E0d + 0 d + E0d + 0 + E0d as E0 =
K ε0
E
V = 3E0d + 0 d 1 qd  k + 1  q
K V=   Q σ = A 
+
Aε 0 2  k 
E0 E0 E0 0 E0 –
q q
+
K –
\ C= =
V qd  k + 1
 
Aε 0 2  k 
+ –
Conductor
Dielectric


+
    1 2Aε 0  k   2k  
=   = C0   1
+ – d k +1 k + 1
+ – 21. This arrangement can be supposed to be a parallel
combination of two capacitors, each with plate area
+
d d d d d
– A/2 and separation d.

72 Physics–12
D:\EG_Physics-12_(26-06-2022)\Open_Files\Ch-2\Ch-2
\ 27-Jul-2022 Ved_Goswami Proof-4 Reader’s Sign _______________________ Date __________

A/2 A/2  1 3
d ′ − t 1 −  = d
 k 2
     1
k1 k2 3  1

or d + t 1 −  
d′ = 1
2  k
As d = 4 mm, t = 3 mm, k = 3
K1Aε 0 K Aε
C1 = , C2 = 2 0  1 3  1
2d 2d We get d′ = × 4 mm + 3mm 1 − 
2  3
Aε 0  K1 + K 2 
C = C1 + C2 =  
d  2 = (6 + 2) mm = 8 mm 1
Aε 0 24. When S is closed:
= (K1 + K2)  1 V1 = P.D. across C1 = 6 V, P.D. across
2d
22. (a) Potential difference across left hand capacitance C2 = 6 V = V2 , C1 = C2 = 1 × 10–6 F
Q L 6Q 3Q as they are connected in parallel.
= VL = = = Q1 = C1V1 = 10–6 × 6
CL 2C C
Q 3Q = 6 × 10–6 C 1
P.D. across right hand capacitance = VR = R = Q2 = 6 × 10 C–6
CR C
P.D. across left hand capacitance = P.D. across right When S is left open and dielectric is introduced between
hand capacitance. the plates as C1 is connected to battery
So no charge will flow. Hence charge remains same. V1′ = P.D. across C1 = 6 V
 1 C1′ = KC1 = 3 × 1
(b) For left hand capacitor, = 3µF = 3 × 10–6 F
Q L 6Q Q1′ = C1′ V1′ 1
VL = = ,
CL C = 3 × 10–6 × 6 = 18 × 10–6 C
For right hand capacitor, As C2 is disconnected from battery, so the charge on
Q R 3Q C2 will not change.
VR = =
CR C
i.e. Q′2 = Q2 = 6 × 10–6 C,
As VL > VR so charge will flow from left hand
capacitance to right hand capacitance. So charge on C2′ = KC2 = 3 × 10–6 F
left hand capacitance will decrease. 1 Q′ 6 × 10−6
V′2 = 2 = = 2V 1
6Q 2Q 3Q C′2 3 × 10−6
(c) VL= = = , VR , As VR > VL
3C C C
25. (i) + –
so charge will flow from right hand capacitance C
to left hand, thus charge on left hand capacitance As Q = CV, Q = 360 µC, P.D. = V
will increase. 1 So CV = 360 µC ...(1)
23. With vacuum as medium of separation between the
When V′ = (V – 120), Q′ = 120 µC
plates
So Q′ = CV′
Aε 0
C0 = 120 µC = C(V – 120) ...(2)
d
Let d′ is the new distance, From equation (1)
Aε 0 360 × 10−6
C=  1 C=  ...(3)
 1 V
d ′ − t 1 −  From equation (2)
 k
2 120 × 10 −6
Given C = C0 C=  ...(4)
3 (V − 120)
2 Aε 0 Aε 0 From equation (3) and (4) we get
=
3 d  1 360 × 10−6 120 × 10 −6
d ′ − t 1 −  =  1
 k V (V − 120)

Electrostatic Potential and Capacitance 73


D:\EG_Physics-12_(26-06-2022)\Open_Files\Ch-2\Ch-2
\ 27-Jul-2022 Ved_Goswami Proof-4 Reader’s Sign _______________________ Date __________

3(V – 120) = V CS = 4 µF CY = 4 CX
2V = 360 1 1 1
So V = 180 Volt = +
CS CX CY
Substitute V = 180 Volt in equation (1)
    C × 180 = 360 × 10–6 1 1 1 5
= + =
C = 2 × 10–6 = 2 µF 1 4 CX 4C X 4CX
(ii) When voltage is increased by 120 V CX = 5 µF, CY = 4 × 5 = 20 µF 1
Then Q = CV = 2 × 10–6 (180 + 120) (ii) qX = qY = CSV = (4 × 12) µC= 48 µC
= 600 µC 1 qX 48 × 10−6
26. (i) Capacitors are connected in series so VX = = = 9.6 V
CX 5 × 10−6
1 1 1
= + , C1 = 20 µF, qY 48 × 10−6
CS C1 C2 VY = = = 2.4 V 1
CS = 4mF CY 20 × 10−6
1 1 1 2

4
=
20 C
+ (iii) As qX = qY, U = 1 q
2 C
1 1 1 4 1
= − = =  UX C 20
C 4 20 20 5 = Y = = 4 1
UY CX 5
C = 5 µF 1
(ii) q1 = q2 = CSV 29. (i) C 1 , C 2 and C 3 are connected in series. Their
= 4 × 10–6 × 12 = 48 × 10–6 C 1 equivalent capacitance C′ is given by
q 48 × 10−6 1 1 1 1 1 1 1
(iii) V1 = 1 = = 2.4 volt = + + = + +
C1 20 × 10−6 C′ C1 C2 C3 10 10 10

q2 48 × 10−6 C′ = 10 µF 1
V2 = = = 9.6 volt
C2 5 × 10−6 3
or V2 = V – V1 = 12 – 2.4 = 9.6 V 1 C′ and C4 are connected in parallel.
C′
27. Let each capacitors be of capacitance C. + –
In series C1 = C2 = C3 = C
1 1 1 1 C4
= + +
CS C1 C2 C3 + –      ½
1 1 1 1 3
= + + =
1 C C C C + 500V –

or      C = 3µF  1 Their equivalent capacitance C is given by
10 40
Now in parallel C = C′ + C4 = + 10 = µF ½
3 3
Cp = C1 + C2 + C3
Cp = 3 + 3 + 3 = 9 µF 1 (ii) q4 = C4V, V = 500 V
When connected to the same source q4 = 10 × 10–6 × 500
1 1 = 5 × 10–3 C
US = CSV2, UP = CPV2
2 2 q1 = q2 = q3 = C′V
US C 1
= S =  1 = 10 × 10–6 × 500
UP CP 9 3
28. (i) Let capacitance of capacitor X is CX. As X and Y
= 5 × 10–3 C 1
have same dimensions. But Y has a di-electric of 3
dielectric constant 4 in between its plates so

74 Physics–12
D:\EG_Physics-12_(26-06-2022)\Open_Files\Ch-2\Ch-2
\ 27-Jul-2022 Ved_Goswami Proof-4 Reader’s Sign _______________________ Date __________

30. As C1, C2 and C3 are in series C1 C2

A
C1 C2 C3
X C5 Y      ½

So C1 = 20 µF C3 C4

C2 = 30 µF C1 C2
C3 = 15 µF
X Y      ½
Q Q Q
So + + =V
C1 C2 C3 C3 C4

Q Q Q 32. (i) As C1 and C2 are in series


 + +  ×106 = 90
 20 30 15  1 1 1 1 1 2 1
= + = + = =
(3Q + 2Q + 4Q) × 106 C12 C1 C2 6 6 6 3

⇒ = 90
60 C12 = 3 µF
9Q C3 is parallel with C12 so net capacitance

⇒ × 106 = 90 C = C3 + C12 = 6 + 3 = 9 µF 1
60
(ii) q3 = C3V3 = 6 × 10–6 × 12
90 × 60
Q= = 72 × 10–6 C
9 × 106
As C1 and C2 are in series.
= 6 × 10–4 C 1 So q1 = q2
−4
Q 6 × 10 600 = C12V = 3 × 10–6 × 12
V2 = = = = 20 V 1
C2 30 × 10 −6
30 = 36 × 10–6 C 1
Energy store in capacitor 1
(iii) U1 = q V , P.D. across
1 1 2 1 1
C2 = C2V22 = × 30 × 10–6 × (20)2 C1 = P.D. across C2 = 6 V
2 2
1
= 15 × 400 × 10–6 = 60 × 10–4 J 1 U1 = × 36 × 10–6 × 6
2
31. This circuit is equivalent to wheatstone bridge
= 108 × 10–6 J = 108 µJ
C1 C 1
As = 3 U2 = × 36 × 10–6 × 6
C2 C4 2
So the bridge is balance so P.D. across C5 = 0 = 108 × 10–6 J = 108 µJ
C1 and C2 are in series, their equivalent 1
U3 = × 72 × 10–6 × 12
C1C2 1×1 1 2
C′ = = = µF ½
C1 + C2 1 + 1 2 = 432 × 10–6 J = 432 µJ 1
33. C1 and C2 are in series so their equivalent C12
Similarly C 3 and C 4 are also in series and their
C1C2 2×2
equivalent capacitor C12 = = = 1 mF ½
1 C1 + C2 2 + 2
C″ = µF. ½ Circuit reduces to
2
A
C′ and C″ are in parallel. So net capacitor
1 1 C3
C = C′ + C″ =  +  µF = 1 µF.
2 2 6V
C5
C12

Energy stored in the network 1F

1 1 D
= CV2 = × 1 × 10–6 × 62 C4
2 2 C3 and C12 are in parallel so their equivalent C123 is
= 18 × 10–6 J 1 C123 = C3 + C12 = 1 + 1 = 2 mF ½

Electrostatic Potential and Capacitance 75


D:\EG_Physics-12_(26-06-2022)\Open_Files\Ch-2\Ch-2
\ 27-Jul-2022 Ved_Goswami Proof-4 Reader’s Sign _______________________ Date __________

A 36. (i) equivalent capacitance = C3 = 2 µF


as the combination of C1, C2 and C4, C5 have been
C5 C123 shorted.
6V
1 F (ii) q = CV = C3V = 2 × 10–6 × 5 = 10–5 C
1 1
C (iii) Total energy stored = qV = × 10–5 × 5 = 2.5 ×
D 2 2
C4
10–5 Joule
C123 and C4 are in series. So their equivalent is 37. (i) Energy stored in capacitor: Consider a capacitor
C123C4 2×2 of capacitance C connect to a battery of potential
C1234 = = = 1 µF ½
C123 + C4 2+2 difference V. Initially the two plates are uncharged.
Suppose +ve charge is transferred from plate 2 to
Now C5 and C1234 are is parallel, their equivalent
1 bit by bit. In this process work has to be done
capacitance C is
because plate 1 is always at higher potential than
C = C1234 + C5 = 1 + 1 = 2 µF ½ plate 2
Energy stored Let at any instant charge on plates be Q′ and –Q′ and
1 1 the PD between plates be V′.
U = CV2 = × 2 × 10–6 × (6)2
2 2 Now a small amount charge dQ′ is transferred from
–6
= 36 × 10 J = 36 mJ 1 plate 2 to plate 1.
34. Let C1 = 6 µF, C2 = 12 µF dW = V′dQ′
V1 = 2V, V2 = ? Q′  Q′ 
dW = dQ′    V′ = 
Charge on capacitor C1, C  C
Q1 = C1V1 Q′
Q
1
Q

= 6 µF × 2V = 12 µC 1½ W = ∫ dW = ∫ dQ ′ = ∫ Q ′dQ ′
0
C C0
As C1 and C2 are in series so
Q
Q1 = Q2. 1  Q′ 2  Q2
Q2 = 12 µC, C2 = 12 µF
=   =  1
C  2  0 2C
V2 = Q 2 = 12µC = 1 V 1½ Q2 1 1
C2 12 µF U == = CV 2 QV  1
2C 2 2
Total battery voltage = V1 + V2 = 2 + 1 = 3 V +Q′ –Q′
35. (i) C4 and C3 are in series
1 = 1 + 1 = 1 +1 (1) (2)

C34 C4 C3 12 4
V
fi C34 = 3 mF ½
C1 and C2 are in parallel
Energy density of electric field: When a capacitor is

C12 = C1 + C2 = 3 + 6 = 9 mF ½
charged electric field is set up in the region between
C12 and C34 are in series the plates i.e., the energy is stored in the form of
1 = 1 + 1 fi C = 9 mF ½ electrostatic potential energy
Cnet 3 9 net 4 1
U= CV2
(ii) (a) Charge on C3 and C4 will be 2
Q = CV Aε 0
For parallel plate capacitor C = and
d
fi Q = 9 × 8 = 18 × 10–6 C ½
4 σd
(b) Energy store V = Ed =
ε0
Q2 Q2 −12 −12
+ = 18 × 18 × 10−6 + 18 × 18 × 10−6 2
2C3 2C4 2 × 4 × 10 2 × 12 × 10 1  Aε 0   σd 
U=  
E = 54 mJ 1 2  d   ε 0 

76 Physics–12
D:\EG_Physics-12_(26-06-2022)\Open_Files\Ch-2\Ch-2
\ 27-Jul-2022 Ved_Goswami Proof-4 Reader’s Sign _______________________ Date __________

Energy stored in Capacitor: The energy of a


1 Aε 0 . σ 2 d 2 1 σ 2 d
= = A charged capacitor is stored in the form of electrical
2 d ε02 2 ε0 potential energy.
1 σ2d To find the energy stored in a capacitor, consider a
A capacitor of capacitance C, across which a potential
U 2 ε0 1 σ2 × ε0 1 difference V is applied between the plates.
= = = ε0 E 2  1
vol. Ad 2 ε0 × ε0 2 Let the charge on one plate be +q and on the other
–q.
or
Suppose at any stage ,the charge on the capacitor
1 1 Aε 0 1 is q.
U = CV 2 = (Ed)2 = Ae0E2d
2 2 d 2 q
Potential of capacitor V =
U 1 C
= e0E2
vol. 2 Small amount of work done in giving an additional
charge dq to the capacitor is
(ii) Before connecting them together
q
C1 = C, C2 = C, V1 = V, V2 = 0 dW = dq ,
C
1 1 1 Total work done in giving a charge Q to the capacitor
Ui = C1V12 + C2 V22 = CV2 ... (1) is
2 2 2
Q
q Q2
C V + C2 V2 W= ∫ C dq or W=1  1
Vcommon = 1 1 2 C
C1 + C2 0

Energy stored in the capacitor
CV + 0 V
= =  1 Q2
2C 2 U= W=1
2 C
1
Uf = C × (Vcom)2 But Q = CV \ U = 1 CV 2
2 net 2
2 Again put
1  V CV 2
= 2 × 2C ×  2  = Q = CV or U = 1 QV
4 2
. .
[ . Cnet = C + C = 2C]
Thus, U= = 1 Q 2 1= CV 2 1 QV  1
2 C 2 2
CV 2 CV 2
Uf – U i = – When Q is in coulomb, V is in Volt ,C is measured
4 2
in fared .then energy U is in joule.
CV 2 Q2 1
= – (–ve value) 1
(ii) Ei = 1 = CV 2
4 2 C 2
So Uf < Ui
As Uf – Ui is negative so energy stored in the
combination is less than the initial energy.
38. (i) When a Capacitor is connected to a circuit with direct
current (DC) source, two processes, “charging” and
“discharging “of the capacitor, takes place.
( ) ( )
2 2
When a capacitor is connected to the d c source CV CV
2 2
1 2 1 2
current flows through the circuit. Both Plates get Ef = + = CV
equal and opposite charges and an increasing 2 C 2 C 4C
potential difference, vc, is created while the Capacitor 1 2
Ef = CV  1
is charging. Once the Voltage at the terminals of the 4
Capacitor, vc, is equal to the supply voltage, vc = V, 1 CV 2
Ef
the Capacitor is fully charged and the current stops = 14 = 1 1
flowing through the circuit, the charging of capacitor Ei CV 2 2
is completed. 1 2

Electrostatic Potential and Capacitance 77


D:\EG_Physics-12_(26-06-2022)\Open_Files\Ch-2\Ch-2
\ 27-Jul-2022 Ved_Goswami Proof-4 Reader’s Sign _______________________ Date __________

Case Based Questions


I. Equipotential Surface: The surface at which 4. Two uniformly charged parallel plates having
potential at all points is same called equipotential densities + s and – s are placed in the XZ plane at a
surface. The work done across any two points on distance ʻdʼ apart, sketch of the equipotential surface
equipotential surface is always zero. The electric field due to electric field between the plates will be
is always normal to the equipotential surface.
Some equipotential surfaces are given below:
(a)
Equipotential

(b)
[due to a + ve point charge]

Properties:
(i) No work is done in moving a test charge over an
equipotential surface. (c)
(ii) Electric field is always normal to the equipotential
surface at every point.
(iii) Equipotential surface are closer together in the
region of strong field and father apart in the region
of weak field.
(d)
1. What is the shape of the equipotential surfaces for a
uniform electric field?
"
(a) Parallel to the direction of E
" 5. The separation between successive equipotential
(b) Anti-parallel to the direction of E surfaces get wider as the distance from the charges
"
(c) Perpendicular to the direction of E increases because
(d) Can have any direction 1
(a) dr µ E (b) dr µ E
2. What is the shape of the equipotential surfaces for an
1
isolated point charge? (c) dr µ 2 (d) dr µ E2
E
(a) square (b) rectangular
(c) oval (d) concentric spheres Ans.1. (c) 2. (d) 3. (b) 4. (a)
" 5. (b)
3. A uniform electric field E of 300 NC–1 is directed
II. A capacitor is a device to store energy. The process
along PQ. A, B and C are three points in the field
of charging a capacitor involves the transferring of
having x and y coordinates (in metres) as shown in
charges from its one plate to another. The work done
the figure. Potential difference between the points B
in charging the capacitor i.e. raising the potential
and C will be
of the capacitor, is stored in the form of electrical
potential energy. This energy is supplied by the
battery at the expense of its chemical energy. This
energy can be recovered by allowing the capacitor
to discharge.
2
=1 Q 1= 2 1
    U = 2 . C 2 . CV 2 QV
1. A parallel plate capacitor of capacitance C is charged
(a) 1500 V (b) – 2100 V to a potential V. It is then connected to another
(c) – 1100 V (d) 2000 V uncharged capacitor having same capacitance. The
78 Physics–12
D:\EG_Physics-12_(26-06-2022)\Open_Files\Ch-2\Ch-2
\ 27-Jul-2022 Ved_Goswami Proof-4 Reader’s Sign _______________________ Date __________

ratio of the energy stored in the combined system to 3. Net capacitance of three identical capacitors in
that stored initially in a single capacitor will be series is 1mF. The ratio of energy stored in two
1 1 1 1 configurations if they are both connected to same
(a) 4 (b) 2 (c) 5 (d) 6
source will be
2. Two parallel plate capacitors of capacitances C1 and
1 1 1 1
C2 such that C1 = 2C2 are connected across a battery (a) 2 (b) 3 (c) 4 (d) 9
of V volts as shown in the figure. Initially, the key 4. Energy (U) stored inside the capacitor be
(K) is kept closed to fully charge the capacitors. affected when it is completely filled with a
The key is now thrown open and a dielectric slab of dielectric material of dielectric constant ‘k’ as
dielectric constant k is inserted in the two capacitors k U
to completely fill the gap between the plates. The ratio (a) U (b) k i (c) kUi (d) k2Ui
i
of the energies stored in the combination before and 5. A capacitor of capacitance C is charged fully by
after the introduction of the dielectric slab will be connecting it to a battery of emf E 0. It is then
disconnected from the battery. If the separation
between the plates of the capacitor is now doubled,
Energy stored by the capacitor becomes
(a) 2 times (b) 4 times (c) 9 times (d) 7 times
Ans.1. (b) 2. (a) 3. (d) 4. (c)
5. (a)
1 1 2
(a) k (b) (c) 2k (d) 3k
k2

IMPORTANT FORMULAE
Formulae Symbols Application

DV = Potential difference
VA = Electric potential at A To find the potential difference using work
1. DV = V – V = WAB
A B q VB = Electric potential at B done from a point A to a point B
q = Charge

2. E = −d V dV/dr = dipole gradient Relation between electric field and potential


dr

3. V = k q r = distance Electric potential due to a point charge


r

4. V = k p cos θ p = dipole moment Electric potential due to dipole at any point


r2
C = Capacitance
5. C = 4pe0r Capacity of isolated spherical conductor
r = radius of conductor

A = area of plates
6. C = ε 0 A Capacitance of a parallel plate capacitor
d d = distance between the plates

Grouped capacitors:
(a) in series.
1 1 1 1 CS = equivalent capacitance in series
7. = + + To calculate equivalent capacitance of a circuit
Cs C1 C 2 C 3 CP = equivalent capacitance in parallel
(b) in Parallel.
CP = C1 + C2 + C3

Electrostatic Potential and Capacitance 79


D:\EG_Physics-12_(26-06-2022)\Open_Files\Ch-2\Ch-2
\ 27-Jul-2022 Ved_Goswami Proof-4 Reader’s Sign _______________________ Date __________

1 Q2 1 1 U = E lectrostatic energy stored in


8. U= = CV 2 = QV Energy stored in a capacitor
2 C 2 2 capacitor

q1q2 U = Potential Energy Potential energy of a system of two point


9. = k
= W
U
r12 W = Work done charges

1 E = Electric field strength


10. u= ε0 E 2 Energy density of a parallel plate capacitor
u = Energy density
2

C0 t = thickness of slab
C= Capacitance of parallel plate capacitor with
11.  t d = distance between the plates
1 −  conducting slab in between
 d C0 = capacitance

ε0 A
C= K = dielectric constant Capacitance of parallel plate capacitor with
12.  1
d − t 1 −  dielectric slab in between
 K

COMMON ERRORS
S No. Errors Corrections
1. Student can’t differentiate b/w electric potential of a Electric potential of a dipole depends on distance and
dipole and a single charge angle, while potential due to a single charge depends
only on distance.
2. Students get confused b/w the terms electric potential Electric potential is a scalar quantity, while potential
and electric potential gradient. gradient is a vector quantity.
3. Student can’t differentiate b/w the graphs In graph
V q V q
A A A A

q V q V
and
V 1
= Slope =
q capacitance
While in graph
q
Slope = = capacitance
V
4. Effect of dielectric in capacitors Proper polarity for dielectric slab
5. Variation of charge potential, energy, field, While connected p.d is constant and while disconnected
capacitance etc. on connected and disconnected with charge is constant.
battery condition.
6. Numeric Problems based on combination of Three ways of solving networks:
capacitors 1. Solve from one end of the circuit if possible.
2. re-draw the circuit in a simpler way
3. Apply wheatstone’s principle if applicable

80 Physics–12
D:\EG_Physics-12_(26-06-2022)\Open_Files\Ch-2\Ch-2
\ 27-Jul-2022 Ved_Goswami Proof-4 Reader’s Sign _______________________ Date __________

REVISION CHART

Electric potential due to a group of


Electric potential (V) at a point due point charges: Electric potential at a Equipotential surface: The
to a point charge q: point due to a group of point charges surface, at every point of
q is equal to the algebraic sum of the which, the electric potential is
V= electric potentials due to individual same, is called equipotential
4πε 0 r charges at that point. surface.

Electrostatic Potential
Electric potential difference between two points in an electric field is defined as the amount of work done, in moving a unit
positive charge, from one point to another, without accelerating it. S.I. unit of potential difference is volt and it is a scalar quantity.

Potential gradient: The rate of change of potential with distance Electric potential due to a dipole:
at a point is called potential gradient. (i) at an axial point
→ −dV Kp
E= V=
r 2 − a2
dr
Potential Gradient is a vector quantity. for r >> a
Electric field is in the direction of decreasing potential. Kp
     V =
SI unit of potential gradient is Vm–1. r2
(ii) on equator V = 0

Capacitance of a parallel plate


Capacitance of a parallel plate capacitor with di-electric slab of
capacitor with vacuum separating thickness Capacitor: Capacitor is an assembly
the plates t<d of two conductors separated by an
Aε 0 Aε 0
C= insulating medium and it is used to
d C=  t t 
d 1 − +  store electric energy by accumulating
SI unit of capacitance is farad  d dk  charge on the conductors.

Capacitance
The electrical capacitance of a conductor is the measure of it’s ability to hold electric charge. The capacitance is defined as the
charge required to increase the potential of the conductor by unit amount.

Capacitance of a parallel plate Energy stored in a capacitor Capacitance of a spherical conductor


capacitor with di-electric separating 1 Q2 1 1 = 4pe0r, r is radius of the sphere.
=U = CV 2 = QV
the plates 2 C 2 2
Aε 0 k
C=
d
k is dielectric constant

Electrostatic Potential and Capacitance 81


D:\EG_Physics-12_(26-06-2022)\Open_Files\Ch-2\Ch-2
\ 27-Jul-2022 Ved_Goswami Proof-4 Reader’s Sign _______________________ Date __________

IMPORTANCE OF EACH TOPIC AND FREQUENTLY ASKED TYPES OF QUESTIONS

☞ Important Topics
1. Questions based on series and parallel combination of the capacitor. 2. Questions based on Electric Potential of a dipole.
3. Questions based on equipotential. 4. Energy stored in a capacitor.

1. In the following fig. calculate the potential difference across capacitor C2. Given potential at A is 90 V. C1 = 20
µF, C2 = 30 µF, and C3 = 15 µF. [Ans. 20V]

A
C1 C2 C3

2. Figure shows three circuits, each consisting of a switch and two capacitors initially charged as indicated. After the
switch has been closed, in which circuit (if any) will the charges on the left hand capacitor (i) increase (ii) decrease
(iii) remain same?
S S S

+ + 6Q + + 3Q + +
6Q 2C C 3Q 3C C 2C 2C
6Q 3Q

(a) (b) (c)

[Ans. (a) remains unchanged, (b) increases, (c) decreases]


3. For what value of C does the equivalent capacitance between A and B is 1 µF in the given circuit?
C 3
A

3
3 3
4 2
B
3 3

All capacitance given in micro Farad (µF)


[Ans. 2 µF]
4. Two metal spheres A and B of radius r and 2r whose centres are separated by a distance of 6r are given charge Q,
are at potential V1 and V2 . Find the ratio of V1 / V2. These spheres are connected to each other with the help of a
connecting wire keeping the separation unchanged, what is the amount of charge that will flow through the wire?
V1 7 q
[Ans. V = 4 , ]
2 3

A B

6r

5. Eight charged water droplets each of radius 1 mm and charge 10 × 10–10C coalesce to form a single drop. Calculate
the potential of the bigger drop. [Ans. 3600 V]
6. Three concentric spherical metallic shells A < B < C of radii a, b, c (a < b < c) have surface densities s, – s and s
respectively. Find (i) the potential of three shells A, B and C. (ii) If shells A and C are at the same potential, obtain
relation between a, b, c.
σ σ
[Ans. (i) VA =
ε0
(a – b + c), VB =
ε0 b
(
a2 − b2 + c2 , V =
σ 2) 2 2
C ε c (a – b + c ), (ii) c = a + b]
0

82 Physics–12
D:\EG_Physics-12_(26-06-2022)\Open_Files\Ch-2\Ch-2
\ 27-Jul-2022 Ved_Goswami Proof-4 Reader’s Sign _______________________ Date __________

7. Find the potential at A and C in the following circuit:


12V

B
A C
1 mF 5 mF

[Ans. VAB = 10 V, VBC = 2V, When B is earthed VB = 0, VA = 10V and VC = –2V]


8. Two identical parallel plate capacitors connected to a battery with the switch S closed. The S
switch is now opened and the free space between the plates of the capacitors is filled with
dielectric of dielectric constant 3, Find the ratio of the total electrostatic energy stored in E A B
both capacitors before and after the introduction of dielectric.
El 5
[Ans. = ]
E 3
9. Eight charged droplets each of radius 1 mm and charge 10 × 10–10 C coalesce to form a single drop. Calculate the
potential of the bigger drop. [Ans. 3600 V]
10. What potential difference must be applied to produce an electric field that can accelerate an electron to 1/10 of
velocity of light? [Ans. 2.6 × 103 V]
11. A 10 µF capacitor can withstand a maximum voltage of 100 V across it, whereas another 20 µF capacitor can
withstand a maximum voltage of only 25 V. What is the maximum voltage that can be put across their series
combination? [Ans. 75 V]
12. Three concentric spherical metallic shells A < B < C of radii a, b, c (a < b < c) have surface densities s, – s and
s respectively. Find the potential of three shells A, B and C. If shells A and C are at the same potential, obtain
relation between a, b, c.[Ans. c = a + b]
13. Four point charges are placed at the corners of the square of edge a as shown in the figure. Find the work done in
Kq2
disassembling the system of charges. Ans. a ( 2 −4 J )
–q +q

+q –q

Electrostatic Potential and Capacitance 83


D:\EG_Physics-12_(26-06-2022)\Open_Files\Ch-2\Ch-2
\ 27-Jul-2022 Ved_Goswami Proof-4 Reader’s Sign _______________________ Date __________

ASSIGNMENT
I. Objective Type Questions (1 Mark)
1. Multiple choice questions:
(i) Consider a uniform electric field in the z direction. The potential is a constant
(a) in all space. (b) for any x for a given z.
(c) for any y for a given z. (d) on the x-y plane for a given z.
(ii) The dielectric strength of air at N.T.P. is 3 × 106 V/m. Then the maximum charge that can be given to a spherical
conductor of radius 3 m is:
(a) 3 × 10–1 C (b) 3 × 102 C (c) 3 × 10–3 C (d) 3 × 10–4 C
2. Fill in the blanks:
(i) Two equipotential surfaces cannot intersect each other because .......... .
(ii) Electric field inside a conductor is .......... .
II. Very Short Answer Type Questions (1 Mark)
3. Why does the electric field inside a dielectric decreases, when it is placed in an external electric field?
4. Four capacitors are connected as shown in the figure. Calculate the equivalent capacitance between the points X
and Y. 
B
X Y
A 2µF 3µF 5µF 10µF

III. Short Answer Type Question-I (2 Marks)


5. (a) Why do two equipotential surfaces not intersect each other?
(b) Draw equipotential surfaces for a system of two equal and opposite charges separated by a small distance.
IV. Short Answer Type Questions-II (3 Marks)
6. Show that electric field intensity is negative of potential gradient.
 
7. Deduce the expression for potential energy of a system of two charges q1 and q2 placed at r1 and r2 respectively
in an external electric field E.
8. Two parallel plate capacitors, X and Y, have the same area of plates and same separation X Y
between them. X has air between the plates while Y contains a dielectric medium of er
= 4.
(i) Calculate capacitance of each capacitor if equivalent capacitance of the combination + –
is 4mF. 15V
(ii) Calculate the potential difference between the plates of X and Y.
(iii) What is the ratio of electrostatic energy stored in X and Y?
9. Find the net capacitance of the capacitor shown below. 
A/2 A/2

k1 k2

84 Physics–12
D:\EG_Physics-12_(26-06-2022)\Open_Files\Ch-3\Ch-3
\ 27-Jul-2022 Ved_Goswami Proof-3 Reader’s Sign _______________________ Date __________

Topics Covered
3 Current Electricity

3.1 Electric Current 3.2 Cells, EMF and Internal Resistance


3.3 Kirchhoff’s Laws and Wheatstone Bridge

C hapter map
Current Electricity

Free electrons in a Electric Charges Cells, EMF, Internal


conductor (+ve, –ve) Resistance (r)

drift velocity
Absence of Presence of 
 −eEτ
electric field electric field vd =
m

v (average) Kirchhoff’s Electric Wheatstone


=0 Law Current Bridge non linear devices
(R varies)
electric energy & power linear & non
V2t linear devices
H = I2Rt = VIt = Ohm’s Law linear devices
R
V = IR (R = constant)
V2
P = I2R = VI = combination
R Resistors
(series, parallel)

Parallel
Series
Factors affecting resistance 1 1 1 1
RS = R 1 + R 2 + R 3 = + +
RP R1 R 2 R3
rl
R= A
graphical variation of r • temperature
with temperature • length (directly)
• area of Cross Section (inversely)
• nature of the material (r)
conductors

Temperature
semi
co-efficient of
conductor
resistance
Rt = R0 (1 + at)
alloys

85
D:\EG_Physics-12_(26-06-2022)\Open_Files\Ch-3\Ch-3
\ 27-Jul-2022 Ved_Goswami Proof-4 Reader’s Sign _______________________ Date __________

Topic 1. Electric Current


•• It is the rate of flow of electric charge with time through Ohm’s Law
any section of the conductor •• The current (I) flowing through a conductor is directly
dq proportional to the potential difference (V) across its two
I = dt
ends provided all physical conditions remain unchanged.
SI unit of electric current is ampere (A)
V ∝ I or V = IR
1A = 1Cs– 1
R is resistance of the conductor.
Flow of electric charges in a metallic conductor S.I. unit of resistance is ohm or W
•• The flow of electric charges through a conductor 1 W = V/A
constitutes an electric current (I). Quantitatively, electric •• Resistivity (r):
current in a conductor across an area held perpendicular Resistance (R) of a conductor is given
to the direction of flow of charge is defined as the amount V ml
of charge flowing across that area per unit time. R= I = 2 ...(1)
ne τA
If a charge dq passes through an area in time t to t + dt, So, the resistance of a conductor depends upon the length
then the current I at time t is given by ‘l’ and area of cross-section (A), the conductor
δq dq R∝l
I = lim δt = dt
δt → 0 R∝ 1
A
Drift Velocity
rl
•• It is the average velocity with which free electrons in the R= ...(2)
A
conductor gets drifted under the influence of the applied
m
external electric field. Comparing eqn. (1) and (2) we get r =
ne 2τ
eEτ If l = 1m, a = 1m2, then R = r
Drift velocity = Vd = m
Thus resistivity of a material is the resistance offered
t is relaxation time, m is mass of an electron, E is applied
by a wire of this material of unit length and unit area of
electric field, e is charge on an electron.
cross-section. It is also known as specific resistance of
Relation between electric current and drift velocity the material
I = neAVd S.I. unit of resistivity is Wm (Ohm metre)
n is no. of free electron per unit volume, A is area of Resistivity is the characteristic of a material, its value
cross-section of the conductor. depends upon the temperature.
•• Conductance : The reciprocal of the resistance of a
Mobility and their relation with electric current conductor is called its conductance (G)
•• Mobility: The conductivity of any material is due to its S.I. unit of conductance is ohm– 1
mobile charge carriers. These may be electrons in metals,
Also written as W– 1 or Siemen written as S
positive and negative ions in electrolytes; electrons and
holes in semiconductors. ohm–1 is also written as mho
The mobility of a charge carrier is the drift velocity •• Conductivity: The reciprocal of the resistivity of a material
acquired by it in a unit electric field. It is given by is called its conductivity (s)
v 1
Thus, s= r
      µ = Ed
S.I. unit of conductivity is ohm–1 m– 1 or (W– 1m– 1)
SI unit of mobility = m2V–1s–1
or Siemen metre– 1 written as Sm– 1 or mho metre– 1
•• Relation between electric current and mobility for a 
conductor: In a metallic conductor, the electric current •• Current density ( J ): It is the current per unit area of cross
is due to its free electrons and is given by section of the conductor.
  
I = enAvd         J = I or J = − neVd
A
But vd = µeE \ I = enA µeE it is a vector quantity. It’s S.I. unit is A/m2

86 Physics–12
D:\EG_Physics-12_(26-06-2022)\Open_Files\Ch-3\Ch-3
\ 27-Jul-2022 Ved_Goswami Proof-4 Reader’s Sign _______________________ Date __________

Relation between current density, resistivity and •• Electrolytes: As the temperature increases the viscous
electric field or ohm’s law in terms of current density force and the inter-ionic attraction in the electrolyte
and electric field.  decreases. So the ions move more freely i.e. the drift
 − eEτ velocity of current carriers (which are ions in case of
Vd = m
electrolytes) increases and the relaxation time increases. As

 − eEτ  1
 ρ ∝ so resistivity decreases with increase in temperature.
J = − neVd = − en  m  τ
The temperature co-efficient of resistivity is negative.
 
 ne 2τE E 
J = m = ρ = σE •• Alloys: Resistance of an alloy does not change much
with change in temperature.
Temperature coefficient of resistance (a) ρ Vs T for alloys
It is defined as the fractional change in resistance for 1°C
change in temperature.
Rt − Ro ρ
a= R ot
Rt − Ro
If t = 1°C, a =
Ro
T
S.I. Unit of temperature co-efficient of resistance is °C–1.
Grouping of resistance: (1) Series grouping (2) Parallel
•• Temperature dependence of resistivity:
grouping.
m
Conductors : As r = ne2τ
Series Grouping
As temperature increase, the number density of free •• The figure shows three resistors of resistance R1, R2 and
electrons (n) does not change much,but the thermal R3 connected in series. In series grouping:
speed of electrons and amplitude of vibration of the
(a) Current through each resistor is same.
positive ions increases. Thus the collisions becomes
more frequent, thereby reducing the relaxation time t. (b) The total resistance of the circuit is equal to the sum
So resistivity increases with increase in temperature. of individual resistances,
ρ Vs T for conductors i.e.,     R = R1 + R2 + ..... Rn
R1 R2 R3

ρ
I

T(K) V
0 50 100 150 (c) Potential difference across any resistor is proportional
Temperature co-efficient of resistivity is positive. to its resistance.
•• Semiconductors: With increase in temperature, the •• Parallel grouping of resistors: The figure shows three
number density of free electrons increases exponentially resistors of resistance R1, R2 and R3 connected in parallel.
and time of relaxation t decreases slightly. The increase In parallel grouping:
in n compensates more than decrease in relaxation time.
(a) There is division of current.
So resistivity decreases with increase in temperature R1
I1
ρ Vs T for semiconductors
I2 R2

I3 R3
ρ I I

V
(b) Potential difference across all resistors is same.
T (c) Current through any resistor is inversely proportional
Temperature co-efficient of resistivity is negative. to its resistance.

Current Electricity 87
D:\EG_Physics-12_(26-06-2022)\Open_Files\Ch-3\Ch-3
\ 27-Jul-2022 Ved_Goswami Proof-4 Reader’s Sign _______________________ Date __________

or the reciprocal of the equivalent resistance of the R1 R2 R3


combination is equal to the sum of the reciprocal of
individual resistances.
1 1 1 1 I
or   R = R + R + ....... + R
1 2 n

•• Ohmic Conductor : The conductors which obey ohm’s V


law, i.e., the ratio of potential difference across its end, V2 across R2 and V3 across R3
to the current flowing through it remains constant are V = V1 + V2 + V3
called ohmic conductors.
= IR1 + IR2 + IR3
or V = const.

I = I(R1 + R2 + R3) ...(1)


V = IReq ...(2)
From (1) and (2) IReq = I(R1 + R2 + R3)
\ Req = R1 + R2 + R3
Current

•• Resistance in Parallel: Consider 3 resistors R1, R2 and


P.D.
R3 connected in parallel voltage across each = V
•• Non-ohmic Conductors : The conductors which do not Let current flowing through R1 be I1, I2 through R2 and
obey ohm’s law are called non-ohmic conductors i.e., for I3 through R3
non-ohmic situations may be of following types: R1
I1
(a) V vs I graph is non linear.
R2
(b) Relation between potential difference and current I2
depends on the sign of V for the same absolute value I3 R3
of V. I I
V
(c) Ratio I is not constant. V
For example: V-I graph for metals at high temperature V V V
I = I1 + I2 + I3 = R + R + R
fig. (i) and V-I graph for a junction diode fig. (ii). 1 2 3

I
 
= V 1 + 1 + 1   ...(1)
 R1 R 2 R 3 
V
–V V I= R  ... (2)
V eq

From (1) and (2), we have


I
–I V  1 1 1 
R eq = V  R + R + R 
Fig. (i)          Fig. (ii) 1 2 3

•• Resistance in Series: Consider 3 resistor of resistances


1 1 1 1
R1, R2 and R3 connected in series. Current through each ⇒ = + +
= I. Let p.d. V1 be across R1, R eq R1 R 2 R 3

EXERCISE 3.1
I. Objective Type Questions (1 Mark) (current density) changes in an exact manner,
1. Choose the correct answers from the given options while the current I remains unaffected. The agent
(MCQs). that is essentially responsible for it is:
(i) Consider a current carrying wire (current I), in (a) source of emf.
the shape of a circle. Note that as the current (b) electric field produced by the charges
progresses along the wire, the direction of J accumulated on the surfaces of wire.

88 Physics–12
D:\EG_Physics-12_(26-06-2022)\Open_Files\Ch-3\Ch-3
\ 27-Jul-2022 Ved_Goswami Proof-4 Reader’s Sign _______________________ Date __________

(c) the charges just behind a given segment of (vii) Resistivity of a given conductor depends upon
the wire which push them just the right way  [AI 2020]
by repulsion. (a) temperature
(d) the charges ahead. (b) length of conductor
(c) area of cross-section
(ii) A metal rod of length 10 cm and a rectangular
(d) shape of the conductor
cross-section of 1 cm × 1 cm is connected to a (viii) The ratio of current density and electric field is
2
battery across opposite faces. The resistance will called [AI 2020]
be (a) Resistivity (b) Conductivity
(a) maximum when the battery is connected (c) Drift velocity (d) Mobility
(ix) The conductivity of a metal decreases with the
across 1 cm × 1 cm faces.
2 increase in temperature on account of
(b) maximum when the battery is connected (a) decrease in number density of electrons
across 10 cm × 1 cm faces. (b) decrease in resistivity
(c) maximum when the battery is connected (c) decrease in relaxation time
across 10 cm × 1 cm faces. (d) increase in mean free path [AI 2020]
2 (x) Two unequal resistors are connected in series
(d) same irrespective of the three faces.
across a battery. Then the [AI 2020]
(iii) The temperature dependence of resistivity r(T)
(a) potential difference across each resistor is the
of semiconductors, insulators and metals is
same
significantly based on the following factors:
(b) current in the smaller resistor is larger
(a) Number of charge carriers can change with (c) potential difference across the bigger resistor
temperature T. is greater
(b) Time-interval between two successive (d) power dissipated in both resistors is the same
collisions can depend on T. (xi) Which of the following has negative temperature
(c) Length of material can be a function of T. coefficient of resistivity? [CBSE 2022]
(d) Mass of carriers is a function of T. (a) metal (b) metal and semiconductor
(iv) If the ammeter in the given circuit shown in the (c) semiconductor (d) metal and alloy
diagram reads 2A, the resistance R is (xii) If n, e, t and m have their usual meanings, then the
[CBSE S.P. 2019-20] resistance of a wire of length l and cross-sectional
area A is given by: [CBSE 2022]
2Ω R
ne 2 A ml
(a) (b)
– + 6V 2mτl ne 2 τA
A
mτA ne 2 τA
(c) (d)
(a) 1W (b) 2W (c) 3W (d) 4W ne 2l 2ml
(v) Two resistors R1 and R2 of 4 W and 6 W are (xiii) The equivalent resistance between A and B of the
connected in parallel across a battery. The ratio network shown in figure is [CBSE 2022]
of power dissipated in them, P1 : P2 will be R
A
 [Delhi 2020]
(a) 4 : 9 (b) 3 : 2 (c) 9 : 4 (d) 2 : 3
(vi) The resistance of a metal wire increases with R
increasing temperature on account of R
(a) decrease in free electron density B

(b) decrease in relaxation time  3


(a) 3R W (b)   R W
(c) increase in mean free path  2
(d) increase in the mass of electron  2
(c) 2R W (d)   R W
 [Delhi 2020]  3

Current Electricity 89
D:\EG_Physics-12_(26-06-2022)\Open_Files\Ch-3\Ch-3
\ 27-Jul-2022 Ved_Goswami Proof-4 Reader’s Sign _______________________ Date __________

2. Fill in the blanks. Reason (R): Drift velocity (vd) of electrons is so


(i) The drift velocity of electron is of the order of large that electrons exit from one end of conductor
.......... while the thermal average velocity of the can travel to the other end almost at once.
electron at room temperature is .......... . II. Very Short Answer Type Questions (1 Mark)
(ii) Manganin has a resistance which is .......... times 1. V-I graph for a metallic wire at two different
as compared to pure copper and its temperature temperatures T1 and T2 is as shown in the figure.
coefficient is .......... °C–1. Which of the two temperatures is higher and why?
(iii) The equivalent resistance of a network .......... due (AI 2015)
to parallel grouping of resistances and becomes
.......... .
T1
(iv) A copper wire of non-uniform area of cross-
section is connected to a d.c. battery. The physical
quantity which remains constant along the wire is V T2

.......... . [Delhi 2020]


3. State True or False
(i) For ‘n’ identical resistance, the ratio of the total I

resistances of series combination to the total 2. I-V graph for two identical conductors of different
R materials A and B is shown in the figure. Which one
resistance of parallel combination is s = n 2.
Rp of the two has higher resistivity? [AI 2015]
(ii) Nichrome is an alloy of Ni, Cr and Cu.
(iii) For insulators, the resistance decrease with the A
increase in temperature.
4. Match the Columns
Column-I Column-II B
(Physical Quantity) (SI Unit) I
(i) Electric current (a) W–1 m–1
(ii) Resistivity (b) Ampere (A)
A
(iii) Conductivity (c) 2
m 0 V
(iv) Current density (d) W m (ohm metre)
3. A conductor is carrying current. Is it charged?
For questions numbers 5 and 6 two statements are given-
one labelled Assertion (A) and the other labelled Reason 4. A steady current flows in a metallic conductor of
(R). Select the correct answer to these questions from non-uniform cross-section. Which of these quantities
the codes (a), (b), (c) and (d) as given below. is constant along the conductor: Current, current
(a) Both A and R are true and R is the correct density, electric field, drift velocity? [NCERT]
explanation of A 5. Is ohm’s law universally applicable for all conducting
(b) Both A and R are true but R is NOT the correct elements? [NCERT]
explanation of A 6. A low voltage supply from which one needs high
(c) A is true but R is false currents must have very low internal resistance. Give
reason. [NCERT]
(d) A is false and R is also false
7. If the electron drift is so small, and electron’s charge
5. Assertion (A): An electric bulb becomes dim, when
is small, how can we still obtain large amount of
electric heater in parallel circuit is switched on.
current in a conductor? [Delhi 2015]
Reason (R): Dimness decreases after sometime.
8. From the variation of resistance of a metallic wire with
6. Assertion (A): When a source of emf is attached temperature given below, calculate the temperature
across a conductor, current is immediately established co-efficient of resistance. [Delhi 2010 C]
in the conductor.
90 Physics–12
D:\EG_Physics-12_(26-06-2022)\Open_Files\Ch-3\Ch-3
\ 27-Jul-2022 Ved_Goswami Proof-4 Reader’s Sign _______________________ Date __________

B 22. Resistivity of copper, constantan and silver are 1.7


C
× 10–8 Wm, 39.1 × 10– 8 Wm and 1.6 × 10–8 Wm,
respectively. Which one has the best conductivity?

Resistance
23. If a wire is stretched to double its original length
A D without loss of mass, how will the resistivity of the
wire be influenced?
24. Two wires of equal lengths, one of copper and the
O
Temp.
E other of manganin have the same resistance. Which
wire will be thicker? [AI 2012]
9. When electrons drift in a metal from lower to higher
potential, does it mean that all the free electrons of 25. Two wires A and B are of same metal, have same area
the metal are moving is the same direction? of cross-section and have their lengths in the ratio
2 : 1. What will be the ratio of current flowing through
 [Delhi 2012] them respectively when the same potential difference
10. How does the drift velocity of electrons in a metallic is applied across the length of each of them?
conductor vary with the increase in temperature? 26. Are the paths of electrons straight line between
11. If the temperature of a good conductor increase how successive collisions (with the positive ions of the
does the relaxation time of electrons in the conductor metal) in the (i) absence of electric field, (ii) presence
change? of electric field?
12. Define temperature coefficient of resistivity. 27. Plot a graph, showing variation of resistance of a
[CBSE S.P. 2018-19] conducting wire as a function of its radius, keeping
the length of the wire and its temperature as
13. How does the mobility of electrons in a conductor
constant. [Foreign 2013]
change, if the potential difference applied across
the conductor is doubled, keeping the length and 28. Give an example of material each for which
temperature of the conductor constant?[Delhi 2019] temperature coefficient of resistivity is: (i) positive,
(ii) negative. [S.P. 2015]
III. Short Answer Type Questions-I (2 Marks)
29. Define the term conductivity of a conductor. On what
14. A potential difference V is applied across a conductor factors does it depend? [Delhi 2008, AI 2016]
of length l. How is the drift velocity affected when V
30. Graph showing the variation of current versus voltage
is doubled and l is halved? [Foreign 2010]
for a material GaAs is shown in the figure. Identify
15. Two conducting wires X and Y of same diameter but the region of: (i) negative resistance (ii) where Ohm’s
different materials are joined in series across a battery. law is obeyed. [AI 2013]
If the number density of electrons in X is twice that
in Y, find the ratio of drift velocity of electrons in the B
two wires. [AI 2010]
Current (mA)

C
16. Out of metals and alloys, which have greater value
of temperature co-efficient of resistance? A
17. Write two special characteristics of manganin due to
which, it is used in making, standard resistance. 0 Voltage (V)
18. Draw a graph showing variation of resistivity 31. Using concept of drift velocity of charge carrier in a
with temperature for nichrome. Which property of conductor, deduce the relationship between current
nichrome is used to make standard resistance coil. density and resistivity of the conductor.[Delhi 2015]
[AI 2013 C] 32. A 4W non insulated resistance wire is bent in the
19. Explain ‘Temperature dependence of resistivity’ for middle by 180° and both the halves are twisted with
conductor. each other. What will be its new resistance?
20. Why does the conductivity of a semiconductor [Delhi 2010 C]
increases with increase of temperature? 33. Figure below represents a part of a closed circuit. Find
21. Two materials Si and Cu are cooled from 300K to the potential difference between the points A and B.
60K. What will be the effect on their resistivity? 1A 4Ω 3V 2Ω
A B
[Foreign 2013]

Current Electricity 91
D:\EG_Physics-12_(26-06-2022)\Open_Files\Ch-3\Ch-3
\ 27-Jul-2022 Ved_Goswami Proof-4 Reader’s Sign _______________________ Date __________

34. When 5V potential difference is applied across a wire (c) increase in length
of length 0.1m, the drift speed of electrons is 2.5 × (d) increase in area of cross-section
10–4 m/s. If the electron density in the wire is 8 × 1028 (Assume that the other factors remains constant in
m–3, calculate the resistivity of the material of wire. each case). [S.P. 2011]
[AI 2016]
41. (i) Derive an expression for drift velocity of free
35. A network of resistors is connected to a 16 V battery electrons. Hence, deduce Ohm’s law.
with internal resistance of 1W., as shown in the
[Delhi 2017]
following figure. Compute the equivalent resistance
of the network. [CBSE S.P. 2018-19] (ii) A wire whose cross-sectional area is increasing
linearly from its one end to the other, is connected
across a battery of V volts. Which of the following
quantities remain constant in the wire?
(a) drift speed (b) current density (c) electric
current (d) electric field
Justify your answer. [Delhi 2017]
42. Define the electric resistivity of a conductor.
36. Derive the expression for drift velocity of free electron Plot a graph showing the variation of resistivity
in terms of relaxation time and electric field applied with temperature in the case of a (a) conductor,
across a conductor. [CBSE S.P. 2019-20] (b) semiconductor.
37. Define the term ‘mobility’ of charge carriers in a Briefly explain, how the difference in the behaviour of
current carrying conductor. Obtain the relation for the two can be explained in terms of number density
mobility in terms of relaxation time. [AI 2020] of charge carriers and relaxation time. [AI 2015]
OR 43. Define relaxation time of the free electrons drifting
Define the term ‘drift velocity’ of electrons in a current in a conductor. How is it related to the drift velocity
carrying conductor. Obtain the relationship between of free electrons? Use this relation to deduce
the current density and the drift velocity of electrons. the expression for the electrical resistivity of the
 [AI 2020] material. [AI 2012]
IV. Short Answer Type Questions-II (3 Marks) 44. (i) Define the term ‘conductivity’ of a metallic wire.
38. A conductor of length L is connected to dc source Write its SI unit.
of emf E. If this conductor is replaced by another (ii) Using the concept of free electrons in a conductor,
conductor of same material and same area of cross- derive the expression for the conductivity of a wire
section but the length 3L, how will the drift velocity in terms of number density and relaxation time.
change? [Delhi 2013] Hence obtain the relation between current density
and the applied electric field E. [Delhi 2018]
39. Plot a graph showing the variation of current density
(J) versus the electric field (E) for two conductors of 45. (i) Differentiate between electrical resistance and
different materials. What information from this plot resistivity of a conductor.
regarding the properties of the conducting material, (ii) Two metallic rods, each of length L, area of
can be obtained which can be used to select suitable cross A1 and A2, having resistivities r1 and r2 are
materials for use in making: (i) standard resistance connected in parallel across a d.c. battery. Obtain
and (ii) connecting wires in electric circuit? expression for the effective resistivity of this
combination. [Delhi 2020]
[Delhi 2015]
V. Long Answer Type Questions (5 Marks)
40. Is current density a scalar or a vector quantity? Deduce
the relation between current density and potential 46. (a) A 16W resistance wire is bent to form a square. A
difference across a current carrying conductor of source of emf 9V is connected across one of its
length l, area of cross-section A, and number density sides as shown. Calculate the current drawn from
of free electrons n. How does the current density, in the source. Find the potential difference between
a conductor vary with: the ends C and D.
(b) If now the wire is stretched uniformly to double the
(a) increase in pot. gradient
length and once again the same cell is connected
(b) increase in temperature in the same way, across one side of the square

92 Physics–12
D:\EG_Physics-12_(26-06-2022)\Open_Files\Ch-3\Ch-3
\ 27-Jul-2022 Ved_Goswami Proof-4 Reader’s Sign _______________________ Date __________

formed, what will now be the potential difference 48. Find the equivalent resistance between the points A
across one of its diagonals? [AI 2015] and B.
D C
R
5Ω

R R
10Ω
5Ω

A B
10Ω
R 5Ω
A B

5Ω
49. A voltmeter V of resistance 400W is used to measure
the potential difference across a 100W resistor in the
9V
circuit given below:
47. Find the equivalent resistance between the points A
and B. (a) What will be the reading of the voltmeter?
5Ω (b) Calculate the p.d. across 100W resistor before the
voltmeter is connected.
84V
10Ω
10Ω

A C B
10Ω 100Ω 200Ω
10Ω

V
5Ω

Answers 3.1
I. Objective Type Questions As the conductors are identical so their l and A are equal.
1. (i) (b) (ii) (a) (iii) (a) and (b) A conductor with higher value of resistance R will have
(iv) (a) (v) (b) (vi) (b) (vii) (a) higher resistivity.
1
(viii) (a) (ix) (c) (x) (c) (xi) (c) Resistance = slope
(xii) (b) (xiii) (c) (Slope)A > (Slope)B
2. (i) 10 m/s, 105 to 106 m/s (ii) 30 to 40, 10–5
–5
RB > RA
(iii) decrease; lesser than the smallest resistance in the  ρl 
So, Resistivity of B > Resistivity of A  R = A  .
circuit.  
(iv) electric current. 3. No, when a potential difference is applied across a
3. (i) True (ii) True (iii) False conductor, the electrons start drifting but the number of
protons at any instant is always equal to the number of
4. (i)-(b) (ii)-(d) (iii)-(a) (iv)-(c)
electrons. Hence the net charge on the conductor is zero.
5. (b) 6. (c)
4. Current will remain constant.
II. Very Short Answer Type Questions 5. No, ohm’s law is not valid for semiconductor diode,
1. For metals, resistance increases with increase in liquid electrolytes.
temperature
6. As Imax = E , r is internal resistance.
Slope of the graph = Resistance r
Slope for temp. T1 > slope for temp. T2 So r should be very low for maximum I.
So T1 > T2. 7. As there are large number of free electrons in a
conductor so it is possible to have large current in spite
2. As resistivity ρ = RA
l of low drift velocity and charge on electrons.

Current Electricity 93
D:\EG_Physics-12_(26-06-2022)\Open_Files\Ch-3\Ch-3
\ 27-Jul-2022 Ved_Goswami Proof-4 Reader’s Sign _______________________ Date __________

8. Temperature co-efficient of resistance m


19. Conductors : As r = ne2τ
Change in resistance
a=
Original resistance × (temp. change) As temperature increase, the number density of free
electrons (n) does not change much,but the thermal
BD OC − OA speed of electrons and amplitude of vibration of the
= =
OA × AD OA × OE positive ions increases. Thus the collisions becomes
9. No, the drift velocity of the electrons are superposed more frequent, thereby reducing the relaxation time t.
over their random thermal motion. So resistivity increases with increase in temperature.
10. The drift velocity decreases with increase in temperature, ρ Vs T for conductors
because of the increase in collision frequency of free
electrons at higher temperature.
11. As the temperature increases, the electrons collide more ρ
frequently with positive ions. Thus the relaxation time
decreases.
12. Fractional change in resistivity per unit change in T(K)
0 50 100 150
temperature. 
[CBSE Marking Scheme, 2018-19] Temperature co-efficient of resistivity is positive.
ml ne 2τA
13. If the potential difference is doubled at a constant 20. ρ = ne2τA or σ = ml  1
temperature, mobility remains unchanged because As temperature increases, more and more electrons
mobility (m) is independent of applied potential become free to conduct electricity. So the conductivity
difference. increases. 1
eEτ eVτ 21. As Si is a semiconductor, its resistivity will increase
14. Drift velocity = vd = m = ml  1
with decrease in temperature. 1
e2Vτ 4eVτ As Cu is a conductor, its resistivity will decrease with
     vd ′ = = = 4vd  1
ml/2 ml decrease in temperature. 1
The drift velocity becomes 4 times the original value. 1
22. As conductivity = Resistivity  1
15. As the wire joined in series
IX = IY So silver has the best conductivity. 1
y 1 v n 23. As resistivity depends upon the nature of its material
=
   nxeAvdx n= eAvdy or dx = 1+1
y vdy nx 2  and not the length, so the resistivity will not change.
 1+1
16. Metals have greater value of temperature co-efficient
ρl RA
of resistance. 2 24. As R = So ρ =  1
A l
17. (i) It has high value of resistivity. 1
For both wires, l and R are same.
(ii) Its temperature co-efficient of resistance is low. 1 As rCu < rmanganin
18. \ ACu < Amanganin 1
1.2
So manganin wire is thicker than copper wire.
VA ρl
25. As, I = , as R =
ρl A
For two wires, V, r, A are same
10–8Wm

r 1.1
So, I∝ 1  1
l
A
I B 1 l
0 200 400 600 800   I=
B
l
=
A
2 1
(Temp.) T(K)
26. (i) In the absence of electric field, the paths are straight
(i) It has small value of temperature co-efficient of line. 1
resistance. 1
(ii) In the presence of electric field, the paths are curved.
(ii) High resistivity. 1  1
94 Physics–12
D:\EG_Physics-12_(26-06-2022)\Open_Files\Ch-3\Ch-3
\ 27-Jul-2022 Ved_Goswami Proof-4 Reader’s Sign _______________________ Date __________

ρl ρl 1 ρl
27. As R = A = 2 ie
. ., R ∝  1 as R= = 4Ω
πr r2 A
4
So R′ = 4 W = 1W  1

Res. (R)
        1 33. P.D. across 4W resistor = 4 × 1 = 4V
P.D. across 2W resistor = 2 × 1 = 2V.
Radius
28. (i) For metals (conductors) like copper temperature Let the potential of the negative terminal of cell be zero.
co-efficient of resistivity is positive. 1 So the positive terminal is at 3V. So potential at A= 3
+ 4 = 7V.
(ii) For semiconductors like silicon, temperature co-
efficient of resistivity is negative. 1 Potential at B = 0 – 2 = – 2V. 1
29. The reciprocal of resistivity is called conductivity Because current flows from higher potential to lower
depends upon the (i) temperature (ii) nature of material. potential
 1+1 so VA – VB = 7 – (– 2) = 9V. 1
30. (i) In the region BC resistance is negative. 1 34. V = 5V, l = 0.1 m, Vd = 2.5 × 10 m/s, n = 8 × 10 m
–4 28 –3,

(ii) Ohm’s law is obeyed in the region where graphs is r = ?,


a straight line. i.e. in the region AB. 1 e = 1.6 × 10–19, m = 9.1 × 10–31 kg
31. When a potential difference V is applied across a I = neAVd ⇒ V = neAV ⇒ VA
= neAVd 1
conductor of length l, and area of cross-section A. Let n R d ρl
be the number density of free electrons in the conductor. V

So, r = l neV
Number of free electrons in length l of the conductor = d

nAl = 5
Total charge contained in length l 0.1 × 8 × 1028 × 1.6 × 10 −19 × 2.5 × 10 −4
q = neAl ½ = 1.5625 × 10–5 Wm 1
When electric field is applied, electrons start drifting 35. Equivalent Resistance
   = R1.R2/ (R1 + R2) + R3 + R4.R5/(R4+ R5) 1
opposite to E.
All the electrons which enter the conductor at one end   = [(4 × 4)/(4 + 4)] + 1 + [(12 × 6)/(12 + 6)] W ½
will pass through the conductor say in time t.   = 7 W. ½
l q [CBSE Marking Scheme, 2018-19]
t= , Current I = = neAVd  ½ 36. When electric field E is applied on conductor force
Vd t

 acting on free electrons
Current density J = I = neVd  ½  
A F = –e E
  
 eEτ m a = –e E
As Vd = 
m  −eE
   \ a =  ½
J = n( − e)  − eEτ  = ne τE
2
So m
 m  m
Average thermal velocity of electron in conductor is
 zero
 m
J = E as ρ = 2  ½
r ne τ (ut)av = 0
e Average velocity of electron in conductors in τ
e
(relaxation time) = vd (drift velocity)
I Vd = (ut)av + at ½

−eEτ
V = 0 +
32. As the wire is non-insulated, so when bent and twisted m

together.  −eEτ
Vd =  1
l m
l′ =, A ′ = 2A
2 Bend wire [CBSE Marking Scheme, 2019-20]
l 37. Mobility (µ) of charge carriers is defined as the
ρl′ ρ 2 ρl R
So, R′ = A′ = 2A = 4A = 4  1 magnitude of drift velocity per unit electric field E.

Current Electricity 95
D:\EG_Physics-12_(26-06-2022)\Open_Files\Ch-3\Ch-3
\ 27-Jul-2022 Ved_Goswami Proof-4 Reader’s Sign _______________________ Date __________

vd eEτ (a) current density j increases with increase in potential


µ= = gradient.
E mE
(b) with increase in temperature, relaxation time t
mC decreases, so current density, j decreases.

SI unit = mCs–1N–1
sN 1
OR (c) j ∝ l , with increase in length, current density j will
As we know, I = neAvd
decrease. 1
I (d) Current density j is independent of area of cross-
The current density j = section. So no change in j. ½
A
41. (i) Expression for drift velocity: In a conductor, the
\      j = nevd
free electrons are responsible to carrying current.
These electrons are in state of random motion
38. We know that Vd = eEτ = eτ V due to thermal energy. On an average the no. of
m m l
electrons travelling in any direction will be equal
1 so drift velocity will become Vd to the number of electrons travelling in the opposite
Vd ∝
l 3 direction.
Alternative method:   
Let u1, u2, ... un be the random velocity of n free
I = neAVd 1 electrons.
  
I V V  u1 + u2 + ... + un
Vd = neA = RneA = ρl ne 1     uavg. = =0 ...(1)
 N
In presence of external field E, the electrons
1 V experience a force opposite to direction of electric
So, Vd ∝ , as l ′ = 3l so Vd′ = d 1
l 3 
field.   
 
39. As J = σ E     F = − eE ⇒ ma = − eE
 
J  − eE
s =  = Slope of line 1 ⇒   a = m  ...(2)
E As the electrons accelerate, they frequently collide
sB > sA or rB < rA with +ve metal ions or electrons. Thus the gain in
(i) Material A is used in making standard resistance velocity lasts for a short time and is lost in the next
as we need wire of higher resistivity for making collision.
standard resistance. 1 Final velocity of the electrons before its suffers next
(ii) Material B will be used for making connecting wires collision will be
  
as sB > sA. Connecting wires should have higher    V1 = u1 + aτ1
conductivity.  
   V 2 = u2 + aτ 2
B 
 
   V = un + a τ n
J
A 1 Average velocity of n electron
     
u1 + a τ1 + u2 + a τ 2 + .... + un + a τ n
Vavg =
Ν
E    
u1 + u2 + .... + un a (τ1 + τ 2 + ... + τ n )
40. Current density is a vector quantity = +
Ν N
Relation between current density and potential 
= 0+aτ 1
difference
eEτ τ1 + τ 2 + ... + τ n
Current I = neAVd and Vd = m 
½ Where t =
N
ne eEτ
Current density J = I = neVd = ½ This average velocity gained by the free electrons
A m of a conductor due to the externally applied electric
2 field is called drift velocity.
J = ne τ V ½ 
m l 
Vd = a τ
V is the P.D. across conductor, m is mass of each   t = a verage relaxation time between two
electron. successive collisions.
96 Physics–12
D:\EG_Physics-12_(26-06-2022)\Open_Files\Ch-3\Ch-3
\ 27-Jul-2022 Ved_Goswami Proof-4 Reader’s Sign _______________________ Date __________

 free electrons increases exponentially with increase in


 eEτ
Vd = − temperature. The relaxation time does not change much
m  1
(– Ve sign shows that it is opposite to electric field.) with increase in temperature.
1
neAeEτ ne 2 τAV ρ ∝ n  1
As I = neA Vd = =
m ml As n increases with increase in temperature, so
V ml resistivity r will decrease with increase in temperature.
or = = Constant
I ne 2 τA 43. Relaxation time: The average time that elapses
Thus, for a given conductor when all physical between two successive collisions of an electron is
conditions are kept const V ∝ I. called relaxation time.
I 1 Relation between drift velocity and relaxation time:
(ii) (a) Vd = , I is const Vd ∝ 

neA A − eEτ
Vd = m  1
Vd will decrease with increase in area of cross-
where e = charge on an electron
section.
 I
E = electric field applied
(b) Current density, j = m = mass of electron
A
 Relation between relaxation time and resistivity:
As I is same in all parts of the wire so j will decrease
When a potential difference V is applied across a
as the area of cross-section will increase.
conductor of length l, then
(c) Electric current will be same in all parts as there is 
 − eEτ eVτ
no branching and the charges cannot accumulate d
V = =−
m lm
anywhere in the wire.
If A is the area of cross-section of the conductor and
(d) Electric field will not be same in all parts of the
n is number density of free electrons, then the current
conductor. 1
through the conductor (I) will be
42. Resistivity: Electrical resistivity of a material is defined
as the resistance of a conductor of that material, having neAeVτ
I = neAVd = ml  1
unit length and unit area of cross section.
Variation of resistivity with temperature V ml
R= = 2  ...(1)
(ii) Semi conductor I ne τA
rl
As we know that R=  ...(2)
A
1
where r is resistivity of the conductor
m
Resistivity

Resistivity

So comparing eqn. (1) & (2) we get, ρ = ne2τ 


1
44. (i) Reciprocal of resistivity of a conductor is called
conductivity. Its SI unit is ohm–1 m–1 or Siemen m–1.
0 50 100 Temperature (K) (ii) Let E is the electric field applied across the conductor
Temperature (K)
of area of cross section A. Mass of an electron is m
Conductors: For conductors, the number density n of and charge is e. If electrons are moving with drift
free electrons is almost independent of temperature velocity vd in the conductor then,
with increase in temperature, the thermal speed of free eE
electrons and the amplitude of vibration of positive ions      vd = τ …(i)
m
also increases. The collisions become more frequent,
thus relaxation time t, decreases.
m 1
As ρ = ⇒ ρ∝ 1
ne 2τ τ
So, resistivity increases with increase in temperature. If n is the charge density in the conductor then,
Semiconductors: In case of semiconductors, as the charge passing through the area A in Dt time is
temperature increases,more number of electrons q = IDt = neAvdDt
become free to conduct electricity. The number of or I = neAvd …(ii)

Current Electricity 97
D:\EG_Physics-12_(26-06-2022)\Open_Files\Ch-3\Ch-3
\ 27-Jul-2022 Ved_Goswami Proof-4 Reader’s Sign _______________________ Date __________

From Eq. (i) 46. (a) Resistance of the branch ADCB


eE 2
ne AE = R1 = R + R + R = 3R
I = neA ⋅ τ or I = ⋅ τ …(iii) 1 16
m m R= = 4Ω
4
V ne 2 A V

or   = ⋅ ⋅τ So R1 = 4 + 4 + 4 = 12Ω  1
R m l
VA ne 2 A V 1 ne 2 R1 and resistance of the branch AB are in parallel.
or = ⋅ ⋅ τ or = ⋅τ 12 × 4 48
ρl m l ρ m Rnet = = = 3Ω  1
4 + 12 16
ne 2
or   s= ⋅τ  …(iv)
m V 9
Current drawn from the cell, I = = = 3A  ½
→ → R 3
Relation between J and E .
From Eq. (iii) 9 3
Current in the branch ADCB = = A ½
2
12 4
I ne τ
     = ⋅E …(v) 3
A m Potential difference across CD = × 4 = 3V  ½
From Eq. (iv) 4
12Ω
ne 2
s= τ 1
m 4Ω
ne 2 Α Β
Putting the value of τ in Eq. (v)
m
I → →
or = sE   or J = σ E  1 9V
A (b) If the wire is stretched to double the length,then
45. (i) Resistance Resistivity area of cross-section will become half of its original
value. Thus
• Resistance is the • Resistivity of a material
property of the is the resistance per Rnew = n2R = 22 × 16 = 64W ½
conductor to resist unit length and per unit 64
Resistance of one arm of square = = 16Ω
the flow of charges area of cross-section. 4
through it. RADCB = 16 + 16 + 16 = 48W
• Its SI unit is ohm (W). • Its SI unit is ohm-metre 48 × 16 48 × 16
(W-m) Rnet = = = 12Ω
48 + 16 64
(ii) We have given here,
Area of cross section of the 1st wire and 2nd wire Current in the branch ADCB,
are A1 and A2 respectively . 9 3
Length of each conductor = L I1 = = A
48 16
The resistance of the 1st conductor and 2nd
conductor are R1 and R2 respectively Current in the branch AB,
Now, 9
I2 = A
r1L r2 L 16
R1 = and R2 =
A1 A2 P.D. between A and D = 3V
For the parallel combination, we can write P.D. between A and B = 9V 1
R1 ⋅ R 2 Potential difference between B and D is 6V.
Req =
R1 + R 2 47. The given circuit is equivalent to
5Ω
ρeq L ρ1ρ2 L2 A1A 2

⇒ = 10Ω 10Ω
A1 + A 2 A1A 2 L ( ρ1A 2 + ρ2 A1 ) A B

ρ1ρ2 ( A1 +A 2 )
C
10Ω 10Ω

⇒ req=
( ρ1A 2 + ρ2 A1 ) 5Ω 1

98 Physics–12
D:\EG_Physics-12_(26-06-2022)\Open_Files\Ch-3\Ch-3
\ 27-Jul-2022 Ved_Goswami Proof-4 Reader’s Sign _______________________ Date __________

Net resistance between A and C 5Ω

10 × 10
= R AC = = 5Ω  1 15Ω
10 + 10
Net resistance between C and B A B
 1½
= RBC = 5WThis circuit reduce to 15Ω
5Ω
5Ω

A 5Ω 5Ω
1 1 1 1
B
1 3 +1+1+ 3 8
  = + + + = =
 1 R 5 15 15 5 15 15
5Ω

15
5Ω
  R= W 1½
8
A 10Ω B 49. 100W and 400W (voltmeter resistance) are joined in
 1 parallel so their net resistance is
5Ω
100 × 400
R1 = = 80Ω  1
As they are connected in parallel 100 + 400
1 1 1 1 2 +1+ 2 5 Total resistance of the circuit
So = + + = =
R 5 10 5 10 10 R = 200 + 80 = 280W
  R = 2W. 1 9
48. The given network is equivalent toThis gets further Current I= A 1
16
reduced to
5Ω (a) Reading of the voltmeter = 80 × 0.3 = 24V 1
(b) Before the voltmeter is connected
10Ω 5Ω Total resistance = 100 + 200
 A B 2 = 300W
5Ω
10Ω
84
I = = 0. 28A  1
300
5Ω
P.D. across 100W = 100 × 0.28 = 28V. 1

Topic 2. Cells, EMF and Internal Resistance


•• Internal Resistance of a Cell : The resistance offered by Note: Internal resistance of a freshly prepared cell is
the electrolyte of a cell, to the flow of current, between usually low, but it increases as more and more
its electrodes, is called the internal resistance of the cell. current is drawn from it.
Internal resistance of a cell depends upon the following •• Electromotive Force (E.M.F.): E.M.F. is defined as the
factors: work done per unit charge by the source, in taking the
(a) the nature of the electrolyte charge once around the circuit.
(b) concentration of the electrolyte i.e., it is directly •• It is equal to the potential difference, between the two
proportional to the concentration of the electrolyte. terminals, of the cell, when no current is drawn from it.
(c) it is directly proportional to the distance between the •• Terminal potential difference: It is the potential drop
electrodes. across the terminals of a cell when a current is being
drawn from it.
(d) It varies inversely, as the common area of the
•• Relation between E.M.F., terminal potential difference
electrodes dipped in the electrolyte.
and internal resistance of a cell: When a cell of E.M.F.
(e) It increases with the decrease in temperature of the (E) and internal resistance r is connected to an external
electrolyte. resistance R.
Current Electricity 99
D:\EG_Physics-12_(26-06-2022)\Open_Files\Ch-3\Ch-3
\ 27-Jul-2022 Ved_Goswami Proof-4 Reader’s Sign _______________________ Date __________

Then E = V + Ir, where V = IR E1 E2 E3 E4 E5


E − V E − V  E − V
or r= I
=
V/R
=
 V 
R r1 r2 r3 r4 r5
I r Eeq = E1 + E2 – E3 + E4 + E5
E But req = r1 + r2 + r3 + r4 + r5
Parallel Grouping of Cells: When positive terminal
R
of all the cell are connected at one point and all their
Variation of E.M.F. and terminal P.D. of a cell with negative terminals are connected at another point, then
external resistance (R): the cells, are said to be connected in parallel.
E1r2 + E 2r1 1
Eeq = r1 + r2 and r = 1 + 1
V eq r1 r2

E E E1
I1
r1
R R
A B
(a) (b) I

A I2 E2

r2
V
B or we can also write
O I IB
E eq E E
(c) = r1 + r 2
req 1 2
For fig. (c) As V = – Ir + E
Compare it with y = mx + c Electric Energy and Power
At A, I=0 •• Electric Energy: The total amount of work done by
the source of emf in maintaining an electric current in a
V = E = intercept on y-axes
circuit for a given time is called electric energy. S.I. Unit
At B, V=0 of electric energy is Joule
E = IBr  or r= E Commercial unit of electric energy is kilowatt hour
IB
written as 1 kWh
or r = – ve of slope of the graph.
1 kWh = 3.6 × 106 J
•• Grouping of Cells : (a) series grouping (b) parallel
grouping •• Electric Power: The rate at which, the work is done,
by the source of emf in maintaining an electric current
Series grouping of cells: When the negative terminal
through a circuit is called electric power.
of one cell, is connected to, the positive terminal,of the
other cell and so on, then the cells are connected to be P= = 2R V2
VI I=
in series.     R
E1 E2 E3 S.I. unit of electric power is 1 watt = 1 W
I
A •• Joule’s Law of Heating:
B C D
Heat produced in a resistor is
r1 r2 r3
(i) directly proportional to the square of current for a
Equivalent E.M.F. of a series combination (Eeq) of n cells
given R
is given by
(ii) directly proportional to resistance R for a given I.
Eeq = E1 + E2 + E3 + .... En
(iii) directly proportional to the time for which the current
The equivalent internal resistance (req) of the series flows through the resistor
combination of n cells is given by
     H = I2Rt
req = r1 + r2 + r3 + .... + rn •• Power rating: It is the electrical energy consumed per
The above expression for Eeq is valid if the n cells assist second by the appliance when connected across the
each other. However if the connections are like marked voltage of the mains.

100 Physics–12
D:\EG_Physics-12_(26-06-2022)\Open_Files\Ch-3\Ch-3
\ 27-Jul-2022 Ved_Goswami Proof-4 Reader’s Sign _______________________ Date __________

EXERCISE 3.2
I. Objective Type Questions (1 Mark) external resistance R. The internal resistance of
1. Choose the correct answers from the given options two sources are r1 and r2 (r2 > r1). If the potential
(MCQs). difference across the source of internal resistance
(i) Two batteries of emf e1 and e2 (e2 > e1) and internal r2 is zero, then R equals to: 1
resistances r1 and r2 respectively are connected in r1 + r2
(a) (b) r2 – r1
parallel as shown in figure. r2 − r1
r1r2 r1 + r2
(c) (d)  [CBSE 2022]
r2 − r1 r1r2
2. Fill in the blanks.
(i) The terminal potential difference of a cell is
(a) The equivalent emf eeq of the two cells is zero when the potential drop across its internal
between e1 and e2, i.e., e1 < eeq < e2. resistance is .......... .
(b) The equivalent emf eeq is smaller than e1. (ii) In order to get a large amount of current from the
cells connected in series, the external resistance
(c) The equivalent emf is given by eeq = e1 + e2
should be .......... .
always.
3. State True or False
(d) eeq is independent of internal resistances r1
and r2. (i) The direction of current inside the cell is from
(ii) If the electric current in a lamp decreases by 5%, positive to negative electrode but outside the cell,
then the power output decreases by it is from negative to positive electrode.
(ii) In order to get the maximum current in the circuit,
(a) 25% (b) 5% (c) 10% (d) 20%
the mixed grouping of cells must be done in such
(iii) Two similar head lamps are connected in parallel
a way that the external resistance is equal to the
to each other. Together, they consume 48W from
net (effective) internal resistance of all the cells.
a 6V battery, the resistance of each filament is
(iii) The relation P = R2I is used when supplied electric
(a) 1.5 W (b) 3 W (c) 4 W (d) 6 W
power is lost or dissipated as heat.
(iv) The heat produced by 100W heater in 2 minutes
4. Match the Columns
is equal to 1
Column-I Column-II
(a) 10.5kJ (b) 16.3kJ
(c) 12.0kJ (d) 14.2kJ (i) Electric energy (a) Heat produced
commercial unit
[CBSE S.P. 2019-20]
(v) The electric power consumed by a 220 V – 100 W (ii) Joule’s law (b) req = r1 + r2 + .....
bulb when operated at 110 V is 1
(iii) Series grouping of cells (c) 1 = 1 + 1 + ....
(a) 25 W (b) 30 W req r1 r2
(c) 35 W (d) 45 W [CBSE 2022] (iv) Parallel grouping of cells (d) kWh
(vi) In a dc circuit the direction of current inside the For questions numbers 5 two statements are given-one
battery and outside the battery respectively are: labelled Assertion (A) and the other labelled Reason
 1 (R). Select the correct answer to these questions from
(a) positive to negative terminal and negative to the codes (a), (b), (c) and (d) as given below.
positive terminal (a) Both A and R are true and R is the correct
(b) positive to negative terminal and positive to explanation of A
negative terminal
(b) Both A and R are true but R is NOT the correct
(c) negative to positive terminal and positive to explanation of A
negative terminal
(c) A is true but R is false
(d) negative to positive terminal and negative to
positive terminal [CBSE 2022] (d) A is false and R is also false
(vii) Two sources of equal emf are connected in series. 5. Assertion (A): In a simple battery circuit the point
This combination is, in turn connected to an of lowest potential is positive terminal of the battery.

Current Electricity 101


D:\EG_Physics-12_(26-06-2022)\Open_Files\Ch-3\Ch-3
\ 27-Jul-2022 Ved_Goswami Proof-4 Reader’s Sign _______________________ Date __________

Reason (R): The current flows towards the point of 9. A uniform wire of resistance 12W is cut into three
the higher potential as it flows in such a circuit from pieces in the ratio 1 : 2 : 3 and the three pieces are
the negative to the positive terminal. connected to form a triangle, A cell of emf 8V and
II. Short Answer Type Questions-I (2 Marks) internal resistance 1W is connected across the highest
of the three resistors. Calculate current through each
1. Distinguish between emf and terminal voltage of a
part of the circuit.
cell. [AI 2015]
10. The plot of the variation of potential difference,
2. A variable resistor R is connected across a cell of emf
across a combination of three identical cells in series,
E and internal resistance r as shown in the figure.
versus current is shown alongside. What is the emf
Draw a graph showing the variation of (i) terminal
and internal resistance of each cell? [AI 2016]
voltage V and (ii) the current I, as a function of R.
[Delhi 2015]
R 6V

0
I 1A
+ –
11. A 10V battery of negligible internal resistance is
E, r connected across a 200V battery and a resistance
3. Define the term ‘power loss’ in a conductor of of 38W as shown in the figure. Find the value of the
resistance R carrying a current I. In what form does current in circuit. [Delhi 2013, 2018]
this power loss appear? Show that to minimise the 10V
power loss in the transmission cables connecting the
power stations to homes, it is necessary to have the
connecting wires carrying current at enormous high
values of voltage. [AI 2015]
4. A parallel combination of two cells of emf’s E1 and E2,
internal resistances r1 and r2 is used to supply current
38Ω 200V
to a load of resistance R, write the expression for the
current through the load in terms of E1, E2, r1 and r2. 12. In the two electric circuits shown in the figure given
[S.P. 2011] below, determine the readings of ideal ammeter (A)
and the ideal voltmeter (V). [Delhi 2015]
5. A cell of emf ‘E’ and internal resistance ‘r’ is
–V + + –
connected across a variable resistor ‘R’. Plot a graph V
showing variation of terminal voltage ‘V’ of the cell 6V 6V
versus the current ‘I’. Using the plot, show how 1Ω 1Ω
the emf of the cell and its internal resistance can be A A
determined. [AI 2014] 9V 9V
6. The emf of a cell is always greater than its terminal 1Ω 1Ω
voltage. Why? Give reason. [Delhi 2013] (a) (b)

7. A cell of emf E and internal resistance r is connected 13. A battery of emf 12V and internal resistance 2W is
across a variable resistor R. Plot a graph showing connected to a 4W resistor as shown in the figure.
the variation of terminal potential V with resistance
R, predict from graph the condition under which V V
2
12V
becomes equal to E. [CBSE 2009]
8. Two heating elements of resistances R1 and R2 when
operated at a constant supply of voltage, V, consume
powers P1 and P2 respectively. Deduce the expressions
A
for the power of their combination when they are, in 4
turn, connected in (i) series and (ii) parallel across (a) Show that a voltmeter when placed across the cell
the same voltage supply. [AI 2011] and across the resistor, in turn, gives the same
reading.
102 Physics–12
D:\EG_Physics-12_(26-06-2022)\Open_Files\Ch-3\Ch-3
\ 27-Jul-2022 Ved_Goswami Proof-4 Reader’s Sign _______________________ Date __________

(b) To record the voltage and the current in the circuit, (i) Now if the potential difference across ‘r’ is
why is voltmeter placed in parallel and ammeter measured using a voltmeter of resistance ‘Rv’,
in series in the circuit? [AI 2016] show that the reading of voltmeter is less than the
14. Two electric bulbs P and Q have their resistances in true value.
the ratio of 1 : 2. They are connected in series across (ii) Find the percentage error in measuring the
a battery. Find the ratio of the power dissipation in potential difference by a voltmeter.
these bulbs. [Delhi 2018] (iii) At what value of RV, does the voltmeter measures
15. A 9 V battery is connected in series with a resistor .The the true potential difference? [S.P. 2015-16]
terminal voltage is found to be 8 V. Current through 20. Two identical cells of emf 1.5 V each joined in parallel
the circuit is measured as 5 A. What is the internal supply energy to an external circuit consisting of two
resistance of the battery? [CBSE S.P. 2018-19] resistances of 7W each joined in parallel. A very high
16. Two bulbs are rated (P1, V) and (P2, V). If they are resistance voltmeter reads the terminal voltage of cells
connected (i) in series and (ii) in parallel across to be 1.4V. Calculate the internal resistance of each
a supply V, find the power dissipated in the two cells. [AI 2016]
combinations in terms of P1 and P2. [Delhi 2019] 21. Write any two factors on which internal resistance
17. A variable resistor R is connected across a cell of emf of a cell depends. The reading on a high resistance
e and internal resistance r. voltmeter, when a cell is connected across it, is 2.2 V.
(a) Draw the circuit diagram. When the terminals of the cell are also connected to a
(b) Plot the graph showing variation of potential drop resistance of 5W as shown in the circuit, the voltmeter
across R as function of R. reading drops to 1.8V. Find the internal resistance of
(c) At what value of R current in circuit will be the cell. [AI 2010]
maximum. [CBSE S.P. 2020-21] +
V

OR
A storage battery is of emf 8V and internal resistance
0.5 ohm is being charged by d.c supply of 120 V using
+ –
a resistor of 15.5 ohm
(a) Draw the circuit diagram.
(b) Calculate the potential difference across the ( )
battery. R = 5Ω K
(c) What is the purpose of having series resistance in 22. (i) Two cells of emf E1 and E2 have their internal
this circuit? [CBSE S.P. 2020-21] resistances r1 and r2, respectively. Deduce an
III. Short Answer Type Questions-II (3 Marks) expression for the equivalent emf and internal
18. Two cells of emfs E1 and E2 and internal resistances r1 resistance of their parallel combination when
and r2 are connected in parallel. Derive the expression connected across an external resistance R. Assume
for the (i) emf and (ii) internal resistance of a single that the two cells are supporting each other.
equivalent cell which can replace this combination. (ii) In case the two cells are identical, each of emf E
[AI 2015] = 5 V and internal resistance r = 2 W, calculate
19. The potential difference across a resistor ‘r’ carrying the voltage across the external resistance R = 10
current ‘I’ is Ir. W. [Delhi 2020]

Answers 3.2
I. Objective Type Questions 5. (d) A is false and R is also false
1. (i) (a) (ii) (c) (iii) (a) (iv) (c) Inside the cell, positive charge flows from negative
(v) (a) (vi) (c) (vii) (b) terminal to the positive terminal (–ve → +ve), so
2. (i) equal to the emf of the cell positive terminal is at higher potential. In a circuit,
(ii) very large as compared to the net internal resistance current (I) flows from higher potential to lower potential
(r) of the cells i.e., from positive terminal to negative terminal (+ve
3. (i) False (ii) True (iii) False → –ve).
4. (i)-(d) (ii)-(a) (iii)-(b) (iv)-(c)

Current Electricity 103


D:\EG_Physics-12_(26-06-2022)\Open_Files\Ch-3\Ch-3
\ 27-Jul-2022 Ved_Goswami Proof-4 Reader’s Sign _______________________ Date __________

1. EMF is potential difference between terminals of a cell, Potential corresponding to point A on the graph = EMF
when no current is drawn from it i.e., the circuit is open. of the cell. ½
 1 Internal resistance r = – (slope of line AB) ½
Terminal voltage of a cell is the potential difference 6. E
between the terminals of a cell, when the circuit is
closed. 1 r
2.
½
V
E I

R
R R  ½+½
E.M.F. is maximum potential difference between two
electrodes of a cell in open circuit and terminal voltage
E is P.D. between two electrodes when cell is in closed
E = EMF of the cell , As I = R + r  1
circuit. 1
3. (i) When a conductor or resistance R carrying current I E = V + Ir
is used for power transmission then the rate of loss of
energy with time is called power loss in a conductor. So, V = E – Ir  ½
V
 ½
7. As V = E − Ir = E − r or V + Vr = E ½
(ii) This power loss appears in the form of heat. ½ R R
(iii) Suppose power P is to be transmitted to a load R E
V=  ½
via transmission cable of resistance Rt. If V is the r 
voltage across load and I the current through it     + 1
R
P
P = VI or I =
 ½
V E

Power wasted in transmission cable


½
P2 2
V
Pt = I R t = 2 R t  ½
V
So in order to minimise the power loss in transmission R
cable, it should be transmitted at high ‑voltage.
When graph becomes parallel to resistance axis then R
4. Their equivalent internal resistance = ∞
rr So, Now the circuit is open therefore V = E. ½
req = 1 2 ½
r1 + r2 8. (i) In series: Let P = net power, R = net res.
2 2
E r + E 2 r1 R1 = V , R 2 = V
Eeq = 1 2 ½ P1 P2
r1 + r2
Net res. = R = R1 + R2, divide both sides by V2
Current in the circuit,
R R R 1 1 1
E1r2+ E 2 r1 So = 12 + 22 or 2 = V 2 + V 2
V2 V V V
E eq r1 + r2 E1 r2 + E 2 r1 R R1 R 2
I= R =
r1 r2
=
R(r1 + r2) + r1 r2 1
total R+
r1 + r2
or    1 = 1 + 1  1
P P1 P2
5.
(ii) In parallel : 1 = 1 + 1
A R R1 R2
Terminal Multiply both sides by V2,
voltage 1 V V V2 2 2
(V)
    R = R1 + R 2 or P = P1 + P2  1
0
B 9. RAB = 2W, RBC = 4W, RCA = 6W
Current (I)
2W and 4W resistors are in series, their equivalent
104 Physics–12
D:\EG_Physics-12_(26-06-2022)\Open_Files\Ch-3\Ch-3
\ 27-Jul-2022 Ved_Goswami Proof-4 Reader’s Sign _______________________ Date __________

B 13. (a) Rnet = 4 + 2 = 6W


E 12
4Ω   I = R = 6= 2A
2Ω net


I1 6Ω ½ P.D. across 4W resistor = 4 × 2 = 8V ½
A
I2
C
Current through the cell flows from negative terminal
I
to the positive terminal so
V = E – Ir = Terminal P.D. = 12 – 2 × 2 = 8V ½
8V 1Ω Thus the reading of voltmeter when placed across
R1 = 2 + 4 = 6W the resistor and across the cell is same = 8V.
R1 and RAC are in parallel so the equivalent resistance (b) An ammeter is used in series because ammeter has
R is given by very low resistance to ensure that all the current
6×6 flows through it, while voltmeter is used in parallel
R = 6 + 6 = 3Ω  ½ to measure potential difference across two points in
the circuit. Voltmeter has very high resistance (R ≈
E 8 ∞) to ensure that its connection does not alter the
So I = R + r = 3 + 1 = 2A  ½
flow of current I. 1
As RABC = RAC = 6W and are connected in parallel 14. In series combination current is same in two bulbs.
so I1 = I2 = 1A. ½ Therefore power dissipation by a bulb is
10. (i) EMF of 3 cells in series = 6V P ∝ R 1
EMF of one cell = 6 = 2V  1 i.e. P1 ∝ R1
3
P2 ∝ R2
(ii) Total internal resistance of combination
E 6 P1 R1 P1 1
= r = I = 1 = 6Ω ∴     P = R   or   P = 2 
1
max 2 2 2

Internal resistance of one cell = 6 = 2Ω  1 15. r=


ε−V
 1
3
11. Net EMF = 200 – 10 = 190V ½ I
R = 38W 9V − 8 V
=  ½
E 190 5A
I = R = 38 = 5A  1½
total = 0.2 W ½
12. For circuit (a): Total EMF = 6 + 9 = 15V [CBSE Marking Scheme, 2018-19]
Total resistance = 1 + 1 = 2W, 16. The power consumption due to the two bulbs is given
I = 15 = 7.5A  ½ as:
2 (i) In series combination: Let V is the supply voltage
For 6V cell, current flows from –ve terminal to positive and R1 and R2 are resistance of the two bulbs then
terminal so potential difference for 6V cell is power consumption due to two bulbs is
V = E – Ir = 6 – 7.5 × 1 V2
P=
= – 1.5 V ½ R1 + R 2
Reading of ammeter = 7.5 A V 2 and P = V 2
Reading of voltmeter = –1.5 V But P1 = 2  ½
R1 R2
For circuit (b): Net emf = 9 – 6 = 3V where V is the supply voltage and R1 and R2 are the
Net resistance = 1 + 1 = 2W resistance of the lamps respectively
Reading of ammeter = current = 3 = 1.5A  ½ ⇒ P= V2 PP
= 1 2  ½
2 2
V +V 2 P1 + P2
As the current through 6V cell is from positive terminal P1 P2
to negative terminal so terminal potential difference
(ii) In parallel combination the same voltage is applied
V = E + Ir = 6 + 1.5 × 1 = 7.5V ½ to both bulbs. Both bulbs are independent. Hence,
Ammeter reading = 1.5A power
Voltmeter reading = 7.5V P = P 1 + P 2 1

Current Electricity 105


D:\EG_Physics-12_(26-06-2022)\Open_Files\Ch-3\Ch-3
\ 27-Jul-2022 Ved_Goswami Proof-4 Reader’s Sign _______________________ Date __________

17. (a) Circuit diagram I1 E1


R
r1

I I
I2 E2

r2
+ – I
 r

(b)     If Eeq is the equivalent emf


e
Then Eeq = V + Ireq
V E eq − V E eq V
So    I = = −  ½ ...(2)
req req req
0 R
Comparing eqn. (1) and (2),
V
(c) We have, V = IR or I = E eq E1 E 2
R req = r1 + r2  ...(3)
when R = 0, current will be maximum. Hence,
maximum current drawn will be at R = 0 V 1 1
req = V  r1 + r2 
OR
120V
1 1 1 r1 r2
or + req = 1
req = r1 r2 or r1 + r2 
(a)   I
From eqn. (3) we get
e r R
E E  E E  r r
Eeq =  r1 + r 2  req =  r1 + r 2  r 1+ 2r
 1 2   1 2  1 2
(b) Effective voltage, Eeff = 120 – 8 = 112V Equivalent
resistance, E1r2 + E 2r1
E eq =
Req = r + R = 0.5 + 15.5 = 16W     r1 + r2  1
E eff 112 19. Without voltmeter

\ I= = = 7A
Req 16 V = Ir  ...(1)
\ Terminal voltage of battery during charging. When voltmeter is connected across r, say current I'
passes through r and remaining current I-I' through
V = E + Ir = 8 + 7 × 0.5
voltmeter.
= 11.5V
So I′r = (I – I′) RV
(c) Series resistance controls the current drawn from
IR V
external supply. In its absence, the current during I′(r + RV) = IRV ⇒ I′ =
the charging will be dangerously high. r + RV
18. Parallel Grouping: When positive terminals of all IR V r Ir V
V′ = I′r = = =
cells are connected to one point and negative terminal r + RV 1 + r 1+
r
to another point,then the cells are said to be connected RV RV
in parallel.    ...(2) [Using (1)]
Consider two cells of EMF E1 and E2, internal resistance So V′ < V 1
r1 and r2 connected in parallel, between the points A and
C.
E1 − V E −V I
I1 = , I2 = 2
r1 r2 I′ r
E1 − V E 2 − V
I = I1 + I2 = +
r1 r2 (I – I′) V
RV
E E 1 1 V − V′
I = r1 + r 2 − V  r + r   ...(1) ½ (ii) Percentage error =
V
× 100
1 2  1 2

106 Physics–12
D:\EG_Physics-12_(26-06-2022)\Open_Files\Ch-3\Ch-3
\ 27-Jul-2022 Ved_Goswami Proof-4 Reader’s Sign _______________________ Date __________

IrR V Equivalent cell


V – V′ = Ir −
r + RV A B
E r
Ir Ir 2 I
Rt = [r + R V − R V ] = R
r + RV r + RV
Potential difference across A and B

\ Percentage error For first cell:
Ir 2 r × 100 V = VA – VB = E1 – I1r1 ...(1)
= × 100 =  1
(r + R V) Ir r + RV For second cell:
V = VA – VB = E2 – I2r2 ...(2)
(iii) As R V → ∞, V′ = Ir = V [Using (2)] 1
E1 V
20. R = Equivalent resistance in the external circuit ⇒ I1 = −  ...(3) from (1)
r1 r1
  = 7 = 3.5 Ω E2 V
2
⇒ I2 = −  ...(4) from (2)
V = 1.4 V. r2 r2
V 1.4
So I = = = 0.4 A  1 E V
R 3.5 For equivalent cell I = −  ...(5)
r r
Also E = V –Irnet

 I = I1 + I2
r
1.5 = 1.4 − 0.4 E V  E1 − V  +  E 2 − V 
2
\ − =   
r r  r1 r1   r2 r2 
0.1 = 0.2r or r = 0.1 = 0.5 Ω  1
0.2
E V  E1 E 2  1 1
E r − = +  − V + 
r r  r1 r2   r1 r2 
E r On comparing we get 1 = 1 + 1
r r1 r2
I 1
\ Equivalent internal resistance is
7Ω r1r2
r=
r1 + r2
E E1 E 2 E r +E r
7Ω Also r
= +
r1 r2
= 1 2r r 2 1
12
21. Internal resistance of a cell depends upon
(i) Distance between the electrodes 1 E1r2 + E 2r1

\ Equivalent emf. is E = r1 + r2
(ii) Common area of the electrodes immersed in the
electrolyte. (ii) Given E1 = E2 = 5 volt, r1 = r2 = 2W
Given E = 2.2 V, V = 1.8 V, R = 5W 1 5× 2 + 5× 2
E= = 5 Volt
 E − V
Internal resistance = r =  V  R 2+2
2× 2
r= =1W
2.2 − 1.8 0.4 2
=  1.8  5 = 1.8 ×5=
1.8
2+2
 
E 5 5
r = 1.1 W 1 I= = = A
( R + r ) (10 + 1) 11
22. (i) A I1 E1 r1 B
\ Voltage across resistance R
5 50
I I2 E2 r2
⇒ V = IR = × 10 = V
11 11
R
= 4.54 Volt

Current Electricity 107


D:\EG_Physics-12_(26-06-2022)\Open_Files\Ch-3\Ch-3
\ 27-Jul-2022 Ved_Goswami Proof-4 Reader’s Sign _______________________ Date __________

Topic 3. Kirchhoff’s Laws and Wheatstone Bridge


•• Kirchhoff’s Laws of Electrical Network Wheatstone bridge
(1) Kirchhoff’s first law/Junction rule/Current rule: If four resistance are connected is the four arms of a quadril.
It states that the algebraic sum of currents at any ateral, a source of emf is connected between two diagonally
junction is zero. opposite points and a galvanometer is connected between
SI = 0 the remaining two diagonally opposite points, then this
I5 arrangement is called a Wheatstone bridge.
I4
B
I1
I3 P Q
I2

or A G C
Sum of currents, entering a junction,is equal to the,
sum of currents leaving the junction R S
I1 + I2 = I3 + I4 + I5
or I1 + I2 – I3 – I4 – I5 = 0 D

(2) Kirchhoff’s second law/loop rule/voltage law: ( )


The algebraic sum of changes in potential around any Balance condition of Wheatstone bridge: When the bridge
closed loop must be zero. is balanced i.e., no current flows through the galvanometer
i.e. SDV = 0 then
D E
      P = R
I1 + I2
R3 Q S
At balance point, potential at point B = potential at point D
C F
R2 I2 E2 When bridge is balanced, the ratio of the resistances in two
adjoining arms is equal to the ratio of the resistances in other
B A two corresponding arms.
R1 I1 E1
Meter Bridge
or
•• It is based on the principle of Wheatstone Bridge. It
The algebraic sum of the e.m.f.’s in any closed loop of
consists of one metre long constantan or manganin wire.
a circuit is equal to the sum of the product of currents
The resistance of the wire is divided into two resistances
and resistances in it.
P and Q. If R is known resistance then the unknown
i.e., SE = SIR resistance S can be determined by using the formula
In loop ABCFA E1 – E2 = I1R1 – I2R2 P
= R
Note: Sign convention for applying loop rule: Q S
1. The emf of a cell is taken as positive if the direction (If k is the resistance per unit length of the wire)
in which loop is traversed is from negative terminal Resistance box Resistance wire
to positive terminal (through the electrolyte).
(R) (S)
2. The emf of a cell is taken as negative if the direction D
in which loop is traversed is from positive terminal
G
to negative terminal (through the electrolyte).
A (P) B (Q) C
3. The product IR is taken as positive if the resistor is l (100 – l)
traversed in the same direction as of the assumed
current.
4. The product IR is taken as negative if the resistor is (Rh)
traversed opposite to the assumed direction of current. + –
( )
Cell Rheostat K

108 Physics–12
D:\EG_Physics-12_(26-06-2022)\Open_Files\Ch-3\Ch-3
\ 27-Jul-2022 Ved_Goswami Proof-4 Reader’s Sign _______________________ Date __________

100 − l rl
R
= lk
=
l or S = R  l  S = A , we get
S (100 − l) k 100 − l  
It can also be used to calculate the specific resistance or 2
r = SA = Sπr ,
resistivity of the material used for making resistor S. Let l l
l be the length of the resistor S and A its area of cross r = radius of wire S
section then using

EXERCISE 3.3
I. Objective Type Questions (1 Mark) 2. Fill in the blanks.
1. Choose the correct answers from the given options (i) For the wheatstone bridge to be balanced, the
(MCQs). necessary conditions for the four resistances
(i) The resistance R is to be measured using a meter (P, Q, R, X) is ............... .
bridge. Student chooses the standard resistance S (ii) Kirchhoff’s second law is based on the law of
to be 100 W. He finds the null point at l1 = 2.9 cm. conservation of ............... .
He is told to improve the accuracy. Which of the
3. State True or False
following is a useful way?
(a) He should measure l1 more accurately. (i) Meter bridge is used for measuring resistivity of
the material.
(b) He should change S to 1000 W and repeat the
experiment. (ii) According to Kirchhoff’s second law, the algebraic
(c) He should change S to 3 W and repeat the sum of charges in potential around any closed loop
experiment. must be zero.
(d) He should give up hope of a more accurate 4. Match the Columns
measurement with a meter bridge. Column-I Column-II
(ii) Kirchhoff’s junction rule is a reflection of (i) Kirchhoff’s first law (a) Junction rule
(a) conservation of current density vector. (ii) Kirchhoff’s second law (b) To measure an
(b) conservation of charge. unknown resistance
(c) the fact that the momentum with which a (iii) Meter bridge (c) Loop rule
charged particle approaches a junction is For questions numbers 5 two statements are given-one
uncharged (as vector), as the charged particle labelled Assertion (A) and the other labelled Reason
leaves the junction. (R). Select the correct answer to these questions from
(d) the fact that there is no accumulation of the codes (a), (b), (c) and (d) as given below.
charges at a junction. (a) Both A and R are true and R is the correct
(iii) Kirchhoff’s first rule at a junction in an electrical explanation of A
network, deals with conservation of (b) Both A and R are true but R is NOT the correct
(a) energy (b) charge explanation of A
(c) momentum (d) both energy and charge (c) A is true but R is false
[Delhi 2020] (d) A is false and R is also false
(iv) Kirchhoff’s first rule SI = 0 and second rule SIR = 5. Assertion (A): Voltmeter can measure the emf of a
SE (where the symbols have their usual meanings) cell more precisely than a potentiometer
are respectively based on–
Reason (R): Voltmeter has a low resistance.
(a) conservation of momentum and conservation
of charge II. Very Short Answer Type Questions (1 Mark)
(b) conservation of energy, conservation of charge 1. On which principle ‘Meter Bridge’ is based upon?
(c) conservation of charge, conservation of III. Short Answer Type Questions-I (2 Marks)
momentum
2. State Kirchhoff’s rules and explain on what basis they
(d) conservation of charge, conservation of energy
are justified. [Delhi 2017, AI 2015]
[CBSE 2022]

Current Electricity 109


D:\EG_Physics-12_(26-06-2022)\Open_Files\Ch-3\Ch-3
\ 27-Jul-2022 Ved_Goswami Proof-4 Reader’s Sign _______________________ Date __________

3. In the circuit diagram shown, AB is a uniform wire 4V 8V


of resistance 15W and length 1m. It is connected to a 0.5 1.0
cell E1 of emf 2V and negligible internal resistance
i i
and a resistance R. The balance point with another
cell E2 of emf 75 mV is found at 30 cm from end A.
Calculate the value of the resistance R. [AI 2015] A 3.0 B
E1 R 4.5
6.0
8. Following circuit was set up in a meter bridge
X
experiment to determine the value X of an unknown
A B resistance. [CBSE SP 2018-19]
G
(a) Write the formula to be used for finding X from
E2 the observations.
4. Use Kirchhoff’s rules to determine the potential (b) If the resistance R is increased, what will happen
difference between the points A and D when no to balancing length?
current flows in the arm BE of the electric network Resistance box
shown in the figure. [AI 2015] R X

D
3Ω E I
F D
I I
I G
1V A B C
R1
R I

2Ω 3V

A C
B
6V I I 4V 9. Calculate the potential difference across the 4
5. Two cells of E.M.F. 10 V and 2V and internal W resistor in the given electrical circuit, using
resistances 10W and 5W respectively, are con nected Kirchhoff’s rules. [AI 2019]
in parallel as shown in the figure. Find the effective
voltage across R. [S.P. 2015-16]
R
I
2V
I2 5

10. Explain the principle of working of a meter bridge.


I1 10V 10 Draw the circuit diagram for determination of an
unknown resistance using it. [AI 2020]
6. In the given circuit, assuming point A to be at zero
potential, use Kirchhoff’s rules to determines the IV. Short Answer Type Questions-II (3 Marks)
potential at point B. [AI 2011] 11. In the meter bridge experiment, balance point was
6V observed at J with AJ = l.
1A D 4A
B R X

2 R1
R
G

A C E l
3V 4A
7. In the circuit shown in the figure, find the current A
J
B

through each resistor. [Delhi 2015 C]

110 Physics–12
D:\EG_Physics-12_(26-06-2022)\Open_Files\Ch-3\Ch-3
\ 27-Jul-2022 Ved_Goswami Proof-4 Reader’s Sign _______________________ Date __________

(i) The values of R and X were doubled and then resistor (X), connected in one arm of the bridge has
interchanged. What would be the new position the resistance 22 KW. The resistance of BC and CD
of balance point? arms are now interchanged and another resistance S
(ii) If the galvanometer and battery are interchanged in place of X (= 22 KW) is used. So that the current
at the balance position, how will the balance point through arm BD is again zero. What is the value of
get affected? [AI 2011] resistor S? [S.P. 2011]
12. Answer the following: B

(a) Why are the connections between the resistors in 2R 2R


a meter bridge made of thick copper wires/strips?
A 3R
[AI 2019] C

(b) Why is it generally preferred to obtain the balance R

X
point in the middle of the meter bridge wire?
D
[AI 2019]
(c) Which material is used for the meter bridge wire V
and why? [AI 2014]
17. Using Kirchhoff’s rules, calculate the current through
13. State Kirchhoff’s rules for an electric network. Using the 40 W and 20 W resistors in the following circuit:
Kirchhoff’s rules, obtain the balance condition in
80 V 20 Ω
terms of the resistances of four arms of Wheatstone A – + B
bridge. [Delhi 2013]
14. Calculate the value of the resistance R in the circuit 40 Ω
shown in the figure so that the current in the circuit is D C
0.2A. What would be the potential difference between
points B and E? [AI 2012] E + – F
40 V 10 Ω
8V 0.2A B 5Ω
A C OR
What is end error in a meter bridge? How is it
15Ω overcome? The resistances in the two arms of the
10Ω 10Ω meter bridge are R = 5 W and S respectively.
30Ω

3V

F
R 0.2A E D

15. State Kirchhoff’s rules. Use these rules to write the


expressions for the currents I1, I2 and I3 in the circuit
diagram shown. [AI 2010]
I1 E1 = 2V r1 = 4Ω
When the resistance S is shunted with an equal
resistance, the new balance length found to be 1.5 l1,
I2 E2 = 1V r2 = 3Ω where l1 is the initial balancing length. Calculate the
value of S. [Delhi 2019]
18. (i) Derive the condition of balance for Wheatstone
I3 E3 = 4V r3 = 2Ω bridge.
(ii) Draw the circuit diagram of meter bridge to
16. In the given Wheatstone bridge, the current in explain how it is based on Wheatstone bridge.
the resistor 3R is zero. Find the value of R, if the [AI 2020]

Current Electricity 111


D:\EG_Physics-12_(26-06-2022)\Open_Files\Ch-3\Ch-3
\ 27-Jul-2022 Ved_Goswami Proof-4 Reader’s Sign _______________________ Date __________

V. Long Answer Type Questions (5 Marks) R S


19. (i) State the principle of working of a meter bridge.
Draw the circuit diagram for finding an unknown G

resistance using a meter bridge. [AI 2019] A B


Derive the relevant formula used.
(ii) ( )
30Ω 21. Find the magnitude and direction of current in 1W
resistor in the given circuit.
R S 6V P 2Ω
A B

G 9V
1Ω
A B
D C
3Ω Q 3Ω
( ) 22. The current is drawn from a cell of emf E and internal
resistance r connected to the network of resistors
In a meter bridge with R and S in the gaps, the null each of resistance r as shown in the figure. Obtain
point is found at 40 cm from A. If a resistance of the expression for (i) the current draw from the cell
and (ii) the power consumed in the network.
30W is connected in parallel with S, the null point
[Delhi 2017]
occurs at 50 cm from A. Determine the values of
r
R and S. [AI 2016] r
r
20. In a meter bridge shown in the figure, the balance A B
r
point is found to be 40 cm from end A. If a resistance r C
of 10W is connected in series with R, balance point r
is obtained 60 cm from A. Calculate the values of R
and S. [AI 2015] E, r

Answers 3.3
I. Objective Type Questions amount of current coming to a junction must be equal
1. (i) (c) (ii) (b & d) (iii) (b) (iv) (d) to the total current leaving a junction. 1
(ii) Kirchhoff’s 2nd law/loop rule/voltage rule: It
2. (i) P = R (ii) energy states that around any closed loop, the algebraic sum
Q X
of change in potential must be 0. i.e., SDV = 0.
3. (i) True (ii) True
4. (i)-(a) (ii)-(c) (iii)-(b) This is based on conservation of energy. As
electrostatic force is a conservative force, so the
5. (c) work done by it along any closed path is zero. 1
II. Very Short Answer Type Questions 3. Balance length = 30 cm = 0.3 m
1. It is based upon principle of Wheatstone Bridge 15
RAX = × 0.3 = 4.5 Ω
2. (a) (i) Kirchhoff’s 1st law/junction rule/current rule: 1
It states that the algebraic sum of currents at any E2 = 75 mV, So 75 × 10– 3 = IRAX = I × 4.5
junction is 0.
75 × 10 − 3 50
Or I= = mA  1
4.5 3
Sum of currents entering a junction is equal to sum
For the loop AE1RBXA
of currents leaving the junction.
50
This law is based on conservation of charge. As the 2 = I (R + R AB ) = × 10 − 3 (R + 15)
conductor is neutral when current is flowing, so the total 3

112 Physics–12
D:\EG_Physics-12_(26-06-2022)\Open_Files\Ch-3\Ch-3
\ 27-Jul-2022 Ved_Goswami Proof-4 Reader’s Sign _______________________ Date __________

2 × 3 × 103 3 9. Applying Kirchhoff’s laws,


R + 15 = = × 103 = 120 8V 2Ω
50 25 A B
I1
R = 120 – 15 = 105W 1
6V
4. IBE = 0 D C
I2 1Ω
For loop AFEBA
2I + 3I + 0 × R1 = 1 + 3 + 6 ⇒ 5I = 10 E
I1 + I 2 4Ω
F


\ I = 2A 1 For closed loop ADEFBA, we get
Along the path AFD, 8 = 4(I1 + I2) + 2I1
VAD = 2 × 2 – 1 + 3 × 2 = 9V 1 \ 8 = 6I1 + 4I2
4 = 3I1 + 2I2 ...(1) ½
E1r2 + E 2r1
5. Effective voltage =  1 For closed loop DEFCD, we get
r1 + r2
6 = 4(I1 + I2) + I2
E1 = 10V, r1 = 10W, E2 = – 2V, r2 = 5W \ 6 = 4I1 + 5I2 ½
10 × 5 − 2 × 10 30 On solving (1) and (2) we get
So Eeff. = = = 2V  1
10 + 5 15 8 2
I1 = A, I2 = A ½
6. Current through 2W resistor is 3A. 7 7
Potential difference across R = 4 W
Using Kirchhoff”s rule for loop BDCEB
V = (I1 + I2)R
6 = 3 × 2 + 4R1
 8 2 10 40
4R1 = 0 or R1 = 0 1 =  +  ×4= ×4 =
 7 7 7 7
So VB = VC = 5.7 Volt ½
VC – VA = 3V given 10. Meter Bridge works on the balanced Wheatstone bridge
as VA = 0 given condition.
So, VC = 3V = VB. 1 X C Y
7. 3W and 6W resistors are connected in parallel their
equivalent (R1) G
A B
1 1 1 2 +1 3 l cm J (100 – l) cm
= + = =
R1 3 6 6 6 I K
R1 = 2W 1 + –
cell
Net EMF = 8 – 4 = 4V
X = unknown resistance
All the resistance are in series,
Y = known resistance
\ Net resistance = R = 1 + 2 + 4.5 + 0.5 = 8W
l = balancing length
Net EMF 4 1
I= = = A 1  l 
Net Res. 8 2 Then X=  Y
 100 − l 
0.5Ω 4V 8V 1Ω
R l
11. (i) Balance condition is X = 100 − l
I When R and X are doubled say
R′ = 2R, X′ = 2X
4.5Ω 2Ω = R1
R′ l′ l′ 2R R
8. (a) X = (100 – ℓ) R/ℓ 1 So = or = =  1
X′ 100 − l ′ 100 − l ′ 2X X
(b) Balancing length will increase on increase of
resistance R. 1 R l
But = 
[CBSE Marking Scheme, 2018-19] X 100 − l

Current Electricity 113


D:\EG_Physics-12_(26-06-2022)\Open_Files\Ch-3\Ch-3
\ 27-Jul-2022 Ved_Goswami Proof-4 Reader’s Sign _______________________ Date __________

So l′ = l and 100 – l′ = 100 – l 1 PI RI P R


When they are interchanged balance length AJ = Dividing (3) and (4) QI1 = SI 2 ⇒ Q = S  1
1 2
(100 – l).
14. Resistance is the branch BCD are connected in series
(ii) If the galvanometer and battery are interchanged at
the balance position, the balance point will not be So RBCD = 5 + 10 = 15W.
affected. 1 This resistance is in parallel with resistance in the
12. (a) As resistance is inversely proportional to area of cross- branch BD and BE.
section, so thick copper wire offers less resistance and So net resistance between the point B and E is
thus the error due to end resistance will be minimised. 1 1 1 1 2 + 3+1 6 1
1 = + + = = =
R BE 15 10 30 30 30 5
(b) The meter bridge is most sensitive when four ⇒ RBE = 5W 1
resistances forming the Wheatstone bridge are equal.
Applying Kirchhoff’s law for loop ABEFA
In such a case the balance point will be somewhere
in the middle of the wire. So for the maximum 8 – 3 = 0.2 × 5 + 0.2R + 0.2 × 15
sensitivity of the bridge, balance point should be in = 1 + 3 + 0.2 R
the middle of the wire. 1 1
0.2R = 1 or R= = 5Ω  1
(c) Constantan or manganin is used in meter bridge 0.2
wire, because they have higher value of resistivity Potential difference between B and E is
and low temperature co-efficient of resistance. 1 VBE = IRBE = 0.2 × 5 = 1 volt 1
13. (i) Kirchhoff’s 1st law: Algebraic sum of currents 8V
meeting at a junction in zero. ½ A B
0.2A
(ii) Kirchhoff’s second law: Around any closed loop of 0.2A

a network,the algebraic sum of changes in potential 15Ω


5Ω
difference must be zero. ½
If resistance are connected in the 4 arms of a
quadrilateral, a source of emf is connected between 3V
two diagonally opposite points and a galvanometer
is connected between the remaining diagonal, then F R = 0.2A
E

it is called a Wheatstone bridge. 15. At junction F, I1 + I2 = I3 ½


The bridge is said to be balanced if current through For loop ABCFA: – 2 + 1 = 4I1 – 3I2
galvanometer is 0. i.e., 4I1 – 3I2 = – 1 ...(1) ½
potential at B = potential at D. For loop FCDEF: – 1 + 4 = 3I2 + 2I3
Balance condition of Wheatstone bridge 3I2 + 2I3 = 3 ...(2) ½
B I –I
1 g I1 E1 = 2V r1 = 4Ω
A B
P Q
Ig
I1
A G C
I2 I2 E2 = 1V r2 = 3Ω
F C
R S
I I2 + Ig
D I3 E3 = 4V r3 = 2Ω
E D
( )
Put I3 = I1 + I2 in eqn. (2)
In loop ABDA: 3I2 + 2(I1 + I2) = 3
– PI1 + GIg + RI2 = 0 ...(1) ½ 2I1 + 5I2 = 3 ...(3) ½
In loop BCDB, Solving equation (1) and (3) we get
– Q(I1 –Ig) + S(I2 + Ig) + GIg = 0 ...(2) ½
I2 = 7 A, I1 = 2 A, I3 = I1 + I 2 = 9 A  1
Under balanced condition, Ig = 0 13 13 13
(1) becomes – PI1 + RI2 = 0, ⇒ PI1 = RI2 ...(3) 16. Let X be the resistance of arm CD
(2) becomes – QI1 + SI2 = 0 ⇒ QI1 = SI2 ...(4) As there is no current through arm BD

114 Physics–12
D:\EG_Physics-12_(26-06-2022)\Open_Files\Ch-3\Ch-3
\ 27-Jul-2022 Ved_Goswami Proof-4 Reader’s Sign _______________________ Date __________

So the bridge is balanced OR


2R R R=5W
So     2R = X According to the wheat stone bridge principle:
B R S
=
2R 2R l1 100 − l1
5 S
A 3R C =  ...(i) ½
l1 100 − l1
R      ½ After shunting means we are connecting resistance in

X
parallel;
D
S
S→
V 2
    X = R and X = 22 kW 1 S S
=  ...(ii) ½
When resistance in the arm BC and CD interchange 1.5l1 2 (100 − 1.5l1)
B Equation (i) can be written as:
2R S 500 – 5l1 = Sl1 ...(iii) ½
And, eqn. (ii) can be written as
3R
A C 10(100 – 1.5l1) = 1.5Sl1 ...(iv) ½
R       ½ From eqn. (iii) and (iv)
2R
500 − 5l1 1000 − 15l1
=
D
l1 1.5l1
V 750 – 7.5l1 = –7.5l1
Let the resistance of new resistor be S 100
2R S l1 =  ½
3
=
    R 2R
500 − 5 × 100 500 − 500
S = 4R = 4 × 22kW = 88kW 1 3 3
S= =
17. In loop ABCD applying KVL 100 100
3 3
80 – 20i1 – 40(i1 – i2) = 0
80 – 60i1 + 40i2 = 0 ...(i) 1 = 1000 × 3 = 10 Ω  ½
3 100
Apply KVL through loop FEDCF 18. (i) Refer to Solution 13.
40 + 40(i1 – i2) – 10i2 = 0 (ii) Meter Bridge: It is based on the principle of
Wheatstone Bridge. It consists of one metre long
40 + 40i1 – 50i2 = 0 ...(ii)
constantan or manganin wire. The resistance of the
4i2 – 6i1 = – 8 ...(i) wire is divided into two resistances P and Q. If R
–5i2 + 4i1 = –4 ...(ii) 1 is known resistance then the unknown resistance S
can be determined by using the formula
Solving eqn. (i) and (ii)
i2 = 4A P
= R (If k is the resistance per unit length
Q S
of the wire)
Put the value of i2 in eqn. (i)
R lk l
4 × (4) – 6i1 = –8 = =
S (100 − l) k 100 − l
16 – 6i = –8
6i1 = 16 + 8 = 24 or S = R  100 − l 
 l 
i1 = 4A

It can also be used to calculate the specific resistance
So, current through 40 W resistor
or resistivity of the material used for making resistor
= i1 – i2 = 4 – 4 = 0 A  1 S. Let l be the length of the resistor S and A its area
Current through 20 W resistor = 4A of cross section then using

Current Electricity 115


D:\EG_Physics-12_(26-06-2022)\Open_Files\Ch-3\Ch-3
\ 27-Jul-2022 Ved_Goswami Proof-4 Reader’s Sign _______________________ Date __________

Resistance box Resistance wire r in resistivity of wire, A is area of cross-section of


(S) the wire
(R)
ρ(100 − l )
D RAJ =
A
G
R ρl A l
A (P) B (Q) C So = =  1
l (100 – l) S Aρ (100 − l ) 100 − l
R 40 2
(ii) In first case: = =  ...(1)
S 60 3
(Rh)
( ) When 30 W resistor is connected across S then,
+ –
Cell Rheostat K R(30 + S) 50
  =
rl 30 S 50
S = A , we get
R R 2 R
2    + =1 ⇒ + = 1 ⇒ R = 10W 1
r = SA = Sπr , S 30 3 30
l l
10 2 10 × 3
r = radius of wire S So   = ⇒S= = 15Ω [Using (1)] 1
19. (i) Meter bridge is based on the principle of Wheatstone S 3 2
bridge. R 40 2
20. = =  1
I1 – Ig S 60 3
P Ig Q When 10W is connected in series with R,
I1
G C
Balance length = 60 cm
A
I2 R + 10 60 3
R So = =  1
S      1 S 40 2
I I2 + Ig R 10 3 10 3 2 5
  + = or = − =
S S 2 S 2 3 6
S 6
E =
10 5
It states that if four resistances P, Q, R and S are
S = 12 W 2
connected in the four arms of a quadrilateral and
a battery and galvanometer are connected in two R 2
=
diagonals as shown in the diagram. S 3
When No current flows through galvanometers 2
R= × 12 = 8 W 1
3
P R
      Q = S  1 21. For loop APQDA
Circuit diagram for meter bridge – 6 = I1 + I2 + 3I1
R S – 6 = 4I1 + I2 ...(1) ½
6V P 2Ω
A B
D
I1 + I2
1Ω
9V     ½
G
l (100 – l)
J B I1 I2
A
D C
Rh Q 3Ω
3Ω
E
( ) For loop PQCBP, I1 + I2 + 3I2 + 2I2 = 9
k
I1 + 6I2 = 9 ...(2) ½
When galvanometer shows zero deflection, then 4I1 + I2 = – 6
R R [I1 + 6I2 = 9] × 4
S = R AJ
JB – – –
ρl 42
RAJ = Resistance of length AJ = A , AJ = l – 23I2 = – 42 or I 2 = A  ½
23

116 Physics–12
D:\EG_Physics-12_(26-06-2022)\Open_Files\Ch-3\Ch-3
\ 27-Jul-2022 Ved_Goswami Proof-4 Reader’s Sign _______________________ Date __________

Substitutes I2 in eqn. (2) Two resistors connected between A and C are is parallel
r
252 207 − 252 − 45 r
½
r
I1 = 9 − = = A 
r
A 2 C 2 B
23 23 23 A
r
B
Current in the branch r
r

− 45 42 − 3   1
PQ = + = A ½ E, r
23 23 23 E, r
22.
r r
r
r
So, rAC = , similarly, rBC =
r 2 2
A
r
B
rAC and rBC are in series so the circuit reduces to
r C
   1 All the three resistors are in parallel
r
1 1 1 1 r
So, = + + ⇒ R AB =
E, r R AB r r r 3

This circuit in equivalent to r 4r


rnet = r + =
r 3 3
E 3E
r r
I = =  1
A C B
  1 rnet 4r
r r
Power Consumed = I2rnet = E 2 3E 2  1
=
r rnet 4r
E, r

Case Based Questions


I. A potential difference (V) is applied across a wire of 4. Relation between drift velocity, mobility and
copper having radius 2.5 mm. It generates a uniform electric field is
electric field 2.5 V/m along the length of the copper v
(a) m = E vd (b) m = Ed
wire. Consequently a current get starts flowing in the
copper wire. Temperature of copper wire is 60°C. (c) mE = vd (d) m = E2 vd
Assuming that each copper atom liberates one free
5. Potential difference that required to be applied
electron and are given the following values:
between the ends of a 4 metre long copper wire
Density of copper = 8.9 g /cm3 to produce the given electric field will be
Resistivity (r) of copper at 25°C = 1.7 × 10–8 W-m
(a) 3.5 V (b) 3 V (c) 5.5 V (d) 10 V
Temperature coefficient of resistivity (a) at 25°C
Ans. 1. (b) 2. (a) 3. (d) 4. (d) 5. (b)
    = 3.9 × 10–3/°C
Atomic mass (A) of copper = 63.5 II. Metre Bridge: It is a device based on wheatstone
bridge and can be used to determine the unknown
1. Resistivity (r) (in W-m) of the material of wire
resistance.
under experimental conditions will be
(a) 6.72 × 10–7 (b) 1.932 × 10–8 It is so called because it consists of 1 m long
constantan or manganin wire stretched taut and
(c) 3.95 × 10–9 (d) 1.66 × 10–5
clamped between two thick metallic strips as shown
2. Current density (J) of the wire is given by (in A/ in the diagram R is an unknown resistance whose
m2) value we can determine using metre bridge.
(a) 12.94 × 107 (b) 6.45 × 106
The jockey is connected to some point D on the wire
(c) 12.25 × 107 (d) 726.5 × 103 at a distance l cm from the end A.
3. Drift velocity (in m/s) of electrons will be The % error in measurement of R can be minimised
(a) 6.1 × 10–3 (b) 2.3 × 10–4 by adjusting the balance point near the middle of the
(c) 6.83 × 10–4 (d) 9.63 × 10–3 bridge.

Current Electricity 117


D:\EG_Physics-12_(26-06-2022)\Open_Files\Ch-3\Ch-3
\ 27-Jul-2022 Ved_Goswami Proof-4 Reader’s Sign _______________________ Date __________

(a) remains same


(b) get displace to right
(c) get displace to left
(d) direction get reverse
3. A metre bridge experimental set up is shown in
figure, the null point D is obtained at a distance
of 40 cm from end A of the metre bridge wire. If
a resistance of 10W is connected in series with
R1, null point is obtained at AD = 60 cm then the
values of R1 and R2 will be

At null point
R R AD
S = R DC
R kl1

⇒  S = k (100 − l1)
l
R = e 1001− l o S
1
(a) R1 = 4 W, R2 = 5 W
It can also be used to calculate the specific resistance
(b) R1 = 6 W, R2 = 4 W
or resistivity of the material used for making resistor
S. Let l be the length of the resistor S and A its area (c) R1 = 8 W, R2 = 16 W
of cross section then using (d) R1 = 8 W, R2 = 12 W
ρl 4. In a metre bridge, the null point is found at a
S = A , we get
distance of 40 cm from A. If a resistance of 12
2
SA = Sπr W is connected in parallel with S, then null point
l r=
l , occurs at 50.0 cm from A. Ratio of R and S will
r = radius of wire S be
1. In the metre bridge experiment, balance point was
observed at J with AJ = l. Then values of R and X
were doubled and then interchanged. Then new
position of balance point would be

(a) 4 : 5 (b) 2 : 3 (c) 2 : 5 (d) 4 : 3


5. In a metre bridge, the null point is found at a
distance of 60 cm from A. If a resistance of 5 W is
(a) (100 – l) from A (b) (100 – l) from B
connected in series with S, then null point occurs
(c) (l – 100) from A (d) (l – 100) from B at 50.0 cm from A. Relation between R and S will
2. If the galvanometer and battery are interchanged be
at the balanced position, then the balance point

118 Physics–12
D:\EG_Physics-12_(26-06-2022)\Open_Files\Ch-3\Ch-3
\ 27-Jul-2022 Ved_Goswami Proof-4 Reader’s Sign _______________________ Date __________

5 7 1. The points with the same potential are –


(a) R = 2 S (b) R = 2 S
(a) b, c, d (b) f, h, j
3 3
(c) R = 2 S (d) S = 2 R (c) d, e, f (d) a, b, j
Ans. 1. (a) 2. (a) 3. (d) 4. (b) 5. (c) 2. The current through branch bg is –
III. An experiment was set up with the circuit diagram 1
(a) 1 A (b) A
shown in figure. [CBSE 2022] 3
Given that R1 = 10 W, R2 = R3 = 5 W, r = 0 W 2
1
and E = 5 V (c) A (d) A
a b c 2 3
3. The power dissipated in R1 is –
E i R2 (a) 2 W (b) 2.5 W
(c) 3 W (d) 4.5 W
d
R1 4. The potential difference across R3 is –
e
r (a) 1.5 V (b) 2 V
j R3 (c) 2.5 V (d) 3 V
Ans. 1. (b) 2. (c) 3. (d) 4. (c)
h g f

IMPORTANT FORMULAE
Formula Symbols Application

1. Q ne I = current, Q = charged, t = time, To find charge


I= = e = charge of electron
t t
2. V = IR (Ohms law) V = Potential difference, Relation between V and I
R = Resistance

3. I = neAVd (n = number density of Vd = Drift Velocity Relation between current and


free electron) A = Areas of cross section drift velocity

4. ρl ml R = Resistance, ρ = Resistivity Relation between (i) R and r (ii)


R= = τ = relaxation time, m = mass of electron R and relaxation time
A n e2 τ A

5. RA m e = charge of electron Relation for resistivity and re-


ρ= = 2 ρ = resistivity laxation time t
l ne τ
6. 1 1 l C = conductance, s = conductivity To find C and s
C= and σ = =
R ρ RA

7. I j = current density Relation between j with Vd and


J= = neVd j = σ E s = conductivity j with E
A
8. Vd eτ m = mobility of electron To find m from Vd
µ= =
E m
9. r = r0 [1 + a (T – T0)} a = temperature coefficient of Variation of r with temperature
resistance/resistivity

Current Electricity 119


D:\EG_Physics-12_(26-06-2022)\Open_Files\Ch-3\Ch-3
\ 27-Jul-2022 Ved_Goswami Proof-4 Reader’s Sign _______________________ Date __________

10. R 2 – R1 T2 – T1 = temperature difference Formula for a


α=
R1 (T2 – T1)
11. Rs = R1 + R2 + R3 +.... Rs = equivalent resistance in series com- Series combination
bination
12. 1 1 1 1 Rp = equivalent resistance in parallel Parallel combination
= + + + ....
R P R1 R 2 R 3 combination
13. P = VI = I2 R = V2/R P = electrical power Relation for P with V, I
and R
14. (I) V = E – Ir r = internal resistance (I) Current is drawn
(II) V = E + Ir (II) cell is being charged
15. nE n = number of cells in series Current drawn when n
I= R = External resistance identical cells are connected in
(R + nr)
series
16. mE m = number of cells in parallel For max. current the external
I= resistance must be equal to the
(mR + r)
total internal resistance
17. mnE m = number of rows For max. current the external
I= n = number of identical cells in each row. resistance must be equal to the
(mR + nr)
total internal resistance
18. SI = 0 (Junction rule) SI, algebraic sum of current SV, algebraic Kirchhoff’s law
SV = 0 (loop rule) sum of potential difference
19. P R P, Q, R, S are resistances in four arms of Balanced condition of Wheat-
= Wheatstone Bridge stone Bridge
Q S
20.  100 − l  S = Unknown resistance Working condition for Wheat-
S= R R = Known resistance stone Bridge
 l 

COMMON ERRORS
S No. Errors Corrections
1. Factors affecting the resistance of a resistor, Correct relations of factors affecting specific resistance
difference between resistance and specific resistance or resistivity has to be explained with examples.
etc.
2. Circuit diagram and numericals (Wheatstone Bridge) Wheatstone's bridge-use ‘Z’ technique for balancing
condition
3. Meter Bridge Meter bridge – ratio of resistances should be taken in
corresponding arms.
4. Comparison of drift velocities in series and parallel Use appropriate relation for vd (for series
combination I = neAvd and for parallel Vd = Eet/m)
5. Kirchhoff’s rule-sign convention Potential rise in positive and fall is negative
conventionally

120 Physics–12
D:\EG_Physics-12_(26-06-2022)\Open_Files\Ch-3\Ch-3
\ 27-Jul-2022 Ved_Goswami Proof-4 Reader’s Sign _______________________ Date __________

REVISION CHART

Kirchhoff’s second law/loop rule/voltage law:


Kirchhoff’s first law/Junction rule/Current rule The algebraic sum of changes in potential around any closed
It states that the algebraic sum of currents at any junction is loop must be zero.
zero. i.e. SDV = 0   or
SI = 0 The algebraic sum of the e.m.f.’s in any closed loop of a circuit
Or is equal to the sum of the product of currents and resistances
Sum of currents, entering a junction,is equal to the, sum of in it.
currents leaving the junction i.e., SE = IR

Kirchhoff’s Law of Electrical Network


According to the junction rule in a circuit, the total of the currents in a juction is equal to the sum of currents out side the
junction. According to the loop rule the sum of the voltages around the closed loop is equal to null.

Meter Bridge Balance condition of Wheatstone bridge


Based on Wheatstone bridge When the bridge is balanced i.e., no current flows through the
galvanometer then
P R
Q = S
B

P Q

A G C

R S

( )

Current Electricity 121


D:\EG_Physics-12_(26-06-2022)\Open_Files\Ch-3\Ch-3
\ 27-Jul-2022 Ved_Goswami Proof-4 Reader’s Sign _______________________ Date __________

REVISION CHART

Electric Energy
Electric Power The total amount of work done by the
The rate at which, the work is done, source of emf in maintaining an electric
by the source of emf in maintaining current in a circuit for a given time
an electric current through a circuit is is called electric energy. S.I. Unit of
Units of Electric Current electric energy is Joule
called electric power.
V2 (a) C.G.S. electro static unit (esu) Commercial unit of electric energy is
2R
P= = VI I= (b) C.G.S. electro magnetic unit (emu) kilowatt hour written as 1 kWh.
R
S.I. unit of electric power is watt (W). (c) S.I. unit (ampere) 1 kWh = 3.6 × 106 J

Electric Current
Current Strength, in a conductor, is defined as the rate of flow of charge across any cross section of the coductor.

Non-ohmic Conductors Ohmic Conductors Ohm’s Law


The conductors which do not obey ohm’s The conductors which obey ohm’s The current (I) flowing through a
law are called non-ohmic conductors law, i.e., the ratio of potential conductor is directly proportional to
i.e., for non-ohmic situations may be of difference accross its end, to the potential difference (V) across
following types: the current flowing through it its two ends provided all physical
(a) V vs I graph is non linear. remains constant are called ohmic conditions remain unchanged.
(b) Relation between potential difference conductors. V ∝ I or V = IR
and current depends on the sign of V V R is resistance of the conductor. S.I.
for the same absolute value of V. or = constant
I unit of resistance is ohm or W
(c) Ratio V is not constant. 1 W = 1 V/A
I
Electric Current & Drift velocity:
dq
I= = neAVd
dt

122 Physics–12
D:\EG_Physics-12_(26-06-2022)\Open_Files\Ch-3\Ch-3
\ 27-Jul-2022 Ved_Goswami Proof-4 Reader’s Sign _______________________ Date __________

REVISION CHART

Current density ( J )
It is the current per unit area of cross
section of the conductor.
Conductivity Temperature coefficient of  I  
The reciprocal of the resistivity of a J = or J = − neVd
resistance (a) A
material is called its conductivity (s) It is defined as the fractional change It is a vector quantity. It’s S.I. unit
1 in resistance for 1°C change in is A/m2
Thus, s = ρ
temperature. Relation between current density,
S.I. unit of conductivity is ohm–1 m– 1 resistivity and electricfield.
Rt − Ro  
or (W– 1m– 1) or Siemen metre– 1 written a=
R ot J = σE = Eρ
as Sm– 1

Resistance
ml ρl m
Resistivity: As R = 2 =     So, ρ = 2
ne τA A ne τ
The resistivity of a material is the resistance offered by a wire of this material of unit length and unit area of cross-section. It is
also known as specific resistance of the material
S.I. unit of resistivity is Wm (Ohm metre)

Internal Resistance of a Cell Conductance Electromotive Force (E.M.F.)


The resistance offered by the electrolyte The reciprocal of the resistance E.M.F. is defined as the work done per
of a cell, to the flow of current, between of a conductor is called its unit charge by the source, in taking the
its electrodes, is called the internal conductance. It’s S.I. unit is charge from its one terminal to the other.
resistance of the cell. ohm– 1 or Siemen. It is equal to the potential difference,
between the two terminals, of the cell,
when no current is drawn from it.
Relation between E.M.F., terminal potential difference and
internal resistance of a cell
When a cell of E.M.F. (E) and internal resistance r is connected
to an external resistance R.
Then, E =V + Ir, where V = IR Terminal potential difference
It is the potential drop across the terminals of a
E − V E − V  E − V cell when a current is being drawn from it.
or r= = = R
I V/R  V 

Current Electricity 123


D:\EG_Physics-12_(26-06-2022)\Open_Files\Ch-3\Ch-3
\ 27-Jul-2022 Ved_Goswami Proof-4 Reader’s Sign _______________________ Date __________

IMPORTANCE OF EACH TOPIC AND FREQUENTLY ASKED TYPES OF QUESTIONS

☞ Important Topics
1. Questions related to Drift Velocity. 2. Questions on Cells, EMF and Internal Resistance
3. Questions based on Kirchhoff’s Laws and Potentiometer
* Maximum weightage is of Drift velocity of Electrons and Resistivitiy.

1. The resistance of a platinum wire at a point 0°C is 5.00 ohm and its resistance at steam point is 5.40Ω. When the
wire is immersed in a hot oil bath, the resistance becomes 5.80Ω. Catculate the temperature of the oil bath and
temperature coefficient of resistance of platinum.[Ans. a = 0.004°C–1; T = 200°C]
2. Calculate the equivalent resistance and current shown by the ammeter in the circuit 2Ω
diagram given. [Ans. R = 2Ω; I = 5A] 4Ω
4Ω 6Ω
1Ω 6Ω
3. A 16 ohm resistance wire is bent to form a square. A source of emf 9 volt is connected
across one of its sides. Calculate the potential difference across anyone of its diagonals. A
[Ans. 1 V] 12V 12V

4. A length of uniform ‘heating wire’ made of nichrome has a resistance 72Ω. At what rate is the energy dissipated if
a potential difference of 120V is applied across (a) full length of wire (b) half the length of wire (wire is cut into
two). Why is it is not advisable to use the half length of wire?
[Ans. (a) 200W (b) 400W. 400W >> 200W but since current becomes
large so it is not advisable to use half the length]
5. Potential difference across terminals of a cell are measured (in volt) 1.6
1.4 A
against different current (in ampere) flowing through the cell. A graph 1.2
was drawn which was a straight line ABC. Using the data given in the 0.8
B

graph, determine (i) the emf. (ii) The internal resistance of the cell.
P.D. (volt)
C
0.4
[Ans. (i) emf = l.4V, (ii) r = 5Ω]
.04 .08 .12 .16 .20 .24 .28
I (ampere)

6. For the circuit given below, find the potential difference between points B and D.
A B
2V, 2Ω

1V, 1Ω 1V, 1Ω

3V, 3Ω
C D
           [Ans. 1.46 Volts]
7. A copper wire of length 3m and radius r is nickel plated till its radius becomes 2r. What would be the effective
resistance of the wire, if specific resistance of copper and nickel are rc and rn respectively?
3ρn ρc
[Ans. R= ]
πr (3ρc + ρn )
2

8. Given two resistors X and Y whose resistances are to be X


A
Y
A
determined using an ammeter of resistance 0.5Ω and a
voltmeter of resistance 20 kΩ. It is known that X is in the (i) (ii)
range of a few ohms, while Y is in the range of several V V

thousand ohm. In each case, which of the two connection shown should be chosen for resistance measurement?
[Ans. Small resistance: X will be preferred; large resistance: Y will be preferred]
9. A voltmeter with resistance 500W is used to measure the emf of a cell of internal resistance 4W, What will be the
percentage error in the reading of the voltmeter? [Ans. 0.8%]

124 Physics–12
D:\EG_Physics-12_(26-06-2022)\Open_Files\Ch-3\Ch-3
\ 27-Jul-2022 Ved_Goswami Proof-3 Reader’s Sign _______________________ Date __________

ASSIGNMENT
I. Objective Type Questions (1 Mark)
1. Multiple choice questions:
(i) The temperature dependence of resistivity r(T) of semiconductors, insulators and metals is significantly based
on the following factors:
(a) Number of charge carriers can change with temperature T.
(b) Time-interval between two successive collisions can depend on T.
(c) Length of material can be a function of T.
(d) Mass of carriers is a function of T.
(ii) If the electric current in a lamp decreases by 5%, then the power output decreases by
(a) 25% (b) 5% (c) 10% (d) 20%
2. Fill in the blanks:
(i) The equivalent resistance of a network .......... due to parallel grouping of resistances and becomes .......... .
(ii) The terminal potential difference of a cell is zero when the potential drop across its internal resistance is ..... .
II. Very Short Answer Type Questions (1 Mark)
3. Give the nature of graph for (i) ohmic (ii) non-ohmic circuit elements.
4. Establish relation between quantities j , s and E .
III. Short Answer Type Questions-I (2 Marks)
5. Distinguish between E.M.F. and terminal potential difference of a cell.
6. Show that to minimise the power loss in the transmission cables connecting the power stations to homes, it is
necessary to have the connecting wires carrying current at enormous high value of voltage.
7. A parallel combination of two cells of emf’s E1 and E2, internal resistances r1 and r2 is used to supply current to a
load of resistance R. Find an expression for the equivalent emf and internal resistance of the given combination.
IV. Short Answer Type Questions-II (3 Marks)
8. Use Kirchhoff’s rule to calculate the current I1 of the circuit given below:
30 Ω
E F
I1
20 Ω 20 V
A C
I3
I2 20 Ω
B D
80 V

9. State Kirchhoff’s laws of electrical network. Using these laws to obtain balance condition for Wheatstone
bridge.


Current Electricity 125


D:\EG_Physics-12_(26-06-2022)\Open_Files\Ch-4\Ch-4
\ 27-Jul-2022 Ved_Goswami Proof-4 Reader’s Sign _______________________ Date __________

Topics Covered
4 Moving Charges and Magnetism

4.1 Oersted’s Experiment, Magnetic Field due to a Current Carrying Conductors


4.2 Force on a Charge in an Electric and Magnetic Field 4.3 Ampere’s Circuital Law
4.4 Force on a Current-carrying Conductor and It’s Applications

C hapter map
Moving Charges and
Magnetism

Magnetic Field on Magnetic force Magnetic effect due


Motion in a
the Axis of a circular On a current carrying to current element
magnetic field
current loop. conductor

•A  mpere’s circuital law Biot Savart Law


•T  he Solenoid
• F orce between two parallel currents wires
•T  orque on magnetic current & Magnetic dipole

Topic 1. Oersted’s Experiment, Magnetic Field due to a Current Carrying


Conductors
•• Oersted’s Experiment: In 1820 Oersted observed that I
when a magnetic compass is placed near to a current P
carrying conductor, it deflects which indicates that a
A r
current carrying conductor produces a magnetic field θ
in the space surrounding it. He also discovered that B
when a conductor carrying in the direction South to
North is placed over a magnetic compass, the needle of
the compass deflects towards West. It is also known as
‘SNOW’ rule.
Combining all these four factors
•• Biot-Savart Law: The magnitude of the magnetic field
 I dl sin θ µ 0 Idl sin θ
dB due to a small current carrying element is: dB ∝ = (for vacuum)
r 2 4π r 2
(i) Directly proportional to the current I flowing through µ0 is constant, called permeability of free space.
it, dB ∝ I 
(ii) Directly proportional to the length dl of the current  µ 0 Idl × r
In vector form dB =
element, dB ∝ dl 4π r 3
 
(iii) Directly proportional to i.e., dB ∝ sin q. Direction of dB : Direction of dB the magnetic induction
 
(iv) Inversely proportional to the square of the distance r is same as that of Idl × r . It can be determined by
1 Maxwell right hand cork screw rule. dB is perpendicular
of the point P from the current element dB ∝
r2 to both Idl and r.
126
D:\EG_Physics-12_(26-06-2022)\Open_Files\Ch-4\Ch-4
\ 27-Jul-2022 Ved_Goswami Proof-4 Reader’s Sign _______________________ Date __________

Special case: (i) If q = 0°, dB = 0 x – distance of the point from the centre of the
Magnetic field is zero at all points on the axis of the loop.
current element.
(ii) If q = 90°
I a
µ Idl
dB = 0 2
4π r
dB is maximum. O
P

I
x

r µ 0 2nIπa 2
B = 3
4π ( a 2 + x 2 ) 2
If point P is very far away i.e., x >>> a
•• SI unit of magnetic field is Tesla or (T) 
µ 0 2nIA 2µ 0 m
as area of the loop A = pa2 then B = =
•• Magnetic field at the centre of a circular current 4πx3 4πx3
carrying coil. Magnetic dipole moment of a current carrying loop m
If there are n turns then magnetic field will be equal to = nIA.
µ0 nI •• Variation of magnetic field along the axis of a circular
B= current carrying loop:
2r
Magnetic field at the centre of current carrying loop is B
nearly uniform.
Circular current carrying coil acts as a dipole with one
face acting as north pole and the other as south pole.

S
Clockwise direction
N
Anticlockwise direction
O
Distance from the centre along axis
•• If current I is flowing through a part of a circular coil of
radius a as shown in the figure, then the magnetic field
of current of current
South Pole North Pole due to this portion of the coil subtending an angle q at
•• Magnetic field pattern due to circular current carrying its centre is
loop µ Iθ
B= 0
N 4πa
Coil
I

I O a

S
•• Magnetic field on the axis of a circular current
carrying coil: A current loop behaves like a magnetic •• Magnetic field at the centre of a semicircular current
dipole. carrying loop
Let n — no. of turns 1  µ0I 
I – current flowing B=  
2  2a 
a – radius of the loop O

Moving Charges and Magnetism 127


D:\EG_Physics-12_(26-06-2022)\Open_Files\Ch-4\Ch-4
\ 27-Jul-2022 Ved_Goswami Proof-4 Reader’s Sign _______________________ Date __________

EXERCISE 4.1
I. Objective Type Questions (1 Mark) (ii) Anticlockwise direction (b) nIA
1. Choose the correct answers from the given options of current
(MCQs). (iii) Magnetic field at the centre (c) South pole
(i) Biot-Savart law indicates that the moving of a circular current carrying
electrons (with velocity v) produce a magnetic coil
field B such that µ nI
(iv) Magnetic dipole moment (d) 0
(a) B is perpendicular to velocity v. 2r
of a current carrying loop
(b) B is parallel to v.
For questions numbers 5 two statements are given-one
(c) it obeys inverse cube law. labelled Assertion (A) and the other labelled Reason
(d) it is along the line joining the electron and (R). Select the correct answer to these questions from
point of observation. the codes (a), (b), (c) and (d) as given below.
(ii) A circular of radius R carries a current I. The (a) Both A and R are true and R is the correct
magnetic field at its centre is B. At what distance explanation of A
from the centre on the axis of the coil, the
(b) Both A and R are true but R is NOT the correct
B
magnetic field will be ? explanation of A
8
(c) A is true but R is false
(a) 2R (b) 3R (c) 2R (d) 3R
(d) A is false and R is also false
(iii) An electron is released from rest in a region
of uniform electric and magnetic fields acting 5. Assertion (A):  In a current acrrying conductor,
 
parallel to each other. The electron will[AI 2020] direction of dB is the direction of the vector dl × dr .
(a) move in a straight line. Reason (R): It is given by right hand screw rule
(b) move in a circle. II. Very Short Answer Type Questions (1 Mark)
(c) remain stationary 1. A circular coil, of radius R, carries a current I. Write
(d) move in a helical path. the expression for the magnetic field due to this coil
(iv) The magnetic field at the centre of a current at its centre. Find out the direction of the magnetic
carrying circular loop of radius R is B 1. The field. [AI 2008 C]
magnetic field at a point on its axis at a distance 2. A circular coil of closely wound N turns and radius
R from the center of the loop is B2. Then the ratio r carries a current I. Write the expressions for the
(B1/B2) is [CBSE 2022] following:
1 (i) The magnetic field at its centre
(a) 2 2 (b) (c) 2 (d) 2 (ii) The magnetic moment of this coil.  [AI 2012]
2
2. Fill in the blanks. III. Short Answer Type Questions-I (2 Marks)
(i) Magnetic dipole moment of a current carrying 3. A straight wire carrying a current of 12A is bent into a
loop is .......... . semi-circular arc of radius 2.0 cm as shown in figure
(a). Consider the magnetic field B at the centre of the
1  µ I
(ii)  0  is the magnetic field at the centre of a arc. (i) What is the magnetic field due to the straight
2  2a 
segments? (ii) In what way the contribution to B from
.......... loop.
3. State True or False

(i) According to Biot Savart Law, magnetic field dB
is directly proportional to cos q.
(a) (b)
(ii) SI unit of magnetic field is weber.
4. Match the Columns the semicircle differs from that of a circular loop and
Column-I Column-II in what way does it resemble? (iii) Would your answer
be different if the wire were bent into a semi-circular
(i) Clockwise direction of (a) North pole
arc of the same radius but in the opposite way as
current shown in figure (b)?

128 Physics–12
D:\EG_Physics-12_(26-06-2022)\Open_Files\Ch-4\Ch-4
\ 27-Jul-2022 Ved_Goswami Proof-4 Reader’s Sign _______________________ Date __________

^
7. Two identical circular loops P and Q, each of radius
4. An element Dl = ∆x i is placed at the origin and r and carrying current I and 2I respectively are lying
carries a large current I = 10 A (see figure). What is in parallel planes such that they have a common axis.
the magnetic field on the y-axis at a distance of 0.5 The direction of current in both the loops is clockwise
m. Dx = 1 cm. [NCERT] as seen from O which is equidistant from the both
loops. Find the magnitude of the net magnetic field
y

P at point O. [AI 2012]


0.5 m

x r r
∆x
O
5. (i) A current ‘I’ enters a uniform circular loop of
radius ‘R’ at point M and flows out at N as shown
in the figure.
I 2I
Obtain the net magnetic field at the centre of the
loop.[Delhi 2015] 8. Two small identical circular loops, marked (1) and
(2), carrying equal currents, are placed with the
N
geographical axes perpendicular to each other as
shown in the figure. Find the magnitude and direction
I2 of the net magnetic field produced at the point O.
O A [Delhi 2008, Foreign 2013]
B
1
M O
I
C1
I1
(ii) Two co-axial circular loops, L1 and L2 of radii
3 cm and 4 cm are placed as shown in figure.
What should be the magnitude and direction of C2 2

the current in the loop L2 so that the net magnetic I

field at the point O be zero? [CBSE 2008] 9. Using Biot-Savart’s law, derive an expression for
magnetic field at any point on axial line of a current
4 cm 3 cm carrying circular loop. Hence, find magnitude of
L1 magnetic field intensity at the centre of circular coil.
O [CBSE S.P. 2019-20]
I1 = 1A L2
10. (i) Write an expression of magnetic moment
associated with a current (I) carrying circular coil
IV. Short Answer Type Questions-II (3 Marks) of radius r having N turns.
6. Two identical circular wires P and Q each of radius R and (ii) Consider the above mentioned coil placed in
carrying current ‘I’ are kept in perpendicular planes such YZ plane with its centre at the origin. Derive
that they have a common centre as shown in the figure. expression for the value of magnetic field due to
Find the magnitude and direction of the net magnetic it at point (x, 0, 0). [Delhi, AI 2020]
field at the common centre of the two coils.
[Delhi 2012] V. Long Answer Type Questions (5 Marks)
Q 11. (i) Express Biot-Savart law in the vector form.
(ii) Use Biot-Savart law to obtain the expression
I for the magnetic field at an axial point, distance
d from the centre of a circular coil of radius R
P
carrying current I.
(iii) Also, find the ratio of the magnitudes of the
I magnetic field of this coil at the centre and at an
axial point for which x = R 3. [AI 2016]

Moving Charges and Magnetism 129


D:\EG_Physics-12_(26-06-2022)\Open_Files\Ch-4\Ch-4
\ 27-Jul-2022 Ved_Goswami Proof-4 Reader’s Sign _______________________ Date __________

12. (i) State Biot-Savart law, giving the mathematical point on its axis at a distance x from its centre.
expression for it. (ii) A conducting rod of length 2 m is placed on a
(ii) Use Biot-Savart law to derive the expression for horizontal table in north-south direction. It carries
the magnetic field due to a circular coil carrying a current of 5 A from south to north. Find the
current at a point along its axis. direction and magnitude of the magnetic force
 [Delhi 2011, AI 2013] acting on the rod. Given that the Earth’s magnetic
(iii) How does a circular loop carrying current behave field at the place is 0.6 ×10–4 T and angle of dip is
as a magnet? [Delhi 2011, AI 2013] π
.[AI 2020]
13. (i) A circular loop of radius R carries a current I. 6
Obtain an expression for the magnetic field at a

Answers 4.1
I. Objective Type Questions  
The direction of dB is same as that of Idl × r
1. (i) (a) (ii) (b) (iii) (a) (iv) (a)
^ ^ ^ ^ ^
2. (i) nIA (ii) semicircular i.e.,  i ×
j  and i × j = k
 
3. (i) False (ii) False
dB is along positive z-direction ½
4. (i)-(c) (ii)-(a) (iii)-(d) (iv)-(b) 5. (i) Let k be the resistance per unit length then
5. (a) Res. of arc MAN = R1 = kl1
II. Very Short Answer Type Questions Res. of arc MBN = R2 = kl2
1. The magnetic field due to a circular coil of radius R, As two arc connected in parallel.
µ0 I So I1R1 = I2R2
carrying current I at its centre is B =
2R I1kl1 = I2kl2 or I1l1 = I2l2 ...(1)
The direction of magnetic field is perpendicular to plane Magnetic field induction at O due to arc MAN
of coil, directed outward if current is anticlockwise and µ Iθ µ I l µ Il
inward if current is clockwise. B1 = 0 1 1 = 0 1 × 1 = 0 121
4π r 4π r r 4πr
2. (i) The magnetic field at the centre due to a circular coil It is directed upward
µ NI
of N turns and radius r carrying current I is B = 0 Magnetic field at O due to Arc MBN
2r
(ii) The magnetic moment of the coil is µ Iθ µ I l µ Il
B2 = 0 2 2 = 0 2 × 2 = 0 22 2
m = NIA = NI × pr2 4π r 4π r r 4πr
3. (i) Magnetic field due to straight segments It is directed into the paper.
 will
 be
zero at the centre of the axis. Because dl and r are But I1l1 = I2l2 [By (1)]
parallel to each other. ½ So B1 = B2
(ii) Magnetic field due to semicircle will be half of the So B1 and B2 are equal in magnitude but opposite in
magnetic field produced by the circular loop of same direction.
radius and carrying same current. ½ 
So Bnet = B1 – B2 = 0. 1
(iii) If the wires were bent into semicircular arc of (ii) Magnetic field at O due to current I1
the same radius but in opposite way the direction
of B will be opposite to the earlier direction, but µ I1 2πa12
B1 = 0 3
; a1 = 3 cm, x1 = 4 cm
4π 2
magnitude will be same. 1
(
a1 + x12 2
)
µ Idl sin θ
4. dB = 0  ½ Magnetic field at O due to current I2
4π r2
I = 10A, dl = 1 cm = 10–2 m, r = 0.5 m µ 0 I 2 2π a2
2
( )
; a2 = 4 cm, x2 = 3 cm
B2 = 3
4π 2
So, dB =
4π × 10 −7 × 10 × 10 −2 sin 90° (
a2 + x2 2 2
)
4π (.5) 2
When observed from O as the current in L1 is
= 4 × 10–8 T 1 anticlockwise, the face of the coil facing the observer

130 Physics–12
D:\EG_Physics-12_(26-06-2022)\Open_Files\Ch-4\Ch-4
\ 27-Jul-2022 Ved_Goswami Proof-4 Reader’s Sign _______________________ Date __________

is acting as north pole so the current in coil L2 facing  µ0r 2I


observer stationed at O should also be N-pole. 7. BP = 3
i.e., current should be in anticlockwise direction. 2(r 2 + x 2 ) 2
 
But B1 = B2 Towards the centre of the coil P
 µ 0 r 2 2I µ 0 Ir 2
µ 2π I1a12 µ0 2πI 2 a22 BQ = =  1
3 3
0 = 2
4π 3
4π 3 2(r + (r + x2 ) 2 2
x2 ) 2
(a 2
1 + x12 ) 2
(a2
2 + x22 ) 2
Towards the centre of coil Q
3   µ 0 Ir 2

I2
=
a12
×
( a22 + )
2 2
x2 Bnet = BP + BQ =
2
3
,
I1 3
a22 2(r + x2 ) 2
( a12 + x12 ) 2
towards the centre of the coil Q. 1
3 P x x Q
I2 (.03) 2 (.04) 2 + (.03) 2  2
= × 
I1 3
(.04) 2 r r
(.03) 2 + (.04) 2  2
  BP BQ
    1
(.03) 2 9 O
= =
(.04) 2 16 2I
I
9
I2 = I1  1 8. Let r → radius of loops
16
Magnetic field at O due to the current in coil 1
6. 1P = IQ = I, RP = RQ = R  2µ 0 Iπr 2 µ 0 Ir 2
µ0 I B1 = 3
= 3
 ½
So B= 4π 2 2 2 2 2 2
2R (r + x ) 2(r + x )
µ0I along OC1
So BP = B Q =  1
2R B
B2

Q
B I θ O
C1 B1
I Bp

θ C2
P BQ
I I
Magnetic field at O due to the current in coil 2
 µ 0 Ir 2 
B2 = 3
along C2 O  ½
Angle between BP and BQ is 90° 2(r 2 + x 2 ) 2
So their resultant  
B1 = B2
B= B2P + BQ2
So B12 + B22 2=
B= = B12 or B 2B1
µ I µ I
= 2 0 = 0  1
2R 2R µ 0 Ir 2

\ B= 3
B 2 (r 2 + x 2 ) 2
tan q = P = 1
BQ
B2
So q = 45° tan q = = 1, θ = 45°  1
B1
µ0 I
So, net field = at an angle of 45° with either field. 
2R Resultant magnetic field is at an angle of 45° with B1.
 1

Moving Charges and Magnetism 131


D:\EG_Physics-12_(26-06-2022)\Open_Files\Ch-4\Ch-4
\ 27-Jul-2022 Ved_Goswami Proof-4 Reader’s Sign _______________________ Date __________

9. According to Biot-Savart’s law, magnetic field due to 



\ B at P ⇒ B = ∫ dBsin θ
a current element is given by

 2 πr
 µ0 idl × i µ0 I µI r
dB = where r = x 2 + a 2 = sin θ ∫ dl = 0 2 × × ( 2πr )
4π r 2 4πr 2
0
4πr r1
µ idl sin 90°  µ 0 Ir 2
\ dB = 0 2 ½
⇒ B = i
4π x + a 2
( )
3
 ½ 2
2 r +x 2 2

Idl  
 µ Idl × r
0
r 11. (i) dB = 4π 3  ½
a r
N dBy Q dB (ii) Magnetic field at axis of a circular current
dBx carrying coil: Consider a circular loop of wire of
 
M
O x P  radius ‘R’ and carrying current I. Let the plane of
dBy N
1 dB the loop be ^ to plane of paper.
r
B A dB cos φ
 θ
dB
And direction 
 of dB is perpendicular to the plane I R S

containing Idl and r . φ
Q
 φ dB sin φ ½
Resolving dB along the x-axis and y-axis. O d φ dB′ sin φ
P
φ
dBx = dB sin q   and  dBy = dB cos q S
Q′
taking the contribution of whole current loop we get
dB′
Bx = ∫ dBx = ∫ dB sin θ C D dB′ cos φ

µ Idl a Consider current element dl at the top of the loop


= ∫ 4π0 x 2 + a 2 2
x +a 2

µ 0 Idl sin 90° µ 0 Idl
dB = =  1
µ0 I µ0 ia × 2πa 4π S2 4πS2
Bx =
4 π ( x + a 2 )3 / 2 ∫ dl = 4π The direction of dB is along PQ.
(x + a2 )
2 2 3/ 2

Consider another current element diametrically


opposite to AB.
And By = ∫ dB = ∫ dB cosθ = 0
y
µ 0 Idl sin 90° µ 0 Idl
µ0 2IA dB′ = =

\ BP = B2x + B2y = Bx = 4π S2 4πS2
(
4π x 2 + a 2 3 / 2 ) This will be directed along PQ′.
 µ 

∴ Bp = 0 2
2m
(
∵ m = IA ½ )

dB = dB′

( )
3/ 2
4π x + a 2

For centre x = 0 As their magnitudes are equal so the vertical
component will cancel each other and the horizontal
 µ 2Iπa 2  I 

\ | B0 | = 0 = µ0   components will get added up.
4π a 3
 2a  µ Idl


in the direction of m  1 B = ∫ dB sin φ = ∫ o 2 sin φ
4πS
[CBSE Marking Scheme, 2019-20] µ Idl R µ IR
 = ∫ 0 3 = 0 3 ∫ dl
10. (i) Magnetic moment M = NI πr 2 n ( ) 4πS S 4πs
(ii) Refer fig. µ 0 IR 2µ 0 IR 2 π
Sol. 9, Magnetic field at point P(x, 0, 0) = × 2 πR =
due to Idl 4πS3 4πS3
 µ Idl sin 90°
 (S2 = R2 + d2)
dB = 0 along PQ
4π r12 2µ 0 IR 2 π
 = 3
along the axis.
For entire coil ∫ dS cos θ = 0 4π ( R + 2
d 2 )2

132 Physics–12
D:\EG_Physics-12_(26-06-2022)\Open_Files\Ch-4\Ch-4
\ 27-Jul-2022 Ved_Goswami Proof-4 Reader’s Sign _______________________ Date __________


2µ 0 IA µ Idl sin θ  µ 0 Idl × r
= 3
where A = πR 2  1 dB = 0 , dB =  ½
2 2 2 4π r2 4π r 3
4π ( R + d )
(ii) See Answer 11 (ii). 2
= Area of the loop
(iii) Using right hand grip rule (thumb rule) we see that
IA = dipole moment of current carrying magnetic field line due to a circular current carrying
loop. loop will come out of its one face and enter into the
(iii) At d = 3R other face depending upon the direction of current. So
2µ 0 IπR 2 2µ 0 IπR 2 one of its face is acting as a north pole and the other
Baxis = = as a south pole. Also a compass needle placed close
3 3
4π [( 2R ) ] 2 by will also get deflected. So we can say that a current
2
4π (R 2 + 3R 2 ) 2
carrying circular loop behaves as a magnet. 1
2µ 0 IπR 2 µ 0 I 13. (i) Refer to Solution 11 (ii).
= =  ½
4π 8R 3 16 R (ii) Current, i = 5 A
µ0 I Magnetic field of earth
Bcentre =  ½
2R Be = 0.6 × 10–4 T
Bcentre µ I 16 R π
= 0 × =8 1 Dip angle q =
Baxis 2R µ0I 6
Length of the rod is 2 m
12. (i) Biot Savart Law: It is valid only for small current
We need to calculate the horizontal component of
element. It can be used to find the magnetic field
the earth magnetic field using formula of horizontal
due to current carrying conductor. 1
component
Consider a small current element of length dl.
BH = Be cos q
Magnetic field due to current element is directly
proportional to Idl. dB ∝ Idl. Also dB ∝ sin q Put the value into the formula
where q = angle between position vector and current π
BH = 0.6 × 10–4 cos
element. B = 0.0000599 T 6
H
I
P BH = 5.99 × 10–5 T
A r We need to calculate the magnitude of the magnetic
θ
B force
Using formula of magnetic force
FB = Bil
Put the value into the formula
1
And dB ∝ FB = 5.99 × 10–5 × 5 × 2
r2
FB = 0.000599 N
Idl sin θ FB = 5.99 × 10–4 N
\ dB ∝  ½
r2 According to the right hand rule,
kIdl sin θ The direction of magnetic force is perpendicular to
dB =
r2 the direction of current.
Value of k depends upon system of units chosen and So, The direction of magnetic force towards west
medium surrounding the conductor because the direction of current is north to east.
µ0 Hence, the magnitude of the magnetic force is
For vacuum k =
4π 5.99 × 10–4 N.

Topic 2. Force on a Charge in an Electric and Magnetic Field


•• Consider a charge q moving with velocity v at angle q F ∝ v sin q
to the magnetic field B. The force F experienced by the F∝q
charge is F ∝ qvB sin q
F∝B F = kqvB sin q

Moving Charges and Magnetism 133


D:\EG_Physics-12_(26-06-2022)\Open_Files\Ch-4\Ch-4
\ 27-Jul-2022 Ved_Goswami Proof-4 Reader’s Sign _______________________ Date __________

v (ii) If v ^ B. i.e., q = 90°


F will be maximum
F = qvB
F^v
θ Whenever F ^ v, path will be circular.
B
× × × ×
F = qvB sin q (when k = 1) v
  
F = q v×B ( ) × ×
O
× ×
q
×

Direction of force experienced is given by cross product × × ×F × ×


rule
    × × × × ×
F ⊥ v and F ⊥ B
O = Centre
Special cases: q = charge (proton)
(i) If v = 0, i.e., charge is at rest then × × v × × × ×
F=0
× × × × × ×
(ii) If v || B i.e., q = 0° or q = 180° (parallel/anti parallel) O
q F O
F=0 × × × × × ×
(iii) If q = 90° i.e., v ^ B × × × × × ×
Then F = qvB
q = –ve charge
F = qvB sin 90° = qvB. O = Centre
(max. value of force experienced) The particle will describe a circular path. The
•• SI Unit of magnetic field: If a charge of 1C is moving centripetal force is provided by the magnetic Lorentz
with a speed of 1 m/s at an angle of 90° to the magnetic force.
field and it experiences a force of 1N, then magnetic mv 2
field is 1 Tesla. = qvB
r     
q = 1C, v = 1 m/s where m = mass of charge
q = 90°, F = 1N mv 2 mv
1=1×1×B×1 r = radius of circle = =
q vB qB
B = 1 Tesla = 1T Time period = time taken to complete one circular
Magnetic field of Earth = 10–4 Tesla path
Smaller unit of magnetic field = 1 Gauss = 1G Circumference 2πr 2π (mv) 2πm
  = = = =
1G = 10–4T Speed v v qB qB
•• Total Lorentz force: When both electric and magnetic Note: Time period is independent of v and r i.e., if particle
fields are present then moves faster, the radius is larger
  
F = Fe + Fm v
   i.e., r is always constant.


F = q E + q v ×B ( ) 1 qB
Frequency = =
[When both fields exist]. T 2πm
•• Shape/Trajectory of Path Followed by a Charge in a
Magnetic Field
(i) If v || B or v is anti-parallel to B
i.e., q = 0° or 180°,
then F = qvB sin 0° or F = qvB sin 180°
F=0
Path will not change. Charge particle will continue to (iii) If v is inclined to B at an angle q
move along line of force i.e. path will be a straight v11 = v cos q
line. It will always remain parallel to B.
v^ = v sin q

134 Physics–12
D:\EG_Physics-12_(26-06-2022)\Open_Files\Ch-4\Ch-4
\ 27-Jul-2022 Ved_Goswami Proof-4 Reader’s Sign _______________________ Date __________

On v11, there is no force acting. E


But on v^ = v sin q, there is force acting. vB = E ⇒ v =
B
It will describe a circular path due to v sin q component This concept can be used for velocity selection.
but will move forward due to v cos q component. •• Magnetic field pattern due to straight current
Hence, it will describe a spiral/helical path around carrying conductor:
the magnetic field.
Radius of the helix
mv 2⊥
= qv^ B
r
mv ⊥ mv sin θ
r= = ,
qB qB
2πr 2πmv sin θ 2πm I
T= = =
v= q Bv sin θ qB
Force on a current carrying conductor in a magnetic
Distance between two spirals = pitch
field: Consider a conductor of length ‘l’ and area of cross
Pitch of helix = linear distance travelled by charged section ‘A’ carrying current ‘I’ along the +ve X direction.
particle in direction of field in one time period B
2πm
Pitch = v11 × T = v cos θ × T = v cos θ ×
qB
2πmv cos θ
= θ
qB +X
I
•• Motion of Charged Particle in (Crossed Electric and
Magnetic Field) Perpendicular Electric and Magnetic
Field: Suppose a beam of electron moving with velocity Magnetic field ‘B’ is applied at an angle q to the direction
v enters a region in which crossed (^) electrical and of current.
magnetic fields exist. Let n = number of free electron per unit volume
+ + + + + + + + + Due to Total number of free electrons in conductor
electrical field
× × ×   = n × volume = nAl
e Drift velocity of electrons will be along –ve x-axis.
× × × ×   
× × ×
Force experienced by 1 electron, f = e ( Vd × B)
  
Due to
magnetic field
Total force on conductor: F = nAle ( Vd × B) where
– – – – – – – – –
  e = charge on electron

Fm = qe (vB)[F=e ln E] 
e ( Vd × B
= qeA 
) = Il × B = Il B sin θ ( )
For no deviation, Fm = Fe (opp. directions) Special Cases: (i) If q = 0°, F = 0 (ii) If q = 90°, F = IlB,
qevB = qeE F = maximum

EXERCISE 4.2
I. Objective Type Questions (1 Mark) (c) They necessarily represent a particle-
1. Choose the correct answers from the given options antiparticle pair.
(MCQs). (d) The charge to mass ratio satisfy
(i) Two charged particles traverse identical helical
paths in a completely opposite sense in a uniform
e + e = 0.
m1 m 2() ()
(ii) An electron is projected with uniform velocity
magnetic field B = B k . 0
along the axis of a current-carrying long solenoid.
(a) They have equal z-components of momenta. Which of the following is true?
(b) They must have equal charges. (a) Electron will be accelerated along the axis.

Moving Charges and Magnetism 135


D:\EG_Physics-12_(26-06-2022)\Open_Files\Ch-4\Ch-4
\ 27-Jul-2022 Ved_Goswami Proof-4 Reader’s Sign _______________________ Date __________

(b) The electron path will be circular about the (vii) A current carrying wire kept in a uniform magnetic
axis. field, will experience a maximum force when it
(c) The electron will experience a force at 45° to is  [CBSE 2022]
the axis and hence execute a helical path. (a) perpendicular to the magnetic field
(d) The electron will continue to move with (b) parallel to the magnetic field
uniform velocity along the axis of the (c) at an angle of 45° to the magnetic field
solenoid.
(d) at an angle of 60° to the magnetic field.
(iii) A cubical region of space is filled with some
uniform electric and magnetic fields. An electron (viii) A straight conducting rod of length l and mass
enters the cube across one of its faces with velocity m is suspended in a horizontal plane by a pair of
v, and a positron enters via opposite face with flexible strings in a magnetic field of magnitude
velocity –­v. At this instant, B. To remove the tension in the supporting strings,
the magnitude of the current in the wire is
(a) the electric forces on both the particles cause
identical accelerations. [CBSE 2022]
(b) the magnetic forces on both the particles cause mgB mgl mg lB
equal accelerations. (a) (b) (c) (d) mg
l B lB
(c) both particles gain or loose the energy at same
(ix) A proton and an alpha particle move in circular
rate.
orbits in a uniform magnetic field. Their speeds
(d) the motion of centre of mass (CM) is
are in the ratio of 9 : 4. The ratio of radii of their
determined by ‘B’ alone.
 rp 
(iv) Time period of a charged particle undergoing a circular orbits   is [CBSE 2022]
circular motion in a uniform magnetic field is  ralpha 
independent of [CBSE S.P. 2019-20]
3 4 8 9
(a) speed of the particle (a) (b) (c) (d)
4 3 9 8
(b) mass of the particle
(c) charge of the particle 2. Fill in the blanks.
(d) magnetic field (i) When a particle of charge q and mass m moving
(v) A charge particle after being accelerated through with velocity v, enters a magnetic field B at right
a potential difference V enters in a uniform angles to the direction of the field, the particle
magnetic field and moves in a circle of radius r. If takes a circular path of radius .......... and time
V is doubled, the radius of the circle will become period of its oscillation is .......... .
[Delhi 2020] 
(ii) A positive particle enters the magnetic field B

(a) 2 r (b) 2 r (c) 4 r (d) r/ 2 perpendicularly with a velocity v . The Lorentz
(vi) Two wires carrying currents I1 and I2 lie, one force will act on it in a direction .......... .
slightly above the other, in a horizontal plane as For questions numbers 3 two statements are given-one
shown in figure. The region of vertically upward labelled Assertion (A) and the other labelled Reason
strongest magnetic field is [CBSE 2022] (R). Select the correct answer to these questions from
the codes (a), (b), (c) and (d) as given below.
II I
I2 (a) Both A and R are true and R is the correct
explanation of A

I1 (b) Both A and R are true but R is NOT the correct


explanation of A
(c) A is true but R is false
(d) A is false and R is also false
3. Assertion (A): Neutron can’t be accelerated by
III IV
cyclotron.
Reason (R): This is because neutrons are electrically
neutral.
(a) I (b) II (c) III (d) IV
136 Physics–12
D:\EG_Physics-12_(26-06-2022)\Open_Files\Ch-4\Ch-4
\ 27-Jul-2022 Ved_Goswami Proof-4 Reader’s Sign _______________________ Date __________

II. Very Short Answer Type Questions (1 Mark) 9. Which one of the following will have the minimum
1. If the magnetic field is parallel to the positive y-axis frequency of revolution, when projected with the
and the charged particle is moving along the positive same velocity (v) perpendicular to the magnetic field
x-axis in given figure which way would the Lorentz (B) (i) (a-particle) and (ii) b-particle.
force be for (a) an electron (negative charge) (b) a [Delhi 2002 C, AI 2015]
proton (positive charge). [NCERT]
z y 10. A particle of charge ‘q’ and mass ‘m’ is moving with

velocity v . It is subjected to a uniform magnetic field

B directed perpendicular to its velocity. Show that it
× × × × × × × × × × × × × ×

describes a circular path. Write the expression of its


× × × × × × × × × × × × × ×
× × × × × × × × × × × × × ×
× × × × × B× × × × × × × × × radius. [Foreign 2012]
× × × × × × × × × × × × × ×
× × × × × × × × × × × × × × 11. Write the expression for Lorentz magnetic force on
x 
× × × × × × × × × × × × × × a particle of charge
 ‘q’ moving with velocity v in a
× × × ×
v
× × × × × × × × × × magnetic field B. Show that no work is done by this
force on the charged particle. [AI 2011]
2. An electron moving through a magnetic field does
not experience any force. Under what condition is 12. (i) A proton and an a particle enter into same
this possible? [AI 2005 C] magnetic field with equal kinetic energy. Find the
 ratio of the radii of the path described by them.
3. When a charged particle moving with velocity v is

subjected to magnetic field B , the force acting on it (ii) A proton and a deuteron having equal momenta
is non-zero. Would the particle gain any energy? enter in a region of uniform magnetic field at right
[AI 2008] angle to the direction of the field. Depict their
4. If a particle of charge q is moving with velocity v trajectories in the field. [Delhi 2013]
along X-axis and the magnetic field B is acting along 13. An a-particle and a proton of the same kinetic energy
  
Y-axis, use the expression F = q (v × B) to find the are in turn allowed to pass through a magnetic field


direction of force F acting on it. B , acting normal to the direction of motion of the
5. A proton and an electron travelling along parallel particles. Calculate the ratio of radii of the circular
paths enter a region of uniform magnetic field, acting paths described by them. [Delhi 2019]
perpendicular to their paths. Which of them will move 14. An a-particle is accelerated through a potential
in a circular path with higher frequency? difference of 10 kV and moves along x-axis. It enters
[Delhi 2018] in a region of uniform magnetic field B = 2 ×10–3 T
6. When a charge q is moving in the presence of electric acting along y-axis. Find the radius of its path. (Take
(E) and magnetic (B) fields which are perpendicular mass of a-particle = 6.4 ×10–27 kg). [AI 2020]
to each other and also perpendicular to the velocity
v of the particle, write the relation expressing v in IV. Short Answer Type Questions-II (3 Marks)

terms of E and B. [AI 2019] 15. A uniform magnetic field B is set up along the
7. An electron with charge –e and mass m travels at a positive x-axis. A particle of charge ‘q’ and mass ‘m’

speed v in a plane perpendicular to a magnetic field moving with a velocity v enters the field at the origin
of magnitude B. The electron follows a circular path in X-Y plane such that it has velocity components
of radius R. In a time, t, the electron travels halfway both along and perpendicular to the magnetic field

around the circle. What is the amount of work done B. Trace, giving reason, the trajectory followed by
by the magnetic field? [CBSE S.P. 2020-21] the particle. Find out the expression for the distance
III. Short Answer Type Questions-I (2 Marks) moved by the particle along the magnetic field in one
8. A narrow beam of protons and deutron, each having rotation. [AI 2015]

the same momentum, enters a region of uniform 16. (a) Write the expression for the force F acting on
magnetic field directed perpendicular to their a particle of mass m and charge q moving with
 
direction of momentum. What would be the ratio of velocity v in a magnetic field B . Under what
the radii of circular paths described by them? conditions will it move in (i) a circular path and
[Foreign 2011] (ii) a helical path?

Moving Charges and Magnetism 137


D:\EG_Physics-12_(26-06-2022)\Open_Files\Ch-4\Ch-4
\ 27-Jul-2022 Ved_Goswami Proof-4 Reader’s Sign _______________________ Date __________


(b) Show that the kinetic energy of the particle moving Y
in magnetic field remains constant.[Delhi 2017]
r
17. Derive an expression for the velocity vC of a positive I
X X
ions passing undeflected through a region where
crossed and uniform electric field E and magnetic
field B are simultaneously present. Y
Find magnitude and direction of electric field required

Draw and justify the trajectory of identical positive
so that the particle goes undeflected.
ions whose velocity has a magnitude less than |VC|.
[CBSE S.P. 2018-19]
OR
18. Consider a beam of charged particle moving with
A particle of mass m and charge q is in motion at
varying speeds. Show how crossed electric and
speed v parallel to a long straight conductor carrying magnetic field can be used to select charged particles
current I as shown here. of a particular velocity?

Answers 4.2
I. Objective Type Questions 8. mpvp = mDvD  (Given)
1. (i) (d) (ii) (d) (iii) (b, c & d) (iv) (a) mv
As r=  1
(v) (b) (vi) (b) (vii) (a) (viii) (c) qB
(ix) (d) rp mP vP q B q
= × D = D 1
rD
2. (i) mv , 2πm q p B mD vD q p
qB Bq
  9. Frequency of revolution when vel is ^ to B is
(ii) perpendicular to both B and v 1 qB
3. (a) f= =  ½
T 2πm
II. Very Short Answer Type Questions q
Freq. ∝
1. Direction can be found by Maxwell’s Right hand cork m
  
screw rule. F = q (v × B) For a-particle= =
q 2e e
 ½
m 4m p 2m p
 
(a) For an electron= i^, B B j^
as v v=
q e 1840 e
 For b-particle = =  ½
m mp mp
F = − e  v i^ × B j^ = − evB  i^ × ^
j
   
 q  q
 ^ As   >   So fb > fa ½
m β  m α
= − evB  k 
 
10. The force acting on a charged particle moving in a
Along –ve Z-axis.  
magnetic field is given by F = q ( v × B)  1

(b) For a proton force is along positive Z-axis. As force is ^ to velocity. So the particle will describe
2. This is possible when the electron is moving either a circular path. The necessary centripetal force will be
parallel or anti-parallel to the magnetic field. i.e., either provided by the magnetic Lorentz force.
q = 0° or q = 180°. mv 2
3. No, the particle will not gain any kinetic energy. When So = qvB, as θ = 90°
r
a charged particle moves in a magnetic field, the force mv
experienced by the charged particle is perpendicular to the r=  1
qB
velocity of the particle. When force is perpendicular to the  

velocity then it is also perpendicular to the displacement. 11. Lorentz magnetic force F = q (v × B)  ½
 
As W = F ⋅ S , so W = change in kinetic energy = 0. As the cross-product of two vectors is perpendicular to
4. Along z-axis. 5. Electron the plane containing the vectors. So F ^ v and F ^ B½
 
6. qE = qvB ⇒ v = E / B 7. zero as W = F ⋅ S = FS cos θ  ½

138 Physics–12
D:\EG_Physics-12_(26-06-2022)\Open_Files\Ch-4\Ch-4
\ 27-Jul-2022 Ved_Goswami Proof-4 Reader’s Sign _______________________ Date __________

As angle between force and displacement is 90°. So Radius of the moving charged particle in uniform
work done = 0 ½ magnetic field is given by
12. (i) KEa = KEP, qa = 2qp, ma = 4mp mv
r= ½
( qB) 
p2
  Ek = KE = So p= 2mE k  ½ Here B is uniform
2m
r m × v × q′
So =
rα m v qpB 2mα E k qp r′ m′ v′ q
= α α × = ×
rp qα B m p v p qα 2m p E k r 1 2 2q = 1

⇒ = × ×  ½
r′ 4 1 q 1
mα q p 1
= × = 2 × = 1 ½ 1 2 2qV
m p qα 2 14. qV = mv ⇒ v =
2 m
(ii) A moving charged particle experiences a force in a 
magnetic field according to Lorentz force (
 v = vi ⊥ B =B j
)
mv
Part of a circle \ Particle deflects along circular path of radius r =

× × × × × × ×
qB
r = m 2qV = 1 2mV
× × × × × × ×

qB m B q
× × × × × × ×

p 1 2 × 6.4 × 10−27 × 104


× × × × × × × r=
2 × 10−3 2 × 1.6 × 10−19
× × × × × × ×
1
r= −3
× 2 × 10−2 = 101 m = 10 m
2 × 10
d
× × × × × × ×

15. If v makes an angle q with x-axis


× × × × × × ×

d × × × × × × ×
v⊥ v
  
F = qv × B = qvB as θ = 90°
As F ^ v.
So it will describe a circular path and F will be θ
perpendicular to v and B both. v||
B

mv 2 mv
So = qvB ⇒ r =  ½ Component of v along B is v cos q = v||
r qB
Component of v perpendicular to B is
1 v sin q = v^
as (mv)P = (mv)D. So r ∝
q As v cos q || B so no force will act on v cos q. v sin q ^
As qP = qD so rP = rD ½ B, so it will describe a circular path due to v sin q. Net
13. A proton has mass, m and charge q, then path will be a helix. 1
Mass of alpha particle, m′ = 4m To find the distance covered by the particle in one
rotation.
Charge on alpha particle, q′ = 2q ½
Let us first find out the time taken to complete one
According to question, rotation.
KE of proton = KE of alpha particle
mv 2⊥
⇒ 0.5 mv2 = 0.5 m′v′2 = qv ⊥ B sin 90°
r
⇒ v2 = 4v′2 mv ⊥
v   r=
⇒ = (4)0.5 = 2 ½ qB
v′

Moving Charges and Magnetism 139


D:\EG_Physics-12_(26-06-2022)\Open_Files\Ch-4\Ch-4
\ 27-Jul-2022 Ved_Goswami Proof-4 Reader’s Sign _______________________ Date __________

2πr 2πm Since initial velocity is perpendicular to E, the trajectory


Time period = T = =  1 would be parabolic.
v⊥ Bq
 OR
Distance (d) covered along B in one rotation r m
v
q
v cos θ × 2πm
  = v cos q T =
qB
I
2πmv cos θ
d=  1 For the charged particle to more undeflected  ½
qB  

This is also called the pitch of the helix. Net force F = FE + Fm = 0
  
16. (a) F = q v × B  ( ) ½
 
FE = − Fm ...(1) ½
(i) For circular path v ^ B ½   
FE FE ′′ electric force, F m →magnetic force
(ii) For helical path, 0 < q < 90°, where q is the
angle between v and B. ½  
   FE = 1 − Fm  ... (2) ½
(b) As F = q v×B ( ) qE = Bqv sin 90° = Bqv
F ^v
 
So, work done = W = F ⋅ S = 0 , as q = 90° ½ E = vB ... (3) ½
W = DKE = 0 ½ µo I
B=  ... (4) 1
So, DK.E = 0 2πr
i.e., KE is constant. ½ Using (4) and (3)
17. E = Ej and B = Bk vµo I
Force on positive ion due to electric field FE = qEj ½ E =  ... (5) ½
2πr
Force due to magnetic field
Magnetic force Fm is towards wire.
FB = q (vc × B) ½
∴ Electric force and electric field should be away from
For passing undeflected,
the line.
FE = – FB
qEj = – q (vc × Bk)
This is possible only if
qvc × Bk = qvcBj or vc = (E/B)i ½
The trajectory would be as shown. ½ [CBSE Marking Scheme, 2018-19]
18. A beam of charged particle moving with varying speeds
is made to pass through a simultaneous crossed magnetic
and electric field of such strength and direction that they
exert equal forces in opposite direction. 2
Force due to electric field = Force due to magnetic field
eE = evB,
E
So v=  2
Justification: For positive ions with speed v < vc B
Force due to electric field = F′E = qE = FE So the electrons speed which satisfy this condition will
due to magnetic field F′B = qvB < FB since v < vc ½ go undeviated and all electrons with speed other than
Now forces are unbalanced, and hence, ion will E
equal to will get deflected. 1
experience an acceleration along E. ½ B

Topic 3. Ampere’s Circuital Law


•• Ampere’s circuital Theorem: The line integral of  

magnetic field around any closed loop is equal to µ0 ∫ B ⋅ dl = m0I
C
times the total current threading the surface bounded by
where I = net current threading the loop
this closed loop.

140 Physics–12
D:\EG_Physics-12_(26-06-2022)\Open_Files\Ch-4\Ch-4
\ 27-Jul-2022 Ved_Goswami Proof-4 Reader’s Sign _______________________ Date __________

•• Magnetic field due to infinitely long, straight current Y


carrying conductor: B = µ0nI
Consider a straight conductor (infinitely long) carrying
current I. B
Consider the amperian loop to a circle centred on the 2
conductor and of radius = r perpendicular to plane of X′ X
board. End of O End of
solenoid Distance solenoid
Consider an element dl of the loop.
Fig. Variation of magnetic field
(B) along the axis of solenoid
Solenoid:
I
r P dl

Magnetic field is tangential to the loop at all points. I


  
∫ B ⋅ dl = ∫ Bdl cos 0° C •• Magnetic field pattern of a finite solenoid
C
Q
⇒ B ∫ dl = B × 2πr
C
According to Ampere’s circuital law,
∫ Bdl = m0I
µo I
So B2pr = moI or B = P
2πr
•• Magnetic field due to a current carrying Solenoid: Let
n be the number of turns per unit length of the solenoid,
carrying current I.
If medium of the core is vacuum × ×× × ×× ×
   B = µ0nI
Note: Magnitude of magnetic field at end of solenoid is
just half of that at its middle.

EXERCISE 4.3
I. Objective Type Questions (1 Mark) (ii) Ampere’s circuital theorem states that
1. Choose the correct answers from the given options    
(MCQs). (a) ∫ B · dl = µ0 I (b) ∫ B · dl = ε0 I
C
(i) Two identical current-carrying coaxial loops carry
current I in an opposite sense. A simple amperian  
 I  

loop passes through both of them once. Calling (c) ∫ B · dl = µ0 (d) ∫ E · dl = µ0 I
the loop as C,
2. State True or False
∫ B.dl = ∓ 2µ 0I
(a) 
C
(i) Magnetic field due to a current carrying solenoid
(b) the value of ∫ B.dl is independent of sense is m0nI.
C
of C.  
 (ii) Magnetic field due to infinitely long, straight
(c) there may be a point on C where B and dl
µ I
are perpendicular. current carrying conductor is 0 .
(d) B vanishes everywhere on C. πr

Moving Charges and Magnetism 141


D:\EG_Physics-12_(26-06-2022)\Open_Files\Ch-4\Ch-4
\ 27-Jul-2022 Ved_Goswami Proof-4 Reader’s Sign _______________________ Date __________

For questions numbers 3 two statements are given-one turns per unit length, find the magnitude and direction
labelled Assertion (A) and the other labelled Reason of the net magnetic field at a point (i) inside on the
(R). Select the correct answer to these questions from axis and (ii) outside the combined system.
the codes (a), (b), (c) and (d) as given below. [Delhi 2014]
(a) Both A and R are true and R is the correct
explanation of A r2
B
(b) Both A and R are true but R is NOT the correct l
explanation of A
(c) A is true but R is false
(d) A is false and R is also false
3. Assertion (A): Magnitude of Magnetic field at end
of a solenoid is just half of that at it its middle A
r1 S1
Reason (R): Magnetic field remains uniform inside I
the solenoid. n1 turns S2

II. Short Answer Type Questions-I (2 Marks) n2 turns

1. A long straight wire AB carries a current I. A proton 3. State Ampere’s circuital theorem. Using ampere’s
P travels with a speed v, parallel to the wire, at a circuital theorem, obtain an expression for the
distance d from it in a direction opposite to the current magnetic field due to a straight infinite current
as shown in the figure. What is the force experienced carrying conductor.
by the proton and what is its direction? [AI 2010] 4. (a) A straight thick long wire of uniform circular
B cross-section of radius ‘a’ is carrying a steady
y ( j^ )
current I. The current is uniformly distributed
I across the cross-section. Use Ampere’s circuital
law to obtain a relation showing the variation of
Proton the magnetic field (Br) inside and outside the wire
d P x (i^ )
with distance r, (r ≤ a) and (r > a) of the field point
from the centre of its cross-section. What is the
z (k^ ) magnetic field at the surface of this wire? Plot a
A graph showing the nature of this variation.
III. Short Answer Type Questions-II (3 Marks) [AI 2008, Delhi 2010]
2. Two long coaxial insulated solenoids, S1 and S2 of a
equal lengths are wound one over the other as shown (b) Calculate the ratio of magnetic field at a point
2
in the figure. A steady current “I” flow through
a
the inner solenoid S1 to the other end B, which is above the surface of the wire to that at a point
connected to the outer solenoid S2 through which the 2
below its surface. What is the maximum value of
same current “I” flows in the opposite direction so as
to come out at end A. If n1 and n2 are the number of the field of this wire?

Answers 4.3
I. Objective Type Questions Direction of B is along –ve z-axis.
1. (i) (b) and (c) (ii) (a)    eVµ 0 I sin 90°
2. (i) True (ii) False F = qV × B, F =  ½
2πd
3. (b)
 ^  ^ ^
II. Short Answer Type Questions-I So direction of F is given by  − j ×  − k   = i
  
1. Magnetic field due to the current carrying wire AB
at P is into the paper and perpendicular to it. F is along the x-axis. ½
µ0 I 2. (i) At a point inside on the axis
B=  1
2πd Magnetic field due to current I in S1

142 Physics–12
D:\EG_Physics-12_(26-06-2022)\Open_Files\Ch-4\Ch-4
\ 27-Jul-2022 Ved_Goswami Proof-4 Reader’s Sign _______________________ Date __________

B1 = µ0n1I along BA ½ P

Magnetic field due to current I in S2


r
B2 = µ0n2I along AB ½
Net B = B1 – B2 = µ0I (n1 – n2) I
O
along BA if n1 > n2 1
(ii) At a point side the combined system, Bnet = 0 1
3. Ampere’s circuital theorem: The line integral of
the magnetic field over a closed loop is equal to µ0 Q
times the current threading the surface bounded by L = 2 pr
the closed loop.
  Ie = current enclosed by the loop = I
Mathematically ∫ B ⋅ dl = µ 0 I  1 According to Ampere’s circuital theorem
 
Magnetic field due to infinitely long, straight ∫ Bdl = m0I
current carrying conductor: Consider a straight C

conductor (infinitely long) carrying current I. B2pr = µ0I


Consider the Amperian loop to a circle centred on the µ0I
So B=  ...(1)
conductor and of radius = r perpendicular to plane of 2πr
paper. 1
For r > a, B ∝  ½
Consider an element dl of the loop. r
When r < a: Imagine the Amperian loop to be
a circle passing through the point P, which lies
inside the wire, and centre coinciding with the
B centre of the cross-section.
I
P dl
r r I
O

L = 2pr
 
Magnetic field is tangential to the loop at all points. ∫ Bdl = m0I,
   C
∫ B ⋅ dl = ∫ Bdl cos 0° I≠r 2
C C Ie =
≠a 2

⇒ B ∫ dl = B × 2πr  1 Because the current is uniformly distributed across
C the cross-section.
According to Ampere’s circuital law, µ 0 Iπr 2  µ I 
   So B2pr = or B =  0 2  r ½
πa 2
 2πa 
∫ B ⋅ dl = m0I For r < a, B ∝ r
⇒ B2pr = moI For the point lying on the surface of the conductor
µ I the loop will have a radius equal to the radius of
⇒ B = o  1 the wire. So on the surface of the wire
2πr
µ I
4. (a) Consider when r > a: Imagine the Amperian loop B2pa = µ0I, B = 0  1
2πa
to be circular loop with centre coinciding with the
centre of the cross-section F. (b) B at a above the surface of the conductor:
2
The point P lies on the circumference of the a 3a
Amperian loop.      r= a+ =
2 2

Moving Charges and Magnetism 143


D:\EG_Physics-12_(26-06-2022)\Open_Files\Ch-4\Ch-4
\ 27-Jul-2022 Ved_Goswami Proof-4 Reader’s Sign _______________________ Date __________

µ0I µ I × 2 µ0I
So B0 = = 0 =
2π × r 2π × 3a 3πa
B
a
B at below the surface:
2 B=
µ0I
2πa
a a
r= a− =
2 2 Bαr 1
Bα r
µ0I µ I a µ I
  2
r= 02 × = 0 Bi =
2πa 2πa 2 4πa
r
a
B0 µ 0 I 4πa 4
  = × =  1 Graph:
Bi 3πa µ 0 I 3

Topic 4. Force on a Current-carrying Conductor and It’s Applications


•• Force between 2 parallel current carrying conductors F2 µ × 1 × 1 4π ×10 −7
The two-infinitely long conductors (1) and (2) carrying = 0 = = 2 × 10 −7 N/m
l 2π × 1 2π
current are placed some distance ‘r’ apart. If two infinitely long parallel conductors carrying
Conductor (2) is placed in magnetic field produced by equal current and placed 1m apart in vacuum, exert
current in the conductor (1). a force of 2 × 10–7 N/m of their length on each other
µ 0 I1 then the current through each of them is said to be
B1 = ...(1) 1 Ampere.
2πr
Conductor (2) is placed in magnetic field B1 which is Note: Two parallel conductors carrying current
pointing into the paper. (i) in same direction, attract each other
(1) (2) (1) (2) (ii) In opposite direction, repel each other, force is very
weak.
•• Circular current loop as a magnetic dipole: A current
B1 B1 loop behaves like a magnetic dipole. If we look at the
F2 F2 upper face of coil carrying current in clock wise direction
then it has south polarity. If we look at the lower face
having current in anticlockwise direction then it has N
I1 I2 I1 I2 polarity.
Magnetic dipole moment M = IA
B2
| M | = Ipr2
•• Magnetic field on the axis of a circular loop
F1 F1 µ 0 Ia 2
B=
B2 2 (a 2 + x 2) 3/2
r r Magnetic field on the axis of a circular loop x >> a
Parallel currents attract Antiparallel currents repel µ 0 Ia 2 µ0 I πa 2
B= =
Conductor (2) will experience a force 2x 3
2π x 3
 
(
F2 = I 2l × B1 = I2lB1 sin 90° ) B=
µ0 M
2π x 3
F2 Iµ I If loop has N turns M = NIA
= I 2 B1 = 2 0 1 •• Magnetic dipole moment of revolving electron
l 2πr e
Direction is towards conductor (1) M = 4π me nh
Similarly conductor 1 is placed in the magnetic field Where me is mass of electron and h is planck’s constant.
produced by current carrying conductor 2. •• Torque on a current carrying coil in a magnetic field:
If I1 = I2 = 1A, r = 1m, in vacuum Consider a rectangular coil PQRS suspended in uniform

magnetic field of induction B.

144 Physics–12
D:\EG_Physics-12_(26-06-2022)\Open_Files\Ch-4\Ch-4
\ 27-Jul-2022 Ved_Goswami Proof-4 Reader’s Sign _______________________ Date __________

PQ = RS = l cos q = cos (90° – a) = sin a


QR = SP = b \ Torque becomes,
I = current following through coil in the direction  
t = nIBA sin a = mB sin a = m ×B
PQRS. Where m = magnitude of magnetic dipole moment of
q = Angle which plane of coil makes with direction rectangular current loop and m = nIA
of magnetic field.     
F4 S
\ τ = m × B = nI A× B ( )
Special Cases:
B F3 = BIl
(i) If coil is set with its plane parallel to direction of

T b cos θ
P I S
F3 θ magnetic field B, then q = 0°, cos q = 1
I
t = nIBA (1) = nIBA (maximum)
θ
B (ii) If coil is set with its plane ^ to direction of magnetic
F1
90° I field B then q = 90°, cos q = 0.
R t = nIBA (0) = (0) (minimum)
•• Moving Coil Galvanometer: It is a device used to detect

P
I F2
the presence of current in a circuit.
Q F1 = BIl
    Principle: It is based on the principle that a current
Let F1 , F2 , F3 and F4 be the forces acting on the 4 currents carrying coil placed in a uniform magnetic field
carrying arms PQ, QR, RS and SP respectively of the experiences a torque and the magnitude of the torque
coil. depends upon the strength of the current.
   Construction:
Force on arm SP = F4 = I (SP × B)
 •• If consists of a rectangular coil of large no. of turns of
    F4 = I (SP) B sin (180° − θ) = IbB sin θ insulated copper were wound on a light non-magnetic,
 
Direction of this force is in the direction of (SP × B) i.e., metallic frame.
in the plane of the coil directed upwards. •• The two ends of the axle of this frame are suspended
Force on arm QR = F2 = IbB sin q from a movable tortion head by means of phosphurbronze
strip in a uniform magnetic field provided by horse shoe
Direction is in the plane of coil directed downwards.
  magnet.
Since F2 and F4 are equal in magnitude and acting in
opposite directions along the same straight line they
cancel out each other. Their resultant effect on coil = 0.
Force on arm PQ = I (PQ) B sin 90° = IlB
According to Flemings left hand rule, direction of this
force is perpendicular to plane of coil directed outwards.
Force on arm RS = I (RS) B sin 90° = IlB
Direction is into plane of coil.
The forces acting on PQ and RS are equal, parallel and
acting in opposite directions along different lines of
action. They form a couple. The effect of this is to rotates
the coil in the anticlockwise direction about the dotted
line as axis.
Torque on coil = either force × arm of couple
Arm of couple = ST = PS cos q = b cos q. Fig. The moving coil galvanometer with radial field
Torque = t = IlB × b cos q •• The pole pieces of magnet are cut into concave shape
= IlbB cos q and a soft iron cylinder is held within the coil. This is
[lb = area = A] = IAB cos q done to make the field radial i.e., the plane of the coil in
If rectangular coil has n turns, then all positions remains parallel to the magnetic field, and
t = nIBA cos q delection ∝ current.
Note: If normal drawn on plane of coil makes angle a Theory: Suppose coil of length l, breadth b is suspended
with direction of magnetic field, then in the radial magnetic field. It will experience a torque
q + a = 90° ⇒ q = 90° – a which will deflect the coil.

Moving Charges and Magnetism 145


D:\EG_Physics-12_(26-06-2022)\Open_Files\Ch-4\Ch-4
\ 27-Jul-2022 Ved_Goswami Proof-4 Reader’s Sign _______________________ Date __________

Deflecting torque = NIAB sin 90° = NBIA Let G = resistance of galvanometer


Where N = no. of turns, B = strength of magnetic field, Ig = max. current which gives full scale
A = area of coil deflection in galvanometer
Due to this torque, the coil rotates. We want to use it to measure a current for 0 – I ampere
The phosphurbronze stick gets twisted. we should connect a shunt resistance (low resistance) in
\ Restoring torque comes into play parallel with galvanometer.
If q = twist produced, restoring torque ∝q, Restoring P.D. across S = P.D. across G
Torque = Kq (I – Ig)S = IgG
Where k = restoring torque per unit twist of wire. Ig G
In equilibrium position of coil, S=
(I − I g )
Deflecting torque = restoring torque I – Ig S
NBIA = kq
k
I= θ
NBA
I∝q
More the current, more will be the deflection
I
\ Galvanometer has linear scale. G
Ig
Note: Significance of radial magnetic field. Due to radial
magnetic field, plane of coil is always parallel to magnetic Resistance of ammeter so formed
field thus the deflection produced is directly proportional 1 1 1
= +
to the current and scale of galvanometer is linear. G′ S G
k G ×S
I= θ
NBA G′ = G + S
•• Current sensitivity (IS): It is defined as the deflection •• Conversion of galvanometer into voltmeter: This can
produced when a unit current flows through the be done by connecting high resistance in series with it.
galvanometer
Let G = resistance of galvanometer.
θ NBA
IS = = It gives full scale deflection for current Ig
I k
Methods of increasing current sensitivity
(i) It can be increased by increasing the no. of turn. R
G
(ii) By increasing value of B and this is done by uncreased Ig
a strong horse shoe magnet.
(iii) By increasing area of coil.
(iv) By decreasing value of ‘k’. If we want to use it to measure a p.d. between 0V to V
then a high resistance R should be connected in series
Note: Value of N can’t be increased beyond centain
with it.
limit, as increase in no. of turns would lead to increase
in resistance of galvanometer and it will become bulky. Net p.d. V = IgG + IgR
Value of A can’t be increase beyond certain limit. In this   V – IgG = IgR
case coil will not be placed in uniform magnetic field and
V − Ig G V
will also make the galvanometer very bulky. R= = −G
•• Conversion of galvanometer to ammeter/To increase Ig Ig
range of galvanometer to measure current: Resistance of voltmeter so formed = R + G.

EXERCISE 4.4
I. Objective Type Questions (1 Mark) the loop with x > 0 is now bent so that it now lies
1. Choose the correct answers from the given options in the y-z plane.
(MCQs). (a) The magnitude of magnetic moment now
(i) A current-carrying circular loop of radius R is diminishes.
placed in x-y plane with centre at origin. Half of (b) The magnetic moment does not change.
146 Physics–12
D:\EG_Physics-12_(26-06-2022)\Open_Files\Ch-4\Ch-4
\ 27-Jul-2022 Ved_Goswami Proof-4 Reader’s Sign _______________________ Date __________

(c) The magnitude of B at (0, 0, z), z >> R 3. Assertion (A): When a current carrying coil placed
increases. in a uniform magnetic field it experiences torque.
(d) The magnitude of B at (0, 0, z), z >> R is Reason (R): Magnitude of the torgque depends upon
unchanged. the strength of the current.
(ii) A circular current loop of magnetic moment M is
in an arbitrary orientation in an external magnetic II. Very Short Answer Type Questions (1 Mark)

field B. The work done to rotate the loop by 30° 1. A circular loop of area A , carrying a current I is

about an axis perpendicular to its plane is placed in a uniform magnetic field B . Write the

3 expression for the torque τ acting on it in a vector
(a) MB (b) MB form.
2
MB 2. State two reasons why a galvanometer cannot be used
(c) (d) zero.
2 as such to measure current in a given circuit.
(iii) The gyro-magnetic ratio of an electron in an 3. An ammeter and a Milliammeter are converted from
H-atom, according to Bohr’s model is the same galvanometer. Out of the two, which current
(a) independent of which orbit it is in. measuring instrument has a higher resistance?
(b) negative.
4. Define the term current sensitivity of moving coil
(c) positive. galvanometer.[Delhi 2020]
(d) increases with the quantum number n.
(iv) Two parallel conductors carrying current of 4.0 5. An electron moves along +x direction. It enters into

A and 10.0 A are placed 2.5 cm apart in vacuum. a region of uniform magnetic field B directed along
The force per unit length between them is –z direction as shown in figure. Draw the shape of
[CBSE 2022] trajectory followed by the electron after entering the
(a) 6.4 × 10 N/m (b) 6.4 × 10 N/m
–5 –2 field. [Delhi 2020]
(c) 4.6 × 10–4 N/m (d) 3.2 × 10–4 N/m 
y B
(v) If an ammeter is to be used in place of a voltmeter,
then we must connect with the ammeter a
[CBSE 2022] x
e
(a) low resistance in parallel z
(b) low resistance in series
(c) high resistance in parallel 6. A square shaped current carrying loop MNOP is
(d) high resistance in series placed near a straight long current carrying wire AB
as shown in the figure. The wire and the loop lie in
2. Fill in the blanks:
the same plane. If the loop experiences a net force F
(i) A galvanometer of resistance 25 W is shunted by
towards the wire, find the magnitude of the force on
a 2.5 W wire. The part of total current that flows
the side ‘NO’ of the loop. [Delhi 2020]
through the galvanometer is given by ..........
(ii) The resistance of a galvanometer is 50 W and the A M N
current required to give full scale deflection is I1 I2
100 µA. In order to convert it into an ammeter
L L
for reading up to 10 A, it is necessary to put a
resistance of .............
For questions numbers 3 two statements are given-one P
B O
labelled Assertion (A) and the other labelled Reason
(R). Select the correct answer to these questions from III. Short Answer Type Questions-I (2 Marks)
the codes (a), (b), (c) and (d) as given below. 7. A circular coil of ‘N’ turns and diameter ‘d’ carries
(a) Both A and R are true and R is the correct a current ‘I’. It is unwound and rewound to make
explanation of A another coil of diameter ‘2d’, current ‘I’ remaining
(b) Both A and R are true but R is NOT the correct the same. Calculate the ratio of the magnetic moments
explanation of A of the new coil and the original coil. [AI 2012]
(c) A is true but R is false 8. A rectangular current carrying loop is kept in a
(d) A is false and R is also false uniform mag. field as shown below:
Moving Charges and Magnetism 147
D:\EG_Physics-12_(26-06-2022)\Open_Files\Ch-4\Ch-4
\ 27-Jul-2022 Ved_Goswami Proof-4 Reader’s Sign _______________________ Date __________

P Q which region(s) is/are there a point on the x-axis, at


which the magnetic field is equal to zero due to these
currents carrying wires? Justify your answer.
S
[CBSE S.P. 2020-21]
N
Region I Region II Region III
× +x
2A 3A
S R
IV. Short Answer Type Questions-II (3 Marks)
(a) What is the direction of magnetic moment of the 17. Derive an expression for the force per unit length
current loop? between two long straight parallel current carrying
(b) When is the torque acting on the loop: (i) max. conductors. Hence define SI unit of current (ampere).
(ii) zero?  [AI 2005, 2009] [AI 2009, 2010, 2012]
9. Write two disadvantages of a moving coil 18. A rectangular loop of wire of size 4 cm × 10 cm carries
galvanometer. a steady current of 2A. A straight long wire carrying
10. A wire AB is carrying a steady current of 12A and is 5A current is kept near the loop as shown. If the loop
lying on the table. Another wire CD carrying 5A is and the wire are coplanar, find:
held directly above AB at a height of 1 mm. Find the (i) the torque acting on the loop and
mass per unit length of the wire CD so that it remains (ii) the magnitude and direction of the force on the
suspended at its position when left free. Give the loop due to the current carrying wire.
direction of the current flowing in CD with respect 5A 4 cm
to that in AB. [Take the value of g = 10 ms–2] I1 A B

[AI 2013]
11. A galvanometer coil has a resistance of 15W and the
metre shows full scale deflection for a current of 4 mA. 2A 10 cm
How will you convert the metre into an ammeter of I2
range 0 to 6A? [NCERT]
12. A galvanometer coil has a resistance of 12W and the
metre shows full scale deflection for a current of 3
mA. How will you convert the metre into a voltmeter 1
D C

of range 0 to 18V? [NCERT] cm

13. Define the term magnetic moment of a current loop. [Delhi 2012]
Write the expression for the magnetic moment when 19. A straight horizontal conducting rod of length 0.45m
an electron revolves at a speed v around an orbit of and mass 60g is suspended by two vertical wires at its
radius ‘r’ in hydrogen atom. [AI 2008] ends. A current of 5.0 A is set up in the rod through
14. Two long straight parallel wire A and B separated by the wires.
a distance d, carry equal current I flowing in same (a) What magnetic field should be set up normal to
direction as shown in the figure. [Delhi, AI 2020] the conductor in order that the tension in the wires
A P B is zero?
I I (b) What will be the total tension in the wires if
x d the direction of current is reversed, keeping the
(a) Find the magnetic field at a point P situated magnetic field same as before? (Neglect the mass
between them at a distance x from one wire. of wires, g = 9.8 m/s2).[AI 2005]
(b) Show graphically the variation of the magnetic
field with distance x for 0 < x < d.
15. A galvanometer of resistance 16 W shows full scale
× × × × × × × × × × × ×
deflection for a current of 4 mA. How will you convert
it into a voltmeter to measure a voltage up to 3 V? × × × × × × × × × × × ×
[AI 2020]
16. Two straight infinitely long wires are fixed in space × × × × × × × × × × × ×
so that the current in the left wire is 2 A and directed 20. An observer to the left of a solenoid of N turns each of
out of the plane of the page and the current in the right cross section area ‘A’ observes that a steady current I in
wire is 3 A and directed into the plane of the page. In it flows in the clockwise direction. Depict the magnetic

148 Physics–12
D:\EG_Physics-12_(26-06-2022)\Open_Files\Ch-4\Ch-4
\ 27-Jul-2022 Ved_Goswami Proof-4 Reader’s Sign _______________________ Date __________

field lines due to the solenoid specifying its polarity and of the total field (external field + field produced
show that it acts as a bar magnet of magnetic moment by the loop) is maximum.
m = NIA. [Delhi 2015] (c) A loop of irregular shape carrying current is

21. Write the expression for the magnetic moment ( m) located in an external magnetic field. If the wire
due to a planar square loop of side ‘l’ carrying a steady is flexible, why does it change to a circular shape?
current I in a vector form.  [NCERT]
In the given figure this loop is placed in a horizontal 27. The current flowing in galvanometer G when key
plane near a long straight conductor carrying a steady k2 is kept open is I. On closing k2, the current in the
current I1 at a distance l as shown. Gives reasons to galvanometer become I/n, where n is an integer.
explain that the loop will experience a net force but Obtain expression for the resistance G of the
no torque. Write the expression for this force acting galvanometer in terms of R, S and n. To what form
on the loop. [Delhi 2010] does this expression reduce when the value of R is
I1 very large as compared to S? [S.P. 2015]
S k2
( )

R
G
l

22. (a) “Increasing the current sensitivity of a ( )


k1
galvanometer may not necessarily increase its E
voltage sensitivity. Justify this statement. 28. A galvanometer of resistance g is converted into a
(b) Outline the necessary steps to convert a voltmeter to measure upto V volts by connecting a
galvanometer of resistance RG into an ammeter resistance R1 in series with the coil. If resistance R2 is
of a given range.  [AI 2011] connected in series with it, then it can measure upto
23. (a) Define the terms (i) current sensitivity and (ii) V/2 volts. Find the resistance, in terms of R1 and R2,
voltage sensitivity. required to be connected to convert it into a voltmeter
(b) Explain the underlying principle used in that can read upto 2V. Also find the resistance G of
converting a galvanometer into a (i) voltmeter the galvanometer in terms of R1 and R2.[Delhi 2015]
and (ii) ammeter.  [AI 2015] 29. Describe the working principle of a moving coil
24. (i) What is the relationship between the current galvanometer. Why is it necessary to use (i) a radial
and the magnetic moment of a current carrying magnetic field and (ii) a cylindrical soft iron core in
circular loop? a galvanometer?
(ii) Deduce an expression for magnetic dipole moment Write the expression for current sensitivity of the
of an electron revolving around a nucleus in a galvanometer.
circular orbit. Indicate the direction of magnetic Can a galvanometer as such be used for measuring
dipole moment? Use the expression to derive the current? Explain. [Delhi 2017]
the relation between the magnetic moment of an 30. A electron of mass me revolves around a nucleus of
electron moving in a circle and its related angular charge + Ze. Show that it behaves like a tiny magnetic
momentum?  [Foreign 2009, AI 2010] dipole. Hence prove that the magnetic moment

associated with it is expressed as µ l = − e L ,
25. Derive the expression for the torque on a rectangular 
current carrying loop suspended in a uniform 2me

magnetic field. [Delhi 2009, 2013] Where L is the orbital angular momentum of the
26. (a) A current carrying circular loop lies on a smooth electron. Give the significance of negative sign.
horizontal plane. Can a uniform magnetic field be [Delhi 2017]
set up in such a manner that the loop turns around 31. (i) Define current sensitivity of a galvanometer. Write
itself (i.e., turns about the vertical axis). its expression.
(b) A current-carrying circular loop is located in (ii) A galvanometer has resistance G and shows full
a uniform external magnetic field. If the loop scale deflection for current Ig.
is free to turn, what is its orientation of stable (a) How can it be converted into an ammeter to
equilibrium? Show that in this orientation, the flux measure current up to I0(I0 > Ig)?

Moving Charges and Magnetism 149


D:\EG_Physics-12_(26-06-2022)\Open_Files\Ch-4\Ch-4
\ 27-Jul-2022 Ved_Goswami Proof-4 Reader’s Sign _______________________ Date __________

(b) What is the effective resistance of this 34. A uniform magnetic field of 3000G is established
ammeter?[Delhi, AI 2020] along the positive z-direction. A rectangular loop of
32. Obtain the expression for the deflecting torque sides 10 cm and 5 cm carries a current of 12A. What
acting on the current carrying rectangular coil of a is the torque on the loop in the different cases shown
galvanometer in a uniform magnetic field. Why is in figure. What is the force on each case? Which case
radical magnetic field employed in the moving coil corresponds to stable equilibrium?[NCERT]
galvanometer?[AI 2020] z z

OR

Derive the expression for the torque acting on the
B
rectangular current carrying coil of a galvanometer. I B
I
Why is the magnetic field made radial? [AI 2020] y y

V. Long Answer Type Questions (5 Marks) (a)


x x (b)
33. A 100 turn closely wound circular coil of radius 10 cm z z
carries a current of 3.2 A. (a) What is the field at the
B
centre of the coil? (b) What is the magnetic moment
of this coil?
I B
I
The coil is placed in a vertical plane and is free to y y
rotate about a horizontal axis which coincides with 30°
its diameter. A uniform magnetic field of 2T in the x (d)
x
horizontal direction exists such that initially the axis z z
of the coil is in the direction of the field. The coil
rotates through an angle of 90° under the influence
of the magnetic field. (c) What are the magnitudes of B B
the torques on the coil in the initial and final position?
(d) What is the angular speed acquired by the coil y
I
y
I
when it has rotated by 90°? The moment of inertia
of the coil is 0.1 kg m2. x (e)
x (f)

Answers 4.4
I. Objective Type Questions Clearly, smaller the value of range I, large is the
1. (i) (a) (ii) (b) and (d) (iii) (a) and (b) shunt resistance. As milliammeter will have range in
(iv) (d) (v) (a) milliampere and an ammeter in ampere, so obviously,
I 1 milliammeter will have a larger shunt resistance and
2. (i) = (ii) 5 × 10–4 W hence it will have a higher resistance.
I0 11
3. (b) 1 1 1 GS
As, = + ⇒ RA =
II. Very Short Answer Type Questions RA G S G +S
  
1. τ = NIA × B, N → No. of turns So higher the shunt resistance S, higher will be the RA
2. (i) A galvanometer has a finite large resistance. When for given G, milliammeter will have higher resistance.
it is connected in series to measure current, the 4. Current sensitivity of a galvanometer is defined as the
net resistance will increase, thus the current to be deflection produced in the galvanometer when a unit
measured will decrease. current flows through it.
(ii) It will give full scale deflection for a very small At equilibrium position of coil in the moving coil
current of the order of micro ampere. It can measure galvanometer, Deflecting torque = restoring torque
only very small current.
k 
I = 
Ig G
3. Shunt resistance, S = NBIA = kq   or θ ...(1)
I − Ig  NBA 

150 Physics–12
D:\EG_Physics-12_(26-06-2022)\Open_Files\Ch-4\Ch-4
\ 27-Jul-2022 Ved_Goswami Proof-4 Reader’s Sign _______________________ Date __________

Mathematically it can be given by


Thus, N ×  2π  d   = N ′  2π  2d  
θ NBA   2   2 
IS = =  using (1)    
I k
here k is the restoring torque per unit twist of wire. N
N′ =  1
5. Force on the electron is given by 2
  
( )
   F = − q v × B = – qvB sin q i.e. F ^ v Now    mA = NIA A = NI πrA2 = ( ) 1
4
NIπd 2
Whenever F ^ v, path will be circular. NI 1
So, the electron will move on a semi circular path in Similarly mB = N ′IA B =
2
( )
πrB2 = NIπd 2
2
( )
the magnetic field.
y 1
mB 2 m 2
= 2= ⇒ B =  1
mA 1 1 mA 1
v x
4
z v 8. (a) Direction of magnetic moment is normal to the loop
6. The force acting on the side MN and force on side PO pointing inward. 1
will nullify each other as the wires are located at equal (b) Torque on the loop t = mB sin q
distance from the infinite wire AB but current flowing (i) Torque will be maximum if m is perpendicular to
in opposite directions in them. B. i.e., the plane of the coil is parallel to magnetic
Distance of side MP from wire AB = L and distance of field. ½
side NO from wire AB = L + L = 2L (ii) Torque will be zero if q = 0° i.e., the plane of coil
is perpendicular to magnetic field. ½
Now, using formula for force between two parallel
9. (i) Its sensitiveness can’t be changed at will. 1
current carrying conductors
(ii) It easily get damaged by overloading. 1
F µ 0  I1I 2  10. Wire AB: I1 = 12A, wire CD: I2 = 5A,
=   , here r is distance between the two
L 2π  r       r = 1 mm
conductors. C D
µ 0  I1I 2  1 mm   ½
i.e. F=  L
2π  r 
The force acting on the whole loop, A B
  On the table
F = FMP − FNO Weight per unit length of wire CD = Magnetic force
µ 0 I1I 2 per unit length
µ 0  I1I 2  µ 0 I1I 2 L
=   L − = mg µ II
2π L 2π ( 2L ) 4π = 012 ½
l 2πr
Towards the wire AB as parallel currents attract.
m µ 0 I1I 2
The force acting on the side NO is given by Mass per unit length =  ½
l 2πrg
 µ II  µ II
FNO = 0  1 2  L = 0 1 2 = F 4π × 10 −7 × 12 × 5
2π  2L  4π = = 120 × 10 −5
Away from the wire AB as antiparallel currents repel. 2π ×10 −3 × 10
7. We know, magnetic moment (m) = NIA where N = No. = 1.2 × 10–3 kg/m ½
of turns The direction of current is CD should be opposite to
the direction of current in AB so that the force on CD
is repulsive i.e., opposite to weight of CD.
11. Given: G = 15W, Ig = 4 mA = 4 × 10–3A ½
d 2d I = 6A, S = ?
N Ig G 4 × 10 −3 × 15
S= =

I − Ig 6 − 4 × 10 −3
Coil A Coil B 6 × 10 −2
= = 10 −2 Ω  1
Then, length of wire remains same 5.996

Moving Charges and Magnetism 151


D:\EG_Physics-12_(26-06-2022)\Open_Files\Ch-4\Ch-4
\ 27-Jul-2022 Ved_Goswami Proof-4 Reader’s Sign _______________________ Date __________

A resistance of 10–2 W should be connected in parallel (b) B


with galvanometer. ½

magnetic
field
12. Given: G = 12W, Ig = 3mA = 3 × 10 A, –3

V = 18V ½
d x
V 18 O distance
R= −G= − 12  1
Ig 3 × 10 −3
= 6000 –12 = 5988 W ½
13. Magnetic moment of a current loop: The torque on
current loop is 15. Here given, G = 16 W, V = 3 volt, Ig = 4 × 10–3 A
t = MB sin q, where q is angle between magnetic V
Using R + G =
moment and magnetic field. ½ Ig
τ
So, M = 3 3000
B sin θ R + 16 = ⇒ R + 16 =
4 × 10 −3 4
If B = 1T, sin q = 1 or q = 90° then M = t. ⇒ R = 750 – 16 = 734 W
That is the magnetic moment of a current loop is We can convert it into a voltmeter by connecting a high
defined as the torque acting on the loop when placed resistance (R = 734 W) in series with galvanometer.
in a magnetic field of 1T such that the loop is oriented
16. In region-I: Magnetic field due to 2A and 3A current
with its plane parallel to the magnetic field. ½
wire is in opposite direction.
Also, m = NIA,
In region-II: Here, also magnetic field due to 2A and
i.e., magnetic moment of a current loop is the product
3A currents wire is in opposite direction.
of number of turns, current flowing in the loop and area
of loop. Its direction is perpendicular to the plane of the But, it is most probable, in region I, that we can have
loop. neutral points, because
2µ o 3µ o
B1 = ; B2 =
v 2πR1 2πR 2
o r for region I, R2 > R1 and I2(3A) > 11(2A) and B1 and B2
are in opposite directions, so net magnetic intensity will
become zero at existing neutral point. (highly probable)
2πr 17. Consider two infinitely long straight parallel conductors
Time period of revolution T =  ½ carrying current I1 and I2 placed r distance apart.
v
e ev Conductor-2 is placed in the magnetic field produced
I= =
T 2πr due to current in conductor-1.
evπr 2 evr µ 0 I1
So M= =  ½ B1 =  1
2πr 2 2πr
14. (a) A P B (1) (2)

I x d–x I
r
Magnetic field at point P, I1
  
BP = BA + BB
µ0 I µ0 I
= (upwards) – (downwards)
2πx 2π ( d − x ) I2

µ I 1 1  µ0 I  d − x − x 
= 0  −  =  
2πx  x ( d − x )  2πx  x ( d − x ) 
µ0 I  d − 2 x  This magnetic field is pointing into the paper on right
=   upwards as BA > BB hand side. So the force/length experienced by conductor
2πx  x ( d − x )  2 is
152 Physics–12
D:\EG_Physics-12_(26-06-2022)\Open_Files\Ch-4\Ch-4
\ 27-Jul-2022 Ved_Goswami Proof-4 Reader’s Sign _______________________ Date __________

F2 µ II
= I 2 B1 = 0 1 2  1
l 2πr T1 T2
SI unit of current : If I1 = I2 = 1A, r = 1m
F I
= 2 × 10–7 N/m
l mg
Definition: If two straight parallel infinitely long
(b) If the direction of the current is reversed, the
conductors carrying equal current, placed 1m apart in direction of the magnet force. BIl will act in the
vacuum exert a force of 2 × 10–7 N per metre of their same direction as mg
length on each other, then current through each of them So Total tension in the wires
T = (T1 + T2) = BIl + mg 1
is said to be1 Ampere. 1
   as mg = BIl
18. (i) τ = M × B , M and B both are directed into the paper So T = mg + mg = 2 mg
so q = 0° = 2×60×10–3 ×9.8 = 1.176 N 1
\ t = 0 1 20. To show that it acts as a bar magnet of magnetic moment
(ii) Force on arm AD m = NIA.
µ 0 I1I 2l 1 Show that solenoid is equivalent to a bar magnet:
F1 = , r1 = 10 −2 m
2π r1 Magnetic field lines of a bar magnet and solenoid show
similar patterns. Let us calculate the axial field of a finite
F1 is attractive
solenoid.
Force on arm CB
µ 0 I1I 2l 1
F2 = , r2 = 5 × 10 −2 m  1
2π r2
F2 is repulsive, so
   µ I I  1 1
So, F = F1 + F2 = o 1 2  −  S N
2π  r r  1 2

4π × 10 −7 × 5 × 2 × 0.1  1 1 
=  −
2π  10 −2
5 × 10 −2  Consider a solenoid consisting of ‘n’ turns per unit
= 20 × 10–8 (100 – 20) = 1.6 × 10–5 N 1 length. Let length of solenoid = 2l and radius = a. Let
P be the point at a distance r from centre of solenoid
Attractive as F1 > F2
where we want to find the field intensity.
5A
A
4 cm
B Each turn of solenoid is at a different distance from the
I1
point P. Consider a circular element of thickness = dx
of the solenoid at a distance x from the centre.
No. of turns in element = n × dx
2A 10 cm
I2 Magnetic field produced by these turns = dB
2µ 0 Iπa 2 ndx
dB = 3  1
2
D C 4π [(r − x) + a 2 ]2
1
cm l l
a
19. (a) l = 0.45 m, m = 60 g = 60 × 10–3 kg, P
I = 5.0 A -x-
For tension in the wires to be zero, weight of the rod r
must be balanced by the magnetic force Total magnetic field = B = ∫ dB
i.e., BIl = mg +l
µ n ⋅ I ⋅ a2 dx
B=
mg 60 × 10 × 9.8
=
−3
= 0.26T  1
B= o
2 ∫ 3
−l 2 2 2
Il 5 × 0.45 [(r −x) + a ]

Moving Charges and Magnetism 153


D:\EG_Physics-12_(26-06-2022)\Open_Files\Ch-4\Ch-4
\ 27-Jul-2022 Ved_Goswami Proof-4 Reader’s Sign _______________________ Date __________

For the points that are very far away i.e., r >>> l and r (ii) Increasing B — the magnetic field.
>>> a. (iii) Increasing A — Area of the coil
+l
µ o nI a 2 µ o n Ia 2 (iv) Decreasing k — Restoring torque per unit twist.
B=
2r 3
∫ dx = 2r 3
[l − (− l )]
θ NBA
−l Voltage sensitivity VS = =  1
2 2 V kR
µ o nI a 2l µ o n Ila
= =  1 VS depends upon the resistance of the coil. If we
2r 3 r3 increase, N or A to increase IS then the resistance of
Multiply, divide by 4p, we get the coil will also increase, then it is not certain that
µ o 4π I na 2 l µ o nIAl VS will increase.
B= =
4π r3 4π r 3 Thus the increase of current sensitivity may not
[n = turns per unit length] necessarily increase its VS.
N
for total no. of turns (N), we have  n =  (b) In order to convert a galvanometer into an ammeter
 2l  we should connect a small resistance in parallel to
B = µ o 4 NIA3l = µ o 2m ( m = NIA) 1 the galvanometer.
3
4π 2l × r 4π r S
This is also the expression for the magnetic field due
    1
to a bar magnet at an axial point. I – Ig

21. The magnetic moment m due to planar square loop of 
G
  I Rg
side ‘l’ carrying a steady current
= l 2 n^ as m IA
I is m I= Ig

Let Ig is the current for which the galvanometer


Where n^ is a unit vector normal to the loop. As per the
 gives full scale deflection. We want to convert it into
diagram given below m is directed into the paper.
an ammeter of range 0 – 1, then let S be the shunt
I1
I l resistance connected in parallel to it.
P Q
Igg = (I – Ig) S
Ig
l So, S= Rg  1
I − Ig
23. (a) (i) Current sensitivity (IS): The deflection produced
S R per unit current passing through the is galvanom-
F1 = Force on PQ = B1Il (attractive) eter is called current sensitivity.
µ I Il µ I I θ
  = 0 1 = 0 1 attractive  1 IS =  ½
2πl 2π I
F2 = Force on SR = B2Il ­(ii) Voltage sensitivity (VS) : The deflection produced
µ I Il µ II per unit potential difference applied across the
  = 0 1 (repulsive) = 0 1 (repulsive)
2π (2l ) 4π galvanometer is called voltage sensitivity.
Forces on PS and QR are equal and opposite, their θ θ IS
resultant is zero.    VS = = =  ½
V IR R
  µ II
Net force = F1 + F2 = 0 1 attractive  1 (b) (i) Conversion of galvanometer to voltmeter: A
4π voltmeter is supposed to have a high resistance.
  
Torque τ = m × B
m and B both are directed into the paper so angle G
Ig R
between them is zero.
     ½
τ = mB sin q = 0 1
So there is no torque but it experience an attractive
To convert a galvanometer into voltmeter of
force. required range a suitable high resistance R should
θ NBA
22. (a) Current sensitivity = IS = = be connected in series with it. If Ig in the full scale
I k deflection for the galvanometer then
Current sensitivity can be increased by
V = IgG + IgR, G is the resistance of the galvanometer.
(i) Increasing N — the no. of turns is the coil.
154 Physics–12
D:\EG_Physics-12_(26-06-2022)\Open_Files\Ch-4\Ch-4
\ 27-Jul-2022 Ved_Goswami Proof-4 Reader’s Sign _______________________ Date __________

V evr
R= −G ½ me = Iπr 2 = ...(1) ½
Ig 2
The direction of the magnetic dipole moment will
(ii) Conversion of a galvanometer to ammeter: An be perpendicular to the plane of the orbit and into
ammeter is supposed to have very low resistance. the paper.
S Also angular momentum
I – Ig L = mevr ...(2) ½
    ½ L is normal to the plane of the paper and in the
G upward direction.
I Ig So, dividing eqn. (1) by (2), we get
A low resistance (S) should be connected in parallel ml evr e
= =
with the galvanometer. L 2 × me vr 2me
IgG = (I – Ig)S  −e 
or ml = L  ½
Ig G 2me
So, S=  ½ 25. Torque on a current carrying coil in a magnetic field
I − Ig
24. (i) If A is area of the loop carrying current I. Then Consider a rectangular coil PQRS suspended in uniform

magnetic moment µl magnetic field of induction B
 
    µ l = IA  ½ PQ = RS = l, QR = SP = b
(ii) Magnetic dipole moment of a revolving electron I = c urrent flowing through coil in the
: Consider an electron moving is a circular path direction PQRS.
around a +vely charged nucleus. Force of attraction q = Angle which plane of coil makes with
between electron and nucleus provides necessary direction of magnetic field.
   
centripetal force. Let F1 , F2 , F3 and F4 be the forces acting on the 4 currents
Electron charge is (– e) where carrying arms PQ, QR, RS and SP respectively of the
e = + 1.6 × 10–19 C coil.
  
Charge on stationary heavy nucleus = + Ze Force on arm SP = F4 = I (SP × B) , as SP = b
Uniform circular motion constitutes a current I. 
       F4 = I (SP) B sin (180° − θ) = IbB sin θ
e Direction of this force is in the plane of the coil directed
I= ...(1) ½
T upwards.
Where T = time period of revolution. Force on arm QR = F2 = IbB sin q ½
Direction of magnetic moment is into the plane of Direction is in the plane of coil directed downwards.   
the paper and is indicated separately by ⊗. F4 S
v
e–  B
P I
r I F3
×
µe

+Ze F1
90° I        ½
R

I F2
Q Q F3 = BIl
Let r = orbital radius of electron and T b cos θ
S
v = orbital speed θ
2pr
   T=  ½ θ
v B
ev
Substituting in (1), we get: I =
2πr
There will be a magnetic moment, denoted by P
m e associated with this circulating current. Its
magnitude is F1 = BIl

Moving Charges and Magnetism 155


D:\EG_Physics-12_(26-06-2022)\Open_Files\Ch-4\Ch-4
\ 27-Jul-2022 Ved_Goswami Proof-4 Reader’s Sign _______________________ Date __________

  (c) The loop takes up a circular shape with its surface


Since F2 and F4 are equal in magnitude and acting in
opposite directions along the same straight line they normal to the magnetic field, in order to maximise
cancel out each other. Their resultant effect on coil = 0. flux. For a given perimeter, a circle enclosed greater
area than any other shape. 1
Force on arm,
E
PQ = I (PQ) B sin 90° = IlB ½ 27. When k2 is open, current is the galvanometer I =
R+G
According to Fleming left hand rule, direction of this
force is perpendicular to plane of coil directed outwards. GS
When k2 is closed, equivalent resistance is R +
Force on arm, G +S
E
RS = I (RS) B sin 90° = IlB Total current I = 
GS
Direction is into plane of coil.  R+ 1
G +S
The forces acting on PQ and RS are equal, parallel
S S E (G + S)
and acting in opposite directions along different lines Ig = I1 = ×
of action. They form a couple. The effect of this is it S+G S + G R(G + S) + GS
rotates the coil in the anticlockwise direction about the SE
dotted line as axis. =
RG + RS + GS
Torque on coil = either force × arm of couple According to question:
Arm of couple ST = b cos q. I E
Torque = t = IlB × b cos q Ig = =  1
n n( R + G )
= IlbB cos q (lb = area = A)
E SE
= IAB cos q 1 =
n(R + G ) RG + RS + GS
Rectangular coil has n turns
t = nIBA cos q    RG + RS + GS = nRS + nSG
Note: If normal drawn on plane of coil makes angle a So   G(R + S – nS) = nRS – RS
with direction of magnetic field, then RS (n − 1) (n −1) RS
G= =
q + a = 90° ⇒ q = 90° – a R + S − nS R + S (1 − n)
cos q = cos (90° – a) = sin a (n − 1) RS
\ Torque becomes When R >> S, G= = (n − 1) S  1
 
R
t = nIBA sin a = mB sin a = m × B  ½ V
28. In first case R1 = −G
Where m = magnitude of magnetic dipole moment of Ig
rectangular current loop and m = nIA Where Ig is the current needed for full scale deflection
    
\ τ = m × B = nI A× B . ½ ( ) of the galvanometer
26. (a) No, for the loop to turn around itself i.e., about V
Ig = ...(1) ½
the vertical axes, torque should be is the vertical R1 + G
direction. Let R2 be the required resistance to change the range 0
     to V/2 in this case
As torque τ = m × B = IA × B
For current carrying loop lying in the horizontal V

plane A will be in vertical direction, so torque Ig = 2 ...(2) ½
cannot be in vertical direction. R2 + G
So it is not possible for the loop to turn around itself Comparing eqn. (1) and (2),
when magnetic field is applied (any direction). 1 V V
(b) For the loop to be in stable equilibrium, the magnetic =
R1 + G 2(R 2 + G)
dipole moment should be parallel to applied 2R2 + 2G = R1 + G
magnetic field i.e.,
R −G
q = 0°. R2 = 1 or G = R1 − 2R 2  1
In this orientation, the magnetic field produced by 2
Let R be the resistance requires to change the range to
the current carrying loop is in the same direction as
2V.
the applied external magnetic field.
2V
So the total magnetic field and their magnetic flux So Ig = ...(3) ½
will also be maximum. 1 R +G

156 Physics–12
D:\EG_Physics-12_(26-06-2022)\Open_Files\Ch-4\Ch-4
\ 27-Jul-2022 Ved_Goswami Proof-4 Reader’s Sign _______________________ Date __________

Comparing eqn. (2) and (3), IgG = (I0 – Ig)S


V  Ig 
2V 2
\ S=  G
=  I0 − I g 
R +G R2 + G
(b) Effective resistance of ammeter (A)
4(R2 + G) = R + G
GS
4R2 + 4G – G = R ⇒ reff. =
G+S
or R = 4R2 + 3G = 4R2 + 3(R1 – 2R2)
= 4R2 + 3R1 – 6R2 32.  Q
B

⇒         R = 3R1 − 2R 2  ½ q
29. It is a device used to detect the presence of current in l
R 
a circuit. M
Principle: It is based on the principle that a current P
carrying coil placed in a magnetic field experiences a I
torque and the magnitude of the torque depends upon the
b
strength of the current. For working and construction, see
S
moving coil galvanometer of important point-4. 1 
(i) Radial magnetic field: Due to radial magnetic field, FPQ = IlB ⊗
plane of the coil is always parallel to the magnetic 
field thus the deflection produced is directly FRS = IlB 
proportional to the current and the scale is linear.1 
FQR = I b B sin (90° – q) = I b B cos q up
(ii) Cylindrical soft iron core helps in making the field

stronger and radial. FSP = I b B sin (90° – q) = I b B cos q down
NBA  
Current sensitivity IS =
K only FPQ and FRS form a couple to apply torque on
N → no. of turns, B → Mag. field strength loop
A → Area of the coil, K → restoring torque per unit t = FPQ (AQ) = (I l B)(b sin q)
twist = I (l b) B sin q = (IA)B sin q
As such a galvanometer cannot be used for t = mB sin q [ m = IA]
measuring the current, because an ideal ammeter
should have zero or very low resistance. A small for radial magnetic field, refer solution 29(i).
shunt resistance need to be connected in its parallel 33. Here, N = 100, r = 10 cm = 0.1 m, I = 3.2 A
in order to increase its range. 1 (a) Magnetic field at the centre of the coil
30. See Solution of Question 24. 2
Nµ 0 I 100 × 4π ×10 −7 × 3.2
Significance of Negative Sign: The –ve sign indicates B = = = 2 × 10 − 3 T  1
2r 2 × 0.1
that angular momentum of the electron is opposite in
direction to the magnetic moment. 1 (b) Magnetic moment m = NIA = 100 × 3.2 × 3.14 ×
31. (i) Current Sensitivity: It is defined as the amount of (0.1)2 = 10 Am2 1
deflection produced per unit magnitude of current (c) B = 2T (Given), t = mB sin q
passed. In the initial orientation magnetic field is along the
φ NAB axis of the coil so q = 0°
CS = or CS =
I k ti = 0
(ii) (a) Finally q = 90° so tf = mB sin 90° = mB
G
I0 Ig tf = 10 × 2 = 20 Nm. 1
I 0 – Ig (d) Torque acting = MB sin q = Ia
I → moment of inertia
S
a → angular acceleration
Galvanometer can be converted into an ammeter

by connecting a small shunt resistance parallel to So = mB sin q
galvanometer (G) coil so that dt

Moving Charges and Magnetism 157


D:\EG_Physics-12_(26-06-2022)\Open_Files\Ch-4\Ch-4
\ 27-Jul-2022 Ved_Goswami Proof-4 Reader’s Sign _______________________ Date __________

dω dθ ^ ^ ^
I × = mB sin θ = 0.06 i × 0.3 k = − 1.8 × 10 −2 j Nm
d θ dt
= 1.8 × 10–2 Nm along –ve y-axis. 1
or I wdw = mB sin qdq 1  
^ ^
Integrating both the sides with limits as (b) IA = 0.06 i Am 2 , B = 0.3 k T
π   
q = 0° to θ = −2
τ = IA × B = 1.8 × 10 i × k
^ ^
2
π ^
ωf
2 = − 1.8 × 10 −2 j Nm
I ∫ ωd ω = mB ∫ sin θ d θ
= 1.8 × 10–2 Nm along –ve y-axis. 1
0 0
ωf π
  ^  ^
(c) IA = 0.06  − j  Am 2 , B = 0.3 k T
I ∫ ωd ω = mB ( − cos θ )02  
0
 −2  ^ ^ −2 ^
I ω2
ωf
π τ = 10 × 1.8  − j × k  = − 1.8 ×10 i Nm
2 = mB ( − cos θ ) 2
0
0 1.8 × 10–2 Nm along –ve x-axis. 1
I ω 2f 
 π  (d) τ = 1.8 × 10 −2 Nm, it is at 30° + 90° = 120° with
= mB  − cos + cos 0° = mB
2  2 
–ve x-axis or240° with +ve x-axis. ½
1
2mB  2 × 20  2
 ^  ^
wf = = = 20 rad/sec  1 (e) IA = 0.06 k Am 2 , B = 0.3 k T
I  .1   
 ^ ^
34. B = 3000 G = 0.3T, So τ = IA × B = 0 as k × k = 0  1
A = 10 × 5 × 10–4 = 5 × 10–3 m2   ^ 
     2 ^

(a) τ = m × B, m = IA (f) IA = 0.06  − k  Am , B = 0.3 k T


   ^ ^
= IA × B = 12 × 5 × 10 −3 i  × 0 .3 k ^
So τ = 0 as k × k = 0 

^
½
 

Case Based Question


I. The galvanometer can be converted into ammeter The resistance of ammeter,
by connecting a low resistance wire in its parallel R g # RS
(shunt). Let an ammeter consists of a galvanometer    RA =
R g + RS
of resistance Rg and a shunt resistance RS. The current The galvanometer can be converted into voltmeter
to be measured and will distributes as Ig and I – Ig by connecting a large resistance in its series. Let a
through galvanometer and shunt wire respectively. voltmeter consists of a galvanometer of resistance
\ Rg and RS are in parallel. Rg and large resistance in its series RS. If current is
\ Ig Rg = (I – Ig) RS passed through it, then potential difference across the
element with which is connected,
V = I( Rg + RS)

The resistance required to convert a galvanometer


or resistance required to convert a galvanometer into voltmeter
into ammeter, V
Ig Rg RS = I – Rg
RS = The resistance of voltmeter,
(I − I g)
RV = Rg + RS

158 Physics–12
D:\EG_Physics-12_(26-06-2022)\Open_Files\Ch-4\Ch-4
\ 27-Jul-2022 Ved_Goswami Proof-4 Reader’s Sign _______________________ Date __________

1. A galvanometer coil has a resistance of 12 W and 3. A galvanometer of resistance 50 W gives full scale
the metre shows full scale deflection for a current deflection for a current of 0.05 A.The diameter of
of 3 mA. In order to convert the metre into a the shunt wire is 2 mm and its resistivity is 5 ×
voltmeter of range 0 to 18 V, value of resistance 10–7 W m. The length of the shunt wire required
will be to convert the galvanometer into an ammeter of
(a) 5560 W (b) 5988 W range 0 to 5A will be
(a) 3.175 m (b) 4.2 m
(c) 5980 W (d) 5790 W (c) 5.21 m (d) 6.23 m
2. A galvanometer coil has a resistance of 15 W and 4. The value of resistance for an ideal voltmeter is
the metre shows full scale deflection for a current (a) small (b) infinite
of 4 mA. In order to convert the metre into an (c) zero (d) very small
ammeter of range 0 to 6 A, the value of shunt will 5. The value of resistance for an ideal ammeter is
be (a) zero (b) infinite
(a) 10 k W (b) 5 m W (c) large (d) very large
(c) 15 m W (d) 10 m W Ans. 1. (b) 2. (d) 3. (a) 4. (b) 5. (a)

IMPORTANT FORMULAE
Formulae Symbols Application

1. Biot Savart Law dB = magnetic field at a point distance r To find magnetic field at a point due to
µ Idl sin θ due to a current element. m0 = permeability current element. To find magnetic field
dB = 0 ×
4π r2 of free space I = current through wire q due to a straight line.
= angle between current element Idl and
position vector r.

2. µ 0 NI a 2 B = magnetic field due to a circular coil of Magnetic field at centre, x = 0


B= 3 N turns at distance x from its centre. (on its B = µ 0 NI
2 (a 2 + x ) 2
2 axis) 2a
a = Radius of coil

3. µ 0 NI B = magnetic field Magnetic field due to a straight conductor


B= r = perpendicular distance from wire to of infinite length
2π r
point of observation.

4. Ampere's circuital law   Magnetic field due to a solenoid B = m0



  ∫ B.dl = Line integral of magnetic field on nI
∫ B.dl = µ0I a closed path.
n → no. turns per unit length of the coil.

5. F = q(v × B

) F = Force Force acting on a charge particle in
V = Velocity of charge particle magnetic field.
F = B q v sinf
q = charge of the particle

6.     Force on charged particle in simultaneous Total Lorentz force


F = q  E + ( V × B )
  Electric and magnetic field.

7.    F = Magnetic force on a current carrying To find force acting on a current carrying


(
F = I L×B ) conductor of length L conductor in a magnetic field.
F = B I L sin f B = magnetic field.

Moving Charges and Magnetism 159


D:\EG_Physics-12_(26-06-2022)\Open_Files\Ch-4\Ch-4
\ 27-Jul-2022 Ved_Goswami Proof-4 Reader’s Sign _______________________ Date __________

8. F µ 0 I1 I2 F Force per unit length between two


= = Force per unit length between two parallel current carrying conductors.
l 2π r l
parallel current carrying I1 and I2
r = distance between the conductor.
(q is the angle between perpendicular to the
coil and the magnetic field)

9. t = BINA sin q t = orque experienced by a current loop of Magnetic field due to a straight conductor
area A in magnetic field B of infinite length
N = Number of turns of coil.
I = current

10.  ig  S = shunt required, Conversion of Galvanometer into Am-


S=
 i − ig
G
 G = Galvanometer Resistance, meter.
  ig = maximum current through galvanom-
eter
(0-i) = range of ammeter

11. R = (V/ig) – G R = High resistance in series Conversion of Galvanometer into Volt-


meter.

COMMON ERRORS
S.No. Errors Corrections
1. Direction of magnetic field Using thumb rule for finding direction of magnetic field due
to a current carrying conductor
2. Relation between current and voltage sensitivity In numerical the usage of ‘increased by’ and ‘increased to’
must be distinguished.

160 Physics–12
D:\EG_Physics-12_(26-06-2022)\Open_Files\Ch-4\Ch-4
\ 27-Jul-2022 Ved_Goswami Proof-4 Reader’s Sign _______________________ Date __________

REVISION CHART

Magnetic field on the axis of a circular


current carrying coil:
Let n — no. of turns
I – current flowing
a – radius of the loop Magnetic field at the centre of a Force on a charge moving in a magnetic
x – distance of the point from the centre circular current carrying coil. field i.e., magnetic Lorentz force:
 
of the loop. Magnetic field at the centre of a
circular current carrying coil of n
(
F = q v×B )
q → charge, v →
µ 0 2nIπa 2 velocity, B→ Mag. field strength
B= 3 µ o nI
turns B = F ^ v and F ^ B
4π ( a 2 + x 2 ) 2 2r
Magnetic field at the centre Particle will describe a circular path
If point P is very far away i.e., x >>> a if v ^ B
of current carrying loop is nearly
as area of the loop A = pa 2 then
 uniform. Definition of 1 Tesla: If a charge of
µ 2nIA 2µ m
B= 0 3 = 03 Circular current carrying coil 1C is moving with a speed of 1 m/s at an
4πx 4πx angle of 90° to the magnetic field and it
acts as a dipole with one face acting
Magnetic dipole moment of a current as north pole and the other as south experiences a force of 1N, then magnetic
carrying loop m = nIA. pole. field is 1 Tesla.

Moving Charges and Magnetism


Biot Savart Law: The magnitude of the magnetic field dB due to a small current carrying element is:
(i) Directly proportional to the current I flowing through it dB ∝ I
(ii) Directly proportional to the length dl of the current element dB ∝ dl
(iii) Directly proportional to sin q.
1
(iv) Inversely proportional to the square of the distance r of the point P from the current element dB ∝
r2
Combining all these four factors
I
I dl sin θ µ Idl sin θ P
dB ∝ = 0 (for vacuum)
r2 4π r 2 A r
θ
µ0 is constant, called permeability of free space.

 µ 0 Idl × r B
In vector form dB =
4π r 3
SI unit of B is 1 Tesla

Moving Charges and Magnetism 161


D:\EG_Physics-12_(26-06-2022)\Open_Files\Ch-4\Ch-4
\ 27-Jul-2022 Ved_Goswami Proof-4 Reader’s Sign _______________________ Date __________

IMPORTANCE OF EACH TOPIC AND FREQUENTLY ASKED TYPES OF QUESTIONS

☞ Important Topics
1. Questions based on Biot Savart Law.
2. Questions based on working and principle of moving coil Galvanometer.
* Maximum weightage is of Moving Coil Galvanometer.

1. Calculate the magnetic field due to a circular coil of 500 turns and of mean diameter 0.1 m, carrying a current of
14A (i) at a point on the axis distance 0.12m from the centre of the coil (ii) at the centre of the coil.
[Ans. (i) 5.0 × 10–3 Tesla; (ii) 8.8 × 10–2 Tesla]
2. An electron of kinetic energy 10 keV moves perpendicular to the direction of a uniform magnetic field of O.S milli
tesla. Calculate the time period of rotation of the electron in the magnetic field. [Ans. 4.467 × 10–8 s]
3. If the current sensitivity of a moving coil galvanometer is increased by 20% and its resistance also increased by
50% then how will the voltage sensitivity of the galvanometer be affected? [Ans. 25% decrease]
4. What is the magnitude of magnetic force per unit length on a wire carrying a current of 8A and making an angle
of 30° with the direction of a uniform magnetic field of 0.15T? [Ans. 0.6 Nm–1]
5. A straight wire of mass 200g and length 1.5 m carries a currenl of 2A. It s suspended in mid-air by a uniform
horizontal magnetic field B. What is the magnitude of the magnetic field? [Ans. 0.657 T]
6. A rectangular loop of sides 25 em and 10 em carrying current of 15A is placed with its longer side parallel to a
long straight conductor 2.0 cm apart carrying a current of 25A. What is the new force on the loop?
[Ans. 7.82 × 10–4 N towards the conductor]
7. In a chamber of a uniform magnetic field 6.5G is maintained. An electron is shot into the field with a speed of 4.8
× 106 ms–1 normal to the field. Explain why the path of electron is a circle.
(a) Determine the radius of the circular orbit (e = 1.6 × 10–19 C, me = 9.1 × 10–31 kg)
(b) Obtain the frequency of resolution of the electron in its Circular orbit. [Ans. (i) 4.2 cm, (ii) 1.8 MHz]

162 Physics–12
D:\EG_Physics-12_(26-06-2022)\Open_Files\Ch-4\Ch-4
\ 27-Jul-2022 Ved_Goswami Proof-4 Reader’s Sign _______________________ Date __________

ASSIGNMENT
I. Objective Type Questions (1 Mark)
1. Multiple choice questions:
(i) A circular of radius R carries a current I. The magnetic field at its centre is B. At what distance from the centre
B
on the axis of the coil, the magnetic field will be ?
8
(a) 2R (b) 3R (c) 2R (d) 3R
(ii) Ampere’s circuital theorem states that
       
(a) ∫ B · dl = µ0 I (b) ∫ B · dl = ε0 I (c) ∫ B · dl = I (d) ∫ E · dl = µ0 I
C µ0
2. Fill in the blanks:
(i) Magnetic dipole moment of a current carrying loop is .......... .
(ii) A galvanometer of resistance 25 W is shunted by a 2.5 W wire. The part of total current that flows through the
galvanometer is given by ..........
II. Very Short Answer Type Questions (1 Mark)
3. Define sensitivity of a galvanometer.
4. Write two advantages of a moving coil galvanometer.
5. Write down an expression for the force per unit length between two straight parallel infinite conductors carrying
current I1 and I2 placed r distance apart and define 1A.
III. Short Answer Type Questions-I (2 Marks)
6. An electron moving horizontally with a velocity of 4 × 10 m/s enters a region of uniform magnetic field of 10–5
4

T acting vertically downward as shown in figure. Draw its trajectory and find out the time it takes to come out of
the region of magnetic field.

7. A proton and an alpha particle enter into same magnetic field with equal kinetic energy. Find the ratio of the radii
of the path described by them.
IV. Short Answer Type Questions-II (3 Marks)
8. Using Biot-Savart law, obtain an expression for the magnetic field at a point lying on the axis of a, circular current
carrying conductor passing through its centre.
9. A monoenergetic electron beam of initial energy 18 keV moving horizontally is subjected to a horizontal magnetic
field of 0.4 G normal to its initial direction. Calculate the vertical deflection of the beam over a distance of 30
cm.
10. Draw a labelled diagram of a moving coil galvanometer. State the principle on which it works. What is the importance
of radial magnetic field?


Moving Charges and Magnetism 163


D:\EG_Physics-12_(26-06-2022)\Open_Files\Ch-5\Ch-5
\ 27-Jul-2022 Ved_Goswami Proof-4 Reader’s Sign _______________________ Date __________

Topics Covered
5 Magnetism and Matter

5.1 Bar Magnet 5.2 Magnetic Property of Materials

C hapter map
Magnetism and Matter

Bar magnet Magnetic Properties


of Materials
Properties of a bar
magnet
Diamagnetic Paramagnetic Ferromagnetic
Terms related to bar
substances substances substances
magnet

Magnetic field at a
Cause of Cause of Cause of
point on the axis of a
diamagnetism paramagnetism ferromagnetism
bar magnet

Magnetic field at a Properties of Properties of Properties of


point on the equator diamagnetic paramagnetic ferromagnetic
of a bar magnet substances substances substances
Force and torque
on a bar magnet Curie’s law Temperature
placed in a uniform dependance of
magnetic field ferromagnetism
Solenoid is
equivalent to a bar Permanent
magnet magnets and
electromagnets

Topic 1. Bar Magnet


Properties of Magnet 3. Magnetic poles always exist in pairs i.e. an isolated
1. Attractive property: A magnet attracts small pieces pole of a magnet is not possible. If we break a magnet
of iron, cobalt, nickel etc. Ends of bar magnet where into smaller parts, each part will have 2 poles, north
this attraction is maximum are called poles. and south.
2. Directive property: A magnet, if suspended/pivoted 4. Unlike poles attract each other, like poles repel each
freely, (free to rotate in horizontal plane) it will always other.
point towards geographical north-south of earth. End •• Terms related to bar magnet
of magnet pointing towards geographic north pole is
called north-seeking pole/in short, north pole. The other 1. Magnetic axis: The line passing through the poles of
pole which points towards, geographic south is called a magnet is called magnetic axis.
south-seeking pole or in short, south pole.

164
D:\EG_Physics-12_(26-06-2022)\Open_Files\Ch-5\Ch-5
\ 27-Jul-2022 Ved_Goswami Proof-4 Reader’s Sign _______________________ Date __________

2. Magnetic equator: The line passing through centre  µ qm µ qm


of magnet and at right angle to magnetic axis is called BN = o = o (^i)
4π (NP) 2 4π (r − l) 2
magnetic equator.
   µ  1 1 
Magnetic dipole: An arrangement of two equal and Bnet = BS + BN = o qm ( i^ )  2
− 
opposite poles separated by a small distance is called 4π  (r − l ) (r + l )2 
a magnetic dipole.
µ o qm( ^i )  4rl
 µ 2mr ( ^i )
3. Magnetic dipole moment of bar magnet (m) =  = o
4π  r 2 − l 2)
 ( ) 2
(
 4π r 2 − l 2 2
 )
m = qm × 2l
Magnetic field is in direction of magnetic moment.
where qm = pole strength
Special case: If r >>> l
2l = dipole length.
 µ 2mr ^ µ 2m ^
Direction of magnet dipole moment is from south pole Bnet = o 4 (i ) = o 3 ( i )
to north pole. 4πr 4πr
If a bar magnet is cut into two equal halves along a line 1
\ B∝ 3
perpendicular to its axis. r
•• Magnetic Field on Equator of Bar Magnet: Consider
N S N S a bar magnet of length 2l and pole strength qm.
N S Point P lies on equatorial line of the magnet at a distance
r from its centre.
Force experienced per unit pole strength due to north
pole
Dipole moment (original) ‘m’ = qm × 2l
m µ o qm µ q
Dipole moment of each part = m′ = qml = 2 BN = 2
= o 2 m 2 along NP
4π PN 4π (l + r )
It a bar magnet is cut into two equal parts longitudinally Force experienced per unit pole strength due to south
i.e., along axis of magnet then new dipole moment pole
qm m
m′ = × 2l = BN
2 2
BN cos  P
N S N S BS cos  

BS
N S
m m/2
r 2
l +r
2

2 2
l +r
•• Magnetic Field Lines Properties:
1. They are closed curves which start in air from north
pole and enter into south pole. N

S O
2. They never cross each other. 2l

3. They come out/enter into a surface of magnet µ 0 qm µ q


BS = 2
= o 2 m 2 along PS
normally. 4π (PS) 4π (l + r )
4. Relative closeness of field lines gives strength of the  
BN = BS
magnetic field at that place.
Vertical components of BN and BS are equal and opposite
Magnetic Field of a Bar Magnet at an Axial to each other so they cancel out.
Point Horizontal components will add up.
•• Consider a bar magnet of length 2l and pole strength qm.  2µ q
Bnet = BN cos θ + BS cos θ = o 2 m 2 cos θ
2l 4π (l + r )
S N P
X-axis l
O cos q = 1
r
(l 2 + r 2) 2
 µ q µ qm 2µ o qm l µ om
BS = o m 2 = o (− ^i) | B net | =
4π (l 2 + r 2)
× 1 = 3
4π (SP) 4π (r + l) 2 (l 2 + r 2) 2 4π (l 2 + r 2) 2

Magnetism and Matter 165


D:\EG_Physics-12_(26-06-2022)\Open_Files\Ch-5\Ch-5
\ 27-Jul-2022 Ved_Goswami Proof-4 Reader’s Sign _______________________ Date __________

Direction of B is anti-parallel (q = 180°) to magnetic µ o n I a 2 2l µ o n Il a 2


moment. = =
2r 3 r3
Special cases: (i) If r >>> l
Multiply and divide by 4p, we get
 µ om µ 0m
B = 3 = 3
4πr µ o 4π I na 2l µ o 4nIAl
4π (r 2) 2 = =
4π r3 4π r 3
•• Show that Solenoid is Equivalent to a Bar Magnet:
[n = turns per unit length]
Magnetic field lines of a bar magnet and solenoid show
similar patterns. Let us calculate the axial field of a finite µ o 2 NIA µ o 2m
B= = [∵ m = NIA] ,
solenoid. 4π r 3 4π r 3
Consider a solenoid consisting of ‘n’ terns per unit length. N = n × 2l
Let length of solenoid = 2l and radius = a. Let P be the point
This is also the expression for the magnetic field due to
at a distance r from centre of solenoid where we want to
a bar magnet at an axial point.
find the field intensity.
•• Torque on a Magnetic Dipole Placed in a Uniform
l l Magnetic Field: Consider a dipole of length 2l placed in
a a uniform magnetic field B. Let q be the angle between
P dipole moment and B.

N qm B ^i
x
dx 2l B
r S
qmB θ
Each turn of solenoid is at a different distance from the T
point P. Consider a circular element of thickness = dx of
the solenoid at a distance x from the centre. 
Force experienced by N-pole = qm B i^
No. of turns in element = n × dx

Magnetic field produced by these turns = dB. Force experienced by S-pole = qm B( − i^ )

2µ o I πa 2 n dx Fnet = 0
dB = 3
4π [(r − x) 2 + a 2 ] 2 These forces will form a couple, which will rotate the
dipole. The torque acting
Total magnetic field 
= B = ∫ dB t = F × perpendicular distance between forces

µo n ⋅ I ⋅ a2
+l
dx = qm B × NT = qm B 2l sin θ = mB sin θ
B=
2 ∫ 3  
[(r − x) 2 + −l 2 2
a ] or m = m×B
For the points that are very far away i.e., r >>> l and Time period of oscillation of a dipole placed in a
r >>> a uniform magnetic field:
+l
µo n I a2 µ o n Ia 2 I
B=
2r 3 ∫ dx =
2r 3
[l − ( − l )] T = 2p
mB
, where I is moment of inertia.
−l

EXERCISE 5.1
I. Objective Type Questions (1 Mark) (ii) Magnetic field due to a bar magnet 2 cm long
1. Choose the correct answers from the given options having a pole strength of 100 Am at a point 10
(MCQs). cm from each pole is
(i) A magnet of magnetic moment M is suspended in (a) 2 × 10–4 T (b) 8p × 10–4 T
a uniform magnetic field B. The maximum value (c) 2 × 10–5 T (d) 4 × 10–4 T
of torque acting on the magnet is (iii) S is the surface of a lump of magnetic material.
1 (a) Lines of B are necessarily continuous across
(a) MB (b) 2 MB (c) 2 MB (d) zero
S.
166 Physics–12
D:\EG_Physics-12_(26-06-2022)\Open_Files\Ch-5\Ch-5
\ 27-Jul-2022 Ved_Goswami Proof-4 Reader’s Sign _______________________ Date __________

(b) Some lines of B must be discontinuous across (ii) Magnetic field lines always cross each other.
S. 4. Match the Columns
(c) Lines of H are necessarily continuous across Column-I     Column-II
S.  
(d) Lines of H cannot all be continuous across (i) Magnetic dipole moment (a) m × B
S. of bar magnet
(iv) The material which is not suitable for making a (ii) Magnetic field of a bar (b) q × 2l
permanent magnet is [AI 2020] magnet at an axial point
(a) Steel (b) Ticonal I
(iii) Torque of magnetic dipole (c) 2p
(c) Lead (d) Alnico MB
(v) The magnetic dipole moment of a current carrying µ0 2mr (i )
coil does not depend upon [AI 2020] (iv) Time period of oscillation of (d)
a dipole placed in a uniform 4π ( r 2 − l 2 ) 2
(a) number of turns of the coil
magnetic field
(b) cross-section area of the coil
(c) current flowing in the coil For questions numbers 5 and 6 two statements are given-
one labelled Assertion (A) and the other labelled Reason
(d) material of the turns of the coil

 (R). Select the correct answer to these questions from
(vi) A bar magnet has magnetic dipole moment M . the codes (a), (b), (c) and (d) as given below.
Its initial position is parallel
 to the direction (a) Both A and R are true and R is the correct
of uniform magnetic field B . In this position, explanation of A
the magnitudes of torque and force acting on it (b) Both A and R are true but R is NOT the correct
respectively are [CBSE 2022] explanation of A
(a) 0 and MB (b) MB and MB (c) A is true but R is false

 
(c) 0 and 0 (d) | M × B | and 0 (d) A is false and R is also false
5. Assertion (A): The magnetic field produced by a
(vii) Which of the following statements is correct?
current carrying solenoid is independent of its length
[CBSE 2022] and cross sectional area.
(a) Magnetic field lines do not form closed loops. Reason (R): The magnetic field inside the solenoid
(b) Magnetic field lines start from north pole and uniform
end at south pole of a magnet. 6. Assertion (A): The poles of a bar magnet cannot be
(c) The tangent at a point on a magnetic field line separated[CBSE 2022]
represents the direction of the magnetic field Reason (R): Magnetic monopoles do not exist.
at that point. II. Very Short Answer Type Questions (1 Mark)
(d) Two magnetic field lines may intersect each 1. Can two magnetic field lines intersect? Justify your
other. answer.
2. Fill in the blanks. 2. Magnetic field lines always form a closed loop.
(i) According to the modern theory of magnetism, Comment.
every atom of a substance behaves just like a 3. What is the basic difference between electric and
................ magnetic field lines?
(ii) Ratio of force applied by the pole of a magnet 4. What is the SI unit of (i) pole strength and
on a unit north pole to the magnetic induction is (ii) magnetic moment of a bar magnet?
called .................... 5. What is the name given to the curves the tangent to
which at any point gives the directions of the magnetic
(iii) A free floating magnetic needle at North pole is
field at that point? Can two such curves intersect each
................. to the surface of earth.
other? Justify your answer.
3. State True or False 6. Must every magnetic configuration have a north pole
(i) In a hydrogen atom, the electron moves in an and a south pole? [NCERT]
orbit of radius 0.50 Å making 1016 revolutions per 7. Magnetic field lines show the direction (at every point)
second. The magnetic moment associated with the along which a small magnetised needle aligns (at the
orbital motion of the electron is 1.26 × 10–23 Am2. point). Do the magnetic field lines also represent the

Magnetism and Matter 167


D:\EG_Physics-12_(26-06-2022)\Open_Files\Ch-5\Ch-5
\ 27-Jul-2022 Ved_Goswami Proof-4 Reader’s Sign _______________________ Date __________

lines of force on a moving charged particle at every 17. A magnetic dipole is under the influence of two
point? [NCERT] magnetic fields. The angle between the field directions
8. Magnetic field arises due to charges in motion. Can a is 60° and one of the fields has a magnitude of 1.2 ×
system have magnetic moments even though its net 102 T. If the dipole comes to stable equilibrium at an
charge is zero? [NCERT] angle of 15°with this field, what is the magnitude of
9. How does the (i) pole strength and (ii) magnetic other field?
moment of each part of a bar magnetic change if it is 18. In figure the magnetic needle has magnetic moment
cut into two equal pieces transverse to length? 6.7 × 10–2 Am2 and moment of inertia I = 7.5 ×
10. A hypothetical bar magnet (AB) is cut into two equal 10–6 kg m2. It performs 10 complete oscillations in
parts. One part is now kept over the other, so that the 6.70s. What is the magnitude of the magnetic field?
pole C2 is above C1. If m is the magnetic moment of [NCERT]
the original magnet, what would be the magnetic
moment of the combination, so formed? N

A C1 C2 B θ

19. Two magnets of magnetic moments m and m 3 are


joined to form a cross. The combination is suspended
11. Name the physical quantity having unit J/T. in a uniform magnetic field B. The magnetic moment
m now makes an angle q with the field direction find
 [CBSE S.P. 2020-21]
the value of angle q. [S.P. 2011]
III. Short Answer Type Questions-I (2 Marks) m 3
B
12. Magnetic field lines can be entirely confined within
the core of a toroid, but not within a straight solenoid. θ
m
Why? [NCERT]
13. A magnetic needle suspended freely in a uniform
magnetic field experience a torque but no net force.
An iron nail near a bar magnet, however, experiences 20. A wire of length l m is turned to form a semicircle
a force of attraction in addition to torque, why? find the ratio of old dipole moment to new dipole
14. Two identical looking iron bars A and B are given, one moment.
of which is definitely known to be magnetised. (We 21. Write the four important properties of the magnetic
do not know which one). How would one ascertain field lines due to a bar magnet. [Delhi 2019]
whether or not both are magnetised? If only one is IV. Short Answer Type Questions-II (3 Marks)
magnetised, how does one ascertain which one? [Use 22. Two identical bar magnets P and Q are placed in two
nothing else but the bars A and B]. [NCERT] identical uniform magnetic fields as shown in figure.
15. A bar magnet is placed in a uniform magnetic field Justify that both the magnets are in equilibrium which
with its magnetic moment making an angle q with of these is in stable equilibrium? Given reason for
the field. your answer?
(i) Write an expression for the torque acting on the
magnet and hence define its magnetic moment. P
B
Q
B
(ii) Write an expression for the potential energy of N S S N
the magnet in this orientation, when is this energy
minimum.
16. A short bar magnet of magnetic moment 0.9J/T is
placed with its axis at 30° to a uniform magnetic field. 23. Derive an expression for the torque experienced by a
It experience a torque of 0.063J. magnetic dipole placed in a uniform magnetic field.
(i) Calculate the magnitude of the magnetic field V. Long Answer Type Questions (5 Marks)
(ii) In which orientation will the bar magnet be in 24. (i) A magnetic needle is placed in a uniform magnetic
stable equilibrium in the magnetic field? field with its axis tilted w.r.t. its position of stable
 [Foreign 2012] equilibrium. Deduce an expression for the time
168 Physics–12
D:\EG_Physics-12_(26-06-2022)\Open_Files\Ch-5\Ch-5
\ 27-Jul-2022 Ved_Goswami Proof-4 Reader’s Sign _______________________ Date __________

period of (small amplitude) oscillation of this (ii) If this magnet is replaced by a combination of two
magnetic dipole about an axis, passing through similar bar magnets, placed over each other, how
its centre and perpendicular to its plane. will the time period vary? [S.P. 2011, 2013]

Answers 5.1
I. Objective Type Questions (i) The pole strength will remain unchanged (since
1. (i) (a) (ii) (a) (iii) (a) and (d) pole strength depends on number of atoms in cross-
(iv) (c) (v) (d) (vi) (c) (vii) (c) sectional area).


2. (i) bar magnet(ii)pole strength (ii) The magnetic moment is reduced to half (since M
(iii) vertical ∝ length and here length is halved).
3. (i) True (ii) False m
4. (i)-(b) (ii)-(d) (iii)-(a) (iv)-(c) 2l
5. (b) 6. (a)
–m
II. Very Short Answer Type Questions +m

1. No, if two magnetic field lines intersect, then two l l


tangents can be drawn at the point of intersection, hence
there will be two directions of magnetic field at that –m +m –m +m
point, which is not possible.
2. As magnetic poles always exists in pair so magnetic  
M /2 M /2
field lines run, north pole to south pole outside the
magnet and south pole to north pole inside the magnet. 10. The magnetic moment of each half bar magnet is m
2
They do not end or start from anywhere and always but they will be oppositely directed, so net magnetic
form a closed loop. moment of combination = 0
3. Electric field lines do not form a closed loop, they start
from a positive charge and end on a negative charge. 11. Magnetic dipole moment
Whereas a magnetic field lines form a closed loop which 12. Magnetic field lines can be entirely confined within the
runs from north pole to south pole outside a magnet and core of a toroid, as it has no ends. So the flux through
from south pole to north pole inside a magnet. any cross-section will be zero as should be the case.1
4. (i) SI unit of pole strength is Am. In case of a straight solenoid if the field lines are entirely
(ii) SI unit of magnetic moment of a bar magnet is Am2 confined between two ends of the solenoid, the flux
or J/T. through the cross-section at each end would be non zero,
5. Such curves are called magnetic lines of force. No, two but the flux of the field B through any closed surface
such lines never intersect each other. If they do so, then must always be zero. 1
two tangents can be drawn at the point of intersection,
13. In uniform magnetic field, the forces acting on the two
giving two directions of force at one point, which is not
ends of the needle are equal but opposite. So net force
possible.
is zero. But a torque will act on the needle. 1
6. Not necessarily.
Magnetic field produced by a bar magnet is non-
It is true only if the source of the magnetic field has a
uniform. It is strong near the poles and decreases as we
net magnetic moment.
move away from the magnet. So the force experienced
7. No, the magnetic field lines do not represent the lines
by the two ends of the nail will be different. When an
of force on a moving charge.
iron nail is placed closed to a north pole of a magnet, the
Lorentz force on a charge is normal to B, the magnetic
needle will get magnetised. The end of the nail closer
field.
to the pole (magnet) will become south pole and get
8. Yes, a system can have magnetic moments even though strongly attracted towards the magnet. 1
its net charge is zero. For example in paramagnetic
materials, atoms have not dipole moment though their 14. Keep one bar on the table and bring different ends of
net charge is zero. the other bar closer to one end of the bar placed on the
  table. If in any situation, there is a repulsive force then
9. When a bar magnet of magnetic moment ( M = m2l ,
both are magnetised. If it is always attractive then only
where m is pole strength) is cut into two equal pieces
one is magnetised. 1
transverse to length.

Magnetism and Matter 169


D:\EG_Physics-12_(26-06-2022)\Open_Files\Ch-5\Ch-5
\ 27-Jul-2022 Ved_Goswami Proof-4 Reader’s Sign _______________________ Date __________

To Find out which bar is magnetised, we will do this


I
activity: A bar magnet exhibits maximum strength at T = 2p  1
the poles and minimum in the central region. Hold mB
one bar say A horizontally. Now take the bar B and 4p2 I
or B=
bring its one end close to one end of the bar A and mT 2
then in the middle region. If at both the places same
4 × (3.14) 2 × 7.5 × 10 − 6
force of attraction is experienced then, bar A, which is = = 0.01T  1
held horizontally is an iron bar and bar B is magnet. 6.7 × 10 − 2 × (0.67) 2
If the force of attraction at the ends is more than the 19. In the position of equilibrium 1
force in the middle, then the bar A held horizontally is mB sin q = 3 mB sin (90° − θ) = 3 mB cos θ
magnetised. 1
tan q = 3 or q = 60° 1
15. (i) Torque acting on a magnetic dipole of dipole moment

m placed at angle q to the magnetic field B is 20. N l S
 
   τ = m × B m = qm × l... (1) ½

   τ = mB sin θ
If B = 1T, q = 90° then t = m
So, magnetic moment may be defined as the torque N
O S
acting on a magnetic dipole placed perpendicular to l = pr
a uniform magnetic field of unit strength. 1 l
(ii) Potential energy U = – mB cos q r = [r = radius of semicircle]
p
Potential energy is minimum when q = 0°. 1 2l
 m   m′ = qm (2r ) = qm ×  ... (2) ½
16. (i) We know, τ = × B π

τ = mB sin q
On dividing (1) by (2), we get
0.9B m q ×l ×π π
0.063 = 0.9 × B sin 30° = = m =  1
2 m′ qm × 2l 2
0.063 21. Magnetic fields can be pictorially represented by
   B= = 0.14 T  1
0.45 magnetic field lines, the properties of magnetic field
(ii) For stable equilibrium potential energy should be lines are as follows:
minimum. • The field is tangent to the magnetic field line.
The energy (U) = – mB cos q • Field strength is proportional to the line density.
When q = 0°, U = – mB (minimum) 1 • Field lines cannot cross.
17. In equilibrium, the net torque on magnetic field must be • Field lines are continuous loops.
zero. Therefore, the torques exerted by the two fields 22. For Magnet P: If qm is pole strength
on the dipole must be equal and opposite Then force on N-pole = qmB towards right
t1 = t2
Force on S-pole = qmB towards left
Given B1 = 1.2 × 10–2 T
Net force on P = 0 1
q1 = 15°, q2 = (60° – 15°) = 45°
Angle between B and dipole moment is 180°. so, q =
mB1 sin q1 = mB2 sin q2  1
180º
B sin θ1
⇒ B2 = 1 So torque acting t = mB sin q = 0
sin θ 2
Potential energy = – mB cos q = + mB
sin 15°
\ B2 = 1.2 × 10–2 × For magnet Q: Then force on N-pole = qmB towards
sin 45° right 1
0.2588 Force on S-pole = qmB towards left

= 1.2 × 10–2 × = 4.4 × 10–3 T 1
0.7071 Net force on P = 0 so, q = 0º
18. m = 6.7 × 10– 2 Am2 Torque t = mB sin 0° = 0
Moment of inertia = I = 7.5 × 10– 6 kg m2 P.E. of Q = – mB cos 0° = – mB 1
6.7 For magnet to be in stable equilibrium, P.E. should be
T = = 0.67 sec , B = ?
10 minimum. So, Q is in stable equilibrium.
170 Physics–12
D:\EG_Physics-12_(26-06-2022)\Open_Files\Ch-5\Ch-5
\ 27-Jul-2022 Ved_Goswami Proof-4 Reader’s Sign _______________________ Date __________

23. Magnetic dipole in uniform magnetic field: Consider B


a bar magnet of length 2l placed in a uniform magnetic N
field B. Let q be the angle between dipole moment and
the magnetic field. θ
N B S T
qm B
   1

2l
θ For small displacement, sin θ  θ
qmB S T mΒ

\ a= − θ 1
^  I
Force experienced by N-pole = qm B ( i ) = FN As a is proportional to q and directed opposite to it,
Force experienced by S-pole the bar magnet will execute simple harmonic motion.
^  mB mB
= qm B ( − i ) = FS  1 w2 = ⇒ w=  1
I I
Where qmB = pole strength 2π I
   T= = 2π
Fnet = FN + FS = 0 N ω mB
These 2 forces are equal, opposite, parallel and their line 1 1 mB
Frequency = =  1
of action is not same. So they form a couple which will T 2π I
try to rotate the dipole. (b) (i) If two magnets are placed as shown below

Torque = t = F × perpendicular distance between the mnet = 0
two forces Net moment of inertia = 2I 1
= qmB × NT = qmB 2l sin q = mB sin q So T= ∞
  
τ = m × B 1 S N m
24. Time period of oscillation of bar magnet placed in
a uniform magnetic field m N S

(a) If a bar magnet of dipole moment ‘m’ is placed in a (ii) mnet = 2m


uniform magnetic field B, let q = angle between m Inet = 2I
and B. There is a torque acting on the dipole
2I I
t = –mB sinq T = 2π = 2π  1
2mB mB
If I is moment of inertia of bar magnet, then T will remain same.
torque, t = Ia (a = angular acceleration)
\ Ia = – mB sin q S N m

− mB
  a= I
sin θ S N m

Topic 2. Magnetic Property of Materials


•• Permeability: Extent to which the magnetic field lines The ratio of the flux density of magnetic induction (B)
can enter a substance is called magnetic permeability. inside the material to the flux density in vacuum.
Magnetic permeability is equal to the ratio of the Magnetising field intensity (H): It is the extent to which
magnitude of magnetic induction (B) to the intensity of a magnetising field can magnetise a substance.
magnetising field. B0 = m0nI
Relative permeability is the ratio of the permeability of H = nI = magnetising field intensity
the substance to permeability of free space A turns A
SI unit of H is = = Am −1
µ B m m
    mr = = It is equal to the number of Ampere turns flowing around
µ 0 B0
the unit length of the solenoid.

Magnetism and Matter 171


D:\EG_Physics-12_(26-06-2022)\Open_Files\Ch-5\Ch-5
\ 27-Jul-2022 Ved_Goswami Proof-4 Reader’s Sign _______________________ Date __________

Intensity of magnetisation (M): It is the magnetic field. Examples of diamagnetic substances are Bismuth,
moment developed per unit volume of a material when copper, lead, zinc, tin,gold, silicon, nitrogen, silver.
placed in a magnetising field. •• Cause of diamagnetism: The simplest explanation for
N S diamagnetism is as follows.
Electrons in an atom orbiting around nucleus possess
orbital angular momentum. These orbiting electrons
qm are equivalent to current-carrying loop and thus possess
  orbital magnetic moment. Diamagnetic substances are
m = qm × 2l
 the ones in which resultant magnetic moment in an atom
m qm × 2l qm is zero. When magnetic field is applied,those electrons
M= = =
V A × 2l A having orbital magnetic moment in the one direction
It is also defined as the pole strength developed per unit slow down and those in the opposite direction speed up.
cross sectional area of the material. This happens due to induced current in accordance with
SI unit = Am–1 Lenz’s law. Thus, the substance develops a net magnetic
moment in direction opposite to that of the applied field
Magnetic susceptibility: It measures the ability of the
and hence repulsion.
substance to take up magnetisation when placed in a
magnetising field (H). Diamagnetism is present in all the substances. However,
the effect is so weak in most cases that it gets shifted by
M ∝ H i.e., M = Constant = χ other effects like paramagnetism, ferromagnetism, etc.
H m
•• Properties of diamagnetic substances:
It is the ratio of intensity of magnetisation to the
magnetising field intensity. 1. Diamagnetic substances are feebly magnetized in the
opposite direction of the magnetizing field.
It is a dimensionless and unitless quantity.
2. They are feebly repelled by magnets.
Relation between magnetic permeability and
magnetic susceptibility: Consider a magnetic material
placed in a magnetic field.
Let H = intensity of magnetising field
B0 = µ0ni = µ0H
BM = µH
Bnet = B0 + Bm
The net magnetic field in the specimen is the sum of the
magnetic field in vacuum and the magnetic field due to 3. They tend to move from stronger to weaker part of
the magnetisation of the specimen. the magnetic field.
Total magnetic field is the sum of magnetic field of 4. The field lines are repelled or expelled and the field
vacuum (B0) and the magnetic field due to magnetisation inside the material is reduced.
of the specimen (Bm). 5. When a thin and long rod of a diamagnetic substance
is suspended freely in a uniform magnetic field, it
Bnet = B0 + Bm = µ0H + µ0M
aligns itself perpendicular to the magnetizing field.
M = intensity of magnetisation
6. Their magnetic susceptibility is negative and small.
Bnet = µ0 (H + M) χ
B  M +
= µ 0 1 + 
H  H T
0
m = m0 (H + M) χdia = constant
m
mr = = (1 + cm) –
m0
•• Diamagnetic substances: Diamagnetic substances 7. The relative permeability is positive but less than one.
are those substances which get feebly magnetised in 8. The magnetic susceptibility is independent of
the opposite direction of the magnetizing field. Such temperature.
substances are feebly repelled by magnets and tend 9. The magnetization of a diamagnetic substance lasts
to move from stronger to weaker part of the magnetic so long as the magnetizing field is applied.

172 Physics–12
D:\EG_Physics-12_(26-06-2022)\Open_Files\Ch-5\Ch-5
\ 27-Jul-2022 Ved_Goswami Proof-4 Reader’s Sign _______________________ Date __________

•• Paramagnetic substances: Paramagnetic substances are 9. The intensity of magnetization is directly proportional
those which get weakly magnetised in the direction of the to the strength of the magnetizing field.
magnetising field, when placed in an external magnetic 10. The magnetization of a paramagnetic substance lasts
field. They have tendency to move from a region of weak so long as the magnetizing field is applied.
magnetic field to strong magnetic field. They get weakly •• Ferromagnetic substances: Ferromagnetic substances
attracted to a magnet. Some examples of paramagnetic are those which get strongly magnetised when placed in
substances are manganese, aluminium, chromium, an external magnetic field in the direction of the applied
platinum, sodium, copper chloride and oxygen at STP. field. They have tendency to move from a region of weak
•• Cause of paramagnetism: The individual atoms (or magnetic field to strong magnetic field. They get strongly
ions or molecules) of a paramagnetic material possess attracted to a magnet. Some examples of ferromagnetic
a permanent magnetic dipole moment of their own. On substances are iron, cobalt, nickel and alloys like alnico.
account of the ceaseless random thermal motion of the •• Causes of ferromagnetism: The individual atoms (or
atoms, no net magnetisation is seen. ions or molecules) in a ferromagnetic material possess a
•• In the presence of an external field, which is strong dipole moment as in a paramagnetic material. However,
enough, and at low temperatures, the individual atomic they interact with one another in such a way that they
dipole moment can be made to align and point in the spontaneously align themselves in a common direction
same direction as the applied field. over a macroscopic volume called domain. Domain
•• Curie’s law: Experimentally, one finds that the contains about 1011 atoms. In the absence of magnetic
magnetisation of a paramagnetic material is inversely field magnetic moment of domains are randomly oriented
proportional to the absolute temperature T. so the net magnetic field is zero. When a magnetic field
M = C B0/T is applied, the domains orient themselves in the direction
Equivalently, cm = C µ0/T of applied mag. field and thus it gets strongly magnetised
This is known as Curie’s law: The constant C is called in the direction of applied field.
Curie’s constant. Thus, for a paramagnetic material both In the absence of magnetic field When magnetic field is applied
c and µr depend not only on the material, but also on the
sample temperature.
As the field increases or the temperature decreases, the
magnetisation increases until it reaches the saturation •• Temperature dependence of ferromagnetism: The
value, at which point all the dipoles are perfectly aligned ferromagnetic property depends on temperature. At
with the field. Beyond this, Curie’s law is no longer valid. high enough temperature, a ferromagnet becomes a
•• Properties of paramagnetic substances paramagnet. The domain structure disintegrates with
1. Paramagnetic substances are feebly magnetized in temperature. This disappearance of magnetisation with
the direction of the magnetizing field. temperature is gradual. The temperature of transition
2. They are feebly attracted by magnets. from ferromagnetic to paramagnetic is called the Curie
3. They tend to move slowly from weaker to stronger temperature TC. The susceptibility above the Curie
part of the magnetic field. temperature, i.e., in the paramagnetic phase is described
by, χ m = C′ .
T − TC
•• Properties of ferromagnetic substances:
1. Ferromagnetic substances are strongly magnetized
4. The field lines gets concentrated inside the material, in the direction of the magnetizing field.
and the field inside is enhanced. 2. They are strongly attracted by magnets.
5. When a thin and long rod of a paramagnetic substance
3. They tend to move from weaker to stronger part of
is suspended freely in a uniform magnetic field, it
the magnetic field.
aligns itself parallel to the magnetizing field.
6. Their magnetic susceptibility is positive and small. 4. The field lines gets strongly concentrated inside the
material, and the field inside is greatly enhanced.
7. The relative permeability is positive but slightly more
than one. 5. When a thin and long rod of a ferromagnetic substance
8. The magnetic susceptibility is inversely proportional is suspended freely in a uniform magnetic field, it
to the absolute temperature. quickly aligns itself parallel to the magnetizing field.

Magnetism and Matter 173


D:\EG_Physics-12_(26-06-2022)\Open_Files\Ch-5\Ch-5
\ 27-Jul-2022 Ved_Goswami Proof-4 Reader’s Sign _______________________ Date __________

6. Their magnetic susceptibility is large. It is of the order A soft iron core in solenoid acts as an electromagnet. In
of several thousand. certain applications, the material goes through an ac cycle
7. The relative permeability is positive and large. of magnetisation for a long period. This is the case in
8. The magnetic susceptibility of a ferromagnetic transformer cores. The hysteresis curve of such materials
substance decreases with temperature according to must be narrow. The energy dissipated and the heating
Curie-Weiss law, cm = C'/(T – TC) where TC is Curie will consequently be lesser. Electromagnets are used in
temperature. electric bells, loudspeakers and telephone diaphragms.
9. At a certain temperature called curie point the Diamagnetic Paramagnetic Ferromagnetic
susceptibility suddenly decreases and the substance 1. Weakly repelled Weakly attracted strongly attracted
becomes paramagnetic. by a magnet by a magnet by a magnet
10. The intensity of magnetization depends on the 2. Feebly magne- Feebly magne- strongly magne-
strength of the magnetizing field and also on the past tized in oppo- tized in the direc- tized in the direc-
site direction of tion of the mag- tion of the mag-
magnetic and mechanical history of the material.
the magnetizing netizing field. netizing field.
11. A ferromagnetic substance retains magnetism even field.
after the magnetizing field is removed. 3. Move slowlyMove slowly Move quickly
from strongerfrom weaker to from weaker
Permanent Magnets and Electromagnets to weaker part stronger part of to stronger part
•• Permanent Magnets: Substances which at room of the magnetic the magnetic of the magnetic
temperature retain their ferromagnetic property for a long field. field. field.
period of time are called permanent magnets. 4. A thin and long A thin and long A thin and long
One can also hold a steel rod and stroke it with one end of rod aligns slowly rod aligns slow- rod aligns quick-
a bar magnet a large number of times, always in the same its longer side ly its longer ly its longer side
perpendicular to side parallel to to the magnetiz-
sense to make a permanent magnet. An efficient way to
parallel the mag- the magnetizing ing field.
make a permanent magnet is to place a ferromagnetic rod netizing field field.
in a solenoid and pass a current. The magnetic field of the
5. µr is positive, but
µr is positive, µr is positive,
solenoid magnetises the rod. The hysteresis curve allows
less than 1. slightly more very large.
us to select suitable materials for permanent magnets. than 1.
The material should have
6. c is small and c susceptibility c susceptibil-
(i) High retentivity so that the magnet is strong. negative. is positive more ity is positive and
(ii) High coercivity so that the magnetisation is not erased than 1. large.
by stray magnetic fields, temperature fluctuations or 7. Susceptibility is S u s c e p t i b i l i t y Susceptibility
minor mechanical stress. independent of varies inversely varies inversely
(iii) Further, the material should have a high permeability. temperature. with temperature with temperature
Steel is one-favoured choice. It has a slightly smaller and follow Cu- and follow Curie-
retentivity than soft iron but this is outweighed by the rie’s law. Weiss law upto
much smaller coercivity of soft iron. Other suitable Curie point and
materials for permanent magnets are alnico, cobalt, steel thereafter behave
and ticonal. like a paramag-
netic substance.
•• Electromagnets: Core of electromagnets are made of
ferromagnetic materials which have 8. No effect of tem- Magnetic mo- Magnetic mo-
perature on ment developed ment developed
(i) High permeability (ii) Low retentivity (iii) Less
magnetisation. decreases with decreases with
B-H curve area increase in tem- increase in tem-
Soft iron is a suitable material for electromagnets. perature. perature.

EXERCISE 5.2
I. Objective Type Questions (1 Mark) of 0.6 T at a temperature of 4 K. When the same
1. Choose the correct answers from the given options sample is placed in an external magnetic field of
(MCQs). 0.2 T at a temperature of 16 K, the magnetisation
(i) Paramagnetic sample shows a net magnetisation will be .................. .
of 8 Am–1 when placed in external magnetic field
174 Physics–12
D:\EG_Physics-12_(26-06-2022)\Open_Files\Ch-5\Ch-5
\ 27-Jul-2022 Ved_Goswami Proof-4 Reader’s Sign _______________________ Date __________

32 2 3. Why is the core of an electromagnet made of


(a) Am–1 (b) Am–1 ferromagnetic materials?
3 3
(c) 6 Am–1 (d) 2.4 Am–1 4. Mention two characteristics of a material for making
permanent magnets. [Delhi 2010]
(ii) A dimagnetic substance is
5. The susceptibility of a magnetic materials is 1.9
(a) repelled when south pole of magnet is brought
× 10–5. Name the type of magnetic materials it
near it
represents.[Delhi 2011]
(b) repelled when north pole of magnet is brought
near it 6. The susceptibility of a magnetic materials is – 4.2
× 10–6. Name the type of magnetic materials it
(c) repelled by both the poles of magnet
represents.[Delhi 2011]
(d) attracted by both the poles of magnet
7. How does the intensity of magnetisation of a
2. Fill in the blanks.
paramagnetic material vary with increasing applied
(i) Domain formation is the necessary feature of magnetic field?
.................... .
8. Why is core of an electromagnet made of ferromagnetic
(ii) At Curie point of ferromagnetic material becomes materials?[AI 2010]
........................... .
9. Soft iron is used to make core of electromagnets
(iii) The magnetic field lines are ........................ by a why?
diamagnetic substance. [Delhi 2020]
10. Which is the material used in making the core of a
(iv) The magnetic field lines are ........................ by a moving coil galvanometer?
paramagnetic substance. [AI 2020]
11. In what way is the behaviour of a diamagnetic
3. State True or False material different from that of a paramagnetic, when
(i) Relative permeability of iron is 5500. Its magnetic kept in an external magnetic field? [AI 2016]
susceptibility is 5501. 12. If c stands for the magnetic susceptibility of a given
(ii) The Hysteresis cycle for the material of permanent material, identify the class of materials for which
magnet should be long and wide. (i) – 1 ≤ c < 0 (ii) 0 < c < e (e is a small positive
For questions numbers 4 two statements are given-one number) write the range of magnetic permeability of
labelled Assertion (A) and the other labelled Reason these materials.
(R). Select the correct answer to these questions from 13. Relative permeability µr of a material has value lying
the codes (a), (b), (c) and (d) as given below. 1 < µr < 1 + e (where e is a small quantity). Identify
(a) Both A and R are true and R is the correct the nature of the magnetic material.
explanation of A 14. Which of the following substances are :
(b) Both A and R are true but R is NOT the correct (i) paramagnetic (ii) diamagnetic
explanation of A Bi, Al, Cu, Ca, Ni, Fe
(c) A is true but R is false 15. How is an electromagnet different from a permanent
(d) A is false and R is also false magnet?
4. Assertion (A): Ferromagnetic substances get strongly 16. What is the basic difference between the atoms
magnetised when placed in an external magnetic field or molecules of a diamagnetic and paramagnetic
in the direction of the applied field. material?
Reason (R): Domains oriented themselves in the 17. The hysteresis loop of a soft iron piece has a much
direction of applied magnetic field. smaller area than that of a carbon steel piece. If
the material is to go through repeated cycles of
II. Very Short Answer Type Questions (1 Mark) magnetisation, which piece will dissipate greater heat
1. Would the maximum possible magnetisation of energy?[NCERT]
a paramagnetic sample be of the same order of 18. A certain region of space is to be shielded from
magnitude as the magnetisation of a ferromagnet? magnetic fields. Suggest a method. [NCERT]
[NCERT] 19. The susceptibility of a magnetic material is – 2.6 ×
2. What are permanent magnets? Give one example. 10–5. Identify the type of magnetic material and state
[Delhi 2013] its two properties. [Delhi 2012]

Magnetism and Matter 175


D:\EG_Physics-12_(26-06-2022)\Open_Files\Ch-5\Ch-5
\ 27-Jul-2022 Ved_Goswami Proof-4 Reader’s Sign _______________________ Date __________

20. Explain why steel is preferred for making permanent (i) Identify the materials A and B.
magnets while soft iron is preferred for making (ii) For the material A, plot variation of intensity of
electromagnets. magnetisation versus temperature.
III. Short Answer Type Questions-I (2 Marks) 30. The following figure shows the variation of
21. The susceptibility of a magnetic materials is 2.6 × magnetisation versus applied magnetic field intensity
10 . Identify the type of magnetic material and state
–5 H, for two magnetic materials A and B.
its two properties. [Delhi 2012] M
22. An iron bar is heated to 1000°C and then cooled in a A
magnetic field free space–will it retain magnetism.
Intensity of
23. Define intensity of magnetisation of a magnetic magnetisation H
material. How does it vary with temperature for a
paramagnetic material? [Delhi 2001] B
24. Out of the two magnetic materials, ‘A’ has relative –M

permeability slightly greater than unity while ‘B’ has (i) Identify materials A and B.
less than unity. Identify the nature of the materials (ii) Draw the variation of susceptibility with
‘A’ and ‘B’. Will their susceptibilities be positive or temperature for B.
negative?[Delhi 2014] 31. State Curie law of magnetism.
25. A uniform magnetic field gets modified as shown 32. Show diagramatically the behaviour of magnetic
below, when two specimens X and Y are placed in it. field lines in the presence of (i) paramagnetic and
(i) Identify the two specimens X and Y. (ii) diamagnetic substances. How does one explain
(ii) State the reason for the behaviour of the field lines this distinguishing feature? [AI 2014, 2015]
in X and Y.
IV. Short Answer Type Questions-II (3 Marks)
33. Two similar bars, made from different materials P and
Q, are placed one by one in a non uniform magnetic
field. It is observed that (i) bar P tends to move from
weak to strong field region (ii) bar Q tends to move
from strong to weak field region.
26. Name materials, one is each case, which are suitable What is the nature of the magnetic materials used for
for (i) a permanent magnet (ii) core of an electric making these two bars?
transformer. Justify your answer.
Show, with the help of a diagram, the behaviour of
27. Why does a paramagnetic substance display greater
the field lines, due to an external magnetic field, near
magnetisation for the same magnetising field, when
each of these two bars.
cooled? How does a diamagnetic substance respond
to similar temperature change? 34. Mention two properties of soft iron due to which it
28. Three identical specimens of magnetic materials is preferred for making an electromagnet.
Nickel, Antimony, Aluminium are kept in a uniform  [AI 2016]
magnetic field. Draw the modifications in the field 35. Distinguish between diamagnetic and ferromagnetic
lines in each case. Justify your answer. materials in respect of their (i) intensity of
29. The following figure shows the variation of intensity magnetization (ii) behaviour in non uniform magnetic
of magnetisation versus the applied magnetic field field and (iii) susceptibility.
intensity H, for two materials A and B [AI 2008] 36. Define magnetic susceptibility to a material. Name
M
two elements, one having positive susceptibility and
B
the other having negative susceptibility. What does
negative susceptibility signify? [Delhi 2008, 2011]

A
37. (a) An iron ring of relative permeability µr has
windings of insulated copper wire of n turns per
metre. When the current in the windings is I, find
H the expression for the magnetic field in the ring.

176 Physics–12
D:\EG_Physics-12_(26-06-2022)\Open_Files\Ch-5\Ch-5
\ 27-Jul-2022 Ved_Goswami Proof-4 Reader’s Sign _______________________ Date __________

(b) The susceptibility of a magnetic material is 0.9853. (b) A circular loop of radius r, having N turns and
Identify the type of magnetic material. Draw the carrying current I, is kept in the XY plane. It is
modification of the field pattern on keeping a piece then subjected to a uniform magnetic field B = Bx
of this material in a uniform magnetic field. i + By j + Bz k. Obtain expression for the magnetic
 [Delhi 2018] potential energy of the coil-magnetic field system
38. Write three points of differences between para-, dia- OR
and ferro- magnetic materials, giving one example
(a) A long solenoid with air core has n turns per unit
for each. [Delhi 2019]
length and carries a current I. Using Ampere’s
V. Long Answer Type Questions (5 Marks) circuital law, derive an expression for the
39. Distinguish the magnetic properties of dia, para and magnetic field B at an interior point on its axis.
ferromagnetic substances in terms of: Write an expression for magnetic intensity H in
(i) Susceptibility (ii) Magnetic permeability the interior of the solenoid.
(iii) Give an example of each of these materials. (b) A (small) bar of material, having magnetic
Draw the field lines due to an external magnetic susceptibility c, is now put along the axis and
field near a (a) diamagnetic (b) paramagnetic near the centre, of the solenoid which is carrying
substances.[AI 2007] a d.c. current through its coils. After some time,
40. (a) Write the expression for the equivalent magnetic the bar is taken out and suspended freely with
moment of a planer current loop of area A, an unspun thread. Will the bar orient itself in
having N turns and carrying a current i. Use the magnetic meridian if (i) c < 0 (ii) c > 1000?
expression to find the magnetic dipole moment of Justify your answer in each case.
a revolving electron. [CBSE S.P. 2018-19]

Answers 5.2
I. Objective Type Questions alignment is complete, Intensity of magnetisation is
1. (i) (b) (ii) (c) independent of applied magnetic field.
2. (i) Ferromagnetic (ii) paramagnetic 8. Because ferromagnetic material like soft iron has high
permeability and high susceptibility.
(iii) opposed (iv) concentrate inside
9. Because soft iron has (i) high permeability and
3. (i) False (ii) True susceptibility (ii) less B-H curve area so lesser energy
4. (a) is wasted during the process of magnetisation and
II. Very Short Answer Type Questions dimagnetisation.
10. Soft iron is used in making the core of a moving coil
1. Yes, it is possible to have maximum magnetisation
of a paramagnetic sample to be of the same order galvanometer, as it has high permeability and low
of magnitude as for a ferromagnetic substance. But hysteresis loss.
saturation requires very high magnetising field. 11. A diamagnetic substance when placed in an external
2. Substances that retain their ferromagnetic property for magnetic field, it will be feebly repelled by the magnet
a long period of time at room temperature are called and move from the region of strong magnetic field to
permanent magnets. Bar magnet is a permanent magnet. the region of weak magnetic field.
12. (i) Diamagnetic − 1 ≤ χ < 0 and 0 ≤ mr < 1
3. The core of an electromagnet made of ferromagnetic
materials because it is (i) high permeability (ii) High (ii) Paramagnetic 0 < c < e and 1 < µr < 1 + e
susceptibility. 13. Paramagnetic
4. For permanent magnet the material must have high 14. Bi and Cu are diamagnetic
susceptibility and high coercivity. Al and Ca are paramagnetic
5. Susceptibility is small positive, so material is 15. The polarity and strength of an electromagnet can be
paramagnetic. changed by changing the strength of the current flowing
6. Susceptibility of material is negative, so given material through it. Whereas once made the strength and polarity
is diamagnetic. of a permanent magnet cannot be changed.
7. Much before saturation, intensity of magnetisation 16. In a diamagnetic material, electrons in atoms or
increases proportionally with increase in applied molecules occur in pair. Their magnetic moments cancel
magnetic field. At sufficiently high values, when dipole out. Net atomic or molecular magnetic moment is zero.

Magnetism and Matter 177


D:\EG_Physics-12_(26-06-2022)\Open_Files\Ch-5\Ch-5
\ 27-Jul-2022 Ved_Goswami Proof-4 Reader’s Sign _______________________ Date __________

In case of paramagnetic materials atomic/molecular 25. (i) X is diamagnetic and Y is paramagnetic. ½ + ½


magnetic dipole moment is non-zero due to the presence (ii) When a diamagnetic substance is placed in a
of unpaired electrons. magnetic field, it gets feebly magnetised opposite
17. Carbon steel piece will dissipate greater heat energy to the applied field,so the field lines are repelled or
because the energy dissipated per cycle is proportional expelled and the magnetic field inside the material
decreases. ½
to the area of the hysteresis loop.
When a paramagnetic substance is placed in a
18. To shield a region from magnetic field, we can surround magnetic field, it gets feebly magnetised in the
it by soft iron rings. Soft iron being ferromagnetic, will direction of the applied field, so the field lines, gets
draw field lines into it. concentrated inside the material. ½
19. As susceptibility is small and negative so the material 26. (i) Steel is used for making permanent magnet because
is diamagnetic. of its high coercivity. 1
Properties: (i) It is feebly repelled by a magnet. (ii) Soft iron is used for making core of a transformer
because it has high permeability and low hysteresis
(ii) It moves slowly from region of strong magnetic loss. 1
field to the region of weak magnetic field. 27. As a paramagnetic substance is cooled, atomic dipoles
20. Steel has high retentivity and high coercivity. The tend to get aligned with the magnetising field (because
material used for making permanent magnet must have of decrease in thermal energy). So it displays a greater
these characteristics, so steel is preferred for making magnetisation when cooled. 1
permanent magnet. There is no change in magnetisation of a diamagnetic
Soft iron has high susceptibility and low coercivity and substance when it is cooled because in case of a
less area of B-H Curve. The material used for making diamagnetic substance, the magnetisation is due to
electromagnet must have these characteristics, so soft Lorentz force which in not dependent on temperature.
iron is preferred for making electromagnets.  1
28. Ni is ferromagnetic
21. As susceptibility is positive and small so it is a
S N
paramagnetic substance. ½+½
Properties: (1) They are feebly attracted by a magnet.
 ½      ½
(2) When thin rod of these materials is suspended freely Aluminium is paramagnetic
in a uniform magnetic field, it will align itself slowly S N
in the direction of applied magnetic field. ½
22. The curie point of iron is 770°C. When it is heated
to 1000°C, iron becomes paramagnetic and looses its
magnetism. When it is cooled in magnetic field free      ½
space, it does not regain its magnetism. 1+1 Antimony is diamagnetic
23. It is defined as the magnetic dipole moment developed
per unit volume of the specimen when it is placed in a
magnetising field. 1     ½ + ½
The intensity of magnetisation of a paramagnetic
substance varies inversely with its temperature T. 29. (i) As χ = M = slope of the line
H
(before saturation) 1 As slope of B > slope of A and both are positive
24. As for material A, µr is slightly greater than one so it is  ½
a paramagnetic substance. ½ So A is paramagnetic ½
Material B: As µr < 1 (slightly) B is ferromagnetic ½
(ii) M
So it is a diamagnetic substance
As µr = 1 + c, ½
 ½
So c = µr – 1
For material A, c will be small and positive. ½
Temp.
For material B, c will be small and negative. ½ M— Intensity of magnetisation
178 Physics–12
D:\EG_Physics-12_(26-06-2022)\Open_Files\Ch-5\Ch-5
\ 27-Jul-2022 Ved_Goswami Proof-4 Reader’s Sign _______________________ Date __________

M 33. (i) P is a paramagnetic magnetic bar. ½


30. (i) χ =
= Slope of the line ½ S N
H
Slope of line A is positive and small so it is 1
paramagnetic. ½
Slope of line B is small and negative, so it is
diamagnetic. ½ (ii) Q is a diamagnetic bar. ½
(ii) χ

N S  1
T  ½

–χ
34. High permeability. 1½
31. Curie’s law of magnetisation states that far away from Low magnetic hysteresis loss. 1½
saturation, the susceptibility of a paramagnetic material
is inversely proportional to the absolute temperature.½ 35.
Property Diamagnetic Ferromag-
netic
Explanation: The intensity of magnetisation M is (i)
1. Intensity of Gets feebly mag- Gets strongly
Directly proportional to the magnetising field intensity magnetisa- netised opposite magnetised in
H, as this field H is responsible for the alignment of tion to the magnetis- the direction 1
atomic magnetic dipoles. ½ ing field. of the magnet-
(ii) Inversely proportional to the absolute temperature ising field.
T, as the thermal energy will oppose the alignment 2. Behaviour in Moves slowly Moves swift-
of atomic magnetic dipoles. non-uniform from region of ly from region
H magnet field strong magnet- of weak mag-
So M∝ or M = CH  ½ isation field to netising field 1
T T
region of weak to region of
M C magnetising strong mag-
c= = field. netising field.
H T
3. Susceptibil- Susceptibility is Susceptibili-
1 ity low and negative. ty is high and 1
So c∝  ½
T positive.
32. (i) A diamagnetic substance gets feebly magnetised in 36. The ratio of intensity of magnetisation to the magnetising
the opposite direction of the magnetising field. Thus field intensity is called magnetic susceptibility. ½
the magnetic field lines becomes less dense in the
Positive susceptibility – Aluminium, Iron ½
diamagnetic material. c is small and negative. ½
Negative susceptibility – Bi, Cu ½
Negative susceptibility means the specimen will get
magnetised opposite to the magnetising field and expels
N S  ½ the magnetic field lines or the magnetic field intensity
inside the specimen is less than the field outside it.
37. (a) Relative permeability of the ring
m
mr =  ½
(ii) A paramagnetic substance gets feebly magnetised in m0
the direction of the applied magnetic field. Thus the where m is the permeability of the iron ring.
field lines become more dense in the paramagnetic
According to Ampere’s circuital law
substance. c is small and positive. ½ → →
S N ∫ B ⋅ dl = m0 m r ( NI)  ½
 ½ → →
∫ B ⋅ dl = m0 m r NI

Magnetism and Matter 179


D:\EG_Physics-12_(26-06-2022)\Open_Files\Ch-5\Ch-5
\ 27-Jul-2022 Ved_Goswami Proof-4 Reader’s Sign _______________________ Date __________

or B(2pr) = m0mrNI 2. The magnetic lines of force tend to pass through the
material.
µ 0 µ r NI
or B= 3. When suspended freely in a uniform magnetic field,
2πr
or B = m0mrnI ½ it aligns itself parallel to the direction of the magnetic
field. 1
where n is the number of terms per unit length.
39. Property Diamag- Paramag- Ferro-
netic netic magnetic
r O (i) S uscep- c is small c is small c is large 1½
t i b i l i t y and nega- and positive and posi-
(c) tive tive
I (ii) M a g . µr is small µr is slightly µr is of 1½
P e r m e - lies be- more than the order
(b) a b i l i t y tween One. of
(µr) 0 and 1. Ex. Alu- t h o u -
½ Ex. Anti- minium sands.
mony Ex. Iron
(iii) (a) Antimony,

Susceptibility of a paramagnetic substance is
small and positive. Hence the given material is a S N
paramagnetic substance. ½       1
Modification of the magnetic field pattern on
keeping a paramagnetic material in a uniform
magnetic field. ½ (b) Aluminium,
38. Properties of diamagnetic substances:
1. When placed in a non-uniform magnetic field, it S N     1
tends to move from stronger to weaker regions of
the magnetic field. For example, when a diamagnetic
liquid in a watch glass is placed on two pole pieces 40. (a) The equivalent magnetic moment is given by
lying close to each other, we observe a depression
m = NiA½
in the middle and when the pole pieces are placed
sufficiently apart, then we observe depressions at The direction of m is perpendicular to the plane
the sides. of current carrying loop. It is directed along the
2. A diamagnetic rod when placed in a uniform direction of advance of a right-handed screw rotated
magnetic field, the rod aligns itself in a direction along the direction of flow of current ½
perpendicular to the direction of the magnetic field. derivation of expression for m of electron revolving
3. The permeability of a diamagnetic substance is less around a nucleus 2
than one. 1
(b) for the loop,m = N (p r2) i (± k) ½
Properties of paramagnetic substances:
Magnetic potential energy = m.B ½
1. Removing the magnetizing field, the paramagnetics
lose their magnetisation. = N (p r2) i (± k).(Bx i + By j + Bz k) ½
2. It develops weak magnetisation along the direction = ± p r2 N i Bz ½
of magnetic field. OR
3. The magnetic lines of force prefer to pass through
the material. 1
Properties of ferromagnetic substances:
(a)    1
1. When placed in a non-uniform magnetic field, it
moves from weaker to stronger parts of the magnetic
field.

180 Physics–12
D:\EG_Physics-12_(26-06-2022)\Open_Files\Ch-5\Ch-5
\ 27-Jul-2022 Ved_Goswami Proof-4 Reader’s Sign _______________________ Date __________

Consider a rectangular Amperian loop abcd. The direction of H is along the axis of the solenoid,
Along cd the field is zero as argued above. directed along the direction of advance of a right-
Along transverse sections bc and ad, the field handed screw rotated along the direction of flow of
component is zero. Thus, these two sections current
make no contribution. Let the field along ab be B. (b) (i) Not necessarily. ½
Thus, the relevant length of the Amperian loop is,
L = h. 1 Reason: material is diamagnetic. After removal of

magnetising field, no magnetisation will remain in
Let n be the number of turns per unit length, then the material and hence earth’s magnetic field will
the total number of turns is nh. The enclosed current not affect it. ½
is, Ie = I (n h), where I is the current in the solenoid.
From Ampere’s circuital law. (ii) Yes ½
BL = μ0Ie, Bh = μ0I (n h) ½ Reason: The material is ferromagnetic. It will

remain magnetised even after removal from the
B = μ0n I
solenoid and hence align with magnetic meridian.
The direction of the field is given by the right-hand
rule. ½
[CBSE Marking Scheme, 2018-19]
H = nI ½

Case Based Question


I. Many Physicists explained ferromagnetism on the
basis on domain theory. In materials like Ni, Fe,
Co, the individual atoms are associated with large
magnetic moments (M). The magnetic moments of
neighbouring atoms interact with each other and
align themselves automatically in the same direction
called domains. Each domain has millions of atoms.
Therefore, each domain have a strong magnetic
moment. In the absence of any external magnetic field
(B0 = 0), these domains are randomly distributed in Fig. Randomly generated domains
the materials and their collective moment is zero. There are two ways for alignment of domains:
B0 = 0 1. B y displacement of the boundaries of the
domains. In case the external field B0 is weak,
the domains aligned in the direction of B0 grow
in size [Shown in (ii) by region ‘A’] while other
decreases in size region, see Fig. (ii).
Weak B0
(i) Unmagnetised sample ←
When ferromagnetic materials are placed in the
magnetic field, the domains align themselves along A
the direction of the field which strongly magnetise
the material along the direction of the magnetic field.
Due to this reason, the ferromagnetic substances show
strongly attraction towards magnets. (ii) Magnetisation process by growing of domains
● The source of ferromagnetism is the spin of the
2. By rotation of domains. In case, the external

electrons.
field B 0 is strong, the domains rotate till
● Ferromagnetic materials have incomplete inner
their magnetic moments get aligned with the
shells. These shells can be completed by using
direction of external field B0. [see Fig.]
Hund’s rule.

Magnetism and Matter 181


D:\EG_Physics-12_(26-06-2022)\Open_Files\Ch-5\Ch-5
\ 27-Jul-2022 Ved_Goswami Proof-4 Reader’s Sign _______________________ Date __________

Strong B0 (a) Paramagnetic material


← (b) Ferromagnetic material
(c) Ferrimagnetic material
(d) Diamagnetic material
2. Which of the following is a strong magnet?
(a) Diamagnetic material
Magnetisation process by rotation of domains
(b) Paramagnetic material
Properties of Ferromagnetic materials: (c) Antiferromagnetic material
● When these substances are placed in a magnetic (d) Ferromagnetic material
field, the lines of force passes mostly into the 3. Magnetic lines of force prefer to pass through
material so that the magnetic induction B becomes ferromagnetic materials because
greater than the magnetising field (B0).
(a) magnetic permeability is much greater than
that of the air.
(b) magnetic permeability is much less than that
of the air.
(c) magnetic susceptibility is very small.
● Ferromagnetic materials become paramagnetic (d) Both (b) and (c).
above a temperature called ‘Curie temperature’ 4. In ferromagnetic materials susceptibility is related
(TC). to temperature as
● Permeability (m) is greater than 1, i.e. m > 1.
(a) χm ∝ (T – Tc) (b) χm ∝ (T – Tc)2
● When these materials are placed in a non-uniform
magnetic field, it moves from weaker parts to the 1
(c) χm ∝ T − Tc (d) χm ∝
stronger parts of the magnetic field. T − Tc
● Magnetic susceptibility (χm) is large and positive, 5. Which of the following exhibits spontaneous
i.e., χm > 1000. Its value decreases with the rise in magnetisation?
temperature. (a) Paramagnetic material
● When a thin rod of a ferromagnetic substance is (b) Ferrimagnetic material

suspended freely in a uniform magnetic field B ( ) (c) Diamagnetic material
, rod gets align itself parallel to the field.
1. In which of the following magnetic moments align (d) Ferromagnetic material
themselves parallel to each other? Ans. 1. (b) 2. (d) 3. (a) 4. (d) 5. (d)

IMPORTANT FORMULAE
Formula Symbols Application
1. Magnetic dipole moment of a bar magnet, qm = pole strength In study of bar magnet
m = qm2l 2l = dipole length
2. Magnetic field strength due to a bar magnet m = magnetic dipole moment To find value of B at axial and
(i) At an axial point equator point
 µ 
2mr
B= o 2
4π (r − l 2 )2

182 Physics–12
D:\EG_Physics-12_(26-06-2022)\Open_Files\Ch-5\Ch-5
\ 27-Jul-2022 Ved_Goswami Proof-4 Reader’s Sign _______________________ Date __________

µ o 2m
For r >>> l, B =
4π r 3
(ii) On equator,
µo m
B= 3

(r 2 + l 2 ) 2

µo m
For r >>> l, B =
4π r 3


3. τ = m × B t = Torque To find work done in rotating the
magnet

4. µ B µr = relative permeability To study magnetic properties like


µr = = susceptibility
µ o Bo

5. Intensity of magnetisation (M) m = mag. dipole moment To study the strength of magnetic
m q V =Volume of the specimen field
= = m
M
V A

6. M χm = mag. susceptibility To classify the magnetic material


χm =
H

7. µr = 1 + cm — To determine characteristic of the


magnetic material

COMMON ERRORS
S No. Errors Corrections
1. Forget to convert Take the units in SI system always
2. Dependence of susceptibility and Should draw in proper quadrant – for dia in fourth quadrant and
temperature for para and ferro in first quadrant.

Magnetism and Matter 183


D:\EG_Physics-12_(26-06-2022)\Open_Files\Ch-5\Ch-5
\ 27-Jul-2022 Ved_Goswami Proof-4 Reader’s Sign _______________________ Date __________

REVISION CHART

Magnetic field strength on equator of a bar magnet:


µ m Magnetic field of a bar magnet at an axial point:
B= o 3
, µ o 2 mr
4π 2 2 2
B=
(r + l ) 4π ( r 2 − l 2 ) 2
µ m µ 2m
for r >>> l , B = o 3 for r >>> l , B = o 3
4πr 4πr

Bar Magnet

Magnetic dipole placed in a uniform magnetic field:


(i) Force acting = 0, A current carrying solenoid is equivalent to a
   bar magnet.
(ii) Torque = τ = m × B
m is magnetic dipole moment.

Diamagnetic substances Paramagnetic substances


Diamagnetic substances are those Ferromagnetic substances
Paramagnetic substances are those which Ferromagnetic substances are those
substances which get feebly magnetized get weakly magnetised in the direction which get strongly magnetised when
in the opposite direction of the of the magnetising field, when placed in placed in an external magnetic field in
magnetizing field. Such substances are an external magnetic field. They have the direction of the applied field. They
feebly repelled by magnets and tend to tendency to move slowly from a region have tendency to move quickly from a
move slowly from stronger to weaker of weak magnetic field to strong magnetic region of weak magnetic field to strong
part of the magnetic field. field. magnetic field.

Magnetic Properties of Materials

Retentivity Magnetic hysteresis Curie temperature


The value of the intensity of magnetisation The lagging behind of magnetisation The temperature of a ferromagnetic
left in the magnetic material, when the behind the magnetising field when a substance above which, it behaves
magnetising field is reduced to zero is specimen is taken through a cycle of as a paramagnetic substance is called
called its retentivity. magnetisation and demagnetisation is curie temperature.
called hysteresis.

Coercivity Curie’s law


The value of reverse magnetising field required to reduce the Experimentally, one finds that the magnetisation of a
residual magnetism to zero is called coercivity. paramagnetic material is inversely proportional to the absolute
temperature T.

184 Physics–12
D:\EG_Physics-12_(26-06-2022)\Open_Files\Ch-5\Ch-5
\ 27-Jul-2022 Ved_Goswami Proof-4 Reader’s Sign _______________________ Date __________

IMPORTANCE OF EACH TOPIC AND FREQUENTLY ASKED TYPES OF QUESTIONS

☞ Important Topics
1. Questions related to Classification of Magnetic Material.
2. Questions based on Properties of diamagnetic, paramagnetic, ferromagnetic.

1. Which of the following substances are diamagnetic?


Bi, Al, Na, Cu, Ca and Ni
2. Will the neon gas be diamagnetic or paramagnetic? Give reason.
3. State and explain Curie law of magnetism.
4. Permanent magents are made of special alloys while the cores of temporary magnets are made of soft iron. Why?
5. Out of the following, identify the materials which can be classfied as: (i) paramagnetic, (ii) diamagnetic:
(a) Aluminium (b) Bismuth (c) Copper (d) Sodium
6. A uniform magnetic field gets modified as shown below when two specimens X and Y are placed in it. Identify
whether specimens X and Y are diamagnetic, paramagnetic or ferromagnetic.

7. A ship is to reach a place 15° south of west. In what direction should it be steered if declination at the place is 18°
west?[Ans. 87° West of North]
8. What is the magnitude of the equatorial and axial fields due to a bar magnet of length 4 cm at a distance of 40 cm
from its mid point? The magnetic moment of the bar magnet is 0.5 Am2.
[Ans. BE = 7.8125 × 10–7 T; BA = 15.625 × 10–7 T]

Magnetism and Matter 185


D:\EG_Physics-12_(26-06-2022)\Open_Files\Ch-5\Ch-5
\ 27-Jul-2022 Ved_Goswami Proof-4 Reader’s Sign _______________________ Date __________

ASSIGNMENT
I. Objective Type Questions (1 Mark)
1. Multiple choice questions:
(i) Magnetic field due to a bar magnet 2 cm long having a pole strength of 100 Am at a point 10 cm from each
pole is
(a) 2 × 10–4 T (b) 8p × 10–4 T (c) 2 × 10–5 T (d) 4 × 10–4 T
(ii) At a certain place, the horizontal component of the earth’s magnetic field is B0 and the angle of dip is 45°. The
total intensity of the field at that place will be
(a) B0 (b) 2 B0 (c) 2B0 (d) B02
2. Fill in the blanks:
(i) According to the modern theory of magnetism, every atom of a substance behaves just like a ................
(ii) Straight line passing through magnetic north and south pole of the earth is called .................
II. Very Short Answer Type Questions (1 Mark)
3. What is the SI unit of (i) pole strength and (ii) magnetic moment of a bar magnet?
4. Relative permeability mr of a material has value lying 1 < mr < 1 + ∈ (where ∈ is a small quantity). Identify the
nature of magnetic material.
III. Short Answer Type Questions-I (2 Marks)
5. The following figure shows the variation of intensity of magnetisation versus the applied magnetic field intensity
H, for two materials A and B
M

H
(i) Identify the materials A and B.
(ii) For the material A, plot variation of intensity of magnetisation versus temperature.
6. What is the magnitude of the equatorial and axial fields due to a bar magnet of length 5.0 cm at a distance of 50
cm from its mid-point? The magnetic moment of the bar magnet is 0.40 Am2.
IV. Short Answer Type Questions-II (3 Marks)
7. Distinguish between diamagnetic and ferromagnetic materials in respect of their (i) intensity of magnetization (ii)
behaviour in non uniform magnetic field and (iii) susceptibility.
8. A magnetic needle is placed in a uniform magnetic field with its axis tilted w.r.t. its position of stable equilibrium.
Deduce an expression for the time period of (small amplitude) oscillation of this magnetic dipole about an axis,
passing through its centre and perpendicular to its plane.
9. Mention two properties of soft iron due to which it is preferred for making an electromagnet.
 

186 Physics–12
D:\EG_Physics-12_(26-06-2022)\Open_Files\Ch-6\Ch-6
\ 27-Jul-2022 Ved_Goswami Proof-4 Reader’s Sign _______________________ Date __________

Topics Covered
6 Electromagnetic Induction

6.1 Faraday’s Law of Electromagnetic Induction and Lenz’s Law


6.2 Self Induction and Mutual Induction 6.3 A.C. Generator and Transformer

C hapter map
Electromagnetic
Induction (EMI)

EMI and Laws Self Induction and Devices based


governing it Mutual Induction on EMI

A.C. Generator
Magnetic flux Self and Mutual
Induction Transformer
Faraday’s &
Lenz’s Law Principle of
Co-efficient of
Transformer
Self and Mutual
Eddy current Induction Losses in a
transformer

Various Methods Solenoid Ways to minimise


of inducing emf losses in a
transformer

Topic 1. Faraday’s Law of Electromagnetic Induction and Lenz’s Law


•• Magnetic Flux: The number of magnetic field lines •• Electromagnetic Induction: The phenomenon of
passing through a given surface normally is called inducing an emf due to the change in magnetic flux liked
magnetic flux. with a coil is called electromagnetic induction.
^
dS B •• Faraday’s Laws of Electromagnetic Induction: 1st law:
Whenever the amount of magnetic flux linked with a coil
changes, an emf is induced in the coil. The induced emf
θ lasts as long as the change in magnetic flux continues.
2nd law: The magnitude of the emf induced is directly
proportional to the rate of change of magnetic flux linked
with a coil.
If B is magnetic field strength, dS is area then magnetic d φB
|e| ∝ where fB = magnetic flux
flux dt
 
df = B ⋅ dS = BdS cos θ kd φ d φ
|e| = = , where k = 1
It is a scalar quantity. dt dt
SI unit of magnetic flux is weber (Wb) •• Lenz’s Law: Direction of induced current is such that it
1 Wb = 1 T m2 will oppose the cause which has produced it.

187
D:\EG_Physics-12_(26-06-2022)\Open_Files\Ch-6\Ch-6
\ 27-Jul-2022 Ved_Goswami Proof-4 Reader’s Sign _______________________ Date __________

− dφ (v) In electric energy meter/electric power meters:


e= The metal disc in electric energy meter rotates due
dt
•• Eddy Current/Foucault Current: They are the currents to eddy currents and these eddy currents are induced
induced in the body of metallic mass when the magnetic by the magnetic field produced by the A.C. flowing
flux threading through them changes. in the coil.
•• Methods of Inducing E.M.F.
d φ / dt
I = εin
= (i) By changing strength of the magnetic field.
R R
•• Disadvantages of Eddy Currents (ii) By changing area of the coil placed is a magnetic
(i) Eddy currents cause unnecessary heating and the heat field.
produced may damage the insulation of coils. (iii) By changing the orientation of the coil w.r.t. the
(ii) The excessive heating may lead to fire causing magnetic field.
financial, economic loss. •• Motional EMF: When the area of the coil through which
(iii) Eddy currents can reduce the efficiency of machines magnetic field passing is changed by moving the coil
as it will try to destroy the relative motion. into or out of the coil, an emf is induced. For example,
when the conductor PQ is moved over a U-shaped loop
•• How to Minimise Eddy Currents
as shown in the figure.
(i) Reduced to some extent by cutting narrow slits in the
× × × × × × × ×
body of the metal. S P
(ii) More effective: By using laminated core instead of a × × × × × × ×
single solid mass. The plane of the sheets are placed R
v
Q
perpendicular to direction of current that would be × × × × × × × ×
set up by the induced emf. Or plane of sheet should Let PQ = l
be parallel to magnetic field.
Induced emf = Blv
•• Applications of Eddy current: Eddy currents are used
If R is the resistance of the loop, then
in:
E Blv
(i) Induction furnace for melting metals: Metal Induced current I = =
R R
specimen is placed in a rapidly varying magnetic
field. Large eddy currents are set up. The large amount The direction of current will be clockwise, it can be
of heat produced will melt metals. determined by Lenz’s law.
(ii) Electric brakes in trains: On applying brakes a As the PQ moves towards left, area of the loop hence
strong magnetic field is set up, in which the drum the magnetic flux through the loop will decrease. The
attached to the wheel rotates. The eddy currents are induced current would tend to increase the flux by
set up in the drum and exert a torque on the drum so inducing magnetic field in the direction of applied field.
as to stop the train. So current will be in clockwise direction.
(iii) Electromagnetic damping: It is used in dead beat This current carrying conductor PQ is placed in the
galvanometer. When the coil of the galvanometer magnetic field. So it will experience a force.
rotates due to the current flowing through it, eddy B2 l 2 v
currents are set up in the metallic core. According to F = BIl sin 90° =
R
Lenz’s law, eddy currents will oppose the cause i.e., To move this conductor, the power supplied by the
the rotation of the coil. Thus, the coil will come to external force = Fv
rest immediately.
B2 l 2 v 2
(iv) In speedometer to record the instantaneous speed P = Fv =
of the vehicle: A magnet which is placed inside an R
aluminium drum rotates with the speed of the vehicle. Blv B2 l 2 v 2
So the eddy currents are induced is the drum which Power dissipated as heatP = EI = Blv × =
R R
oppose the motion of the magnet. Torque acting on the
So we see that the electrical energy is produced at the
drum in opposite direction deflects the drum through
cost of mechanical energy.
an angle depending upon the speed of the vehicle.

188 Physics–12
D:\EG_Physics-12_(26-06-2022)\Open_Files\Ch-6\Ch-6
\ 27-Jul-2022 Ved_Goswami Proof-4 Reader’s Sign _______________________ Date __________

EXERCISE 6.1
I. Objective Type Questions (1 Mark) 3. State True or False
1. Choose the correct answers from the given options (i) A coil having area A, total number of turns
 N is
(MCQs). rotated in a uniform magnetic field B with an
(i) A circular coil is placed near a straight conductor angular velocity w. The maximum emf induced in
as shown below. When the current in the straight the coil is equal to NBAw.
conductor increases, the current in the coil is (ii) The induced emf produced in a wire of length
.................... . l moving in a magnetic field B with a constant
velocity v is given by Bvl and the induced current in
Bvl
a loop of this wire having resistance R is .
Coil R
For questions numbers 4 two statements are given-one
I labelled Assertion (A) and the other labelled Reason
Increasing (R). Select the correct answer to these questions from
the codes (a), (b), (c) and (d) as given below.
(a) clockwise (b) anticlockwise
(c) normal to the plane of coil (a) Both A and R are true and R is the correct
(d) None of them explanation of A
(ii) Which of the following gives the direction of the (b) Both A and R are true but R is NOT the correct
induced e.m.f.? explanation of A
(a) Faraday’s law (b) Lenz’s law (c) A is true but R is false
(c) Ampere (d) Biot-Savart’s law
(d) A is false and R is also false
(iii) Which of the following based on the law of
conservation of energy? 4. Assertion (A): Only a change in magnetic flux (Df)
(a) Faraday’s law (b) Lenz’s law will maintain an induced current in the copper coil.
(c) Ampere (d) Biot-Savart’s law Reason (R): The presence of large magnetic flux
(iv) A coil of area 100 cm2 is kept at an angle of 30° through a copper coil maintains a current in the coil
with a magnetic field of 10–1 T. The magnetic field if the circuit is continuous.
is reduced to zero in 10–4 s. The induced emf in II. Very Short Answer Type Questions (1 Mark)
the coil is [CBSE 2022] 1. Write S.I. unit of magnetic flux. Is it a scalar or a
(a) 5 3 V (b) 50 3 V vector quantity?
(c) 5.0 V (d) 50.0 V 2. On what factors does the magnitude of the emf
2. Fill in the blanks. induced in the circuit due to magnetic flux depend?
(i) Two circular conductor A and B are placed  [Foreign 2013]
perpendicular to each other as shown in the 3. Predict the direction of induced current in metal
figure. If the current in one of the coils is changed, rings 1 and 2 when current I in the wire is steadily
then current induced in the other coil is equal to decreasing? [Delhi 2012]
....................... .
B 1
A
2

4. The spherical bobs, one metallic and the other of glass,


(ii) When the rate of change of current through a of the same size are allowed to fall freely from the same
closed circuit is unity, then the induced emf height above the ground, which of the two would reach
produced in it is equal to ................. . the ground earlier and why?
(iii) A copper rod of length l is rotated about one end 5. If the resistance R in circuit ‘a’ decreased, what will
perpendicular to the uniform field B with constant be the direction of induced current in circuit ‘b’?
angular velocity w. The induced emf between its  [Delhi 2011]
two end is ...................
Electromagnetic Induction 189
D:\EG_Physics-12_(26-06-2022)\Open_Files\Ch-6\Ch-6
\ 27-Jul-2022 Ved_Goswami Proof-4 Reader’s Sign _______________________ Date __________

13. Two bar magnets are quickly moved towards a


metallic loop connected across a capacitor ‘C’
as shown in the figure. Predict the polarity of the
R b
capacitor. [AI 2011]
a
A
C
S N B S N

6. A bar magnet falls through a metal ring. Will its 14. Figure shows a current carrying solenoid moving
acceleration be equal to g? [Delhi 2008] towards a conducting loop. Find the direction of the
7. Predict the direction of induced currents in metal current induced in the loop. [Delhi 2015]
rings 1 and 2 lying in the same plane where current
I in the wire is increasing steadily.
1 i

2 I 15. When a bar magnet is pushed towards (or away) from


the coil connected to a galvanometer, the pointer in
8. Predict the direction of induced current in a metal ring
the galvanometer deflects. Identify the phenomenon
when the ring is moved toward a straight conductor
causing this deflection and write the factors on which
with constant speed v. The conductor is carrying
the amount and direction of the deflection depends?
current I in the direction shown in the figure.
 [Delhi 2012] 16. A metallic rod held horizontally along east-west
direction, is allowed to fall under gravity. Will there
v
be an emf induced at its ends? Justify your answer.
I [Delhi 2013]
9. The motion of copper plate is damped when it is 17. The closed loop PQRS is moving into a uniform
allowed to oscillate between the two poles of a magnetic field acting at right angles to the plane of
magnet. What is the cause of this damping? the paper as shown. State the direction of the induced
10. When current in a coil changes with time, how is the current in the loop. [Delhi 2012]
back emf induced in the coil related to it? × × × × × × × × × × ×
P Q
 [AI 2008]
× × × × × × × × × × ×
11. A bar magnet is moved towards a solenoid as shown
in the figure. What is the direction of current in R? × × × × × × × × × × ×
 [Delhi 2013] v
R × × × × × × × × × × ×
X Y

× × × × × × × × × × ×S R
18. Two loops of different shapes are moved in the region
S N
of a uniform magnetic field pointing downward. The
12. A flexible wire of irregular shape, abcd, as shown in
loops are moved in the directions shows by arrows.
the figure, turns into a circular shape when placed in
What is the direction of induced current in each loop?
a region of magnetic field which is directed normal
 [Foreign 2010]
to the plane of the loop away from the reader. Predict × × × × × × × × × × × × × ×
the direction of the induced current in the wire. g
[Delhi 2014] × × × × × × × × × × × × × ×
c
× × × × × × × × × × × h f
a × × × × × × × × × × × × × ×
× × × × × × × ×c × × ×
× × × × × × × × × × × × × ×
× × × × × × × × × × × b e
b d
a × × × × × × × × × × × × × ×
× × × × × × × × × × ×
19. Twelve wires of equal length are connected in the
× × × × × × × × × × × form of a skeleton cube which is moving with velocity

190 Physics–12
D:\EG_Physics-12_(26-06-2022)\Open_Files\Ch-6\Ch-6
\ 27-Jul-2022 Ved_Goswami Proof-4 Reader’s Sign _______________________ Date __________

  
v in the direction of magnetic field B. Find the emf uniform magnetic field B , as shown in the figure.
in each arm of the cube. [Delhi 2016] What is the potential difference developed between
D C the two ends of the rod, P and Q? [AI 2020]
× × × × × × × × × × × ×

A B × × × × × × ×B× × × × ×
v × × × × × × O× × × × × ×
× × P× × × × × × × Q× × ×
H
G × × × ×w× × × × × × × ×
× × × × × × × × × × × ×
E F B × × × × × × × × × × × ×
20. A coil is removed from a magnetic field (i) rapidly 27. Draw a graph showing variation of the value of the
(ii) slowly. In which case more work will be done? induced emf as a function of rate of change of current
21. Consider a magnet surrounded by a wire with an on/ flowing through an ideal inductor. [AI 2020]
off switch S′ (as shown in figure). If the switch is III. Short Answer Type Questions-I (2 Marks)
thrown from the off position (open circuit) to the on 28. A coil Q is connected to low voltage bulb B and placed
position (closed circuit), will a current flow in the near another coil P as shown in the figure. Give reason
circuit? Explain. [NCERT Exemplar] to explain the following observations.
S N S N
B
~
S′
Circuit closed Circuit open
Q P
22. The closed loop (PQRS) of wire is moved out of a
uniform magnetic field at right angles to the plane of (a) The bulb B lights up.
the paper as shown in the figure. Predict the direction (b) Bulb B gets dimmer if coil Q is moved towards
of the induced current in the loop. [Foreign 2012] left. [Delhi 2016]
× × × × × × × × × × × 29. Name the SI unit of magnetic flux and show that
P Q
it equals volt-sec. Give three possible ways of
× × × × × × × × × × ×
producing an induced emf in a coil giving an example
× × × × × × × × × × × in each case. [S.P. 2013]
30. A rectangular wire frame, as shown in the figure is
× × × × × × × × × × × placed in uniform magnetic field directed upward
S R and normal to the plane of the paper. The part AB is
× × × × × × × × × × × connected to a spring. The spring is stretched and then
23. A long straight current carrying wire passes normally released when the wire AB has come to the position A′B′
through the centre of circular loop. If the current (t = 0). How would the induced emf vary with time?
through the wire increases, will there be an induced Neglect damping. [S.P. 2008]
emf in the loop? Justify. [Delhi 2017]
A
24. Plot a graph showing variation of induced e.m.f. with A′

the rate of change of current flowing through a coil.


 [Delhi 2020]
25. A series combination of an inductor (L), capacitor (C)
and a resistor (R) is connected across an ac source of
emf of peak value E0 and angular frequency (w). Plot
a graph to show variation of impedance of the circuit
with angular frequency (w). [Delhi 2020] B′ B
26. A metallic rod PQ of length l is rotated with an angular 31. Consider a metallic pipe with an inner radius of 1 cm.
velocity w about an axis passing through its mid-point If a cylindrical bar magnet of radius 0.8 cm is dropped
(O) and perpendicular to the plane of the paper, in through the pipe, it takes more time to come down

Electromagnetic Induction 191


D:\EG_Physics-12_(26-06-2022)\Open_Files\Ch-6\Ch-6
\ 27-Jul-2022 Ved_Goswami Proof-4 Reader’s Sign _______________________ Date __________

than it takes for a similar unmagnetised cylindrical C1


iron bar dropped through the metallic pipe. Explain. C2
[NCERT Exemplar]
32. Like electrostatic shielding, is electromagnetic
N S
shielding also possible?
33. A wire in the form of a tightly wound solenoid is
connected to a DC source, and carries a current. If
the coil is stretched so that there are gaps between A A

successive elements of the spiral coil, will the current 41. A small piece of metal wire is dragged across the
increase or decrease? Explain. [NCERT Exemplar] gap between the pole piece of a magnet in 0.5 s. The
34. A solenoid is connected to a battery so that a steady magnetic flux between the pole pieces is known to be
current flows through it. If an iron core is inserted into 8 × 10–4 Wb. Estimate the induced emf in the wire.
the solenoid, will the current increase or decrease?  [Delhi 2004]
Explain. [NCERT Exemplar]
42. A jet plane is travelling westward at a speed of 1800
35. Consider a metal ring kept on top of a fixed solenoid km/h. What is the potential difference developed
(say on a cardboard) (in figure). The centre of the ring between the ends of a wing 25m long, its earth’s
coincides with the axis of the solenoid. If the current magnetic field at the location has a magnitude of 5.0
is suddenly switched on, the metal ring jumps up. × 10–4 T and the dip angle is 30°. [AI 2009]
Explain. [NCERT Exemplar]
43. A circular copper disc 10 cm in radius rotates at
20p rad/s about an axis through its centre and
Ring perpendicular to the disc. A uniform magnetic field
of 0.2 T acts perpendicular to the disc:
(i) Calculate the potential difference developed
between the axis of the disc and the rim.
(ii) What is the induced current in the circuit whose
terminals are connected between centre of disc
36. (a) State Lenz’s law. and point of rim and the resistance of the circuit
(b) A metallic rod held horizontally along east-west is 2W.
direction, is allowed to fall under gravity. Will 44. A 0.5 m long metal rod PQ completes the circuit
there be an emf induced at its ends? Justify your as shown in the figure. The area of the circuit is
answer. [Delhi 2013] perpendicular to the magnetic field of flux density
0.15 T. If the resistance of the total circuit is 3W
37. Two identical loops, one of copper and the other of
calculate the force needed to move the rod in the
aluminium, are rotated with the same angular speed
direction as indicated with a constant speed of 2 ms–1.
in the same magnetic field. Compare (i) the induced
emf and (ii) the current produced in the two coils. × × × × × × × × × × ×
Justify your answer. [AI 2010] Q
× × × × × × × × × × ×
38. Show that Lenz’s law is in accordance with the law
of conservation of energy. × × × × × × × × × × ×
39. Twelve wires of equal length ‘l’ are connected
to form a skeleton cube which moves with a × × × × × × × × × × ×
(i) velocity v perpendicular to the magnetic field B.
× × × × × × × × × P× ×
What will be the induced emf in each arm of the cube?
(ii) If v || B, what is the induced emf? 45. A rectangular conductor LMNO is placed in a
40. A magnet is quickly moved in the direction indicated uniform magnetic field of 0.5 T. The field is directed
by an arrow between two coils C1 and C2 as shown perpendicular to the plane of the conductor. When the
in figure. What will be direction of induced current arm MN of length of 20 cm is moved towards left
in each coil as seen from the magnet? Justify your with a velocity of 10 ms–1, calculate the emf induced
answer. [Delhi 2011] in the arm. Given the resistance of the arm to be 5W

192 Physics–12
D:\EG_Physics-12_(26-06-2022)\Open_Files\Ch-6\Ch-6
\ 27-Jul-2022 Ved_Goswami Proof-4 Reader’s Sign _______________________ Date __________

(assuming that other arms are of negligible resistance) the arrangement is poisoned between the poles of a
find the value of the current in the arm. [AI 2013] permanent magnet producing uniform magnetic field
× × × × × × × × × × ×
B
B = 0.4 T. The rails, the rod and the magnetic field are
L
M in three mutually perpendicular directions as shown
× × × × × × × × × × × in the figure. If the ends A and C of the rails are short
circuited, find the
× × × × × × × × × × ×
v × × × ×
× × × × × × × × × × × P
A A´
O N
× × × × × × × × × × ×
× × × ×
46. A jet plane is travelling west at 450 ms–1. If the ⊗ v
B
horizontal component of earth’s magnetic field at that × × × ×
place is 4 × 10–4 T and the angle of dip is 30°, find C C´
the emf induced between the ends of wings having a Q
span of 30m. [AI 2008] × × × ×
47. A square loop MNOP of side 20 cm is placed (i) external force required to move the rod with
horizontally in a uniform magnetic field acting uniform velocity = 10 cm/s and
vertically downwards as shown in the figure. The (ii) power required to do so. [Delhi 2020]
loop is pulled with a constant velocity of 20 cm s–1 50. A conductor of length l is rotated about one of its ends
till it goes out of the field. [AI 2015] at a constant angular speed w in a plane perpendicular
× × × × × × × × × × ×
M 20 cm to a uniform magnetic field B. Plot graphs to show
N
× × × × × × × × × × × variations of the emf induced across the ends of the
v conductor with (i) angular speed w and (ii) length of
× × × × × × × × × × × the conductor l. [AI 2020]
P O 51. A coil of wire enclosing an area 100 cm2 is placed
× × × × × × × × × × × with its plane making an angle 60° with the magnetic
1m
field of strength 10–1 T. What is the flux through the
(i) Depict the direction of the induced current in the
coil? If magnetic field is reduced to zero in 10–3 , then
loop as it goes out of the field. For how long would
find the induced emf? [CBSE S.P. 2020-21]
the current in the loop persist?
(ii) Plot a graph showing the variation of magnetic IV. Short Answer Type Questions-II (3 Marks)
flux and induced emf as a function of time. 52. Describe, with the help of a suitable diagram, how
48. Two loops, one rectangular of dimensions 10 cm × 2.5 one can demonstrate that emf can be induced in a
cm and second of square shape of side 5 cm are moved coil due to the change of magnetic flux. Hence state

out of a uniform magnetic field B perpendicular to Faraday’s law of electromagnetic induction.
the planes of the loops with equal velocity n as is  [AI 2015]
shown in the figure. 53. A bar magnet M is dropped so that it falls vertically
× × × × × × × × × × × ×
through the coil C. The graph obtained for voltage
10 cm
× × × × × × × × × × × ×
produced across the coil versus time is showing in
v 2.5 cm figure.
× × × × × × × × × × × × (i) Explain the shape of the graph.
(ii) Why is the negative peak longer than the positive
5 cm
× × × × × × × × × × × × peak?
v 5 cm
M
× × × × × × × × × × × × P–D(mV)
(i) In which case will the emf induced be more? Magnet

(ii) In which case will the current flowing through the


two loops be less? R C
Time (ms)
Justify your answer. V
49. A conducting rod PQ of length 20 cm and resistance
0.1 W rests on two smooth parallel rails of negligible
resistance AA´ and CC´. It can slide on the rails and
Electromagnetic Induction 193
D:\EG_Physics-12_(26-06-2022)\Open_Files\Ch-6\Ch-6
\ 27-Jul-2022 Ved_Goswami Proof-4 Reader’s Sign _______________________ Date __________

54. A circular coil of N-turns and radius R, is kept normal × × × × × × × ×


to a magnetic field, given by B = B0 cos wt. Deduce S P
an expression for the emf induced in this coil. State × × × × × × × ×
the rule which helps to detect the direction of induced v
current. R
× × × × × × × Q
×
55. Derive an expression for (i) induced emf and
(ii) induced current when a conductor of length l is 59. A current is induced in coil C1 due to the motion
moved with a uniform velocity v, normal to a uniform of current carrying coil C2. (a) Write any two ways
magnetic field B. Assume the resistance of conductor by which a large deflection can be obtained in the
to be R. galvanometer G. (b) Suggest an alternative device
to demonstrate the induced current in place of a
56. The arm PQ of the rectangular conductor is moved galvanometer. [Delhi 2011]
from x = 0, outwards. The uniform magnetic field is
perpendicular to the plane and extends from x = 0 to C1 C2
x = b and is zero for x > b. Only the arm PQ possesses
substantial resistance r. Consider the situation when
the arm PQ is pulled outwards from x = 0 to x = 2b
and it then moved back to x = 0 with constant speed
v. Obtain expressions for the flux, the induced emf,
the force necessary to pull the arm and the power
dissipated as Joule heat. Sketch the variation of these
quantities with distance. G

[AI 2010, S.P. 2019-20][NCERT] 60. (a) Describe a simple experiment (or activity) to
show that the polarity of emf induced in a coil
× × × × × × × × × × ×
K L M
is always such that it tends to produce a current
× × × × × × × × × × × which opposes the change of magnetic flux that
S
produces it.
× × × × × × × ×P × × ×
61. A wheel with 8 metallic spokes each 50 cm long is
× × × × × × × × × × × rotated with a speed of 120 rev/min in a plane normal
l v
× × × × × × × × × × ×
to the horizontal component of the Earth’s magnetic
field. The earth’s magnetic field at the plane is 0.4 G
Q
× × × × × × × × × × × and the angle of dip is 60°. Calculate the emf induced
R between the axle and the rim of the wheel. How will
× × × × × × × × × × ×
the value of emf be affected if the number of spokes
× × × × × × × × × × ×
x = 2b were increased? [Delhi 2013]
x= 0 x= b
62. The magnetic field through a single loop of wire,
57. What are eddy currents? How are they produced? In 12 cm in radius and 8.5W resistance, changes with
what sense eddy currents are considered undesirable time as shown in the figure. The magnetic field is
in a transformer? How can they be minimised? Give perpendicular to the plane of the loop. Plot induced
two applications of eddy currents. [AI 2011] current as a function of time. [S.P. 2015-16]
58. Figure shows a rectangular loop conducting PQRS in
which the arm PQ is free to move. A uniform magnetic
2.0
field acts in the direction perpendicular to the plane of
the loop. Arm PQ is moved with a velocity v towards
the arm RS. Assuming that the arm QR, RS and SP
B(T)

1.0
move negligible resistances and the moving arm PQ
has the resistance r, obtain the expression ac (i) the
current in the loop (ii) the force and (iii) the power
required to move in the arm. [Delhi 2003] 0 2.0 4.0 6.0
t (s)

194 Physics–12
D:\EG_Physics-12_(26-06-2022)\Open_Files\Ch-6\Ch-6
\ 27-Jul-2022 Ved_Goswami Proof-4 Reader’s Sign _______________________ Date __________

63. When a conducting loop of resistance of 10 W and × × × × × × ×


area 10 cm2 is removed from an external magnetic × × × × × × ×
× × × × × × ×
field acting normally, the variation of induced current × × × × × × v×
in the loop with time is shown in the figure. × × × × × × ×
(a) Sketch the variation of magnetic flux, the induced
current and power dissipated as Joule heat as
(I in A) function of time.
0.4 A (b) If instead of rectangular loop, circular loop is
pulled out; do you expect the same value of
induced current? Justify your answer. Sketch the
variation of flux in this case with time.
[CBSE S.P. 2020-21]
(0, 0) 1.0 (t in s)
V. Long Answer Type Questions (5 Marks)
Find the
65. What is induced emf? Write Faraday’s law of
(i) total charge passed through the loop. electromagnetic induction. Express it mathematically.
(ii) change in magnetic flux through the loop. A conducting rod of length ‘l’ with one end pivoted,
(iii) magnitude of the magnetic field applied. is rotated with a uniform angular speed w in a vertical
[Delhi 2020] plane, normal to a uniform magnetic field ‘B’.
64. A rectangular loop which was initially inside the (i) Deduce an expression in the emf induced in this
region of uniform and time - independent magnetic rod. [Delhi 2013, 2012]
field, is pulled out with constant velocity v as shown (ii) If resistance of rod is R, what is the current induced
in the figure. in R. [Delhi 2008]

Answers 6.1
I. Objective Type Questions
Clockwise
1. (i) (a) (ii) (b) (iii) (b) (iv) (a) 7.
2. (i) Zero (ii) self inductance L Anticlockwise
1
(iii) Bwl2 8. Clockwise.
2 9. As the plate oscillates,the changing magnetic flux
3. (i) True (ii) True 4. (c) through the plate induces a strong eddy current in the
II. Very Short Answer Type Questions direction, which opposes the cause.
1. S.I. unit of magnetic flux is weber. It is a scalar quantity. Also, copper being diamagnetic substance, it gets
2. The magnitude of induced emf in the circuit depends magnetized in the opposite direction,so the plate motion
dφ gets damped.
on the time rate of change of magnetic flux E =
dt 10. The back emf induced in the coil oppose the change in
3. Loop 1- clockwise. current.
Loop 2-anticlockwise 11. The direction of current will be from Y to X.
4. Glass bob will reach the ground earlier. In case of Reason: As S pole is moved towards the coil, the RHS
metallic bob eddy current will be produced due to it’s end of the coil will behave as S pole (Lenz’s law). So
motion in earth’s magnetic field these eddy current will current as seen by an observe standing on RHS, the
oppose the cause (i.e., the fall) which has produced direction of current will be clockwise.
it, glass bob being bad conductor will not have eddy 12. As the loop changes from irregular to circular shape,
current. the area and hence the flux linked with the loop will
5. As R decreases, the current in circuit ‘a’ will increase. increase. According to Lenz’s law the induced current
The current induced in ‘b’ will oppose it. The induced will decrease the flux. For that the current should in
current will be in anticlockwise direction. anticlockwise direction.
6. No, its acceleration a < g. 13. According to Lenz‘s law movement of both the magnet
Because as the magnet falls, induced current will be when induce a current in clockwise direction when seen
produced is the coil, which will oppose the fall of the from R.H.S. So the top plate i.e., Plate A of the capacitor
magnet so a < g. becomes +ve and end B becomes negative.

Electromagnetic Induction 195


D:\EG_Physics-12_(26-06-2022)\Open_Files\Ch-6\Ch-6
\ 27-Jul-2022 Ved_Goswami Proof-4 Reader’s Sign _______________________ Date __________

14. Clockwise from the point of observation. As the R.H.S. 23. No emf will be induced in the loop magnetic field is
end of the solenoid will behave as south pole. When the parallel to the plane of the loop and the magnetic flux
solenoid is moved towards the loop the L.H.S. face of threading the loop is zero.
the loop will act as S-pole and R.H.S. face as N-pole.
Magnetic
So the current when seen from the point of observation lines
marked, current will be in clockwise direction.
15. Phenomenon is electromagnetic induction. The amount
loop
of deflection depends upon the speed with which the
bar magnet is pushed towards or away from the coil
direction of deflection will depend upon. 24. According to Faraday’s 2nd law
(i) Which pole of the magnet is moved? dφ
e∝ –
(ii) The direction (towards or away from the coil) in dt
which the magnet is moved. dφ
e= –k
16. Yes, there will be induced emf, as it will intercept the dt
horizontal component of earth’s magnetic field. d
e= –k ( LI )
17. Due to the motion of coil, the magnetic flux linked dt
with the coil increases. So by Lenz’s law, the current dI
induced in the coil will oppose this increase, hence tend e= –kL
to produce a magnetic field upward, so current induced dt
in the coil will flow anticlockwise. dI

⇒ e∝ –
18. As Loop, abc is entering the magnetic field, so magnetic dt
flux linked with it begins to increase. According emf
to Lenz’s law, the current induced will oppose the
increase in magnetic flux, so current induced will be ↑
e
anticlockwise which tends to decrease the magnetic dI
field, passing through the loop. O dt
As loop efgh is leaving the magnetic field, so flux
linked with it will decrease, the induced current will
be clockwise to produce magnetic field downward to 25. The graph showing the variation of impedance (Z) of
oppose the decrease in magnetic flux. the circuit with angular frequency (w) is as follows:
  Z
19. As v || B, so the electrons do not experience any force
 
as q ( v × B) = 0, So no emf will be induced in any arm.
Impedance

20. Work done will be more when the coil is removed R


rapidly, as in this case the rate of change of magnetic w
w0
flux linked with the coil will be more and so more
w0 shows the resonance frequency for the LCR circuit.
current will be induced.
–l/2 O dx +l/2
21. As there is no relative motion between the magnet and 26.
the loop. So there is no change in magnetic flux with x
time. So no induced emf and hence no induced current de = B·dx × wx = Bx × dx
will be produced. l l/2
2  x2 
22. The direction of induced current will be clockwise. e = Bω ∫ xdx = Bω  
 2  −l / 2
Reason: As the loop is moving out of the magnetic field, −l
2
the magnetic flux linked with the loop will decrease. l2 l2 
According to Lenz’s law, the induced current will tend = Bω  −  = 0
to increase the magnetic flux, so the magnetic field due 8 8
to the induced current should be in the same direction as  d I
the applied field. So I should be in clockwise direction. 27. We know that e = L  
 dt 

196 Physics–12
D:\EG_Physics-12_(26-06-2022)\Open_Files\Ch-6\Ch-6
\ 27-Jul-2022 Ved_Goswami Proof-4 Reader’s Sign _______________________ Date __________

dI 32. Yes, eddy current may be used for electromagnetic


So emf (e) linearly proportional to shielding. When a magnetic field directed towards
dt
a metallic sheet is suddenly switched on, large eddy
currents are produced is the sheet. The higher the
conductivity of the sheet, the better the shielding of the
varying magnetic field. 1+1
Induced emf

33. The current will increase, because the wires are pulled
apart the flux will leak through the gaps. Lenz’s law
demands that induced emf resist this decrease, which
O Rate of change of current
can be done by an increase in current. 1+1
34. The current will decrease. As the iron core is inserted
28. (a) Bulb B lights up due to mutual induction between in the solenoid, the magnetic field increase and the flux
coils P and Q. 1 increase. Lenz’s law implies that induced emf should
(b) As coil Q is moved towards left, the mutual induction resist this increase, which can be achieved by a decrease
between coil P and Q will decrease. 1 in current. 1+1
29. S.I. Unit of magnetic flux is weber (Wb) ½ 35. When the current is suddenly switched on, the magnetic
dφ flux passing through the ring increases. If the current in
As |E|=
dt the coil is as shown in the diagram, the induced current
1 Wb in the ring will be in anticlockwise direction as seen
1 volt = from the top according to Lenz’s law. 1
1 sec
As the flow of current is in opposite direction to that
So 1 Wb = 1 volt sec in the solenoid, they will repel each other and the ring
Methods of producing induced emf: As f = BA cos will move upward. 1
q so to induced emf, magnetic flux should be changed 36. (a) Lenz’s law: The direction of induced emf is such
with time. that it will oppose the cause which has produced it.
(i) By changing B, the magnetic field: This can be done  1
by changing the distance between the coil and the (b) Yes, emf will be induced at its ends, because the
magnet. ½ horizontal component of earth’s magnetic field is
(ii) By changing area ‘A’ of the coil placed in magnetic intercepted by it. 1
field. dφ d
37. (i) Induced emf ε = = (BA cos ωt ) = BA w

\ This can be done by moving a conducting loop dt dt
into or out of the region of magnetic field. ½ sin wt
As B, A, w are same for both loops, so induced emf
(iii) By changing the relative orientation of the coil and
is same in both loop. 1
magnetic field i.e., by changing q the angle between
the coil and the magnetic field : This is done by ε εA
(ii) Current induced, I = =
rotating a coil about an axis ^ to the magnetic field. R ρl
 ½ As area A, length l and emf e are same for both
30. When the spring is stretched and released, the conductor loops but resistivity r is less for copper, therefore
AB would oscillate about its mean position. As the current I induced is larger in copper loop. 1
magnetic flux linked with the loop will change, an 38. When a magnet is moved towards or away from a closed
induced emf will be set up across its ends. This induced coil, induced current always opposes the cause which
emf will vary with time. 1+1 has produced it. e.g. When north pole of a magnet is
31. When the bar magnet moves through the pipe, the eddy brought closer to a coil, its face towards the magnet
currents are induced in the metallic pipe. 1 will develop north polarity and repels the north pole
According to Lenz’s law, these currents will oppose of the magnet. To move the magnet towards the coil,
the cause i.e., the motion of the magnet, which has mechanical work has to be done to over come the force
produced it. So the acceleration will be less than g, the of repulsion developed between the north poles of the
acceleration due to gravity. Thus the magnet will take magnet and the coil. This mechanical work done is
more time than the similar unmagnetised cylindrical converted into electrical energy, so conservation of
iron bar. 1 energy is not violated. 1+1

Electromagnetic Induction 197


D:\EG_Physics-12_(26-06-2022)\Open_Files\Ch-6\Ch-6
\ 27-Jul-2022 Ved_Goswami Proof-4 Reader’s Sign _______________________ Date __________

39. (i) when v ^ B Induced EMF = ein


Arms AE, BF, CG and DH are parallel to velocity v, Bwr 2 0.2 × 20π × (0.1) 2
so no emf is induced in these arms. The arms AB, DC, = =
2 2
EF and HG are parallel to magnetic field B. So no emf
will be induced in these arms. = 6.28 × 10–2 V 1
−2
v E 6.28 × 10
D C D C (ii) As R = 2W, Iin = in =
R 2
A B
A B = 3.14 × 10–2 A 1
v
44. l = 0.5 m, B = 0.15T, R = 3W, v = 2m/s
G Force needed to move the rod = F
G
H
H
= B2l2v/R 1
E B F E B F 2 2 2 −3
Case I = (0.15) (0.5) × = 3.75 × 10 N  1
Case II
3
The arms AD, BC, FG and EF are ^ to both B and v. 45. Induced emf in the moving rod in the magnetic field is
Hence emf will be induced in these arms. given by
Induced emf = LBv 1 e = + Blv = 0.5 × 0.2 × 10 = 1V 1
(ii) As velocity of each conductor is parallel to B. ε 1
Current in the rod l = = = 0.2A  1
So no Lorentz force and hence no induced emf. 1 R 5
40. The near face of C1 will have the current in clockwise 46. l = 30 m, v = 450 m/s, BH = 4 × 10–4 T,
direction. As the north pole of the magnet is moving
d = 30°, E = ?
away from the coil. So the magnetic flux linked with
coil C1 will decrease. According to Lenz’s law, current As the jet will cut the vertical component so the emf
(induced) will be in clockwise direction, to increase the will be induced
magnetic flux linked with it. BV = BH tan d = 4 × 10–4 × tan 30°
In Coil L 2 induced current will be in clockwise 4
direction. 1 = × 10 −4 T  1
3
Reason: When the magnet moves toward C2, the flux
linked with it will increase. The induced current will be 4
e = Blv = × 10 −4 × 30 × 450
in clockwise direction. So as to decrease the flux through 3
C2. 1 −2
= 180 3 × 10 = 3.12 V  1
∆φ ∆φ
41. Induced emf e = − = (numerically) 1 47. Clockwise Direction.
∆t ∆t
Time in which the loop will begin to come out of the
8 × 10 −4 magnetic field.
= = 1.6 × 10 −3 V = 1.6 mV  1
0.5 Distance = 80 cm, V = 20 cm/s
42. B = 5 × 10–4 T, d = 30°, l = 25 m, distance 80
    t = = = 4 sec  ½
1800 × 5 speed 20
v= = 500 m/s 1 Time in which the whole loop will be out of the field
18
The wings of the jet plane will cut the vertical 100
t= = 5 sec  ½
component of earth’s magnetic field 20
BV = B sin d = 5 × 10–4 × sin 30° From t = 0 sec to t = 4 sec flux in constant.
= 2.5 × 10–4 T ½
ein = BVlv = 2.5 × 10 × 25 × 500
–4
Flux
= 3125 × 10–3 = 3.125 V ½ (f)
       ½
43. (i) r = 10 cm = 0.1 m, B = 0.2 T, w
= 20p rad/s O 4 5
Time (second)

198 Physics–12
D:\EG_Physics-12_(26-06-2022)\Open_Files\Ch-6\Ch-6
\ 27-Jul-2022 Ved_Goswami Proof-4 Reader’s Sign _______________________ Date __________

Induced emf, E = –

=−
(
φ f − φs )
dt ∆t
Induced     ½ 52. Take a coil consisting of a large number of turns of
EMF
insulated copper wire and connects in series with a
Time (second) galvanometer. 1
O 4 5
Bring the N-pole of a bar magnet suddenly
48. (i) ein = Blv
towards the coil. The galvanometer shows
B and v are same in both cases.
a deflection. But if the magnet is placed
l = 2.5 cm for the upper loop near the coil, the galvanometer shows
N
l = 5 cm for the lower loop zero deflection. As the magnet moves 1
So include emf is more in 5 cm × 5 cm loop 1 closer to the coil, the strength of the
E magnetic field B, passing through the G
(ii) I = , and R ∝ length  
R coil increases. So the flux φ = B ⋅ A
Perimeter of rectangle = 2 (10 + 2.5) = 25 cm also increases. But when magnet is kept
Perimeter of square = 4 × 5 = 20 cm stationary, strength of B and hence the magnetic flux
Rsq. < Rrect. Esq. > Erect. 1 does not change. So the change in magnetic flux linked
or I in rectangle is lesser than the current in the with a coil results in induced current. 1
square. OR
49. (i) Given l = 20 cm = 20 × 10–2 m,
Similarly if the rod PQ is moved over a u-shaped
R = 0.1 W, B = 0.4 T,
conductor placed in a uniform magnetic field B. Here
v = 10 cm/s = 10 ×10–2 m/s B ^ v.v is velocity of the conductor. ½
 V  Blv  B2 l 2 v
F = i l B =   lB =  l B = The area of the loop, through which the magnetic field
 R  R  R is passing changes. The galvanometer again shows
deflection. This happens because f = BA cos q changes
(0.4)2 × (20 × 10−2 ) ( )
2
× 10 × 10 −2
= because of the change in area of the coil through which
0.1 magnetic field is passing. Thus the change in magnetic
= 6.4 × 10 N –3 flux linked with a coil results in induced current. ½
 B2 l 2 v  × × × × × × × × × × ×
(ii) Power = P = F·v =  v
 R 
P
× × × × × × × × × × ×
= 6.4 × 10–3 × 10 × 10–2
= 6.4 × 10–4 W × × × × × × × × × × ×    ½
v
Bwl 2 × × × × × × × × × × ×
50. Induced emf = e = Q
2 G
× × × × × × × × × × ×
(i) E   (ii) E Laws: (i) Whenever magnetic flux limited with a coil
changes an emf is induced in the coil. It lasts so long
w as the magnetic flux is changing. ½
L l (ii) Induced emf is directly proportional to the time rate
of change of magnetic flux. ½
(iii) Induced emf opposes the cause which has produced
51. Flux through the coil f = BA cos q
it. ½
= 10–1 × 100 × 10–4 × cos 30°
53. (i) As the magnet falls towards the coil, the magnetic
3 flux linked with the coil increases and emf is induced.
= × 10−3 Wb
2 As the speed with which the magnetic is falling is
Here, ff = 0; dt = 10–3s increasing so the rate of change of flux linked with

Electromagnetic Induction 199


D:\EG_Physics-12_(26-06-2022)\Open_Files\Ch-6\Ch-6
\ 27-Jul-2022 Ved_Goswami Proof-4 Reader’s Sign _______________________ Date __________

the coil will increase and hence the induced emf V Blv
will increase till the magnet just reaches the top of =
Iinduced =
 ½
R R
the coil. At this point emf becomes maximum. As Current will flow only if the circuit is complete.
the magnet starts moving through the coil, the flux × × × × × × × × × × ×
decreases. The induced emf also decreases. ½ S P
(–)
When the magnet is completely inside the coil, the × × × × × × × × × × ×
flux linked with the coil does not change so the    ½
induced emf becomes zero. ½ × × × × × × × × × × ×
v E
As the magnet begins to move out of the coil, the × × × × × × × × × × ×
flux linked with the coil begins to decrease. The rate
(+)
of decrease of magnetic flux increases with time, R
× × × × × × × × × Q× ×
thus increasing the induced emf, but in opposite
direction. This induced emf acquires its maximum 56. Consider the forward motion from
value when the magnet is completely out of the coil x = 0 to x = 2b
at its lower end. Now as the magnet moves away. The flux FB linked with the loop is
The flux again decreases till it becomes zero. 1 fB = Blx for 0 ≤ x < b = Bl b for b ≤ x < 2b ½
(ii) Negative peak is longer than the positive peak because The induced emf is
the magnet moves out of the coil faster than it moved –dφB
into the coil. 1 e= = – Blv for 0 ≤ x < b
dt
54. The flux f linked with the coil is = 0 for b ≤ x < 2b ½
f = NBA, A = pR2 When the induced emf is non-zero, the current I is (in
Blv
f = NpR2B0 cos wt 1 magnitude) I =
r
So induced emf Outward Inward
dφ d
e= − = − NπR 2 B0 cos ωt K L M L K
dt dt
= NpR2B0w sin wt 1 Blb
M0 Flux

It will be a sinusoidally varying emf.


The direction of the induced current can be determined b 2b b O
by Lenz’s law, which states that, the direction of the x=0
induced current will be such that it will oppose the
cause which has produced it. 1 Blv
Induced

55. Consider a U-shaped conductor placed in a magnetic


EMF

field which is ^ to plane to conductor and pointing


inwards. – Blv ½
Conductor PQ is movable on this U-shaped conductor.
Let PQ be moved inwards. B2l2v
r
When the conductor is moved inwards, the electrons in
the conductor will experience a Lorentz force. End P
Force

will become –ve and Q will become +ve. So an electric 2 2

field will be set up from Q to P. This motion of electrons –Blv


r
will continue till Fe = Fm. 1 Outward Inward
Fe = Fm
qE = qvB 2 2
Blv
V  V
= vB E =  v → velocity r
l  l
Power

V = Blv        V → induced EMF 1 2b b 0  ½


b
x= 0
If the circuit is complete and the resistance of the (b)
conductor is then R,

200 Physics–12
D:\EG_Physics-12_(26-06-2022)\Open_Files\Ch-6\Ch-6
\ 27-Jul-2022 Ved_Goswami Proof-4 Reader’s Sign _______________________ Date __________

The force required to keep the arm PQ in constant dφ dx


motion is IlB. Its direction is to the left. In magnitude E= − = Bl = Blv  1
dt dt
B2 l 2 v × × × × × × × ×
F= 0≤ x<b
r S P′ P
=0 b ≤ x < 2b  ½
× × × × × × × ×
v PQ = l    ½
The joule heating loss is PJ = I2r
B2 l 2 v 2 0≤ x<b
R
× × × × × × Q′ Q
=
r dx
=0 b ≤ x < 2b  ½ When v = rate of change of position
dt
57. Eddy current/Foucault current: They are the Conductor PQ carrying current I is placed in the
currents induced in the body of metallic mass when the magnetic field. So it will experience a force.
magnetic flux threading the metal block changes. ½ F = IlB sin q = IlB sin 90° = IlB
To show the production of Eddy Current: Take a Blv Bl B2l 2 v
strong electromagnet. Keep it vertical. On the upper = =  ½
R R
end keep a cardboard and on the cardboard keep a thin So the power required to move is
metallic disc. As soon as the current in the electromagnet
= Power supplied by external force = Fv
will be switched on, the disc will be thrown upward.
2 2 2 2 2
Because due to the changing magnetic field passing P = Fv = B l v × v = B l v  1
through the disc (when current is growing) the flux R R
linked with the disc changes and hence eddy currents 59. (a) Deflection in the galvanometer can be increased by:
are induced. If the top face of the electromagnet is north (i) Increasing the rate at which the coil C2 is moved, i.e.,
then the direction of current in the disc will be such that move C2 with high speed. 1
it’s end facing electromagnet will become north pole (ii) Placing a soft iron laminated core at the centre of C1.
and will tend to oppose the increasing magnetic field  1
which has produced. ½ (b) Induced current can be demonstrated by using a
They are undesirable in a transformer as lot of energy torch bulb in place of galvanometer. 1
will be lost as heat energy due to production of eddy 60. (a) Make the connections as shown in the circuit
current. ½ diagram.
They can be minimised by using laminated core or by
cutting narrow slits in the body of the metal. ½
Application of eddy current: Eddy currents are used
in: c

(1) Induction furnace for melting metals: A metal


specimen is placed in a rapidly changing magnetic field
G b E    1
so large eddy currents are set up. The heat produced is
Galvanometer
sufficient to melt metals. It is mainly used in extraction
of metals from his ores. ½
a
(2) Electric brakes (in trains): A strong magnetic field is Insert the plug key between a and b. The current
applied to the rotating drum attached to the wheel the flows through the coil. At the upper face of the coil
eddy currents are set up in the drum and exert a torque current is in anticlockwise direction. Suppose the
on the drum. So as to stop the train. ½ deflection in the galvanometer is towards right. The
58. Consider a conductor PQ of length l free to move on a upper face of the coil will be N-pole. Now remove
U-shaped conducting nail the plug from a and b. Insert the plug between b and
A uniform magnetic field B is applied into the paper. c and move the N-pole of the bar magnet towards the
Let the conductor PQ be moved with speed v inward. coil. We see that the galvanometer shows a sudden
Let RQ = x, φi = Blx deflection towards right. So the upper face of the coil
will have a current in the anticlockwise direction.
Let QQ′ = dx
Thus the top face is acting as N-pole. Thus induced
φf = Bl (x – dx) N-pole will repel the N-pole of the bar magnet. So
dφ = φf – φi = – Bldx the induced current will decrease the magnetic flux.

Electromagnetic Induction 201


D:\EG_Physics-12_(26-06-2022)\Open_Files\Ch-6\Ch-6
\ 27-Jul-2022 Ved_Goswami Proof-4 Reader’s Sign _______________________ Date __________

This shows that the polarity of the induced current 1


is such that it opposes the change of magnetic flux Area of the I-t curve = × 1 × 0.4
2
that produce it.1+1
= 0.2 [from the graph] 1
120 × 2π \Total charge passing through the loop is 0.2 C.
61. w = 120 rev / min = = 4π rad/s  1
60 (ii) Given, Resistance of the loop. R = 10 W and DQ =
l = 0.5m, B = 0.4 × 10–4 T, d = 60° 0.2 C [from part (i)]
Wheel will be cutting horizontal component of earth’s Suppose the change in magnetic flux be Df.
magnetic field We know that
BH = B cos d = 0.4 × 10–4 cos 60° = 0.2 ×10–4 T ∆φ
2 −4 2 DQ =
Bl ω 0.2 × 10 × (.5) × 4π R
= E=
2 2 ⇒ Df = DQ × R = 0.2 × 10 = 2 Weber 1
= 3.14 × 10 V
–5
1 (iii) Given, total change in area, DA = 10 cm2
Induced emf does not depend upon the number of = 0.001 m2
spokes. So it will remain same. 1 by definition of magnetic flux, we know that
62. From t = 0 to t = 2 sec Df = B (DA)
− d φ − dBA 2 = B (0.001)
Ein = =
dt dt B = 2000 T 1
− AdB 1 64. (a)
= = − 3.14 × (.12)2 × = − 0.023 V
dt 2
0.023
= − 2.7 mA  1

Iin =
8.5
t
From t = 2 sec to t = 4 sec
dB
= 0 So Ein = 0 So Iin = 0
dt I
dB − 1
From t = 4 sec to t = 6 sec, =
dt 2 t

− AdB
Ein = = + 0.023 V, Iin = 2.7 mA  1
dt H

3 t

2
(b) In case of circular loop, rate of change of area of
the loop during its passage out of the field is not
Current in mA

1
constant, hence induced current varies accordingly.
65. (i) Induced EMF: It is the emf produced due to change
0   1 in magnetic flux linked with it.
2 4 6 Time (t)
Faraday’s Laws of electromagnetic induction:
–1
Ist law: Whenever the amount of magnetic flux linked
–2 with a coil changes, an emf is induced in the coil. The
2.7
induced emf lasts as long as the change in magnetic
–3 flux continues. 1
2 law: The magnitude of the emf induced is directly
nd

dq proportional to the rate of change of magnetic flux


63. (i) I = ⇒ ∫ dq = ∫ Idt linked with a coil. 1
dt
So, area under the I-t curve gives charge flowing in d φB
|E| ∝ where fB = magnetic flux
the loop. dt

202 Physics–12
D:\EG_Physics-12_(26-06-2022)\Open_Files\Ch-6\Ch-6
\ 27-Jul-2022 Ved_Goswami Proof-4 Reader’s Sign _______________________ Date __________

kd φ dφ Emf induced across this element.


|E| = = where k = 1 = Bvdx as (E = Blv)
dt dt
= Bxωdx as (v = xω) 1
Direction of induced current is such that it will oppose ω
× × × × × × × × ×
the cause which has produced it. A
×
× × × × × × × × × × ×
− dφ × D ×
E= C B
dt × × × × × × × × × × ×
× ×
− Nd φ × × × × ×O× × × × × ×

For N turns in the coil E =  1
dt × ×
Derivation: Consider a rod of length l, pivoted at one × × × × × × × × × × ×
×
end being rotated with uniform angular speed w in a ×
× × × × × × × × × × ×
vertical plane, normal to a uniform magnetic field B. × ×
× × × × × × × ×
Consider a small element CD of the rod at a distance EMF induced across the rod
l l
OC = x from the pivoted end. x2 1
E = ∫ Bωxdx = Bω = Bωl 2
Let v be its linear velocity. 0
2 0
2
EMF induced across the ends of this element E Bωl 2

(ii) Induced current I = =  1
= Bvdx as (E = Blv) R 2R

Topic 2. Self Induction and Mutual Induction


•• Self Induction : The phenomenon of inducing an emf in Or
a coil due to change in current in the same coil is called − LdI
self induction. As E=
dt
Due to self induction, a coil opposes the change is current E
flowing through it by inducing an emf in itself. L=
dI
Self inductance or co-efficient of self induction: If dt
current I is flowing through a coil then dI 1A
If =
s,
E = 1V , then
flux f∝I dt
or f = LI, 1V × s
where L is constant of proportionality L= = 1 Ωs = 1 H
1A
φ
L = , If I = 1A then L = f 1 Henry: Self inductance is said to be 1 Henry if an emf
I of 1V is induced in it, when the current in it is changing
Self inductance is equal to the flux linked with a coil when at the rate of 1A/s.
a current of 1A is passing through it.
Self Inductance of a Long Solenoid Coil
Or
Self inductance of a coil depends on geometry of the coil
− dφ − d − LdI E
As E= = (LI) = Or L = and is given by
dt dt dt dI / dt
m0 N 2 A
Self inductance of a coil is equal to the magnitude of L=
the emf induced in a coil when the current in the coil is l
changing at the rate of 1A/s l is length of the solenoid, N is total number of turns of
SI unit of self inductance: solenoid and A is area of cross-section of the solenoid.
φ If in place of air core, a material of relative permeability
As L= µr is used then,
I
SI unit of self inductance is= 1=
1Wb m0 m r N 2 A
Henry 1 H Lm =
A l
1 Henry: If the magnetic flux linked with a coil carrying Lm
current of 1A is 1 weber, then self inductance of the coil So, µr =
L
is said to be 1 Henry.

Electromagnetic Induction 203


D:\EG_Physics-12_(26-06-2022)\Open_Files\Ch-6\Ch-6
\ 27-Jul-2022 Ved_Goswami Proof-4 Reader’s Sign _______________________ Date __________

•• Mutual Induction: It is the property of two coils, by If a magnetic flux of 1Wb is linked with a coil due to
virtue of which a coil opposes a change in the current a current of 1A passing through the neighbouring coil,
flowing through the other by inducing an emf. then mutual inductance is 1H.
Coefficient of mutual induction: If the current at any OR
point of time in coil-1 is I1 then the magnetic flux linked
Mutual inductance of 2 coils is said to be 1 Henry if an
with the coil-2.
induced emf of 1V is induced in a coil when the current
i.e., f2 ∝ I1
in the neighbouring coil changes at the rate of 1A/s.
f2 = M21I1
(M21 = Mutual inductance of coil 2 due to coil 1) •• Co-efficient of Mutual Induction of Two Long Coaxial
φ2
Solenoids: If l is length of each coil, area of cross
M21 = section is A, and N1, N2 are total number of turns of the
I1
If I1 is 1A, then two solenoids. For air core
M21 = f2. m0 N1 N 2 A
M=
Mutual inductance: It is equal to the flux linked with l
a coil when the current flowing through neighbouring Note: If two coils are not of equal area of cross section,
coil is 1A. the area of cross-section should always be of the inner
1Wb coil, the same is true for length also.
SI unit = = 1H = 1 Henry = 1 Ws
A So we see that (i) M ∝ N1N2
− d φ2 (ii) M ∝ common cross-sectional area
Also E2 =
dt (iii) M also depends upon the material of the core.
−d dI (iv) M depends upon the relative separation and
E2 = (M 21I1 ) = − M 21 1
dt dt orientation of the coils larger the separation between
− E2 the coils, smaller will be the mutual inductance of the
M21 =
dI1 coils.
dt
Definition: Mutual inductance of two coils is equal to
magnitude of induced emf when current flowing through
neighbouring coil is changing at the rate of 1A/s.
Definition of 1H: As
f2 = M21I1 or
φ2
M21 = Minimum mutual Induction Maximum mutual Induction
I1

EXERCISE 6.2
I. Objective Type Questions (1 Mark) (c) increases when one of them is rotated about
1. Choose the correct answers from the given options an axis.
(MCQs). (d) is the same as M21 of coil 2 with respect to
(i) The self inductance L of a long solenoid of length coil 1.
l and area of cross-section A, with a fixed number (iii) A circular coil expands radially in a region of
of turns N increases as [NCERT Exemplar] magnetic field and no electromotive force is
(a) l and A increases produced in the coil. This can be because
(b) l decreases and A increases  [NCERT Exemplar]
(c) l increases and A decreases (a) the magnetic field is constant.
(d) Both l and A decreases. (b) the magnetic field is in the same plane as the
circular coil and it may or may not vary.
(ii) The mutual inductance M12 of coil 1 with respect
to coil 2 [NCERT Exemplar] (c) the magnetic field has a perpendicular (to
the plane of the coil) component whose
(a) increases when they are brought nearer.
magnitude is decreasing suitably.
(b) depends on the current passing through coils.

204 Physics–12
D:\EG_Physics-12_(26-06-2022)\Open_Files\Ch-6\Ch-6
\ 27-Jul-2022 Ved_Goswami Proof-4 Reader’s Sign _______________________ Date __________

(d) there is a constant magnetic field in the 4. Assertion (A): In the phenomenon of mutual
perpendicular (to the plane of the coil) induction, self induction of each of the coil persists.
direction. Reason (R): Self-induction arises when strength
(iv) The self-inductance of a solenoid of 600 turns is of current in one coil changes. In mutual induction,
108 mH. The self-inductance of a coil having 500 current is changing in both the individual coils.
turns with the same length, the same radius and
II. Very Short Answer Type Questions (1 Mark)
the same medium will be [CBSE 2022]
1. Define the term self-inductance of a coil, give its S.I.
(a) 95 mH (b) 90 mH
unit. [Delhi 2009, AI 2010]
(c) 85 mH (d) 75 mH
2. How does the self-inductance of an air coil change,
(v) A constant current is flowing through a solenoid.
when (i) the number of turns in the coil is decreased
An iron rod is inserted in the solenoid along its
(ii) an iron rod is introduced in the coil.
axis. Which of the following quantities will not
increase? [CBSE 2022] 3. A plot of magnetic flux (f), versus current (I) is shown
(a) The magnetic field at the centre in the figure for two inductors A and B. Which of the
two has large value of self-inductance?[Delhi 2010]
(b) The magnetic flux linked with the solenoid
(c) The rate of heating φ
(d) The self-inductance of the solenoid
A
2. Fill in the blanks.
B
(i) When current changes from + 2 A to – 2 A in
0.05 sec, an emf of 8 V is induced in a coil. The
coefficient of self inductance of the coil is ....... .
(ii) An emf of 100 mV is induced in a coil when
current in another nearby coil becomes 10 A from I
0 in 0.1 S. The coefficient of mutual induction 4. Define mutual inductance and write its SI units.
between the two coils will be .......... . [Delhi 2015]
(iii) The magnetic flux linked with a coil changes by 5. If the self-inductance of an iron core inductor
2 × 10–2 Wb when the current changes by 0.01A. increases from 0.01 mH to 10mH on introducing the
The self inductance of the coil is ....................... . iron core into it, what is the relative permeability of
 [S.P. 2019-20] the core material used? [AI 2002]
(iv) The number of turns of a solenoid are doubled
6. If the rate of change of current of 2 ampere/second
without changing its length and area of cross-
induces an emf of 40mV in the solenoid, what is the
section. The self-inductance of the solenoid will
become ................... times. [AI 2020] self-inductance of this solenoid?
3. State True or False 7. A large circular coil of radius R and a small circular
(i) An inductance L having a resistance R is connected coil of radius r are put in the vicinity of each other.
to an alternating source of angular frequency w. If the coefficient of mutual induction for this pair
wL equals. 1 mH, what would be the flux linked with the
The quality factor (Q) of the inductance is . large coil when a current of 0.5 A flows through the
R
(ii) Volt s–1A–2 is the unit of self inductance of a coil. smaller coil? [Delhi 2008 C]

For questions numbers 4 two statements are given-one 8. Write an expression for the energy stored in an
labelled Assertion (A) and the other labelled Reason inductor of inductance ‘L’, when a steady current
(R). Select the correct answer to these questions from is passed through it. Is the energy electric or
the codes (a), (b), (c) and (d) as given below. magnetic?
(a) Both A and R are true and R is the correct 9. How does the mutual inductance of a pair of coils
explanation of A change when
(b) Both A and R are true but R is NOT the correct (i) Distance between the coils is increased?
explanation of A
(ii) Number of turns in the coils is increased?
(c) A is true but R is false
 [AI 2013]
(d) A is false and R is also false

Electromagnetic Induction 205


D:\EG_Physics-12_(26-06-2022)\Open_Files\Ch-6\Ch-6
\ 27-Jul-2022 Ved_Goswami Proof-4 Reader’s Sign _______________________ Date __________

III. Short Answer Type Questions-I (2 Marks)


10. Name and define the unit used for measuring the co- K
E
efficient of mutual inductance. State the relation of L B
this unit with the units of magnetic flux and electric
current.
11. Two circular coils, one of radius r and the other of
A copper coil L wound on a soft iron core and a lamp
radius R are placed co-axially with their centres B are connected to a battery E through a tapping key
coinciding. For R >> r, obtain an expression for the K. When the key is closed, the lamp glows dimly. But
mutual inductance of the arrangement.[Delhi 2008] when the key is suddenly opened, the lamp flashes
12. Current in a circuit falls steadily from 5.0 A to 0.0 for an instant to much greater brightness explain.
A in 100 ms. If an average emf of 200V is induced, 19. Define mutual inductance between two long coaxial
calculate the self-inductance of the circuit. solenoids. Find out the expression for the mutual
[Foreign 2011] inductance of inner solenoid of length l having the
13. An average induced emf of 0.4V appears in a coil radius r1 and the number of turns n1 per unit length
when current in it is changed from 10A in one due to the second outer solenoid of same length and
direction to 10A in opposite direction in 0.4 second. n2 number of turns per unit length. [Delhi 2012]
Find the coefficient of self induction of the coil. 20. The magnetic flux linked with a large circular coil,
[S.P. 2011] of radius R, is 0.5 × 10–3 Wb when a current of 0.5
14. Figure shows an arrangement by which current flows A, flows through a small neighbouring coil of radius
through the bulb (X) connected with coil B, when ac r. Calculate the coefficients of mutual inductance for
is passed through coil A. the given pair of coils.
If the current through the small coil suddenly falls
to zero, what would be the effect in the larger coil?
 [Delhi 2008 C]
~ X
21. (a) Obtain the expression for the magnetic energy
stored in a solenoid in terms of magnetic field B,
area A and length l of the solenoid.
Coil A Coil B (b) How does this magnetic energy compare with
(i) Name the phenomenon involved. the electrostatic energy stored in a capacitor?
(ii) If a copper sheet is inserted in the gap between  [NCERT]
the coils, explain how the bulb brightness of the 22. (a) Define the term ‘self-inductance’ and write its S.I.
would change? [AI 2010] Unit.
(b) Obtain the expression for the mutual inductance of
IV. Short Answer Type Questions-II (3 Marks)
two long co-axial solenoids S1 and S2 wound one
15. How is the mutual inductance of a pair of coils over the other, each of length L and radii r1 and
affected when: r2 and n1 and n2 number of turns per unit length,
(i) Separation between the coils is increased? when a current I is set up in the outer solenoid S2.
(ii) The number of turns in each coil is increased?  [Delhi 2017]
(iii) A thin iron sheet is placed between the two coils, 23. The current flowing through an inductor of self
other factors remaining the same? inductance L is continuously increasing. Plot a graph
Explain your answer in each case. [AI 2013] showing the variation of:
(i) Magnetic flux versus current
16. Define self inductance of a coil. Show that magnetic
(ii) Magnetic potential energy stored versus the
energy required to build up the current I in a coil of
current. [Delhi 2014]
1 2
self inductance L is give by LI . [Delhi 2012] 24. Two concentric circular loops of radius 1 cm and 20
2
cm are placed coaxially.
17. Derive expression for self inductance of a long air
(i) Find mutual inductance of the arrangement.
cored solenoid of length l, cross sectional area A and
(ii) If the current passed through the outer loop is
having number of turns N.[Delhi 2009, 2012, 2015]
changed at a rate of 5 A/ms, find the emf induced
18. How does the self inductance of an air core will in the inner loop. Assume the magnetic filed on
change when (i) the number of turns in the coil is
decreased (ii) an iron rod is introduced in the coil? the inner loop to be uniform. [Delhi 2014]

206 Physics–12
D:\EG_Physics-12_(26-06-2022)\Open_Files\Ch-6\Ch-6
\ 27-Jul-2022 Ved_Goswami Proof-4 Reader’s Sign _______________________ Date __________

Answers 6.2
I. Objective Type Questions 9. (i) Decreases (ii) Increases.
1. (i) (b) (ii) (a) and (d) (iii) (b) and (d) 10. SI unit of mutual inductance is 1 Henry (1 H). ½
(iv) (d) (v) (c) Definition: If an induced emf of 1 volt is set in a coil
2. (i) 0.1 H (ii) 1 mH (iii) 2H (iv) 4 when the current in the neighbouring coil changes at
3. (i) True (ii) False 4. (a) the rate of 1 A/s then the mutual inductance of two coils
is said to be 1 Henry. 1
II. Very Short Answer Type Questions
1. The self inductance is defined as the magnitude of emf φ2
As f2 = MI1 or M =
induced in the coil, when the rate of change of current I1
in the coil is 1 ampere/second. So 1 H = 1 Wb/A ½
The S.I. unit of self-inductance is henry (H). 11. Let the current in the coil C1 be I1. The magnetic field
2. (i) Self inductance of a coil ∝ N2 at the centre of this current carrying coil is
When number of turns in coil is decreased, the self R
inductance will decrease.
C2
(ii) When an iron rod is introduced in the coil, the self-       ½
r
inductance of coil increases.
φ
3. As f = LI or L = C1
I
The slope of f Vs I graph is equal to the value of self m0 I1
inductance B1 =  ...(1) ½
2R
 φ  φ The flux linked with the coil C2
  >  
        I A I B µ 0 I1 2
So self inductance of inductor A > self inductance of f2 = B1A2 = πr  ½
2R
inductor B.
4. Mutual Inductance: It is equal to the magnetic flux φ2 µ πr 2
M= = 0  ½
linked with a coil when a current of 1A is flowing I1 2R
through the neighbouring coil. 12. E = − LdI or L = − E  1
OR dt dI
Mutual inductance of two coil is equal to the magnitude dt
of the emf induced in a coil, when the current in the − 200 × 100 × 10 −3
neighbouring coil is changing at the rate of 1A/s. = = 4H 1
−5
SI unit of mutual inductance is Henry. 13. E = 0.4V, change in current = 10 – (– 10) = 20A. ½
5. Relative permeability
|E| = LdI dt  ½
L
medium 10 mH
µr = = = 1000
Lair 0.01 mH E 0.4 × 0.4
⇒ L = dI = = 8 × 10–3 H 1
∆i 20
6. Induced emf, e = − L dt
∆t 14. (i) Mutual inductance. ½
e 40 × 10 −3 (ii) When a copper sheet is inserted in the gap between

\ L= = = 20 ×10 −3 H = 20 mH the coils, opposing eddy currents will be produced
 ∆i  2 in copper, which will result in decrease in magnetic
 
∆t flux linked with the coil B. So lesser induced emf
7. M = 1 mH = 10–3 H, I = 0.5A, f = ? will be produced. thus, the brightness of the bulb
Flux linked with the larger coil due to the current in the will decrease. 1½
smaller coil 15. Mutual inductance of a pair of coils will
= f = MI = 10–3 × 0.5 = 5 × 10– 4 Wb. (i) Decrease, when the separation between the coils is
1 2 increased. With increase in separation between the coils
8. Energy stored E = LI , I is current flowing.
2 the flux linked with the second coil will decrease, which
It is magnetic in nature. will result in decrease, of mutual induction. 1

Electromagnetic Induction 207


D:\EG_Physics-12_(26-06-2022)\Open_Files\Ch-6\Ch-6
\ 27-Jul-2022 Ved_Goswami Proof-4 Reader’s Sign _______________________ Date __________

(ii) Increase, when the number of turns in each coil is If the core is made of material of relative permeability
increased.  1 µr is placed in the coil.
(iii) Decrease when a thin iron sheet is placed between N2A
the two coils. As opposing eddy currents will be set Lm = m r m0  1
l
up in the iron sheet, which will result in decrease
in magnetic flux linked with the coil. Hence mutual m0 N 2 A
18. As L=  1
inductance will decrease. 1 l
16. (i) Self inductance refer to Ans No.1 (Very short). 1 For air core solenoid
(ii) Consider a coil of self inductance L connected with N → No. of turns in the coil
a source of varying emf. A → Area of cross section of the coil
Because of the changing current, induced emf will
l → Length of the coil
be produced due to self induction. This induced emf
will oppose the change of current. (i) As number of turns N decreases, the self inductance
So work needs to be done against this back emf in L will also decrease. 1
establishing the current. (ii) When an iron rod is introduced in the coil then
This work done is stored as magnetic potential self inductance L1 = µrL. So self inductance also
energy in the coil. 1 increase.
Let the current at any point of time be I and the As the resistance of copper is much less than the
induced emf is E. resistance of the bulb, so when the key K is put
dW dW LdI on,most of the current flow through the coil L and
Power P = = E I or = I very small current flows through the bulb thus it
dt dt dt
glows dim.
  dW = LIdI
But when key K is suddenly opened, a large induced
I0 I0
emf is set up across the copper coil (in order to
W= ∫ dW = ∫ LIdI = ∫ LIdI oppose the sudden change in current) which tends to
0 0
maintain the current through the lamp. Thus the lamp
I0 I0
 I2  flashes for a moment to a much greater brightness.
= L ∫ IdI = L  
 1
0  2 0
19. Mutual inductance: It is equal to the magnitude of the
1 2
   UB =
L I0  1 induced emf when the current in the neighbouring coil
2 is changing at the rate of 1A/s. 1
17. Self inductance of a long solenoid: Consider a long Expression for mutual inductance: n1, n2: No. of turns
solenoid of length ‘l’ and area of cross section ‘A’. per unit length in coil 1, 2, respectively.
Radius ‘r’ <<< l.
I2: current flowing through coil 2
Let n = no. of turns per unit length
Magnetic field inside the coil B2= mon2I2
Let current through solenoid = I
B = µ0nI No. of turns in coil 1 = N1 = n1l
Flux liked with Total flux through coil 1 = f1 = N1B2A1
1 turn = BA = µ0nIA f1 = N1A1µ0n2I2
Total no. of turns = nl φ1 NN A
Total flux = f = µ0nIA (nl) = µ0n2lAI 1 M12 = = µ0 1 2 1  1
I2 l
φ N2: Total no. of turns in coil 2.
= µ 0 n 2 lA
L=
I
Coil 1
Total turns = N
2
 N µ N2A
/
/
/
/
/

L = µ 0   lA = 0  1   1
 l l
Coil 2
/

I2

I 20. fB2 = 0.5 × 10–3 Wb, I1 = 0.5A, M = ?

208 Physics–12
D:\EG_Physics-12_(26-06-2022)\Open_Files\Ch-6\Ch-6
\ 27-Jul-2022 Ved_Goswami Proof-4 Reader’s Sign _______________________ Date __________

φ B2
0.5 × 10 −3 B2 = µ0n2I ½
M= = 10 −3 H 
= 1 The magnetic flux passing through the coil
I1 0.5
S1 = f = N1B2 A1
If the current through the small coil suddenly falls to
zero, so the rate of fall of current will be high,so the = n1L µ0n2Ipr12 ½
induced emf will be high. 1+1 φ
As mutual inductance M =
21. (a) The magnetic energy stored in a solenoid I
1 2 \ M = µ0n1n2Lpr12 ½
UB = LI . 23. (i) As f = LI so f ∝ I
2
Let n → no. of turns/length f Vs I will be a straight line ½
2
1  B  1 2
L (ii) Mag. P.E. = U = LI  1
UB =  1
2  µ 0 n  2
(Since B = µ0nI, for a solenoid) So, U ∝ I 2 ½
2
1  B 
= (µ 0 n 2 Al )  As L = µ0n2Al U
2  µ 0 n 
1 Flux
B2 Al 
= 1 φ
2m0
Where A is area of cross section of the coil. I I
(b) The magnetic energy per unit volume is, ½+½
UB 24. (i) We know that f = MI
uB =  (Where V is volume through and magnetic field at the centre of the bigger loop
V
which the field lines passes.)
 m I 4π×10−7 I
UB B2 B = 0 = = p × 10–6 I 1
= =  1 2R 2 × 20 ×10−2
Al 2µ 0
Flux through the smaller loop can be found as
22. (a) As f = LI, if I = 1A, then f = L
The self inductance of a coil is equal to the magnetic f = BAS = p × 10–6 I × p(0.01)2
flux linked with a coil when 1A current is flowing = p2 × 10–10 × I
through it. φ
S.I. unit of self-inductance is 1 Henry. ½ M= = p2 × 10–10 = 9.86 × 10–10 × I 1
I
(b) Consider two long co-axial solenoids S1 and S2

(ii) emf induced is given by
wound one over the other, each of length L and radii
r1 and r2 and n1 and n2 number of turns per unit length dφ d dI
e= − = − ( MI ) = −M
when a curent I is setup in the outer solenoid S2, a dt dt dt
magnetic field is produced. ½
dI
S1 e = –9.86 × 10–10 ×
dt
      ½
S2 e = –9.86 × 10–10 × 5
I = –4.93 × 10–9 volt 1

Topic 3. A.C. Generator and Transformer


•• AC Generator: It is a device which converts mechanical Theory: Consider a coil consisting of N turns of insulated
energy into alternating current. copper wire rotating in a uniform magnetic field B. Let
Principle: It is based on principle of electromagnetic the angle between B and area vector at an instant be q
induction when a closed coil is rotated in a uniform and the coil in rotating with angular speed w, then
magnetic field with its axis ^ to the magnetic field, the Magnetic flux f = NBA cos q = NBA cos wt
magnetic flux linked with the coil changes and an induced − dφ
emf hence a current is set up. Ein = = NBAω sin ωt
dt

Electromagnetic Induction 209


D:\EG_Physics-12_(26-06-2022)\Open_Files\Ch-6\Ch-6
\ 27-Jul-2022 Ved_Goswami Proof-4 Reader’s Sign _______________________ Date __________

B Working: Suppose plane of the coil is ^ to plane of


paper with AB at front and CD at the back. As the coil is
θ
rotated anticlockwise, AB moves inwards and CD moves
^n
outwards. Flux linked with coil changes. According to
Fleming’s Right Hand Rule, induced current in AB is from
A to B. Path of current is A BCDR2 B2RB1 R1A. In the load
E is maximum when resistance, current is from B2 to B1 as shown in figure 1.
sin wt = 1
Y
So Ein = E0 Sin wt
Graph of Eind Vs time
E
Ein
E0
E0 = NBAw T/2 T
X
t O T/4 3T/4 t

After half the rotation of the coil, AB is at the back and


Construction: It consists of: CD is front. On rotation CD moves inwards and AB
(i) Field magnets: N and S are the pole pieces of a strong moves outwards. Direction of induced current in AB is
horse shoe magnetic in which the coil in rotated. from B to A. So path of current is BAR1 B1RB2R2DCB
(ii) Armature: ABCD is a rectangular armature coil. It the current in the load resistance is reversed in direction
consists of a large number of turns of insulated copper as shown in figure 2.
wire wound over a laminated soft iron core.
•• Transformer: It is a device used to change the voltage
(iii) Slip rings R1 and R2 are hollow metallic rings to
of A.C.
which two ends of the coil are connected.
(iv) Carbon brushes B1 and B2 are flexible metal plates (a) Principle: Transformer are based on mutual induction
or carbon rods. They are fixed. They are kept in light i.e., when changing current is passed through one of
contact with R1 and R2, respectively. Purpose of the the two inductively coupled coils, an induced emf is
brushes is to pass on the current from the coil to the produced in the other coil.
external load resistance. Theory and working: Let NP be the no. of turns in
B C the primary and NS be the no. turns in the secondary
coil.
S N
As the alternating current flows through the primary,
A D
B1, B2 → Carbon brushes it generates an alternating magnetic flux in the core
B1 R1 R1, R2 → Slip rings which also passes through the secondary.
Emf induced in the primary
Current flows from A to B
R2 Current flows from B2 to B1
B2

R
EP = − N P , assuming coils to be ideal.
dt
(Fig 1) EP = Einput –Einduces = 0, as R = 0
C B dφ
So, EP = Ein = − N P
dt
S N dφ
Induced emf in the secondary coil: ES = − NS
dt
D A Assuming that there is no loss in the flux.
S S E N
B1 R1 = = k. where k = transformation ratio
Current flows from D to C EP NP
Current flows from B1 to B2
R2 Step up transformer: ES > EP, k > 1
B2
Possible when NS > NP
R No. of turns in secondary > No. of turns in primary.
E N , k<1
Step down transformer: S = S < 1
(Fig 2) EP NP

210 Physics–12
D:\EG_Physics-12_(26-06-2022)\Open_Files\Ch-6\Ch-6
\ 27-Jul-2022 Ved_Goswami Proof-4 Reader’s Sign _______________________ Date __________

No. of turns in secondary < No. of turns in primary Can be minimised by using a material whose BH
NS < NP area curve is low i.e., by using soft iron.
Construction: It consists of two coils of insulated (iv) Flux loss: Magnetic flux produced by primary
copper wire having different no. of turns wound on coil may not fully pass through the secondary.
the same soft iron laminated core. Can be minimised by using a core of ferromagnetic
– The coil, to which electrical energy is supplied is substance and winding the primary and secondary
called the primary coil and the coil from which energy over one another.
is drawn/out put is taken is called secondary coil. (v) Humming loss: As the transformer works some
– To minimise the energy loss due to eddy currents of the energy is converted into a humming sound.
laminated core is used. Some energy is lost as sound energy.
(c) A step up transformer coverts a low voltage into high
voltage, it does not violate principle of conservation
(Output) of energy as the increase in voltage is at the cost of
AC ~ Load current. When voltage increases the current decreases.
Input resistance
Pinput = Poutput
Primary
Secondary
IPEP = ISES
IP ES NS
Laminated soft iron core      I = =
S EP NP
– The soft iron core is used to minimise flux loss •• Uses of Transformer
and also to minimise the energy loss due to the (i) Used in transmission of electrical energy from
magnetisation and demagnetisation of the core when generating station to the consumers to reduce the
AC is passed through the coil because soft iron is transmission loss.
ferromagnetic and B-H area curve is small. (ii) Used in voltage regulators for TV, refrigerators, A.C.
(b) Energy losses in a transformer: etc.
(i) Copper-loss: Some energy is lost due to joule’s (iii) Used in stabilised power supply
heating of the copper wire. Resistance of the coils (iv) Step down transformer is used to obtain large current
should be minimised by using thick copper wire for electric welding.
of low resistance (v) Use of transformer in long distance transmission of
P = I2R electric power.
I is constant. If P power is delivered to a load R via cable of
resistance Rt
(ii) Eddy current loss: Alternating magnetic flux
induces eddy currents in the iron core, which leads P
Then, Power P = VI ⇒ I =
to energy loss in the form of heat can be minimises V
by using laminated core. Power wasted during transmission
(iii) Hysteresis loss: AC carries the core to the process 2 P2 R t
Pt = I R t = ,
of magnetisation and demagnetisation. Work is V2
done in each of these cycle resulting into loss of If power is transmitted at high voltage, power loss
energy. will be lesser.

EXERCISE 6.3
I. Objective Type Questions (1 Mark) (ii) A step up transformer operates on a 230 volt line
1. Choose the correct answers from the given options and a load current of 2 ampere. The ratio of the
(MCQs). primary and secondary windlings is 1 : 25. The
(i) In a pure inductive circuit with a.c. source, the current in the primary is
current lags behind emf by (a) 25 amp (b) 50 amp
(c) 15 amp (d) 12.5 amp
p p
(a) p (b) 2p (c) (d) (iii) Which of the following instrument do not make
2 4
use of eddy currents?

Electromagnetic Induction 211


D:\EG_Physics-12_(26-06-2022)\Open_Files\Ch-6\Ch-6
\ 27-Jul-2022 Ved_Goswami Proof-4 Reader’s Sign _______________________ Date __________

(a) Electrical brakes (b) Both A and R are true but R is NOT the correct
(b) Dead beat galvanometer explanation of A
(c) Inductor motor (c) A is true but R is false
(d) Transformer (d) A is false and R is also false
(iv) The current in the primary coil of a pair of coils 5. Assertion (A): Transformer is a device used to change
changes from 7 A to 3 A in 0.04 s. The mutual the voltage of A.C.
inductance between the two coils is 0.5 H. The Reason (R): It is based on the principle of Mutual
induced emf in the secondary coil is induction.
(a) 50 V (b) 75 V (c) 100 V (d) 220 V 6. Assertion (A): A step-up transformer cannot be used
(v) The core of a transformer is laminated to reduce as a step-down transformer.
the effect of Reason (R): A transformer works only in one
(a) flux leakage (b) copper loss direction.
(c) hysteresis loss (d) eddy current II. Very Short Answer Type Question (1 Mark)
2. Fill in the blanks. 1. Mention the two characteristic properties of the
(i) A device that rely upon the relationship between material suitable for making core of a transformer.
the electric current and the magnetic field is [AI 2012]
.........................., and ................................... .
2. A solenoid with N loops of wire tightly wrapped
(ii) .................. is a device used to change the voltage around an iron-core is carrying an electric current I.
of AC. If the current through this solenoid is reduced to half,
(iii) If the angular speed of the armature of a dynamo then what change would you expect in inductance L
is doubled then the amplitude of the induced e.m.f of the solenoid. [CBSE S.P. 2020-21]
will become .................... . III. Short Answer Type Questions-I (2 Marks)
(iv) Laminated iron sheets are used to minimise 3. Show diagrammatically two different arrangements
.................... currents in the core of transformer. used for winding the primary and secondary coils in a
[AI 2020] transformer. Assuming the transformer to be an ideal
3. State True or False one, write the expression for the ratio of its
(i) The current flowing in a step down transformer (i) Output voltage to input voltage
220 V to 22 V having impedance 220 W is 1 mA. (ii) Output current to input current [Foreign 2012]
(ii) A cooper rod of length l is rotated about one end 4. State the principle of working of a transformer. Can
perpendicular to the uniform field B with constant a transformer be used to step up or step down a d.c.
angular velocity w. The induced emf between its voltage? Justify your answer. [AI 2011]
1 5. How is the large scale transmission of electric energy
two end is Bwl2.
2 over long distances done with the use of transformers?
4. Match the Columns  [AI 2012]
Column-I Column-II 6. How much current is drawn by the primary coil of
(i) Step-up transformer (a) k < 1 a transformer which steps down 220 V to 22 V to
(ii) step-down transformer (b) k > 1 operate a device with an impedance of 220 W.
(iii) used to obtain large (c) AC generator [AI 2008]
current for electric welding 7. How much current is drawn by the primary of a
(iv) convert mechanical energy (d) A step-down transformer connected to 220 V supply when it
into alternating current transformer delivers power to a 110V –550 W refrigerator?
[AI 2016]
For questions numbers 5 and 6 two statements are given-
one labelled Assertion (A) and the other labelled Reason 8. Calculate the current drawn by the primary coil of a
(R). Select the correct answer to these questions from transformer which steps down 200V to 20V to operate
the codes (a), (b), (c) and (d) as given below. a device of resistance 20W. Assume the efficiency of
the transformer to be 80%.
(a) Both A and R are true and R is the correct
explanation of A 9. An ac generator consists off a coil of 100 turns and
cross-sectional area, 3m2, rotating at an angular
212 Physics–12
D:\EG_Physics-12_(26-06-2022)\Open_Files\Ch-6\Ch-6
\ 27-Jul-2022 Ved_Goswami Proof-4 Reader’s Sign _______________________ Date __________

frequency of 60 radians/second in a uniform magnetic 15. With the help of a labelled diagram, explain the
field of 0.04 T. The resistance of the coil is 500 ohm. working of a step-up transformer. Give reason to
Calculate (i) the maximum current drawn from the explain the following: [AI 2020]
generator and (ii) the maximum power dissipated in (i) The core of the transformer is laminated.
the coil. [Delhi 2002]
(ii) Thick copper wire is used in windings.
10. An aeroplane is flying horizontally from west to
V. Long Answer Type Questions (5 Marks)
east with a velocity of 900 km/hour. Calculate
the potential difference developed between the 16. A coil of number of terns N, area A is rotated at a
ends of its wings having a span of 20 m. The constant angular speed w, in a uniform magnetic field
horizontal component of the Earth’s magnetic field B and connected to a resistor R. Deduce expression
is 5 × 10–4 T and the angle of dip is 30°. for:
 [Delhi 2018] (i) Maximum emf induced in the coil.
IV. Short Answer Type Questions-II (3 Marks) (ii) Power dissipation in the coil.
11. The primary coil of an ideal step up transformer  [Delhi 2008]
has 100 turns and transformation ratio is also 100. 17. (i) Describe briefly with the help of a labelled
The input voltage and power are 220V and 1100 W diagram,the working of a step up transformer.
respectively. Calculate:
[Foreign 2012]
(i) The number of turns in the secondary coil
(ii) Write any five sources of energy loss in a
(ii) The current in the primary coil
transformer. Also write its uses.
(iii) The voltage across the secondary coil
(iii) A step up transformer converts a low voltage into
(iv) The current in the secondary coil high voltage. Does it not violate the principle of
(v) The power in the secondary coil. conservation of energy? Explain.
12. A circular coil of radius 8.0 cm and 20 turns rotates about  [Delhi 2009, 2011]
its vertical diameter with an angular speed of 50 rad/s
18. (i) State the principle of an a.c. generator.
in a uniform horizontal magnetic field of magnitude
3 × 10–2 T. Obtain the maximum and average emf (ii) Explain briefly, with the help of labelled diagram,
induced in the coil. If the coil forms a closed loop its working and obtain the expression for the emf
of resistance 10W. Calculate the maximum value of generated in the coil.
current in the coil. Calculate average power loss due (iii) Draw a schematic diagram showing the nature of
to joule heating. Where does this power come from? the alternating emf generated by the rotating coil
[S.P. 2008] in the magnetic field during one cycle.
13. State the principle of an ac generator and explain its 19. (i) Draw a labelled diagram of AC generator. Derive
working with the help of a labelled diagram. Obtain the expression for the instantaneous value of the
the expression for the emf induced in a coil having emf induced in the coil.
N turns each of cross-sectional area A, rotating with (ii) A circular coil of cross-sectional area 200 cm2 and
→ 20 turns is rotated about the vertical diameter with
a constant angular speed ‘w’ in a magnetic field B ,
directed perpendicular to the axis of rotation. angular speed of 50 rad s–1 in a uniform magnetic
field of magnitude 3.0 × 10–2T. Calculate the
 [Delhi 2018] maximum value of the current in the coil.
14. (i) What is the principle of transformer?
OR
(ii) Explain how laminating the core of a transformer
(i) Draw a labelled diagram of a step-up transformer.
helps to reduce eddy current losses in it.
Obtain the ratio of secondary to primary voltage
(iii) Why the primary and secondary coils of a in terms of number of turns and currents in the
transformer are preferably wound on the same two coils.
core? 3
(ii) A power transmission line feeds input power at
OR
2200 V to a step-down transformer with its primary
Show that in the free oscillations of an LC circuit, windings having 3000 turns. Find the number of
the sum of energies stored in the capacitor and the turns in the secondary to get the power output at
inductor is constant in time. [CBSE S.P. 2018-19] 220 V. [Delhi 2017]

Electromagnetic Induction 213


D:\EG_Physics-12_(26-06-2022)\Open_Files\Ch-6\Ch-6
\ 27-Jul-2022 Ved_Goswami Proof-4 Reader’s Sign _______________________ Date __________

20. (i) Draw the diagram of a device which is used to V. The resistance of the two wire line carrying
decrease high ac voltage into a low ac voltage and power is 0.5 W per km. The town gets the power
state its working principle. Write four sources of from the line through a 4000-220 V step-down
energy loss in this device. transformer at a sub-station in the town. Estimate
(ii) A small town with a demand of 1200 kW of the line power loss in the form of heat.
electric power at 220 V is situated 20 km away
from an electric plant generating power at 440  [Delhi 2019]

Answers 6.3
I. Objective Type Questions Ioutput E input NP
1. (i) (c) (ii) (b) (iii) (d) (iv) (a) (ii) = =  ½
Iinput E Output NS
(v) (d)
2. (i) Generator, electric motor 4. Transformer works on the principle of mutual
induction,which states that whenever changing current
(ii) Transformer (iii) double (iv) eddy
is passed through one of the two inductively coupled
3. (i) False (ii) True
coils, emf will be induced in the other coil.1
4. (i)-(b) (ii)-(a) (iii)-(d) (iv)-(c)
5. (a) 6. (b) Transformer cannot be used to step up or step down
a d.c. voltage, because when d.c. source is connected
II. Very Short Answer Type Questions across the primary coil, there will be no change of
1. Two characteristic properties current. So there will be no change is magnetic flux
(i) Low hysteresis loss. i.e. less B-H curve area linked with secondary coil. So there will be no induced
(ii) High permeability emf.1
2. remains same 5. At the power station, the voltage of AC is stepped up
by using step up transformer. This reduces the current
3.
that flows through the cable. As energy lost as heat is
I2Rt so it reduces the resistive power loss. Then at the
consumer end, the voltage is decreased using step down
Primary

Secondary

transformer.1+1
½ 6. Current in secondary coil,
VS 22
IS = = A = 0.1A  1
Z 220
(a) For an ideal transformer VSIS = VPIP
Soft \ Current in primary coil
iron-core VS IS 22 × 0.1
IP = V = 220 = 0.01A  1
P
Primary

Secondary

7. PO = 550 W, EP = 220 V, ES = 110 V, IP = ?


½ P 550
P0 = ES IS or IS = O = = 5A  1
ES 110
ES I 110 I P
(b) = P, =
EP IS 220 5

Figure (a) and (b) show the two different arrangements 5 × 110
used for winding the primary and secondary coils is a
⇒ IP = = 2.5 A 1
220
transformer.
E out NS 8. EP = 200V, ES = 20V, RL = 20W
(i) for an ideal transformer =  ½
Ein NP Efficiency = h = 80%
NS – No. of turns in secondary coil ES 20
IS = = = 1A
NP — No. of turns in primary coil R L 20

214 Physics–12
D:\EG_Physics-12_(26-06-2022)\Open_Files\Ch-6\Ch-6
\ 27-Jul-2022 Ved_Goswami Proof-4 Reader’s Sign _______________________ Date __________

P0 ES IS (i) E0 = 20 × 3 × 10–2 × 3.14 × (8 × 10–2)2 × 50 = 0.603 V


h = =  1 As E = E0 sin wt ½
Pin E P I P
(ii) EaV = 0 as the average of sin wt over a cycle is zero.
ES IS 20 × 1 × 100  ½
So IP = = = 0.125 A  1
ηE P 80 × 200 (iii) Maximum current
9. N = 100, A = 3m2, w = 60 rad/s, E 0 0.603 ½
=
I0 = = 0.0603A 
B = 0.04 T, R = 500 W R 10
NBAω 100 × 0.04 × 3 × 60 (iv) Average power dissipated as heat
(i) Imax = = = 1.44 A  1
R 500 1 1
=
= E 0 I0 = (0.603) (0.0603) 0.018 W  ½
  (ii) Pmax = ImaxVmax 2 2
Vmax = NBAw = 100 × 0.04 × 3 × 60 The source of this power is the external agent which
    = 720V is rotating the coil.
P max = 1.44 × 720 = 1036.8 W 1 13. ac Generator is a device which converts mechanical
5 energy into electrical energy. Its working is based upon
10. V = 900 km/h = 900 × m/s the principle of E.M. Induction, i.e. when magnetic flux
18
linked with a coil changes, induced emf/current is set
⇒ V = 250 m/s, BH = 5 × 10–4 T up in the coil. ½
l = 20 m, d = 30° Construction: An ac generator consists of the following
BV = BH tan d parts:
or BV = 5 × 10–4 × tan 30° (i) Permanent field magnet N-S for strong magnetic
1 field.
BV = 5 × 10–4 ×  1
3 (ii) Armature coil: It consists of a large number of turns
Potential difference developed of insulated copper wire wound on a laminated
e = BVl V soft iron core. It can be rotated about an axis
perpendicular to magnetic field.
5 × 10 −4 (iii) Slip rings: R1 and R2 are two metallic rings. The
e= × 20 × 250
3 two ends of the armature coil are connected to these
rings. Both the rings rotates with the rotation of
ε = 1.4 V armature coil.
    1
B C
11. NP = 100, k = 100, EP = 220V, Pin = 1100W
S N
NS
(i) Transformation ratio = K = A D
NP B1, B2 → Carbon brushes
B1 R1 R1, R2 → Slip rings
NS Current flows from A to B

100 = , So NS = 10000  ½ R2
B2 Current flows from B2 to B1
100
R
Pin 1100
(ii) Pin = E P IP or I P = = = 5A  ½
EP 220 (Fig 1)

N E C B
(iii) K = S = S , ES = KE P = 100 × 220
NP EP
S N
  = 22000 V ½
D A
IP I 5
(iv) K = ⇒ IS = P = = 0.05A  ½
IS K 100 B1 R1
Current flows from D to C
Current flows from B1 to B2
(v) Pout = ? , PO = ESIS = 22000 × 0.05 R2
B2
  = 1100 W ½+½
R
12. N = 20, r = 8 cm = 8 × 10–2 m, w = 50 rad/s
B = 3 × 10–2 T, R = 10W ½
Max. induced emf. = Eo = NBAw = NBpr2w ½ (Fig 2)

Electromagnetic Induction 215


D:\EG_Physics-12_(26-06-2022)\Open_Files\Ch-6\Ch-6
\ 27-Jul-2022 Ved_Goswami Proof-4 Reader’s Sign _______________________ Date __________

 ½

Fig. (3)
(iv) Brushes: B1, B2 are two carbon brushes. These are Expression for induced emf.
fixed but have the sliding contacts with slip rings. Let B is the magnitude of magnetic field.
Current induced in the coil is taken out to the circuit
and its direction changes after every half rotation of A is the area of each turn of the coil.
the coil. Since the current changes its magnitude and N is the total number of turns with coil.
direction and hence it is called alternating in nature.
w is the angular velocity with which armature coil
 ½ →
rotates in B .
Working: As the armature coil rotates, the magnetic
flux linked with the coil changes, and an induced The magnetic flux f linked with the coil at any
emf/current flows through it. instant t.
Suppose the plane of the armature coil is perpendicular     f = NBA cos q
to the plane of the paper in which magnetic field is where q is the angle which the normal to the plane
applied as in Fig. (3). →
of the coil makes with B .
When the coil is rotated anti-clockwise, AB side
of the armature coil moves inwards and CD moves dφ
Induced emf e = −
outwards. The magnetic flux linked with the coil dt
changes. Current is induced in the coil from A to B
d
and from C to D in the arms AB and CD respectively or e= − (NBA cos q)
and in the external circuit current flows from B2 to dt
B1. d
or e= − (NBA cos wt)
After 180° rotation (half rotation of the coil) AB is dt
at back and CD is at the front. Therefore on rotation or e = NBAw sin wt
further, AB moves outwards and CD moves inwards.
Now the current induced in AB from B to A and in or e = e0 sin wt
CD it is from D to C, and in result, the current in the where e0 = NBAw 1
external circuit flows from B1 to B2 i.e. the direction (Maximum/peak value of emf)
of current in the coil and in the external circuit
changes. This is repeated, the induced current in the 14. (i) Principle of transformer see point-3 in theory.  1
external circuit changes direction and magnitude (ii) Laminations are thin, making the resistance higher.
after every half rotation of the armature coil. Such Eddy currents are confined within each thin
current is alternating in nature. ½ lamination. This reduces the net eddy current. 1

216 Physics–12
D:\EG_Physics-12_(26-06-2022)\Open_Files\Ch-6\Ch-6
\ 27-Jul-2022 Ved_Goswami Proof-4 Reader’s Sign _______________________ Date __________

(iii) For maximum sharing of magnetic flux and (i) Core is laminated to block or minimise the paths
magnetic flux per turn to be the same in both of eddy currents to minimise heat loss against
primary and secondary. 1 resistance of core.
OR (ii) Thick copper wire is used in order to reduce the
At an instant t, charge q on the capacitor and the resistance of transformer coil to minimise heat loss.
current i are given by: 16. Consider a coil consisting of N turns of insulated copper
q(t) = qo cos wt wire rotated in a uniform magnetic field B. Let the angle
i(t) = – qo w sin wt between magnetic field and area vector at any point of
Energy stored in the capacitor at time t is time be q. The coil is rotated with angular velocity w.

1 1 q2 q02 f = NBA cos q


2 2
CV = UE = = cos (ωt ) 1 q = wt
2 2 C 2C
So f = NBA cos wt 1
Energy stored in the inductor at time t is
− dφ
1 2 1 E= = − NBAω ( − sin ωt )
UM = Li = L q02 w 2 sin 2 (wt ) dt
2 2
= wNBA sin wt
q02
=
2C
2
sin (ωt ) ( ω = 1 / LC ) 1 E = 0 when wt = 0
E = max. when wt = p/2
Sum of energies Emax = NBAw = E0
q02 q02 Ein = E0 sin wt 1
UE + UM = (cos 2 ωt + sin 2 ωt ) =
2C 2C
Power dissipated in the coil: If R is the resistance of
This sum is constant in time as q0 and C, both are the coil, then the
time-independent. 1 B
[CBSE Marking Scheme, 2018-19]

15. θ

n^     1
vi ~ P S ~ vs

E
AC voltage vi is applied as primary P of transformer Iinduced =
R
(with turns NP). By self induction, potential difference
dφ E 2 N 2 B2 A 2 ω 2 sin 2 ωt
developed is ep = –Np = vi Pinst = EI = =  1
dt R R
T T
Also, by mutual induction potential difference developed
∫ EIdt ∫ sin
2 2 2 2 2
N BA ω ωt dt
in secondary (turns Ns)
0 0
Pav = =
dφ T R×T
es = –Ns = vs
dt N 2 B2 A 2 w 2
= , 1
dφ 2R
Here, = time rate of change of magnetic flux of
dt  T T
each turn  as ∫ sin 2 ωt dt = 
 0 2 
es N v

\ = s = s 17. Given in theory part important point No.3.
ep N p vi 18. Given in theory part important point No.3.

Electromagnetic Induction 217


D:\EG_Physics-12_(26-06-2022)\Open_Files\Ch-6\Ch-6
\ 27-Jul-2022 Ved_Goswami Proof-4 Reader’s Sign _______________________ Date __________

19. (i) Given in theory part. 3 laminated. (thin sheets of soft iron insulated from
(ii) A = 200 cm = 200 × 10 m = 2 × 10 m
2 –4 2 –2 2 one another).  1

N = 20, w = 50 rad/s, B = 3 × 10–2 T 1 Hysteresis: The magnetization of the core is
repeatedly reversed by the alternating magnetic

E0 = NBA w = 20 × 3 × 10–2 × 2× 10–2 × 50
field. The repeating core magnetization process

= 6000 × 10–4 = 0.6 volt 1 takes place at the cost of electrical energy and this
OR energy appears as heat. The heat generated can be
kept to a minimum by using a magnetic material

(i) Given in theory part. 3
which has a low hysteresis loss. Hence, soft iron

(ii) EP = 2200 V, nP = 3000, nS = ?, core is often taken for the core material because the
ES = 220V magnetic domains within it changes rapidly with low
ES energy loss. ½
n
= S  1
(ii) Total electric power required,
EP nP
P = 1200 kW = 1200 × 103 W
220 nS
= Supply voltage, V = 220 V
2200 3000
Voltage at which electric plant is generating power,
3000 × 1 V = 440 V
nS = = 300 1
10 Distance between the town and power generating
station, d = 20 km
20. (i) A device which is used to increase or decrease ac
voltage is called transformer. A transformer which
converts high voltage into low voltage is called step
down transformer. 1 P
S
Working principle: A transformer works on the Primary
Secondary
coil
principle of Faradeys laws of electromagnetic coil
S′
induction in which an EMF is induced in secondary P′
coil of the transformers by the magnetic flux High Low
generated by the voltages and currents flowing in input Output
the primary coil. 1 Resistance of the two-wire lines carrying power
Sources of Losses: = 0.5 W/km
Resistance of windings: The low resistance copper Total resistance of the wires,
wire are used for the windings still has resistance
R = (20 + 20)0.5 = 20 W
and thereby contribute to heat loss I2R.
Flux leakage: The whole flux produced by the A step-down transformer of rating 4000-220 V is
primary coil may not be linked to the secondary used in the sub-station.
coil due to a wrong design of the core. Input voltage, V1 = 4000 V
Eddy currents: The changing magnetic field not Output voltage, V2 = 220 V
only induces currents in the secondary coil but also
rms current in the wire lines is given as:
currents in the iron core itself. These currents flow
in little circles in the iron core and are called eddy P 1200 × 103
currents. The eddy currents cause heat loss. The heat I= = = 300 A 1
V1 4000
loss, however, can be reduced by having the core

Case Based Question


I. We can induced Current, not only in conducting coils, it. The induced currents flow in closed paths. These
but in conducting sheets or blocks also. Whenever currents look like eddies (whirl-pools) in water and
the magnetic flux (Df) linked with metal sheets or so they are known as eddy currents.
blocks changes, an electromotive force is induced in

218 Physics–12
D:\EG_Physics-12_(26-06-2022)\Open_Files\Ch-6\Ch-6
\ 27-Jul-2022 Ved_Goswami Proof-4 Reader’s Sign _______________________ Date __________

“Eddy currents are the currents induced in solid (b) Induction furnace
metallic masses (coils, sheets or blocks) when the (c) LED lights
magnetic flux passing through them changes.”
(d) Magnetic breaks in trains
Eddy currents also oppose the change in magnetic
flux, which is the cause of their own production, so 2. Which of the following law follows when eddy
their direction can be found by Lenz’s law. currents are generated?
Experimental demonstration of eddy currents: (a) Lenz’s law (b) Gauss’s law
Take a pendulum having a flat aluminium plate as (c) Both (a) and (b) (d) None of these.
shown in figure. In the absence of any magnetic
 3. Eddy currents are considered undesirable in a
field ( B = 0), the pendulum swings freely. As the transformer because
electromagnet is switched on, the oscillations of the
pendulum get damped and in few seconds it comes (a) they create noise.
to rest. This is because as the aluminium plate moves (b) they causes wear and tear.
in between the North and South poles, magnetic flux (c) they dissipate energy in the form of heat.
passing through it get change. So, eddy currents are
(d) Both (a) and (b)
set up in it, which oppose the motion of the aluminium
plate in the magnetic field according to Lenz’s law. 4. Eddy currents travel in
(a) closed paths
(b) non-conductive materials.
Aluminium (c) axially when testing rod with an encircling
plate
S
coil.
Eddy (d) perpendicular to the test surface.
currents
5. Which of the following is true about Eddy
B N currents?
(a) By Lenz’s law, an eddy current creates a
magnetic field that opposes the magnetic field
that created it.
(b) Eddy currents are loops of electrical current
Applications of Eddy Currents:
induced within the conductors by a changing
• Electric brakes • Speedometers magnetic field in the conductor.
• Induction furnace/motor (c) Eddy currents are a cause of energy loss in
• Electro-magnetic damping/shielding alternating current inductors, generators and
1. Which of the following does not use the application transformers.
of eddy current? (d) All of the above.
(a) Electric power meters Ans. 1. (c) 2. (a) 3. (c) 4. (a) 5. (a)

IMPORTANT FORMULAE
Formula Symbols Application
1. dφ e = induced emf. Faraday's law of electromagnetic induction:
ε=
dt
2. e = – B l v B = magnetic field Motional emf
V = Velocity
l = length of the metal rod

Electromagnetic Induction 219


D:\EG_Physics-12_(26-06-2022)\Open_Files\Ch-6\Ch-6
\ 27-Jul-2022 Ved_Goswami Proof-4 Reader’s Sign _______________________ Date __________

3. V = ½ B w l2 V = e mf developed between the ends of To find emf developed between the end of the
the rod. w = angular velocity rod rotating in the magnetic field.
l = length of the rod

4. f = LI f = Magnetic flux Relation between f and L


L = Self-Inductance of the coil

5. e = – L dl/dt L = Self-Inductance of the coil. To find self-induced emf in a coil.

6. e = – M dl /dt M12 = Coefficient of mutual inductance Self-induced of a solenoid


1 12 2
dI 2
=
Rate of charge of current in the
dt
secondary coil

7. L = m m n2 Al mr = Relative permeability Self inductance of along solenoid


r 0
m0 = permeability of free

8. e = nB A w sin wt n = no. of turns of the coil Equation of a.c. generator


(w = 2 pν) B = magnetic field
A = area of the coil
v = frequency of AC

9. 1 2 U = Energy stored in inductor To find energy stored in the inductor in its


U= LI magnetic field.
2

COMMON ERRORS
S No. Errors Corrections
1. Applications of Lenz's Law Current should be marked in such a way to oppose the
increase or decrease of flux.

220 Physics–12
D:\EG_Physics-12_(26-06-2022)\Open_Files\Ch-6\Ch-6
\ 27-Jul-2022 Ved_Goswami Proof-4 Reader’s Sign _______________________ Date __________

REVISION CHART
Methods of Inducing E.M.F.
• By changing strength of the magnetic
Lenz’s Law
field. Electromagnetic Induction
Direction of induced current is such that it
• By changing area of the coil placed in The phenomenon of inducing an
will oppose the cause which has produced it.
a magnetic field. electrical current/emf due to the change
− dφ
• By changing the orientation of the coil E= in magnetic flux linked with a coil is
w.r.t. the magnetic field. dt called electromagnetic induction.

Electromagnetic Induction and Laws


Faraday’s law of induction is a basic law of electromagnetism predicting how a magnetic field
will interact with an electric circuit to produce an electromotive force (EMF).

Faraday’s Laws of Electromagnetic Eddy Current/Foucault Current Magnetic Flux


Induction They are the currents induced in the body The number of magnetic field lines
1 st law: Whenever the amount of of metallic mass when the magnetic flux passing through a given surface normally
magnetic flux linked with a coil changes, threading through them changes. is called magnetic flux.
an emf is induced in the coil. The If B is magnetic field strength, dS is area
induced emf lasts as long as the change I = εin
d φ / dt then magnetic flux
=  
in magnetic flux continues. R R B ⋅ dS = BdS cos θ
2 nd law: The magnitude of the emf It is a scalar quantity.
induced is directly proportional to the
SI unit of magnetic flux is weber (Wb)
rate of change of magnetic flux linked
with a coil. 1 Wb = 1 Tm2
d φB
|E| ∝ where fB = magnetic flux
dt
kd φ d φ
|E| = = , where k = 1
dt dt
Transformer
It is a device used to change the voltage of
A.C.
Principle: It is based on mutual induction AC Generator
when the changing current is passed through It is a device which converts mechanical
one of the two coils inductively coupled, an energy into alternating current.
emf is induced in the other coil. Principle: It is based on principle of
For a transformer electromagnetic induction when a closed
Energy losses in a transformer ES NS I P coil is rotated in a uniform magnetic field
• Copper loss k= = =
E P N P IS with its axis ^ to the magnetic field, the
• Eddy current loss magnetic flux linked with the coil changes
Step up transformer: For a step up
• Hysteresis loss and an induced emf hence a current is
transformer k > 1, ES > EP and NS > NP, IP > IS
• Flux loss set up.
Step down transformer : For a step down
• Humming loss
transformer
k < 1, ES < EP, NS < NP, IP < IS

Electromagnetic Induction 221


D:\EG_Physics-12_(26-06-2022)\Open_Files\Ch-6\Ch-6
\ 27-Jul-2022 Ved_Goswami Proof-4 Reader’s Sign _______________________ Date __________

REVISION CHART

Self inductance of a long solenoid coil 1 Henry: If the magnetic flux linked
consisting of N turns. Co-efficient of Mutual Induction or with a coil carrying current of 1A is 1
m N2A Mutual Induction of Two Long Coaxial weber, then self inductance of the coil
\ L= 0 , A is area of cross- is said to be 1 Henry.
section l Solenoids
m0 N1 N 2 A Or
If in place of air core, a material of M=
relative permeability µr is used then, l 1 Henry: Self inductance is said to be
N 1 , N 2 are number of turns, A is core 1 Henry if an emf of 1V is induced in
m0 m r N 2 A L it, when the current in it is changing at
Lm = or µr = m common area of cross section and l is length
l L of the two coils. the rate of 1A/s.

Self Induction and Mutual Induction


• Self Induction: The phenomenon of inducing an emf in a coil due to change in current in the same coil is called self induction.
Due to self induction, a coil opposes the change in current flowing through it by inducing an emf in itself.
• Mutual inductance or Coefficient of mutual induction: It is equal to the flux linked with a coil when the current flowing
through neighbouring coil is 1A.
Or
Mutual inductance of two coils is said to be 1 Henry if an induced emf of 1V is induced in a coil when the current in the
neighbouring coil changes at the rate of 1A/s.

Mutual inductance of two coils is equal to Self inductance or co-efficient of self Coefficient of mutual induction: If the
magnitude of induced emf when current induction current at any point of time in coil-1 is
flowing through neighbouring coil is Using, f = LI I1 then the magnetic flux linked with
changing at the rate of 1A/s. Self inductance is equal to the flux the coil-2.
Definition of 1H: As linked with a coil when a current of 1A is f2 = M21I1
φ2 passing through it. (M21 = Mutual inductance of coil 2 due
f2 = M21I1 or M21 =
I1 − LdI E to coil 1)
As E = or L=
If a magnetic flux of 1Wb is linked with dt dI
a coil due to a current of 1A passing dt
through the neighbouring coil, then Self inductance of a coil is equal to the
mutual inductance is 1H. magnitude of the emf induced in a coil Mutual Induction
Or when the current in the coil is changing It is the property of two coils, by virtue
at the rate of 1A/s of which a coil opposes a change in the
Mutual inductance of 2 coils is said to
be 1 Henry if an induced emf of 1V is It’s SI unit is 1 Henry (1H) current flowing through the other by
induced in a coil when the current in the 1Wb inducing an emf.
neighbouring coil changes at the rate of 1H = = 1 Ωs
1A
1A/s.

222 Physics–12
D:\EG_Physics-12_(26-06-2022)\Open_Files\Ch-6\Ch-6
\ 27-Jul-2022 Ved_Goswami Proof-4 Reader’s Sign _______________________ Date __________

IMPORTANCE OF EACH TOPIC AND FREQUENTLY ASKED TYPES OF QUESTIONS

☞ Important Topics
1. Working and principle of A.C. Generator. 2. Working and principle of Transformer.
3. Questions based on induced emf and self / mutual induction.
* Maximum weightage is of Inductance.

1. The coefficient of self-induction of a coil is 0.015 henry (H). If the current changes from 0.20 A to 1.0 A in
0.010 sec, what is the emf induced? [Ans. 1.2 V]
2. What is the maximum induced emf in a 400-turn coil of area 400 cm2 revolving at 30 rps about an axis in the plane
of coil and perpendicular to a field of 0.050 tesla(T)? [Ans. 150 V]
3. If you have one 4-H and one 12-H inductor, what inductances can you achieve by using them together? What
configurations would you use to obtain these inductances? [Ans. Series, 15H, parallel, 3 H]
4. What is the maximum flux through a coil that has an area of 400 cm2 and a value of 0.500 T (weber/m2) for B?
 [Ans. 0.020 Wb]
5. What is the energy required to establish a magnetic field by a 5.00 H inductor carrying a current of 10.0 A?
 [Ans. 250 J]
6. The north pole of a bar magnet is inserted in a flat coil (closed) of 20 turns resting on a table. If the flux changes
from 10–4 Wb to 10–3 Wb in 0.25 sec, what is the induced emf? What is the direction of the induced current in the
coil as you look down upon the coil-Clockwise or counterclockwise? [Ans. 0.07 V, counterclockwise]
7. A search coil has 60 turns, each of an area of 5 cm2. It is moved from a magnetic field of 0.5 Wb/m2 to one of 0.0
Wb/m2 in 0.15 msec. What is the induced emf? [Ans. 100 V]
8. A coil with a radius of 2.00 cm and 20.0 turns is removed from a magnetic field of 0.300 T in 4.00 msec. Find the
emf induced in the coil. [Ans. 1.88 V]
9. A rectangular coil of 100 turns has dimensions of 10 cm by 15 cm. It rotates about an axis through the midpoint of
the short sides. The axis of rotation is perpendicular to the direction of the magnetic field of strength 0.50 T, and
it is rotating at 600 rpm.
(a) When is the emf induced in the coil a maximum?
(b) Is the induced emf ever zero? If so, when?
(c) What is the maximum induced emf?
[Ans. (a) when φ = 0 and Δφ/Δt is max; (b) yes: when φ is max, and Δφ/Δt = 0; (c) 47 V]

Electromagnetic Induction 223


D:\EG_Physics-12_(26-06-2022)\Open_Files\Ch-6\Ch-6
\ 27-Jul-2022 Ved_Goswami Proof-4 Reader’s Sign _______________________ Date __________

ASSIGNMENT
I. Objective Type Questions (1 Mark)
1. Multiple choice questions:
(i) Which of the following based on the law of conservation of energy?
(a) Faraday’s law (b) Lenz’s law (c) Ampere (d) Biot-Savart’s law
(ii) The mutual inductance M12 of coil 1 with respect to coil 2
(a) increases when they are brought nearer.
(b) depends on the current passing through coils.
(c) increases when one of them is rotated about an axis.
(d) is the same as M21 of coil 2 with respect to coil 1.
2. Fill in the blanks:
(i) When the rate of change of current through a closed circuit is unity, then the induced emf produced in it is equal
to ................. .
(ii) When current changes from + 2 A to – 2 A in 0.05 sec, an emf of 8 V is induced in a coil. The coefficient of
self inductance of the coil is ............ .
II. Very Short Answer Type Questions (1 Mark)
3. How does the mutual inductance of a pair of two coils change when the distance between the coils is increased?
4. The closed loop PQRS is moving into a uniform magnetic field acting at right angles to the plane of the paper as
shown. State the direction of the induced current in the loop.
× × × × × × × × × × ×
P Q
× × × × × × × × × × ×

× × × × × × × × × × ×
v

× × × × × × × × × × ×

× × × × × × × × × × ×S R

5. Why does the core of a transformer is made of a magnetic material of high permeability?
III. Short Answer Type Questions-I (2 Marks)
6. (a) State Lenz’s Law.
(b) A metallic rod hold horizontally along east-west direction is allowed to fall under gravity. Will there be an emf
induced at its ends? Justify your answer.
7. Two circular coils, one of radius r and the other of radius R are placed co-axially with their centres coinciding, for
R > r, obtain an expression for the mutual inductance of the arrangement.
IV. Short Answer Type Questions-II (3 Marks)
8. Define self inductance of a coil and derive expression for self inductance of a long air cored solenoid of length l,
cross sectional area A and having number of turns N.
9. Define self inductance of a coil. Show that magnetic energy required to build up the current I in a coil of self
1 2
inductance L is given by LI .
2
10. (a) State the principle of an a.c. generator.
(b) Explain briefly, with the help of labelled diagram, its working.



224 Physics–12
D:\EG_Physics-12_(26-06-2022)\Open_Files\Ch-7\Ch-7
\ 27-Jul-2022 Ved_Goswami Proof-4 Reader’s Sign _______________________ Date __________

Topics Covered
7 Alternating Current

7.1 Peak and rms value of A.C./Alternating Voltage, Resistance and Reactance
7.2 Series LR, RC and LCR Circuit

C hapter map
Alternating Current Circuit
Series RCL Circuit

Passive Element Instantaneous Current Active Element


in an AC circuit
I(t) = Imax sin (wt – f)
R (Ω) Resistor L (H) Inductor C (F) Capacitor Generator Instantaneous Voltage
rms Value V(t) = Vmax sin wt

Resistance Inductive Capacitive I


Irms = max
Reactance Reactance 2 Phasor

Frequency of generator
Z (Ω) Impedance of the circuit w (rad/s)

Real number rms Value


Vmax
Module: |Z (Ω)| Vrms =
2
Angle

Phase angle: f (rad)

Topic 1. Peak and rms Value of A.C./Alternating Voltage, Resistance and


Reactance
•• Alternating Current (a.c.): When the current changes I = I0 sin wt = I0 sin 2pvt
continuously in magnitude and direction periodically,
I0 = Peak or maximum value of current
then the current is said to be alternating.
v = Frequency of a.c.
Current I is instantaneous value of current
I0 •• Direct Current (d.c.): The current which flows with a
T/2 constant magnitude in the same direction is called direct
0 t current.
T

225
D:\EG_Physics-12_(26-06-2022)\Open_Files\Ch-7\Ch-7
\ 27-Jul-2022 Ved_Goswami Proof-4 Reader’s Sign _______________________ Date __________

pointer will appear to be stationary at zero value due to


persistence of vision.
Current A.C. can be measured by using hot wire ammeter. They
work on the heating effect of current
H ∝ I2
Time The heat produced does not depend upon the direction
of current.
•• Alternating Voltage: The voltage whose magnitude
changes continuously and the direction reverses •• Root Mean Square (rms) Value of A.C.: It is that value
  

periodically, then voltage is said to be alternating. of steady current which produces the same amount of heat
in a given resistor in the same time as is produced by the
A.C. in the same resistor in one time period.
Voltage
It is also called effective value or virtual value of a.c.
V0 I0
Ieff = Iv = I=
rms = 0.707 I0
T 2
0 Time (t)
T/2
V0
Similarly Veff = =
Vv V=
rms = 0.707 V0
2
V = V0 sin wt = V0 sin 2pnt
•• Phasor: A rotating vector that represents a sinusoidally
V = Instantaneous value of voltage varying quantity is called a phasor.
V0 = Peak or maximum value of voltage
•• Phasor Diagram: A diagram that represents alternating
n = Frequency of the alternating voltage current and voltage of the same frequency as phasor
T = Time period of alternating voltage. along with proper phase angle between them is called a
•• Average or Mean Value of a.c. Over Half a Cycle: phasor diagram.
It is that value of steady current which sends the same •• A.C. Source connected across a pure resistor:
charge through a resistor as is sent by the given a.c. in
As V = V0 sin wt
the same resistor in its half time period.
I = I0 sin wt
2I0
  Iav over half a cycle = = 0.637 I0
π R

•• Average Value of Alternating Voltage Over Half a


Cycle: It is that value of steady voltage which sends the
same charge through a resistor in the same time as is
sent by the given alternating voltage through the same
resistor in its half cycle. ~
2V0 The voltage is in phase with the current
   Vav over half a cycle = = 0.637 V0
π V0 V
•• Average or Mean Value of a.c. Over a Cycle: It is that   I0 = , I rms = rms
R R
value of steady current which sends the same charge
through a resistor in the same time as is sent by the given
a.c. through the same resistor in its one cycle. Average
A
of a.c. over a cycle = 0 V0
•• Average of alternating voltage over a cycle is zero.
B
•• Ordinary moving coil galvanometer cannot be used to I0
ωt
measure a.c. because a moving coil galvanometer works 0
on the magnetic effect of current, which depends on the
•• Average power consumed by a pure resistor connected
direction of current. In one half cycle of a.c., the pointer to a.c. source
will move in one direction and in the second half it will
move in the opposite direction. As average of a.c. over a V0 I0
= Vrms I rms =
cycle is zero, so even if the frequency of a.c., is low, the 2

226 Physics–12
D:\EG_Physics-12_(26-06-2022)\Open_Files\Ch-7\Ch-7
\ 27-Jul-2022 Ved_Goswami Proof-4 Reader’s Sign _______________________ Date __________

•• A.C. Source Connected Across a Pure Inductor: When


a source of alternating voltage V = V0 sin wt is connected
to a pure inductor L, it drives a current V0
I0
 π
I = I0 sin  ωt − 
 2 90°
ωt
Current lags behind potential by p/2
Peak value of current Peak value of current
V V
I0 = 0 I0 = 0
XL XC
rms value of current
Vrms V
Irms = Irms = rms
XL XC
L A 1
V0 XC = is called capacitive reactance. Capacitive
/

ωC
reactance is the opposition offered by a capacitor to flow

0
90° of current through it.
It’s SI unit is ohm.
~ Variation of XC with frequency n is as shown in the graph.
V = V0 sin ωt
I0
•• A capacitor blocks d.c. but allows a.c. to pass.
X L = wL is called inductive reactance. Inductive Because for d.c., frequency = 0
reactance is the opposition offered by a pure inductor to 1 1
XC = = =∞
flow of a.c. through it. ωC 2πvC
V
So I0 = 0 = 0
XC
XL

XC
Frequency ()
It’s SI unit is ohm.
Variation of XL with frequency Frequency (v)
1
L= slope of the line. For a.c. frequency v has finite value so XC and hence I0
2p also have finite value.
•• A.C. Source Connected Across a Pure Capacitor •• A pure inductor offers no opposition to flow of d.c. but
When a source of alternating voltage V = V0 sin wt is opposes a.c. because for d.c.,
connected across a capacitor of capacitance. ‘C’. The Frequency = 0
current leads voltage by p/2. So, XL = wL = 0
 π For a.c., frequency has a finite value so XL and hence I0
I = I0 sin  ωt + 
 2 will have a finite value.

EXERCISE 7.1
I. Objective Type Questions (1 Mark) 3
1. Choose the correct answers from the given options (a) 5 2 A (b) 5 A
2
(MCQs).
(i) If the rms current in 50 Hz a.c. circuit is 5 A, the 5 5
(c) A (d) A
value of current 1/300 seconds after its value 6 2
becomes zero is [NCERT Exemplar]
Alternating Current 227
D:\EG_Physics-12_(26-06-2022)\Open_Files\Ch-7\Ch-7
\ 27-Jul-2022 Ved_Goswami Proof-4 Reader’s Sign _______________________ Date __________

(ii) When a voltage measuring device is connected (ii) A device ‘X’ is connected to an a.c. source V = V0
to AC mains, the meter shows the steady input sin wt. The variation of voltage, current and power
voltage of 220 V. This means[NCERT Exemplar] in one complete cycle is shown in the following
(a) input voltage cannot be AC voltage, but a DC figure.
voltage. (a) Which curve shows power consumption over
(b) maximum input voltage is 220 V. a full cycle?
(c) the meter reads not v but 〈v2〉 and is calibrated (b) Identify the device ‘X’. [S.P. 2015-16]
2
to read 〈 v 〉 . A

(d) the pointer of the meter stuck by some

V.I.P.
B
mechanical defect.
(iii) The rms current in a circuit connected to a 50 Hz C
t
ac source is 15 A. The value of the current in the
1 
circuit   s after the instant the current is zero,
is   600 [CBSE 2022]
15 2. A pure inductor of 25 mH is connected to a source of
(a) A (b) 15 2 A 220V. Find the inductive reactance and rms current
2 in the circuit, if the frequency of the source is 50 Hz.
2 [NCERT]
(c) A (d) 8 A
15 3. The graphs below represent the variation of opposition
2. Fill in the blanks. offered by the circuit element to the flow of a.c. with
(i) The frequency of a.c. supplied to our houses in the frequency of the applied emf. Identify the circuit
India is ................ . elements.
(ii) A rotating vector that represents a sinusoidally
varying quantity is called a .................. .
Opposition Opposition
3. State True or False to current to current
(i) The capacitive reactance in an a.c. circuit is
effective resistance due to capacity. v v
(a)
(ii) Capacitive reactance is inversely proportional to (b)
the frequency of a.c. 4. Explain why current flows through an ideal capacitor
For questions numbers 4 two statements are given-one when it is connected to an a.c. source but not when it
labelled Assertion (A) and the other labelled Reason is connected to a d.c. source in a steady state.
(R). Select the correct answer to these questions from [AI 2016]
the codes (a), (b), (c) and (d) as given below. 5. An electrical element X when connected to an a.c.
(a) Both A and R are true and R is the correct source, it has a current leading the voltage by p/2
explanation of A radian. Identify X and write an expression for its
reactance.
(b) Both A and R are true but R is NOT the correct
6. In an a.c. circuit, the instantaneous voltage and current
explanation of A
are V = 200 sin 300 t volt and (I = 8 cos 300 t ampere
(c) A is true but R is false respectively. Is the nature of the circuit capacitive or
(d) A is false and R is also false inductive? Give reason. [AI 2015]
4. Assertion (A): A pure inductor offers no opposition 7. What is the power dissipated in an a.c. circuit in which
to flow of d.c. E = 230 sin (wt + p/2); I = 10 sin wt?
Reason (R): Because for d.c., frequency is zero, 8. State which of the two, capacitor or an inductor, tends
therefore XL = wL = 0. to become short when the frequency of the applied
alternating voltage has a very high value?
II. Very Short Answer Type Questions (1 Mark)
[S.P. 2015]
1. (i) An alternating voltage given by V = 140 sin 314t 9. The instantaneous current in an ac circuit is i = 5 sin
in connected across a pure resistor of 50W find the 314 t, what is (i) rms value and (ii) frequency of the
rms current through the resistor. [Delhi 2012] current.

228 Physics–12
D:\EG_Physics-12_(26-06-2022)\Open_Files\Ch-7\Ch-7
\ 27-Jul-2022 Ved_Goswami Proof-4 Reader’s Sign _______________________ Date __________

10. The current flowing through a pure inductor of 21. An alternating current from a source is given by
inductance 2 mH is I = 15 cos 300t ampere. What i = 10 sin 314t. What is the effective value of current
is the (i) rms and (ii) average value of current for a and frequency of source? [CBSE S.P 2020-21]
complete cycle? [Foreign 2011] III. Short Answer Type Questions-I (2 Marks)
11. Prove that an ideal capacitor in an ac circuit does not 22. A 15 µF capacitor is connected to a 220 V, 50 Hz
dissipate power. [Delhi 2008] source. Find the capacitive reactance and the current
12. What is the power dissipated in an ac circuit (rms and peak) in the circuit. If the frequency is
in which voltage and current are given by doubled, what happens to the capacitive reactance
 π and the current? [Delhi 2009][NCERT]
V = 230 sin  ωt +  and I = 10 sin wt?
 2 23. A coil when connected across a 10V d.c. supply draws
13. The instantaneous current and voltage of an a.c. circuit a current of 2A. When it is connected across 10V – 50
are given by Hz a.c. supply the same coil draws a current of 1A.
I = 10 sin 314t ampere, V = 50 sin 314t volt Explain why? Hence determine the self inductance
of the coil.
What is the power dissipation in the circuit?
24. Why is the use of a.c. voltage preferred over d.c.
[AI 2008] voltage? Give two reasons. [AI 2014]
14. What will be effect on inductive reactance XL and 25. A light bulb is rated 100 W for 220 V ac supply of 50
capacitive reactance XC, if frequency of ac source is Hz. Calculate [AI 2012, 2013]
increased?
(a) The resistance of the bulb.
15. Distinguish between the terms ‘effective value’ and
‘peak value’ of alternating current. (b) The rms current through the bulb.
16. Sketch a graph showing variation of reactance of a 26. An ac voltage V = V0 sin wt is applied across a
capacitor with frequency of the applied voltage. capacitor. Show that average power consumed per
[AI 2015] cycle is zero.
17. In India domestic power supply is at 220 V, 50Hz, 27. What is meant by mean value of a.c.? Derive an
while in USA it is 100 V, 50 Hz. Give one advantage expression for mean value of a.c. over first half cycle
and one disadvantage of 220 V supply over 110 V T
or from t = 0 to t = .
supply. 2
18. The peak value of emf in ac is E0. Write its (i) rms 28. What is meant by RMS value of current, derive an
(ii) average value over a complete cycle. expression for the rms value of alternating current
[Foreign 2011] 2π
over a cycle i.e., t = 0 to t = T or t = 0 to t = .
ω
19. An alternating current from a source is represented
by I = 10 sin 314 t 29. Find root mean square value of an alternating emf.
V = V0 sin wt.
Write the corresponding values of
(i) it’s effective value and IV. Short Answer Type Questions-II (3 Marks)
(ii) frequency of the source. 30. Figure a, b and c show three alternating circuits with
equal currents. If the frequency of the alternating emf
20. (i) What is the rms value of alternating current shown
in figure? be increased, what will be the effect on currents in
the three cases? Explain.
I R L C

2A

T/2 3T/2 t
0
T ~ ~ ~
(a) (b)
– 2A
31. (a) For given a.c. i = im sin wt,show that the average
power dissipated in a resistor R over a complete

(ii) What is the power dissipated by in ideal inductor 1
in ac circuit? Explain. [Delhi 2008] cycle is im2 R.
2

Alternating Current 229


D:\EG_Physics-12_(26-06-2022)\Open_Files\Ch-7\Ch-7
\ 27-Jul-2022 Ved_Goswami Proof-4 Reader’s Sign _______________________ Date __________

(b) A light bulb is rated at 100W for a 220 V a.c. with frequency of the applied alternating voltage.
supply. Calculate the resistance of the bulb. An ac voltage V = V0 sin wt is applied across a
[AI 2013] pure inductor of inductance L. Find an expression
32. Define the term capacitive reactance. Show for the current i, flowing in the circuit and show
graphically the variation of capacitive reactance with mathematically that the current flowing through it
frequency of applied alternating voltage. lags behind the applied voltage by a phase angle
An ac voltage V = V0 sin wt is applied across a pure p
of . Also draw (a) phasor diagram (b) graphs
capacitor of capacitance C. Find an expression for 2
current flowing through it. Show mathematically the of V and i versus wt for the circuit.
current flowing through it leads the applied voltage [Delhi 2008 C, AI 2016]
p (ii) A 50W–100V electric bulb is to be used on a
by angle . [AI 2008C]
2 200V –50 Hz ac supply. Calculate the inductance
33. (i) Explain the term inductive reactance. Show of the lamp so that it may glow with its normal
graphically the variation of inductive reactance brightness. (Take p = 3.14) [S.P. 2003]

Answers 7.1
I. Objective Type Questions 1
1. (i) (b) (ii) (c) (iii) (a) 5. X is a capacitor. XC =
ωC
2. (i) 50 cps (ii) Phasor 6. V = 200 sin 300t
3. (i) True (ii) True
 π
4. (a) I = 8 cos 300t = 8 sin  300 t + 
 2
II. Very Short Answer Type Questions
As current leads potential by p/2 the circuit is capacitive.
1. (i) V0 = 140 V, R = 50W
7. As phase difference = f = p/2
V V × 0.707 140 × 0.707
Irms = rms = 0 = = 1.98A π
R R 50 P = ermsIrms cos f = ε rms I rms cos = 0
2
(ii) (a) Curve A = Power
1 1
(b) Device X is a capacitor. 8. Capacitor : For capacitor XC = =
ωC 2πvC
2. L = 25 mH = 25 × 10–3 H, Vrms = 220 V
As the frequency becomes very high, XC tends to
v = 50 Hz, w = 2pv = 100p rad/s becomes negligible, so the current will be very high.
XL = wL = 100p × 25 × 10–3 = 7.85 ohm 9. Given : I = 5 sin 314t...(i)
V 220
Irms = rms = = 28A Comparing it with I = I0 sin wt...(ii)
XL 7.85 We get I0 = 5A, w = 314
3. For figure (a): The circuit element is a pure resistor. I0 5
For figure (b): The circuit element is a pure inductor. \ (i) rms value Irms == = A 3.5 A
2 2
For an inductor XL = wL i.e. XL ∝ L
ω 314
1 1 (ii) Frequency v = = = 50 Hz
4. As capacitive reactance. XC = = 2π 2 × 3.14
ωC 2πvC
For d.c. v = 0 so XC is infinity 10. Peak value of current (I0) = 15A
V I0 15 15 2
So the current I = =0 (i) Irms = = = × = 7.5 2A
XC 2 2 2 2
(ii) Iav = 0
No current will flow through a capacitor, if it is
connected to a d.c. source for a.c., v is finite, so XC has 11. Power dissipated in ac circuit, P = VrmsIrms cos f where
V R
a finite value thus I = will have finite value. cos f =
XC Z
So the current flow through an ideal capacitor when it is For an ideal capacitor R = 0
connected to an a.c. source but not when it is connected R
\ cos f = =0
to a d.c. source in a steady state. Z
230 Physics–12
D:\EG_Physics-12_(26-06-2022)\Open_Files\Ch-7\Ch-7
\ 27-Jul-2022 Ved_Goswami Proof-4 Reader’s Sign _______________________ Date __________


\ P = VrmsIrms cos f = VrmsIrms × 0 = 0 (zero) 314
(ii) w = 2pv = 314, v = = 50 Hz
1 2 × 3.14
12. Power dissipated P = V0 I0 cos φ
2 1

p  I 2 + I 22 + I32  2
V0 = 230 V, I0 = 10A, f = 20. (i) Irms =  1 
2  3 
1 π 1

\ × 230 × 10 cos = 0.
P=  22 + ( − 2) 2 + 22  2
2 2
=   =2
13. Phase difference between V and I is zero.  3 
I0 = 10A, V0 = 50V, f = 0
(ii) The power P = VrmsIrms cos f
1
Power Pav = Vrms Irms cos f = V0 I0 cos φ R
2 where, cos f = For an ideal inductor
Z
1
= × 50 × 10 cos 0° = 250 W π
2 f= so P = 0
14. The inductive reactance XL = wL = 2pfL will increase 2
with the increase of frequency f, while capacitive 21. Given i = 10 sin 314t
reactance \ i0 = 10 and w = 314
1 1 i0 10
XC = = ieff = = = 7.07 A
ωC 2πfC 2 2
will decrease with the increase of frequency. ω 314
and v= = = 50Hz
15. Alternating current changes in magnitude and direction 2π 2 × 3.14
both. The maximum value of the alternating current is
called the peak value. It is denoted by I0. The square 22. Vrms = 220V, v = 50 Hz, C = 15 µF = 15 × 10–6F,
root of mean of the square value of current is called the XC = ?, Irms = ?, I0 = ?
‘effective value’ or ‘rms value’ of current.
1 1 1
I XC = = =
Effective value = Ieff = 0 ωC 2πvC 2 × π × 50 × 15 ×10 −6
2
Effective value of ac is the value of that steady current 1 × 106
= = 212 ohm  1
which will produce the same amount of heat in a 314 ×15
resistor as in produced by AC in one cycle. Vrms 220
Irms = = = 1.04A
16. X C 212
Reactance

I0 = 2 I rms = 1.414 × 1.04 = 1.47 A  ½


XC
When the frequency is doubled, XC will become half
212
of its previous value i.e., = 106 Ω  ½
2
Frequency 23. In case of d.c. the opposition to current is by the
17. Disadvantage: High voltage can be dangerous. resistance. But in case of a.c. the opposition to the
Advantage: Line loss is lesser. current is because of impedance of the coil and
18. E0 = Peak value of emf 1

E
(
Z = R 2 + X 2L ) 2 .
(i) rms value: Erms = 0 As Z > R so current reduces when a.c. source is
2
connected.
(ii) Average value: Eav = 0
V 10
I I case: R = = = 5Ω  ½
19. (i) Ieff = Irms = 0 = 0.707 × I0 I 2
2
V 10
= 0.707 × 10 = 7.07 A II case: Z =
= = 10Ω  ½
I 1

Alternating Current 231


D:\EG_Physics-12_(26-06-2022)\Open_Files\Ch-7\Ch-7
\ 27-Jul-2022 Ved_Goswami Proof-4 Reader’s Sign _______________________ Date __________

As Z2 = R2 + X 2L 27. It is that value of steady current which sends the same


charge in a circuit as is sent by the given AC is its half
1 1
or XL =  Z2 − R 2  2 = 102 − 52  2 time period.
    Consider an AC source connected across a resistance
= 75 = 5 3 ohm.  ½ R

XL = 2pvL = 5 3
5 3 5 3 5 3
L= = = = 0.0276H
2πv 2 × 3.14 × 50 314
 ½
~
V = V0 sin ωt
24. (i) A.C. can be transmitted to distant places without any
significant line loss by using step up transformers.1 Let I = instantaneous value of current
(ii) A.C. can be reduced by using a choke coil without dq
I= or dq = Idt ½
any significant loss of energy. As average power dt
consumed by a choke coil in one cycle is zero. 1 T/2
2
Vrms
220 × 220 Total charge q = ∫ dq = ∫ Idt
25. (a) R = = 484Ω 1
= 0
P 100
T T/2
P 100
(b) Irms = = = 0.45A 1 2
 − cos ωt 
Vrms 220 q= ∫ I0 sin ωt dt = I0  ω 

26. Consider a capacitor of capacitance C connected across 0
0
an a.c. source.
C I0  2π  T  
= −  cos   − cos 0°
ω T 2 
I0 2I
= − [ − 1 − 1] = 0 ...(1) ½
ω ω
~ Let Iav be the steady current
V = V0 sin ωt T
q = Iav × ...(2)
Let the potential difference across the capacitor at an 2
instant of time is V = V0 sin wt 2I T
So current I = I0 cos wt, because current leads potential Comparing (1) and (2) 0 = Iav ×
ω 2
p
by . 4I0 4I0 T 2I0
2 Iav = = =
So Pinst = VI = V0I0 sin wt cos wt ω T 2π T π
dW VI 2I0
= 0 0 sin 2ωt Iav = = 0.63I0
dt 2 3.14
VI 2I
dW = 0 0 (sin 2ωt dt )  1 So Iav = 0.63I0 = 0  1
2 π
T
V0 I0 28. RMS value: It is that value of steady current which
W= ∫ dW = ∫ 2
sin 2ωt dt will produce the same heating effect in a resistor as it
0 produced by the AC when passed for the same time.½
T
− V0 I0  cos 2ωt  Consider source of AC connected across resistor R.
=
2  2ω  0 R

− V0 I0  2π 
=
4ω cos 2 × T × T − cos 0 = 0
W ~
Pav = = 0 1 E = E0 sin ωt
T
232 Physics–12
D:\EG_Physics-12_(26-06-2022)\Open_Files\Ch-7\Ch-7
\ 27-Jul-2022 Ved_Goswami Proof-4 Reader’s Sign _______________________ Date __________

Let instantaneous value of current = I   sin 2 ωt  


T
V02
Amount of heat produce in time dt = dH =  T −   
2R  2ω  0 
dH = I2 Rdt.
2
= V0  1  2π 
T

H= ∫ dH = ∫ I R dt
2
2R T − 2ω  sin 2 × T × T − sin 0 
0
 
2
= V0 [T ] 
T T
...(1) ½
= R∫ I02 ∫ sin
2
sin ωt dt = RI02 2
ωt dt 2R
0 0
2
T Vrms T
2  1 − cos 2 ωt  Also H= ...(2) ½
= RI0 ∫   dt  ½ R
 2
0 Comparing (1) and (2)
RI02  
T T
2
Vrms T V2T
= ∫ dt − ∫ cos 2 ωt dt  = 0
2  0 0  R 2R
RI02   sin 2 ωt  
T
2 V02
=  T −    Vrms =
2  2ω  0  2
V0
RI02  1  2π    So Vrms = = 0.707 V0  ½
=
2 T − 2ω  sin 2 × T × T − sin 0  2
 
30. In circuit (a), there will be no change in current, as
RI02  1  in case of a resistor, the opposition to current is not
=
2 T − 2ω ( 0 − 0)  affected by change in frequency. 1
(b) In case be as frequency v increases
RI02 RI02 T 2 RT
= = [T ] = I0 ...(1) ½ XL = 2pvL will also increase.
2 2 2
Thus the current will decrease. 1
If Irms = steady current, then heat produced 1
H = I2rms RT ...(2) (c) In case of a capacitor XC =
2pvC
Comparing equation (1) and (2) As the frequency of the source will increase the XC
RT will decrease thus the current will increase. 1
I02 2
= I rms RT
2 31. (a) When a.c. source is connected across a
I2 resistor, current and potential are is phase so
I 2rms = 0 V = Vm sin wt
2
dW
I0 I0 Instantaneous power P =
Irms == = 0.707 I0 . ½ dt
2 1.414
\ dW = Pdt = VIdt = Vm sin wt Im sin wt dt
29. Amount of heat produced in dt time = dH
T
V2
∫ dW = ∫ Vm Im sin ωt dt 
2
dH = dt W= ½
R 0
T
V2 T
H= ∫ dH = ∫ dt = Vm I m (1 − cos 2 ωt ) dt
0
R ∫ 2
0
T T
V02 sin 2 ωt V02 Vm I m 
T T

∫ R ∫ sin ωt dt
2
= dt =
R = ∫ dt − ∫ cos 2ωt dt 
0 0 2  0 0 
T
V02  1 − cos 2 ωt  Vm I m 
R ∫0 
=   dt  ½ = T − [sin 2ωt ]0 
T
2 2  
V02  
T Τ
Vm I m V I
=  ∫ dt − ∫ cos 2 ωt dt  = ( T − 0) = m m T  ½
2R  0 0  2 2

Alternating Current 233


D:\EG_Physics-12_(26-06-2022)\Open_Files\Ch-7\Ch-7
\ 27-Jul-2022 Ved_Goswami Proof-4 Reader’s Sign _______________________ Date __________

W Vm I m I m RI m I 2m R  XC = Capacitive reactance
Pav == = = 1
T 2 2 2  π
I = I0 sin  ωt +   1
(b) P = 100W, Vrms = 220V, R = ?  2
2 33. (i) Inductive reactance: The opposition offered by
Pav = Vrms or R = Vrms
2
an inductor to flow of currnent is called inductive
R Pav
reactance.
2
R = (220)  1 Inductive reactance XL = wL, w is angular frequency.
100 = 484Ω
L is self inductance of the coil.
32. Capacitive reactance: It is the opposition offered to
1
current by a capacitor XC =
ωC
XL
      ½
XC
     ½
v
Expression for the current when a.c. source
is connected across an inductor: Consider an
v Frequency
inductor connected across an a.c. source.
Expression for current when capacitor is connected
L
across A.C. Source: Consider a capacitor of capacitance

/
C connected across an AC source
Let V be the potential difference between plates of
capacitor at any point of time.
q
V= as V = V0 sin ωt ~
C V = V0 sin ωt

V0 sin wt q or q = CV0 sin wt Induced emf = – L


dI
C dt
dφ LdI
I= = V0 C (ω cos ωt )  1
dt V– =0
dt
C
LdI
= V = V0 sin ωt
dt
V
dI = 0 sin ωt dt
L
~ V0
V = V0 sin ωt I= ∫ dI = ∫ L
sin ωt dt

 π V0 ( − cos ωt )
I = V0 ωC sin  ωt +  = + C ½
 2 L ω
π As the applied emf is sinusoidal so we expect the
∴Current is leading potential by current also to be sinusoidal. Thus the average of
2  ½ current must be 0 over a time period. As the average
Max value of of cos wt = 0 over a time period, So the constant C
V0 V = 0.
I = I0 = V0 wC = = 0 V0 V
1 XC I= ( − cos ωt ) = − 0 cos ωt
ωC ωL ωL
1 V0 π
= XC [Reactance of capacitor] 
ωC   = sin  ωt − 
ωL  2

234 Physics–12
D:\EG_Physics-12_(26-06-2022)\Open_Files\Ch-7\Ch-7
\ 27-Jul-2022 Ved_Goswami Proof-4 Reader’s Sign _______________________ Date __________

V0  π (ii) Rating is 50 W –100V


Max. current Imax = when sin  ωt −  =1 P 50 1
ωL  2 Irms = = = A
Vrms 100 2
 π V across inductor in series RL circuit
I = I0 sin  ωt − 
 2 1

VL = ( 2
Veff − VR2 ) 2 2
as Veff = VL2 + VR2
p

\ V (potential) leads I (current) by  ½ 1
2
2 2
= (200 − 100 ) 2
Y
VL = 100 3V  ½
Graph :
V or I
V V0 The bulb will glow with original brightness of the
1
current through it is – A. So
V0
V 2
I
I0
2 VL 100 3
½ XL = = = 200 3Ω
t
O /2 3/2 t O X
/2 I rms 0.5
As XL = wL = 2pvL
(a) I I0
XL 200 3
(b) L= = = 1.10 H  ½
Y 2πv 2 × 3.14 × 50

Topic 2. Series LR, RC and LCR Circuit


•• A.C. source across a series RL circuit: Peak P.D. across V0 1
resistor Z= =  R 2 + X 2L  2
I0  
VOR = I0R
Potential leads current by f,
R L
BD V0L I0 X L X L
tan f = = = =
OB V0R I0 R R
•• A.C. Source Across a Series RC Circuit
Peak P.D. across R = V0R = I0R
~
V = V0 sin ωt Peak P.D. across C = V0C = I0XC
R C
Y
D
A
V0 I0
VOL B
φ
C VOR ~
V = V0 sin ωt
X′ X
O
I0
Peak P.D. across inductor B
A
VOL = I0XL
The resultant VOR
φ VO
OD2 = OB2 + BD2 D
O
V02 = (I0R)2 + (I0XL)2
1 VOC
(
V0 = I0 R 2 + X 2L ) 2 C

Alternating Current 235


D:\EG_Physics-12_(26-06-2022)\Open_Files\Ch-7\Ch-7
\ 27-Jul-2022 Ved_Goswami Proof-4 Reader’s Sign _______________________ Date __________

Resultant OD2 = OB2 + BD2 1


V0 = I 0  R 2 + ( X C − X L ) 2  2
Vo2 = (I0R)2 + (I0XC)2  
1 1
V0 V
Z = 0 =  R + ( X C − X L ) 
2 2 2
Z= =  R 2
+ X 
2 2

I0 
C
I0
Potential lags behind current by f I0
A
BD VOC X C
tan f = = = B
OB VOR R C VOR VO F
•• A.C. Source Across a Series LCR Circuit:


VOL
Peak P.D. across R = I0R; Peak P.D. across L = I0XL; O
Peak P.D. across C = I0XC
E
R L C
VOC
D
Case II
Current leads potential by f,
~ BF VOC − VOL X C − X L
V = V0 sin ωt tan f = = =
OB VOR R
(i) If VL > VC
Circuit is capacitive.
OF2 = OB2 + BF2
•• Variation of Current with Frequency in a Series LCR
V02 = V 2 + ( V − V )2 Circuit
OR OL OC
2
1 
2 1 
  V0 = I0  R 2 + (X L − X C )2  2 As Z2 = R +  ωL − 
   ωC 
V0 1 V
=  R 2 + (X L − X C ) 2  2 and I=
Z= Z
I0
Case I: If w → 0, Z → ∞
BF VOL − VOC X L − X C So I → 0, i.e., Z is very high and I is very low.
tan f = = =
OB VOR R Case II: As w → ∞ i.e., w is very high
Voltage leads current by f. Circuit is inductive. Z → ∞, I → 0
VOL F So again Z is very high and I is very low.
I0 1
V0 vr =
E A 2π LC
B
VOL – VOC VOR
Peak current

O
VOC

D V0C
Case I
v = vr Frequency
(ii) If
VL < VC
OE = VOC – VOL Case III: When XL = XC

In this case 1
i.e., wL =
OF2 = OB2 + BF2 ωC
1
+ ( VOC − VOL )
2

2
V02 = VOR or w=
LC
236 Physics–12
D:\EG_Physics-12_(26-06-2022)\Open_Files\Ch-7\Ch-7
\ 27-Jul-2022 Ved_Goswami Proof-4 Reader’s Sign _______________________ Date __________

Z is minimum and Z = R 3. Power dis- Power dissipation Power dissipation de-


So I is maximum. sipated by across XC or XL is pends on the power
This frequency is called the resonant frequency of the resistance is zero. factor cos f
circuit. maximum R
1 cos φ =
1 Z
So wr = or nr =
LC 2p LC 4. C u r r e n t a n d In case of XL, volt- Phase angle between
•• Impedance (Z): It is the total opposition to the flow of voltage are in age leads current current and voltage
a.c. in a circuit containing (R and L), (R and C), or (R, phase by p/2, in case of depends on the value
L and C). XC voltage lags be- of XL and XC.
hind current by p/2.
R = resistance,
L = Inductor Q-factor measures sharpness of resonance and it is
defined as the ratio of resonant frequency to the difference
C = Capacitor
in two frequencies taken on both sides of resonant
frequency such that at each frequencies the current
1
amplitude becomes times the values at resonance
Z
X frequency. 2
ωr ω
φ Q-factor = = r
ω 2 − ω1 2 ∆ω
R
Current amplitude = I0
Impedance of a circuit can be calculated by using
impedance triangle. I = ε0
0 1
Impedance triangle is a right angled triangle whose  2 
2 2

base represents resistance R, perpendicular represents  R + (X L − X C ) 


At w1 = wr + Dw
reactance X and hypotenuse represents the impedance
Z of the circuit. From this triangle, the phase angle f I0 ε0
=
between voltage and current can be calculated as ( at ω1 ) 1
  1  
2 2
X R 2
 R +  ω1L − 
tan f =   or  cos f =
R Z   ω1C  

•• Difference Between Resistance, Reactance and
I max ε
Impedance = 0 = 0
Reactance XL or
2 R 2
Resistance (R) Impedance Z
XC ε0 ε0
\ =
1. The opposition The opposition The total opposition 1
R 2
to the flow of offered by a pure offered by LR or RC   1  
2 2

 R 2 +  ω1L − 
current by a inductor or pure ca- or LCR circuit to the
  ω1C  
pure resistor is pacitor to the flow flow of a.c. is called 
called Resis- of a.c. in called Re- impedance. 1
tance actance   1  
2 2
2
2. Resistance Reactance de- It depends upon the R 2 =  R +  ω1L − ω C  
does not pends upon the frequency of a.c. ac-  1 
depend upon frequency of the cording to the relation 2
frequency of a.c. For an induc- Z=  1 
2R2 = R 2 +  ω1L −
a.c. tor, it is direct y  ω1C 
R 2 + (X L − XC )
2
proportional to 2
frequency XL =  1 
R2 =  ω1L −
2pvL. For a capac-  ω1C 
itor, it is inversely
proportional to 1
frequency R = ω1L −
ω1C
1
XC = 1
2pvC R = (ω r + ∆ω ) L −
(ω r + ∆ω ) C

Alternating Current 237


D:\EG_Physics-12_(26-06-2022)\Open_Files\Ch-7\Ch-7
\ 27-Jul-2022 Ved_Goswami Proof-4 Reader’s Sign _______________________ Date __________

 ∆ω  1 R
= ω r L 1 + − So w2 = ωr +
 ω r   ∆ω  2L
ω r 1 + C
 ω r  w1 = ωr −
R
−1
2L
 ∆ω  1  ∆ω  •• Power Consumed in a.c. Circuit: Average power in an
= ω r L 1 +  −  1+
 ωr  ωr C  ω r  a.c. circuit per cycle is given by
V
 ∆ω  1  ∆ω  Pav = Vrms Irms cos f = 0 I0 cos φ
= ω r L 1 + − 1− 2
  
ωr  ωr C  ω r  cos f is called the power factor
 ∆ω  1  ∆ω  Pav
= ω r L 1 + − 1− Power factor = cos f =
  
ωr  ωr C  ω r  Vrms I rms
Power factor: It is the ratio of true power (Pav) to the
L  ∆ω ∆ω  = 2 L ∆ω ...(1) apparent power (VrmsIrms)
R= 1 + − 1 + 
C ωr ωr  C ωr R R
 Power factor = cos φ = = 1
Z
 2 
2 2
ωr 1 L  R + (X L − X C ) 
Q-factor = =
2 ∆ω R C  [From (1)] (i) For pure resistive circuits f = 0
2
Vrms
Q-factor is also defined as the ratio of voltage across So   Pav = Vrms I rms =
L (inductance) or C (capacitance)at resonance to the R
p
applied voltage. (ii) For a pure inductive circuit f = , so Pav = 0
2
V IX p
Q-factor = L = L (iii) For a pure capacitive circuit: f = , so Pav = 0
VR IR 2
XL ωr L 1 L 1 L •• Wattless Current: The current in AC circuit is said to be
= = = × = wattless if average power consumed in the circuit is 0. It
R R LC R R C
is the component Irms sin f of the a.c. In an inductive or
R p
•• Band width 2 Dw = capacitive circuit as f = so power factor = 0 and so
L 2
the current is wattless.

EXERCISE 7.2
I. Objective Type Questions (1 Mark) (c) the iron core of the inductor should be
1. Choose the correct answers from the given options removed.
(MCQs). (d) dielectric in the capacitor should be removed.
[NCERT Exemplar]
(i) An alternating current generator has an internal
(iii) The selectivity of a series LCR a.c. circuit is large,
resistance Rg and an internal reactance Xg. It is
when
used to supply power to a passive load consisting
(a) L is large and R is large
of a resistance Rg and reactance XL. For maximum
(b) L is small and R is small
power to be delivered from the generator to load
(c) L is large and R is small
the value of XL is equal to
(d) L = R  [Delhi 2020]
(a) zero (b) Xg (iv) In a circuit the phase difference between the
(c) –Xg (d) Rg [NCERT Exemplar] p
alternating current and the source voltage is .
(ii) To reduce the resonant frequency in an LCR series 2
circuit with a generator Which of the following cannot be the element(s)
(a) the generator frequency should be reduced. of the circuit?
(b) another capacitor should be added in parallel (a) Only C (b) only L
to the first. (c) L and R (d) L or C  [CBSE 2022]

238 Physics–12
D:\EG_Physics-12_(26-06-2022)\Open_Files\Ch-7\Ch-7
\ 27-Jul-2022 Ved_Goswami Proof-4 Reader’s Sign _______________________ Date __________

(v) The impedance of a series LCR circuit is 3. State True or False


1 1 (i) The average power dissipation in a pure capacitor
(a) R + XL + XC (b) + + R2 in ac circuit is zero.
X C2 X 2L
(ii) The phase difference between the current and
(c) X 2L − X C2 + R 2 (d) R 2 + (X L − X C )2 voltage at resonance is p.
4. Match the Columns
[CBSE 2022]
(vi) When an alternating voltage E = Eo sin wt is Column-I Column-II
 π 1 L
applied to a circuit, a current I = Io sin  ωt +  (i) RL-circuit, tan f = (a)
 2 R C
flows through it. The average power dissipated in XL
the circuit is (ii) RC-circuit, tan f = (b)
R
(a) Erms . Irms (b) EoIo
X
E o Io (iii) LCR-circuit, tan f = (c) C
(c) (d) Zero  [CBSE 2022] R
2
XL − XC
(vii) The voltage across a resistor, an inductor, and a (iv) Q-factor (d)
R
capacitor connected in series to an ac source are For questions numbers 5 two statements are given-one
20 V, 15 V and 30 V respectively. The resultant labelled Assertion (A) and the other labelled Reason
voltage in the circuit is (R). Select the correct answer to these questions from
(a) 5 V (b) 20 V (c) 25 V (d) 65 V the codes (a), (b), (c) and (d) as given below.
[CBSE 2022] (a) Both A and R are true and R is the correct
(viii) A circuit is connected to an ac source of variable explanation of A
frequency. As the frequency of the source is (b) Both A and R are true but R is NOT the correct
increased, the current first increases and then explanation of A
decreases. Which of the following combinations (c) A is true but R is false
of elements is likely to comprise the circuit? (d) A is false and R is also false
(a) L, C and R (b) L and C 5. Assertion (A): The current in a.c. circuit is said to
(c) L and R (d) R and C [CBSE 2022] be wattless if average power consumed in the circuit
(ix) A 15 W resistor, an 80 mH inductor and a capacitor is zero. It is the component Irms sinf of the a.c.
of capacitance C are connected in series with a Reason (R): In an inductive (L) or capacitive (C)
50 Hz ac source. If the source voltage and current circuit as f = p/2 so power factor = 0 and so the
in the circuit are in phase, then the value of current is wattless.
capacitance is
II. Very Short Answer Type Questions (1 Mark)
(a) 100 mF (b) 127 mF 1. In the circuit shown, when the frequency of supply is
(c) 142 mF (d) 160 mF[CBSE 2022] doubled, how should the values of L and C be changed
 25  so that the glow in the bulb remains unchanged?
(x) A 300 W resistor and a capacitor of   mF are
 π [S.P. 2008]
connected in series to a 200 V – 50 Hz ac source.
The current in the circuit is B
L C
(a) 0.1 A (b) 0.4 A
(c) 0.6 A (d) 0.8 A [CBSE 2022]
2. Fill in the blanks.
(i) An electric bulb marked 40 W and 200 V is used
in a circuit of supply voltage 100 V. Its power
~
2. A bulb B and a capacitor C are connected in series
would be ......................
to an a.c. source. A dielectric slab is now introduced
(ii) In an ideal inductor L = 4 H and w = 100 rad/s. between the plates of the capacitor. How will the
The power developed is ................ brightness of the bulb change?

Alternating Current 239


D:\EG_Physics-12_(26-06-2022)\Open_Files\Ch-7\Ch-7
\ 27-Jul-2022 Ved_Goswami Proof-4 Reader’s Sign _______________________ Date __________

3. Sketch a graph showing the variations of impedance


of LCR circuit with the frequency of the applied 14

voltage. 12

4. The power factor of an a.c. circuit is 0.5.What is the 10

Reactance (Ω)
phase difference between voltage and current in the 8
circuit? [AI 2016] 6
5. Define the term ‘wattless current’. [AI 2015] 4
6. Define the term ‘quality factor’ of resonance in series 2
LCR circuit. What is its SI unit? [AI 2015]
0
7. In a series LCR circuit, VL = VC ≠ VR. What is the 100 200 300 400
value of power factor? [Delhi 2015] Frequency (Hz)

8. An inductor is connected in series with a bulb to an (c) Using the same axes, draw a graph of reactance
a.c. source. What happens to brightness of bulb when against frequency for the inductor given in
number of turns in the inductor is reduced? part (a).[AI 2020]
[Delhi 2002] (d) If this capacitor and inductor were connected in
series to a resistor of 10W, what would be the
9. In series LCR circuit, the plot of Imax vs w is shown in impedance of the combination of 300 Hz.
the figure. Find the bandwidth and mark in the figure.
[NCERT Exemplar] [Delhi 2003 C]
1.5
12. (i) For a series LCR circuit, connected to an a.c.
source, identify the graph that corresponding to
1

Imax.(A)
ω> . Give reason.
1.0 LC

V I V I
0.5

ωt ωt

0 0.5 1.0 1.5 2.0


(rad/s)  (a) (b)

10. An a.c. circuit consists of a series combination of (ii) If a charged capacitor C is short circuited through
circuit elements X and Y. The current is ahead of an inductor L, the charge and current in the circuit
voltage in phase by p/4. If element X is a pure resistor oscillate simple harmonically.
of 100W, name the circuit element Y, and calculate (a) In what form the capacitor and the inductor stores
the rms value current if rms voltage is 141V. energy?
11. (i) A capacitor of unknown capacitance, a resistor (b) Write two reasons due to which the oscillations
of 100W and an inductor of self inductance L = become damped. [S.P. 2015-16]
(4/p2) henry are connected in series to an ac 13. At an airport, a person is made to walk through the
source of 200V and 50 Hz. Calculate the value door way of a metal detector, for security reasons.
of the capacitance and impedance and current of If she/he is carrying anything of metal, the metal
the circuit when the current is in phase with the detector emits a sound. On what principle does this
voltage. Calculate the power dissipated in the detector work? [NCERT]
circuit.[AI 2013 C, 2016]
14. A light bulb and an open coil inductor are connected
(ii) Figure given below shows how the reactance of to an ac source through a key as shown in figure.
a capacitor varies with frequency. The switch is closed and after sometime, an iron
(a) Use the information on graph to calculate the value rod is inserted into the interior of the inductor. The
of capacity of the capacitor. [AI 2020] glow of the light bulb (a) increases (b) decreases (c)
(b) An inductor of inductance ‘L’ has the same is unchanged as the iron rod is inserted. Give your
reactance as the capacitor at 100 Hz. Find the answer with reasons. What will be your answer if ac
value of L. [AI 2020] source is replaced by dc source? [NCERT]

240 Physics–12
D:\EG_Physics-12_(26-06-2022)\Open_Files\Ch-7\Ch-7
\ 27-Jul-2022 Ved_Goswami Proof-4 Reader’s Sign _______________________ Date __________

(ii) A series RL circuit in which inductive impedance


is ‘b’ times the resistance in the circuit.
Find the value of power factor of the circuit in
each case. [AI 2008 C]
~ 22. (i) In the given circuit, the potential difference across
the inductor L and resistor R are 200 V and 150
15. An inductor L of reactance XL is connected in series
V respectively and the rms, value of current is
with a bulb B to an ac source as shown in figure.
5A. Calculate (i) the impedance of the circuit and
Explain briefly how does the brightness of the bulb
(ii) the phase angle between the voltage and the
change when (i) number of turns of the inductor is
current.
reduced and (ii) a capacitor of reactance XC = XL is
included in the circuit.  [Delhi 2015] L R
Bulb

~ ~
AC source (ii) Figure shows how the reactance of an inductor
16. In a series LCR circuit, the voltage across an indicator, varies with frequency.
a capacitor and a resistor are 30V, 30V and 60V (a) Calculate the value of the inductance of the
respectively. What is the phase difference between inductor using the information given in the graph.
the applied voltage and current in the circuit?
(b) If this inductor is connected in series to a resistor
17. A resistor ‘R’ and an element ‘X’ are connected in of 8 ohm, find what would be the impedance at
series to an ac source of voltage. The voltage is found 300 Hz?
p
to lead the current in phase by . If ‘X’ is replaced
4 8
by another element ‘Y’, the voltage lags behind the
p
current by . 6
4
Reactance (Ω)

(i) Identify elements ‘X’ and ‘Y’.


(ii) When both ‘X’ and ‘Y’ are connected in series with 4
’R’ to the same source, will the power dissipated
in the circuit be maximum or minimum? Justify 2
your answer. [Foreign 2013]
18. Define power factor. State the conditions under which 0
it is (i) maximum and (ii) minimum. [Delhi 2010] 100 200 300 400 500
19. An electric lamp connected in series with a capacitor Frequency (Hz)
and an ac source is glowing with of certain brightness. 23. An ac voltage of 100V, 50 Hz is connected across a 20
How does the brightness of the lamp change on ohm resistor and 2 mH inductor in series. Calculate
reducing the (i) capacitance and (ii) frequency? (i) impedance of the circuit (ii) rms current in the
[Delhi 2010] circuit.
20. What is the impedance of a capacitor of capacitance
24. Calculate the (i) impedance (ii) wattless current of
C in an ac circuit using source of frequency n Hz?
the given ac circuit?
[Delhi 2020]
OR C R
What is the value of impedance of a resonant series
40V 30V
LCR circuit? [Delhi 2020]
III. Short Answer Type Questions-I (2 Marks)
21. An AC voltage V = Vm sin wt is applied across:
(i) A series RC circuit in which capacitive impedance
~
2A
is ‘a’ times the resistance in the circuit.
Alternating Current 241
D:\EG_Physics-12_(26-06-2022)\Open_Files\Ch-7\Ch-7
\ 27-Jul-2022 Ved_Goswami Proof-4 Reader’s Sign _______________________ Date __________

25. Calculate the quality factor of a series LCR circuit maximum? What can we conclude about nature of
with L = 2.0 H, C = 2 µF, and R = 10W. Mention the impedance of set up at frequency w0?
significance of quality factor in LCR circuit. [S.P. 2008]
[Foreign 2012] I0
26. A series LCR circuit is connected to an ac source (200
V, 50 Hz). The voltage across the resistor,capacitor X

and inductor are respectively 200 V, 250V and 250V.


(i) The algebraic sum of the voltages across the three Y

elements is greater than the voltage of the source.


How is this paradox resolved? Z
(ii) Given the value of the resistance of R is 40W
ω0 ω
calculate the current in the circuit.
[Foreign 2013]
(ii) The graphs shown below, depict the variation
250V 250V 200V
of current Im vs. angular frequency (w) for two
/

different series LCR circuits.


1.0

Imax(A)
~
200V, 50 Hz 0.5 I
II
27. Draw a plot showing the variation of the current I as
a function of angular frequency ‘w’ of the applied ac wv
source for the two cases of a series combination of (i) 0.0
0.5 1.0 1.5 2.0
inductance L1 capacitance C1 and resistance R1 and (ii)  M rad/s
inductance L2, capacitance C2 and resistance R2 where
R2 > R1. Write the relation between L1, C1 and L2, Observe the graphs carefully:
C2 at resonance. Which one, of the two, would be (a) State the relation between the L and C values
of the two circuits, when the current in the
better suited for the fine tuning in a receiver set? Give
two circuits is maximum.
reason. [Foreign 2013]
(b) Indicate the circuit for which
28. Explain the term ‘sharpness of resonance’ in ac circuit.    1. power factor is higher
OR    2. quality factor (Q) is larger.

An ac source of emf V = V0 sin wt is connected to Give reasons for each case. [Delhi 2009 C]
a capacitor of capacitance C. Deduce the expression 30. Given below are two electrical circuits A and B.
for the current (I) flowing in it. Plot the graph of (i) Calculate the ratio of power factor of circuit B to the
V vs wt, (ii) I vs. wt.  [AI 2020] power factor of circuit A.

IV. Short Answer Type Questions-II (3 Marks) XL = 3R R XL = 3R R XC = R


29. (i) Three students X, Y and Z performed an
experiment for studying variation of a.c. with
angular frequency in a series LCR circuit, and
obtained the graphs shown in figure. They all used
a.c. sources of same rms value and inductance
~ ~
of same value. What can we conclude about (A) (B)
(a) capacitance value (b) resistance value used 31. A resistor of 200W and a capacitor of 15.0 µF are
by them? In which case will the quality factor be connected in series to a 220 V, 50 Hz ac source.

242 Physics–12
D:\EG_Physics-12_(26-06-2022)\Open_Files\Ch-7\Ch-7
\ 27-Jul-2022 Ved_Goswami Proof-4 Reader’s Sign _______________________ Date __________

Calculate the current in the circuit and the rms voltage Calculate:
across the resistor and the capacitor. Is the algebraic (i) The angular frequency of the source which derives
sum of these voltages more than the source voltage? the circuit at resonance.
If yes, resolve the paradox. [AI 2008C][NCERT]
(ii) The current at the resonating frequency.
32. An inductor 200 mH, capacitor 500 µF, resistor (iii) The rms potential drop across the inductor at
10W are connected in series with a 100V, variable
resonance.[Delhi 2012]
frequency ac source. Calculate the
37. (i) State the condition for resonance to occur in
(i) Frequency at which the power factor of the circuit
series LCR a.c. circuit and derive an expression
is unity.
for resonant frequency. [Delhi 2010]
(ii) Current amplitude at this frequency. (ii) Draw a plot showing the variation of the peak
(iii) Q-factor. [Delhi 2008] current (i0) with frequency of the a.c. source used.
33. An inductor of unknown value, a capacitor of 100 Define the quality factor Q of the circuit.
µF and a resistor of 10W are connected in series to [AI 2008]
a 200V, 50Hz ac source. It is found that the power 38. An ac source of voltage V = V0 sin wt is connected,
factor of the circuit is unity. Calculate the inductance
one by one, to three circuit elements, X, Y and Z. It
of the inductor and current amplitude. [Delhi 2008]
is observed that the current flowing in them
34. The figure shows a series LCR circuit with L = 5.0 H,
(i) is in phase with applied voltage for element X.
C = 80 µF, R = 40W connected to a variable frequency
240V source. Calculate p
(ii) lags the applied voltage, in phase, by angle for
R element Y. 2

p
(iii) leads the applied voltage, in phase, by angle
2
~ C Identify the three circuit elements.
Find the expression for the current flowing is the
circuit, when the same ac source is connected
L
across a series combination of the elements X, Y
/

and Z.
(i) The angular frequency of the source which drives If the frequency of the applied voltage is varied,
the circuit to resonance.
set up the condition of the frequency when the
(ii) The current at the resonating frequency.
current amplitude in the circuit is maximum. Write
(iii) The rms potential drop across the capacitor at the expression for this current amplitude.
resonance.[Delhi 2012]
39. A circuit containing an 80mH inductor and a 250 µF
35. A series LCR circuit is connected to an ac source.
capacitor in series connected to a 240 V, 100 rad/s
Using the phasor diagram, derive the expression for
the impedance of the circuit. Plot a graph to show supply. The resistance of the circuit is negligible.
the variation of current with frequency of the source, (i) Obtain rms value of current.
explaining the nature of its variation. [AI 2012] (ii) What is the total average power consumed by the
36. A series LCR circuit with L = 4.0 H, C = 100 µF and circuit?[Delhi 2015]
R = 60 W is connected to a variable frequency 240 V 40. A resistor of 100 W and a capacitor of 100/ pmF are
source as shown in connected in series to a 220V, 50 Hz a.c. supply.
R (i) Calculate the current in the circuit.
(ii) Calculate the (rms)voltage across the resistor and
the capacitor. Do you find the algebraic sum of
C these voltages more than the source voltage? If
yes, how do you resolve the paradox? [AI 2015]
~ Source
41. (i) Determine the value of phase difference between
L the current and the voltage in the given series LCR
/

circuit.

Alternating Current 243


D:\EG_Physics-12_(26-06-2022)\Open_Files\Ch-7\Ch-7
\ 27-Jul-2022 Ved_Goswami Proof-4 Reader’s Sign _______________________ Date __________

R = 400Ω C such that XC = XL is put in series, the power factor


becomes P2. Find out P1/P2.[AI 2015]
47. (i) Figure shows the variation of resistance and
~ V = V sin (1000t + φ)
0
C = 2 µF reactance versus angular frequency. Identify the
curve which corresponds to inductive reactance
and resistance.

/
L = 100 mH
A
(ii) Calculate the value of the additional capacitor
which may be joined suitably to the capacitor C B
that would make the power factor of the circuit C
unity.  [AI 2015]
42. Obtain the resonant frequency and Q-factor of a series
LCR circuit with L = 3H, C = 27µF, R = 7.4Ω. It is
desired to improve the sharpness of resonance of circuit
by reducing its full width at half maximum by a factor
of 2. Suggest a suitable way. [CBSE S.P. 2019-20] ω

43. A resistance R and a capacitor C are connected in series (ii) Show that series LCR circuit at resonance behaves
to a source V = V0 sin wt. as a purely resistive circuit. Compare the phase
(i) The peak value of the voltage across the (a) relation between current and voltage in series LCR
resistance and (b) capacitor. circuit for (a) XL > XC (b) XL = XC using phasor
diagrams?
(ii) The phase difference between the applied voltage
and current. Which of them is ahead? (iii) What is an acceptor circuit and where it is used?
[Delhi 2020] [S.P. 2015-16]
44. A resistance R and a capacitor C are connected in 48. A device X is connected across an ac source of voltage
series to an ac source V = V0 sin wt. V = V0 sin wt. The current through X is given as
(i) Obtain the expression for the instantaneous current I = I0 sin[wt + (p/2)].
(I) in the circuit. (i) Identify the device X and write the expression for
(ii) Show graphically variations of V and I as a its reactance.
function of wt. [AI 2020] (ii) Draw graphs showing variation of voltage and
V. Long Answer Type Questions (5 Marks) current with time over one cycle of ac, for X.
45. (i) An alternating voltage V = Vm sin wt applied to (iii) How does the reactance of the device X vary
a series LCR circuit drives a current given by with frequency of the ac? Show this variation
graphically.
i = im sin (wt + f). Deduce an expression for the
average power dissipated over a cycle. (iv) Draw the phasor diagram for the device X.
[Delhi 2018]
(ii) For circuits used for transporting electric power,
49. (i) Show that an ideal inductor does not dissipate
a low power factor implies large power loss in
power in an ac circuit.
transmission. Explain.
(ii) The variation of inductive reactance (XL) of an
(iii) Determine the current and quality factor at inductor with the frequency (f ) of the ac source
resonance for a series LCR circuit with L = 1.00 of 100 V and variable frequency is shown in the
mH, C = 1.0 nF and R = 100 W connected to an fig. [Delhi 2020]
a.c. source having peak voltage of 100V.
(a) Calculate the self-inductor of the inductor.
[Foreign 2011] (b) When this inductor is used in series with a
46. (i) Show that the average power consumed in an capacitor of unknown value and a resistor of
inductor L connected to an a.c. source is zero. 10 W at 300 s–1, maximum power dissipation
(ii) In a series LR circuit XL = R and the power factor of occurs in the circuit. Calculate the capacitance
the circuit is P1. When a capacitor with capacitance of the capacitor.

244 Physics–12
D:\EG_Physics-12_(26-06-2022)\Open_Files\Ch-7\Ch-7
\ 27-Jul-2022 Ved_Goswami Proof-4 Reader’s Sign _______________________ Date __________

(XL in W) 50. (a) Derive the expression for the current flowing in an
ideal capacitor and its reactance when connected
60
to an ac source of voltage V = V0 sin ωt. 
[CBSE SP 2020-21]
40

(b) Draw its phasor diagram.
20 (c) If resistance is added in series to capacitor what
changes will occur in the current flowing in the
circuit and phase angle between voltage and
0 100 200 300 current.
f (in Hz)

Answers 7.2
I. Objective Type Questions
1. (i) (c) (ii) (b) (iii) (c) (iv) (a)
(v) (d) (vi) (d) (vii) (c) (viii) (a)
(ix) (b) (x) (b)
2. (i) 10 W (ii) 0 (zero)
3. (i) True (ii) False
4. (i)-(b) (ii)-(c) (iii)-(d) (iv)-(a)
5. (a) 1
4. Power factor = cos f = 0.5 = or f = 60°
2
II. Very Short Answer Type Questions
There is a phase difference of 60°
1
 
2 2 5. The current in ac. circuit is said to be wattless if the
1. As Z =  R 2 +  ωL − 1   average power consumed in the circuit is zero.
  ωC  
1
 
2 2 E
=  R 2 +  2πvL − 1   and IV = V
  Z
 2πvC 
The glow of the bulb will remain same when IV remains
same.
So when v is doubled, L should be halved and C should
be halved. OC = Irms sin f is wattless current.
1
1
1 6. Quality factor: It is defined as the ratio of the resonant
 2 V
2. Z =  R + 
2 2 2
XC  =  R 2 + 2 2  and Irms = rms frequency to the difference in two frequencies at which
 ω C  Z 1
the current amplitude becomes times the value at
When a dielectric slab is introduced, the capacitance 2
the resonant frequency
‘C’ will increase. Thus Z will decrease. Decrease in Z
will give rise to increase in current. So the glow of the ωr ω L
Mathematically Q = = r
bulb will increase. Bandwidth R
1 Or
3. As (
Z = R + (XL − XC )
2
)
2 2
It may also be defined as the ratio of the voltage drop
across the inductor (or capacitor) at resonance to the
1
applied voltage.
 
2 2
=  R 2 +  2πvL − 1   It has no unit. It is a pure number.
 
 2πvC 
7. As VL = VC So XL = XC
At v = vr (resonance frequency) 1

Z = R (minimum). Z =  R 2 + ( X L − X C )2  2 = R
 

Alternating Current 245


D:\EG_Physics-12_(26-06-2022)\Open_Files\Ch-7\Ch-7
\ 27-Jul-2022 Ved_Goswami Proof-4 Reader’s Sign _______________________ Date __________

R Hz, C = ?, Z = ?, I = ?, P = ?

\ =1
cos f = When current and voltage are in phase then
Z
So power factor is unity. 1
XC = X L or ω 2 =
8. Current through the bulb LC
V
Irms = rms 1 1 1
Z So C= = 2 2 = 2 π2
ω L 4π v L 4π (50) 2 × 4
2
1
Z =  R 2 + ω 2 L2  2 1
  = = 25 ×10 −6 = 25µF
When the no. of turns n in the inductor decreases, L will 4 × 25 ×102 × 4
decrease as L ∝ n2 so Z will decrease, which will lead At resonance Z = R = 100W
to increase in the current flowing through the bulb, so Vrms 200
the brightness of the bulb will increase. Irms = = = 2A
Z 100
9. Bandwidth is the difference of two frequencies when At resonance, power dissipated is maximum
1 Pav = VrmsIrms cos 0° = 200 × 2 = 400W
I= I0max
2
From figure we see that
(ii) (a) For v = 100 Hz, XC = 6W
Dw = 1.2 – 0.8 = 0.4 rad/s 1 1
XC = ⇒C=
1.5 2πvC 2πv X C
1

Imax.(A)
C= = 2.65 × 10 − 4 F
1.0 2 × 3.14 × 100 × 6
0.707
(b) v = 100 Hz, XL = 6W
XL 6
0.5 XL = ω L ⇒ L =
=
ω 2 × 3.14 × 100
L = 9.459 × 10–3 H
0 (c) As XL = 2pvL i.e., XL ∝ v so
0.5 0.8 1.0 1.2 1.5 2.0
(rad/s) 
v 100 200 300 400
10. In case of a capacitor current is ahead of voltage, so Y XL 6 12 18 24
must be a capacitor
π
f = , R = 100 Ω 24
4 22
20
R p 100 18
cos f = ⇒ cos =
Reactance (W)

16
Z 4 Z 14
1 100
12
= or Z = 100 2 = 141.4Ω 10
2 Z 8
6
V 141 ~ 4
Irms = rms = − 1A 2
Z 141.4 0
100 200 300 400 fre. (Hz)
1

Z =  R 2 + ( X ′L − X C′ ) 
2 2
(d) As
Z
 
XC X L X ′L
As = L=
φ
ω ω′
ω′ v′ 300
R So X ′L = XL = XL = × 6 = 18Ω
ω v 100
4 1 1
11. (i) R = 100Ω, L = H, Vrms = 200V , v = 50
C= =
π2 X C ω X C′ ω ′

246 Physics–12
D:\EG_Physics-12_(26-06-2022)\Open_Files\Ch-7\Ch-7
\ 27-Jul-2022 Ved_Goswami Proof-4 Reader’s Sign _______________________ Date __________

X C ω 6 × 100 So impedance will decrease. Thus the current I will


So X C′ = = = 2Ω increase. Thus the bulb will grow brighter.
ω′ 300
1 (ii) When a capacitor of reactance XC = XL is introduced
So Z = 10 + (18 −2)  2
2 2 then
1

Z =  R 2 + ( X L − X C )  2 = R. i.e.
1 2

= [100 + 256] 2 = 18.87Ω  
Z will decrease so current in the circuit will increase,
1
12. (i) When ω > , XL = wL will be large so the brightness of the bulb will increase.
LC
16. VL = 30V, VC = 30V, VR = 60V
1
and X C = will be small. So XL > XC. Phase difference (f) in series LCR circuit is given by
ωC
Thus the circuit will be inductive. In inductive X − X L VC − VL 30 − 30
tan f = C = = =0
circuit voltage leads current. i.e., current lags behind R VR 60
voltage. f = 0 (zero)
1 17. (i) X is an inductor, because in series R-L circuit voltage
So graph (a) corresponds to ω >
LC leads current.
(ii) (a) Capacitor stores energy in the form of electric Y is a capacitor, because in series R-C circuit voltage
field. The inductor stores energy in the form of lags behind current.
magnetic field. (ii) When R & X are connected in series,
(b) The oscillation becomes damped because (i) of π
φ=
the resistance of the connecting wires and the 4
material of the coil (ii) some energy is radiated XL
As tan φ =
in the form of e.m. waves, so the oscillations R
become damped. π XL
tan =
13. When we walk through a metal detector, we are walking 4 R
through a coil of many turns connected to a capacitor. XL
1=
If we carry some metal on our body, impedance of the R
circuit changes. \ XL = R
This change in impedance changes the current When R & Y are connected in series
significantly, as the circuit is tuned suitably. This change π XC
tan =
in current is detected and the alarm rings. 4 R
14. When an iron rod is inserted into the interior of the XC
1=
inductor, it self inductance L will increase. So XL = wL R
will also increase. So R = XC
V when both X, Y are connected in series with R.
As Irms = rms
XL Z2 = R2 + (XL –XC)2
So Irms will decrease. as XC = XL = R, So
So the brightness of the bulb will decrease. Z = R,
If ac source is replaced by a dc source of the same As circuit is purely resistive so power consumed is
voltage, the reactance of the bulb will be zero. Thus maximum.
the bulb will glow more i.e., the brightness of the bulb 18. The power factor (cos f) is the ratio of resistance and
will increase on introducing an iron rod, there will be impedance of an ac circuit i.e.,
no change in the brightness of the bulb. R
15. (i) When number of turns in the inductor is reduced, Power actor, cos f =
Z
the self inductance of the coil will decrease. (i) Maximum power factor is 1 when Z = R i.e., when
As XL = wL so XL will also decrease. circuit is purely resistive.
1
(ii) Minimum power factor is 0. This happens when
The impedance Z =  R + X L  2
2 2
circuit is purely inductive or capacitive.

Alternating Current 247


D:\EG_Physics-12_(26-06-2022)\Open_Files\Ch-7\Ch-7
\ 27-Jul-2022 Ved_Goswami Proof-4 Reader’s Sign _______________________ Date __________

19. (i) When capacitance is reduced, capacitive reactance  4


1   f = tan −1   = 53°  ½
XC = increase, hence impedance of circuit  3
ωC

(ii) (a) As XL = wL
V
Z = R 2 + X C2 increases, so the current I = XL
Z XL 1
L= ==
decrease. As a result the brightness of the bulb is ω 2πv 2π
reduced. slope of XL Vs v graph
B 1 ( 6 − 2) 1 4
= × = ×
C 2π (300 − 100) 2π 200
= 3.18 × 10–3 H ½
(b) v = 300 Hz, XL = 6W (from the graph), R = 8W
~ 1
Z = Impedance =  R 2 + X 2L  2
Source

(ii) When frequency is decreased, capacitive reactance
1
1
XC = increases and hence impedance of = 8 + 6  2 = 10Ω 
2 2
½
2πf C
circuit increases, so current decreases. As a result 23. Given V = 100 V, v = 50 Hz, R = 20 W,
brightness of bulb will be reduced. L = 2 mH = 2 × 10–3 H
1 1 (i) Impedance of R-L circuits
20. Xc = =
ωC 2πnC 2 2
= Z = R + ( ωL )
1
then Z= R 2 + X C2 = R 2 + ½
4π n C2 2 2
= R 2 + (2πvL) 2 
OR
= (20) 2 + (2 × 3.14 × 50 × 2 × 10 −3 ) 2
R 2 + (X L − X C )
2
Z=
= 400 + (0.628) 2 ≈ 20Ω  ½
R 2 2
21. Power factor cos f = , when Z = R + X
Z Vrms 100
(i) X = XC = aR, (ii) rms current = =
I rms = = 5A  1
Z 20

\ Z= R 2 + ( aR ) 2 = R 1 + a 2 24. (i) Potential difference across capacitance = VC = IXC
\ Capacitive reactance
R 1 V 40

\ cos f = =  1 XC = C = = 20Ω
2
R 1+ a 1 + a2 I 2
(ii) X = XL = bR VR 30
Resistance = R = = = 15Ω  1

\ Z= I 2
R 2 + (bR ) 2 = R 1 + b 2
R 1 Impedance = Z = R 2 + X C2 = (15) 2 + (20) 2

\ cos f = =  1
R 1 + b2 1 + b2    = 225 + 400 = 625 = 25Ω  1
22. (i) Voltage applied (ii) The phase lead (f) of current over applied voltage
V= =
2
(200) + (150) 2 is
VL2 + VR2 XC
tan f =
= 400 × 102 + 225 × 102 = 250 V R
Wattless current = Iwattless = I sin f
V 250
Impedance of circuit = =
Z = = 50 W ½ X 
I 5 = I. C 
Phase angle between voltage and current  Z 
X L VL 200 4 20
tan f = = = = = 2× A = 1.6A  ½
R VR 150 3 25

248 Physics–12
D:\EG_Physics-12_(26-06-2022)\Open_Files\Ch-7\Ch-7
\ 27-Jul-2022 Ved_Goswami Proof-4 Reader’s Sign _______________________ Date __________

1 1
wr = =  ½
L1C1 L2 C2
       ½
Z

XC So L1C1 = L2 C2
φ
For fine tuning L1, C1 and R1 is better suited because
R Q-factor is high and the resonance will be sharp. ½
1 L 1 2 28. Q-factor measures sharpness of resonance and it
25. We have, Q = = = 100  1 is defined as the ratio of resonant frequency to the
R C 10 2 × 10 −6
difference in two frequencies taken on both sides of
It signifies the sharpness of resonance, ½ resonant frequency such that at each frequencies the
Higher is the value of Q-factor, sharper is the resonance. 1
current amplitude becomes times the values at
 ½ 2
26. (i) In case of a resistor the current (I) and potential (V) resonance frequency.
are in phase. ωr ω
p Q-factor = = r
In case of an inductor, V leads I by ω 2 − ω1 2 ∆ω
2
p
So VL leads VR by  ½
2
p I0
In case of a capacitor V lags behind I by  ½ max
2 Band width
p = 2 = 2 – 1
So VC lags behind VR by . Thus VL and VC
2 I0
have a phase difference of p so the phasor is 2 Band Width
OB = VL – VC
I

VR
VL
      ½ 1 t 2
O
OR
VC Capacitive reactance: It is the opposition offered to
current by a capacitor
1
1
So Veff =  VR2 + (VL − VC ) 2  2 XC =
  ωC
As VL = 250 V, VC = 250V and they are out of phase
so Veff = 200V ½
(ii) As R = 40W, VL = VC given
So XL = XC XC
1
Z =  R + (X L − X C )  2 = R
2 2

Veff 200 v
So I= = = 5A  1 Frequency
R 40
27.     
R1 Expression for current when capacitor is connected
across A.C. Source: Consider a capacitor of capacitance
I R2 > R1 C connected across an AC source
Let V be the potential difference between plates of
R2 ½ capacitor at any point of time.
q
V= as V = V0 sin ωt
Frequency C
r

Alternating Current 249


D:\EG_Physics-12_(26-06-2022)\Open_Files\Ch-7\Ch-7
\ 27-Jul-2022 Ved_Goswami Proof-4 Reader’s Sign _______________________ Date __________

q 1
V0 sin wt = or q = CV0 sin wt (ii) (a) As wr =
C LC
dφ As wr is same for two circuits
I= = V0 C (ω cos ωt )
dt So L1C1 = L2 C2
C
i.e.,
L1C1 = L2C2 ½
(b) (1) At Resonance Z = R
R
Power factor = cos φ = = 1  ½
Z
~ Power factor is same for both the circuit and is equal
V = V0 sin ωt to 1.
π (2) Circuit I has higher Q-factor, as its resonance peak
I = V0 ωC sin  ωt +  is sharper. ½
 2
30. Impedance of circuit A
π ZA = R 2 + X 2L = R 2 + 9R 2 = 10R  ½
∴Current is leading potential by
2
Impedance of circuit B
Max value of 1
ZB =  R + (X L − X C )  2 
2 2
V0 V0 1
I = I0 = V0 wC = 1 =
XC
1
ωC
ZB =  R 2 + (3R − R ) 2  2
1
= XC [Reactance of capacitor]
ωC 1
=  R 2 + 4R 2  2 = 5R  ½
XC = Capacitive reactance
 π R
I = I0 sin  ωt +  Power factor = cos φ =
 2 Z
(cos φ) A R ZB ZB 5R 1
= × == =  ½
(cos φ) B ZA R ZA 10R 2
I0
V0 31. Given R = 200W, C = 15.0 µF = 15.0 × 10–6 F,
V = 220V, f = 50 Hz
90°
Capacitance reactance
ωt
1 1
XC = =
29. (i) (a) For resonance XL = XC ½ ωC 2πfC
As L is same 1
Resonant frequency is same for all of them = = 212Ω  ½
2 × 314 × 50 × 15.0 × 10 −6
1 Impedance of circuit,
wr =
LC = ½
(200)2 + (212) 2 = 291.5 W
So capacitor used should also be same. ½
(b) The maximum value of current at resonance is RMS current,
Vrms 220
V0 Irms = = A = 0.75 A ½
(I0)max = Z 291.5
R
Voltage across resistance,
(I0) max for student Z is least so resistance used by
student Z is highest. Resistance used by student X VR = RI = 200 × 0.75 = 150V ½
is lowest. ½ Voltage across capacitor,
At w = w0, impedance is purely resistive. VC = XCI = 212 × 0.75 = 159 V ½

250 Physics–12
D:\EG_Physics-12_(26-06-2022)\Open_Files\Ch-7\Ch-7
\ 27-Jul-2022 Ved_Goswami Proof-4 Reader’s Sign _______________________ Date __________

Algebraic sum of VR and VC = VR + VC 34. (i) We know


= 150 + 159 = 309 V > 212V 1 1
wr = = = 50 rad/s  1
This is because these voltage are not in same phase but LC 5 × 80 × 10 −6
p (ii) Current at resonance
VC lags behind VR by an angle
2 Vrms 240
Irms = = = 6A  1
R 40

\ V= VR2 + VC2 = (150) 2 + (159) 2 (iii) Vrms across capacitor
≈ 220 V ½ Vrms = IrmsXC
32. Given L = 200 mH = 200 × 10–3 H,
1 1
    C = 500 µF = 500 × 10–6 F, R = 10W, = I rms ωC = 6 × 50 × 80 × 10 −6
   Vrms = 100V ½
(i) Angular (resonant) frequency w, at which power 6 × 106
= = 1500 V  1
factor of the circuit is unity, is resonance frequency 4 × 103
1 1 35. Consider a resistor, an inductor and a capacitor are
wr = =
LC 200 ×10 −3
× 500 × 10 −6 connected in series to an a.c. source.
In case of resistor, potential and current are in phase.
= 100 rad s–1  ½ p
In case of an inductor, potential leads current by
Resonant frequency vr 2
ωr 100 100 p
vr = = = = 15.9 Hz  ½ In case of a capacitor, potential lags behind by
2π 2 × 3.14 6.28 2
Let VR = Peak Potential difference across resistance
(ii) At resonant frequency vr impedance, 0
VR = I0R
Z= R 0
VL = Peak Potential difference across inductance

\ Current amplitude, 0
C L
R
V0 V 2 100 2

/
I0 = = = = 10 2A  ½ I
Z R 10 V I V
= 14.1 A ½ V
I

ω r L 100 × 200 × 10 −3
(iii) Q-factor =
R
=
10
= 2 ½ ~
VL = I0XL
33. For power factor unity, XL = XC 0
VC = Peak potential difference across capacitor
1 0
i.e.,
wL =    VC = I0XC
ωC 0
If VL > VC
1 1 1 0 0

⇒ L= 2
= 2
= 2 2  ½ OD = VL – VC
ω C (2πv) C 4π v C 0 0
OE2 = OA2 + EA2 ⇒ OE2 = OA2 + OD2
Given n = 50 Hz, C = 100 µF = 100 × 10–6 F
( )
2
V02 = VR2 0 + VL0 − VC0
= 10–4 F
1 = I0  R + (X L − X C )  
2 2 2
1
L= = 0.10 H  ½
4 × (3.14) × (50) 2 × 10 −4
2
1

V0 V0 = I0  R 2 + (X L − X C ) 2  2
 
Current amplitude, I0 =  ½
Z 1
At resonance Z = R Impedance = Z =  R 2 + (X L − X C )2  2
 
V 200 2 Potential leads current by an angle f
So = 0
I0= = 20 2A  ½
R 10 AE VL − VC
tan f = =
= 20 × 1.414 A = 28.28 A 1 OA VR

Alternating Current 251


D:\EG_Physics-12_(26-06-2022)\Open_Files\Ch-7\Ch-7
\ 27-Jul-2022 Ved_Goswami Proof-4 Reader’s Sign _______________________ Date __________

−1  VL − VC   X − XC  100
f = tan  = tan −1  L  = = 7.96 Hz  1
 VR    R 12.56
 ½ (ii) Current at the resonating frequency: At resonance,
V0 Z=R
Current I0 =
Z V 240
So Iv == = 4A  1
E R 60
B
(iii) Rms potential drop across inductor at resonance
Iv × 1 × L
VL0 V0 I0 (Vrms)L = IvXL = IvwrL =
LC
D A
φ 4 ×1× 4 16 × 100
VR0
ωt = = = 800V  1
O
   ½ 4 × 100 ×10 −6 2
37. (i) condition for resonance: For resonance to occurs
the current and potential difference should be in
VCO phase.
C X L − XC
So tan f = =0
Variation of current with frequency of source R
So
XL = X C
Case I: If frequency is low i.e., w → 0
If wr is the resonant frequency then
1 1
vr  wr L =
2 LC ωr C
1 1
Peak current

wr = or vr =  1
LC 2p LC
    ½
(ii) E0
I0 
R
E0
I0 
2R
Bandwidth
= 2
1
 
v = vr Frequency

1 I0
  1  2
2
Z =  R 2 +  ωL −  
  ωC  
Z → ∞ , So I0 → 0
1 r  2 
Quality factor (Q): It gives the sharpness of
Case II: If frequency is very high i.e, w → ∞ resonance. The quality factor is defined as the
Z → ∞ , So I0 → 0 ratio of resonant frequency to the width of half
Case III: For frequency w when XL = XC power frequencies.
1 1 ωr R ω L
wL = ⇒ ω2 = Q= , 2Dw = so Q = r 1
ωC LC 2∆ω L R
2Dw is called Bandwidth.
1 1
So ,v= w= ½ 38. (i) X is a pure resistor, (ii) Y is a pure inductor
LC 2 p LC (iii) Z is a pure capacitor1½
At this frequency Z is minimum and I0 is maximum. When L, C and R are connected in series with an
36. (i) Resonant frequency a.c. source, then impedance
1 1
vr =
Z =  R + (X L − X C )  2
2 2
2p LC

1 1 × 102 V V
= = I= = 1 
6.28 × 2 Z
2 × 3.14 4 × 100 × 10 −6 ½  2

2 2
 R + (X L − X C ) 

252 Physics–12
D:\EG_Physics-12_(26-06-2022)\Open_Files\Ch-7\Ch-7
\ 27-Jul-2022 Ved_Goswami Proof-4 Reader’s Sign _______________________ Date __________

Frequency condition for the current amplitude to be Algebraic sum of voltage across R and C is more
maximum in w than the source voltage.
= Resonant frequency = wr This is so because potential and current across a
1 1 capacitor are not in phase. VR and VC have a phase
wr = or vr = ½ p
LC 2π LC difference of
Maximum current amplitude 2
V V= VR2 + VC2  ½
Iomax = 0  ½
R 41. (i) In a series LCR circuit
39. (i) L = 80 mH = 80 × 10–3 H X − XC X − XL
C = 250 µF = 250 × 10–6 F tan f = L or C  ½
R R
Vrms = 240V, w = 100 rad/s L = 100 mH = 100 × 10–3H = 10–1 H
240 C = 2µF = 2 × 10–6 F
IrmsXC – Irms XL = 240, I rms =
XC − XL w = 1000 rad/s
XL = wL = 100 × 80 × 10–3 = 8W 1 XL = wL = 1000 × 10–1 = 100 W
1 1 1 1
XC = ωC = XC = = = 500Ω  ½
100 × 250 × 10 −6 ωC 1000 × 2 ×10 −6
1 1000 X C − X L 500 − 100
= = = 40Ω  ½ tan f = = =1
25 × 10 −3
25 R 400
240 240 p
= = 7.5A  f = 45­° or  ½
Irms = ½ 4
40 − 8 32
p
Current leads potential by
L C 4
VL
(ii) For power factor to be unity, R = Z i.e. XL = XC ½
Let the new capacitance be C′
Irms
1 1
VC = 100 or C′ =
ωC ′ 1000 × 100
−5
(ii) Average power consumed by the circuit is zero. 1 = 10 F = 10µF  ½
40. R = 100 W We need to increase the capacitance from 2 µF to
100 100 × 10 −6 F 10µF. So we should connect a capacitor of 8µF in
C= µF = parallel with 2µF capacitor. ½
π π
Vrms = 220V, v = 50 Hz 42. I0 = V0/R = 10/10 = 1 A ½
–6
wr = 1/√LC = 1/√ (1 × 1 × 10 )
1 1
(i) XC = =
2πvC 2 × π × 50 × 100 × 10 −6 = 103 rad/s ½
π V0 = I0 XL = I0 wr L ½
3 3
= 1 × 10 × 1 = 10 V ½+½
= 100 W ½
1 1 3
Q = wr L/R = (10 × 1)/10 = 100 ½
Z =  R + X C2  2
2
= [1002 + 1002 ] 2 [CBSE Marking Scheme, 2018-19]

= 100 2   ½  resonant frequency for LCR circuit is given by


Vrms 220 1 1
v0 = =
Irms == = 1.1 2A = 1.56A  ½ 2p LC 2 × 3.14 3 × 27 × 10 −6
Z 100 2
(ii) VR = RMS potential across resistor = 17.69Hz 1
= IrmsR = 1.56 × 100 = 156V ½ Or   w0 = 2pv0 = 111 rad/s. ½
VC = RMS potential across capacitor  quality factor of resonance
= 1.56 × 100 = 156V Q= 1 L
 ½
VR + VC = 156 + 156 = 312V ½ R C

Alternating Current 253


D:\EG_Physics-12_(26-06-2022)\Open_Files\Ch-7\Ch-7
\ 27-Jul-2022 Ved_Goswami Proof-4 Reader’s Sign _______________________ Date __________

1 3 45. (i) Consider an a.c. source connected across a series



∴ Q= = 45.0 1 LCR circuit.
7.4 27 × 10 −6 As V = Vm sin wt, I = Im sin (wt + f)
To improve sharpness of resonance circuit by a factor The instantaneous power
2, without reducing w0; reduce R to half of its value i.e.
  P = VI = Vm sin wt Im sin (wt + f)
R = 3.7Ω
dW
43. (i) Peak voltage across = VmIm sin wt (sin wt cos f + cos wt sin f)
dt
(a) Resistance R is given by
dW = VmIm sin2wt cos f dt
V0 R
VR = I0R = + VmIm sin wt cos wt sin dt
R 2 + X C2 T
 1 − cos 2ωt 
(b) Capacitor C is given by W= ∫ dW = Vm Im cos φ ∫  2  dt
V0 X C 0
VC = I0XC = T
R 2 + X C2 V I sin φ
+ m m
2 ∫ sin 2 ωt dt 1
−1  VC  −1  X C 
0
(ii) f = tan  = tan 

 VR   R  T T

= Vm I m cos φ  ∫ dt − ∫ cos 2 ωt dt 
= Phase Difference between V and I 2  0 
0
Current (I) is ahead of applied voltage (V).
T
44. (i) Z=  − cos ωt  Vm I m sin φ
R 2 + X c2 = impedance +
 2ω  0
2
V V sin ωt
\ I= = 0 Vm I m cos φ
Z R 2 + X c2 = ×T−0+0 1
vR 2
vC
W Vm I m
R C Pav = = × cos φ = VrmsIrms cos f 1
I T 2 2
(ii) As Pav = VrmsIrms cos f ½
V= v0 sin t If power factor i.e., cos f is small, then current will
VR = IR, VC = I X C be considerably high at a given voltage.
As power loss = I2R will be high. ½
in Phase of I,   V lags I by p
2 (iii) V = 100V, L = 1 mH = 10–3H,
C = 1.0 nF = 10–9 F, R = 100 W
At resonance, Z = R
vR
I V V 100
Q So I= = = = 1A  ½
Z R 100
vc v 1 L 1 10 −3 1000
V= = Q-factor = = = = 10  ½
VR + VC2 (IR ) 2 + (IX C ) 2 R C 100 10 −9 100
V V0 sin ωt 46. (i) consider an inductor of self inductance L connected

⇒ I= = across an a.c. source
R 2 + X C2 R 2 + X C2 L
f = phase difference between V & I
V X
tan f = C = C    ½
VR R
(ii) V
~
V
I
or 0 sin ωt
I
Instantaneous emf e = e0 sin wt
0 t
p
In case of an inductor, potential leads current by
2
½

254 Physics–12
D:\EG_Physics-12_(26-06-2022)\Open_Files\Ch-7\Ch-7
\ 27-Jul-2022 Ved_Goswami Proof-4 Reader’s Sign _______________________ Date __________

 π (b) When XL = XC
So I = I0 sin  ωt −  = − I0 cos ωt
 2 VL − VC
Instantaneous power tan f = = 0 ½
VR
dW
P= = εI = ε 0 sin ωt ( − I0 cos ωt )
dt
dW = – e0I0 sin wt cos wt dt 1
− ε 0 I0
T       ½
W= ∫ dW = 2 ∫0
sin 2ωt dt

T
− ε 0 I0  − cos 2ωt 
=   =0
2  2ω  0 I
0
VR A
W
= 0
Pav = 1
T (iii) Acceptor circuit: It is a series LCR circuit. It is
(ii) In series LR circuit: used in tuning of radio sets.  1
XL 48. (i) V = V0 sin wt
XL = R , tan φ = =1
R π 
I = I0 sin  ωt +  ½
1  2
Z =  R 2 + X 2L  2 = 2R  ½
As current I leads the voltage V by a phase angle of
R R 1 ≠ , so the circuit is capacitive.
Power factor = cos φ = = = = P1 ½ ½
Z 2R 2 2
When XC = XL, put in series with the given L and
(ii) 
R then Z = R
R
P2 = Power factor = cos φ = = 1 1
R
P1 1 1
So = =  1
P2 2 ×1 2
47. (i) Curve B is for inductive reactance
Curve C for resistance. 1
(ii) Impedance of a series LCR circuit is 1
(iii) Xc =
Z2 = R 2 + ( X L − X C ) 2 ωC
At resonance, XL = XC 1
So Z= R Xc =  ½
2pf C
So the circuit behave as a purely resistive circuit at
resonance. 1 1
Phase relationship between current and voltage in Xc ∝
f
series LCR circuit.
(a) When XL > XC, The circuit is mainly inductive, so
potential will, lag behind current. ½

    ½
B

VL – VC ½
V
(iv) Phasor diagram showing the variation of current
and voltage for a pure capacitor (current leads the
φ
I
voltage by phase angle of 90°). 1
0 VR A

Alternating Current 255


D:\EG_Physics-12_(26-06-2022)\Open_Files\Ch-7\Ch-7
\ 27-Jul-2022 Ved_Goswami Proof-4 Reader’s Sign _______________________ Date __________

V  π
Maximum current, I0 = 0 × 1 when sin  ωt + 
1  2
   1 =1
ωC
 π
\ I = I0 sin  ωt + 
 2
V
Comparing with ohm's law: I = to equation
49. (i) Power = VI cos f R
For pure inductive circuit, the phase difference V0
p p I0=
between current and voltage is , (i.e. f = ) 1
2 2 ωC
p 1
\ Power = V.I. cos = V.I. × 0 = 0 We have, capacitive reactance, Xc=
2 ωC
Therefore, zero power is dissipated. This current is (b) Phasor Diagram

referred to as wattless current.
(ii) (a) We know XL = wL and w = 2pf, where f is V0
frequency (in Hz). V = V0 sin t

X 20
\ L= L = I0
2πf 2π (100)
90º
t
0
40 60
= =  [Using graph] I = I0 sin (t + )


2π ( 200) 2π (300)
2

= 31.84 × 10–3 H ≈ 32 mH (c)


VR VC
(b) Given f = 300 s–1 we know that power dissipation is R C
maximum when XL = XC (i.e., at resonance)
I
1 1 I 90°

⇒ wL = ⇒C= 2 VR VC
ωC ωL 
1
⇒ C= VR
4p f 2 L
2

1
=
4 × 3.14 × 3.14 × 300 × 300 × 32 ×10−3 

= 8.8 µF
50. (a) We have: V = V0 sin wt VC

q
Also, V= ; q = charge on capacitor
C
q
V0 sin wt =   or, q = CV0 sin wt
C Here, effective impedance, Z = R 2 + X C2 As
\ I == dq d (CV sin wt) Z > R, current will be reduced. In RC circuit, voltage
I = 0
dt dt lags behind the current by a phase angle f, where
= CV0 cos wt. w Vc − Io X c X
tan φ = = = − c negative sign is for
VR Io R R
V0  π
\ I = sin  ωt +  lagging of alternating voltage behind alternating
1  2
current.
ωC

256 Physics–12
D:\EG_Physics-12_(26-06-2022)\Open_Files\Ch-7\Ch-7
\ 27-Jul-2022 Ved_Goswami Proof-4 Reader’s Sign _______________________ Date __________

Case Based Question


I. In resonance, amplitude of a mechanical oscillation of frequency w0 for which Z will have a minimum
becomes maximum when the frequency of driving value so that it is also then possible to determine the
force becomes equals to the natural frequency of maximum value amplitude (peak current).
system. ʻResonanceʼ is also possible in some specific
electric circuits series LCR circuit is considered
important because it can exhibit ʻresonanceʼ which
makes it very useful in designing tuning circuits.
We know that peak current (I0) in a series LCR ac
circuit is related to peak volt across the circuit by the
relation

Consider a series LCR circuit connected to an AC


source of constant peak voltage V0 and variable
frequency w0. Value of L, C and R are given as L =
5 50
H, C = π mF and R = 400 W for value of w = w1.
π
For another value of w = w2, rms voltage across the
resistance is 50 volts.
1. Voltage (rms) across the resistor (VR) is
V (a) 80 V (b) 100 V (c) 90 V (d) 110 V
I0 = Z0
2. Voltage (rms) across the capacitor (VC) is
where Z= R 2 + (X L − XC) 2
(a) 12 V (b) 20 V (c) 15 V (d) 40 V
R 2 + bωL − ωC l
2
1 3. Voltage (rms) across the inductor (VL) is
=
is the impedance of the circuit (a) 50 V (b) 25 V (c) 100 V (d) 12.5 V
It is clear that impedance depends on frequency, Z 4. Frequency w1 is nearly equal to
= Z(w). Hence, for a given V0, peak current (I0) will (a) 324.2 rad/s (b) 312 rad/s
be different for different frequencies. For a certain (c) 225 rad/s (d) 625 rad/s
frequency w0, impedance Z become minimum so
5. Frequency w2 is nearly equal to
that peak current (amplitude) I0 will have a maximum
value compared to other frequencies. This is called (a) 50 rad/s (b) 36 rad/s
ʻResonanceʼ in LCR series circuit and w0 in the (c) 73 rad/s (d) 18 rad/s
resonant frequency. We can easily find the value Ans. 1. (a) 2. (d) 3. (c) 4. (a) 5. (c)

COMMON ERRORS
S No. Errors Corrections
1. Confusion in Instantaneous Peak and rms. If not specified, the current or voltage marked (or voltage with
value of AC current/voltage. frequency should be considered as r.m.s.
2. AC through pure inductor and capacitor Voltages across different elements should be added vectorially
phase difference between voltage and current only.
3. Numerical problems based on LCR series Conversion of units into SI and proper calculation should be
circuit (condition of resonance), power factor done.

Alternating Current 257


D:\EG_Physics-12_(26-06-2022)\Open_Files\Ch-7\Ch-7
\ 27-Jul-2022 Ved_Goswami Proof-4 Reader’s Sign _______________________ Date __________

IMPORTANT FORMULAE
Formula Symbols Application
1. 2 I = average current To fined current in the circuit
Iav = I = 0.637 I0 I0 = Peak current
π 0

2. I0 Iv = virtual current To find peak or max. value of


Ieff or Irmsor Iv = = 0.707 I0 Irms = Root mean square current current
2
Ieff = Effective current

3. LR circuit Z = Impedance (ohm) To draw Phasor diagram


V 2 2 To determine power factor
(i) Z = rms = R + X L
I rms
Where XL = wL
w = Angular frequency (rad/s)
V f = Phase angle
(ii) Irms = rms
Z L = Inductance (Henry)
X L ωL cos f = Power factor
(iii) tan f = =
R R
R
(iv) cos f =
Z
4. CR circuit C = capacitance (Farad) To determine Instantaneous
V current
(i) Z = rms = R 2
+ XC2
I rms
1
Where XC =
ωC
X 1/ωC
(ii) tan f = C = I = Instantaneous current
R R
(iii) I = I0 sin (wt + f)

5. LCR circuit To find impedance and phase


Vrms 2 difference between V & I.
(i) Z = I = R 2 + ( X L − XC )
rms
X –X
(ii) tan f = L C
R
R
(iii) cos f =
Z
Z
XL–XC
f
R

6. 1 Tuning of a radio receiver


Resonant frequency, fr =
2π LC

7. Q = Quality Factor Sharpness of resonance


ωL 1 L
Q – factor = r =
R R C

258 Physics–12
D:\EG_Physics-12_(26-06-2022)\Open_Files\Ch-7\Ch-7
\ 27-Jul-2022 Ved_Goswami Proof-4 Reader’s Sign _______________________ Date __________

REVISION CHART

Average Value of Alternating Voltage


Average or Mean Value of a.c. Average or Mean Value of a.c. Over
Over Half a Cycle
Over a Cycle: Half a Cycle
It is that value of steady voltage which
It is that value of steady current which It is that value of steady current which
sends the same charge through a resistor
sends the same charge through a sends the same charge through a resistor
in the same time as is sent by the given
resistor in the same time as is sent by as is sent by the given a.c. in the same
alternating voltage through the same
the given a.c. through the same resistor resistor in its half time period.
resistor in its half cycle.
in its one cycle. Average of a.c. over
2V0 a cycle = 0. Average of alternating 2I
Vav over half a cycle = = 0.637 V0 Iav over half a cycle = 0 = 0.637 I0
π voltage over a cycle is zero. π

Peak, average and rms Value of AC/Alternative Voltage, Resistance and reactanace

A.C. Source connected across a pure A.C. Source Connected Across a Root Mean Square (rms) Value of
resistor: Pure Inductor: Current lags behind A.C.  
As V = V0 sin wt, I = I0 sin wt p It is that value of steady current which
potential by
The voltage is in phase with the current 2 produces the same amount of heat in
V If V = V0 sin wt, a given resistor in the same time as
V and I are in phase I rms = rms
R p is produced by the A.C. in the same
   I = I0 sin (wt – )
2 resistor in one time period.
V2
Power consumed = Iv2 R = rms Vrms It is also called effective value or virtual
R Irms = value of a.c.
= IrmsVrms XL
Inductive reactance XL = wL Ieff = Iv = Irms = I0 / 2
SI unit of X L is ohm. Power Veff = Vv = Vrms = V0 / 2
consumed = zero
•   Phasor: A rotating vector that
A.C. Source Connected Across a represents a sinusoidally varying
Capacitor: Current leads the voltage quantity is called a phasor.
p • P hasor Diagram: A diagram that
by . If V = V0 sin wt,
2 represents alternating current and
Alternating current (a.c.) voltage of the same frequency as
 π
I = I0 sin  ωt +  When the current changes phasor along with proper phase angle
 2
continuously in magnitude and between them is called a phasor
1 direction periodically, then the current diagram.
Capacitive reactance X C =
ωC is said to be alternating.
SI Unit of XC is ohm. power consumed
is zero.

Direct Current (d.c.)


The current which flows with a
constant magnitude in the same
direction is called direct current.

Alternating Current 259


D:\EG_Physics-12_(26-06-2022)\Open_Files\Ch-7\Ch-7
\ 27-Jul-2022 Ved_Goswami Proof-4 Reader’s Sign _______________________ Date __________

REVISION CHART
A.C. source across a series LCR
circuit
If XL > XC
1

Impedance Z =  R 2 + ( X L − X C )  2
2
  Wattless Current
If XC > XL Power Consumed in a.c. Circuit
1
The current in AC circuit is said to
be wattless if power consumed in the Average power in an a.c. circuit per
Z = R 2 + ( XC − X L )
22
  circuit is 0. It is the component Irms cycle is given by
If XL > XC , Voltage leads current sin f of the a.c. In an inductive or V0
If XL < XC , current leads Voltage p
Pav = Vrms Irms cos f = I0 cos φ ,
capacitive circuit as f = 2
If f is phase difference between V and I 2 cos f is called the power factor
V − VC X L − X C so power factor = 0 and so the current
tan f = L = Pav
R is wattless. Power factor = cos f =
R Vrms I rms

Series LR, RC and LCR circuit

Power factor Condition of resonance in series Q-factor measures sharpness of resonance


It is the ratio of true power (Pav) to the LCR circuit: and it is defined as the ratio of resonant
apparent power (VrmsIrms) XL = XC or VL = VC frequency to the difference in two
Power factor = frequencies taken on both sides of resonant
1 frequency such that at each frequencies the
R R Resonant frequency = wr =
cos φ = = 1 LC 1
Z current amplitude becomes times the
 R 2 + (X L − X C )2  2 At resonance: 2
 
• X L = XC , values at resonance frequency.
• For pure resistive circuits f = 0 • VL = VC But out of phase ωr ωr
V2
So Pav = Vrms I rms = rms If f is phase difference between V Q-factor = ω − ω = 2 ∆ω
2 1
R and I
• For a pure inductive circuit Or
So V = VR
Q-factor is also defined as the ratio
p • V and I are in phase i.e. f = 0 of voltage across L (inductance) or C
f= , so Pav = 0
2 • Z is minimum and Zmin = R (capacitance)at resonance to the applied
• For a pure capacitive circuit: • Current amplitude is maximum voltage.
p V IX
f= , so Pav = 0 V0 V0  X L = X C  Q-factor = L = L
2 Imax = =  ∴ Z = R  VR IR
Z R  
• Power cunsumed is maximum = XL ωr L 1 L 1 L
  = = = × =
Irms Vrms R R LC R R C
R
•• Band width 2 Dw =
L
R
Variation of I0max with w So w2 = ωr + ,
2L
w2 – w1 =Dw = Band width
R
w1 = ωr −
2L

260 Physics–12
D:\EG_Physics-12_(26-06-2022)\Open_Files\Ch-7\Ch-7
\ 27-Jul-2022 Ved_Goswami Proof-4 Reader’s Sign _______________________ Date __________

IMPORTANCE OF EACH TOPIC AND FREQUENTLY ASKED TYPES OF QUESTIONS

☞ Important Topics
1. Questions based on peak and rms values. 2. Questions based on LCR-Circuit.
3. Questions based on Q-factor and phasor diagram.
* Maximum weightage is of AC Voltage to a Series LCR Circuit.

1. A coil of wire has a resistance of 30.0 W and an inductance of 0.100 H.


(a) What is its inductive reactance if connected to a 60-cycle line?
(b) What is its impedance?
(c) What would the current be if the coil were connected to a 120-V DC line?
(d) What would the current be if it were to a 120-V AC, 60-cycle line?
[Ans. (a) 37.7 Ω; (b) 48.2 Ω; (c) 4 A; (d) 2.49 A]
2. A 120-W rheostat and a 15 mF capacitor are connected in a series circuit to 120-V, 60-cycle emf.
(a) What is the reactance of the capacitor?
(b) What is the total impedence of the circuit?
(c) What is the current through the circuit?
(d) What is the voltage drop across each circuit element?
[Ans. (a) 177 Ω; (b) 214 Ω; (c) 0.561 A; (d) VC = 99.3 V, VR = 67.3 V]
3. If a 110-V line is connected to the primary of a step-up transformer, it delivers 2 amps on the secondary coil. The
ratio of turns on the two windings is 25. Assume no losses in the transformer. Find
(a) the secondary voltage (b) the primary current
[Ans. (a) 2750 V; (b) 50 A]
4. What is the impedance of a circuit which has a resistance of 30 Ω and an inductive reactance of 40 Ω?
[Ans. 50 Ω]
5. What is the phase angle for a circuit that has an inductance reactance of 30 Ω, a capacitive reactance of 20 Ω and
20 Ω resistance? [Ans. f = 26.6°, voltage leads current]
6. What is the capacitance needed in a circuit to produce resonance in a 60-cycle circuit having an inductance of 1
H?[Ans. 7.04 μF]
7. What is the ratio of primary turns to secondary turns in a transformer which is designed to operate a 6-V bell system
when connected to a 114-V line? [Ans. 19:1]
8. What is the reactance of a 2.00-μF capacitance at a frequency of 1, 60, 440, 106 Hz? What does this indicate?
[Ans. 7.96 × 104 Ω, 1.3 × 103 Ω, 181 Ω, 7.96 × 10–2 Ω]
9. A step-up transformer has a turns ratio of 200:1, and 100 V are applied to the primary side of this transformer.
(a) Find the secondary output voltage.
(b) If the secondary current is 100 mA, find the primary current.
(c) Find the power output of the transformer. [Ans. (a) 2 × 104 V; (b) 20 A; (c) 2 × 103 watts]

Alternating Current 261


D:\EG_Physics-12_(26-06-2022)\Open_Files\Ch-7\Ch-7
\ 27-Jul-2022 Ved_Goswami Proof-4 Reader’s Sign _______________________ Date __________

ASSIGNMENT
I. Objective Type Questions (1 Mark)
1. Multiple choice questions:
(i) If the rms current in 50 Hz a.c. circuit is 5 A, the value of current 1/300 seconds after its value becomes zero
is
3 5 5
(a) 5 2 A (b) 5 A (c) A (d) A
2 6 2
(ii) An alternating current generator has an internal resistance Rg and an internal reactance Xg. It is used to supply
power to a passive load consisting of a resistance Rg and reactance XL. For maximum power to be delivered
from the generator to load the value of XL is equal to
(a) zero (b) Xg (c) –Xg (d) Rg
2. Fill in the blanks:
(i) The frequency of a.c. supplied to our houses in India is ................ .
(ii) In an ideal inductor L = 4 H and w = 100 rad/s. The power developed is ................
II. Very Short Answer Type Questions (1 Mark)
3. Define the term 'resonance'. 
4. The power factor of an a.c. circuit is 0.5. What is the phase difference between the voltage and current in the
circuit?
5. An alternating current from a source is represented by I = 10 sin 314 t. Write the corresponding values of (i) its
effective value and (ii) frequency of the source.
III. Short Answer Type Questions-I (2 Marks)
6. Define the term ‘quality factor’.
7. What is meant by rms value of current? Derive an expression for the rms value of a.c. from

t = 0 to t =
.
ω
IV. Short Answer Type Questions-II (3 Marks)
8. In a series LR circuit XL = R and the power factor of the circuit is P1. When a capacitance of capacitor C such that
XC = XL is put in series, the power factor become P2. find out P1/P2.
9. (a) A resistor ‘R’ and an element ‘X’ are connected in series to an ac source of voltage. The voltage is found to
p
lead the current in phase by . If ‘X’ is replaced by another element ‘Y’, the voltage lags behind the current
p 4
by .
4
(i) Identify elements ‘X’ and ‘Y’.
(ii) When both ‘X’ and ‘Y’ are connected in series with ’R’ to the same source, will the power dissipated in the
circuit be maximum or minimum? Justify your answer.
(b) Define Q-factor of series LCR - circuit.
10. Explain the term inductive reactance. Show graphically the variation of inductive reactance with frequency of the
applied alternating voltage.
An ac voltage V = V0 sin wt is applied across a pure inductor of inductance L. Find an expression for the current i,
flowing in the circuit and show mathematically that the current flowing through it lags behind the applied voltage
by a phase angle of p . Also draw (a) phasor diagram (b) graphs of V and I versus wt for the circuit.
2
 rrr

262 Physics–12
D:\EG_Physics-12_(26-06-2022)\Open_Files\Ch-8\Ch-8
\ 27-Jul-2022 Ved_Goswami Proof-4 Reader’s Sign _______________________ Date __________

Topics Covered
8 Electromagnetic Waves

8.1 Displacement Current 8.2. Electromagnetic Waves 8.3. Electromagnetic Spectrum

C hapter map
Displacement Current

Ampere’s Circuital Modification of Ampere’s Maxwell’s prediction of


Maxwell Equations
Theorem Circuital Theorem Electromagnetic Waves

Electromagnetic Waves

Wavelength Electromagnetic Radiation Frequency

Electromagnetic Spectrum

Radio Micro Infrared Visible Ultra violet Gamma


X-rays
Waves Waves Rays light Rays Rays

Topic 1. Displacement Current


•• Displacement Current Consider two planar loops C1 and C2, C1 just left of the
 
Ampere’s Circuital Theorem ∫ B ⋅ dl = µo I capacitor and C2 just on right of plate of for the loop C1
 
Where I is the current passing through the conductor
C ∫ B ⋅ dl = µ0I...(1)
C1
threading the surface bounded by the loop. But the area bounded by C2 lies in the region between the
•• Inconsistency in Ampere’s circuital theorem: Consider plates of capacitor so no current flows across it.
the charging of a capacitor. According to Ampere’s  
circuital theorem, \ ∫ B ⋅ dl = 0 ...(2)
  C2

∫ B ⋅ dl = µo I But as C1 and C2 infinitesimally close, so right hand


C
sides of eqn. (1) and eqn. (2) should be equal. i.e.,
when I = current threading surface bounded by loop.  
  

C2
∫ B · dl = ∫ B · dl
C1 C2

This result is inconsistent with equations (1) and (2).


So we need to modify Ampere circuital theorem.
C1 •• Maxwell’s Modification of Ampere’s Circuital
Theorem: Maxwell followed a symmetry consideration.
He said that as changing magnetic field induces electric

263
D:\EG_Physics-12_(26-06-2022)\Open_Files\Ch-8\Ch-8
\ 27-Jul-2022 Ved_Goswami Proof-4 Reader’s Sign _______________________ Date __________

field hence a changing electric field must induce a 1. Gauss’s law in electrostatics
magnetic field.   qenclosed
As currents are the usual source of magnetic field. So
∫ E ⋅ dS = ∈0
S
Maxwell named this changing electric field [time varying A stationary charge will produce an electric field.
electric field] as displacement current.
2. Gauss’s law in magnetism
Displacement Current : It is a current which comes into  
existence whenever the electric field and hence electric
∫ B ⋅ dS = 0
flux changes with time. Magnetic monopoles
S do not exist.
ε0 d φE 3. Faraday’s laws of electromagnetic induction
Id = − d φB  
dt
where fE = electric flux
e=
dt
= ∫ ⋅ dl
E

Modified Ampere’s circuital theorem is   − d φ B − d    



  ∫ E ⋅ dl = =  ∫ B ⋅ dS
dt  
∫ B ⋅ dl = m0 (IC + ID) dt C 
C
− d φB
where ID = displacement current =
dt
   dφ 
E
This means that time varying magnetic field gives rise
∫ B ⋅ dl = µ 0  IC + ε0
 dt  to changing electric field.
C
•• To show that displacement current between plates of 4. Modified Ampere’s Circuital Theorem
capacitor is equal to conduction current:    E dφ 
∫ B ⋅ dl = µ 0  IC + ∈0
 dt 

Consider a capacitor being charged by a battery. C
σ This implies that time varying electric field gives rise
Electric field between plates of capacitor E =
ε0 to a changing magnetic field.
q
E= •• Maxwell’s Prediction of Electromagnetic Waves:
A ε0
From his four equations given earlier he said that a time
  q q varying electric field is a source of changing magnetic
f = E ⋅ dS = ×A=
A ε0 ε0 field and the time varying magnetic field is a source of
d φE changing electric field.
d  q  dq
Id = ε0 = ε0  = = IC This means that change in either field will produce the
dt dt  ε0  dt
other field. Also the variation in electric and magnetic
Thus displacement current = Conduction current. field occur in mutually perpendicular direction and shows
•• Maxwell Equations: A group of four equations wave like properties.

EXERCISE 8.1
I. Objective Type Questions (1 Mark) (d) The given electromagnetic wave is not plane
1. Choose the correct answers from the given options polarised.
(MCQs).  

(i) An electromagnetic wave travels in vacuum along
(ii) Expression ∫ B ⋅ ds = 0 shows.
(a) Gauss’s law of electrostatics
+Z direction  [NCERT Exemplar]
(b) Gauss’s law of magnetism
E = (E i + E j ) cos( kz − ωt ).
1 2
(c) Faraday’s law of e.m. induction
(a) The associated magnetic field is given as
(d) Modified Ampere’s circuital theorem
1
B = (E1 i + E2 j ) cos(kz – wt). 2. Fill in the blanks.
c
(b) The associated magnetic field is given as (i) The varying electric field between the plates of
1 the capacitor gives rise to ................ and hence
B = 2 (E1 i – E2 j ) cos(kz – wt).
c .................. is also produced.
(c) The given electromagnetic field is circularly (ii) .................... produces magnetic field in the same
polarised. way as does the conduction current.
264 Physics–12
D:\EG_Physics-12_(26-06-2022)\Open_Files\Ch-8\Ch-8
\ 27-Jul-2022 Ved_Goswami Proof-4 Reader’s Sign _______________________ Date __________

3. State True or False 9. Name the current which can flow even in the absence
(i) The displacement current flows in the dielectric of of electric charge. [CBSE S.P. 2018-19]
the capacitor, when the potential difference across 10. How displacement current produced between the
its plates changes (increases/decreases) with time. plates of parallel plate capacitor during charging.
(ii) Displacement current is the same as peak current.  [Delhi 2020]
For questions numbers 4 and 5, two statements are 11. Differentiate between conduction current and
given-one labelled Assertion (A) and the other labelled displacement current. [Delhi 2020]
Reason (R). Select the correct answer to these questions III. Short Answer Type Questions-I (2 Marks)
from the codes (a), (b), (c) and (d) as given below. 12. A capacitor, made of two parallel plates each of
(a) Both A and R are true and R is the correct plate area A and separation d, is being charged by an
explanation of A external ac source. Show that the displacement current
(b) Both A and R are true but R is NOT the correct inside the capacitor is the same as the current charging
explanation of A the capacitor. [AI 2013]
(c) A is true but R is false 13. When an ideal capacitor is charged by a dc battery, no
current flows. However, when an ac source is used,
(d) A is false and R is also false
the current flows continuously. How does one explain
4. Assertion (A): Modified Ampere’s circuital theorem
this, based on the concept of displacement current.
   d φE 
can be given by ∫ B.dl = µ0  Ic + ∈0 [Delhi 2012]
 dt 
c 14. A capacitor of capacitance of ‘C’ is being charged by
Reason (R): It implies that time varying electric field connecting it across a dc source along with an ammeter.
gives rise to a constant magnetic field. Will the ammeter show a momentary deflection during
5. Assertion (A): The electromagnetic waves are the process of charging? If so, how would you explain
transverse in nature. this momentary deflection and the resulting continuity
Reason (R): These waves propagate in straight of current in the circuit? Write the expression for the
line. current inside the capacitor. [AI 2012]
II. Very Short Answer Type Questions (1 Mark) 15. The charge on a parallel plate capacitor varies as q
1. Is the steady electric current the only source of = q0 cos 2pvt. The plates are very large and close
magnetic field? Justify your answer. [Delhi 13 C] together (Area = A, Separation = d). Neglecting the
d φE edge effects, find the displacement current through
2. Why the quantity ε 0 is called the displacement the capacitor.
current? dt
16. How does Ampere-Maxwell law explain the flow
3. A capacitor of capacitance C, is being charged up by of current through a capacitor when it is being
connecting it across a.d.c. voltage source of voltage V. charged by a battery? Write the expression for the
How do the conduction and displacement currents, in displacement current in terms of the rate of change
the set up compare with each other. of electric flux. [Delhi 2017]
(a) during the charging up process?
IV. Short Answer Type Questions-II (3 Marks)
(b) after the capacitor gets fully charged? [Delhi 2013]
17. Write the generalised expression for the Ampere’s
4. What oscillates in e.m. waves? Are these waves circuital law in terms of the conduction current and
longitudinal or transverse? the displacement current. Mention the situation when
5. A capacitor has been charged by a dc source. What there is:
are the magnitude of conduction and displacement (i) only conduction current and no displacement
currents, when it is fully charged? [Delhi 2013] current.
6. What is the origin of displacement current? (ii) displacement current and no conduction current.
7. A variable frequency ac source is connected to a  [Foreign 2013]
capacitor. How will the displacement current change 18. The figure below represents a capacitor made of two
with decreases in frequency? circular plates each of radius r = 12 cm and separated
[NCERT Exemplar][AI 2015 C] by d = 5.0 mm. The capacitor is being charged by an
8. How is the speed of em-waves in vacuum determined external source. The charging current is constant I =
by the electric and magnetic fields? [Delhi 2017] 0.15 A.

Electromagnetic Waves 265


D:\EG_Physics-12_(26-06-2022)\Open_Files\Ch-8\Ch-8
\ 27-Jul-2022 Ved_Goswami Proof-4 Reader’s Sign _______________________ Date __________

(i) Calculate the rate of change of electric field d

between the plates.


(ii) Find the displacement current across the plates.
(iii) Is Kirchhoff’s first role valid at each plate of
capacitor? Explain. [NCERT] ( )
+ –

Answers 8.1
I. Objective Type Questions medium between the capacitor plates that leads to
1. (i) (a) (ii) (b) polarisation and displacement of charges.
2. (i) Displacement current, magnetic field 11. Conduction current (I c) is established by actual
(ii) Displacement current movement of free electrons through a metallic conductor
3. (i) True (ii) False 4. (c) 5. (b) whereas displacement current (Id) is established by
polarisation of molecules of a dielectric substance under
II. Very Short Answer Type Questions
the effect of an external electric field.
1. No. The displacement current is also a source of
12. In figure conduction current is flowing in the wires,
magnetic field, as in case of the space between the plates
causes charge on the plates
of a capacitor being charged or discharged.
dq
d φE So, IC = (i) ½
2. Because the dimensions of ε 0 is the dt
dt
same as that of electric current and it exists due to C
the displacement of charges across the plates of the
capacitor ~
3. (a) During the charging up process conduction current Displacement current between plates
= displacement current = non zero. d φE
Id = ε0 , ...(ii) ½
(b) After the capacitor gets fully charged conduction dt
current = displacement current = 0. where fE = Electric flux
4. Electric and magnetic fields oscillate in e.m. waves. Using Gauss’s Theorem, if one of the plate is inside the
E.M. waves are transverse in nature. tiffin type Gaussian surface
5. During charging of a capacitor by a dc source, σ q
E = =  ½
ε dφ ε0 Aε0
IC = ID = 0 E
dt q
When capacitor is fully charged then Then fE = EA =
ε0
(fE) = Max
d  q dq
So IC = Id = 0 So, Id = ε 0 = ...(iii)
dt  ε 0  dt
6. Displacement current is produced by time varying
electric field. From equation (i) and (iii), IC = Id ½
7. With decrease in frequency, reactants 13. When an ideal capacitor is connected to a d.c. battery,
1 current flows momentarily, and it stops when the
XC = will increase which will lead to decrease capacitor is charged.
ωC
in conduction current. As ID = IC; hence displacement But when an a.c. source is used, the current flows
current will also decrease. continuously as a capacitor allows alternating current
E 1
to flow and offers an opposition of X C = . 1
8. Speed of em-waves in vacuum = c = 0 where E0 and ωC
B0
In case of a.c. the conduction current is changing. So the
B0 are the amplitudes of electric and magnetic field.
electric field between the plates is also changing with
9. Displacement current 1 time. Due to time changing electric flux displacement
[CBSE Marking Scheme, 2018-19] current is produced in the region between the plates.
10. Displacement current (Id) is produced by time varying d φE
 As Id = ε 0 and id = ic at all times. 1
electric flux (f) and electric field ( E ) across the diectric dt

266 Physics–12
D:\EG_Physics-12_(26-06-2022)\Open_Files\Ch-8\Ch-8
\ 27-Jul-2022 Ved_Goswami Proof-4 Reader’s Sign _______________________ Date __________

14. Yes, the displacement current between the plate of (ii) In the region in between the plates, there is only
capacitor are produced because of the changing electric displacement current and no conduction current. 1
field in between the plates of the capacitor. ½ 18. (i) Let C be the capacitance of capacitor and q the
So, outside the plates is conduction current and in the instantaneous charge on plates, then
region between the plates there is displacement current. q = CV
They also have equal magnitude. ½
dq dV
So there is continuity in the circuit. =I = C
     dt dt
Current inside the capacitor = displacement current
d φE
If E is the electric field between the plates, then V/d
ID = ε 0  1 = E or V = Ed
dt
15. Conduction current IC = Displacement current ID ½ dq d dE
= I = C (Ed ) i.e., I = Cd
  dt dt dt
IC = I D = dq = d q0 cos (2πvt )  1
dt dt dE I I I I
= – q02pv sin (2pvt) ½
∴ = = = = ...(i)
dt Cd ε 0 A ε 0 A ε 0 πr 2
16. When a capacitor is charged with a battery, the potential d
d
difference and hence electric field between the plates
increases from zero to its maximum value in a certain is equal to conduction current.½
period (though small) of time. 1 Here, I = 0.15 A, r = 12 cm = 12 × 10–2 m
Hence the electric flux between the plates changes \ Rate of change of electric field between the plates
with time, which is the cause of displacement current dE 0.15
In the connecting wires there is conduction current   =
dt 8.85 × 10 × 3.14 × (12 × 10 −2 ) 2
−12
IC, But in between the plates of the capacitor, there is
displacement current ID = IC. There is continuity in the    
= 9.74 × 1011 Vm–1s–1  ½
current, thus the capacitor gets charged- (ii) Displacement current
d φΕ dE  I 
ID = ε0  1        I d = ε 0 A dt = ε 0 A  ε A  [using (i)] ½
dt 0
 
17. ∫ B ⋅ dl = µ 0 (IC + ID )  1 = I = conduction current = 0.15A ½
C
(iii) Yes, Kirchhoff’s law holds at each plate of capacitor
(i) When a capacitor is being charged, in the connecting since displacement current is equal to conduction
wires there is only conduction current and no current. 1
displacement current. 1

Topic 2. Electromagnetic Waves


•• EM Waves: It is a wave radiated by an accelerated where E0 = amplitude
charge and which propagates through space as coupled BZ = B0 sin (kx – wt)
electric and magnetic field oscillating perpendicular to •• Sources of EM Waves: A stationary charge produces
each other and to direction of propagation. only electric field, whereas a charge in uniform motion
Y produces a constant value of magnetic field. So a
stationary or a steady current cannot produce a e.m.
waves. An accelerated charge will produce a magnetic
E field changing with time, which in turn will produce a
X
time varying electric field. Thus an accelerated charge
produces e.m. waves.
B The frequency of the e.m. wave is equal to the frequency
Magnetic field
of oscillation of the charge. The energy carried by the
wave comes from the source which makes the charge
Z
oscillate.
For a wave travelling along +ve x-axis •• Properties of EM Waves
Ey = E0 sin (kx – wt) 1. Transverse in nature.

Electromagnetic Waves 267


D:\EG_Physics-12_(26-06-2022)\Open_Files\Ch-8\Ch-8
\ 27-Jul-2022 Ved_Goswami Proof-4 Reader’s Sign _______________________ Date __________

2. Produced by accelerated charged particles. place in space. Ratio of electric and magnetic fields is
3. Obey principle of superposition. always constant and is equal to speed of EM waves
4. Energy in EM waves is divided equally between in that medium.
electric and magnetic fields. i.e., E0 = c
1 B0
5. In free space, they travel with a velocity c =
µ0 ε0 7. E.M. waves transport linear momentum and energy
where m0 = absolute permeability. as they travel through space. Momentum
e0 = absolute permittivity of the medium. U
1
p= , U → energy carried
In material medium, speed is v = c
µε 8. E.M. waves are not deflected by electric or magnetic
where e = permittivity of that medium. field.
6. Variation in amplitudes of electric and magnetic fields 9. The electric field of an e.m. wave is responsible for
in EM waves takes place at the same time and at same its optical effects, because E0 >> B0.

EXERCISE 8.2
I. Objective Type Questions (1 Mark) For questions numbers 4 and 5, two statements are
1. Choose the correct answers from the given options given-one labelled Assertion (A) and the other labelled
(MCQs). Reason (R). Select the correct answer to these questions
(i) A linearly polarized electromagnetic wave given from the codes (a), (b), (c) and (d) as given below.
(a) Both A and R are true and R is the correct
as E = E0 i cos(kz – wt) is incident normally on a
explanation of A
perfectly reflecting infinite wall at z = a. Assuming
that the material of the wall is optically inactive, (b) Both A and R are true but R is NOT the correct
the reflected wave will be given as explanation of A
(c) A is true but R is false
[NCERT Exemplar]
(d) A is false and R is also false
(a) E = –E i c os(kz – wt).
r 0 4. Assertion (A): Infrared radiations play an important
(b) Er = E0 i cos(kz + wt). role in maintaining the average temperature of earth.
Reason (R): Infrared radiations are sometimes
(c) E = –E i cos(kz + wt).
r 0
referred to as heat waves.
(d) Er = E0 i cos(kz – wt). 5. Assertion (A): When a charged particle moves in a
(ii) Light with an energy flux of 20 W/cm2 falls on a circular path, it produces electromagnetic waves.
non-reflecting surface at normal incidence. If the Reason (R): Charge particle has acceleration.
surface has an area of 30 cm2, the total momentum
II. Very Short Answer Type Questions (1 Mark)
delivered (for complete absorption) during 30
minutes is [NCERT Exemplar] 1. Out of microwaves, ultraviolet rays and infrared rays,
(a) 36 × 10 kg m/s. (b)36 × 10–4 kg m/s.
–5 which radiations will be most effective for emission
of electrons from a metallic surface?
(c) 108 × 104 kg m/s. (d)1.08 × 107 kg m/s.
2. What is the evidence that shows that sound is not an
2. Fill in the blanks.
e.m. wave?
(i) .................... are transverse in nature.
3. Give the ratio of velocities of light rays of wavelength
(ii) .................... is responsible for the optical effects
4000Å and 8000Å in vacuum.
due to an electromagnetic radiation and hence it
is also called as ....................... . 4. The wavelength of electromagnetic radiation is
doubled. What will happen to the energy of the
3. State True or False
photon?
(i) The speed of electromagnetic waves in a medium
5. What are the direction of electric and magnetic field
1
is . vectors relative to each other and relative to the
µ0 ε0 direction of propagation of electromagnetic waves?
(ii) Normal light is due to magnetic field oscillations. [AI 2012]

268 Physics–12
D:\EG_Physics-12_(26-06-2022)\Open_Files\Ch-8\Ch-8
\ 27-Jul-2022 Ved_Goswami Proof-4 Reader’s Sign _______________________ Date __________

6. A plane electromagnetic wave travels in vacuum along (b) How are the magnitudes of the electric and
z-direction. What can you say about the direction of magnetic fields related to the velocity of the em
electric and magnetic field vectors? [Delhi 2011] waves? [Delhi 2013]
7. (i) What is the ratio of speed of g-rays and radio 12. Find the wavelength of electromagnetic waves of
waves in vacuum? frequency 6 × 1012 Hz in free space. Give its two
(ii) Name the physical quantity which remains same application. [Delhi 2011]
for microwaves of wavelength 1 mm and UV 13. The oscillating magnetic field in a plane
radiations of 1600 Å in vacuum. [Delhi 2012] electromagnetic wave is given by
8. Depict the field diagram of an electromagnetic wave    By = (8 × 10–6) sin (2 × 1011 t + 300px) T
propagating along positive x-axis with its electric field (i) Calculate the wavelength of the electromagnetic
along y-axis. [Delhi 2020] wave.
9. Give the ratio of velocity of the two light waves of (ii) Write down the expression for the oscillating
wavelengths 4000Å and 8000Å travelling in vacuum. electric field. [Delhi 2008]
 [CBSE SP 2020-21] 14. Write the expression for the speed of light in a material
medium of relative permittivity e r and relative
III. Short Answer Type Questions-I (2 Marks)
magnetic permeability µr. [AI 2020]
10. Explain briefly how electromagnetic waves are
produced by an oscillating charge. How is the IV. Short Answer Type Questions-II (3 Marks)
frequency of the em waves produced related to that 15. In a plane electromagnetic wave, the electric
of the oscillating charge? [Foreign 2012] field oscillates sinusoidally at a frequency of
2.0 × 1010 Hz and amplitude 48 Vm–1.
11. What is meant by the transverse nature of
electromagnetic waves? Draw a diagram showing (a) What is the wavelength of a wave?
the propagation of an electromagnetic wave along (b) What is the amplitude of the oscillating magnetic
X-direction, indicating clearly the directions of field?
oscillating electric and magnetic field associated with (c) Show that the average energy density of the E field
it. [AI 2008] equals the average energy density of the B-field
[c = 3 × 108 ms–1] [NCERT]
OR
16. Suppose that the electric field amplitudes of an
(a) An em wave is travelling in a medium with
 electromagnetic wave is E0 = 120 N/C and that its
a velocity v = vi. Draw a sketch showing the frequency is v = 50.0 MHz.
propagation of the em wave, indicating the
(a) Determine B0, w, k and l
direction of the oscillating electric and magnetic  
fields. (b) Find expression for E and B.  [NCERT]

Answers 8.2
I. Objective Type Questions hc 1
1. (i) (b) (ii) (b) 4. Energy of a photon E = hv = , i.e., E ∝
λ λ
2. (i) Electromagnetic waves So if l is doubled, energy E of the photon is halved.
(ii) Electric vector, light vector 5. Electric and magnetic field vectors are perpendicular
3. (i) False (ii) False to each other and also to the direction of propagation
of e.m. wave.
4. (b) 5. (a)
6. Electric field vector along X-axis
II. Very Short Answer Type Questions Magnetic field vector along Y-axis.
1. Ultraviolet rays. As ultraviolet rays have the highest As electronic field and magnetic field are perpendicular
frequency among the wave given, and hence the most to each other and also perpendicular to the direction of
energetic. propagation of wave.
2. Sound requires a medium for propagation whereas e.m. Speed of γ -rays in vacuum
waves do not require a medium for propagation. 7. (i) =1
Speed of radiowaves in vacuum
3. Ratio is One. Because e.m. waves of all wavelengths (ii) Speed (c = 3 × 108 m/s). As both of them are e.m.
travel with same speed in vacuum. waves.

Electromagnetic Waves 269


D:\EG_Physics-12_(26-06-2022)\Open_Files\Ch-8\Ch-8
\ 27-Jul-2022 Ved_Goswami Proof-4 Reader’s Sign _______________________ Date __________

8. Y 2π 1
λ= = m
300π 500  ½
E0
E (ii) = c or E 0 = B0 c = 8 × 10 −6 × 3 × 108
B0
X
  E0 = 24 × 102 = 2.4 × 103 V/m ½
B
Magnetic field ( )
As B is along j , velocity is along − i

Thus E is along +ve z-axis i.e., ( + 


k)
Z
9. 1:1 because in Vacuum, all colours travel with the same Because 
k × j = − i
velocity. Thus, EZ = 2.4×103 sin (2 × 1011t + 300px) V/m ½
10. An oscillating or accelerated charge supposed to be
1 c  1 
a source of an electromagnetic wave. An oscillating 14. v = =  c = 
charge produces an oscillating electric field in space µ 0µ r ε 0 ε r µr εr  µ0ε0 
which produces an oscillating magnetic field which is
15. Given : v = 2 × 1010 Hz, E0 = 48 Vm–1
again a source of electric field. These oscillating electric
and magnetic field, hence, keep on regenerating each c 3 × 108
(a) λ = = = 1.5 × 10 −2 m  1
other and an electromagnetic wave is produced. 1 v 2 × 1010
The frequency of em wave = Frequency of oscillating E0 48
charge. 1 (b) B0 = = = 1.6 × 10 −7 T  1
c 3 × 108
11. (a) Transverse waves are those waves in which the
vibrations are perpendicular to the direction of (c) To show that the energy density of electric field =
propagation of waves. ½ average energy density of the magnetic field.
Energy density of E. field
Y
1 2
= ue = ε 0 E ...(i)
E B E B 2
½ Energy density of M-field
X
1 2
B B = u B = B ...(ii)
E E
Direction of propagation
2µ 0
Z
1 1
In e.m. waves the electric and magnetic field are Now, ue = ε o E 2 = ε 0 (cB)2  ½
2 2
vibrating ^ to the direction of propagation of waves.
2
E ^ v, B ^ v, E ^ B ½  E  1  1  1
2
E0 
 B = c  = 2 ε 0  µ ε  B = 2µ 0 × B
2

(b) Speed of e.m. wave =  ½  0 0 


B0
Thus, ue = uB ½
E0 → Amplitude of electric field
B0 → Amplitude of magnetic field  1 
8  as c = 
c 3 × 10
 µ0 ε0 
12. As λ = = =5 × 10 −5 m  ½
v 6 × 1012 16. (a) E0 = 120 N/C, v = 50 MHz = 5 × 107 Hz
This wavelength corresponds to infrared waves. ½ B0 = E0/c = 120 / (3 × 108)
Application of infrared rays : = 4 × 10–7 T ½
(i) They are used in taking photograph during fogs.½ w= 2pv = 2 × 3.14 × 5 × 107

(ii) They are used in green houses to warm the plants. = 3.14 × 108 rad/s ½
 ½
c 3 × 108
13. Comparing it with general equation of a plane λ= = = 6m
v 5 × 107  ½
progressive wave y = A sin (wt + kx)
2π 2π 2π π 3.14
k= = = = = 1.05 m −1 
(i) k = = 300π  ½ λ 6 3 3 ½
λ
270 Physics–12
D:\EG_Physics-12_(26-06-2022)\Open_Files\Ch-8\Ch-8
\ 27-Jul-2022 Ved_Goswami Proof-4 Reader’s Sign _______________________ Date __________

(b) Let the wave be propagating along +ve x-axis 


E y = 120 sin (3.14 × 108 t − 1.05 x)j N/C  ½
Electric field will be along y-axis and magnetic field 
along z-axis then B z = 4 × 10 −7 sin (3.14 × 108 t − 1.05 x) 
kT ½

Topic 3. Electromagnetic Spectrum


•• Electromagnetic spectrum: Increasing order of wavelength :
g rays X rays UV Visible Infra red (IR) Micro waves Radio waves
< 10 nm 10 nm to 1 nm 1 nm to 400 nm 400 nm to 700 nm 1 mm to 700 nm 1 mm to 3 m
–3 –3
>3m
All EM waves travel with same speed in vacuum.
The orderly distribution of electric magnetic radiations Uses :
according to their wavelength/frequency is called (i) Can cause fluorescence, therefore used to check
electromagnetic spectrum. mineral samples.
•• Gamma Rays: (ii) To study molecular structure/arrangement of
– Origin : Nuclear origin, in decay of radioactive electrons in external shell through UV absorption
substances. spectra.
– Highly energetic, high penetrating power. (iii) In detection of forged documents, finger prints in
Uses: forensic laboratory.
(iv) To purify water as they can kill germs.
(i) Provide information about structure of atomic
nuclei. (v) Used to sterilize surgical instruments as they can
destroy bacteria.
(ii) In radio therapy
(vi) Used in burglar alarms due to its ability to cause
(iii) To detect leakage/blockage in underground
emission of photoelectrons.
pipelines.
•• Visible Light:
•• X-rays:
– Emitted due to atomic excitation.
– Produced by sudden deceleration of high energy
Frequency : 8 × 1014 Hz to 4 × 1014 Hz
electrons.
Uses : (i) Helps to see objects.
Frequency : 3 × 1019 Hz to 1 × 1016 Hz
•• Infra Red Rays (IR):
Uses :
– Emitted due to excitation of atoms and molecules.
(i) In surgery to detect fractures, foreign bodies like
bullets, stones and diseased organs in human body Frequency : 4 × 1014 Hz to 1 × 1013 Hz
as X-rays can pass through human flesh but are Uses :
stopped by bones, metals. (i) Keep earth warm and hence help to sustain life.
(ii) In radio therapy, to control untraceable skin (ii) In solar water heaters and cookers.
decrease malignant growth. (iii) Coal deposits in interior of earth are the result of
(iii) In detective departments to detect explosives, conversion of forest wood into coal due to IR rays.
used by custom department to detect gold, silver (iv) To take photographs during conditions of fog and
in body of smugglers. Also used to distinguish haze as they can travel longer distances through
real diamond and gems from artificial ones. atmosphere under conditions of fog, smoke w/o
(iv) In engineering to detect flaws, cracks, holes in much scattering (scattered less as compared to
metal products. visible light).
(v) In scientific research to study crystal structure (v) To treat muscular strain.
arrangement of molecules, atoms in complex (vi) Used to produce dehydrated fruits.
substances. (vii) In weather forecasting, through IR photography.
•• Ultra-violet rays (UV): (viii)Used to provide electrical energy to satellites by
– Produced by excitation of atoms in spark and arc using solar cells.
lamps. •• Micro Waves:
Frequency : 5 × 1017 Hz to 8 × 1014 Hz – Oscillating currents in special vacuum tubes.

Electromagnetic Waves 271


D:\EG_Physics-12_(26-06-2022)\Open_Files\Ch-8\Ch-8
\ 27-Jul-2022 Ved_Goswami Proof-4 Reader’s Sign _______________________ Date __________

Frequency : 3 × 1011 Hz to 1 × 109 Hz •• Radio Waves:


Uses : – Oscillating circuits.
(i) In radars, other communication systems. Frequency : 3 × 109 Hz to 1 × 104 Hz
(ii) To cook food Uses : (i) TV and radio communications.

EXERCISE 8.3
I. Objective Type Questions (1 Mark) (iii) UV-rays (c) Radio therapy
1. Choose the correct answers from the given options (iv) IR-rays (d) Purify water
(MCQs). For questions numbers 5 two statements are given-one
(i) If lv, lx and lm represent the wavelengths of visible labelled Assertion (A) and the other labelled Reason
light, X-rays and microwaves respectively, then (R). Select the correct answer to these questions from
(a) lm > lx > lv (b) lv > lm > lx the codes (a), (b), (c) and (d) as given below.
(c) lv > lx > lm (d) lm > lv > lx (a) Both A and R are true and R is the correct
(ii) The correct arrangement of colours in the explanation of A
descending order of their wavelength is (b) Both A and R are true but R is NOT the correct
explanation of A
(a) yellow, violet, green, orange.
(c) A is true but R is false
(b) orange, yellow, green, violet.
(d) A is false and R is also false
(c) violet, green, yellow, orange.
5. Assertion (A): Microwave communication is
(d) yellow, green, orange, violet. preferred over optical communication.
(iii) A welder wears special glasses to protect his eyes Reason (R): Microwaves provide large number of
mostly from the harmful effect of channels and band width compared to optical signals.
(a) very intense visible light. II. Very Short Answer Type Questions (1 Mark)
(b) infrared radiation. [AI 2020] 1. The velocity of propagation (in vacuum) and the
(c) ultraviolet rays. (d) microwaves. frequency of (i) X-rays and (ii) radio-waves are
(iv) Electromagnetic waves used as a diagnostic tool denoted by (VX, nX) and (VR, nR) respectively. How
in medicine are do the values of (a) VX and VR (b) nx and nR compare
(a) X-rays. with each other. [S.P. 2017]
(b) ultraviolet rays. [AI 2020] 2. How are microwaves produced? [AI 2011, 2015]
(c) infrared radiation. 3. Justify the statement that a global nuclear war will be
(d) ultrasonic waves. followed by global nuclear winter.
2. Fill in the blanks. 4. How are radio waves produced? [AI 2011]
(i) .................... produces heat when absorbed by the 5. Why does microwave oven heat up a food item
matter and ............... are used in radio astronomy containing water molecules most efficiently?
and radar system. [NCERT Exemplar]
(ii) ....................... are used for producing dehydrated 6. Which out of the following are electromagnetic
fruits. waves: X-rays, sound waves and radiowaves?
3. State True or False 7. Which part of the electromagnetic spectrum has the
(i) X-rays has the lowest frequency. large penetrating power? [Delhi 2010]
(ii) The frequency of visible light is of the order of 8. Which part of the electromagnetic spectrum is used
1015 Hz. in operating a RADAR? [Delhi 2010]
4. Match the Columns 9. Which part of the electromagnetic spectrum is
Column-I Column-II absorbed from sunlight by ozone layer?[Delhi 2010]
(i) Gamma (a) To detect fractures 10. Name the electromagnetic waves, which
(ii) X-rays (b) In weather forecasting (i) maintain the Earth’s warmth and

272 Physics–12
D:\EG_Physics-12_(26-06-2022)\Open_Files\Ch-8\Ch-8
\ 27-Jul-2022 Ved_Goswami Proof-4 Reader’s Sign _______________________ Date __________

(ii) are used in aircraft navigation. [Foreign 2012] IV. Short Answer Type Questions-II (3 Marks)
11. Welders wear special goggles or face masks with glass 23. Write any four characteristics of electromagnetic
windows to protect their eyes from electromagnetic waves. Give any uses each of:
radiations. Name the radiations and write the range (i) Radio-waves (ii) Microwaves.
of their frequency. [AI 2013] [Delhi 2010, 2011]
12. Write the following radiations in ascending order in 24. Identify the following electromagnetic radiation as
respect of their frequencies: X-rays, microwaves, per the wavelength given below:
ultraviolet rays and radio waves and gamma rays. (i) 10– 4 nm (ii) 10–3 m
[Delhi 2010] (iii) 1 nm. Write one application of each. [AI 2008)
13. Name the part of the electromagnetic spectrum whose 25. (i) Which one of the following electromagnetic
wavelength lies in the range 10–10 m. Give its one radiations has least frequency:
use. [AI 2010]
UV radiations, X-rays, Microwaves [AI 2015]
14. Arrange the following in descending order of
(ii) How do you show that electromagnetic waves
wavelength. X-rays, Radio-waves, Blue light, infrared
carry energy and momentum? [AI 2015]
light. [Foreign 2010]
26. How are infrared waves produced? Why are these
15. Name the electromagnetic radiations used for (a)
referred to as ‘heat waves’? Write their one important
water purification, and (b) eye surgery.[Delhi 2018]
use. [Delhi 2011, AI 2015]
16. Name the electromagnetic waves that are widely used
27. The following table gives the wavelength range of
as a diagnostic tool in medicine. [CBSE S.P. 2018-19]
some constituents of the electromagnetic spectrum.
17. Write the mathematical form of Ampere-Maxwell
[Delhi 2008 C]
circuital law. [Delhi 2020]
18. Mention one use of part of electromagnetic spectrum S. No. Wavelength Range
to which a wavelength of 21 cm (emitted by hydrogen 1. 1 mm to 700 nm
in interstellar space) belongs.  [CBSE S.P. 2020-21]
2. 400 nm to 1 nm
III. Short Answer Type Questions-I  (2 Marks)
3. 1 nm to 10–3 nm
19. Compare the following:
(i) Wavelengths of the incident solar radiation 4. <10–3 nm
absorbed by the earth’s surface and the radiation Select the wavelength range and name the
re-radiated by the earth. electromagnetic waves that are:
(ii) Tanning effect produced on the skin by UV
(i) widely used in the remote switches of household
radiation incident directly on the skin and that electronic devices.
coming through glass window.
(ii) produced in nuclear reactions. [Delhi 2008 C]
[CBSE S.P. 2018-19]
28. Why are infra-red radiations referred to as heat waves?
20. (i) Why are infra-red waves often called heat waves?
Name the radiations which are next to these radiations
Explain.
in the electromagnetic spectrum having (i) shorter
(ii) W hat do you understand by the statement, wavelength (ii) longer wavelength.
“Electromagnetic waves transport momentum”?
29. Electromagnetic waves with wavelength:
21. Gamma rays and radio waves travel with the same
velocity in free space. Distinguish between them in (i) l1 is used in satellite communication
terms of their origin and the main application. (ii) l2 is used to kill germs in water purifier
 [Delhi 2020] (iii) l3 is used to detect leakage of oil in underground
22. Which of the following electromagnetic waves pipelines.
has (i) minimum wavelengths, and (ii) minimum (iv) l4 is used to improve visibility in runways, during
frequency? Write one use of each of these two waves. fog and mist conditions.
[AI 2020] (a) Identify and name the part of electromagnetic
Infrared waves, Microwaves, g-rays and X-rays. spectrum to which these radiations belong.

Electromagnetic Waves 273


D:\EG_Physics-12_(26-06-2022)\Open_Files\Ch-8\Ch-8
\ 27-Jul-2022 Ved_Goswami Proof-4 Reader’s Sign _______________________ Date __________

(b) Arrange these wavelengths in ascending order 31. Find the wavelength of electromagnetic waves of
of their magnitude. frequency 4 × 109 Hz in free space. Give its two
(c) Write one more application of each. applications.
 [NCERT Exemplar] 32. Name the parts of the electromagnetic spectrum which
is: [Delhi 2015]
30. Name the constituent radiation of electromagnetic
(i) Suitable for radar systems used in aircraft
spectrum which
navigation.
(i) is used in satellite communication (ii) Used to treat muscular strain.
(ii) is used for studying crystal structure (iii) Used as a diagnostic tool in medicine.
(iii) is similar to the radiations emitted during the decay Write in brief, how these waves can be produced.
of radioactive nuclei 33. (i) Arrange the following electromagnetic radiation in
(iv) has its wavelength range between 390 nm and the ascending order of their frequencies:
700 nm.  [CBSE 2022]
(v) is absorbed from sunlight by ozone layer X-rays, microwaves, gamma rays, radio waves.
(vi) produces intense heating effect. [Foreign 2010] (ii) Write two uses of any two of these radiation.

Answers 8.3
I. Objective Type Questions 6. X-rays and radiowaves are electromagnetic waves.
1. (i) (d) (ii) (b) (iii) (c) (iv) (a) 7. g-rays.
2. (i) Microwaves, radiowaves (ii) Infrared. 8. Microwave are used in operating a RADAR.
3. (i) False (ii) True 9. Ultraviolet light is absorbed by ozone layer.
4. (i) -(c) (ii) -(a) (iii) -(d) (iv) -(b)
10. (i) Infrared rays (ii) Microwaves
5. (a) Both A and R are true and R is the correct explanation
11. Ultraviolet radiations.
of A Microwaves are electromagnetic waves (EMW)
with wavelength as longer than those of infrared Frequency range 1015 – 1017 Hz.
(I.R.) light but relatively short from radio waves. 12. Ascending order of frequencies is : Radio-waves,
The microwave range includes ultra high frequency microwaves, ultraviolet rays, X-rays and g-rays.
(UHF), super high frequency (SHF) and extremely 13. The electromagnetic wave having wavelength 10–10 m
high frequency (30 – 300 GHz) signals. Optical is X-rays.
communication is any form of telecommunication
that uses light as the transmission medium. Hence, X-rays are used to study crystal structure and to detect
its channels and bandwidth (w) are lesser than fractures in bones.
microwave communication. 14. Radio-waves, Infrared rays, Blue light, X-rays
II. Very Short Answer Type Questions 15. (a) Ultraviolet (b) Ultraviolet
1. (a) VX = VR (b) nX > nR 16. X-rays1
2. Microwaves are produced in special vacuum tubes [CBSE Marking Scheme, 2018-19]
called klystrons due to oscillating currents.     dφ 
3. The clouds produced by a global nuclear war would
17. ∫ B ⋅ dl = µ0  I + ε0 E 
 dt 
l
cover most parts of the sky. These clouds will prevent
the solar light from reaching many parts of the globe. 18. Radio wave used in remote control, air traffic control.
So this would cause a nuclear winter. 19. (i) Radiation re-radiated by earth has greater
4. Radio waves are produced because of the accelerated wavelength. 1
motion of electrons in conducting wires or oscillating (ii) Tanning effect is significant for direct UV radiation; it
circuits for example LC circuit. is negligible for radiation coming through the glass. 1
5. Microwave oven heat up a food item containing water [CBSE Marking Scheme, 2018-19]
molecules most efficiently because the frequency of 20. (i) Infrared waves are often called as heat waves,
microwaves matches with the natural frequency of because they induces resonance in molecules and
water molecules. There is a resonance. increase internal energy of a substance. 1

274 Physics–12
D:\EG_Physics-12_(26-06-2022)\Open_Files\Ch-8\Ch-8
\ 27-Jul-2022 Ved_Goswami Proof-4 Reader’s Sign _______________________ Date __________

(ii) E.M. waves transport momentum, means when these 26. Infrared rays are produced by hot bodies or by vibrations
waves strikes a surface a pressure is exerted on the of molecules or atoms. 1
surface. 1 They are referred to as heat waves because IR rays get
21. Gamma rays have nuclear origin and produced in decay easily absorbed by water molecules in most materials. 1
of radioactive substances whereas radio waves are Due to the increased thermal agitation their temperature
produced from rapid acceleration and deceleration of increases and hence get heated. They are used in
electrons in aerials. muscular pain relief. 1
Gamma rays are used to detect leakage/blockage in 27. (i) Infrared waves (wavelength range 1 mm to 700 nm)
underground pipelines and in treatment of cancer.  1½
As the name itself suggests, radio waves are used in (ii) Gamma rays (wavelength range <10 nm). –3

radio and television communications and broadcasting
28. Refer Ans. to Q. No. 26. 1
the signals.
(i) EM waves having shorter to wavelength than
22. (i) g-rays, use of g-rays : Treatment of cancer
infrared waves are visible, UV, X-rays and g-rays. 1
(ii) microwaves, use of microwave: in radars.
(ii) EM waves having longer wavelength than infrared
23. Characteristics of e.m. waves: waves are microwaves, short radio waves, television
(i) They are transverse in nature. and FM radio. 1
(ii) They do not need a medium for propagation. ½ 29. (a) l1 → Microwaves, l2 → ultraviolet,
(iii) They are produced by accelerated charge particles. l3 → X-rays, l4 → Infrared 1
 ½
(b) Ascending order of wavelength l3, l2, l4, l1 1
(iv) They obey superposition principle.
(c) (i) Microwaves → used in RADAR
Uses:
(ii) Ultraviolet rays → Used in food preservation½
(i) Radiowaves: (a) Used in radio astronomy. (b) In
(iii) X-rays → Detection of bone-fractures
radio and television communications 1
(ii) Microwaves : (a) For cooking in microwave ovens. (iv) Infrared rays → In green house to keep the plants
warm. ½
(b) In RADAR systems for aircraft navigation.
(iii) In long distance communication system i.e., satellite 30. (i) Microwaves ½
communications. 1 (ii) X-rays ½
24. (i) 10 nm → gamma radiation
–4 (iii) g-rays ½
Application: Radio therapy or to initiate nuclear (iv) Visible light ½
reactions. 1 (v) Ultraviolet rays ½
(ii) 10–3 m → microwaves (vi) Infrared ½
Application: in RADAR for aircraft navigation, 31. Wavelength,
microwave cooking (oven). 1 c 3 × 108 3 300
(iii) 1 nm → X-rays   λ = = = m= cm = 7.5 cm  1
v 4 × 109 40 40
Application : In medical science for detection of
This wavelength corresponds to microwave region. 1
fractures in bones. 1
These are used in (i) Radar (ii) Microwave oven for
25. (i) Microwaves have least frequency. 1
cooking. 1
(ii) When e.m. waves falls on a surface, its electric
32. (i) Microwaves (ii) Infrared rays (iii) X-rays
and magnetic field, make the charges present in the
matter oscillate. The charges acquire momentum and Microwaves : In magnetron or klystron vacuum tubes. 1
energy from the e.m. waves. This shows that e.m. I-R rays : In hot bodies by vibrations of atoms and
waves carry energy and momentum. 1 molecules. 1
For example when the sun’s radiations fall on an X-rays : By bombarding high energy electrons on a
object, it gets heated. The energy absorbed from the metal target. 1
e.m. waves heats the object. Also when radiations 33. (i) Radiowaves < Microwaves < X-rays < Gamma rays
fall on a surface, it also exerts pressure called (ii) X-rays can be used in Medical Science, Security
radiation pressure. 1 purposes, Astronomy etc. Infrared rays find use in
Thus e.m. waves carry momentum. Electrical heaters, cooking food etc like microwaves.

Electromagnetic Waves 275


D:\EG_Physics-12_(26-06-2022)\Open_Files\Ch-8\Ch-8
\ 27-Jul-2022 Ved_Goswami Proof-4 Reader’s Sign _______________________ Date __________

Case Based Questions


I. A stationary charge produces only an electrostatic
field while a charge in uniform motion produces a
magnetic field, that does not change with time. An
oscillating charge is an example of accelerating
charge. It produces an oscillating magnetic field which
in turn produces an oscillating electric field and so on.
The oscillating electric and magnetic field regenerate
each other as a wave which propagates through space.

1. What type of wave is solar radiation?


2. Name the radiation that causes greenhouse effect.
3. Which radiation is absorbed by ozone layer?
Ans. 1. Solar radiation is transverse em wave.
2. Infrared ray causes greenhouse effect.

3. Ozone layer absorbs harmful UV rays coming

from the sun.
III. The spectrum of electromagnetic wave is as follows:
1. How are magnitudes of electric and magnetic fields
related to the velocity of the electromagnetic wave. → Decreasing wavelength
2. The magnetic field Radio- Micro- Infrared Visible Ultra- X- g-
 in a plane electromagnetic wave
is given by B = B0 sin (kx + ωt) ˆj T. Write the wave wave region violet rays rays
expression for the corresponding electric field. rays

3. A plane electromagnetic wave of frequency 25 MHz ● Radiowaves. These waves have longest wavelength
travels in a free space along x-direction. At a particular and minimum frequency.
point in space and time, E = 6.3 ĵ V/m. What is the Frequency range 500 × 103 Hz to 1000 × 106 Hz
magnitude of magnetic field at that time?
Wavelength range 600 m to 0.1 m
E
Ans. 1. Velocity of em wave = 0 Source Accelerated motion of charges
B0
in conducting bodies
Where E0 = amplitude of electric field
● Microwaves.
and B0 = amplitude of magnetic field.
 Frequency range 109 Hz to 1012 Hz
2. E = E0 sin (kx + ωt) kˆ T

Wavelength range 0.3 m to 0.001 m
E 6.3
3. Bz = =
8
= 2.1 × 10 −8 T = 0.021 µT Source Klystrons, Magnetrons and
c 3 × 10 Gunn diodes
II. Radio waves are produced by the accelerated motion ● Infrared waves. Infrared waves produce heating
of charges in conducting wires. Microwaves are effect, so they are also known as heat waves or
produced by special vacuum tubes. Infrared waves are thermal radiation. The water molecules or NH3,
produced by hot bodies and molecules also known as CO2 molecules present in different materials absorb
infrared (IR) waves and increase the thermal motions
heat waves. UV rays are produced by special lamps and ultimately heat up the materials and their
and very hot bodies like sun. surroundings region very well.

276 Physics–12
D:\EG_Physics-12_(26-06-2022)\Open_Files\Ch-8\Ch-8
\ 27-Jul-2022 Ved_Goswami Proof-4 Reader’s Sign _______________________ Date __________

Frequency range 1011 Hz to 5 × 1014 Hz 1. Which of the following wave is not employed in

case of remote sensing?
Wavelength range 5 × 10–3 m to 10–6 m
(a) X-rays (b) Visible rays
Source Hot bodies
(c) Thermal IR (d) Radio waves
● Visible light. It is a very small part of the
electromagnetic spectrum for which the our retina is 2. EM waves varies from ............ to ............ .
sensitive. (a) Metres to nano-metres
Frequency range 4 × 1014 Hz to 7 × 1014 Hz (b) Metres to micro-metres
Wavelength range 8 × 10 m to 4 × 10 m –7 –7
(c) Nano metres to micro-metres
Source Radiated by excited atoms in (d) Centimetres to nano-metres
ionised gas and incandescent 3. Which among the following is having more
bodies. wavelengths?
● Ultraviolet (UV) light. These are present in sunlight. (a) X-rays (b) Cosmic waves
Frequency range 1016 Hz to 1017 Hz
(c) Radio waves (d) Gamma rays
Wavelength range 3.5 × 10–7 m to 1.5 × 10–7 m
4. Which of the following indicates the correct set
Source High voltage gas discharge of combination in radio waves?
tubes, Sun (a) Shorter wavelength - high frequency
● X-rays. As X-rays can pass through many forms of
(b) Longer wavelength - less frequency
matter, so they are used in medical and industrial field.
(c) Shorter wavelength - less frequency
Frequency range 1018 Hz to 1020 Hz
(d) Longer wavelength - high frequency
Wavelength range 10–8 m to 10–11 m
5. Radio waves are
Source Sudden collision of fast
moving electrons by a Heavy (a) uniform motion of electrons in the conducting
body.
metal target.
● Gamma-rays (g-rays). These waves have highest (b) uniform motion of protons in the conducting
body.
frequency range and lowest wavelength range.
Penetrating power of these rays are very high. (c) accelerated motion of protons in conducting
body.
Frequency range 1018 Hz to 1022 Hz
(d) accelerated motion of electrons in conducting
Wavelength range 10–14 m to 10–10 m body.
Source Radioactive nuclei and nuclear Ans. 1. (a) 2. (a) 3. (c) 4. (a) 5. (d)
reactions.

COMMON ERRORS
S. No. Errors Corrections
1. Confusion in direction of propagation of wave From phase term, direction of propagation of wave is
obtained and
2. directions of magnetic and electric fields. from the unit vector, direction of fields can be obtained.
3. Electromagnetic spectrum in the order of increasing Either the order of frequency or the wavelength should be
or decreasing wavelength or frequency remembered from which the other can be calculated.

Electromagnetic Waves 277


D:\EG_Physics-12_(26-06-2022)\Open_Files\Ch-8\Ch-8
\ 27-Jul-2022 Ved_Goswami Proof-4 Reader’s Sign _______________________ Date __________

IMPORTANT FORMULAE
Formula Symbols Application
1. 1 c is the speed of electromagnetic To find speed of e.m. wave in vacuum.
c= wave in vacuum, m0 is permeability
µ0 ε0
constant and e0 is the permittivity
constant for vacuum.
2. 1 n is the speed of electromagnetic To find speed of electromagnetic
ν= wave in material medium. wave in material medium.
µε
3. Laws at a glance Used in various electric and magnetic
 
(i) ∫ B · dA = 0 fB = Flux in magnetic field field related problems.
S

(Gauss’s Law for electricity)


  fE = Flux in electric field
(ii) ∫ B · dA = 0
S
(Gauss’s Law for magnetism)

 
– dφ B
(iii) ∫ E ⋅ dl
dt
=
S
(Faraday’s Law)

  dφ ε
(iv) ∫ B ⋅ dl dt
= m0ic + m0 e0
S
(Ampere – Maxwell Law)
4. d φE ID = Displacement current Charging or discharging of a
ID = ε 0 capacitor.
dt

5. Modified Ampere’s Circuit Theorem IC = Conduction current In charging/discharging of a capacitor


   in a circuit
∫ B ⋅ dl = µ0(IC + ID)
c
d φE
= µ0(IC + ε 0 )
dt

6. Speed of an e.m. wave in vacuum B0 = amplitude of the magnetic field To determined speed of light
E0 in vacuum
c=
B0

7. Equation of a plane e.m. wave is k = propagation vector To determine frequency angular


By = B0 sin (wt ± kx) frequency and wavelength of e.m.
wave.
Ez = E0 sin (wt ± kx)
w = 2pn = 2p , k = 2π
T λ

278 Physics–12
D:\EG_Physics-12_(26-06-2022)\Open_Files\Ch-8\Ch-8
\ 27-Jul-2022 Ved_Goswami Proof-4 Reader’s Sign _______________________ Date __________

REVISION CHART

Displacement Current
It is a current which comes into existence whenever the
Source EM Waves electric field and hence electric flux changes with time.
An accelerated change is the ε0 d φE
Id =
source of e.m. waves. dt

Electromagnetic Waves
It is a wave radiated by an accelerated charge and which propagates through space as coupled electric and
magnetic field oscillating perpendicular to each other and to direction of propagation of wave.
For a wave travelling along +ve x-axis
Ey = E0 sin (kx – wt), By = B0sin (kx – wt)

Properties of EM Waves Modified Ampere’s


•• They are transverse in nature. circuital Law
•• The energy transported by e.m. waves s divided equality The line integral of magnetic
between electric and magnetic field. field Β over a closed path
is equal to µ0 times the sum
•• The energy transported by e.m. waves per second per
of conduction current IC and
unit is represented by a vector called Poynting Vector S .
the displacement current ID
Ε ×Β threading the closed path.
S=
µ0
   ε0 d φE
•• The ratio of amplitude of electric field and magnetic field
is constant and that constant is equal to speed of e.m. wave
∫c B ⋅ dl = µ0(IC + dt )
E0
in free space, i.e. c =
B0
1
•• Speed e.m. waves in free space c = = 3 ×108 m/s
µ0 ε0
1
In a material medium, v =
µε

Electromagnetic Waves 279


D:\EG_Physics-12_(26-06-2022)\Open_Files\Ch-8\Ch-8
\ 27-Jul-2022 Ved_Goswami Proof-4 Reader’s Sign _______________________ Date __________

IMPORTANCE OF EACH TOPIC AND FREQUENTLY ASKED TYPES OF QUESTIONS

☞ Important Topics
1. Questions related to Electromagnetic Spectrum.
2. Questions on arranging of wavelength and frequency of emw in decreasing or increasing order.
3. Questions based on formula v = nl.
* Maximum weightage is of Electromagnetic Waves.

1. Electromagnetic waves with wavelength:


(a) l1 are used to treat muscular strain.
(b) l2 are used by a FM radio station for broadcasting.
(c) l3 are used to detect fracture in bones.
(d) l4 are absorbed by the ozone layer of the atmosphere.
Identify and name the part of electromagnetic spectrum to which these radiation belong. Arrange these wavelengths
in decreasing order of magnitude.
[Ans. (a) l1 → Infra red radiations, (b) l2 → VHF radiowaves, (c) l3 → X-rays, (d) l4 → Ultraviolet rays. The
wavelengths in decreasing order of magnitude are: l2 > l1 > l4 > l3 ]
2. Identify the type of e.m. waves, whose method of production, is associated with
(a) a klystron value
(b) vibrations of atoms and molecules
(c) decay of atomic nuclei.
Also give the approximate range of wavelength of each of these e.m. waves.
[Ans. (a) Microwaves, l from 0.1 m to 1 mm, (b) Infrared radiation, l from 1 mm to 700 nm,
(c) Gamma rays, l < 10–3 nm]
3. In an electromagnetic wave propagating along the x-direction, the magnetic field oscillates at a frequency of 3 ×
1010 Hz and has an amplitude of 10–7 tesla, acting along the y-direction.
(a) What is the wavelength of the wave?
(b) Write the expression representing the corresponding oscillating electric field.
[Ans. (a) l = 100 cm (b) Ez= 30 sin 2p (3 ×1010t – 100x)Vm–1 ]
4. A radio can tune to any station in the 7.5 MHz to 12 MHz band. What is the corresponding wavelength band?
 [Ans. 40 m – 25 m]
5. The amplitude of the magnetic field part of a harmonic electromagnetic wave in vacuum is B0 = 510 nT. Waht is
the amplitude of the electric field part of the wave? [Ans. 153 NC–1]
6. Suppose that the electric field amplitude of an electromagnetic wave is E0 = 120 NC–1 and that its frequency is v =
50.0 MHz. (a) Determine, B0, w, k and l. (b) Find expressions for Ε and Β
[Ans. (a) B0 = 4 × 10–7 T, w = 3.14 × 108 rad s–1, k = 1.05 m–1, l = 6.00 m,

(b) Β = 4 × 10–7 sin (1.05x – 3.14 × 108t) j T]+
7. What is the cut-off frequency beyond which the ionosphere does not reflect electromagnetic radiations?
 [Ans. Ionosphere cannot reflect electromagnetic radiations having frequency higher than 40 MHz]
8. What is the nature of the waves used in radar? What is their wavelength range?
[Ans. Microwaves. These are electromagnetic waves of the wavelength range 10–3 m to 0.3 m.]

280 Physics–12
D:\EG_Physics-12_(26-06-2022)\Open_Files\Ch-8\Ch-8
\ 27-Jul-2022 Ved_Goswami Proof-4 Reader’s Sign _______________________ Date __________

ASSIGNMENT

I. Objective Type Questions (1 Mark)


1. Multiple choice questions:
 
(i) Expression  ∫ B ⋅ ds = 0 shows.
(a) Gauss’s law of electrostatics (b) Gauss’s law of magnetism
(c) Faraday’s law of e.m. induction (d) Modified Ampere’s circuital theorem
(ii) Light with an energy flux of 20 W/cm falls on a non-reflecting surface at normal incidence. If the surface has
2

an area of 30 cm2, the total momentum delivered (for complete absorption) during 30 minutes is
(a) 36 × 10–5 kg m/s. (b) 36 × 10–4 kg m/s.
(c) 108 × 104 kg m/s (d) 1.08 × 107 kg m/s.
2. Fill in the blanks:
(i) .................... produces magnetic field in the same way as does the conduction current.
(ii) .................... are transverse in nature.
II. Very Short Answer Type Questions (1 Mark)
3. Name the electromagnetic radiation which has the largest penetrating power.
4. The bombardment of a metal target, by high energy electrons, can result in the production of e.m. waves. Name
these produced waves.
III. Short Answer Type Questions-I (2 Marks)
5. State Maxwell equations.
6. How can we express mathematically a plane electromagnetic wave propagating along X-axis? Also represent it
graphically.
IV. Short Answer Type Questions-II (3 Marks)
7. Obtain an expression for the energy density of an electromagnetic wave.
8. What is meant by electromagnetic spectrum? Give its four uses.
9. Write one property and one use each of infrared rays, ultraviolet rays and radiowaves.
10. Identify the following electromagnetic radiations as per the frequencies given below. Write one application of each.
(a) 1020 Hz (b) 109 Hz (c) 1011 Hz


Electromagnetic Waves 281


D:\EG_Physics-12_(26-06-2022)\Open_Files\Ch-9\Ch-9
\ 27-Jul-2022 Ved_Goswami Proof-4 Reader’s Sign _______________________ Date __________

Topics Covered
9 Ray Optics

9.1 Reflection of Light by Spherical Mirror 9.2 Refraction and Total Internal Reflection
9.3 Refraction at Spherical Surfaces 9.4 Refraction Through a Prism
9.5 Optical Instruments

C hapter map
Ray Optics

Reflection by Convex Refraction Optical instruments


and Concave mirrors
Astronomical,
Compound Microscope Newtonion and
Mirror formula Magnification Cassegrain Telescope
v0  D
m=− 1 +  f  f 
Convex and Concave lens u0  fe  m = − 0 1 + e 
fe  D

Laws of Absolute and Relative Total Internal Reflection Refraction through glass slab
Refraction Refractive index (TIR) and Prism and combination of medium

Topic 1. Reflection of Light by Spherical Mirror


•• Reflection: The phenomenon of coming back of light reflection or refraction either actually meet or appear to
into the same medium after striking a surface capable of meet.
reflecting light is called reflection. (Sending it back) •• Relation between radius of curvature and focal
•• Laws of Reflection: length: Consider a concave
(i) Angle of incidence = mirror of radius of curvature R i
A
Angle of reflection i r and focal length f. C θ 2θ
r
P
(ii) Incident ray, reflected The incidence ray parallel to FN
ray and the normal at principal axis strikes the mirror
the point of incidence, all lie in the same plane. at A and is reflected along AF.
•• Concave Mirror: Concave Mirror is a part of hollow ∠i = ∠r (laws of reflection)
glass sphere whose outer surface is polished and the
∠i = ∠q (alternate interior angles)
reflection takes place from the surface which is caving in.
•• Convex Mirror: Convex mirror is a part of hollow glass So ∠r = ∠q
sphere whose inner surface is polished and reflection AN
In DANC tanq =
takes place on surface which is vexing out. NC
AN
Note : When a ray of light which strikes mirror through In DANF tan2q =
NF
centre of curvature, it will be normal to the _ q so
For small angles, tan q ∼
surface of mirror i.e., Angle of incidence = 0°, AN AN
_
q∼ ,
so angle of reflection is also 0°. NC 20  NF
Focus/Principal Focus: It is a point on principal axis at or NC = 2NF
which all the rays of light parallel to principal axis after For small aperture point. N → Point P.
282
D:\EG_Physics-12_(26-06-2022)\Open_Files\Ch-9\Ch-9
\ 27-Jul-2022 Ved_Goswami Proof-4 Reader’s Sign _______________________ Date __________

Thus PC = 2PF B′
i.e. R = 2f
Object B
•• For convex Mirror: To Prove R = 2f: Consider a convex
between P C
mirror of radius of curvature R and focal length f.
&F F A P A′
Incident ray parallel to principal axis strikes the mirror
at A and is reflected along AB, further projected to F.
∠i = ∠r (laws of reflection) Convex Mirror
∠i = ∠2 (corresponding angles)
⇒ ∠r = ∠2 B
∠r = ∠1 (vertically opposite angle) Object
B′
\ ∠1 = ∠2 N between F
B
So AF = FC r & infinity A F P A′ F C
For small aperture, point. i
A
A → point. P 1
So AF = PF P
2
F C \\\\

which gives PF = FC

\\ \
\\\\
\\\\\
PC = PF + FC

\\\\\\\\\\\\\\\\\\\\\\\
Object at
R = PF + PF = PF + PF infinity P F C
= 2PF = 2f.
•• (a) Concave Mirror:

\\\ \\\
\\\

\
\\

Object at (b) Mirror Position Position Nature of Image


infinity C F P
of object of image
Concave At infinity At focus Real, inverted
highly diminished
B Concave Beyond C Between Real, inverted
Object A′ C and F diminished
P
beyond C A C
B′
F
Concave At C At C Real, inverted,
same size
Concave Between Beyond Real, Inverted,
C and F C larger than object
B
Concave At focus Infinity Real, inverted,
Object at C A highly enlarged
F P
C Concave Between Behind Virtual, erect,
F and P mirror larger than object
Convex At infinity At F Virtual, erect,
Object B highly diminished
between F A′
P Convex Far from Between Virtual, erect,
&C C A F
mirror pole and diminished
B′
focus
Cartesian Conventions:
B Rules:
Object at A i
P 1. All distances to be measured from pole or optical
C r
Focus F centre.
2. Distances measured in direction of incident ray are
considered positive (+).

Ray Optics 283


D:\EG_Physics-12_(26-06-2022)\Open_Files\Ch-9\Ch-9
\ 27-Jul-2022 Ved_Goswami Proof-4 Reader’s Sign _______________________ Date __________

3. Distances measured opposite to incident ray are AP PF


considered negative (–). =
A ′P (PA ′ − PF)
4. Distances above principal axis are positive (+) and −u − f
below it are taken to be negative (–). =
−v − v − (− f )
•• Use of Mirrors:
u f
1. • Concave mirrors are used in search lights and =
v v− f
headlight of cars. Source of light (e.g., bulb) is
placed at focus of mirror so that reflected beam uv – uf = vf
becomes parallel so we get a strong, parallel beam. 1 1 1
Divide both sides by uvf − =
• Also used by dentist to focus light on tooth for f v u
examination. 1 1 1
= +
• Solar cookers: to get high intensity. f u v
Size of image
2. Convex mirrors are used in rear view mirror in •• Magnification: Magnification =
Size of object
automobile as they always form small and erect image hi −v
and has a larger field of view. Magnification = =
ho u
3. Spherical aberrations in a mirror: The inability of
where ho = height of object, hi = height of image
a spherical mirror of large aperture to bring all the
rays of wide beam of light falling on it to focus at hi − v
\ m= =
one point is called spherical aberration. ho u
m is +ve for virtual image.
Marginal rays
m is – ve for real image
•• Mirror Formula : Concave Mirror (Virtual Image)
B′
Paraxial rays
B L
F
C F A N P A′
•• Spherical Aberration: This Results in a Blurred Image
Methods of reducing aberrations:
1. By using mirrors of small aperture. Object AB placed between F and P of concave mirror.
2. Cut off the marginal rays Virtual, erect image A′B′ is formed behind the mirror.
3. Better method : Use parabolic mirror. DABP ~ DA′B′P
•• Mirror Formula : Concave Mirror (Real Image): An AB AP
= ...(1)
object AB is placed beyond A ′B′ A ′P
B L
centre of curvature of DLNF ~ DB′A′F
concave mirror. A′ LN NF
Real, inverted image A C F N P =
B′ A ′ FA′
A′B′ is formed between B′ For small aperture, NF ~ PF
F and C. AB PF
DABP ~ DA′B′P =  [ LN = AB] ...(2)
A ′B′ FA′
AB AP AP PF
=  ...(1) Comparing (1) and (2) =
A ′B′ A ′P A ′P FA′
DLNF ~ DB′A′F
AP PF
LN NF =
= A ′P (A ′P + PF)
A ′B′ FA′
For small aperture, NF ~ PF −u − f
=
AB PF v v + (− f )
=  ... (2) [ LN = AB] – u (v – f ) = – fv
A ′B′ FA′
AP PF – uv + uf = – fv
Comparing (1) and (2) = Divide by uvf throughout
A ′P FA′
284 Physics–12
D:\EG_Physics-12_(26-06-2022)\Open_Files\Ch-9\Ch-9
\ 27-Jul-2022 Ved_Goswami Proof-4 Reader’s Sign _______________________ Date __________

−1 1 −1 AB AP
+ = \   = ...(1)
f v u A ′B′ A ′P
1 1 1 DLNF ~ DB′A′F
+ = LN FN
v u f =
•• Mirror Formula (Convex Mirror) B′ A ′ A ′F
For small aperture → NF ~− PF
AB PF
=  [ LN = AB] ...(2)
L
A ′B′ A′ F
B AP PF PF
Comparing (1) and (2) = =
B′ A′ P A′ F (PF − A ′P)
−u f
A′ F C =
A P N v ( f − v)
– u(f – v) = fv
– uf + uv = fv
Consider an object AB placed in front of a convex mirror Dividing throughout by uvf
beyond F. Image formed is virtual, erect and magnified. −1 1 1 1 1 1
+ = or = +
DABP ~ DA′B′P v f u f v u

EXERCISE 9.1
I. Objective Type Questions (1 Mark) II. Very Short Answer Type Questions (1 Mark)
1. Choose the correct answers from the given options 1. An object AB is kept in front of a concave mirror as
(MCQs). shown in the figure A
(i) To get three images of a single object, one should (i) How will the position and
have two plain mirrors at an angle of intensity of the image be B C F
P
(a) 60° (b) 90° (c) 120° (d) 30° affected if the lower half
(ii) A ray of light strikes a silvered surface inclined to of the mirror’s reflecting
another one at an angle of 90°. Then the reflected surface is painted black? [AI 2012]
ray will turn through (ii) Complete the ray diagram showing the image
(a) 0° (b) 45° (c) 90° (d) 180° formation of the object.
2. Fill in the blanks. 2. Draw a ray diagram to show the image formation by
a concave mirror when the object kept between its
(i) A ray of light is incident on a plane mirror at an
focus and the pole. Using this diagram, derive the
angle of incidence 30°. The ray after reflection is
magnification formula for the image formed.
deviated through ................... .
[Delhi 2011]
(ii) A spherical mirror forms a real image of a point
object placed in front of it. The distance of the III. Short Answer Type Questions-I (2 Marks)
image and object from the mirror is 30 cm and 3. Use the mirror equation to show that:
30 cm respectively. The focal length and nature (i) An object placed between f and 2f of concave
of the mirror is ...................... . mirror produces a real image beyond 2f.
3. State True or False [Delhi 2015]
(i) The focal length (f) of spherical mirror of radius (ii) A convex mirror always produces a virtual image
R independent of the location of the object.
curvature R is .
2 [AI 2016]
(ii) Focal length of a concave mirror when it is dipped
(iii) An object placed between the pole and focus of
in water does not changes.
a concave mirror produces a virtual and enlarged
Assertion-Reason Questions (Instructions as usual) image. [NCERT][AI 2011]
4. Assertion (A): Aconvex mirror cannot form real images.
4. A 5 cm long needle is placed 10 cm from a convex
Reason (R): Convex mirror converges the parallel
mirror of focal length 40 cm. Find the position, nature
rays that are incident on it. [CBSE S.P. 2020-21]
Ray Optics 285
D:\EG_Physics-12_(26-06-2022)\Open_Files\Ch-9\Ch-9
\ 27-Jul-2022 Ved_Goswami Proof-4 Reader’s Sign _______________________ Date __________

and size of the image of the needle. What happens to IV. Short Answer Type Questions-II (3 Marks)
the size of the image when the needle is moved farther 8. A concave mirror produces a real and magnified image
away from the mirror? [S.P. 2011] of an object kept in front of it. Draw a ray diagram
5. If you sit in a parked car, you glance in the rear view to show the image formation and use it to derive the
mirror R = 2 m and notice a Jogger approaching. If mirror equation. [AI 2015]
the Jogger is running at a speed of 5m/s, how fast is 9. (i) Draw a ray diagram to show image formation
when the concave mirror produces a real, inverted
the image of the Jogger moving when the Jogger is
and magnified image of the object.
(a) 39 m (b) 29 m. [NCERT]
(ii) Obtain the mirror formula and write the expression
6. A mobile phone lies along the principal axis of a for the linear magnification. [Delhi 2018]
concave mirror as shown in figure. Show by suitable 10. Define the term ‘focal length of a mirror’. With the
diagrams the formation of its image. Explain why the help of a ray diagram, obtain the relation between its
magnification is focal length and radius of curvature. [Delhi 2020]
not uniform and B A 11. An object is placed in front of a concave mirror. It
distortion will P is observed that a virtual image is formed. Draw the
CC
occur depending
F ray diagram to show the image formation and hence
on the location of
B′ 1 1 1
A′ derive the mirror equation = + .[Delhi 2020]
the mobile w.r.t. f u v
the mirror. [NCERT][Delhi 2014] V. Long Answer Type Questions (5 Marks)
7. Calculate the distance of an object of height h from 12. Derive mirror equation for a convex mirror. Using it,
a concave mirror of radius of curvature 20 cm, so as show that a convex mirror always produces a virtual
to obtain a real image of magnification 2. Find the image, independent of the location of object.
location of image also. [AI 2016] [CBSE S.P. 2019-20]

Answers 9.1
I. Objective Type Questions Now, DAPB ~ DA′PB′
1. (i) (b) (ii) (d) A ′B′ B′ P
2. (i) 120° (ii) 120 mm, concave \ =
AB BP
3. (i) True (ii) True
Applying the new Cartesian sign convention, we have
4. (c) ‘A’ is true but ‘R’ is false. Convex mirror diverged
the light rays. A′B′ = –h2 [downward image height]
II. Very Short Answer Type Questions AB = +h1 [upward object height]
1. (i) If the lower half of the mirror’s reflecting surface is B′P = –v [image distance on left]
painted black, there will be no change in the position BP = –u [object distance on left]
of the image, but its intensity will get reduced. −h2 −v
(ii) Image formed will be inverted, diminished, between \ =
h1 −u
C and F.
Magnification,
A h2 v
m= =−
B′
P h1 u
B C F
1 1 1 1 1 1
= +   or = −
A′
3. (i) Mirror equation is
f v u v f u
2. Concave mirror: Figure shows the ray diagram for For a concave mirror, f is negative, i.e., f < 0.
the formation of image A′B′ of a finite object AB by a For a real object (on the left of mirror). u is negative.
concave mirror.
1
A M For u between f and 2f implies lies
u
B 1 1 1 1 1
B C F N P between and i.e., > >
f f 2f 2f u f
A v
u (as u, f are negative)

286 Physics–12
D:\EG_Physics-12_(26-06-2022)\Open_Files\Ch-9\Ch-9
\ 27-Jul-2022 Ved_Goswami Proof-4 Reader’s Sign _______________________ Date __________

1 1 1 1 1 1
or − < < (a) u = – 39 m, = +
2f u f f v u
1 1 1 1 1 1 1 1 1
or − < − < − So, = −
f 2f f u f f v f u
1 1 1 1 1 1 1 1 1 1 40
or < < 0 as = + i.e., is negative. = − =1+ =
2f v f v u v v 1 − 39 39 39
This implies that v is negative and greater than 2f. 39
v= m ½
This means that the image lies beyond 2f and it is 40
real. ½ Position of Jogger after 1 sec = – 39 + 5 = – 34 m
(ii) For a convex mirror, f is positive i.e., f > 0 uf
For a real object on the left u is negative. Value of position of Jogger after 1 sec, v1 =
u− f
1 1 1 1 1 1 − 34 × 1 34
= + implies = − v1 = = m
f v u v f u − 34 − 1 35
1 39 34
As u is negative and f is positive; must be positive, −
v v − v1 40 35 5 1
Speed = = = = m/s  ½
so v must be positive i.e., image is behind the mirror. Time 1 1400 280
Hence, image is virtual whatever the value of u may uf − 29 × 1 29
(b) u = –29 m, v = = = m ½
be. ½ u − f − 29 − 1 30
1 1 1 After 1 sec u = – 29 + 5 = – 24 m
(iii) For a mirror, = −  ....(1)
v f u uf − 24 × 1 24
   v1 = = = m ½
For a concave mirror, f is negative f < 0 u − f − 24 − 1 25
As u is also negative, so f < u < 0 29 24

1 1 1 v − v1 30 25
This implies, − > 0 as − is a +ve quantity Average speed = =
f u u t 1
1 145 − 144 1
So, > 0 or v is positive i.e., image is on the right = = m/s  ½
v 150 150
and hence virtual. ½ Though the Jogger is moving with a const speed,
v f but the speed of the image is increasing.
Magnification, m = − = − 6. The part of the phone which is at C will have its image
u u− f
at C and of the same size as the object. The other end
As u is negative and f is positive, magnification
of the phone is placed between F and C. So it will be
f slightly magnified and formed beyond C (See figure in
m= > 1 i.e., image
f −u question). 1
is enlarged. ½ Thus the different part of the phone are magnified by
4. h0 = 5 cm, u = – 10 cm, f = + 40 cm different amount because of their different positions
1 1 1 1 1 1 +1 1 +1+ 4 5 from the mirror. 1
= + ⇒ = − = + = = R
f v u v f u 40 10 40 40 7. f = = – 10 cm, m = – 2
v = 8 cm 1 2
−v −v
As v is + ve, so the image is virtual and erect. Concave mirror m = ,–2 ,
u u
h −v hI −8 v = 2u ½
Magnification m = I = or = = 0.8
hO u 5 − 10 1 1 1
     = +  ½
hI = 0.8 × 5 = 4 cm 1 f v u
As the needle is moved farther away from the mirror, −1 1 1 1+ 2 3
    = + = =
the image will move towards the focus and the size will 10 2u u 2u 2u
decrease. ½ − 3 × 10
u= = – 15 cm  ½
R 2 2
5. Convex mirror, = f = =1m v = 2u = 2 × – 15 = – 30 cm ½
2 2
Ray Optics 287
D:\EG_Physics-12_(26-06-2022)\Open_Files\Ch-9\Ch-9
\ 27-Jul-2022 Ved_Goswami Proof-4 Reader’s Sign _______________________ Date __________

8. An object AB is placed B DA′B′F ~ DMPF


in front of concave B′ A ′ B′ F
mirror. A′ =
C A F
P     PM FP
Real, inverted image
B′ A ′ B′ F
A′B′ is formed beyond or   =  …(1)
C.
B′ AB FP
   DA′B′P ~ DABP (∠i = ∠r)
DABP ~ DA′B′P
B′ A ′ B′ P
AB AP \   =  …(2)
=  ...(1) ½ BA BP
A ′B′ A ′P From Eq. (1) and Eq. (2)
DLNF ~ DB′A′F
B′ F B′ P
LN NF =
=     FP BP
A ′B′ FA ′
For small aperture, NF ~− PF B′ P − FP B′ P
=  1
AB PF FP BP
=  ...(2) ½ Applying using conventions
A ′B′ FA ′     PF = –f, PB′ = –v, PB = u
Comparing (1) and (2) −v + f −v v− f v
AP PF PF \ = or =
= =  ½ −f −u f u
A ′P FA ′ (PA ′ − PF)
or vu – fu = fv or vu = fv + fu
−u − f u f dividing both sides by uvf
= or =
− v − v − (− f ) v v −f
1 1 1
uv – uf = vf  ½ or = +  1
f u v
Divide both sides by uvf,
1 1 1 1 1 1 This is mirror formula.
− = or = +  ½ Linear magnification (m)
f v u f v u
9. (i) ½ height of the image h′
m= =
height of the object h
Using sign convention
−h′
m=
   +h

\ from Eq. (2)
−h′ v
m= =  1
h u
10. Focal length: It is the distance between the focus and
the pole of the mirror.
(ii) AB is the object placed beyond C whose image A′B′
is formed in between F and C. Relation between f and R: Consider a ray AB parallel
to the principal axis, incident at point B of a spherical
A M
concave mirror of small aperture. After reflection from
the mirror, this ray converges to point F. Obeying the
laws of reflection. Thus F is the focus of the mirror, C
is the centre of curvature, CP = radius of curvature and
BC is a normal to mirror at point B.
B′ F i According to the law of reflection,
C P
B
∠i = ∠r
r

As AB is parallel to PC,
A′
∠a = ∠i
(b) \ We get ∠r = ∠a using DBFC

Hence CF = FB
288 Physics–12
D:\EG_Physics-12_(26-06-2022)\Open_Files\Ch-9\Ch-9
\ 27-Jul-2022 Ved_Goswami Proof-4 Reader’s Sign _______________________ Date __________

For a mirror of small aperture, – u(v – f) = – fv


FB ~ FP   ∴ CF ~ FP – uv + uf = – fv
Hence CP = CF + FP = FP + FP = 2 FP Divide by uvf throughout
R −1 1 −1
or R = 2f or f = + =
2 f v u
A 1 1 1
B
i + =
r v u f
P 12.

C F M
f A
R A′
11. Mirror Formula: Concave Mirror (Virtual Image) 1
B B′ F C
Object AB placed between F and P of concave mirror.
Virtual, erect image A′B′ is formed behind the mirror N
B

B L Deduction of mirror formula


1 1 1
+ =  [See theory last point] 1+1
C F A N P A v u f
For a convex mirror f is always +ve.

∴ f> c
DABP ~ DA′B′P
Object is always placed in front of mirror hence u < 0
AB AP
=  ... (1) (for real object)
A ′B′ A ′P 1 1 1
DLNF ~ DB′A′F + =
LN NF v u f
= 1 1 1
B′ A ′ FA′ = −  1
For small aperture, NF ~− PF v f u
AB PF As u < 0 i.e. u is –ve
=  ...(2) 1
A ′B′ FA′ hence, >0
Comparing (1) and (2) v
AP PF PF fi v > 0 i.e. +ve for all values of u.
= =
A ′P FA′ (A ′P + PF) Image will be formed behind the mirror and it will be
−u − f virtual for all values of u. 1
=
v v + (− f )

Topic 2. Refraction and Total Internal Reflection


•• Refraction: The phenomenon of change in path of light sin i m
as it passes obliquely from one transparent medium to = 2
sin r m1
another is called refraction. or m1 sin i = m2 sin r
•• Laws of refraction: •• Absolute Refractive Index: It is ratio of speed of light
1. Incident ray, refracted ray i (1) in vacuum to speed of light in the given medium.
and the normal at the point c
m=
of incidence lie in the same v
plane. r
(2) Note: When light undergoes reflection or refraction, the
frequency of light does not change because frequency is
2. The ratio of sin i to sin r, is
characteristic of source.
always constant for a given
colour of light and is called the refractive index of c v λ ( vac.) λ vac c
m= = = =
second medium with respect to first medium. v v λ ( med.) λ med vm

Ray Optics 289


D:\EG_Physics-12_(26-06-2022)\Open_Files\Ch-9\Ch-9
\ 27-Jul-2022 Ved_Goswami Proof-4 Reader’s Sign _______________________ Date __________

•• Relative Refractive Index: It is the ratio of refractive sin r


index of one medium to the refractive index of another For Refraction at face RS gµa =  ...(2)
sin e
medium. 1 sin r
•• Cause of Refraction of Light: As light travels with We know gma =
=  ...(3)
µ
a g sin i
different speed in different medium, so it bends while
sin r sin r
going into one medium from another. Comparing (2) and (3) =
sin e sin i
Refractive index specifies degree of bending of ray of sin e = sin i
light when going from one medium to another. ∠e = ∠i
•• Denser Medium (Optically): N So emergent ray is parallel to incident ray.
More bending of light, ∠r is •• To Find Lateral Displacement : (Refer to Previous
less, refractive index, is more, ∠i (1)
Figure): When light passes through a glass slab,
speed is lower. emergent ray is parallel to incident ray but it is displaced
•• Optically Rarer Medium: sideways, i.e., there is lateral displacement.
∠r
Lower bending of light, ∠r is (2)
CE
more, refractive index is lesser, In DCEB sin ( i – r) =
CB
speed is more. CE = CB sin (i – r) ...(1)
•• Principle of Reversibility of Light: If BN ′
the final path of a ray of light after it has In DBN′C, cos r =
BC
suffered several reflections or refractions BN ′ t
is reversed, it retraces its path. BC = = = t sec r ...(2)
cos r cos r
1 Using BC from (2) in (1), CE = CB sin (i – r)
To Prove : 1µ 2 =
2 m1 d = t sec r sin (i – r)
If light goes from medium one to medium two then  
µ1 sin i = µ2 sin r  cos i 
We can show that d = t sin i 1 − 1 
µ 2 sin i  (µ 2 − sin 2 i ) 2 
1µ 2 =
= ...(1)  
µ1 sin r
= t sin i [1 – x]
On reversing the path of the ray of light cos i
µ2 sin r = µ1 sin i where x= 1

µ1 sin r (µ 2 − sin 2 i ) 2
2µ 1 = µ
= ...(2) Factors on which lateral displacement (d) depends:
sin i
2 (i) Thickness of glass slab i.e. d ∝ t
1 (ii) Angle of incidence i.e. d ∝ sin i
2µ 1 =
1 m2 (iii) Refractive index i.e. d ∝ µ
(If µ is large, cos i divided by larger no, 1 – x will be
•• Refraction through Glass Slab: To show that emergent
larger no.)
ray is parallel to incident ray. Consider a glass slab of
•• Refraction Through Combination of Mediums:
thickness (t).
N
A Air
i1
i
P B Q
i- r1
r M Water
t r E
r r1
R
S C e D
N′
r2
We have to show that ∠i = ∠e Glsss r2
CE = lateral displacement = d
For Refraction at face PQ
sin i i1
aµg = sin r  ...(1) Air

290 Physics–12
D:\EG_Physics-12_(26-06-2022)\Open_Files\Ch-9\Ch-9
\ 27-Jul-2022 Ved_Goswami Proof-4 Reader’s Sign _______________________ Date __________

sin i1 sin r1 sin r2 Total Internal Reflection : Phenomenon in which a ray


aµw = , wµg = , gµa = of light travelling at an angle of incidence greater than
sin r1 sin r2 sin i1
the critical angle from denser to a rarer medium is totally
sin i sin r1 sin r2 reflected back into denser medium is called total internal
= aµw × wµg × gµa = × × =1
sin r1 sin r2 sin i reflection.
1 a mg •• Condition for Total Internal Reflection
wµg = =
g a × aµw
µ a mw (i) Light must travel from denser to rarer medium.
•• Relation Between Real Depth and Apparent Depth: (ii) ∠i > ∠ic (in denser medium)
Consider an object O placed in a medium of refractive
ic = critical angle; i = incident angle
index µ, say water of refractive index aµw.
•• Relation Between Critical Angle and Refractive Index
Rarer (R.I.):
Real depth = AO
r
A Apparent depth = AO′
2. Rarer (air)
r B
i
O′
i
Denser medium
ic
O 1. Denser
AB
In DOAB sin i = (glass)
OB
AB
In DO′AB sin r =
O ′B Consider a light going from medium 1 to medium 2.
sin i AB O ′B O ′B Medium 1 is denser than medium 2.
= w µa = × =
sin r OB AB OB sin ic sin ic
For normal viewingpoint B → point A 1µ2 =
=
sin r sin 90°
O ′A 1 1
wµa = OA =
1 m2 sin ic
1 OA Real depth
aµw = = = 1
µ O ′ A Apparent depth 2µ1 =
w a
If real depth increases, apparent depth will also increase, sin ic
1
to keep the ratio constant If Light is going from glass to air then aµg =
sin ic
Shift in position = Real depth – Apparent depth 1
      = OA – O′A = OO′ 1.5 = ⇒ sin ic = 0.66
sin ic
If real depth = t = thickness [ R.I. of glass w.r.t. air aµg = 1.5]
 t  1 ic = 41.8° ≈ 42°.
\   OO′ =  t −  = t 1 − 
 µ  µ •• Optical Fibre: It is a hair-thin long strand of quality
Shift depends upon thickness of medium and refractive quartz or glass coated with a material of slightly lower
index of the medium. refractive index. It works on the principle of total internal
•• Total Internal Reflection: reflection.
Critical angle: That angle of incidence in a dense
Construction: It consists of three main parts: (i) Core,
medium for which angle of refraction is 90° is called
(ii) Cladding, (iii) Buffer.
critical angle.
Core: Central cylindrical core is made of high quality
glass, plastic, silica of refractive index µ2.
Rarer Cladding: Core is surrounded by a glass or plastic
r1 r2 90°
jacket of refractive index m1 with m2 > m1. In a typical
Denser i1 i2 ic i > ic optical fibre, refractive indices of core and cladding
is nearly 1.52 and 1.48 respectively.
Buffer Coating: It is used to provide safety and
i << ic i < ic i = ic strength to the core and cladding of optical fibre.
Ray Optics 291
D:\EG_Physics-12_(26-06-2022)\Open_Files\Ch-9\Ch-9
\ 27-Jul-2022 Ved_Goswami Proof-4 Reader’s Sign _______________________ Date __________

Example. Invert an image with deviation of rays through


180°.
Cladding P
Core
Q A
1

Q′ 45°
2 45°
P′
45°

45°
C B
Example. Invert an image without deviation of rays.
Core

A B′
Cladding

Buffer Coating B A′
Brilliancy of diamonds is due to total internal reflection.
As the refractive index of diamond is very large, its
critical angle is very small, about 24.4°. The faces of
a diamond are so cut that the light entering the crystal 45° 45°
suffers total internal reflection repeatedly and hence it
Example. Refraction of light affects length of the day.
gets collected inside and it comes out through only a
Apparent shift in position of sun at sunrise and sunset. Due
few faces.
to atmospheric refraction, the sun is visible before actual
If m = 2.42
sunrise and after actual sunset.
1 1
sin ic = = = 0.413 ⇒ ic = 24° With altitude, density and hence refractive index of air
µ 2.42 layers decreases. Light rays starting from the sun travel from
Hence, the diamond sparkles when seen in the direction rarer to denser medium (layers). They bend more and more
of emerging light. towards the normal. But an observer sees an object in the
Examples based on Total Internal Reflection (TIR) direction of the rays reaching his eyes. So, to an observer
Prism and Refraction standing on earth, the sun appears to be in a position S′ (even
Example. Ray diagram using prism to deviate a ray through though its actual position is S) above horizon. The apparent
90°. shift in direction of sun is by about 0.5°. Thus, sun appears
Right angled isosceles prism. Critical angle for glass is to rise early about 2 minutes and set later by 2 minutes.
nearly 42°. Apparent
A S′ Position of Sun Atmosphere
45°
Q
Horizon
P
45°
Observer
45°
S Actual
Position of Sun Earth
S 45°
45°
Example. Sun near horizon appears to be flattened during
90° sunrise and sunset.
B R
C The sun near the horizon appears flattened due to
atmospheric refraction. The density and refractive index
P′ S′
of the atmosphere decreases with altitude, so the rays

292 Physics–12
D:\EG_Physics-12_(26-06-2022)\Open_Files\Ch-9\Ch-9
\ 27-Jul-2022 Ved_Goswami Proof-4 Reader’s Sign _______________________ Date __________

from the top and bottom position of the sun on the horizon totally reflected. These rays then move up through layers of
are refracted by different degrees. This causes apparent increasing refractive index and therefore undergo refraction
flattening of the sun. But the rays from the sides of the sun in a direction opposite to that in the first case. These rays
on a horizontal plane are generally refracted by the same reach the observer’s eyes and he sees an inverted image of
amount. So the sun still appears circular along its sides. the object, as if it was formed in a pond of water.
Example. Image formation in a desert.
Mirage: It is an optical illusion observed in deserts or
over hot extended surfaces, due to which a traveller sees a
shimmering of water some distance ahead of him.
The surrounding objects appear inverted. On hot summer
}} Cold Air
(Denser)

Hot Air
day, surface of earth becomes very hot. Layer of air over (Rarer)
the earth are more heated than the higher ones. Hence the
density and refractive index of air layers increase as we
move high up. As the rays of light from a distant object like
a tree travels towards the earth through layers of decreasing
refractive index, they bend more and move away from
the normal. A stage is reached when ∠i > ∠ic the rays are

EXERCISE 9.2
I. Objective Type Questions (1 Mark) 5. Assertion (A): Optical fibres are used for
1. Choose the correct answers from the given options telecommunication.
(MCQs). Reason (R): Optical fibres are based on the
(i) The principle behind optical fibre is phenomenon of total internal reflection.
(a) total external reflection II. Very Short Answer Type Questions (1 Mark)
(b) total internal reflection 1. Can the absolute refractive index of a medium be less
(c) Both (a) and (b) (d) Diffraction than unity?
(ii) Which of the following is used in optical fibre? 2. Can the refractive index of a medium be less than
(a) Total internal reflection unity?
(b) Scattering (c) Reflection 3. When monochromatic light travels from one medium
to another, its wavelength changes but frequency
(d) Interference
remains the same. Explain [Delhi 2011]
2. Fill in the blanks.
4. A beam of light converges at a point on the screen.
(i) A bird flying high in the air appears to be A plane parallel glass plate is introduced in the
................... as the refraction takes place from path of this converging beam. How will the point
rarer to denser medium. of convergence be affected? Draw the relevant ray
(ii) In glass violet light travels .............................. than diagram.
red light.
5. When light travels from an optically denser medium
(iii) Out of red, blue and yellow light, the scattering to a rarer medium, why does the critical angle of
of ................. light is maximum. [AI 2020] incidence depend on the colour of light? [AI 2015]
3. State True or False 6. Name one phenomenon which is based on total
(i) Blue colour of sky is due to phenomenon of internal reflection. [AI 2016]
scattering.
III. Short Answer Type Questions-I (2 Marks)
(ii) A fish at a depth of 12 cm in water is viewed by
an observer on the bank of a lake. Height of the 7. For the same angle of incidence, the angles of
image of fish raised by 12 cm. (µ = 4/3) refraction in three different media A, B and C are
15°, 25° and 35° respectively. In which medium the
Assertion-Reason Questions (Instructions as usual)
velocity of light is minimum? [AI 2012]
4. Assertion (A): The diamond shines due to multiple
total internal reflection. 8. Why does a ray of light bend towards normal as it
goes from air to glass?
Reason (R): The critical angle of diamond is 24.4°.

Ray Optics 293


D:\EG_Physics-12_(26-06-2022)\Open_Files\Ch-9\Ch-9
\ 27-Jul-2022 Ved_Goswami Proof-4 Reader’s Sign _______________________ Date __________

9. (a) State the conditions for total reflection to occur 15. The figure shows a ray of light falling normally on
[Delhi 2013, 2016, 2019] the face AB of an
A
(b) Write the relation between the refractive index and equilateral glass prism
critical angle for a given pair of optical media. having refractive index
[Delhi 2013, 2019] 3/2, placed in water of
10. (a) For a ray of light travelling from a denser medium refractive index 4/3. Will
of refractive index n 1 to a rarer medium of this ray suffer total internal
n2 reflection on striking the
refractive index n2, prove that = sin ic is the C
n1 face AC? Justify your B
critical angle of incidence for the media. answer.  [Delhi 2018]
(b) Explain with the help of a diagram, how the above IV. Short Answer Type Questions-II (3 Marks)
principle is used for transmission of video signals 16. In the figure given below, three light rays red (R),
using optical fibres. [Delhi 2008, 2016] green (G) and blue (B) are incident on an isosceles
11. A small bulb is placed at the bottom of a tank right-angled
containing water to a depth of 80 cm. What is the area
a
prism abc at face
of the surface of water through which light from the
ab. Explain with
bulb can emerge out? Refractive index of water is 1.33.
(Consider the bulb to be a point source.) [NCERT] reason, which B
G
ray of light will R
12. A right angled prism made from a material of
refractive index m is kept in air. A be transmitted
A ray PQ is incident on side AB through the face
normally as shown here. Find in ac. The refractive 45°
terms of m the maximum value P Q index of the b c
i R
of q upto which the incident ray prism for red, green, blue light are 1.39, 1.44, 1.47
necessarily undergoes total respectively.
internal reflection at the face AC. Trace the path of rays after passing through face ab.
θ
 [S.P. 2015] B C
[AI 2008, Delhi 2009, Foreign 2011]
13. A ray PQ is incident normally on the face AB of a
triangular prism of 17 State the conditions of total internal reflection.
A
refracting angle of 60°, Refractive indices of the given prism material for
made of a transparent P 60° Red, Blue and Green colours are 1.39, 1.48 and 1.42
material of refractive Q respectively. Trace the path of rays through the prism.
2  [CBSE S.P. 2019-20]
index , as show in
3 A
the figure. Trace the path
of the ray as it passes B C G
through the prism. Also calculate the angle of B
emergence and angle of deviation. [Delhi 2014 C] R
14. A ray of light is incident normally on the face AB of
a right-angled glass
45°
prism of refractive A B B C
60°
i n d e x am g = 1 . 5 . 18. Calculate the angle of emergence (e) of the ray of light
The prism is partly incident normally on the face AC
immersed in a liquid of a glass prism ABC of refractive
of unknown refractive
index 3 . How will the angle of
index. Find the value
of refractive index emergence change qualitatively,
of the liquid so that if the ray of light emerges from
the ray grazes along the prism into a liquid of refractive
the face BC after index 1.3 instead of air?
refraction through the  [AI 2020]
prism. [AI 2015] C
294 Physics–12
D:\EG_Physics-12_(26-06-2022)\Open_Files\Ch-9\Ch-9
\ 27-Jul-2022 Ved_Goswami Proof-4 Reader’s Sign _______________________ Date __________

Answers 9.2
I. Objective Type Questions Thus a ray of light bends towards normal as it goes from
1. (i) (b) (ii) (a) air to glass. 1
2. (i) higher than in reality (ii) slower 9. (a) Conditions for total internal reflection are :
(iii) Blue 3. (i) True (ii) False (i) Light must travel from denser to rarer medium.
4. (b) 5. (a)  ½
II. Very Short Answer Type Questions (ii) Angle of incidence in denser medium must be
greater than critical angle (ic). ½
c Speed of light in vacuum
1. No, as m = = as 1
v Speed of light in medium (b) µ = , i is critical angle. m is refractive index
c > v so m > 1. sin ic c
Speed of light in medium 1 v1 of the denser medium w.r.t. rarer medium. 1
2. Yes, as 1m2 = = . 10. (a) When a ray of light goes from a denser medium of
Speed of light in medium 2 v2
refractive index n1 to a rarer medium of refractive
If v1 < v2 , i.e. medium one is denser as compare to
index n2 and the angle of incidence is equal to critical
medium 2 then µ < 1. e.g. µ for H2O w.r.t glass in less
angle ic, then angle of refraction will be 90°.
than one.
3. When monochromatic light travels from one medium
to another its wavelength and speed both change such Rarer (n2)
v v
that 1 = 2 = frequency. 90°
λ1 λ 2 ½
ic
So, frequency remains unchanged.
Denser (n1)
4. The point of convergence will shift away from the glass
plate, as shown in the ray diagram given below. The
screen has to be moved towards right to receive the According to Snell’s law, n1 sin ic = n2 sin 90° or
point of convergence again. n2
= sin ic ½
Original Screen n1
point of (b) Optical fibre: It is a hair-thin long strand of good
convergence
quality quartz or glass coated with a material of
slightly lower refractive index. It works on the
principle of total internal reflection. Video signal
can be transmitted through it with a negligible loss
Point of
convergence in signal strength. A large number of optical fibres
Glass plate
with glass plate are held together to form a light pipe. ½
1 When video signals modulated with light is incident
5. As sin ic = on its one face at a small angle of incidence, it
m
As m, the refractive index depends on the colour of light suffers refraction from air to quartz and strikes the
quartz surface at an angle > ic and thus it suffers
so ic will also depend on the colour of light.
total internal reflection multiple times as both the
6. Optical fibres used in endoscopy in medical science are conditions of total internal reflection are satisfied.
based on total internal reflection. The signal finally comes out at the other end.
sin i c
7. From Snell’s law µ = = Coating (n1)
sin r v
sin r n2
n2 > n1
or v = c  1
sin i ½
For given i, v ∝ sin r, r is minimum for medium A, so
velocity of light is minimum in medium A. 1
A
sin i c Coating B
8. As m = = , m is refractive index of
sin r v 11. Light from the source will come out through a circular
glass w.r.t. air as c > v. So sin i > sin r. i.e., ∠i > ∠r.1
path of radius r = OB = OA

Ray Optics 295


D:\EG_Physics-12_(26-06-2022)\Open_Files\Ch-9\Ch-9
\ 27-Jul-2022 Ved_Goswami Proof-4 Reader’s Sign _______________________ Date __________

14.
A O B
A B
60°
ic
µg
½
h ic

ic ic

30°
60°
S ½
1 1 3
sin ic = =   ½
µ 1.33 4 µw

OS = h = 80 cm = 0.8 m,
OB = h tan ic
C
2
 3 7 mg sin 60° = mw sin 90° ½
cos ic = 1 −   = 
 4 4 3
1.5 × = µ w × 1 or µw = .75 × 1.732 = 1.299 1
sin ic 3 2
=
tan ic =
cos ic 7 ½ 15. In the prism QR ray makes an angle of 60° with the
Area of the path = p(OB)2 = p(h tan ic)2 normal, i.e. angle of incidence in the denser medium
2
is 60° i.e. ∠i = 60°.
 3  This ray is going from glass prism to water. ½
= 3.14 (0.8) 2 
 7  3
= 2.58 m2 ½ The refractive index of prism =
2
π π 4
12. ∠i = ∠A, θ = − A = − i  ½ Refractive index of water =
2 2 3
For light to undergo TIR, i > ic ½ For total internal reflection, angle of incidence and angle
of refractions (emergence) = 90°.
So, the maximum value of q corresponds to minimum A
value of i which is ic P 60°
π π 1
So θ max = − ic = − sin −1  1 Q
2 2 µ 30 ½
° R
13. A I 60°

P 60° B C

Q
Applying Snell’s Law
½ 4 2 8
µg sin ic = µw sin 90° or sin ic = × =
3 3 9
30
°

60° or sin ic = 0.88 ½


But in question angle of incidence in prism is 60° i.e.
3
B C  sin 60° = = 0.816
2
2 This angle of incidence is lesser than critical angle.
A = 60°, µ = Hence, No TIR (Total Internal Reflection). ½
3
1
1 3 16. As µ =  ½
Using sin ic = =  ½ sin ic
µ 2
so m for critical angle of 45° is
So     ic = 60°
1
On face AC, i = 60°, so r = 90°   µ = = 2 = 1.414  ½
sin 45°
Angle of emergence = 90° ½
Angle of incidence for all colours is 45°.
Angle of deviation = 30° ½
296 Physics–12
D:\EG_Physics-12_(26-06-2022)\Open_Files\Ch-9\Ch-9
\ 27-Jul-2022 Ved_Goswami Proof-4 Reader’s Sign _______________________ Date __________

a (b) i > ic
45°
1
 Sin ic =
 ½
B m
G
R
½  1 
∴ (ic)Red = Sin–1  = 46°
 1.39 
45° R
 1 
b c
(ic)Green = Sin–1  = 44.8°
 1.42 
B G
• The ray will be allowed to pass through ac if i < ic.  1 
(ic)Blue = Sin–1  = 43° 1
Red colour: m = 1.39,  1.48 
1  Angle of incidence at face AC is 45° which is
sin ic = , i > 45°, i.e., ic > i, or i < ic
1.39 c more than the critical angle for Blue and Green
Then it will be refracted. ½ colours therefore they will show TIR but Red colour
Green colour : m = 1.44,
will refract to other medium.
1
sin ic = , i < 45°, i.e., ic < i, or i > ic 18. As per Snell’s law µ1 sin (i) = µ2 sin (r)
1.44 c
Then TIR will take place. ½ µprism = 3 and µ2 = 1 (air)
Blue colour : m = 1.47, µprism sin (30°) = 1 × sin (e)
1
sin ic = , i < 45°, i.e., ic < i, or i > ic 1
1.47 c 3 × = sin (e) ⇒ e = 60° 1½
Then TIR will take place. ½ 2
Now when the external medium is changed to liquid of
17. Two conditions for T IR –
(a) Light must travel from denser to rarer medium ½ µL = 1.3 then,
µprism sin (30°) = µL sin (e)
3 sin (30°) = 1.3 sin (e)
 3 
1 e = sin–1 
 = 41.82° 1½
 2 ×1.3 
Hence, the angle of emergence (e) reduces to 41.82°
from 60°.

Topic 3. Refraction at Spherical Surfaces


•• Refraction at Spherical Surfaces: µ1 (rarer)
A µ2 (denser)
Pole : It is the centre of the refracting surface i
r
Centre of Curvature : Centre of sphere of which the
α γ β
surface is a part. O P N C I
Principal axis : Line joining the pole and the centre of
curvature extended on both sides.
•• Refraction at Convex Surface: Object placed in rarer
medium, real image. Consider a spherical surface of radius of curvature R.
Assumptions in refraction at spherical surfaces. A point object O is placed on principal axis in rarer
– Object is point object. medium of refractive index m1.
– Angle of incidence is small. m1 < m2
– Aperture of surface is small. m1 sin i = m2 sin r

Ray Optics 297


D:\EG_Physics-12_(26-06-2022)\Open_Files\Ch-9\Ch-9
\ 27-Jul-2022 Ved_Goswami Proof-4 Reader’s Sign _______________________ Date __________

m2 sin i ~ i m2r = m1i ...(1)


= − In DACO, i= a+g ...(2)
m1 sin r r
[For small angles sin i = i] In DACI, r= g+b ...(3)
m2r = m1i ...(1) Substituting values of i and r in (1)
In DACO, i = a + g...(2) (i is exterior angle) m2 (g + b) = m1 (a + g) ...(4)
In DACI, g= r+b (g is exterior angle) AN
tan a =
r= g–b ...(3) ON
AN
Substituting values of i and r in (1) tan b =
NI
m2 (g – b) = m1 (a + g) ...(4)
AN
AN tan g =
tan a = NC
ON For small angles, tan a ~− a, tan b ~− b, tan g ~− g.
AN
tan b =  AN AN   AN AN 
Eqn. (4) becomes, µ 2  + = µ1  +
NI  NC NI   ON NC 
AN
tan g = For small aperture, point. N → point. P
NC µ2 µ2 µ µ
For small angles, tan a ~− a, tan b ~− b and tan g ~− g. + = 1 + 1
PC PI OP PC
 AN AN   AN AN  µ2 µ2 − µ1 µ1
Eqn. (4) becomes m2  − = m1  + +
 NC NI    NI NC 
R

v
=
u R
For small aperture, point N → point P. µ 2 µ1 µ 2 − µ1
− =
µ2 µ2 µ1 µ v u R
− = + 1 •• Refraction at convex surface,object in denser
PC PI PO PC
µ2 µ2 −µ1 µ1 medium, image is real:
− = + n
R v u R 2 (denser)
A
µ1 (rarer)
r
µ1 µ µ1 µ 2
fi − 2 = − i
u v R R
µ2 µ µ 2 µ1
− 1 = −
  
R R C N P
v u O I
µ2 µ µ 2 − µ1
− 1 =
v u R
•• Refraction at Convex Surface: Object in rarer
medium, image is virtual Consider an object (point obj.) O placed on principal axis
M
A
of refracting surface. Object is placed in denser medium.
µ1 (rarer) i µ2 (denser) Real image is formed in the rarer medium.
r
Using Snell’s law m2 sin i = m1 sin r
β α γ m2 sin r
=
I O P N C m1 sin i
For small angles, sin i ~− i, sin r ~− r.
m2 r
=
m2 > m1 m1 i
Consider a spherical surface of radius of curvature R. m2i = m1r ...(1)
A point object O is placed on principal axis in rarer In DACO, g= i+a
medium of refractive index m1. i= g–a ...(2)
Using Snell’s law, m1 sin i = m2 sin r In DACI, r= g+b ...(3)
m2 sin i Substituting i and r values in (1), we get
= m2 (g – a) = m1(g + b) ...(4)
m1 sin r
For small angles, sin i ~− i, sin r ~− r AN AP
=
a ~− tan a =
m2 i NO PO
= [For small aperture pt. N → pt P.]
m1 r

298 Physics–12
D:\EG_Physics-12_(26-06-2022)\Open_Files\Ch-9\Ch-9
\ 27-Jul-2022 Ved_Goswami Proof-4 Reader’s Sign _______________________ Date __________

AN PA AN1
b ~− tan b = = b1 ~− tan b1 =
NI PI N1I1
AN PA AN1
g ~− tan g = = g1 = tan g1 =
NC PC N1C1
 PA AP   PA AP  As the lens is thin, all the distances measured from its
Eqn. (4) becomes µ 2  − = µ1  +
 PC PO    PC PI  surface may be taken equal to those measured from its
µ2 µ2 µ µ optical centre.
− = 1 + 1
PC PO PC PI  1 1   1 1 
µ µ µ1  + = µ2  −

µ2 µ2
− = 1 + 1  N1O N1C1    N1C1 N1I1 
− R −u −R v µ1 µ µ µ
µ1 µ 2 µ1 + µ 2 + 1 = 2 − 2
fi − = PO PC1 PC1 PI1
v u R
µ1 µ1 µ2 µ2
•• Lens Maker Formula: + = −
− u R1 R1 v1
Assumptions: µ − µ1
µ 2 µ1
– Object is point object lying on principal axis. − = 2 ...(A)
v1 u R1
– Lens used is thin, so that the distances measured from
For refraction at XP2Y, I1 will act as virtual object. The
its surface may be taken equal to those measured from
incident ray is in the denser medium. Refraction is taking
optical centre.
place at a convex surface.
– Aperture of the lens is small.
As light is going from denser to rarer medium, it bends
– All the rays are paraxial, i.e., they make small angle away from the normal. Final image is formed at I.
with the normal to the lens faces and to the principal
µ1 µ 2 µ1 − µ 2
axis. − = ...(B)
X v v1 R2
M′ µ µ µ − µ1 µ1 − µ 2
A Adding (A) and (B) 1 − 1 = 2 +
v u R1 R2
i1 i B
r1
µ1 µ1  1 1 
− = (µ 2 − µ1 )  −
α P2 γ1 β1 v u  R1 R 2 
O C2 P1 N1 P C1 I I1
1 1  µ2  1 1 
Dividing both sides by m1, − =  − 1  −
v u  µ1   R1 R 2 
1  µ2  1 1 
=  − 1  −
Y f  µ1   R1 R 2 
Consider a thin convex lens with point object O placed •• Spherical Lens: It is a part of transparent medium
on its principal axis. bounded by two surfaces, atleast one of which has to be
Let R1 = radius of curvature of surface XP1Y a part of a sphere.
R2 = radius of curvature of surface XP2Y They are of two types:
Consider refraction at surface XP1Y. (i) Convex lens/converging lens (thicker at centre,
m1 sin i1 = m2 sin r1 thinner at edges).
For small angles, sin i ~− i, sin r ~− r (ii) Concave/diverging lens (thinner at centre, thicker at
edges).
m1i1 = m2r1 ...(1)
In DAOC1, i1 = a + g1 ...(2) Note: There is lateral displacement when ray passes
In DAC1I1, g1 = r1 + b1 through optical centre of thick lens. Incident
ray is parallel to emergent ray.
⇒ r1 = g1 – b1 ...(3)
(iii) Principal focus: 1st principal focus: It is a point
Substituting values of i1 and r1 in (1)
on the principal axis such that rays starting from this
m1 (a + g1) = m2 (g1 – b1) point for a convex lens or appearing to go towards
AN1 this point in a concave lens after refraction through
a ~− tan a =
N1O the lens become parallel to principal axis.

Ray Optics 299


D:\EG_Physics-12_(26-06-2022)\Open_Files\Ch-9\Ch-9
\ 27-Jul-2022 Ved_Goswami Proof-4 Reader’s Sign _______________________ Date __________

A ′B′ F A′
= 2  ...(2)
MO F2 O
A ′O F A′
F1 O O F1 Comparing (1) and (2), = 2
AO OF2
A ′O (OA ′ − OF2 )
=
AO OF2
2nd principal focus : It is a point on the principal axis v v −f
such that the rays parallel to principal axis pass through =   fi vf = – uv + fu
−u f
it (convex lens) or appear to pass through it (concave
lens) after refraction. 1 −1 1
Dividing throughout by uvf, = +
u f v
1 1 1
= −
f v u
O F2 F2 O •• Lens formula (Convex Lens : Virtual Image)
Consider a convex lens of focal length f, an object AB is
placed between optical centre and focus of the lens. The
image (A′B′) formed will be virtual, erect and enlarged.
Convex lens can be broken into three parts;central part
B′
as a thin glass slab and two prisms placed on the two
sides of the glass slab touching the glass slab. When light
passes through a prism, it is always deviated towards its
base which is lying on the glass slab so it bends towards B M
principal axis.
A′ F1 A O F2

F2 F2
DA′B′O ~ DABO
A ′B′ A ′O
=  ...(1)
AB AO
DMOF2 ~ DA′B′F2
A concave lens has central part as thin glass slab and two
prisms with their apex towards glass slab. When light A ′B′ A ′F2
=  ...(2)
passes through it, the light is always deviate towards OM OF2
base of prism and then it bends away from principal A ′O A ′F2 (A ′O + OF2 )
Comparing (1) and (2), = =
axis (diverges). AO OF2 OF2
•• Lens Formula (Convex Lens, Real Image): Consider −v −v+ f
a lens of focal length f with object AB placed between =
−u f
f and 2f. – vf = – u (– v + f) Or – vf = uv – uf
Dividing throughout by uvf,
B M
−1 1 1 1 1 1
= − ⇒ − =
F2 2F2 A′ u f v v u f
2F1 A F1 O •• Lens Formula (Concave Lens : Virtual Image)
Consider a
B′
concave lens of B M
DABO ~ DA′B′O focal length f. B′
A ′B′ A ′O An object AB
=  ...(1) is placed in A F2 A¢ O F1
AB AO
front of it at a
DMOF2 ~ DA′B′F2
point between
f and infinity.
300 Physics–12
D:\EG_Physics-12_(26-06-2022)\Open_Files\Ch-9\Ch-9
\ 27-Jul-2022 Ved_Goswami Proof-4 Reader’s Sign _______________________ Date __________

An virtual, erect and diminished image A'B' is formed.


DABO ~ DA′B′O M
A ′B′ A ′O δ
=  ...(1)
AB AO δ
DMOF2 ~ DB′A′F2 O F2
A ′B′ A ′F2
=  ...(2)
OM OF2
A ′O (OF2 − OA ′) Consider a parallel beam of light falling on a convex lens
Comparing (1) and (2), =
AO OF2 of focal length f.
−v − f − ( − v) OM = h = 1
=
−u − f OM h 1
tan d = = =
v − f +v OF2 f f
fi =
u −f 1
– fv = u (– f + v) = 1 unit of power == 1= Dioptre 1 D
Metre
Or – fv = – uf + uv 1 D is the Power of lens whose focal length in 1 m.
−1 − 1 1 1 1 1 •• Combination of Thin Lenses: Magnification: If lenses
Dividing throughout by uvf, = + ⇒ − = are combined, net magnification is the product of all
u v f v u f
Linear Magnification magnifications.
m = m1 × m2 × m3 ....
B M Power : Consider two thin lenses of focal length f1 and
f2 combined to form single lens. The lenses are placed
F2 2F2 A′ coaxially.
2F1 A F1 O

C1 C2 I I1
B′
A ′B′ A ′O
From figure, = u v
AB AO
v1
Height of image
m= Object O is placed on the principal axes.
Height of object
hi v 1 1 1
−hi v For refraction at first lens, = −  ...(1)
In this case, = ⇒ = f1 v1 u
ho � u ho u
hi v 1 1 1
= m= For refraction at 2nd lens, = −  ...(2)
ho u f2 v v1
m is (–) for real image, m is (+) for virtual image. Adding (1) and (2) we get,
•• Power of a Lens: Power of lens is ability of lens to bend 1 1 1 1 1
the rays of light. + = − =
f1 f 2 v u f of combination
It is defined as the tangent of the angle by which it 1 1 1
converges or diverges a beam of light falling at unit = P1, = P2, =P
f1 f2 f
distance from optical centre.
Thus P= P1 + P2

EXERCISE 9.3
I. Objective Type Questions (1 Mark) (a) short sightedness or myopia
1. Choose the correct answers from the given options (b) long sightedness or hypermetropia
(MCQs).
(c) Presbyopia
(i) A person uses spectacles of power +2D. He is
suffering from (d) Astigmatism

Ray Optics 301


D:\EG_Physics-12_(26-06-2022)\Open_Files\Ch-9\Ch-9
\ 27-Jul-2022 Ved_Goswami Proof-4 Reader’s Sign _______________________ Date __________

(ii) A convex lens is dipped in a liquid whose


refractive index is equal to the refractive index
of the lens. Then its focal length will O
(a) become zero (b) become infinite
(c) reduce (d) increase
5. The line AB in the ray diagram represents a lens. State
(iii) A biconcave lens of power P vertically splits into
whether the lens is convex or concave. [AI 2015]
two identical plano concave parts. The power of
each part will be [Delhi 2020] A
P P
(a) 2P (b) (c) P (d)
2 2
(iv) A biconvex lens of glass having refractive index
1.47 is immersed in a liquid. It becomes invisible
and behaves as a plane glass plate. The refractive
index of the liquid is B
(a) 1.47 (b) 1.62 (c) 1.33 (d) 1.51
III. Short Answer Type Questions-I (2 Marks)
 [Delhi, AI 2020]
6. The radii of curvature of the faces of a double convex
2. Fill in the blanks. lens are 10 cm and 15 cm. If the focal length of the
(i) A convex lens and concave lens, each having lens is 12 cm, find the refractive index of the material
same focal length of 25 cm are put in correct to of the lens. [Delhi 2010]
form a combination of lenses. The power of the
7. A convex lens of focal length f1 is kept in contact with
combination is ...................... .
a concave lens of focal length f2. Find the focal length
(ii) A concave lens made of a material of refractive of the combination. [AI 2013]
index 1.5 is immersed in a medium of refractive
index 1.5. A parallel beam of light is incident on
the lens. The beam will ......................... .
3. State True or False O I1
(i) The focal length of a lens of focal power 100 D f1
is 1 m.
(ii) The image of an object formed by a concave lens
is always virtual. I
O I1
Assertion-Reason Questions (Instructions as usual)
4. Assertion (A): A convex lens of focal length 30
cm can’t be used as a simple microscope in normal u v
v
setting. 8. A convex lens is placed in contact with a plane mirror.
Reason (R): For normal setting, the angular A point object at a distance of 20 cm on the axis of
magnification of simple microscope is M = D/f this combination has its image coinciding with itself.
[CBSE S.P. 2020-21] What is the focal length of the lens? [Delhi 2014]
II. Very Short Answer Type Questions (1 Mark) 9. The focal length of an equi-convex lens is equal to
1. An object is placed at the principal focus of a the radius of curvature of either face. What is the
concave lens of focal length f. Where will its image refractive index of the material of the lens? [AI 2015]
be formed? [AI 2008] 10. An object is placed in front of convex lens made
2. Under what condition does   a biconvex lens of glass of glass. How does the image distance vary if the
having a certain refractive index act as a plane glass refractive index of the medium is increased in such a
sheet when immersed in a liquid? [Delhi 2012] way that still it remains less than the glass?
3. Sun glasses or goggles have curved surfaces but they [S.P. 2010]
do not have any power. Why? 11. The radii of curvature of both the surfaces of a lens
4. The lens shown in fig is made of two different are equal. If one of the surfaces is made plane by
transparent materials. A point object ‘O’ is place on its grinding, how will the focal length and power of the
axis. How many images of the object will be formed? lens change? [AI 2015]

302 Physics–12
D:\EG_Physics-12_(26-06-2022)\Open_Files\Ch-9\Ch-9
\ 27-Jul-2022 Ved_Goswami Proof-4 Reader’s Sign _______________________ Date __________

12. A convex lens of focal length 25 cm is placed coaxially the path of emergent rays when (i) n2 = n1 (ii) n2 >
in contact with a concave lens of focal length 20 cm. n1 (iii) n2 < n1
Determine the power of the combination. Will the 21. Find the position of the image formed of the object
system be converging or diverging in nature? ‘O’ by the lens combination given in the figure.
[Delhi 2013]  [Delhi 2019, Foreign 2011]
f = 10 cm f = – 10 cm f = + 30 cm
2
13. A biconvex lens has a focal length times the radius
3
of curvature of either surface. Calculate the refractive O
index of the lens material. [Delhi 2010]
14. A lens forms a real image of an object. The distance
from the object to the lens is u cm and the distance
of the image from the lens is v cm. The given graph – 30 cm 5 cm 10 cm
shows variation of v with u. Lens 1 Lens 2 Lens 3
22. You are given three lenses L1, L2 and L3 each of focal
v (cm) length 20 cm. An object is kept at 40 cm in front of
60
L1, as shown. The final real image is formed at a focus
40 f of L3. Find the separations between L1, L2 and L3.
20
[AI 2012]
L1 L2 L3
0
–80 –60 –40 –20
u (cm)
(i) What is the nature of the lens? O
(ii) Using the graph, find the focal length of this lens. I

15. Light from a point source in air falls on a spherical


40 cm 20 cm
glass surface (n =1.5 and radius of curvature = 20 cm).
23. An illuminated object and a screen are placed 90 cm
The distance of the light source from the glass surface
apart. Determine the focal length and nature of the
is 100 cm. At what position the image is formed?
lens required to produce a clear image on the screen,
[NCERT] twice the size of the object. [AI 2010]
16. An object is placed 30 cm in front of a plano-convex 24. (a) An equiconvex lens with radii of curvature of
with its spherical surface of radius of curvature 20 cm. magnitude r each, is put over a liquid layer poured
If the refractive index of the material of the lens is on top of a plane mirror. A small needle, with its
1.5, find the position and nature of the image formed. tip on the principal axis of the lens, is moved along
 [AI 2020] the axis until its inverted real image coincides
17. Using lens maker’s formula, derive the thin lens with the needle itself. The distance of the needle
1 1 1 from the lens is measured to be ‘a’. On removing
formula = − for a biconvex lens. [AI 2020] the liquid layer and repeating the experiment the
f v u distance is found to be ‘b’.
3
18. The focal length of an equiconcave lens is times of Given that two values
4
radius of curvature of its surfaces. Find the refractive of distances measured
index of the material of the lens. Under what condition represent the focal length
will this lens behave as a converging lens? [AI 2020] values in the two cases,
obtain a formula for the Liquid
19. Write two characteristics of image formed when refractive index of the
an object is placed between the optical centre and liquid.
focus of a thin convex lens. Draw the graph showing
(b) If r = 10 cm, a = 15 cm, b = 10 cm, find the
variation of image distance v with object distance u refractive index of the liquid. [Delhi 2008 C]
in this case. [CBSE S.P. 2020-21]
25. A beam of light converges at a point P. Now a lens is
IV. Short Answer Type Questions-II (3 Marks) placed in the path of the convergent beam 12 cm from
20. The refractive index of a material of a concave lens is P. At what point does the beam converge if the lens is:
n1. It is immersed in a medium of refractive index n2. (i) a convex lens of focal length 20 cm.
A parallel beam of light is incident on the lens. Trace (ii) a concave lens of focal length 16 cm?  [AI 2015]

Ray Optics 303


D:\EG_Physics-12_(26-06-2022)\Open_Files\Ch-9\Ch-9
\ 27-Jul-2022 Ved_Goswami Proof-4 Reader’s Sign _______________________ Date __________

26. An object is placed 40 cm from a convex lens of focal medium of refractive index (n1) to a denser medium
length 30 cm. If a concave lens of focal length 50 cm of refractive index (n2), is incident on the convex
is introduced between the convex lens and the image side of spherical refracting surface of radius of
formed such that it is 20 cm from the convex lens, curvature R. [Delhi 2011]
find the change in the position of the image. 34. A symmetric biconvex lens of radius of curvature R
[AI 2015] and made of glass of refractive index 1.5, is placed
27. A screen is placed at a distance of 90 cm from an on a layer of liquid placed on top of a plane mirror as
object. The image of the object is formed on the screen shown in the
by a convex lens for two different locations of the lens figure. An optical
separated by 20 cm. Calculate the focal length of the needle with its tip
lens used. [NCERT] on the principal
28. An object is placed 15 cm in front of a convex lens axis of the lens is
of focal length 10 cm. Find the nature and position of moved along the
the image formed. Where should a concave mirror of axis until its real,
radius of curvature 20 cm be placed so that the final inverted image
image is formed at the position of the object itself? coincides with the needle itself. The distance of the
 [AI 2015]
needle from the lens is measured to be x. On removing
29. A sunshine recorder globe of 30 cm diameter is made the liquid layer and repeating the experiment, the
of glass of µ = 1.5. A ray enters the globe parallel distance is found to be y. Obtain the expression for
to the axis. Find the position from the centre of the
the refractive index of the liquid in terms of x and y.
sphere where the ray crosses the axis.
 [Delhi 2018]
30. A convex lens of focal length 10 cm is placed coaxially
35. A screen is placed 80 cm from an object. The image
5 cm away from a concave lens of focal length 10 cm.
If an object is placed 30 cm in front of the convex of the object on the screen is formed by a convex
lens, find the position of the image formed by the lens placed between them at two different locations
combined system. [AI 2009] separated by a distance 20 cm. Determine the focal
31. A convex lens made up of glass of refractive index. 1.5 length of the lens. [Delhi 2020]
is dipped in turn in (i) a medium of refractive index V. Long Answer Type Questions (5 Marks)
1.65 (ii) a medium of refractive index 1.33 (a) will it 36. A spherical surface of radius of curvature R, separated
behave as a converging or diverging lens in two cases a rarer and a denser medium as shown in the figure.
(b) How will its focal length changes in two media.
 [AI 2011] Rarer Denser
32. A biconvex lens with its two faces of O P C
equal radius of curvature R is made of R R
a transparent medium of refractive
index m1. It is kept in contact with a (i) Complete the path of the incident ray of light,
medium of refractive index m2 as shown showing the formation of real image. Hence
in the figure. µ1 derive the relation connecting object distance u
µ2
(a) Find the equivalent focal length of image distance ‘v’, radius of curvature R and the
the combination.  [AI 2015] refractive indices n1 and n2 of two media.
(b) Obtain the condition when this combination acts [S.P. 2016]
as a diverging lens. (ii) Briefly explain, how the focal length of a convex
(c) Draw the ray diagram for the case m1 > (m2 + 1)/2, lens changes with increase in wavelength of
when the object is kept far away from the lens. incident light.
Point out the nature of the image formed by the 37. Draw a ray diagram for formation of image of a
system. [AI 2015] point object by a thin double convex lens having
33. Obtain lens makers formula using the expression radii of curvature R1 and R2. Hence, derive lens
n2 n1 (n2 − n1 ) maker’s formula for a double convex lens. State the
− = assumptions made and sign convention used.
v u R
Here the ray of light propagating from a rarer [Delhi 2009, Foreign 2013]

304 Physics–12
D:\EG_Physics-12_(26-06-2022)\Open_Files\Ch-9\Ch-9
\ 27-Jul-2022 Ved_Goswami Proof-4 Reader’s Sign _______________________ Date __________

Or (ii) A point object is placed at a distance of 12 cm of


Using the relation for refraction at a single spherical the principal axis of a convex lens of focal length
refracting surface, derive the lens makers formula. 10 cm. A convex mirror is placed coaxially on the
38. Write the basic assumptions used in the derivation other side of the lens at a distance of 10 cm. If
of lens – maker’s formula and hence derive this the final image coincides with the object, sketch
expression.  [CBSE S.P. 2019-20] the ray diagram and find the focal length of the
convex mirror. [AI 2020]
39. (i) Derive lens maker’s formula for a biconvex lens.
Answers 9.3
I. Objective Type Questions 6. R1 = 10 cm, R2 = – 15 cm, f = 12 cm
1. (i) (a) (ii) (b) (iii) (b) (iv) (a) 1  1 1 
= ( µ − 1) −  ½
2. (i) Zero (ii) remain parallel f 
 R1 R 2 
3. (i) False (ii) True
1 1 1  5  µ −1
4. ‘A’ is false and ‘R’ is also false. As we know normal = ( µ − 1)  +  = (µ − 1)   =
12  10 15   30  6
setting means least distance of distinct vision. So for
1
a normal human it is 25cm, But in this situation it is µ − 1 = = 0.5 , m = 1.5 1½
given 30 cm. So it can't be used for simple microscope 2
in normal setting. 7. For convex lens of focal length (+ f1)
1 1 1
h = −  ...(1)
−u D f1 v ′ u
magnification (m) = ⇒m=  …(i) For concave lens of focal length (– f2)
h u
I1 will act as virtual object.
−D
1 1 1 1 1 1
– = −  ...(2) 1
When v = –D, using − = , we get f2 v v′
v u f
1 1 1 Adding equation (1) and (2),
− = 1 1 1 1
− D −u f − = −  ...(3)
D D D D f1 f 2 v u
⇒ −1 + = ⇒ = +1 For an equivalent lens (using lens formula)
u f u f
D 1 1 1
\ m= +1  [using (i)] = − , where f is the focal length of combination.
f f v u ...(4)
So, both ‘A’ and ‘R’ are false. 1 1 1
From equation (3) and (4), = −  1
II. Very Short Answer Type Questions f f1 f 2
1 1 1 1 1 1 8. 20 cm, Rays coming out of lens are incident normally
1. = − , = +
f v u v f u on the plain mirror so reflected rays will trace the path
Here u = – f and for a concave lens f = – f of incident ray, hence forming an image on the object
itself, thus object and image overlapped each other at
1 1 1 f
∴ =− − ⇒v= − focus (F) of the convex lens. 1
v f f 2
2. When refractive index of liquid = Refractive index of
 1
glass.
F
3. For sun glasses R1 = R2 = R, as both the surfaces are
equally curved.
f
 1 1  9. R1 = R, R2 = – R, f = R
So P = (µ − 1)  − = 0.
 R1 R 2  1  1 1   1 1
4. Refractive index of each material is different, so the = (µ − 1)  −  = (µ − 1)  +   1
f  R1 R 2   R R
lens will have two different focal lengths, one for each
material, hence two images will be formed. 1 2
= (µ − 1)  [Given f = R]
5. AB is a concave lens. R R

Ray Optics 305


D:\EG_Physics-12_(26-06-2022)\Open_Files\Ch-9\Ch-9
\ 27-Jul-2022 Ved_Goswami Proof-4 Reader’s Sign _______________________ Date __________

m – 1 = 0.5 µ 2 µ1 µ 2 − µ1
m = 1.5 1 Using − =  ½
v u R
1  µ2  1 1  1.5 1 1.5 − 1
10. = − 1  − + =  ½
f  µ1   R1 R 2  v 100 20
If m1 increases but m2 > m1 1.5 1 1 5−2 3
= − = =
1 v 40 100 200 200
will decrease 1
f 1.5 × 200
v=
= 100 cm  1
So f will increase 3
1 1 1 1  1 1 
So according to = −
f v u 16. Using = ( µ − 1)  − 
f  R1 R 2 
v will increase. 1
11. Case (1): R1 = R, R2 = – R
1 0.5
 1 1 = = 5 1
= (1.5 − 1)  −  =
1  1 1 2 f  20 ∞  20 200 40
As P = = (µ − 1)  +  = (µ − 1)
\ f = 40 cm
f  R R R
When both surfaces are curved 1 1 1 1
Now using = −
Case (2) : R1 = R, R2 = ∞ i.e., one surface is plane. f v u
1  1 1  1 P′ 1 f ×u 40 × ( −30 )
′ P = = (µ − 1)  −  = (µ − 1)   or = ⇒ v= =
f′  R ∞  R P 2 f +u 40 − 30
P −40 × 30
P′ =  1 ⇒ v= = –120 cm
2 10
Power will become half of it’s earlier value. Image is virtual, erect and enlarged in front of lens 120
12. f1 = 25 cm, f2 = – 20 cm ½ cm away.
Power of the combination 17. According to lens maker’s formula
1 1 1 1 1  1 1 
P= = +
f f1 f 2 − = ( µ 21 − 1)  −  ...(1)
v u  R1 R 2 
1 1 When object is at placed at infinity, u = ∞
P= − = 4 − 5 = −1D  1
0.25 0.2 Image is obtained at focus v = f
System will be diverging in nature as P is – ve. ½ Using these value in equation (1)
2 1 1  1 1 
13. f = R, R1 = R, R2 = – R, m2 = ?, m1 = 1
3 − = ( µ 21 − 1)  − 
1  µ2  1 1  f ∞  R1 R 2 
=  − 1  −  ½
f  µ1   R1 R 2  1  1 1 
= ( µ 21 − 1)  −  ...(2)
3  2 f  R1 R 2 
= (µ 2 − 1)  
2R  R on equating equation (1) and (2), we get
3 3
(µ 2 − 1) = or µ 2 = 1 + = 1.75  1½ 1 1 1
4 4 = −
14. (i) Lens is convex, because only convex lens forms a f v u
real image. 1 3
18. f = – R, R1 = –R, R2 = R
(ii) Take u = 20 cm, 4
Corresponding value of v is 20 cm  1
1 1 
So u = – 20 cm, v = 20 cm = ( µ − 1)  − 
1 1 1 1 1 2
f  R1 R 2 
      = − = + =
f v u 20 20 20 4  1 1
− = ( µ − 1)  − − 
f = 10 cm 1 3R  R R
15. u = –100 cm, R = 20 cm, (convex surface) 4  2
m1 = 1, m2 = 1.5
⇒ − = ( µ − 1)  − 
3R  R
306 Physics–12
D:\EG_Physics-12_(26-06-2022)\Open_Files\Ch-9\Ch-9
\ 27-Jul-2022 Ved_Goswami Proof-4 Reader’s Sign _______________________ Date __________

2 2 5 1 1 1 1 1 1 1 1 0

⇒ =µ–1 ⇒ µ=1+ = = − or = + =− + =
3 3 3 f 2 v2 u2 v2 f 2 u2 10 10 10
When equiconcave lens immersed in a medium, having
v2 = ∞ 1
5 For lens 3, this will act as object
refractive index more than , it behave as a converging
lens. 3 So u3 = ∞, f3 = 30 cm
19. Two characteristics- virtual and enlarged image and same 1 1 1 1 1 1 1 1 1
= − or = + = + =
1 1 1 f3 v3 u3 v3 f3 u3 30 ∞ 30
side of lens. As u and v both negative, we get = −
Interpret v u f v3 = 30 cm. 1

y = mx + c, So the final image is formed at a distance of 30 cm, on
B the R.H.S. of the lens 3.
22. For L1 : f1 = 20 cm, u1 = – 40 cm
B
1 1 1
= −
A F A f1 v1 u1
1 1 1
= +
plot of the graph 20 v1 40
u = –ve
1 1 1 2 −1 1
v = –ve v = − = =
v1 20 40 40 40
For real Image
v1 = 40 cm. 1
u For L3 : v3 = 20 cm, f3 = 20 cm, u3 = ?
1 1 1 1 1 1
= − ; = −
f3 v3 u3 20 20 u3
20. The path of rays in three cases is shown in fig.
n2 n1 n2 1
n2 n1 n2 = 0 i.e., u3 = ∞ 1
u3
L1 L2 L3


(i) for n1 = n2 1 O
½
(ii) for n2 > n1 1 I1
(iii) n2 < n1 1
20 cm
n2 n1 n2 40 cm 40 cm 20 cm

So the lens 2 has formed the image at ∞


So for lens 2, the object is placed at it’s focus because
For n2 < n1 the image formed by L1 will act as object for L2
21. For lens 1: u1 = – 30 cm, f1 = 10 cm, v1 = ? So the distance between L1 & L2 is 40 + 20 = 60 cm½
1 1 1 As the image formed by L2 is at infinity i.e., a parallel
= −       Or beam is falling on L3. So the distance between L2 & L3
f1 v1 u1
can have any value.
1 1 1 1 1 3 −1 2 23.
= + = − = = L2
v1 f1 u1 10 30 30 30
v1 = 15 cm 1
The image formed by lens 1 will act as object for the ½
Object Screen
lens 2. This will be at a distance of 10 cm on the right
side of lens 2. So this is a virtual object
For lens 2: u2 = 10 cm, f2 = – 10 cm, v2 = ? u v

Ray Optics 307


D:\EG_Physics-12_(26-06-2022)\Open_Files\Ch-9\Ch-9
\ 27-Jul-2022 Ved_Goswami Proof-4 Reader’s Sign _______________________ Date __________

|u| + |v| =90 cm


Image is real so m is – ve
v v
m = , − 2 = , v = – 2u ½
u u  ½
If we consider only magnitude |v| = 2|u| Object P

i.e., |u| + 2 |u| = 90


|u| = 30 cm 1 12 cm
But according to sign convention (ii) Concave lens
u = – 30 cm, v = 60 cm
1 1 1
Using lens formula = −  ½
f v u P
1 1 1 1+ 2 3
= + = =
f 60 30 60 60 12 cm
f = 20 cm 1 u = 12 cm, f = – 16 cm
1  1 1  1 1 1 1 1 1 −1 1 − 3 + 4 1
24. Using = (µ − 1)  − R = r, R2 = – r = − Or = + = + = =
f v u v f u 16 12 48 48
f1  R1 R 2  1
1 2 v = 48 cm. 1
(a) = (µ − 1) 26. For convex lens
f1 r
u1 = – 40 cm, f1 = 30 cm
f1 = b
1 1 1
1 2(µ − 1) r = −  ½
So = or b =  ...(1) ½ f1 v1 u1
b r 2(µ − 1)
Focal length of the combination i.e., plane concave 1 1 1 1 1 4−3 1
= + = − = =
liquid lens and the convex lens is a. v1 f1 u1 30 40 120 120
1 1 1 v1 = 120 cm 1
As = +
F f1 f 2
          f1 = b, F = a
O
1 1 1 1 1
So = − = −  ...(2) ½ I1  ½
f 2 F f1 a b
Focal length of a plano concave lens (liquid lens) is
1  −1 1  (µ − 1) 20 cm
= (µ l − 1)  − =− l  ...(3) ½
f2  r ∞ r 120 cm
(µ − 1) 1 1 This image will act as an object for the concave lens.
Comparing eqn. (2) & (3), − l = − For concave lens f2 = – 50 cm,
r a b
µ −1 1 1 u2 = 120 – 20 = 100 cm
or l = −
r b a 1 1 1 1 1 1
= − Or = +
 1 1 f 2 v2 u2 v2 f 2 u2
µl = r  −  + 1  ½
 b a 1 1 1 − 2 +1 −1
= + = = Or v2 = –100 cm
1 1  1 v − 50 100 100 100
(b) µ l = 10  −  + 1 = 10   + 1 = 1.33  1  2 1
 10 15   30 
27. x + x + 20 = 90 or x = 35 cm
25. (i) Convex lens
For L1, u = – 35 cm, v = 90 – 35 = 55 cm 1
u = 12 cm, f = 20 cm L1 L2
1 1 1 1 1 1
= − , = −
f v u 20 v 12
1 1 1 3+5 8 2  ½
= + = = = I
v 20 12
O
60 60 15
15
=v = 7.5 cm  1
2 x 20 cm x

308 Physics–12
D:\EG_Physics-12_(26-06-2022)\Open_Files\Ch-9\Ch-9
\ 27-Jul-2022 Ved_Goswami Proof-4 Reader’s Sign _______________________ Date __________

1 1 1 1 1 7 + 11 18 Distance of the image from the centre of the sphere =


= − = + = =  15 + 7.5 = 22.5 cm. ½
f v u 55 35 385 385 ½
30. For convex lens: f1 = 10 cm, u1 = – 30 cm, v1 = ?
385
= f = 21.4 cm  1 1 1 1
18 = −
28. Convex lens u = – 15 cm, f = 10 cm, v = ? f1 v1 u1
1 1 1 1 1 1 1 1 1 1 1 1 3 −1 1
= − , = + = + or = − = =
f v u 10 v 15 10 v1 30 v1 10 30 30 15
1 1 1 3−2 1 ⇒ v1 = 15 cm 1
= − = =
v 10 15 30 30
v = 30 cm 1
O
I1  ½
C

O
P 1
20 cm
120 cm
30 cm I1 will act as a virtual object for concave lens with u2
For the final image to be formed at O itself the rays of = 15 – 5 = 10 cm, f2 = – 10 cm ½
light should retrace their path after striking the mirror. 1 1 1 1 1 1 1 1
= − or = + = + =0
So the image formed by the lens should lie at centre of f 2 v2 u2 v2 f 2 u2 − 10 10
curvature of the mirror. So v2 = ∞ 1
So PC = 20 cm. The final image is formed at infinity.
So, mirror should be placed at a distance of 30 + 20 =
31. (i) refractive index of medium = m1 = 1.65,
50 cm from the lens. 1
29. For refraction of the first surface i.e., when light enters m2 = 1.5, R1 = R, R2 = – R
glass. 1 µ  1 1 
=  2 − 1  −
  R1 R 2 
B A
C f  µ1
µ1
 ½  1.5   2  − .15  2 
= − 1   =
P1 O P2 I I1  1.65   R  1.65  R 
µ2
It will behave as a diverging lens as f is –ve. 1
30 1  1.5  2 .17 2
u = – ∞, m1 = 1, m2 = 1.5, = R = 15 cm (ii) If m1 = 1.33, then =  − 1 =
2 f  1.33  R 1.33 R
Let P1I1 = v1
(a) f is + ve so converging lens. ½
µ µ µ − µ1
Using 2 − 1 = 2 1 2 1
v u R (b) In air = (1.5 − 1) = or f = R  ½
1.5 1 1.5 − 1 1.5 1 fa R R
− = or = In medium of refractive index 1.65,
v1 − ∞ 15 v1 30
165 R
P1I1 = v1 = 45 cm 1 f =− = − 5.5 R
15 2  ½
For refraction at the second surface, I1 will act as virtual In medium of refractive index 1.33,
object. 133 R 7.8
So u2 = 45 – 30 = 15 cm as object distance is to be = f = R = 3.9R  ½
17 2 2
measured from pole of the refracting surface. f is maximum in medium of refractive index
R2 = – 15 cm 1.65.
µ1 µ 2 µ1 − µ 2 32. (a) Consider the two lenses individually,
Using relation − =
v u R R1 = R, R2 = – R
Because the refraction occurs from denser to a rarer air air
medium µ1 1  1 1 2
= (µ1 − 1)  +  = (µ1 − 1)
1 1.5 1 − 1.5 1 2 f1  R R R
− = or = ⇒ v = 7.5 cm  1
v 15 − 15 v 15

Ray Optics 309


D:\EG_Physics-12_(26-06-2022)\Open_Files\Ch-9\Ch-9
\ 27-Jul-2022 Ved_Goswami Proof-4 Reader’s Sign _______________________ Date __________

1  n2  1 1 
1  1 1 1 or =  − 1  −  ½
for = (µ 2 − 1)  −  = − (µ 2 − 1) ×
  f  n1   R1 R 2 
µ2
f2 ∞ R R
X n2 (denser)
N1 N2
Focal length of the combined lens (F) n1 (rarer)
A O
1 1 1 2 1
= + = (µ1 − 1) − (µ 2 − 1) ×
F f1 f 2 R R
a  ½
1  2 µ − µ − 1
= ( 2µ1 − 2 − µ 2 + 1) =  1 2 O C2 P1 P2 C1 I I1
 
R R
R
⇒ F =  1
2µ1 − µ 2 − 1 v1
Y
(b) For this combination to act as a diverging lens F
34. When the lens is placed on a layer of liquid placed on
should be –ve.
a plane mirror.
i.e., + 2m1 – m2 – 1 < 0 or 2m1 < m2 + 1
There are two lenses in combination biconvex lens and
µ +1
or m1 < 2  1 plane concave lens.
2 The image overlapped at object when it placed on focus
µ +1
(c) If µ 1 > 2 , the combination will act as a 1 1 1
2 = +
f f1 f 2
converging lens.
f is the focal length in combination.
µ1 µ2
f1 = y, f = x
1 1 1 1 1 1
= + or = −
x y f2 f2 x y
1 y−x
or   =  1
Nature of the image : Real, inverted, ½ f2 xy
Size: highly diminished. ½ 1  1 1 
33.For a ray of light propagating from a rarer (n1) medium Here = (µ − 1)  −
y  R1 R 2 
to a denser medium
n n (n − n ) 1  1 1 1 1 2
2 − 1 = 2 1 or   = (1.5 − 1)  +  or = ×
v u R y  R R y 2 R
I1 which is the image for the surface XP1Y will act or   y = R 1
as a virtual object for the surface XP2Y For liquid lens
So v = v1, and R = R1 R1 = –R = –y, R2 = •
n2 n1 n2 − n1 1  1 1
For XP1Y, − =  ...(1) 1 \ = (µ l − 1)  − − 
v1 u R1 f2  y ∞
For surface XP2Y, v1 is object distance and the y − x µl − 1 x− y
incident ray is in denser medium of refractive = ⇒ ml = 1 +
xy −y x
index n2. So now positions of n1 and n2 will be
interchanged. 2x − y y

⇒   µl = or µ l = 2 −  1
n1 n2 n1− n2 x x
So − =  ..(2) 1
v v1 R2 35. Case 1:
Adding eqn. (1) & (2) we get L

n1 n1  1 1 
− = (n2 − n1 )  −
v u  R1 R 2 
x cm (80 – x) cm
1 1 n  1 1 
or − =  2 − 1  − 80 cm
v u  n1   R1 R 2  Distance of object u = –x cm
310 Physics–12
D:\EG_Physics-12_(26-06-2022)\Open_Files\Ch-9\Ch-9
\ 27-Jul-2022 Ved_Goswami Proof-4 Reader’s Sign _______________________ Date __________

Distance of image v = (80 – x) cm Consider a spherical at surface of radius of curvature


Let focal length = f R. A pt. object O is placed on principal axis in rarer
using lens formula: medium of refractive index n1. n1 < n2
n1 sin i = n2 sin r from Snell’s law
1 1 1
= − , we get n sin i i
f v u 2 = _ 
~ [For small angle sin i = i]
n1 sinr r
1 1 1 n2r = n1i ...(1)
= +  ... (1)
f (80 − x ) x In DACO, i = a + g ...(2) ½
Case 2: (i is exterior angle)
L In DACI, g = r + b (g is exterior angle)
r = g – b ...(3)
Substituting values of i and r in (1)
(x + 20) (60 – x) n2 (g – b) = n1 (a + g) ...(4) ½
AN AN AN
tan α = , tan β = , tan γ =
Distance of object u = – (x + 20) cm ON NI NC
Distance of image v = (60 – x) cm For small angles, tan a ~− a, tan b ~− b and tan g ~− g.
using lens formula: Eqn. (4) becomes,
1 1 1  AN AN   AN AN 
= − , we get n2  − = n1  +  1
f v u  NC NI   NO NC 
For small aperture, pt N → pt. P.
1 1 1
= +  ... (2) n2 n2 n1 n1
f ( 60 − x ) ( x + 20 ) PC − PI = PO + PC
Equating equation (1) and (2), we get: n2 n2 − n1 n1
1 1 1 1 R − v = u + R
+ = +
(80 − x ) x ( 60 − x ) ( x + 20 ) n1 n2 n1 n2
u − v = R − R
80 80

⇒ = n2 n1 n2 − n1
x ( 80 − x ) ( 60 − x ) ( x + 20 ) − =
v u R  1
⇒ 80x – x2 = 1200 + 40x – x2 (ii) As wavelength of the incident light increases, the
⇒ 40x = 1200 ⇒ x = 30 cm refractive index n2 of the lens will decrease
put the value of x in equation (1) 1 µ  1 1 
1 1 1 =  2 − 1  −
= + f  µ1   R1 R 2 
f (80 − 30 ) 30 So
1
will decrease i.e., f will increase. 1
1 1 1 8 f

⇒ = + = 37. Lens Maker Formula
f 50 30 150 X
f = 18.75 cm i1
A
i B
M′

36. (i) Object placed in rarer medium, real image


r1

Assumptions in refraction at spherical surfaces. α P2 γ1 β1


 1
– Object is point object. O C2 P 1 N 1 P C1 I I1

– Angle of incidence is small.


– Aperture of surface is small.
n1 (rarer) Y
A
i n2 (denser) Assumptions:
– Object is point object lying on principal axis.
r
α γ β  1
O P N C I – Lens used is thin, so that the distances measured from
its surface may be taken equal to those measured
from optical centre.

Ray Optics 311


D:\EG_Physics-12_(26-06-2022)\Open_Files\Ch-9\Ch-9
\ 27-Jul-2022 Ved_Goswami Proof-4 Reader’s Sign _______________________ Date __________

– Aperture of the lens is small. 1 1 µ  1 1 


– All the rays are paraxial, i.e., they make small angle − =  2 − 1  −
v u  µ1   R1 R 2 
with the normal to the lens faces, and to the principal
axis. 1 µ  1 1 
=  2 − 1  −  1
Consider a thin convex lens with pt. object f  µ1   R1 R 2 
O placed on its principal axis. 38. Basic assumptions in derivation of Lens-maker’s
Let R1 = radius of curvature of surface XP1Y formula:
R2 = radius of curvature of surface XP2Y (i) Aperture of lens should be small
Consider refraction at surface XP1Y. (ii) Lenses should be thin
m1 sin i1 = m2 sin r1 (using Snell’s law) (iii) Object should be point sized and placed on principal
For small angles, sin i ~− i, sin r ~− r axis.
N1
m1i1 = m2 r1 ...(1) i1
A

In DAOC1, i1 = a + g1 ...(2) R1


~
In − AC1I1, g1 = r1 + b1 ⇒ r1 = g1 – b1 ...(3) ½
Substituting values of i and r in (1) O B R1 C1 I1
u v
m1 (a + g1) = m2 (g1 – b1) 1
AN1 C
a ~− tan a = N O N2
1 A r2
AN1 R1
b1 ~− tan b1 =
N1I1 D I I1 ½
C2 v C1
AN1 R2
g1 ~− tan g1 = v
N1C1 n2 n1
C
As the lens is thin, all the distances measured from its Suppose we have a thin lens of material of refractive
surface may be taken equal to those measured from its index n2, placed in a medium of refractive index n1,
optical centre. let O be a point object placed on principal axis then
µ1 µ µ µ for refraction at surface ABC we get image at I1,
   + 1 = 2 − 2
PO PC1 PC1 PI1 n2 n1 n −n
∴ − = 2 1  ...(i) ½
µ1 µ1 µ 2 µ 2 v′ u R1
+ = −
   − u R1 R1 V1 But the refracted ray before goes to meet at I1 falls
on surface ADC and refracts at I2 finally; hence I1
µ 2 µ1 µ 2 − µ1
   − =  ...(A) 1 works as a virtual object 2nd refracting surface
V1 u R1 n − n2
n n
For refraction of XP2Y, I1 will act as virtual object. The ∴ 1 − 2 = 1  ...(ii) ½
v v′ R2
incident ray is in the denser medium, refraction is taking
Equation (1) + (2)
place at a convex surface.
n1 n1  1 1 
As light is going from denser to rarer medium it bends − = ( n2 − n1 )  −
away from the normal. Final image is formed at I. v u  R1 R 2 
µ1 µ 2 µ1 − µ 2 1 1  1 1 
− =  ...(B) 1
∴ − = ( n21 − 1)  −  ...(iii)
V V1 R2 v u  R1 R 2 
Adding (A) and (B), If u = ∞, ѵ = f
µ1 µ 2 µ 2 − µ1 µ1 − µ 2 1  1 1 
− = + = ( n21 − 1)  −  ...(iv)1
V u R1 R2 f  R1 R 2 
µ µ  1 1  Which is lens maker’s formula.
1 − 1 = (µ 2 − µ1 )  −
V u  R1 R 2  39. (i) Refer solution 37
Dividing both sides by m1 (ii) For the convex lens:

312 Physics–12
D:\EG_Physics-12_(26-06-2022)\Open_Files\Ch-9\Ch-9
\ 27-Jul-2022 Ved_Goswami Proof-4 Reader’s Sign _______________________ Date __________

u = –12 cm, f = +10 cm


1 1 1

\ = −
10 v ( −12 )
1 1 1 6−5 1
10cm

⇒ = − = =
O L M I v 10 12 60 60
⇒ v = 60 cm

Mirror

\ MI = LI – LM = 60 – 10 = 50 cm
Lens
 MI = R = (radius of curvature)

\ R = 50 cm
1 1 1
As = − R 50
f v u fmirror =
= = 25 cm
2 2

Topic 4. Refraction Through a Prism


•• Prism: It is a wedge shaped piece of transparent medium ∠d = ∠MQR + ∠MRQ
bounded by two plane surfaces inclined to each other. = (i – r1) + (e – r2) = i + e – (r1 + r2) ...(1)
ngle of the Prism : The angle included between the two Also A + ∠QNR = 180° ...(2)
refracting faces is called the angle of the prism. r1 + r2 + ∠QNR = 180° ...(3)
AB and AC are the refracting surfaces. Comparing eqn. (1) & (2)
∠BAC is angle of the prism. r1 + r 2 = A
BC is called the base of the prism. So eqn. (1) becomes
A d = i + e – A or i + e = A + d
Variation of angle of deviation with angle of incidence:
Section of a prism As angle of incidence increases, d will initially decrease,
become minimum and then begin to increase.
At minimum deviation.
C (i) ∠i = ∠e   (ii) r1 = r2
B
(iii) refracted ray is parallel to BC i.e., incident ray
•• Refraction Through a Prism: To show that ∠i + ∠e and emergent ray are symmetrically situated w.r.t.
= ∠A + ∠d. prism.
When monochromatic light strikes the face AB, as it
 A + δm 
goes from rarer to denser medium, it bends towards sin 
 2 
normal. When this refracted ray strikes the face AC, it •• To show that: µ =
bends away from normal as light is going from denser A
sin
to rarer medium. 2
A
Note : When light passes through a prism, it bends
towards it’s base. Normal Normal
at Q at R
Refracting M
A δ
edge Angle of δ Q
on
Normal Refracting prism e v iati r1 r2 R e
of d Normal
at Q face gle at R
M A n N S
P
Q R
i r2 e C
r1 B
ay
e nt r
P Incid N
S
deviation

C
Angle of

B
δ = δm
∠i is angle of incidence
∠e is angle of emergence
∠d is angle of deviation
i i = i′ i′

Ray Optics 313


D:\EG_Physics-12_(26-06-2022)\Open_Files\Ch-9\Ch-9
\ 27-Jul-2022 Ved_Goswami Proof-4 Reader’s Sign _______________________ Date __________

Proof: We know thati + e = A + d •• Dispersion: The phenomenon of splitting of a beam


At d = dmin, i = e and r1 = r2 of white light into its constituent of colours on passing
A through a dispersive medium is called dispersion.
So A = r1 + r2 = r + r Or r= •• Spectrum: The wavelength-wise arrangement of the
2
A + δm constituents of white light on passing through a dispersive
Also i + i = A + dm Or i= medium is called the spectrum.
2
According to Snell’s Law,
 A + δm  Screen
sin 
sin i  2  Red
µ= =
sin r A Orange
sin Yellow
2 Glass Green
ite
A Wh ght Prism Blue
L i Indigo
Normal Normal Violet
at Q
M at R •• Cause of Dispersion: Each colour has its own
δ wavelength. According to Cauchy’s relation, refractive
Q
r1 r2 R e index varies with wavelength of light as
B C
µ = A + 2 + 4 + ....
N S
P
λ λ
B C where A, B, C = constants.
For prism of small angle d = A (µ –1)
lV < lR
deviation
Angle of

δ = δm \ mV > mR  ⇒ dV > dR


So deviation for violet > deviation for red
i i = i′ i′
•• Refraction through Prism of Small Angle:
A
µ

δ λ
Q
i
r1 r2 R e Violet colour will be deviated maximum and red colour
will be deviated minimum.
•• Angular Dispersion: It is the difference between angle
B C of deviation of two extreme colours i.e., violet and red.
sin i Or
For refraction at AB, µ =
sin r1 It is the angle in which all colours of white light are
i contained.
For small angles, µ= since i ≈ sin i, r1 ≈ sin r1 Angular dispersion = dV – dR = (µV – 1) A – (mR – 1) A =
r1
i= µr1 (mV – mR)A
sin e Angle of dispersion depends on angle of prism and nature
For refraction at AC, µ = of material of prism.
sin r2
e A
For small angles, µ= ⇒ e = µ r2
r2
i+e= A+d N
δV δR
µr1 + µr2 = A+d White
fi µ(r1 + r2) = A+d Light
R
µA – A = d N′ V
A (µ – 1) = d
B C

314 Physics–12
D:\EG_Physics-12_(26-06-2022)\Open_Files\Ch-9\Ch-9
\ 27-Jul-2022 Ved_Goswami Proof-4 Reader’s Sign _______________________ Date __________

•• Dispersive Power: It is defined as the ratio of angular δ V − δ R (µ V − µ R ) A


dispersion to mean deviation produced by the prism. It w= =
δY (µ Y − 1) A
is represented by w. (µ V − µ R )
Mean deviation is the deviation for the mean colour =
(µ Y − 1)
which is yellow. where mY = refractive index for mean colour.
Angular dispersion Dispersive power depends only on nature of material of
w=
Mean deviation prism and not on angle of prism.

EXERCISE 9.4
I. Objective Type Questions (1 Mark) Assertion-Reason Questions (Instructions as usual)
1. Choose the correct answers from the given options 4. Assertion (A): The edges of the images of white
(MCQs). object formed by concave mirror on the screen appear
(i) When light is passed through a prism, the colour white.
which deviates least is Reason (R): Concave mirror does not suffer from
(a) Red (b) Violet (c) Blue (d) Green chromatic aberration.
(ii) The refractive index of glass is 1.520 for red light II. Very Short Answer Type Questions (1 Mark)
and 1.525 for blue light. Let D1 and D2 be angles 1. Write the relationship between angle of incidence ‘i’,
of minimum deviation for red and blue light angle of prism ‘A’ and angle of minimum deviation
respectively in a prism of this glass. Then for a triangular prism. [Delhi 2013]
(a) D1 = D2 (b) D1 > D2 2. Out of blue and red light which is deviated more by
(c) D1 < D2 a prism? Give reason. [Delhi 2010]
(d) Depends on the angle of prism
III. Short Answer Type Questions-I (2 Marks)
(iii) For a glass prism, the angle of minimum deviation
3. A ray of light, incident on an equilateral glass prism
will be smaller for the light of
(a) red colour (b) blue colour ( )
µ g = 3 moves parallel to the base line of
(c) yellow colour (d) green colour the prism inside it. Find the angle of incidence for
this ray. [Delhi 2012, AI 2015]
[Delhi, AI 2020]
A
2. Fill in the blanks.
(i) When a ray of white light passes through a glass 60°

prism, red light is divided through a ......... angle Q R


than violet light because the refractive index for i
r
violet light is ......... than that of red light. P
(ii) The dispersive power of for a given prism is
independent of .................. but depends .......... . B
60°
C
(iii) An equilateral prism is made up of material of 4. A ray PQ incident normally on the A
refractive index 3 . The angle of minimum refracting face BA is refracted in P 90°
deviation of light passing through the prism is the prism BAC made of material of Q
..................... . [CBSE S.P. 2019-20] refractive index 1.5. Complete the
(iv) A ray of light on passing through an equilateral path of ray through the prism. From
glass prism, suffers a minimum deviation equal which face will the ray emerge?
to the angle of the prism. The value of refractive 60°
Justify your answer. [AI 2016] B C
index of the material of the prism is .............. .
 [Delhi, AI 2020] 5. A ray PQ incident on the refracting face BA is
3. State True or False refracted in the prism BAC as shown in the figure
and emerges from the other refracting face AC as RS
(i) Rainbow is formed due to the combination of
such that AQ = AR. If the angle of prism A = 60° and
dispersion and total internal reflection.
(ii) Maximum deviation takes place when there is 30° refractive index of material of prism is 3 , calculate
incidence. angle q. [AI 2016]

Ray Optics 315


D:\EG_Physics-12_(26-06-2022)\Open_Files\Ch-9\Ch-9
\ 27-Jul-2022 Ved_Goswami Proof-4 Reader’s Sign _______________________ Date __________

A Find the angle of the prism.


[NCERT Exemplar, AI 2016]
9. Draw a graph to show the angle of deviation d with the
variation of angle of incidence i for monochromatic
Q R θ ray of light passing through a prism of refracting angle
P  A + δm 
S sin 
 2 
A. Deduce the relation µ = .
B C A
6. A ray of light incident on one of the faces of a glass sin
2
prism of angle ‘A’ has angle of incidence 2A. The [Delhi 2008, 2011, Foreign 2011]
refracted ray in the prism strikes the opposite face 10. (i) A ray of light incident on face AB of an equilateral
which is silvered, the reflected ray from it retracing glass prism, shows minimum deviation of 30°.
its path. Trace the ray diagram and find the relation Calculate the speed of light A
between the refractive index of the material of the through the prism.
prism and the angle of the prism. [AI 2015] (ii) Find the angle of incidence
IV. Short Answer Type Questions-II (3 Marks) at face AB so that the
7. Trace the path of a ray of light passing through a glass emergent ray grazes along
prism (ABC) as shown in the figure. If the refractive the face AC. [Delhi 2017] B C
index of glass is 3 , find out of the value of the angle 11. A ray of light through a prism of refractive index 2
of emergence from the prism. [Foreign 2012] as shown in the figure.
A A
60°
M N
r1 r2

60° µ= 2
B C
60°
B C
Find:
(
8. For a glass prism µ = 3 the angle of minimum ) (i) the angle of minimum deviation for this prism.
deviation is equal to the angle of the prism. (ii) the angle of incidence (∠r2) at face AC.
Answers 9.4
I. Objective Type Questions So blue colour is deviated more than the red colour.
1. (i) (c) (ii) (c) (iii) (a) 3. As ray QR is parallel to the base BC of the prism, so it
2. (i) smaller, greater is the case of minimum deviation. ½
(ii) angle of prism (∠A) and µV, µR and µY
A sin i
(iii) 60° (iv) 3 3. (i) True (ii) False Thus, r = = 30° , µ 21 = ½
2 sin r 
4. (a)
sin i 1 3
II. Very Short Answer Type Questions So, 3= ⇒ sin i = 3 × ⇒ sin i =
sin 30° 2 2
A + δm
1. Angle of incidence i=   i = 60° 1
2
2. Deviation caused by a prism d = (m – 1) A Refractive 4. m = 1.5
index (m) is more for blue than red.
1 1
 B     sin ic = = = 0.666  ½
 Using µ = A + 2 + ..... µ 1.5
 λ 
ic = 42°
As lV < lR
So mV > mR i < ic ½
So dV > dR light will emerge from the face AC.

316 Physics–12
D:\EG_Physics-12_(26-06-2022)\Open_Files\Ch-9\Ch-9
\ 27-Jul-2022 Ved_Goswami Proof-4 Reader’s Sign _______________________ Date __________

A
A + δm
sin
P30° 8. µ = 2 , µ = 3 , dm = A ½
Q A
60° sin
30°
 1 2
A+A
sin
µ2
So 3 = 2 = sin A  ½
µ1
A A
sin sin
B
60°
C 2 2
5. If AQ = AR then QR || BC so q is angle of minimum A A
2 sin cos
deviation. =    = 2 2 2 cos A  1
 A + δm   60° + θ  θ A 2
 sin
sin  sin  sin  30° + 
 2   2   2 2
Using µ = = =
A
A sin 30° 1 A 3
sin cos = ∴ = 30° i.e., A = 60° 1
2 2 2 2 2
1 9. See theory.
1  θ 10. (i) dm = 30°, A = 60°
3 × = sin  30° + 
2  2
 A + δm   60° + 30° 
θ sin   sin  
or 30° + = 60° ⇒ θ = 60°   2   2 sin 45°
1 µ = = =
2  60°  sin 30°
 A sin 
sin i sin 2A 2 sin A cos A sin    2  
 2
6. µ = = or µ =
sin r sin A sin A = 2  1
m = 2 cos A 1 c 3 × 10 8
c
A So v= =  [ µ = ]
m 2 v
A 8
= 2.122 × 10 m/s ½
90 ° – A (ii) If ∠r 2 = ic , then the emergent ray will graze along
 1 the face AC.
2A A A

B C i r1 r2
7. ∠i = 0° at the face AC. So ∠r = 0
Angle of incidence on face AB is 30° B C
1 1 1 1 1
sin ic = = = = 0.5774 1 µ= Or sin ic =
µ 3 1.732 sin ic µ
Thus i < ic so light will be refracted 1
sin ic = i.e. ic = 45° ½
sin 30° 1 3 2
= ⇒ sin e = 3 sin 30° = As r1 + r2 = A
sin e 3 2
So r1 = A – r2 = 60° – 45° = 15° ½

\ ∠e = 60° 1
sin i
A µ=
sin r1
30°
⇒ sin i = µ sin r1 = 2 sin 15°
= 1.414 × 0.2588 = 0.3659
 1 So i = 21.5° ½
30° 11. (i) The angle of minimum deviation
e  A + δm 
sin 
 2 
m=
60°
 A
B C sin  
 2

Ray Optics 317


D:\EG_Physics-12_(26-06-2022)\Open_Files\Ch-9\Ch-9
\ 27-Jul-2022 Ved_Goswami Proof-4 Reader’s Sign _______________________ Date __________

 A + δm   1  A + δm
sin   sin–1   =
 2   2 2
2 = °
 60  A + δm
sin   45° =
 2  2
 A + δm  90° = 60° + dm
2 sin 30° = sin 
 2  dm = 30°
(ii) Angle of incidence
1  A + δm  A + δm
2× = sin 
2  2  i=
2
1  A + δm  60° + 30°
= sin  ∴ i= = 45°
2  2  2

Topic 5. Optical Instruments


• (a) Simple Microscope [image formed at least distance 1 −1 1
of distinct vision]: It is a single convex lens with short = +
f D u
focal length fitted with a handle to hold it. Also called
D D
a magnifying glass. = −1+
B f u
D D
m= =1+
u f
B
Smaller the focal length, more the magnifying power.
 •• Simple Microscope (When final image is formed at
A F A O F infinity): When an object placed at focus point of a
convex lens so that parallel rays enter the eye, the image
is formed at infinity. In this case, our eye lens is relaxed.
B D

B
 E
A

(b) When an object AB is placed between optical centre A β


and focus of a convex lens, virtual, erect and enlarged F O F
image is formed on the same side of the lens where
B
the object is placed.The position of object AB is so
adjusted that the final image is formed at least distance α
A E
of distinct vision D. D
Magnifying power : It is the ratio of angle subtended Magnifying Power is defined as ratio of angle formed
by the image and the object at the eye when both are at by image when formed at infinity to the angle formed
least distance of distinct vision. by object at eye when placed at least distance of distinct
β vision.
m=
α β
m=
A ′B′ α
tan b ~− b for small angles =
OA ′ AB
AB
For small angles tan b ~− b =
and tan a ~− a = −f
AE AB
tan a ~− a =
β A ′B′ D −D
m= = = ,
α AB u β AB − D D
as OA′ = AE = D m= = × =
α − f AB f

318 Physics–12
D:\EG_Physics-12_(26-06-2022)\Open_Files\Ch-9\Ch-9
\ 27-Jul-2022 Ved_Goswami Proof-4 Reader’s Sign _______________________ Date __________

•• Compound Microscope : Image at least distance of distinct vision: The object AB to be magnified is placed
between f and 2f of the objective lens. Magnified, real, inverted image A′B′ is formed. A′B′ will act as object for the
eye piece. The distance between objective and eye piece is so adjusted that A′B′ lies within the focal length of the eye
piece. Eye piece acts as a simple microscope. The final image A′′B′′ is formed at the least distance of distinct vision.
The final image is inverted with respect to object AB.
B

E
A

D Eye
uo vo

B
A  A
2Fo A F o O F
o Fe  O

Objective
lens B

Eyepiece

B

Magnifying power : It is the ratio of the angle subtended u0 ~ f0 since object is very close to focus.
at the eye by the final image to the angle subtended at L is distance between the two lenses.
the eye by the object when both are at least distance of •• Telescope: It is a device which is used to see far off
distinct vision. objects clearly. There are two kinds of telescopes :
A ′′B′′ refracting type and reflecting type.
tan b ~− b =
O ′A ′′ Refracting type telescope :
AB (i) Astronomical telescope, (ii) terrestrial telescope.
tan a ~− a =
AE (i) Astronomical [final image formed at ∞] also
β A ′′B′′ AE A ′′B′′ O ′A ′′ A ′′B′′ called normal adjustment.
m = = × = × =
α O ′A ′′ AB O ′A ′′ AB AB It consists of two lenses —Objective and eye piece.
A ′′B′′ A ′B′ A ′′B′′ A ′B′ Objective is a convex lens of large focal length
m = × = × = me × mo
AB A ′B′ A ′B′ AB and large aperture. It focuses the distant object.
 D  v  Larger aperture is taken to form a bright image and
m = 1 +  ×  0  high resolving power. With larger focal length the
 f e   − u0 
magnifying power increases.
[– ve sign shows that image is inverted] Eye piece is a convex lens of small focal length
 D  L  and smaller aperture.
m ~− – 1 +   
 fe   fo 

Ray Optics 319


D:\EG_Physics-12_(26-06-2022)\Open_Files\Ch-9\Ch-9
\ 27-Jul-2022 Ved_Goswami Proof-4 Reader’s Sign _______________________ Date __________

fo fe

Q
C
Eye
α Α′
α
Ο β Ο′

Β′

m∞
Lo Fro
Le
Magnifying power: It is the ratio of angle subtended by image at eye and angle subtended by object at eye when both
β
are formed at infinity. m=
α
A ′B′
For small angles tan b ~− b =
O ′A ′
A ′B′
tan a ~− a =
OA ′
β A ′B′ OA ′ OA ′ + f o
m= = × = =
α O ′A ′ A ′B′ O ′A ′ − f e
(–) Shows that it is inverted image. Not to be taken in calculations
Distance between the two lenses = fo + fe = length of tube.
•• Astronomical Telescope [final image formed at least distance of distinct vision]: Object is placed at infinity.
Objective forms the image in focal plane.
Image formed by objective will act as object for eye piece. Distance between eye piece and objective is so adjusted
that AB lies within focal length of eye piece. i.e. O′B < fe
Position of AB is so adjusted that final image is formed at least distance of distinct vision.

fo
ue

 fe 

e
Ey

Le

Lo

320 Physics–12
D:\EG_Physics-12_(26-06-2022)\Open_Files\Ch-9\Ch-9
\ 27-Jul-2022 Ved_Goswami Proof-4 Reader’s Sign _______________________ Date __________

Magnifying power of a telescope is defined as ratio of R


angle subtended at eye by final image formed at least fo
distance of distinct vision to the angle subtended by For image formed at infinity, m = = 2
fe fe
object at eye when seen directly.
β (i) Newtonion Telescope (Reflecting type): In
m= newtonian telescope, objective is a concave mirror of
α
AB AB large focal length and large aperture. A plane mirror
For small angles b ~− tan b = ; a ~− tan a = (secondary mirror) is placed just before the focus
O ′B OB
point of the objective such that it makes an angle of
β OB f f
m= = = o = o 45° with the principal axis of concave mirror. Eye
α O ′ B O ′ B − ue piece is a convex lens of smaller focal length and
1 1 1 smaller aperture.
According to lens formula, − =
v u f
v = – D and u = – ue M1
where D = least distance of distinct vision
1 1 1 1 1 1 M2 Objective
− = Or = +
− D − ue fe ue fe D 45°
F M2 is
 1 1 f  f 
m = − f 0 ×  +  = − o 1 + e  Secondary
 fe D  fe  D mirror
\ Magnification will increase.
•• Reflecting Telescope: (i) Cassegrain Telescope:
Objective is concave mirror of large focal length and
large aperture having a hole around its pole. The light
coming from a far off object after reflection from the
Eyepiece
objective is made to fall on a secondary mirror placed
in between its pole and focus. The light reflected from •• Advantage of Reflecting Telescope over Refracting
the convex mirror goes into the eye piece, which acts as Type: The objective lens is replaced by concave mirror
a simple microscope. of large aperture has high gathering power and absorbs
very little amount of light than lens of large aperture.
So final image formed in reflecting telescope is very
bright.
Parallel rays A
– Due to large aperture of mirror, its resolving power
from distant is also high.
object B – As the objective is the mirror, there are no chromatic
Eyepiece aberration.
– Use of parabolic mirror will also reduce spherical
aberration.
For image formed at least distance of distinct vision – Lens of large aperture tends to be heavy and hence
fo  fe  difficult to make and support by its edges whereas the
m= 1 +  . mirror of equivalent optical quality weighs less and
fe  D can be supported over its entire back surface.

EXERCISE 9.5
I. Objective Type Questions (1 Mark) (c) Real and enlarged (d) Virtual and enlarged
1. Choose the correct answers from the given options (ii) In a compound microscope, the objective lens of
(MCQs). fo and eyepiece of fe are placed at distance L such
(i) The image formed by an objective of a compound that L equals
microscope is (a) fo + fe (b) fo – fe
(a) Virtual and diminished (c) much greater than fo or fe
(b) Real and diminished (d) much less than fo or fe

Ray Optics 321


D:\EG_Physics-12_(26-06-2022)\Open_Files\Ch-9\Ch-9
\ 27-Jul-2022 Ved_Goswami Proof-4 Reader’s Sign _______________________ Date __________

(iii) The final image formed in an astronomical Reason (R): The range of a telescope tells us how far
refracting telescope with respect to the object is away a star of some standard brightness can spotted
[CBSE S.P. 2019-20] by telescope.
(a) Real inverted (b) Real erect II. Very Short Answer Type Questions (1 Mark)
(c) Virtual erect (d) Virtual inverted 1. You are given following three lenses. Which two
(iv) The focal length of the objective of a compound lenses will you use as an eyepiece and as an objective
microscope is [AI 2020] to construct an astronomical telescope?
Lenses Power Aperture
(a) greater than the focal length of eyepiece
L1 3 D 8 cm
(b) lesser than the focal length of eyepiece.
L2 6 D 1 cm
(c) equal to the focal length of eyepiece L3 10 D 1 cm
(d) equal to the length of its tube. 2. Define magnifying power of a telescope. Write its
(v) Larger aperture of objective lens in an astronomical expression. [Delhi 2012]
telescope [Delhi 2020] 3. An astronomical telescope uses two lenses of powers
(a) increases the resolving of the image 10 D and 1 D. What is its magnifying power in normal
(b) decreases the brightness of the image adjustment? [AI 2010]
(c) increases the size of the image 4. How is the working of a telescope different from the
microscope? [Delhi 2012]
(d) decreases the length of the telescope
5. Which two main considerations are kept in mind
2. Fill in the blanks. while designing the ‘objective’ of an astronomical
(i) In a compound microscope, the intermediate image telescope?
is ........................ . III. Short Answer Type Questions-I (2 Marks)
(ii) The objective of an astronomical telescope has a 6. With the help of a ray diagram explain the working
large aperture to ..................... . of a reflecting telescope. Mention two advantage of
(iii) A compound microscope is used because a a reflecting telescope over a refracting telescope.
realistic simple microscope does not have .............  [CBSE 2022]
magnification. [AI 2020] 7. Write two important limitations of a refracting
3. State True or False telescope over a reflecting type telescope.[AI 2013]
8. Define the magnifying power of a compound
(i) The length of the astronomical telescope for normal
microscope when the final image is formed at infinity.
adjustment is fo + fe. Why must both the objective and the eyepiece of
(ii) The magnifying power of the erecting lens in a a compound microscope has short focal lengths?
terrestrial telescope is two. Explain. [Delhi 2017]
4. Match the Columns Or
Column-I (Optical Column-II Explain two advantages of a reflecting telescope over
Instrument) (Magnifying Power) a refracting telescope.
9. Why should the objective of a telescope have large
(i) Simple microscope D focal length and large aperture? Justify your answer.
(a)
(when image at D) f  [Delhi 2017, Foreign 2013]
10. With the help of a ray diagram, show how a compound
(ii) Simple microscope v  D microscope forms a magnified image of a tiny object,
(b) − 0 1 + f  at least distance of distinct vision. Hence derive an
(when final image at •) u0
expression for the magnification produced by it.
f0  D   [CBSE 2022]
(iii) Compound microscope (c) − 1+  IV. Short Answer Type Questions-II (3 Marks)
f e  f 11. Draw a labelled ray diagram of a refracting telescope.
Define its magnifying power and write expression for
(iv) Cassegrain telescope (d) 1 + D it. [AI 2013]
(at ‘D’) f 12. Draw a schematic diagram of a reflecting telescope
Assertion-Reason Questions (Instructions as usual) (Cassegrain). Write two important advantages that a
5. Assertion (A): By increasing the diameter of the reflecting telescope has over a refracting telescope.
objective of telescope, we can increase its range. [Delhi 2008, 2012, 2015]

322 Physics–12
D:\EG_Physics-12_(26-06-2022)\Open_Files\Ch-9\Ch-9
\ 27-Jul-2022 Ved_Goswami Proof-4 Reader’s Sign _______________________ Date __________

Or (i) Identify the optical instrument.


(i) Draw a schematic labelled ray diagram of a (ii) Calculate the angular magnification produced by
reflecting type telescope. the instrument. [Delhi 2020]
(ii) Write two important advantages justifying why 21. What is the difference in the construction of an
reflecting type telescopes are preferred over astronomical telescope and a compound microscope?
refracting telescopes. The focal lengths of the objective and eyepiece
13. Draw ray diagram to explain the working of a of a compound microscope are 1.25 cm and 5.0
refracting astronomical telescope when the final cm, respectively. Find the position of the object
image is formed at near point. [Delhi 2019] relative to the objective in order to obtain an angular
magnification of 30 when the final image is formed
Define its magnifying power and obtain expression at the near point. [AI 2020]
for it.
V. Long Answer Type Questions (5 Marks)
14. Draw a neat labelled ray diagram of a compound
microscope. Explain briefly the working. [AI 2010] 22. A small telescope has an objective lens of focal length
140 cm and an eye-piece of focal length 5.0 cm. What
15. Draw the labelled ray diagram for the formation is the:
of image by a compound microscope. Derive an
expression for its total magnification (or magnifying (i) magnifying power of telescope for viewing distant
objects when the telescope is in normal adjustment
power), when the final image is formed at the
(i.e., when the final image is at infinity)?
near point. Why both objective and eyepiece of a
compound microscope must have their short focal (ii) the final image is formed at the least distance of
lengths? [Foreign 2008, Delhi 2009, 2010, 2013] distinct vision (D = 25 cm)?
16. (i) A giant refracting telescope at an observatory has (iii) What is the separation between the objective and
an objective lens of focal length 15 m. If an eye eye lens when final image is formed at infinity?
piece of focal length 1.0 cm is used, what is the (iv) If this telescope is used to view a 100 m tall tower
angular magnification of the telescope? 3 km away, what is the height of the image of the
(ii) If this telescope is used to view the moon, what tower formed by the objective lens?
is the diameter of the image of the moon formed (v) What is the height of the final image of the tower
by the objective lens. The diameter of the moon if it is formed at the least distance of distinct vision
is 3.48 × 106 m and radius of lunar orbit is 3.8 × D = 25 cm? [AI 2013 C][S.P. 2012]
108 m. [Delhi 2008, 2019, AI 2011, 2015] 23. (i) Draw a ray diagram for final image formed at
17. A compound microscope consists of an objective lens distance of distinct vision (D) by a compound
of focal length 2.0 cm and an eyepiece of focal length microscope and write expression for its magnifying
6.25 separated by a distance of 15 cm. How far from the power.
objective should an object be placed in order to obtain (ii) An angular magnification (magnifying power) of
the final image at (i) the least distance of distinct vision 30X is desired for a compound microscope using
(D = 25 cm) and (ii) infinity? as objective of focal length 1.25cm and eye piece
18. A compound microscope uses an objective lens of focal length 5cm. How will you set up the
of focal length 4 cm and eyepiece lens of focal compound microscope? [CBSE S.P. 2019-20]
length 10 cm. An object is placed at 6 cm from the 24. (i) Draw the ray diagram of an astronomical telescope
objective lens. Calculate the magnifying power of the when the final image is formed at infinity.
compound microscope. Also calculate the length of (ii) An astronomical telescope has an objective lens
the microscope. [AI 2011] of focal length 20 m and eyepiece of focal length
19. The focal lengths of the objective and eyepiece of 1 cm.
microscope are 1.25 cm and 5 cm respectively. Find (a) Find the angular magnification of the
the position of the object relative to the objective telescope.
in order to obtain an angular magnification of 30 in
(b) If this telescope is used to view the Moon,
normal adjustment.  [Delhi 2012]
find the diameter of the image formed by
20. An optical instrument uses an objective lens of power the objective lens. Given the diameter of the
100 D and an eyepiece of power 40 D. The final Moon is 3.5 × 106 m and radius of lunar orbit
image is formed at infinity when the tube length of is 3.8 × 108 m. [AI 2020]
the instrument is kept at 20 cm.

Ray Optics 323


D:\EG_Physics-12_(26-06-2022)\Open_Files\Ch-9\Ch-9
\ 27-Jul-2022 Ved_Goswami Proof-4 Reader’s Sign _______________________ Date __________

Answers 9.5
I. Objective Type Questions in between its pole and focus. The light reflected from
1. (i) (c) (ii) (a) (iii) (d) the convex mirror goes into the eye piece, which acts
(iv) (b) (v) (a) as a simple microscope.
2. (i) real, inverted and magnified
(ii) have high resolution (iii) multilevel
3. (i) True (ii) False
4.(i)-(d) (ii)-(a) (iii)-(b) (iv)-(c) 5. (b) Parallel rays
from distant
A

II. Very Short Answer Type Questions object B


1. An astronomical telescope has an objective of larger Eyepiece

aperture and larger focal length while an eye piece is


of small aperture and small focal length.
Therefore, we will use L1 as an objective and L3 as an For image formed at least distance of distinct vision
eyepiece. fo  fe 
2. Magnifying power in normal adjustment: It is equal m= 1 +  .
fe D
to the ratio of the angle subtended by the final image
at the eye when formed at infinity as seen through the R
telescope to the angle subtended by the object at the fo
For image formed at infinity, m = = 2
eye when seen directly. fe fe
f o Focal length of objective (ii) Newtonion Telescope (Reflecting type): In
=m =
f e Focal length of eye piece newtonian telescope, objective is a concave mirror of
large focal length and large aperture. A plane mirror
− fo f − Pe
3. m = or | m | = o or m = − f o × Pe = (secondary mirror) is placed just before the focus
fe fe Po point of the objective such that it makes an angle of
Pe = 10 D, Po = 1 D 45° with the principal axis of concave mirror. Eye
10 piece is a convex lens of smaller focal length and
So m = − = − 10 smaller aperture.
1
4. Difference in working of telescope and microscope. M1
(i) Objective of telescope forms the image of a very far
off object at or within focus point of its eyepiece. M2 Objective
The microscope does the same for a small object 45°
kept just beyond the focus of its objective.
F M2 is
(ii) The final image formed by a telescope is magnified Secondary
relative to its size as seen by the unaided eye while mirror
the final image formed by a microscope is magnified
relative to its original size.
(iii) The objective of a telescope has large focal
length and large aperture while the objective of a
microscope has very small values.
5. (i) Objective should be of larger focal length in order Eyepiece
to have high magnifying power. •• Advantage of Reflecting Telescope over Refracting
(ii) Objective should be of larger aperture, in order to Type: The objective lens is replaced by concave mirror
have high resolving power. of large aperture has high gathering power and absorbs
6. Reflecting Telescope: (i) Cassegrain Telescope: very little amount of light than lens of large aperture.
Objective is concave mirror of large focal length and So final image formed in reflecting telescope is very
large aperture having a hole around its pole. The light bright.
coming from a far off object after reflection from the – Due to large aperture of mirror, its resolving power
objective is made to fall on a secondary mirror placed is also high.

324 Physics–12
D:\EG_Physics-12_(26-06-2022)\Open_Files\Ch-9\Ch-9
\ 27-Jul-2022 Ved_Goswami Proof-4 Reader’s Sign _______________________ Date __________

– As the objective is the mirror, there are no chromatic Or


aberration. Advantages of reflecting type telescope over refracting
– Use of parabolic mirror will also reduce spherical telescope.
aberration. (i) There is no chromatic or spherical aberration in
– Lens of large aperture tends to be heavy and hence reflecting type telescope.
difficult to make and support by its edges whereas the (ii) Reflecting type telescope can be made with large
mirror of equivalent optical quality weighs less and aperture.
can be supported over its entire back surface. (iii) Reflecting type telescope has only one surface for
7. Image formed by a refracting telescope will have grinding, so it is easier to make.
(i) Spherical aberration (ii) Chromatic aberration.1+1 (iv) It is cheaper than refracting telescope. 1
8. Magnifying power: It is equal to the ratio of the angle (v) Image is brighter because of its large light gathering
subtended by the image at eye when formed at infinity power, no absorption of light due to reflecting
to the angle subtended by the object at eye when seen surfaces.
without microscope i.e. directly. (vi) Resolving power is high due to large aperture of the
For a compound microscope objective. 1
L D fo
m = mo × me = ×  1 9. For a telescope magnifying power m =  1
fo fe fe
To increase the magnifying power, both the objective So for higher magnifying power objective should have
and eyepiece must have short focal length. 1 large focal length. 1
10. Compound Microscope : Image at least distance of distinct vision: The object AB to be magnified is placed
between f and 2f of the objective lens. Magnified, real, inverted image A′B′ is formed. A′B′ will act as object for the
eye piece. The distance between objective and eye piece is so adjusted that A′B′ lies within the focal length of the
eye piece. Eye piece acts as a simple microscope. The final image A′′B′′ is formed at the least distance of distinct
vision. The final image is inverted with respect to object AB.
B

E
A

D Eye
uo vo

B
A  A
2Fo A Fo O O
F Fe
o 

Objective
lens B

Eyepiece

B

Ray Optics 325


D:\EG_Physics-12_(26-06-2022)\Open_Files\Ch-9\Ch-9
\ 27-Jul-2022 Ved_Goswami Proof-4 Reader’s Sign _______________________ Date __________

Magnifying power : It is the ratio of the angle subtended (–ve) shows that it is inverted image. Not to be taken
at the eye by the final image to the angle subtended at in calculations Distance between the two lenses = fo
the eye by the object when both are at least distance of + fe = Length of tube. ½
distinct vision. 12. Objective is concave mirror of large focal length and
A ′′B′′ large aperture having a hole around its pole. The light
tan b ~− b =
O ′A ′′ coming from a far off object after reflection from the
AB objective is made to fall on a secondary mirror (convex)
tan a ~− a = placed in between its pole and focus. The light reflected
AE
from the convex mirror goes into the eye piece, which
β A ′′B′′ AE A ′′B′′ O ′A ′′ A ′′B′′ acts as a simple microscope. ½
m = = × = × =
α O ′A ′′ AB O ′A ′′ AB AB Objective
A ′′B′′ A ′B′ A ′′B′′ A ′B′
m = × = × = me × mo Secondary Mirror
AB A ′B′ A ′B′ AB A
 D  v 
Parallel rays 1
m = 1 +  ×  0 
from a distant
B
 f e   − u0  object Eye piece

[– ve sign shows that image is inverted]


 D  L 
m ~− – 1 +    Advantage of reflecting telescope over a re-
 fe   fo  fracting telescope:
u0 ~ f0 since object is very close to focus. (i) Reflecting telescope is free from chromatic
L is distance between the two lenses. aberration. ½
11. In normal adjustment: Refracting telescope in normal (ii) Image is free from spherical aberration. ½
adjustment. (iii) Image formed is brighter and resolving power is
fo fe high. ½
B
13. f0

A α Α′ Eye
α β
Ο Ο′
1
Β′
fe Β
1

  Ο
Lo Le
Α
Objective Eye piece

Magnifying power is the ratio of angle sub- Le


tended by image at eye and angle subtended by Lo
Eyepiece
object at eye when both are formed at infinity.
Objective
Refracting telescope with final image at least
β
m=  ½ distance of distinct vision
α
For small angle Magnifying power of a telescope is defined as ratio of
angle subtended at eye by final image formed at least
A ′B′
tan b ~− b = distance of distinct vision to the angle subtended by
O ′A ′ object at eye when seen directly
A ′B′ β
tan a ~− a = m=
OA ′ α
β A ′B′ OA ′ OA ′ AB
m = = × = For small angles b ~− tan b =
α O ′A ′ A ′B′ O ′A ′ O ′B
+ fo AB
=  1 a ~− tan a =
− fe OB

326 Physics–12
D:\EG_Physics-12_(26-06-2022)\Open_Files\Ch-9\Ch-9
\ 27-Jul-2022 Ved_Goswami Proof-4 Reader’s Sign _______________________ Date __________

β OB f f β A ′′B′′ AE
m=
= = o = o  ...(1) ½ m= = ×  1
α O ′ B O ′ B − ue α O ′A ′′ AB
According to lens formula A ′′B′′ O ′A ′′ A ′′B′′
= × =
1 1 1 O ′A ′′ AB AB
− =
v u f A ′′B′′ A ′B′ A ′′B′′ A ′B′
m= × = × = me × mo
v = – D and u = – ue AB A ′B′ A ′B′ AB
Which D = least distance of distinct vision  D  v 
m = 1 +  ×  o   1
1 1 1  f e   − u0 
− =
− D − ue f e –ve sign shows that the final image is inverted.
1 1 1 Both objective and eye piece should be of small focal
= +  ...(2) ½
ue f e D length in order to have higher magnifying power. ½
From (1) and (2), we get 16. (i) fo = 15 m, fe = 1 cm = 10–2 m
f − 15
 1 1  − fo  fe  Angular magnification m = − o = −2 = – 1500
m = − fo ×  +  = 1 +   1 f e 10
 fe D  fe D –ve sign shows that image is inverted. 1
14. Ray Diagram: For image at least distance of distinct (ii) Let D be the diameter of the moon in meters. r is
vision radius of the lunar orbit. Then
Moon
Eyepiece
Objective fo
uo
vo
α
B
Fo A
α 1
AF OF Image
2Fo o A Fe  O r

1
Objective
B d is the diameter of the image of the moon.

D d 3.48 × 106 d
α= = ⇒ =  ½
r fo 3.8 × 108 15
B 3.48 × 106 × 15
d= = .1373 m = 13.73 cm  ½
Working: The object AB to be magnified is placed 3.8 × 108
between f and 2f of the objective lens, magnified, real, 17. (i) When the final image is formed at
inverted image A′B′ is formed. A′B′ will act as object D = 25 cm, fo = 2 cm, fe = 6.25 cm,
for the eye piece. The distance between objective and
For eye piece fe = 6.25 cm, ve = – 25 cm
eye piece is adjusted, so that A′B′ lies within the focal
length of the eye piece. 1 1 1 1 1 1 1
= − , = −
Eye piece acts as a simple microscope. Magnified image f e ve ue 6.25 − 25 ue
A′′B′′ is formed at the least distance of distinct vision. 1 −1 1 − 4 −1 − 5
The final image is inverted with respect to object AB. = − = =
ue 6.25 25 25 25
 1
ue = – 5 cm 1
15. For ray diagram refer to the previous question
Distance between objective and eye piece = 15 cm,
Derivation fo r magnifying power: It is the ratio of the vo + ue = 15
angle subtended by the final image at eye to the angle
vo = 15 – |ue| = 15 –5 = 10 cm
subtended by the object at eye when both are placed at
the near point of eye. For objective lens, fo = 2 cm, vo = 10 cm, u0 = ?
A ′′B′′ 1 1 1 1 1 1
β = − , = −
i.e., m = , tan b ~− b = f o vo u0 2 10 u0
α O ′A ′′
AE 1 1 1 1− 5 − 4
a ~− tan a =
= − = =
AB u0 10 2 10 10

Ray Optics 327


D:\EG_Physics-12_(26-06-2022)\Open_Files\Ch-9\Ch-9
\ 27-Jul-2022 Ved_Goswami Proof-4 Reader’s Sign _______________________ Date __________

5 vo vo
u = − = − 2.5 cm mo =
, −6=
o 2  1 uo uo
Object should be placed 2.5 cm in front of the So vo = – 6uo 1
objective.
(ii) When final image is formed at ∞ 1 1 1
Using = −
i.e, ve = ∞ so the object should be placed at focus f o vo uo
pt. of eye piece 1 1 1 − 1 −6 − 7
i.e., ue = –6.25 cm = − = =
1.25 − 6uo uo 6uo 6uo 
Distance between lenses h = 15 cm i.e.,
vo + |ue| = 15 − 7 × 1.25
u0 = = − 1.46 cm  1
vo = 15 – 6.25 = 8.75 cm, fo = 2 cm ½ 6
1 1 1 1 1 1 20. (i) Given, power of objective lens = 100 D and power
= − , = − of eyepiece = 40 D, tube length = 20 cm
f 0 vo u0 2 8.75 u0
We know, power of lens
1 1 1 − 6.75 1
= − = =
u
o 8. 75 2 17.5 focal length ( in metre )
17.5
uo = − = − 2.6 cm 1
6.75  1
\Focal length (in metre) =
power of lens
18. fo = 4 cm, fe = 10 cm, uo = – 6 cm, m = ?
1
v  D
\ Focal length of objective lens (f0) = = 1 cm
m = − o 1 +  100
| u0 |  fe  
To find vo using lens formula, 1
and Focal length of eyepiece (fe) = = 2.5 cm
1 1 1 1 1 1 40
= − , = −
Since the objective lens has a smaller focal length
f 0 vo u0 4 vo −6
than the eyepiece, the instrument is a compound
1 1 1 3−2 1 microscope.
= − = =
vo 4 6 12 12  (ii) Magnification when the image is formed at infinity
vo = 12 cm is given by
12  25  35  L  D 
So m = So − 1 +  = − 2 × = −7 1
6  10  10 m =  f  f  ,

 o  e 
Length of the microscope = ue + vo
1 1 1 1 1 1 where L is length of tube
For eyepiece, = − , = − 20 × 25
f e ve ue 10 − 25 ue ⇒ m = = 200
1× 2.5
1 1 1 −2 − 5 − 7
=− − = = hence, magnification produced by instrument is 200
ue
25 10 50 50 times.
− 50 21. Telescope: Objective lens is of large focal length and
ue = = − 7.14 cm  1
7 large aperture.
Distance between lenses = |vo| + |ue| = 12 + 7.14 Microscope: Objective lens is of small focal length and
= 19.14 cm 1 small aperture.
19. fo = 1.25 cm, fe = 5 cm, uo = ?, m = 30 fo = 1.25 cm, fe = 5.0 cm,
In normal adjustment, final image is formed at infinity ve = –25 cm, m = 30
and 1 1 1 1 1 1
D 25 For objective, = − ⇒ + =
m= = =5 f o vo uo f o uo vo
e
fe 5  1
m − 30 f o uo 1.25 × uo v 1.25
m = mo × me fi mo = = =−6 ⇒ vo =
= ⇒ o =
me 5 f o + uo 1.25 + uo uo 1.25 + uo

328 Physics–12
D:\EG_Physics-12_(26-06-2022)\Open_Files\Ch-9\Ch-9
\ 27-Jul-2022 Ved_Goswami Proof-4 Reader’s Sign _______________________ Date __________

 D   vo  100 1
angular magnification m = − 1 + ×  (iv) tan a ~− a = 3
= rad
f e   uo  3 × 10 30

 25   1.25 
30 = − 1 + × 
 5   1.25 + uo  α
fo β
O
 1.25  h
30 = −6   ⇒ uo = –1.5 m Image
 1.25 + uo 
22. fo = 140 cm, fe = 5 cm Let the height of the image formed by objective
(i) In normal adjustment be h.
− fo 140 h h h 1
m= =− = − 28 , m is negative,
α= = , So =
fe 5 f o 140 140 30
i.e., final image is inverted. 1 140
− fo  fe  =h = 4.67 cm  1
(ii) m = 30
1 + 
fe D (v) Magnification produced by eye piece (me)
140  5 D 25 I
=−
1 +  = − 28 × 1.2 = − 33.61  1 me = 1 + =1+ = 6 ⇒ me = ,
5 25 fe 5 h
(iii) Distance between the objective and eye piece in I is height of final image
normal adjustment 140
I = hme = × 6 = 28 cm  1
= fo + fe = 140 + 5 = 145 cm 1 30

23. (i) Ray Diagram: (with proper labelling)

B
A A Fe
A FO Fo
1
Objective
uo B
vo
Eye piece
B
D

vo  D L  D  1 1 1 6
– = + =
Magnifying power m =
uo 1+ f  = f 1+ f   ue 5 25 25
e o e
−25
½ Or ue = = –4.17 cm 1
(ii)  m = mome = –30 (real, inverted) 6
 m = mo × me
 fo = 1.25cm and fe = 5.0 cm
m −30
Let us setup a compound microscope such that the \ m0= = = –5
final image be formed at D, then me 6
D 25 \ m0 = +vo
me= 1 +
=1+ = 6 ½ uo
fe 5
fi v0 = –5uo ½
and position of object for this image formation can
be calculated – 1 1 1 1 1 1
− = Or − =
vo uo f0 −5u0 u0 1.25
1 1 1 1 1 1
− = Or − = −6 1
ve ue fe −25 ue 5 Or =
5uo 1.25

Ray Optics 329


D:\EG_Physics-12_(26-06-2022)\Open_Files\Ch-9\Ch-9
\ 27-Jul-2022 Ved_Goswami Proof-4 Reader’s Sign _______________________ Date __________

uo = –1.5 cm fo 20
fi vo = 7.5cm ½ (ii) (a) m = − = − −2 = – 2000
fe 10
Tube length = vo + |ue|
= 7.5cm+ 4.17 cm (b)
L = 11.67cm 1 do = 3.5×106m fo = 20 m
Object should be placed at 1.5cm distance from the a
a
objective lens. u = 3.8×108m di = ?
24. (i) Objective lens Eye piece
Object
at ∞
fo& fe O1 d o di
O A1
tan a = =
u fo
B1
e
ag 3.5 ×106
l im t ∞
⇒ di = × 20 = 0.18 m
n a a
Fi 3.8 ×108

Case Based Questions


I. Total internal reflection is the phenomenon of If the object is placed at infinity, the image will be
reflection of light into denser medium at the interface formed at focus for both double concave lens.
of denser medium with a rarer medium. For this Therefore, lens maker's formula is
phenomenon to occur necessary condition is that
1  µ 2 − µ1   1 1 
light must travel from denser to rarer medium and         =  − 
f  µ1   R1 R 2 
the angle of incidence in the denser medium must be
greater than critical angle (C) for the pair of media When lens is placed in air µ1 = 1 and µ1 = µ, the lens
in the contact. The critical angle depends on nature 1  1 1 
maker's formula takes the form = (µ − 1)  − 
of medium and the wavelength of light we can show f  R1 R 2 
1
that µ = 1. A thin lens of glass (µ = 1.5) of focal length 10 cm is
sin C
immersed in water (µ = 1.33). What will be the new
1. Critical angle for water-air interface is 48.6°. What focal length?
is the refractive index of water?
2. The radius of curvature of each face of a biconcave
2. If light is going from glass to air, what will be the lens with refractive index 1.5 is 30 cm. What would
value of critical angle? be the focal length of the lens in the air?
3. Name a few phenomena caused due to internal 3. An underwater swimmer cannot see very clearly even
reflection of light.
in absolutely clear water. Why?
1 1 1 4
Ans. 1. µ = = = = 1  1 1 
sin C sin 48.6 0.75 3 Ans. 1. Given f = 10 cm, = (1.5 − 1)  − ...(1)
f  R1 R 2 
1 1
2. a µ g = ⇒ 1.5 = 1  1.5  1 1 
sin C sin C and = − 1  − ...(2)
f w  1.33   R1 R 2 
1
sin C = = 0.66 ⇒ C = 41.8°
1.5 divide (2) by (1)
3. Mirage in hot summer days and brilliance of
f w 0.5 × 1.33
diamond.    = =4
f 0.17
II. The lens maker's formula relates the focal length of a
lens to the refractive index of the lens material and the fw = 4f = 4 × 10 = 40 cm
radii of curvature of its two surfaces. This formula is 2. Here given R = 30 cm
called so because it is used by manufactures to design 1  1 1 
lenses of required focal length from a glass of given     = (µ − 1)  −
f  R1 R 2 
refractive index.

330 Physics–12
D:\EG_Physics-12_(26-06-2022)\Open_Files\Ch-9\Ch-9
\ 27-Jul-2022 Ved_Goswami Proof-4 Reader’s Sign _______________________ Date __________

 1 1 5. The following diagram shows same diamond cut in


= (1.5 − 1)  −  = − 0.5 × 2 = − 1 two different shapes.
 − 30 30  30 30
\ f = – 30 cm
3. The eye lens is surrounded by a different medium
(i.e water) than air. This changes the focal
length of the eye lens. As a result eye could not
accommodate all the image as it do in air.
III. Sparking Brilliance of Diamond: The total internal The brilliance of diamond in the second diamond will
reflection of the light is used in polishing diamonds be:
to create a sparking brilliance. By polishing the (a) less than the first (b) greater than first
diamond with
(c) same as first
specific cuts, it is
(d) will depend on the intensity of light
adjusted the most
Ans. 1. (b) 2. (a) 3. (c) 4. (d) 5. (d)
of the light rays
approaching the IV. Optical Fibre: It is a hair-thin long strand of quality
quartz or glass coated with a material of slightly lower
surface are
refractive index. It works on the principle of total
incident with an internal reflection.
angle of incidence
Construction: It consists of three main parts
more than critical
(i) Core, (ii) Cladding, (iii) Buffer
angle. Hence, they
Optical fibres are fabricated with high quality
suffer multiple reflections and ultimately come out
composite index glass/quartz fibre each fibre consist
of diamond from the top. This gives the diamond a
of core and cladding. When a signal in the form of
sparking brilliance. light is directed at one end of the fibre at a suitable
1. Light cannot easily escape a diamond without multiple angle, it undergoes repeated total internal reflection
internal reflections. This is because: along the length of the fibre and finally comes out
(a) Its critical angle with reference to air is too large at the other end. Since light undergoes total internal
(b) Its critical angle with reference to air is too small reflection at each stage there is no appreciable loss in
intensity of the light signals.
(c) The diamond is transparent
(d) Rays always enter at angle greater than critical
angle
2. The critical angle for a diamond is 24.4°. Then its Core
Cladding

refractive index is
(a) 2.42 (b) 0.413 (c) 1 (d) 1.413 1

3. The basic reason for the extraordinary sparkle of


2
suitably cut diamond is that
(a) It has low refractive index
(b) It has high transparency
(c) It has high refractive index
(d) It is very hard
4. A diamond is immersed in a liquid with a refractive
index greater than water. Then the critical angle for
total internal reflection will Core
(a) will depend on the nature of the liquid
(b) decrease Cladding
(c) remains the same (d) increase
Buffer Coating

Ray Optics 331


D:\EG_Physics-12_(26-06-2022)\Open_Files\Ch-9\Ch-9
\ 27-Jul-2022 Ved_Goswami Proof-4 Reader’s Sign _______________________ Date __________

1. Relation between critical angle (ic) and refractive


index (m) is
(a) m = sin ic (b) m2 = sin ic Water Water

1
(c) sin ic = µ (d) sin ic = µ
18° 18°
2. If ic = 35.3, then the value of ʻmʼ will be (a) (b)
(a) 3 (b) 2
3
(c) 2 (d) 3
Water Water
3. If m = 2.42, then the value of critical angle will be
(a) 41° (b) 35° 18° 18°
(c) 32° (d) 24°
(c) (d)
4. Speed of light in any medium, for which critical angle
is 45°, is given by 2. A point source of light is placed at the bottom of a tank
filled with water, of refractive index m, to a depth d.
(a) 2.12 × 108 m/s (b) 3.12 × 108 m/s The area of the surface of water through which light
(c) 4 × 108 m/s (d) 5 × 108 m/s from the source can emerge is:
5. Which relation is correct in regards of refractive
πd 2 πd 2
indices of core and cladding? (a) (b)
2(µ 2 − 1) (µ 2 − 1)
(a) mcore << mcladding (b) mcore = mcladding
(c) mcore> mcladding (d) mcore ≤ mcladding πd 2 2πd 2
(c) (d)
Ans. 1. (c) 2. (d) 3. (d) 4. (a) 5. (c) 2 µ2 − 1 (µ 2 − 1)
V. A ray of light travels from a denser to a rarer medium.
After refraction, it bends away from the normal. 3. For which of the following media, with respect to air,
When we keep increasing the angle of incidence, the the value of critical angle is maximum?
angle of refraction also increases till the refracted ray (a) Crown glass (b) Flint glass
grazes along the interface of two media. The angle
(c) Water (d) Diamond
of incidence for which it happens is called critical
angle. If the angle of incidence is increased further 4. The critical angle for a pair of two media A and B of
the ray will not emerge and it will be reflected back in refractive indices
the denser medium. This phenomenon is called total (a) 0° (b) 30° (c) 45° (d) 60°
internal reflection of light.
5. The critical angle of pair of a medium and air is 30°.
1. A ray of light travels from a medium into water at The speed of light in the medium is
an angle of incidence of 18°. The refractive index of
the medium is more than that of water and the critical (a) 1 × 108 ms–1 (b) 1.5 × 108 ms–1
angle for the interface between the two media is 20°. (c) 2.2 × 108 ms–1 (d) 2.8 × 108 ms–1
Which one of the following figures best represents
the correct path of the ray of light? Ans. 1. (a) 2. (b) 3. (d) 4. (b) 5. (b)

COMMON ERRORS

S No. Errors Corrections


1. Directions of ray diagrams Arrow marks should be shown in ray diagrams to
represent the direction of the ray of light
2. Numerical Problem based on lens/mirror formula. Proper sign convention should be used.
3. Resolving power; magnifying power of optical Dependence of various factors should be related with
instruments. equations of resolving power and magnifying power.

332 Physics–12
D:\EG_Physics-12_(26-06-2022)\Open_Files\Ch-9\Ch-9
\ 27-Jul-2022 Ved_Goswami Proof-4 Reader’s Sign _______________________ Date __________

IMPORTANT FORMULAE
Formula Symbols Application
1. 1 1 1 u = object distance To find focal length of mirror
Mirror formula + = v = image distance,
v u f
f = focal length of the mirror
2. m = magnification To find magnification caused by a
v m is negative for real images and mirror
m= -
u + ve for virtual images
3. R — To determine focal length of a
f=
2 mirror
4. Linear magnification for mirror hi = height of image To find height of object/image (+ve
h –v –f f–v h0 = height of object in case of mirror)
m= i = = =
h0 u f–u f
5. real depth To assess the depth of an object in
µ= a medium
apparent depth
6.  1 t = thicknesss To find refractive index
Normal shift d = t 1 − µ 
 
7. 1 ic = critical angle To study total inernal reflection.
µ=
sin ic Optical fibres
8. d = t sec r sin (i – r) d = lateral shift To study bending of light rays
9. • When object is situated in rarer µ2 = refractive index of denser To find value of u or v or R, while
medium medium two of them are given.
− µ1 µ 2 µ 2 − µ1 µ1 = refractive index of rarer
+ =
u v R medium
• When object is situated in
denser medium
− µ 2 µ1 µ1 − µ 2
+ =
u v R
10. R1, R2 = Radius of curvature f = Lens makers formula. To find focal
1  µ 2 − µ1   1 1  focal length of the lens m1, m2 = length of lens
=  −  Refractive index of medium 1 and
f  µ1   R1 R 2 
2 respectively
11. 1 1 1 — To find the focal length of lenses
Lens formula = −
f v u
12. Linear magnification for lens — To determine liear magnification
h v f f–v
m= i = = =
h0 u +
f u f
13. f = effective focal length of To find focal length of the
1 1 1 1 combination combination of lenses.
= + +
f f1 f 2 f 3 f1, f2, f3 = focal length of objective
14. Simple microscope: D = Least distance of distinct vision Used as reading glass and in
D laboratory
m = 1 + , when image at ‘D’
f
D
m = , when normal adjustment
f

Ray Optics 333


D:\EG_Physics-12_(26-06-2022)\Open_Files\Ch-9\Ch-9
\ 27-Jul-2022 Ved_Goswami Proof-4 Reader’s Sign _______________________ Date __________

15. Compound microscope: L = length of the tube of To study the biological specimen
(i) Final image at near point ‘D’ microscope To select the lens of required focal
v0  D L  D length
m=  1 +  = 1 +  Used by jewellers and watch makers
u0  fe  fo  fe 
(ii) For normal adjustment
v0 D L D
m= ×  ×
u0 f e fo fe
16. m: magnifying power of a compound To find magnifying power of a
microscope compound for normal adjustment.
–L D f0 = focal length of objective
m= × fe = focal length of eyepiece
fo fe
L = Distance between objective lens
and eye lens
17. m = magnifying power of a telescope To find magnifying power of a
fe = focal length of eyepiece telescope. For normal adjustment.
β
0 f b = angle subtended at the eye by
m = α = −f the image
e
a = angle subtended at the eye by
the object
18. Astronomical Telescope: Used in astronomical research.
When final image is formed at To find separation between objective
least distance of distinct vision: and eyepiece lenses.
f0  fe 
m=− 1 + 
fe D
magnifying power in normal ad-
justment
f0
m=−
fe

19. ∠i + ∠e = ∠A + ∠d i → angle of incidence For a prism


e → angle of emergence
A → angle of prism
d → angle of deviation
20. i = e, r1 = r2 At minimum deviation. For a prism
21.  A + δm  A = Angle of prism To find refractive index by
sin   m2 = refractive index of prism using a prism.
µ  2 
µ 21 = 2 = m1 = refractive index of medium
µ1 A
sin dm = angle of minimum deviation.
2
22. d = dv – dr d = Angular dispersion To find the dispersive power
δv − δr µv − µr w = Dispersive power
ω= =
δy µy −1

334 Physics–12
D:\EG_Physics-12_(26-06-2022)\Open_Files\Ch-9\Ch-9
\ 27-Jul-2022 Ved_Goswami Proof-4 Reader’s Sign _______________________ Date __________

REVISION CHART

Magnification (mirror)
Size of image −v
Magnification (m) = =
Size of object u
hi − v
where ho = height of object, hi = height of image, m = =
ho u
m is +ve for virtual image, m is – ve for real image

Reflection
The phenomenon of coming back of light into the same medium after striking a surface capable
of reflecting light is called reflection.

Convex Mirror Concave Mirror


Convex mirror is a part of hollow glass Concave mirror is a part of hollow glass
sphere whose inner surface is polished and sphere whose outer surface is polished and the
reflection takes place on surface which is reflection takes place from the surface which
vexing out. is caving in.

Refraction
The phenomenon of change in path of light as it passes obliquely from one transparent medium
to another is called refraction.

Total Internal Laws of Absolute and Refraction through


Reflection (TIR) Refraction Relative Refractive glass slab and
and Prism Index combination of
medium

Optical Instruments
Human Eye: Eye lens, ciliary muscles, Iris, Pupil. power of accommodation, far point, near
point, range of vision.
Defects Remedies
Myopia (short-sightedness) concave lens
Hypermetropia (long-sightedness) and Presbiopia convex lens
Astigmatism cylindrical lens

Compound Microscope Astronomical, Newtonion and Cassegrain


(image at D) Telescope (image at D)
v0  D f0  f 
m=− 1 +  m=−  1+ e 
u0  fe  fe  D

Ray Optics 335


D:\EG_Physics-12_(26-06-2022)\Open_Files\Ch-9\Ch-9
\ 27-Jul-2022 Ved_Goswami Proof-4 Reader’s Sign _______________________ Date __________

IMPORTANCE OF EACH TOPIC AND FREQUENTLY ASKED TYPES OF QUESTIONS

☞ Important Topics
1. Questions related to Critical angle, TIR, Glass Prism.
2. Questions on mirror formula, lens maker formula.
3. Questions based on microscope, telescope.
4. Numerical questions based on refraction on spherical surface (both convex and concave).

1. An object of length 2.5 cm is placed at a distance of 1.5 f from a concave mirror where f is the focal length of the
mirror. The length of object is perpendicular to principal axis. Find the size of image. Is the image erect or inverted?
2. Find the size of image formed in the situation shown in figure. [Ans. 5 cm, Inverted]

1.5 cm 40 cm
20 cm
O C 1.6 cm approx.
µ1 = 1 µ2 = 1.33

3. A ray of light passes through an equilateral prism in such a manner that the angle of incidence is equal to angle of
emergence and each of these angles is equal to 3/4 of angle of prism. Find angle of deviation. [Ans. 30°]
4. Critical angle for a certain wavelength of light in glass is 30°. Calculate the polarising angle and the angle of re-
fraction in glass corresponding to this. [Ans. ip = tan–1 2]
5. A light ray passes from air into a liquid as shown in figure. Find refractive index of liquid.
 [Ans. airµliquid = 3/ 2 ]

60° Air

Liquid
15
°

6. At what angle with the water surface does fish in figure see the setting sun?
µ = 1 air Sun

Water µw = 4/3

[Ans. At critical angle, fish will see the sun.]


7. In the following diagram, find the focal length of lens L2 . [Ans. 40 cm]

0 cm
f1 = 2
o
′ L1
I I
L2
15 cm

20 cm 80 cm

336 Physics–12
D:\EG_Physics-12_(26-06-2022)\Open_Files\Ch-9\Ch-9
\ 27-Jul-2022 Ved_Goswami Proof-4 Reader’s Sign _______________________ Date __________

8. Using the data given below. state which two of the given lenses will be preferred to construct a (i) telescope (ii)
Microscope. Also indicate which is to be used as objective and as eyepiece in each case.
Lenses Power (P) Aperture (A)
L1 6D 1 cm
L2 3D 8 cm
L3 10 D 1 cm
[Ans. For telescope, less L2 is chosen as objective as it aperture is largest, L3 is chosen as eyepiece as its l ocal length
is smaller.
For microscope lens L3 is chosen as objective because of its small focal length and lens L1, serve as eyepiece
because its local length is not larges.]
9. Two thin converging lens of focal lengths 15 cm and 30 cm respectively are held in contact with each other. Cal-
culate power of the combination. [Ans. P = 10 D]

Ray Optics 337


D:\EG_Physics-12_(26-06-2022)\Open_Files\Ch-9\Ch-9
\ 27-Jul-2022 Ved_Goswami Proof-4 Reader’s Sign _______________________ Date __________

ASSIGNMENT

I. Objective Type Questions (1 Mark)


1. Multiple choice questions:
(i) To get three images of a single object, one should have two plain mirrors at an angle of
(a) 60° (b) 90° (c) 120° (d) 30°
(ii) A convex lens is dipped in a liquid whose refractive index is equal to the refractive index of the lens. Then its
focal length will
(a) become zero (b) become infinite
(c) reduce (d) increase
2. Fill in the blanks:
(i) A ray of light is incident on a plane mirror at an angle of incidence 30°. The ray after reflection is deviated
through ................... .
(ii) A convex lens and concave lens, each having same focal length of 25 cm are put in correct to form a combination
of lenses. The power of the combination is ...................... .
II. Very Short Answer Type Questions (1 Mark)
3. Calculate speed of light in the medium whose critical angle is 45°.
4. A lens of glass is immersed in water. What will be the effect on the power of the lens?
III. Short Answer Type Questions-I (2 Marks)
5. Draw a ray diagram to show the image formation by a concave mirror when the object kept between its focus and
the pole. Using this diagram, derive the magnification formula for the image formed.
6. Why does sky appear blue? Explain.
7. (a) State the conditions for total reflection to occur. 
(b) Write the relation between the refractive index and critical angle for a given pair of optical media. 
IV. Short Answer Type Questions-II (3 Marks)
8. You are given three lenses L1, L2 and L3 each of focal length 20 cm. An object is kept at 40 cm in front of L1, as
shown. The final real image is formed at a focus f of L3. Find the separations between L1, L2 and L3.
L1 L2 L3

O
I

40 cm 20 cm

9. A spherical surface of radius of curvature R, separated a rarer and a denser medium n1 Rarer n2 Denser
as shown is the fig.
Complete the path of the incident ray of light, showing the formation of real image. O P C
Hence derive the relation connecting object distance u image distance v and radius
of curvature R and refractive indices n1 and n2 of two media.
10. (a) Define SI unit of power of a lens.
(b) A plano convex lens is made of glass of refractive index 1.5. The radius of curvature of the convex surface is
25 cm.
(i) Calculate the focal length of the lens.
(ii) If an object is placed 50 cm in front of the lens, find the nature and position of the image formed.
[CBSE 2022]


338 Physics–12
D:\EG_Physics-12_(26-06-2022)\Open_Files\Ch-10\Ch-10
\ 27-Jul-2022 Ved_Goswami Proof-4 Reader’s Sign _______________________ Date __________

Topics Covered
10 Wave Optics

10.1 Wavefronts and Huygen’s Principle 10.2 Interference of Light Waves and Y.D.S.E.
10.3 Diffraction at Single Slit

C hapter map
Wave Optics

Wavefronts and Interference of waves Diffraction of light at


Huygen’s principle a single slit

Verification of laws Principle of superposition Single slit diffraction and


of reflection and of waves expression for width of
refraction of light central maxima

Youngs double slit Difference between


experiment interference and
diffraction

Topic 1. Wavefronts and Huygen’s Principle


•• Wavefront: It is a continuous locus of all the points Ray
in the same phase of vibrations. Wavefronts are
perpendicular to the direction of propagation of energy Cylindrical
S
(i.e., ray of light). wavefront

•• Type of Wavefront: (a) Spherical Wavefront (b)


Cylindrical Wavefront (c) Plane Wavefront
(a) Spherical Wavefront: Wavefronts are spherical
when source of light is a point source. (c) Plane Wavefront: Plane wavefront is a limited
or a small part of a large spherical or cylindrical
Ray wavefront.
Plane wavefronts

S
Spherical
wavefronts
(b) Cylindrical Wavefront: When the source of light
is linear in shape, for example fine rectangular slit
then wavefront is cylindrical in shape.

339
D:\EG_Physics-12_(26-06-2022)\Open_Files\Ch-10\Ch-10
\ 27-Jul-2022 Ved_Goswami Proof-4 Reader’s Sign _______________________ Date __________

•• The following diagrams shows wavefronts The new wavefront at any later time is given by the
corresponding to forward envelope of the secondary wavelets at that
(i) A parallel beam of light. time. For example
•• Reflection of light using Huygens Principle: Consider
Wavefront
a plane wavefront AB reflected at the interface of the two
media. When the wavefront strikes the reflected surface
Ray at point B the secondary wavelets travel back into the
same medium.
Incident wavefront Reflected wavefront
N N′

Y D
A
(ii) A converging beam of light.
P
Spherical i r
wavefront i r
X B C Y
During the time, the disturbance from A reach the point
Ray C, the secondary wavelets from B have spread over a
hemisphere of radius BD = vt, where v is the speed of
the wave and t is the time taken for the wave to reach
from A to C.
(iii) A diverging beam of light. Also AC = vt
The tangent plane CD is the new reflected wavefront.
Let i and r be the angle of incidence and angle of
reflection respectively.
∠ABC = ∠i, ∠BCD = ∠r
Ray In DABC and DDCB,
∠BAC = ∠CDB = 90°
Spherical BC = BC (common)
wavefront AC = BD = vt
•• Huygen’s Principle: So DABC ≅ DDCB
(i) Each point on a wavefront act as a source of So by cpct ∠ABC = ∠BCD or ∠i = ∠r
secondary wavelets, which add up to give a i.e., the angle of incidence is equal to angle of reflection.
wavefront at any later time. Thus the first law of reflection is verified.
(ii) The secondary wavelets spread out in all directions Since the incident ray PB, the normal BN and the
with the speed of light in the given medium. reflected ray BD are respectively perpendicular to
incident wavefront AB, the reflecting surface XY and
the reflected wavefront CD, therefore they all lie in the
plane of paper. So we can say that incident ray, reflected
ray and the normal at the point of incidence all lie in the
same plane. Thus the second law is verified.
•• Reflected Wavefronts:
(i) When plane wavefront strikes a concave mirror :
Incident Reflected
wavefront wavefront
A
A′
B F
B′
C′
C
Reflected wavefront is spherical.
340 Physics–12
D:\EG_Physics-12_(26-06-2022)\Open_Files\Ch-10\Ch-10
\ 27-Jul-2022 Ved_Goswami Proof-4 Reader’s Sign _______________________ Date __________

(ii) When plane front strikes a convex mirror : •• When a plane wavefront is refracted by a convex lens:
Incident
wavefront
A Refracted
Reflected wavefront wavefront
A

F
B B
P F C C
C

Incident wavefront
•• When a plane wavefront is reflected by a plane
mirror:
Incident wavefront Reflected wavefront
•• Refracted Wavefront when a plane front goes from N′
N
rarer to a denser medium:
N Y D
A
Incident
wavefront S
v1 > v2 B
i r
v1t i r
X Y
S B C
i Rarer–v1 •• Verification of laws of refraction of light using
i
X
A r C Y Huygens Principle: When light (plane wavefront)
r Denser–v2 goes from rarer to denser medium : Consider a plane
v2t
wavefront AB striking on a plane surface XY, which
Refracted
wavefront separates two medium of refractive index m1 and m2 and
D m2 > m1.
N
•• Refracted wavefront when a plane front goes from Incident
denser to a rarer medium: wavefront
B

v1t
Incident B P
wavefront v2 > v1
v1t i Rarer–v1
i i
X Y
Denser–v1 A r C
Denser–v2
i r
C v2t
A r
Rarer–v2 Refracted
r v2t wavefront
D
Refracted
v1 – v2 wavefront Let v1 and v2 be the speed of wave in rarer and denser
E medium respectively.
•• When a plane wavefront is refracted by a prism: The wavefront first strikes at the point A and then at the
other points towards C. According to Huygen’s principle,
Incident
wavefront the secondary wavelets from point A are growing in
A Refracted
wavefront
medium of refractive index m2 whereas the secondary
wavelets from B are still travelling in the first medium
B
A′ of refractive index m1.
Let the time taken by the disturbance to travel from B to
B′ C is t, then BC = v1t
C During this time, the secondary wavelets from A must
C′ have spread over a hemisphere of radius = AD = v2t in
the denser medium. So CD is the refracted wavefront,
v1t > v2t.

Wave Optics 341


D:\EG_Physics-12_(26-06-2022)\Open_Files\Ch-10\Ch-10
\ 27-Jul-2022 Ved_Goswami Proof-4 Reader’s Sign _______________________ Date __________

Let i and r be the angle of incidence and angle of refracted ray and normal at the point of incidence all in
refraction respectively. the same plane.
BC •• On reflection of light there is no change in speed,
In DABC. sin i =
AC frequency or wavelengths of light. i.e., the speed,
frequency or wavelength of the reflected light is same
AD
In DADC, sin r = as that of the incident light.
AC
•• (i) When light is refracted then there is no change in
sin i BC AC BC v1t v1 frequency of the light: i.e., Frequency of the incident
Thus, = × = =
sin r AC AD AD v2t v2 light is same as that of refracted light. Because the
frequency of light is the characteristic of the source
This ratio of speed of the wave in two medium is called
of light emitting it and not the medium.
the refractive index of the second medium w.r.t. first
medium. Hence Snell’s law is proved. (ii) When light goes from one medium to another the
speed of light change, thus the wavelength of the
As the ray PA is perpendicular to incident wavefront
light will change.
AB, AN is perpendicular to XY the dividing surface and
The relation between speed, wavelength and frequency
AD is perpendicular to the refracted wavefront CD. XY,
is, V = nl
AN and AD are respectively ^ to AB, AN and CD and
all these are perpendicular to the plane of the paper so V → speed, n → frequency, l → wavelength
then the second law of refraction i.e., the incident ray, As light goes from denser to rarer medium, the speed of
light increases, thus wavelength of light also increase.

EXERCISE 10.1
I. Objective Type Questions (1 Mark) 2. Fill in the blanks.
1. Choose the correct answers from the given options (i) Wavefronts are .................... to the direction of
(MCQs). propagation of energy.
(i) The wave theory of light in its original form was (ii) When the source of light is .................... in shape,
first postulated by then wavefront is cylindrical in shape.
(a) Christian Huygens (iii) When plane wavefront strikes a concave mirror,
(b) Newton reflected wavefront is ...................... .
(c) Thomson 3. State True or False.
(d) de Broglie (i) Wavefront is a discontinuous locus of all the points
(ii) Huygen’s wave theory of light can not explain: in the same phase of vibrations.
(a) Photoelectric effect
(ii) We can verify the first law of reflection with the
(b) interference
help of Huygen principle.
(c) diffraction
(d) polarisation (iii) Verification of laws of refraction of light using
(iii) Light appears to travel in straight line since: Huygen principle is not possible.
(a) Its wavelength is very large 4. Match the Columns
(b) Its wavelength is very small Column-I Column-II
(c) It is absorbed by atmosphere Spherical
wavefront
(d) none of these (i) A parallel beam (a) Ray
(iv) Which type of wavefront not exist of light
(a) spherical (b) cylindrical
Wavefront
(c) plane (d) line
(v) For the light diverging from a point source (ii) A converging beam (b) Ray

(a) the wavefront is spherical. of light


(b) the intensity decreases in proportion to the
distance squared.
(c) the wavefront is parabolic. (iii) A diverging beam (c)
(d) the intensity at wavefront does not depend on of light
Ray

distance Spherical
wavefront

342 Physics–12
D:\EG_Physics-12_(26-06-2022)\Open_Files\Ch-10\Ch-10
\ 27-Jul-2022 Ved_Goswami Proof-4 Reader’s Sign _______________________ Date __________

For Q. 5 there are two statements labelled as assertion B


(A) and reason (R). Select the correct answer to these
questions from the codes (a), (b), (c) and (d) as given
below. 1
(a) Both A and R are true and R is the correct P1 P2
explanation of A. A
2
(b) Both A and R are true but R is not the correct
explanation of A. 10. Define wavefront of a travelling wave. Using
(c) A is true but R is false. Huygen’s principle, obtain the law of refraction at
a plane interface when light passes from a denser to
(d) Both A and R are false.
rarer medium. [AI 2020]
5. Assertion (A): When a monochromatic light is
incident on a surface separating two media, the IV. Short Answer Type Questions-II (3 Marks)
reflection and refraction of light both have the same 11. How is a wavefront defined? Using Huygen’s
frequency as the incident frequency. construction draw a figure showing the propagation
Reason (R): The frequency of monochromatic light of a plane wave reflecting at the interface of the two
depends on media. media. Show that the angle of incidence is equal to
angle of reflection. [Delhi 2008, 2019]
II. Very Short Answer Type Questions (1 Mark) OR
1. What type of wavefront will emerge from a (i) point Define a wavefront. Using Huygens’ principle, verify
source and (ii) distant light source? [Delhi 2009] the laws of reflection at a plane surface.
2. A light wave enters from air into glass. How will the [CBSE 2018]
following be affected: 12. Draw wavefronts corresponding to
(i) energy of the wave (i) a parallel beam of light
(ii) frequency of the wave? (ii) converging beam (iii) diverging beam.
13. A plane wavefront, of width x, is incident on an
3. When monochromatic light travels from one medium
air-water interface and the corresponding refracted
to another, its wavelength changes but frequency
wavefront has a width z as shown. Express the
remains same. Explain. [Delhi 2011]
refractive index of air with respect to water, in terms
4. How is a wavefront different from a ray? of the dimension shown. [S.P. 2011]
[AI 2015 C]
5. What is the shape of the wavefront on earth for
sunlight? [NCERT Exemplar] Air
III. Short Answer Type Questions-I (2 Marks) y
x
6. What is a wavefront? Can two wavefronts cross each i B
other? Give reason for your answer. A
r
7. Draw the shape of the refracted/reflected wavefront w z
when a plane wavefront is incident on (i) Prism Water

(ii) Convex mirror. Give brief explanation for the


construction. [S.P. 2011]
8. Use Huygens’ principle to show the propagation 14. State Huygen’s principle of wave theory. Sketch the
of a plane wavefront from a denser medium to a wavefront emerging from (i) point source of light and
rarer medium. Hence find the ratio of the speeds of
(ii) linear source of light like a slit. [Delhi 2009 C]
wavefronts in the two media. [AI 2015, 2016]
15. What is a wavefront? How does it propagate? Using
9. Define the term ‘wavefront of light’. A plane
Huygen’s principle, explain reflection of a plane
wavefront AB propagating from denser medium (1)
into a rarer medium (2) is incident on the surface wavefront from a surface and verify the laws of
P1P2 separating the two media as shown in figure. reflection. [AI 2020]
Using Huygen’s principle, draw the secondary V. Long Answer Type Questions (5 Marks)
wavelets and obtain the refracted wavefront in the 16. (a) State Huygen’s principle. Using the principle
diagram.[Delhi 2020] draw a diagram to show how a plane wavefront

Wave Optics 343


D:\EG_Physics-12_(26-06-2022)\Open_Files\Ch-10\Ch-10
\ 27-Jul-2022 Ved_Goswami Proof-4 Reader’s Sign _______________________ Date __________

incident at the interface of the two media gets 17. Define the term, ‘refractive index’ of a medium. Verify
refracted when it propagates from a rarer to a Snell’s law of refraction when a plane wavefront is
denser medium. Hence, verify Snell’s law of propagating from a denser to a rarer medium.
refraction. [Delhi 2019]
(b) When monochromatic light travels from a rarer to 18. (a) Define a wavefront.
a denser medium, explain the following, giving (b) Draw the diagram to show the shape of plane
reason: wavefront as they pass through (i) a thin prism
(i) Is the frequency of reflected and refracted light and (ii) a thin convex lens. State the nature of
same as the frequency of incident light? refracted wavefront.
(ii) Does the decrease in speed imply a reduction in (c) Verify Snell’s law of refraction using Huygens’s
the energy carried by light wave? [Delhi 2013] principle. [CBSE S.P. 2020-21]

Answers 10.1
I. Objective Type Questions 6. Wavefront is a continuous locus of all the points
1. (i) (a) (ii) (a) (iii) (b) (iv) (d) vibrating in the same phase. No, two wavefronts cannot
(v) (a) and (b) cross each other. 1
2. (i) perpendicular (ii) linear Because, if they intersect, then at the point of
(iii) spherical intersection, there will be two directions of propagation
3. (i) False (ii) True (iii) False
of energy, which is not possible. 1
4.(i)-(b) (ii)-(a) (iii)-(c)
5. (c) 7.
Incident
II. Very Short Answer Type Questions A
wavefront
Refracted
1. (i) Spherical wavefront wavefront
(ii) Plane wavefront. A′
2. (i) Energy of the wave may decrease if the part of the B
light wave is reflected back into air. If there is no
  ½
reflection, then energy will not change. B′
(ii) Frequency of the wave remains same. C

3. Frequency of the refracted wave is the same as that of the C′


incident wave because frequency in the characteristic
of the source emitting the wave while wavelength is A′
the characteristic of the medium in which the wave is Incident
propagation. So frequency remains same. wavefront
A
4. Wavefront is a surface at each point of which the
phase is same: whereas a ray indicates the direction of B B′ P
F
propagation of a wave. A ray is normal to the wavefront   ½
at each point. C

C′ Reflected
wavefront
Wavefront
Refraction from a Prism : When the incident
S
wavefront strikes the prism at B, the secondary wavelets
travel in glass, where as the wavelets from A are still
Ray

travelling in air. As speed of light in air is more than


speed of light in glass so the distance AA′ > BB′ thus
5. Spherical with huge radius as compared to the earth’s the refracted wavefront is A′B′ not parallel to AB. ½
radius so that it is almost a plane.
344 Physics–12
D:\EG_Physics-12_(26-06-2022)\Open_Files\Ch-10\Ch-10
\ 27-Jul-2022 Ved_Goswami Proof-4 Reader’s Sign _______________________ Date __________

Reflection from convex mirror: When the incident This ratio of speed of the wave in two medium is called
wavefront strikes the mirror, wavefront strikes the the refractive index of the second medium w.r.t. first
mirror at P first, the reflected wavelets from point P start medium.
back into the same medium after (reflection where as 9. It is a continuous locus of all the points of medium
the wavelets from A and C are still moving towards the vibrating in the same phase at a given instant of time.
mirror, thus the reflected wavefront will be spherical.½
8. Consider a plane wavefront AB, striking on a plane surface B
vt Denser (1)
XY, which separates two medium of refractive index m1
and m2 and m1 > m2. vt
Let v1 and v2 be the speed of wave in denser and rarer A ct
medium respectively. Rarer (2)
The wavefront first strikes at the point A and then at
the other points towards C. According to Huygens’ 10. Wavefront: It is a locus of all the disturbances
principle, the secondary wavelets from point A are oscillating with energy in same phase at a given instant.
growing in medium of refractive index m2 whereas the For law of reflection, see theory part.
secondary wavelets from B are still travelling in the 11. Consider a plane wavefront AB reflected at the interface
first medium of refractive index m1. of the two media. When the wavefront strikes the
Let the time taken by the disturbance to travel from B reflected surface at point B the secondary wavelets
to C is t, then BC = v1t. travel back into the some medium.
During this time, the secondary wavelets from A must During the time the disturbance from A reach the point
have spread over a hemisphere of radius = AD = v2t in C, the secondary wavelets from B have spread over a
the rarer medium. So CD is the refracted wavefront v1t hemisphere of radius BD = vt. Where v in the speed of
< v2t. the wave and t is the time taken from the wave to reach
from A to C.
Also AC = vt ½
Incident B
wavefront The tangent plane CD is the new reflected wavefront.
i v1t
Denser–v1
Let i and r be the angle of incidence and angle of
i
C
reflection respectively.
A r
Rarer–v2
r
∠ABC = ∠i, ∠BCD = ∠r ½
v2t
Refracted
In DABC and DDCB,
v1 < v2 wavefront ∠BAC = ∠CDB = 90°
D
½ BC = BC (common)
Let i and r be the angle of incidence and angle of AC = BD = vt
refraction respectively.
So DABC ≅ DDCB ½
BC
In DABC, sin i =  ½ So by CPCT, ∠ABC = ∠BCD
AC
or ∠i = ∠r ½
AD
In DADC, sin r =  ½ Incident wavefront Reflected wavefront
AC
N N′
sin i BC AC BC
Thus, = × = Y
sin r AC AD AD A D
1
v1t v1 P
= =  1 r
v2t v2 i
i r
X Y
B C

Wave Optics 345


D:\EG_Physics-12_(26-06-2022)\Open_Files\Ch-10\Ch-10
\ 27-Jul-2022 Ved_Goswami Proof-4 Reader’s Sign _______________________ Date __________

12. See theory part under the heading ‘Type of wavefront’. (ii) No, there will be no decrease in energy of the wave,
as the energy carried by a wave depends upon its
13.
amplitude and not on speed of the wave. 1½
17. Consider the triangles BAD and ACD figure below.
Air
y BD c1t
x sin i = sin ∠BAD = =  ½
i B 1½ AD AD
A AC c2t
w r sin r = sin ∠ADC = =  ½
z AD AD
Water
sin i ct c
= 1 = 1 = a constant 1
sin r c2t c2
N A¢
sin i y / AB y P
a
mw == = v1t
sin r w / AB w Incident
wavefront
w B Denser
w
ma = . 1½ Medium 1 i i
y v1
X
i
Y 1
D
14. Huygen’s Principle: A r

(i) Each point on a wavefront is a source of secondary Medium 2 r v2t


v2
waves, which add up to give a wavefront at any later
v1 <v2 C Refracted Rarer
time. wavefront
The secondary wavelets spread out in all directions This constant is called the refractive index of the second
with the speed of light in the given medium. 1 medium (2) with respect to the first medium (1). 1
(ii) The new wavefront at any later time is given by the
c1 n
forward envelope of the secondary wavelets at that = 2 = 1n2 1
c2 n1
time. 1
18. (a) Definition of wavefront refer ans. 15
See theory part under the heading ‘Huygen’s principle’.
(b) Refer ans. 7 Ray diagram showing shapes of
 ½
wavefront
See theory part under the heading ‘Cylindrical
Wavefront’. ½
Incident
15. Wavefront: It is a continuous locus of all the points plane
wavefront
vibrating in the same phase. The wavefront is represented
(i) Transmitted
by a ray bundle and the propagation is accomplished Convex lens plane
wavefront
by the transferring those rays across the space.
Now Refer Ans.11
F
16. (a) See theory part. 1+1
(b) (i) The frequency of the reflected and the refracted (ii) Spherical wavefront
at radius R
light are the same as the frequency of the incident
light. 1½ (c) Proof of Snell’s law, refer ans. 17

Topic 2. Interference of light Waves and Y.D.S.E.


•• Principle of Superposition of Waves: When two or Interference: The phenomenon of redistribution
more waves reach at a point simultaneously, the resultant of energy in the region when two waves of same
displacement at a point is given by algebraic sum of frequency travelling in the same direction with zero or
displacement due to individual waves. constant initial phase difference superimpose is called
If y1, y2, y3, ... yn are the displacements caused by interference.
individual waves then the resultant displacement y is
given by y = y1 + y2 + y3 + ... yn
346 Physics–12
D:\EG_Physics-12_(26-06-2022)\Open_Files\Ch-10\Ch-10
\ 27-Jul-2022 Ved_Goswami Proof-4 Reader’s Sign _______________________ Date __________

Resultant wave For constructive interference i.e., maxima


cos f = 1 or f = 0, 2p, 4p, 8p, ....
Wave 2 f = 2np where n = 0, 1, 2, 3, ...
Path difference
t
Disp. Wave 1 λ λ
Dx = φ= × 2 nπ = nλ
2π 2π
Dx = nl (n = 0, 1, 2, ....)
It is of two types : (a) Constructive (b) Destructive A max = a12 + a22 + 2a1 . a2 = (a1 + a2)2
2

(a) Constructive interference : When two waves Amax = a1 + a2


of same frequency travelling in same direction For distructive interference i.e., minima
superimpose in phase then the resultant amplitude
cos f = – 1
is the sum of the individual amplitudes.
(b) Destructive interference : When two waves f = p, 3p, 5p, 7p ....
superimpose out of phase, resultant is shown by line = (2n – 1) p, (n = 1, 2, 3, ...)
R in the fig. Path difference
λ λ λ
Wave 1 = φ= × (2n − 1) π = (2n − 1)
2π 2π 2
Resultant A min = a1 + a2 – 2a1a2 = (a1 – a2)2
2 2 2

Disp R Time Amin = (a1 – a2)


Resultant intensity:
Wave 2 I ∝ a2
•• Condition for constructive and destructive I = ka2
interference: Suppose displacement of two light waves I1 = ka12, I2 = ka22
from the two sources at a point P at any time t is given by
ka2 = ka12 + ka22 + 2ka1a2 cos f
y1 = a1 sin wt
I = I1 + I 2 + 2 I1I 2 cos φ
y2 = a2 sin (wt + f)
Where f = phase difference between two waves To Show that Energy is Conserved in Interference:
In interference pattern, intensity at maxima and minima:
a1, a2 = amplitudes
Imax ∝ (a1 + a2)2
w = angular frequency
Imin ∝ (a1 – a2)2
y = y1 + y2 = a1 sin wt + a2 sin (wt + f)
Avg. Intensity:
= a1 sin wt + a2 sin wt . cos f
I + I min
+ a2 sin f cos wt Iavg. = max
2
= (a1 + a2 cos f) sin wt + a2 sin f cos wt
(a1 + a2 ) 2 + (a1 − a2 ) 2
Let a1 + a2 cos f = A cos q ...(1) Iavg ∝ = a12 + a22
2
a2 sin f = A sin q ...(2)
If there is no interference, I at every point will be
y = A cos q sin wt + A sin q cos wt
i.e., I = I1 + I2
y = A sin (wt + q)
I1 ∝ a12; I2 ∝ a22
It is a progressive wave and of the same frequency, phase
I = I1 + I2 = ka12 + ka22= k(a12 + a22)
angle q and amplitude A
Average energy in interference pattern is same as total
Square and add (1) and (2)
energy when the waves interfere. So there is no violation
A2 cos2 q + A2 sin2q = (a1 + a2 cos f)2 + (a2sin f)2 of conservation of energy.
A2 (cos2 q + sin2 q) = a12 + a22 cos2 f •• Young’s Double Slit Experiment (YDSE): A source
+ 2a1a2 cos f + a22 sin2 f of monochromatic light illuminates a narrow slit ‘S’.
A2 = a12 + a22 (cos2 f + sin2 f) Two narrow slits S1 and S2 are arranged symmetrically
+ 2a1a2 cos f and parallel to the slit S at a small distance from slit S.
A = a1 + a2 + 2a1a2 cos f
2 2 2
Distance between S1 and S2 in very small.

Wave Optics 347


D:\EG_Physics-12_(26-06-2022)\Open_Files\Ch-10\Ch-10
\ 27-Jul-2022 Ved_Goswami Proof-4 Reader’s Sign _______________________ Date __________

When an observation screen is placed little far away from 4. The interfering waves should be is the same state of
the two slits, alternate bright and dark bands appear on polarisation.
the screen. These bands are called interference fringes. 5. The amplitudes of the interfering waves should be
When one of the slits is closed, bands of darkness and equal for better contrast between fringes.
lightness will disappear and intensity becomes uniform. 6. The distance between two coherent sources should
This experiment confirms the wave nature of light. As be small and the distance between the two sources
SS1 = SS2 So these waves are initially in phase. So S1 and the screen should be large, for sufficient width
and S2 act as coherent sources of monochromatic light. of the fringes.
At the point O, waves are meeting in phase because the •• Theory behind Young’s Double Slit Experiment: A
path difference is 0 and intensity of light is maximum. narrow slit S is placed in front of a monochromatic source
At places of minima, distructive interference takes place of light. Two narrow slits S1 and S2 are placed in front of
because crest of one wave falls on trough of the other. it such that SS1 = SS2.
Trough Let S1S2 = d
Crest P is the point of observation on screen which is placed
S – Source at a distance D from the two slits S1 and S2.
S1, S2 are two
narrow slits
Path difference between the waves reaching at P from
S1 the two sources.
S Dx = S2P – S1P
2
S2  d
S2P2 = S2A2 + AP2 D 2 +  y + 
 2
2
2 2 2  d
Screen S1P2 = S1B + BP = D +  y − 
2  
2 2
Superposition of wavefronts  d  d d
S2P2 – S1P2 =  y +  −  y −  = 4 y = 2 yd
•• Relation between Slit Width and Intensity of Light:  2  2 2
(S2P + S1P) (S2P – S1P) = 2yd
I1 a2 w
= 12 = 1 2 yd 2 yd yd
I2 a2 w2 S2P – S1P = = =
S1P + S2 P 2 D D
where w1, w2 = width of slit (1) and (2)

P
Coherent Sources: The two sources which continuously S1
B
emit light of same frequency with 0 or constant phase y
difference between them are called coherent sources. d/2

Need of Coherent Sources: Light is emitted by S


d
O
individual atoms and not in the bulk. Even the smallest
d/2
source of light contains millions of atoms and emission
of light takes place independently. S2 A
D
In an atom, on an average, 108, jumps take place in a
second. So, the phase difference will change 108 time Approximation : D >> d ⇒ S1P + S2P ≈ 2D
in 1 second. yd
Therefore interference pattern will change very fast Path Difference : Dx = S2 P − S1P =
D
i.e., 108 times in 1 second, and our eye cannot see such For Maxima, Dx = nl (for nth maxima)
change and a uniform illumination is seen on the screen. yn d
⇒ = nl
•• Conditions for Sustained Interference Pattern: D
1. The two sources must be coherent i.e. they should give y2 d
out waves of zero or constant initial phase difference. For n = 0,    =0 (Central maximum)
D
2. The two sources should give waves of same y1d λD
wavelength continuously. For n = 1, = λ ⇒ y1 =
D d
3. The two sources should be narrow. (Position of 1st maxima)
348 Physics–12
D:\EG_Physics-12_(26-06-2022)\Open_Files\Ch-10\Ch-10
\ 27-Jul-2022 Ved_Goswami Proof-4 Reader’s Sign _______________________ Date __________

y2 d 2λD λD
For n = 2, = 2 λ ⇒ y2 = = [2n − 1 − 2n + 2 + 1]
D d 2d
(Position of 2nd maxima) 2λD λD
Distance between two successive maxima: = =
2d d
nλD (n − 1) λD λD
yn – yn – 1 = − = All fringes are of equal width.
d d d
Between two maxima, there is always a minima Graph:
Fringe width of dark fringe :
I
λD
bdark =
d
For minima, Dx = λd = (2n − 1) λ –3l –2l –l O l 2l 3l Position on screen
D 2 d d d d d d
D λ λD
For n = 1, y′1 = = (First dark fringe) Note: For interference fringes to be seen, the condition
d 2 2d which must be satisfied :
3D λ 3 λD
For n = 2, y′2 = = (Second dark fringe) s λ
d 2 2 d <
S d
5D λ 5 λD
For n = 3, y′3 = = (Third dark fringe) s = size of source S
d 2 2 d
Distance between two successive minima: S = Distance of source S from plane of
bbright = yn′ − yn′ − 1 two slits.
Otherwise interference pattern produced by different
λD λD
= (2n − 1) − [2 (n − 1) − 1] sources will overlap
2d 2d

EXERCISE 10.2
I. Objective Type Questions (1 Mark) (iv) In a Young‘s double slit experiment, the source is
1. Choose the correct answers from the given options white light. One of the holes is covered by a red
(MCQs). filter and another by a blue filter. In this case
(i) Two waves having intensity in the ratio 25 : 4
produce interference. The ratio of the maximum (a) there shall be alternate interference patterns
to minimum intensity is of red and blue.
(a) 5 : 2 (b) 7 : 3 (b) there shall be an interference pattern for red
(c) 49 : 9 (d) 9 : 49 distinct from that for blue.
(ii) Two waves y1 = A1 sin (wt – b1) and y2 = A2 sin (c) there shall be no interference fringes.
(wt – b2) superimpose to form a resultant wave
(d) there shall be an interference pattern for red
whose amplitude is
mixing with one for blue.
(a) A12 + A 22 + 2A1A 2 cos (β1 − β 2 ) (v) Two sources S1 and S2 of intensity I1 and I2 are
placed in front of a screen [fig. (a)]. The pattern
(b) A12 + A 22 + 2A1A 2 sin (β1 − β 2 ) of intensity distribution seen in the central portion
(c) A1 + A2 is given by figure (b).
(d) | A1 + A2|
I
(iii) In a wave, the path difference corresponding to a S1 x
phase difference of f is x
S1
π π λ λ
(a) φ (b) φ (c) φ (d) φ X′
x=0
X
2λ λ 2π π (a) (b)

Wave Optics 349


D:\EG_Physics-12_(26-06-2022)\Open_Files\Ch-10\Ch-10
\ 27-Jul-2022 Ved_Goswami Proof-4 Reader’s Sign _______________________ Date __________

In this case which of the following statements are (c) A is true but R is false.
true? (d) Both A and R are false.
(a) S1 and S2 have the same intensities. 5. Assertion (A): In interference all the fringes are of
same width.
(b) S1 and S2 have a constant phase difference.
Reason (R): In interference fringe width is
(c) S1 and S2 have the same phase. independent of position of the fringe.
(d) S1 and S2 have the same wavelength. 6. Assertion (A): In Young's double slit experiment, the
(vi) The shape of the interference fringes in Young’s fringes become distinct if one of the slits is covered
double slit experiment when D (distance between with cellophane paper.
slit and screen) is very large as compared to fringe Reason (R): The cellophane paper decreases the
width is nearly 1 wavelength of light.
(a) straight line (b) parabolic II. Very Short Answer Type Questions (1 Mark)
(c) circular (d) hyperbolic 1. If s is the size of source giving light of wavelength l,
[S.P. 2019-20] separation between the slits is d, D is its distance from
the plane of slits is, what should be the criterion for the
2. Fill in the blanks.
interference fringes to be seen.
(i) ........................... interference occurs when two
waves of same frequency travelling in same 2. No interference pattern is detected when two coherent
direction superimpose in phase sources are infinitely close to each other. Why?
3. State the essential conditions for two light waves to
(ii) Two sources with zero or constant phase difference
be coherent.
between them are called ............. sources.
4. A slit, S is illuminated by a monochromatic source of
(iii) Alternative bright and dark band are called
light to give two coherent sources P1 and P2. These
.................... fringes.
give bright and dark bands on a screen. At a point R,
3. State True or False. on the screen, there is a dark fringe what relationship
2π must exist between the lengths P1R and P2R?
(i) Path difference = × phase difference.
λ R
(ii) In Young’s double slit experiment, the fringe width
is minimum for violet colour.
(iii) Non, reflecting glass owes its property to P1
constructive interference.
(iv) Fringe width of the fringes is increase when the S M
apparatus is immersed in water.
4. Match the Columns
P2
Column-I Column-II
(i) Destructive interference (a) cos f = +1
(ii) constructive interference (b) Young
5. How will the intensity of maxima and minima, in
s λ the Young’s double slit experiment change, if one of
(iii) Condition for interference (c) <
S d the two slits is covered by a transparent paper which
of fringes
transmits only half of the light intensity?
(iv) Double slit experiment (d) cos f = –1 [S.P. 2011]
For Q. 5 and 6 there are two statements labelled as 6. Explain the statement ‘light added to light can produce
assertion (A) and reason (R). Select the correct answer darkness’.
to these questions from the codes (a), (b), (c) and (d) as 7. What will be the effect in interference fringes if red
given below. light is replaced by blue light?
(a) Both A and R are true and R is the correct 8. Two slits in Young’s double slit experiment are
explanation of A. illuminated by two different lamps emitting light of
(b) Both A and R are true but R is not the correct same wavelength. Will you observe the interference
explanation of A. pattern? Justify your answer. [AI 2008C]

350 Physics–12
D:\EG_Physics-12_(26-06-2022)\Open_Files\Ch-10\Ch-10
\ 27-Jul-2022 Ved_Goswami Proof-4 Reader’s Sign _______________________ Date __________

9. What change in the interference pattern in Y.D.S. 20. The distance between two slits in Young’s interference
experiment will be observed when light of smaller experiment is 0.03 cm. The fourth bright fringe is
frequency is used? obtained at a distance of 1 cm from central fringe
10. Why are coherent sources necessary to produce a on a screen placed at a distance of 1.5 m from slits.
sustained interference pattern? [Delhi 2013] Calculate the wavelength of light used.
11. How does the angular separation of interference 21. Find the ratio of intensities at two points on a screen
fringes change in Young’s experiment, if the distance in Young’s double slit experiment when waves from
between the slits is increased? [Delhi 2008] the two slits have a path difference of (i) 0 and (ii)
12. Write the conditions on path difference under which λ
.
(i) constructive (ii) destructive interference occur in 4
Young’s double slit experiment. [Delhi 2020] 22. Two plane monochromatic waves propagating in
13. The intensity of light at same points on the screen in the same direction with amplitudes A and 2A and
Young’s double slit experiment is zero. Give reason. p
[AI 2020] differing in phase by rad. superpose. Calculate
3
III. Short Answer Type Questions-I (2 Marks) the amplitude of the resultant wave.
14. A region is illuminated by two sources of light. The 23. Find the ratio of intensities at two points P and Q on
intensity I at each point is found to be equal to I1 + a screen in a Young’s double slit experiment when
I2, where I1 is the intensity of light at the point when
waves from S1 and S2 have a phase difference of :
source 2 is absent, I2 is similarly defined. Are the
source coherent or incoherent? Explain. [NCERT] p p
(i ) and (ii ) respectively.
15. The arrangement used by Thomas Young to produce 3 2
an interference pattern is shown in the figure below. 24. In Young’s double slit experiment, the two parallel
Justify why there would be no change in the fringe slits are made one millimetre apart and a screen is
width when the main illuminated slit S is shifted to placed one metre away. What is the fringe separation
the position S′ as shown. [S.P. 2015] when blue green light of wavelength 500 nm is used?
[NCERT]
S1 25. Laser light of wavelength 630 nm incident on a pair
S′ of slits produces an interference pattern in which the
φ Q bright fringes are separated by 8.1mm. A second light
O
S φ produces an interference pattern in which the fringes
are separated by 7.2 mm. Calculate the wavelength
of the second light. [AI 2009]
S2
O′ 26. Yellow light of wavelength 6000 Å produces fringes
Screen of width 0.8 mm in Young’s double slit experiment.
What will be the fringe width if the light source
16. Two identical coherent waves, each of intensity 1,
is replaced by another monochromatic source of
are producing an interference pattern. Find the value
wavelength 7500 Å and the separation between the
of the resultant intensity at a point of (i) constructive
slits is doubled?
interference and (ii) destructive interference.
17. How will the angular separation and visibility of 27. In Young’s experiment, the width of the fringes
fringes in Young’s double slit experiment change obtained with light of wavelength 6000Å is 2.0 mm.
when (i) screen is moved away from the plane of the Calculate the fringe width if the entire apparatus is
slits (ii) width of source slit is increased? [AI 2008] immersed in a liquid medium of refractive index 1.33.
18. What is the effect on the interference pattern in 28. The ratio of the intensities at minima to the maxima
Young’s double slit experiment when: in the Young’s double slit experiment is 9:25. Find
(i) Screen is moved closer to the plane of slits? the ratio of the widths of the two slits. [AI 2014]
(ii) Separation between two slits is increased. 29. Two coherent sources have intensities in the ratio
19. The resultant intensity at any point on the screen 25 : 16. Find the ratio of the intensities of maxima to
varies between zero and four times the intensity due minima, after interference of light occurs.
to one slit, in young’s double slit experiment. Give 30. Two sources of intensity I1 and I2 undergo interference
reason.
in Young’s double slit experiment. Show that

Wave Optics 351


D:\EG_Physics-12_(26-06-2022)\Open_Files\Ch-10\Ch-10
\ 27-Jul-2022 Ved_Goswami Proof-4 Reader’s Sign _______________________ Date __________

2
I max ( a1 + a2 )
P
= where a1, a2 are the amplitudes of S1
I min ( a1 − a2 )2
S
disturbance for two sources S1 and S2. O
S2
31. Describe any two characteristic features which
distinguish between interference and diffraction
phenomena. Derive the expression for the intensity (i) State the condition for constructive and destructive
at a point of the interference pattern in Young’s double interference.
slit experiment.[Delhi 2019]
(ii) Obtain an expression for the fringe width.
32. Two waves from two coherent X
sources S and S′ superimpose at X as (iii) Locate the position of the central fringe.
shown in the figure. If X is a point on [AI 2013 C]
the second minima and SX – S′X is
40. A beam of light consisting of two wavelengths, 800
4.5 cm. Calculate the wavelength of S S¢ nm and 600 nm is used to obtain the interference
the waves. [CBSE S.P. 2020-21]
fringes in a Young’s slit experiment on a screen
IV. Short Answer Type Questions-II (3 Marks) placed 1.4 m away. If the two slits are separated by
33. What is the effect on the interference fringes in a 0.28 mm, calculate the least distance from the central
Young’s double slit experiment when: bright maximum where the bright fringes of the two
(i) The source slit is moved closer to the two double wavelength coincide. [AI 2012]
slits plane.
41. In Young’s double slit experiment using monochro-
(ii) Width of the source slit is increased
matic light of wavelength l, the intensity at a point
(iii) The monochromatic sources is replaced by a
on the screen where path difference is l is K units.
source of white light. [NCERT]
What is the intensity of light at a point where path
34. In Young’s double slit experiment, explain with reason
λ
in each case, how the interference pattern changes, difference is ?
when 3
(i) Width of the slits is doubled 42. The intensity at the central maxima (O) in a Young’s
(ii) Separation between the slits is increased, and double slit experiment is I0. If the distance OP equals
(iii) Screen is moved away from the plane slits. one-third of the fringe width of the pattern, show that
35. What are coherent sources of light? Why are coherent I0
sources required to produce interference of light? the intensity at point P would be .
4
Give an example of interference in every day life.
[Foreign 2011, 2012]
[Delhi 2008 C]
36. Can two identical and independent sodium lamps act
as coherent sources? Given reason for your answer. P
[AI 2008] S1
37. How does the fringe width of interference fringes
change, when the whole apparatus of young’s d O
experiment is kept in water (refractive index = 4/3)?
[Delhi 2008, 2011] S2
38. Two narrow slits are illuminated by a single
monochromatic sources. Name the pattern obtained D
on the screen? One of the slit is now completely
covered. What is the name of the pattern now 43. In Yo u n g ’s d o u b le s lit ex p er imen t u s in g
obtained. Draw intensity pattern obtained in the two
monochromatic light of wavelength l, the intensity
cases. Also write two differences between the patterns
obtained in the above two cases. of light at a point on the screen where path difference
is l is k units. Find the intensity at a point on the
39. Figure shows a modified Young’s double slit
λ
λ screen where path difference is . [AI 2015]
experimental set up. Here SS2 – SS1 = . 4
4

352 Physics–12
D:\EG_Physics-12_(26-06-2022)\Open_Files\Ch-10\Ch-10
\ 27-Jul-2022 Ved_Goswami Proof-4 Reader’s Sign _______________________ Date __________

44. Consider two coherent sources S1 and S2 producing minimum intensity of the fringe in the interference
monochromatic waves to produce interference pattern.
pattern. Let the displacement of the wave produced (ii) What kind of fringes do you expect to observe
by S1 be given by if white light is used instead of monochromatic
y1 = a cos wt. and the displacement by S2 be y2 = a
light?  [Delhi 2018]
cos (wt + f). 49. What is the effect on the interference fringes in
Find out the expression for the amplitude of the Young’s double slit experiment due to each of the
resultant displacement at a point and show that following operations? Justify your answers.
φ (i) The screen is moved away from the plane of the
the intensity at that point will be I = 4a 2 cos 2 .
2 slits.
Hence establish the conditions for constructive and
destructive interference. [AI 2015] (ii) The separation between slits is increased.

45. In a two slit experiment with monochromatic light, (iii) The source slit is moved closer to the plane of
double slit. [Delhi 2020]
fringes are obtained on a screen placed at some
distance D from the slits. If the screen is moved 5 × V. Long Answer Type Questions (5 Marks)
10–2 m towards the slits, the change in fringe width 50. (i) What are coherent sources of light? State two
is 3 × 10–5 m. If the distance between the slits is 10–3 conditions for two light sources to be coherent.
m, calculate the wavelength of the light used. (ii) Derive a mathematical expression for the width of
interference fringes obtained to Young’s double
46. In Young’s double slit experiment, using light of
slit experiment with the help of a suitable diagram.
wavelength 400 nm, interference fringes of width ‘X’
[AI 2008 C, Delhi 2011)
are obtained. The wavelength of light is increased to
Or
600 nm and the separation between the slits is halved.
If one wants the observed fringes width on the screen Derive an expression for fringe width in a two slit
to be the same in the two cases, find the ratio of the interference experiment.
distance between the screen and the plane of the 51. (i) There are two sets of apparatus of Young’s double
interfering sources in the two arrangement. slit experiment. In set A, the phase difference
between the two waves emanating from the slits
47. In Young’s double slit experiment, deduce the does not change with time, whereas in set B, the
condition for (i) constructive, and (ii) destructive phase difference between the two waves from the
interference at a point on the screen. Draw a graph slits changes rapidly with time. What difference
showing variation of the resultant intensity in the will be observed in the pattern obtained on the
interference pattern against position ‘X’ on the screen. screen in the two set ups? 5
[AI 2012] (ii) Deduce the expression for the resultant intensity
48. (i) If one of two identical slits producing interference in both the above mentioned set ups (A and B),
in Young’s experiment is covered with glass, assuming that the waves emanating from the
so that the light intensity passing through it is two slits have the same amplitude A and same
wavelength l.  [CBSE S.P. 2018-19]
reduced to 50%, find the ratio of the maximum and

Answers 10.2
I. Objective Type Questions II. Very Short Answer Type Questions
1. (i) (c) (ii) (a) (iii) (c) (iv) (c) 1. The size of source should be small. S1
(v) (b) and (c) (vi) (a) s λ Source
The condition is <
2. (i) Constructive (ii)coherent S d
2. Fringe width of interference
s
(iii) Interference
d

3. (i) True (ii) True (iii) False (iv) False Dλ 1


fringes β = , as β∝ . S
4. (i)-(d) (ii)-(a) (iii)-(c)(iv)-(b) d d S2
5. (a) 6. (c) When d is infinitely small, fringe

Wave Optics 353


D:\EG_Physics-12_(26-06-2022)\Open_Files\Ch-10\Ch-10
\ 27-Jul-2022 Ved_Goswami Proof-4 Reader’s Sign _______________________ Date __________

width b will be too large. In such a case even a single (ii) For distructive interference i.e. minima cos f = –1
fringe may occupy the whole field of view. Hence, the f = p, 3p, 5p, 7p,…
interference pattern cannot be detected. f = (2n–1)p, (n =1,2,3…)
3. (a) Two waves should have a constant phase difference. λ λ  1
(b) Two waves must be continuous and of same path difference Dx = φ= (2n–1)p =  n −  λ
2π 2π  2
frequency.
13. Due to redistribution of light energy because of
4. For a dark fringe, Path difference
interference of light which undergo constructive and
λ destructive interference at different points.
P2R – P1R = (2n − 1) 2
14. If phase difference between interfering waves is f then
Where n = 1, 2, 3, ... Resultant intensity
5. As a1 ≠ a2 I = I1 + I 2 + 2 I1I 2 cos φ  ½
So intensity at minima will not be zero. As I = I1 + I2 given
So intensity at minima will increase and the intensity So 2 I1I 2 cos φ = 0
at maxima will decrease. So the contrast will be poor. So cos f = 0, as I1 ≠ 0, I2 ≠ 0 ½
6. If two waves of same amplitude in the same state of The phase difference varies from 0 to 2p such that
polarisation meet at a point in opposite phase, the average of cos f over a cycle is zero. So the two sources
resultant amplitude and hence intensity of light becomes are giving out wave with a phase difference which in
zero at that point. There will be destructive interference, not constant. ½
so it produces darkness.
So the sources are incoherent. ½

7. β = , i.e., b ∝ l; the wavelength of blue light is less 15. If source S is at the perpendicular bisector of S1S2 then
d
than that of red light; hence if red light is replaced by SS1 = SS2, so the path difference from two slits S1 and
blue light, the fringe width decreases. S2 will be zero at O i.e., SQ and O will lie in one line.
8. The light emitted by different lamps is not coherent, When S is shifted to S′ by such that S′QS = f then
so the waves emitted by sources can not produce S′S1 < S′S2. ½
interference.
λD cD S1
9. (i) As b= =
d vd S′
1 φ Q O
So b∝ S φ
v
As n decrease, b will increase.
S2
10. In order to have sustained interference pattern, the phase
O′
difference between the superimposing wave should Screen
not change with time. In case of coherent source, they
give out waves of same frequency and a constant phase So the point of zero path difference i.e., central maxima
difference. will appear at an angle – f, which means it is shifted
λ by the same angle on the other side of the bisector. ½
11. As Avg. fringe separation ω = where d is distance λD
between slits d The fringe width β = will remain same as there is
d
So as d is increased, w will decrease. no change in, l, D or d. 1
So Angular separation of interference fringes will 16. IR = I1 + I 2 + 2 I1I 2 cos φ  ½
decrease with increase in distance between the slits.
I1 = I2 = I
12. (i) For construction interference i.e.maxima cosf = 1 or
f = 0, 2p, 4p, 8p,… f = 2np, where n = 0,1,2,3,… So IR = I + I + 2I cos f = 2I (1 + cos f) ½
path difference, (i) Constructive interference
λ λ f = 2np, n = 0, 1, 2, 3, ...
Dx = φ= ×2np = nl
2π 2π    So cos 2np = 1
\ Dx = nl (n = 0, 1, 2,…)    So, IR = 2I (1 + 1) = 4I ½

354 Physics–12
D:\EG_Physics-12_(26-06-2022)\Open_Files\Ch-10\Ch-10
\ 27-Jul-2022 Ved_Goswami Proof-4 Reader’s Sign _______________________ Date __________

(ii) Destructive interference yn d 10 − 2 × 3 × 10 −4


f = (2n – 1) p, n = 1, 2, 3, ... So l= =  ½
nD 4 × 1.5
cos f = –1
   So IR = 2I (1 –1) = 0 ½ 3
= × 10 −6 = 5 × 10 −7 m  1
β λ. 6
17. (i) Angular separation = =
D d 21. Using I = a12 + a22 + 2a1a2 cos φ
It is independent of D; therefore, angular separation As a1 = a2 = a
remains unchanged if screen is moved away from So I = 2a2 + 2a2 cos f
the slits. But the actual separation between fringes
= 2a2 (1 + cos f)
λD
β= increases, so visibility of fringes increases. (i) When path difference = 0 So f = 0°
 d 1 I1 = 2a2 (1 + cos 0°) = 4a2 1
(ii) When width of source slit is increased, then the λ
angular fringe width remains unchanged but fringes (ii) When path difference =
4
becomes less and less sharp; so visibility of fringes Phase difference
s λ 2π
decreases. If the condition < is not satisfied, = × Path difference
S d λ
the interference pattern disappears. 1 2π λ π
λD = × =  ½
18. As b = λ 4 2
d π
2  2
D → distance between slits and screen I2 = 2a 1 + cos  = 2a
 2 
d → distance between slits
I1 2
(i) When screen is moved closer to the plane of the slits, = 4a = 2  ½
D will decrease so b will also decrease. 1 I2 2a 2 1
(ii) When the separation between the slits i.e., d is π
1 22. A1 = A , A 2 = 2 A , φ = rad
increased, b will decrease as β ∝ .  1 3
d 1
19. (i) In Young’s double slit experiment (
A′ = A12 + A 22 + 2A1A 2 cos φ ) 2  1
a1 = a = a2 1
So resultant amplitude A is given by π 2
=  A 2 + (2A) 2 + 2A 2A cos 
A2 = a2 + a2 + 2a2 cos f = 2a2 (1 + cos f)  3
φ 1
A2 = 4a 2 cos 2 as I ∝ A2 1
   =  5A 2 + 4A 2 × 1  = [7 A 2 ] 2 = 7 A 
2
2 1
 2
φ
So I = 4 I0 cos2
 1 23. I = I1 + I2 + 2I1I2 cos f
2
In Young double slit experiment
For constructive interference
f =0 or 2np I1 = I2 = I0
So Imax = 4 I0 ½ So I = 2I0 (1 + cos f) 1
At minima,f =p, 3p, 5p, ... I ∝ 1 + cos f
i.e., f = (2n – 1)p π 1
IP 1 + cos 1+
So Imin = 0 ½ So, = 3 = 2 = 3 1
IQ π 1 2
So, intensity varies from 0 to 4 I0 1 + cos
20. d = 0.03 cm = 3 × 10– 4 m 2
24. Here d = 1 mm = 10–3 m, D = 1 m
n= 4
l = 500 nm = 500 × 10– 9 m ½
yn = 1 cm = 10– 2 m
Fringe width,
D = 1.5 m
D λ 1 × 500 × 10 −9
nλD b= = m 1
yn =  ½ d 10 −3
d
= 5 × 10– 4 m = 0.5 mm. ½

Wave Optics 355


D:\EG_Physics-12_(26-06-2022)\Open_Files\Ch-10\Ch-10
\ 27-Jul-2022 Ved_Goswami Proof-4 Reader’s Sign _______________________ Date __________

25. l1 = 630 nm = 630 × 10–9 m, b1 = 8.1 mm 2


5 
= 8.1 × 10–3 m,
I max ( r + 1) = 2  + 1
4
l2 = ?, b2 = 7.2 mm = 7.2 × 10–3 m
\ = = 81 : 1  1
λ1D λ D
I min ( r − 1)2  5 − 1 2
As b1 = , β2 = 2  1  
d d 4 
β2 λ 2 30. Amplitude of the resultant wave is given by

So    = 1
β1 λ1
(2 2
a = a1 + a2 + 2a1a2 cos φ 2  ½ )
β2 7.2 Where f is the phase difference between interfering
or l2 = λ 1 = 630 × = 560 nm 1
β1 8.1 waves
26. l1 = 6000 Å, b1 = 0.8 mm, slits separation As intensity I ∝ a2
= d1 = d, b2 = ?, l2 = 7500 Å, d2 = 2d or I = ka2 = k ( a12 + a22 + 2a1a2 cos φ )
λ1D λ D At maxima,
As b1 = β2 = 2  ½ f = 2np, n = 0, 1, 2, ...
d1 d2
So cos f = 1 ½
β2 d1 λ 2 7500 d 5 1 5
       = × = ×
β1 λ1 d 2 6000 2d
= × = 
4 2 8
1 So (
Imax = k a12 + a22 + 2a1a2 = k(a1 + a2)2 )
5 5 At minima, f = p, 3p, 5p , ...
b2 =
× β1 = × 0.8 = 0.5mm  ½
8 8 i.e., f = (2n – 1) p where n = 1, 2, .....
λd λ′d β′ λ ′ λ 1 So cos f = – 1 ½
27. b= , β′ = So = = =  1
d d β λ µλ µ Imin = k ( a12 + a22 )
− 2a1a 2 = k ( a1 − a2 )
2

b′ = 1 β = 2 mm = 1.5mm  1 I max ( a1 + a2 )
2

µ 1.33 =
    Imin ( a1 − a2 )2  ½
28. If w1 and w2 are the slit width of two slits
w1 I1 a12
31. Difference between Interference and Diffraction
Then = =  ½ Interference Diffraction
w2 I 2 a22
2 Interference may be defined Diffraction on the other hand
 a1  as waves emerging from two can be termed as secondary
    Imin ( a 1 − a2 ) 2  a − 1  ½ different sources, producing waves that emerge from the
= = 2 
I max ( a 1 + a2 )2  a1 + 1  different wavefronts. different parts of the same
a  wave.
 2 
In interference the intensity In diffraction, there is a
 a1 
 − 1 of all the positions on max- variance of the intensity of
9 a a ima are of similar intensity positions.
= 2  , If 1 = r
25  a1 a
+ 1 2 in interference.
a 
      2  The width of the fringes in The width of the fringes is
r −1 3 interference is equal in in- not equal in interference.
= terference.
    r + 1 5  ½
5r – 5 = 3r + 3 It is absolutely dark in the In the case of diffraction,
2r = 8 or r = 4 region of minimum intensity, there is a variance in the in-
in the case of interference. tensity of interference.
    a1 = 4 ; a1 = 4a2 1
a2
2
P
w1 a12  4  16 S1
    = 2 =  =  ½ B
w2 a2  1  1 d/2
y

w1 I1 S d O
29. Amplitude ratio = r = =  ½
w2 I2 d/2
I1 25 5 A
r = = = S2
 ½ D
I2 16 4

356 Physics–12
D:\EG_Physics-12_(26-06-2022)\Open_Files\Ch-10\Ch-10
\ 27-Jul-2022 Ved_Goswami Proof-4 Reader’s Sign _______________________ Date __________

By the superposition principle, the resultant displacement s λ


at point P is given by, the condition < is not satisfied, the interference
S d
y = y1 + y2 pattern will disappear. 1
y = a1 sin wt + a2 sin (wt + f)
(iii) White light consists of seven colours with their
= a1 sin wt + a2 sin wt cos f wavelengths ranging from 400nm to 780nm. Each
+ a2 cos wt sin f colour will produce its central maxima at same point.
y = (a1 + a2 cos f) sin wt So the central maxima is white. Central white fringe
+ a2 sin f cos wt ...(i) will be surrounded by a few coloured fringes. The
Let   a1 + a2 cos f = A cos q...(ii) closest fringe on either side of the central white fringe
      a2 sin f = A sin q...(iii) will be violet and the farthest will be red. After that
Then, equation (i) becomes there will be overlapping of fringes due to different
wavelengths. 1
y = A cos q sin wt + A sin q cos wt
y = A sin(wt + q) Dλ
34. The fringe width b =
Squaring and adding both sides of the equations (ii) d
and (iii), we obtain (i) When the width of the slit is doubled; the intensity
A2 cos2 q + A2 sin2 q = (a1 + a2 cos f)2 + a22 sin2 f of interfering wave becomes four times, intensity
of maxima becomes 16 times i.e., fringes become
A2 = a21 + a22 (cos2 f + sin2 f) + 2a1a2 cos f
brighter. 1
A2 = a21 + a22 + 2a1a2 cos f
The intensity of light is directly proportional to the 1
(ii) β ∝, when separation between the slits
square of the amplitude of the wave. The intensity of d
light at point P on the screen is given by, is increased the fringe width decreases i.e., fringes
I = a21 + a22 + 2a1a2 cos f...(iv) 1 comes closer. 1
32. Path difference Dx =SX – S′X = 4.5cm for minimum, (iii) b ∝ D, when screen is moved away from the plane of
the slits, the fringe width increases i.e., fringes become
 1 farther away. 1
Dx =  n −  λ as n = 2
2
 
35. Coherent sources: Two sources giving light waves of
3λ same frequency and constant phase difference are called
\ 4.5 = ⇒ l = 3cm
2 coherent sources. ½
33. (i) Let s be the width of the source slit, S be the distance Necessity of coherent sources to produce interference
of double slit plane from the source slit. of light: Intensity at any point in the region of
For interference fringes to be distinctly visible superposition is I = a12 + a22 + 2a1a2 cos φ  ½
s λ
< should be satisfied. If the sources are not coherent, then the phase difference
S d between the superimposing waves will keep changing
with time. On an average 108 jumps of electrons takes
place is one second in a source of light. So the phase
d change will take place at a very fast rate as a result of
s  ½ this no fringes will be observed. 1
Example of interference in every day life: Soap
S
bubbles appear coloured in white light. These colours
appear due to superposition of light reflected from
As the source slit is brought closer, the S decreases, the upper and lower surface of soap film. The colours
the interference pattern becomes less and less sharp. observed are those for which the condition of maxima
But as long as the fringes are visible, the fringe width in reflected light is satisfied. 1
is unaffected. ½
36. No, two independent sources of light cannot act as
(ii) As a broad source is equivalent to a large number of coherent sources. 1
point sources placed close by. Each of these sources
will produce their own interference pattern. So there Reason: The emission of light is due to millions of
will be overlapping of fringes produced by each atoms and their number goes on changing in quite
source. The fringe pattern will be less sharp. But if random manner. When an electron jumps back from

Wave Optics 357


D:\EG_Physics-12_(26-06-2022)\Open_Files\Ch-10\Ch-10
\ 27-Jul-2022 Ved_Goswami Proof-4 Reader’s Sign _______________________ Date __________

higher energy level (excited state) to lower energy level, 39. Initial path difference between S1 and S2,
a photon is released. 1 λ
Dy0 = SS2 − SS1 =
On an average is one second around 10 such jumps
8
4
takes place. So the phase difference between them Path difference between disturbances from S1 and S2 at
cannot remain same. 1 point P,
Dλ xd
37. Fringe width β = , b a l for same D and d. Dy =
d D
Total path difference between the two disturbances at
When the whole apparatus is immersed in a transparent
P,
liquid of refractive index µ = 4/3, the wavelength
decreases to λ xd
Dy0 = Dy = +
λ λ 4 D
l′ = = .   1 \ For constructive interference,
µ 4/3
 λ xd 
Dλ ′ D 3λ 3 Path diff. =  +  = nλ; n = 0, 1, 2, ...  1
So b′ = = = β 1 4 D
d d4 4
P
3 x
So fringe width is times the original fringe width.1
4 S1
38. When two narrow slits are illuminated by a single S O x
S2 O′ 0
monochromatic source, interference pattern is obtained
on the screen. D

When one of the slit is completely covered then single xn d  1


slit diffraction pattern is obtained on the screen. ½ or = n −  λ ...(1)
D  4
Interference
For destructive interference,
  ½ λ xd λ
I Path diff. =  +  = (2n − 1) ...(2)
4 D 2
xn′ d  3 λ x′ d 3 λ
or =  2n − 1 −  or n =  2n −  
D  2 2 D  2 2
–2λ –λ –λ/2 O λ/2 λ 2λ 1
Path difference
Fringe width,
(i) All maxima’s are of equal intensity. λD
b = xn + 1 − xn =  ½
(ii) All maxima’s are of equal width. d
Diffraction ½ The position x0 of central fringe is obtained by putting
n = 0 in equation (1). Therefore,
λD
\ x0 = –  ½
I

½ 4d
The negative sign shows that the central fringe is
obtained at a point O below the (central) point O.
40. l1 = 800 nm = 800 × 10– 9 m,

O
l2 = 600 nm = 600 × 10–9 m
–2λ –λ λ 3λ/2 2λ
Path difference
D = 1.4 m, d = 0.28 mm = 0.28 × 10– 3 m
(i) All maxima’s are not of equal intensity. Central nλD
Formula used, yn =  1
maxima has maxima intensity and the intensity of d
subsequent maxima’s decreases rapidly. ½ For the least distance of coincidence of fringes,
(ii) Central maxima is twice in width as compared to difference in order of the fringes for wavelength l1 and
secondary maximas. ½ l2 must be one.

358 Physics–12
D:\EG_Physics-12_(26-06-2022)\Open_Files\Ch-10\Ch-10
\ 27-Jul-2022 Ved_Goswami Proof-4 Reader’s Sign _______________________ Date __________

nλ1D (n + 1) λ 2 D If path difference = l, then phase difference = 2p.


( yn ) λ1 = ( yn + 1 ) λ 2 i.e., = 2π
d d So I = 4I0 cos 2 = 4I0 = K (given) 1
nl1 = (n + 1)l2 1 2
nl1 – nl2 = l2 or λ
When path difference =  ½
λ2 600 × 10 600 −9 4
n= = = =3
λ1 − λ 2 (800 − 600) × 10 −9
200 2π π
Phase difference = =  ½
4 2
nλ1D 3 × 800 × 10 −9 × 1.4
yn = = π 1 K
d 0.28 × 10 −3 So I = 4I0 cos 2 =K× =  1
4 2 2
= 12 × 10– 3 m 1 44. According to principle of superposition of waves
φ y = y1 + y2 = a cos wt + a cos (wt + f)
41. Using I = 4I0 cos 2  1
2
φ φ
When path difference is l, y = 2a cos cos  ωt +   1
2  2
Phase difference = 2 p
 2π   C−D C + D
So I = 4I0 cos 2   ∵cos C + cos D = 2 cos cos 
 2 2 2 
I = 4I0 = K (given) ...(1) The amplitude of the resultant wave
λ φ
When Dx = = 2a cos = A
3 2
2π 2π λ 2π As I ∝ A2
Phase Difference f = × ∆x = × =  1 2
λ λ 3 3  φ
I = K  2a cos  ,
 2
 π 1 K
I′ = 4I0 cos 2   = K × = [Using (1)] 1 K is proportionality const.
 3 4 4
φ
λD I = 4Ka 2 cos 2  1
42. Fringe width = β = 2
d For constructive interference
β λD
OP = y = = 2 φ

3 3d cos = 1 or cos φ = ± 1
2 2
yd λDd λ φ
Path difference = ∆x = = =  1 = np or f = 2np ½
D 3Dd 3 2
λ For destructive interference
Dx = φ , f is phase difference
2π φ φ
cos 2 = 0 or cos = 2
2π 2π λ 2π 2 2
f= ∆x = × =
λ λ 3 3 φ p
i.e., = (2n + 1) or f = (2n + 1) p ½
In Young’s double slit experiment 2 2
IR = I1 + I 2 + 2 I1I 2 cos φ 45. Fringe width
λD λD′
φ          β = and β′ =
= 2I + 2I cos φ = 4I cos 2  1 d d
2 λ
φ    β − β′ = d ( D − D′ )  1
IR = 4I cos 2
2 Given D – D′ = 5 × 10 m, b – b′ = 3 × 10 m
–2 –5

At O, f = 0, So IR = 4I = I0 given d = 10– 3 m
φ
2
 1 I0 −5 l × 5 × 10 −2
IP = I0 cos
2
= I0   =  1 So 3 × 10 =  1
3 2 4 10 −3
φ 3 × 10 −5 × 10 −3
43. As I = 4I0 cos 2 l= = 6 × 1000Å= 6000Å 1
2 5 × 10 −2

Wave Optics 359


D:\EG_Physics-12_(26-06-2022)\Open_Files\Ch-10\Ch-10
\ 27-Jul-2022 Ved_Goswami Proof-4 Reader’s Sign _______________________ Date __________

46. l1 = 400 nm, b1 = x, slit separation d = d1,  λ


d No effect on angular fringe width  φ = 
l2 = 600nm, d 2 = 1 , β 2 = x  ½  d

2 (ii) Both linear fringe width and angular fringe width
1 1
To find
D1
, decrease  β ∝ , φ ∝  .
D2  d d
s λ
λ1D1 λ D (iii) If condition < is satisfied interference will be
As    β1 = , β2 = 2 2  ½ S d
d1 d2
obtained otherwise no interference will be seen.
β1 λ1 D1 d 2 V. Long Answer Type Questions
= × ×
    β2 λ 2 D 2 d1  1 50. (i) Coherent source of light: Those sources which
x 400 D1 d1 gives out light of same frequency with a constant
= × × initial phase difference are called coherent sources.
    x 600 D 2 2d1
 1
D1 6 3 Condition for two light sources to be coherent:
   = ×2 =  1
D2 4 1
(a) Two sources should give out waves of same
47. Refer theory topic No. 2 under the heading of condition frequency continuously. 1
for constructive and destructive Interference. 1+1+1 (b) They should give out waves with constant initial
48. (i) Let the maximum intensity be Imax and minimum phase difference.
intensity Imin This can be achieved by obtaining two sources from
Imax ∝ (a1 + a2)2 one source only i.e., either by division of wavefront
Imin ∝ (a1 – a2)2 or by using the light reflected from the two surfaces
of the same body. 1
I0 ∝ a12 or a1 ∝ I0
(ii) Expression for fringe width: Refer theory topic
I0 I0 No. 2 under the heading: YDSE. 1+1
a22 ∝ or a2 ∝
2 2 51. (i) Set A: stable interference pattern, the positions of
maxima and minima does not change with time. 1
2
 I0  Set B: positions of maxima and minima will change

I max  I0 + 

\ = 2  1 rapidly with time and an average uniform intensity
I min 
2 distribution will be observed on the screen. 1
I0 
 I0 −  (ii) Expression for intensity of stable interference pattern
2
in set –A 2
I max ( 2 + 1) 2 If the displacement produced by slit S1 is
or =
I min ( 2 − 1) 2 y1 = a cos wt
I max (3 + 2 2 ) then the displacement produced by S2 would be
or =  ½
I min (3 − 2 2 ) y2 = a cos (wt + f)
(ii) Time varying fringes patterns in which relative and the resultant displacement will be given by
positions of fringes changes with time because of
y = y1 + y2
the presence of various wavelengths in white light.
 ½ = a [cos wt + cos (wt + f)]
φ φ
The interferences patterns due to different constituent = 2a cos   cos  ωt + 
colours of white light overlap (incoherently)  2  2
coloured fringes of different width are obtained. 1 The amplitude of the resultant displacement is
49. (i) Linear fringe with (b) increased as  φ
2a cos   and therefore the intensity at that point
λD  2
b= ⇒b∝D
d will be

360 Physics–12
D:\EG_Physics-12_(26-06-2022)\Open_Files\Ch-10\Ch-10
\ 27-Jul-2022 Ved_Goswami Proof-4 Reader’s Sign _______________________ Date __________

 φ  φ
I = 4 I0 a cos 2   Function cos 2   will randomly vary between 0
 2  2
f= 0 1
and 1, the average value will be .
∴ I = 4 I0 2
In set B, the intensity will be given by the average 1
intensity \ I = 4 I0 × = 2 I0
2
 φ
< I > = 4 I0 < cos 2   > 1 [CBSE Marking Scheme, 2018-19]
 2

Topic 3. Diffraction at Single Slit


•• Diffraction: Diffraction is the phenomenon of bending of Then the point P will be of minimum intensity. It is
light around the corners of an obstacle or an opening in because, if the slit is assumed to be divided into two
its path. Due to diffraction light spreads into the regions equal parts then, wavelets from the corresponding points
of geometrical shadow. of the two halves of the slit will have a path difference
This is due to the superposition of wavelets from the λ
of and hence they will produce minima.
different parts of the same source. 2
Diffraction effect is more pronounced (observable) When For First Minima,
the size of aperture or obstacle is less than or of the order d sin q1 = l  
of wavelength of waves.
Diffraction of sound waves is more pronounced as λ
compared to light because the size of the openings or fi sin q1 =
d
obstacle which waves come across are comparable to Since q1 is very small,
wavelength of sound (a metre) so diffraction of sound
λ
is observed not for light. sin q1 ≈ q1 =
•• Diffraction at Single Slit: Consider a parallel beam of d
monochromatic light incident normally upon a slit AB. If BN = 2l, then slit AB can be imagined to be divided
Then each point of AB becomes a source of secondary into 4 equal parts, then the path difference between the
disturbance or wavelets. secondary wavelets originating from the corresponding
Now consider a point O on the screen which in placed λ
points of each part = hence they produce minima.
at distance D from slit AB. Since point O is equidistant 2
from A and B, therefore, the secondary wavelets from AB So for second minima
reach the point O in the same phase and hence constructive d sin q2 = 2l
interference takes place at O. So O is position of central 2λ
maxima. fi sin q2 =
d
Since q2 is very small,
A P

d
{ C
  O
{ y1 sin q2 ≈ q2 =

d
N Or in general, for minima
B d sin qn = nl
Light Source
D nλ
qn =
d
Let us now consider the secondary wavelets travelling in
3λ 5λ
a direction making an angle q with CO. Let these waves Secondary Maxima: If BN = 2 or 2
meet at a point P. The intensity at P will depend on path
then the point P on the screen will be a maxima called
difference between the secondary wavelets emitted from
1st secondary maxima. It is because, the slit can be
the corresponding points of the wavefront.
considered to be divided into three equal parts, the
Path difference = BN = d sin q wavelets from 1st two parts will reach P in opposite
If d sin q = l phase, cancelling the effect of each other, the wavelets

Wave Optics 361


D:\EG_Physics-12_(26-06-2022)\Open_Files\Ch-10\Ch-10
\ 27-Jul-2022 Ved_Goswami Proof-4 Reader’s Sign _______________________ Date __________

from the 3rd part of the slit remain uncancelled and Width of 1st, secondary maxima,
5λ 2λ λ λ
produce maxima at BN = d sin θ = we get second w1 = θ2 − θ1 = − =
maxima 2 d d d
5λ All secondary maxima have equal width. The intensity
q= [ sin q ≈ q, For small value] of 1st secondary maxima is much less than that of central
2d
In general, maxima as only 1/3rd of the slit contributes to this
λ maxima.
qn = ( 2n + 1) , where n = 1, 2, 3,..............
2d •• Graph for Variation of Intensity: Broad central
•• Width of Central Maxima: The central maxima extends maximum is in direction q = 0° of incident light. On,
on both sides of centre of slit. So angular width of central either side, it has secondary maxima of decreasing
maxima intensity.

w = 2θ1 =
d
If OP = y, then I
y1
= tan q1
D
y –3l –2l –l O l 2l 3l Position on screen
tan q1 ≈ q1 = 1 d d d d d d
D
Linear width,
Intensity of secondary maximum decreases with the order
2λD of the maximum.
bo = 2y1 = 2q1D =
d

EXERCISE 10.3
I. Objective Type Questions (1 Mark) (iv) Consider the diffraction pattern for a small
1. Choose the correct answers from the given options pinhole. As the size of the hole is increased
(MCQs). (a) the size decreases.
(i) Consider sunlight incident on a slit of width 104 (b) the intensity increases.
Å. The image seen through the slit shall (c) the size increases.
(a) be a fine sharp slit white in colour at the centre. (d) the intensity decreases.
(b) a bright slit white at the centre diffusing to (v) The resolving power of a telescope can be
zero intensities at the edges. increased by increasing
(c) a bright slit white at the centre diffusing to (a) wavelength of light.
regions of different colours. (b) diameter of objective.
(d) only be a diffused slit white in colour. (c) length of the tube.
(ii) The condition for observing Fraunhoffer
(d) focal length of eyepiece. [AI 2020]
diffraction from a single slit is that the light
wavefront incident on the slit should be 2. Fill in the blanks.
(a) spherical (b) cylindrical (i) The penetration of light into the region of
geometrical shadow is called .................... .
(c) plane (d) none of these
(ii) Diffraction of ................. waves is more
(iii) Consider sunlight incident on a pinhole of width pronounced as compared to light.
103 Å. The image of the pinhole seen on a screen
(iii) In Young’s double slit experiment, the path
shall be
difference between two interfering waves at a
(a) a sharp white ring. point on the screen is 5l/2, l being wavelength
(b) different from a geometrical image. of the light used. The ..................... dark fringe
(c) a diffused central spot white in colour. will lie at this point. [Delhi 2020]
(d) a diffused coloured region around a sharp (iv) If one of the slits in Young’s double slit experiment
central white spot. it fully closed, the new pattern has .......... central
maximum in angular size. [Delhi 2020]
362 Physics–12
D:\EG_Physics-12_(26-06-2022)\Open_Files\Ch-10\Ch-10
\ 27-Jul-2022 Ved_Goswami Proof-4 Reader’s Sign _______________________ Date __________

3. State True or False. 7. State the essential condition for diffraction to occur.
(i) Width of the central diffraction maximum on 8. In the single slit diffraction experiment, some
immersing the apparatus in a denser medium of coloured fringes around the central white maximum
1 are observed on the screen when one uses a source
refractive index µ becomes times its width of white light. Give reason. [AI 2020]
m
1 III. Short Answer Type Questions-I (2 Marks)
in air and wavelength becomes times the 9. Ray optics is based on the assumption that light
m
wavelength travels in a straight line diffraction effects (observed
(ii) Light waves spreading from two sources produce when light propagates through small apertures/slits or
interference only if they have same frequency and around small obstacles) disprove this assumption. Yet
constant phase difference . the geometrical optics assumption is so commonly
For Q. 4 there are two statements labelled as assertion used in understanding location and several other
(A) and reason (R). Select the correct answer to these properties of images in optical instruments. What is
questions from the codes (a), (b), (c) and (d) as given the justification? [NCERT]
below. 10. When a low flying aircraft passes overhead, we
(a) Both A and R are true and R is the correct sometimes notice a slight shaking of the picture of
explanation of A. our TV screen. Suggest a possible explanation.
(b) Both A and R are true but R is not the correct [NCERT]
explanation of A. 11. A parallel beam of monochromatic light falls normally
(c) A is true but R is false. on a narrow slit of width ‘a’ to produce a diffraction
(d) Both A and R are false. pattern on the screen placed parallel to the plane of
4. Assertion (A): Intensity of secondary maxima the slit. Use Huygen’s principle to explain that the
decreases with the order of the maximum. central bright maxima is twice as wide as the other
maxima. [Delhi 2014 C]
Reason (R): Intensity of the central maximum is
due to the constructive interference of wavelets 12. Diffraction is common in sound and not common in
from all parts of the slit, while subsequent secondary light waves. Why?
maximum from fractional parts of the slit. 13. In a single slit diffraction experiment, the slit width is
made double that of the original width. What would
II. Very Short Answer Type Questions (1 Mark)
happen to the size and intensity of central diffraction
1. When a tiny circular obstacle is placed in the path of band? Give reason for your answer.
light from a distant source, a bright spot is seen at the [AI 2008, Foreign 2012, Delhi 2012]
centre of the shadow of the obstacle. Explain why?
14. State with reason, how would the linear width of
[NCERT][Delhi 2009, 18] central maximum change if (i) monochromatic yellow
2. Give two differences between interference and light is replaced with red light, and (ii) distance
diffraction of light. [AI 2008, Delhi 2009] between the slit and the screen is increased.
3. How does the angular separation between fringes in 15. A parallel beam of light of 500 nm falls on a narrow
single slit diffraction experiment change when the slit and the resulting diffraction pattern is observed
distance of separation between the slit and screen is on a screen 1 m away. It is observed that the first
doubled? minimum is at a distance of 2.5 mm from the centre
4. In a single slit diffraction experiment, the width of the of the screen. Calculate the width of the slit.
slit is made doubled the original width. How does this [AI 2013]
affect the size and intensity of the central diffraction 16. A slit of width ‘a’ is illuminated by light of
band?[AI 2012][Delhi 2018] wavelength, 700 nm. What will be the value of slit
5. How does the intensity of the central maximum width ‘a’ when
change if the width of the slit is halved in a single (a) First minimum falls at an angle of diffraction 30°?
slit diffraction experiment? [Foreign 2002] (b) First maximum falls at an angle of diffraction 30°?
6. A parallel beam of monochromatic light falls normally [AI 2009]
on a single narrow slit. How does the angular width 17. A parallel beam of light of wavelength 600 nm is
of the principal maximum in the resulting diffraction incident normally on a slit of width ‘a’. If the distance
pattern depend on the width of the slit?[AI 2008 C] between the slit and the screen is 0.8 m and the
Wave Optics 363
D:\EG_Physics-12_(26-06-2022)\Open_Files\Ch-10\Ch-10
\ 27-Jul-2022 Ved_Goswami Proof-4 Reader’s Sign _______________________ Date __________

distance of 2nd order maximum from the centre of OR


the screen is 1.5 mm, calculate the width of the slit. What should be the width of each slit to obtain n
[AI 2008] maxima of double slit pattern within the central
18. A parallel beam of light of wavelength 500 nm falls maxima of single slit pattern? [CBSE S.P. 2020-21]
on a narrow slit and the resulting diffraction pattern IV. Short Answer Type Questions-II (3 Marks)
is observed on a screen 1 metre away. It is observed 26. Draw the diagram showing intensity distribution of
that the first minimum is at a distance of 2.5 mm from light on the screen for diffraction of light at a single
the centre of the screen, find the width of the slit. slit. How is the width of central maxima affected if
[Foreign 2010] (i) the width of the slit is doubled; (ii) the wavelength
OR of the light used is increased?
A parallel beam of light of wavelength 500 nm falls
What happens to the width of the central maxima if
on a narrow slit and the resulting diffraction pattern is
the whole apparatus is immersed in water and why?
obtained on a screen 1 m away. If the first minimum is
[Foreign 2009]
formed at a distance of 2.5 mm from the centre of the
27. Two wavelength of sodium light 590 nm and 596 nm
screen, find the (i) width of the slit, and (ii) distance
are used, in turn, to study the diffraction taking place
of first secondary maximum from the centre of the
at a single slit of aperture 2 × 10–4 m. The distance
screen.[AI 2020]
between the slit and the screen is 1.5 m. Calculate the
19. Estimate the angular separation between first order
separation between the position of the first maxima
maxima and third order maxima of the diffraction
of the diffraction pattern obtained in the two cases.
pattern due to a single slit of width 1 mm. When light
[Delhi 2013]
of wavelength 600 nm is incident normally on it.
28. Light of wavelength 550 nm is incident as parallel
[AI 2015 C]
beam on a slit of width 0.1 mm. Find the angular
20. Light of wavelength 500 nm, falls from distant source,
width and the linear width of the principal maxima
on a slit 0.50 mm wide. Find the distance between the
in the resulting diffraction pattern on a screen kept
two dark bands, on either side of the central bright
at a distance of 1.1 m from the slit, which of these
band of the diffraction pattern observed on a screen
width would not change if the screen were moved to
placed 2m from the slit. [AI 2004 C]
a distance of 2.2 m from the slit? [S.P. 2008]
21. Compare and contrast the pattern which is seen with
29. The following table gives data about the single slit
two coherently illuminated narrow slits in Young’s
diffraction experiment:
experiment with that seen for a coherently illuminated
Wavelength of light Half angular width of
single slit producing diffraction.
the principal maxima
22. A narrow slit is illuminated by a parallel beam of
l q
monochromatic light of wavelength l equals to 6000
pl qq
Å and the angular width of the central maxima in the
Find the ratio of the width of the slits used in the two
resulting diffraction pattern is measured. When the
cases. Would the ratio of the half angular width of
slit is next illuminated by light of wavelength l′, the
the first secondary maxima, in the two cases, be also
angular width decreases by 30%. Calculate the value
equal to q?[S.P. 2013]
of the wavelength l′.[CBSE S.P. 2018-19]
30. In Young’s double slit experiment, the distance ‘d’
23. In the diffraction due to a single slit experiment, the
between the slits S1 and S2 is 1 mm. What should be
aperture of the slit is 3 mm. If monochromatic light
the width of each slit so as to obtain 10 maxima of
of wavelength 620 nm is incident normally on the
the double slit pattern within the central maxima of
slit,, calculate the separation between the first order
the single slit pattern? [NCERT]
minima and the 3rd order maxima on one side of the
31. Name the phenomenon which is responsible for
screen. The distance between the slit and the screen
bending of light around sharp corners of an obstacle.
is 1.5 m. [Delhi 2019]
Under what conditions does this phenomenon take
24. In a single slit diffraction experiment, the width of the
place? Give one application of this phenomenon in
slit is decreased. How will the (i) size (ii) intensity
everyday life. [S.P. 2015]
of the central bright band be affected. Justify your
answer.[Delhi 2020] 32. In a single slit diffraction experiment, light of
25. Draw the graph showing intensity distribution of wavelength l illuminates the slit of width ‘a’ and the
fringes with phase angle due to diffraction through diffraction pattern is observed on a screen.
single slit.  [Delhi 2020]

364 Physics–12
D:\EG_Physics-12_(26-06-2022)\Open_Files\Ch-10\Ch-10
\ 27-Jul-2022 Ved_Goswami Proof-4 Reader’s Sign _______________________ Date __________

(a) Show the intensity distribution in the pattern with 34. (a) Explain two features to distinguish between
the angular position q. the interference pattern in Young’s double slit
(b) How are the intensity and angular width of central experiment with the diffraction patterns obtained
maxima affected when due to a single slit.
(i) width of slit is increased, and (b) A monochromatic light of wavelength 500 nm is
(ii) separation between slit and screen is decreased? incident normally on a single slit of width 0.2 mm
V. Long Answer Type Questions  (5 Marks) to produce a diffraction pattern. Find the angular
33. (a) Obtain the conditions for the bright and dark width of the central maximum obtained on the
fringes in diffraction pattern due to a single narrow screen.
slit illuminated by a monochromatic source. Estimate the number of fringes obtained in
Explain clearly why the secondary maxima go Young’s double slit experiment with fringe width
on becoming weaker with increasing n. 0.5 mm, which can be accommodated within
(b) When the width of the slit is made double, how the region of total angular spread of the central
would this affect the size and intensity of the maximum due to single slit.  [Delhi 2017]
central diffraction band? Justify. [AI 2012]

Answers 10.3
I. Objective Type Questions between the slit and screen. So angular separation
1. (i) (a) (ii) (c) (iii) (b) and (d) between fringes will remain same.
(iv) (a) and (b) (v) (b) 4. In single slit diffraction experiment linear fringe width
2λD
2. (i) Diffraction(ii) sound is β =
d

(iii) 3rd (iv) If d is doubled, the width of central maxima is halved.
d Thus size of central maxima is reduced to half. Intensity
3. (i) True (ii) True of diffraction pattern varies square of slit width. So, when
4. (a) the slit gets double, it makes the intensity four times.
II. Very Short Answer Type Questions 5. If d is slit width, intensity of central maximum I ∝ d2
1. The waves from a distant source are diffracted by the 1
edges of the circular obstacle. These diffraction waves so if width is halved, the intensity becomes times.
4
interfere constructively at the centre of the obstacle, so
2λD
a bright spot is seen there. 6. Linear Width of central maximum = , where d is
2. Difference between interference and diffraction d
1
Interference Diffraction
width of the slit, βCentral ∝
d
(i) It is due to the super- (i) It is due to the super-

position of two waves position of secondary Angular width =
d
coming from two wavelength originating
That is angular width of principal maximum decrease
coherent sources. from different parts of
with increase of width of the slit.
the same wave-front
7. The essential condition for diffraction to occur is the
(ii) T h e w i d t h o f t h e (ii) The width of the
size of the obstacle or aperture should be less than or
interference bands is diffraction bands is not of the order of wavelength of light used.
equal. the equal.
8. White light has different component of varying
(iii) The intensity of all (iii) The intensity of central wavelength. All components undergo constructive
maxima (fringes) is maxima is maximum interference at the central position and form white
same. and goes on decreasing
maximum but around it the positions of destructive
rapidly with increase in
interference for one component is overlapped by
order of maxima.
constructive component of other colour. That’s why
3. In single slit diffraction experimental angular separation coloured fringes are observed around the central
between fringes is independent of the separation maximum.

Wave Optics 365


D:\EG_Physics-12_(26-06-2022)\Open_Files\Ch-10\Ch-10
\ 27-Jul-2022 Ved_Goswami Proof-4 Reader’s Sign _______________________ Date __________

9. Diffraction effect is pronounced only when the 15. l = 500 nm, D = 1 m, y = 2.5 mm, a = ?
size of the obstacle/slit is less than or comparable For minima, Path diff. dq = nl ½
to wavelength of light used. In ordinary optical For first minima,
instruments, the sizes of the aperture are much larger
n= 1
than the wavelength of light, so the diffraction effect
are negligible. Hence the assumption that light travels dy = l ½
in straight line is justified. 1+1 D
λD 500 × 10 −9 × 1
10. The shaking of the pictures on our TV screen is due d= = = 2 × 10 −4 m  1
y 2.5 × 10 −3
to the interference of the direct signal with the signals
reflected from the low flying aircraft. 1+1 16. l = 700 nm = 700 × 10–9 m
a = ?, n = 1, q = 30°
11. In single slit diffraction if the path difference dq1, is
(a) For 1st minima : Path diff. = a sin q = l
equal to l then first minima will be observed
λ λ 700 × 10 −9
i.e., For 1st minima θ1 = , where d is slit width. ½ So a = = = 1.4 × 10–6 m 1
d sin θ sin 30°
There will be minima on either side of central maxima. 3λ
(b) For the first maxima a sin q =
So the width of central maxima is 2q1. 2

The second minima will be at an angle θ 2 =  ½ 3λ 3 × 700 × 10 −9
d or a = = = 2.1 × 10–6 m 1
2 sin θ 2 × sin 30°
So the width of 1st secondary maxima
17. l = 600 nm = 600 × 10–9 m,
2λ λ λ
= q2 – q1 = − =  1 Slit width = a = ?, D = 0.8 m, n = 2
d d d
yn = 1.5 mm = 1.5 × 10–3 m
Thus the width of secondary maxima is half the width
of central maxima. For nth maxima;
λD
12. The essential condition for diffraction to be more yn = (2n + 1)  1
pronounced if the size of the aperture or obstacle 2a
is of the order of the wavelength of wave. As the λD (2n + 1) 600 × 10 −9 × 0.8 × (5)
or a= =
wavelength of sound is comparable to the size of the 2 yn 2 × 1.5 × 10 −3
obstacle or aperture found around us, so diffraction of = 0.8 × 10–3 m = 0.8 mm 1
sound is common in everyday life. In case of light, the
18. l = 500 nm = 500 × 10 m,–9
D = 1 m,
wavelength of light is much smaller (~ 10– 6m) than the
size of the objects around us. So diffraction of light is n = 1, y = 2.5 mm = 2.5 × 10 m, a = ?
–3

not seen easily. 1+1 For minima,a sin q = nl 1


λ λD −9
500 × 10 × 1
2λD i.e., a= = =
13. Linear Width of central maximum = θ y 2.5 × 10 −3
d
When slit width (d) is doubled, width of central = 0.2 × 10–3 m = 0.2 mm 1
1 OR
maximum is halved. Its area become times and hence
4
(i) Same as Ans. 18 above
intensity becomes 4 times the initial intensity. 1+1 (ii) θ = ( 2n + 1) λ , where n=1,2,…for

n
14. The linear width of central bright maximum is given 2a
by secondary maxima
2Dλ For first secondary maximum, n =1
b0 =
a λ 3λ
(i) If monochromatic yellow light is replaced with \ θ1 = ( 2 × 1 + 1) =
2a 2a
red light, the linear width of the central maximum Distance of first secondary maximum from the centre
increases because lred > lyellow. 1 of the screen
(ii) If the distance (D) between the slit and the screen is 3λ 
= y1= q1D =  D
increased, the linear width of the central maximum  2a 
increases. 1 we have a = 2.0×10–4 m from Ans. 18

366 Physics–12
D:\EG_Physics-12_(26-06-2022)\Open_Files\Ch-10\Ch-10
\ 27-Jul-2022 Ved_Goswami Proof-4 Reader’s Sign _______________________ Date __________

 3 × 500 × 10−9  22. Angular width


\ y1=  −4  × 1 = 3.75×10 m
–3
2q = 2l/d ½
 2 × 2 × 10  Given l = 6000 Å
λ In Case of new l (assumed l′ here), angular width
19. For maxima, qn = (2n + 1)
2d decreases by 30% ½
n is order of secondary maxima n = 1, 2, 3, ...  100 − 30 
=  2q = 0.70 (2q) ½
Angular position of 1st order maxima q1 = 3λ  ½  100 
2d 2l′/d = 0.70 × (2 l/d)
nλ ∴ l′= 4200 Å ½
Angular position of nth minima qn =
d [CBSE Marking Scheme, 2018-19]
3λ 23. a = 3 mm = 3 × 10–3 m
For 3rd order minima, q3′ =  ½
d l = 620 nm = 620 × 10–9 m, D = 1.5 m
Distance of 1st order minima from central maximum
q3′ – q1= 3λ − 3λ = 3λ  ½
d 2d 2d λD
xn = n
3 × 600 × 10 −9 a
= = 9 × 10 −4 rad  ½ 620 × 10 −9 × 1.5
2 × 10 −3 x1 = 1 ×
20. l = 500 nm = 500 × 10–9 m, d = 0.5 mm = 0.5 × 10–3 m, 3 × 10 −3
D=2m = 3.1 × 10–4 m ½
Distance between the two dark bands, on the either Distance of 3rd order maxima from central maximum
side of the central bright bands is the width of central  1  λD
maxima = b0 1 xn =  n +   ½
2 a
2λD 2 × 500 × 10 −9 × 2  1  620 × 10 −9 × 1.5
b0 = =
d 0.5 × 10 −3 x3 =  3 + ×
 2 3 × 10 −3
= 4 × 10–3 m = 4 mm 1
7
21. x3 = × 3.1 × 10–4 m
Central First 2
bright
Intensity
bright = 10.85 × 10–4 m ½
Separation between 1st order minima and 3rd order
 ½ maxima
0 λ λ 3λ 2λ x3 – x1 = 10.85 × 10–4 m – 3.1 × 10–4 m
Path diff.
2 2 = 7.75 × 10–4 m = 0.775 mm ½
24. (i) The size of the central maximum = 2l/a
Interference Pattern Diffraction Pattern where a = slit width.
1. All fringes are of equal 1. All fringes are not of So, If a is decreased, the size of the central maximum
width. equal width. The central will increase.
maxima is twice the (ii) However, the intensity changes due to the following
width of secondary factors:
maxima. • D  ecreasing the width of the slit, causes less light
2. Intensity of light at 2. Minima’s can never be energy to tall on the screen as compared to that of
minima may be zero perfect darkness i.e., the original one.
if the two superposing the intensity of light at • The light energy is now spread into a larger area on
waves are of same minima will never be the screen because the size of the central maximum
amplitude. zero. get enlarged.
3. All maxima’s will have 3. The intensity of central
Intensity (I)
same intensity. maxima is maximum
and goes on decreasing
rapidly with increase in 25. (a)
order of maxima. –3l –2l –l 0 l 2l 3l
a a a a a a
½+½+½
Wave Optics 367
D:\EG_Physics-12_(26-06-2022)\Open_Files\Ch-10\Ch-10
\ 27-Jul-2022 Ved_Goswami Proof-4 Reader’s Sign _______________________ Date __________

(b) Let ‘a’ be the width of each slit. linear separation 3 λ1D
between n bright fringes, For l1, y1 =  ½
2 a
nλ D
x = nb = 3 590 × 10 −9
d y1 = × × 1.5 = 6.64 mm  ½
x nλ 2 2 × 10 −4
Corresponding angular separation, q1= =
D d 3 λ2D
Now, the angular width of central maximum in For l2, y2 =
2 a
the diffraction pattern of a single slit, θ2 = 2λ 3 596 × 10 −9 × 1.5
a = × = 6.705 mm
As q1= q2 so nλ = 2λ 2 2 × 10 −4
d a
y2 – y1 = 6.705 – 6.64 = 0.065 mm 1
2d 28. l= 550 nm = 550 × 10 m, D = 1.1 m
–9
\ a= ;
n d= 0.1 mm = 1 × 10–4 m, w0 = ?, b0 = ?,
where d = separation between slits 2λ 2 × 550 × 10 −9
w0 = 2θ = =
26. 1 d 1 × 10 −4
I
= 1.1 × 10–2 rad 1
Linear width of principal maxima
2λD
–3l –2l –l O l 2l 3l Position on screen b0 = = 1.1 × 10 −2 × 1.1 = 12.1 mm  1
d d d d d d d
When the screen is moved to 2.2 m from the slit,
(i) Width of central maxima βo = 2λD the angular width will not change, linear width will
d increase. 1
d → slit width, D is distance between slit and screen.
29. Let the slit width be a1 and a2 in the two cases
If d is doubled, width of central maxima will become
half of initial value. 1 λ pλ
q= and qθ =  1
(ii) b0 ∝ l a1 a2
If l is increased, bo will also increase. 1 qθ = pλ × a1 or a1 = q 
1
27. D = 1.5 m, a = 2 × 10 m,–4
θ a2 λ a2 p
l1 = 590 nm = 590 × 10–9 m Half angular width of second case
l2 = 596 nm = 596 × 10–9 m
Half angular width of firsst case
3 pλ
2 a2 a q
= = p 1 = p × = q 1
y      ½ 3 λ a2 p
θ 2 a1
a O 30. The linear distance of n bright fringes in an interference
pattern on the screen is
nλD
yn = , As yn << D 1
D d
y nλ
 1 So the angular separation is given by qn = n = ½
For maxima: Path diff. =  n + λ D d
2 10λ
For n = 10, q10 = , d is distance between two slits.
 1 d
i.e., a q = n +  λ  ½
 2 The angular width of the central maxima in single slit
For 1 maxima, n = 1
st diffraction pattern of a slit of width ‘a’
y 2λ
tan θ −
~ θ= = , a is slit width ½
    D a
ay 3 10λ 2λ d 0.1
So = λ So = or a= = = cm 0.02 cm  1
D 2 d a 5 5

368 Physics–12
D:\EG_Physics-12_(26-06-2022)\Open_Files\Ch-10\Ch-10
\ 27-Jul-2022 Ved_Goswami Proof-4 Reader’s Sign _______________________ Date __________

31. Diffraction: 1 λ
Since q1 is very small, sin q1 ≈ q1 =
Size of the slit/obstacle should be less than or d
comparable to the wavelength of light used. 1 In general when d sin q = nl minima will be
Diffraction is used in ultrasound scanning to estimate obtained.
the size and shape of the tumours, ulcers in human body. 3λ 5λ
 1 Secondary maxima: If BN = 2 or 2
Intensity
32. (a) Then the point P on the screen will be a maxima
called 1st secondary maxima. It is because, the slit
can be considered to be divided into three equal
parts, the wavelets from 1st two parts will reach P in
–2l –3l –l 0 l 3l 2l opposite phase, cancelling the effect of each other,
2
2 Angular position the wavelets from the 3rd part of the slit remain
uncancelled and produce maxima at P
(b) i. Intensity increases, Angular width (q) decrease



nλ BN = d sin θ =
 q = where d = width of slit 2
d 5λ
   ii. Intensity increases due to nearness, no effect on We get 2nd secondary maxima q =  1
2d
angular width λ
33. (a) Consider a parallel beam of monochromatic light In general qn = (2n + 1) we get maxima.
2d
incident normally upon a slit AB. Then each point
Secondary maxima becomes weaker with
of AB becomes a source of secondary disturbance
increasing n: The intensity of central maxima is
or wavelets.
due to constructive interference of wavelets from
Now consider a point O on the screen which is all parts of the slit, whereas the first secondary
placed at distance D from slit AB. Since point O is maxima is due to the contribution of wavelets from
equidistant from A and B, therefore, the secondary the one-third part of the slit and in second secondary
wavelets from AB reach the point O in the same maxima, the contribution is due to the wavelets
phase and hence constructive interference takes place from the one fifth part of the slit and so on. Hence
at O. the intensity of secondary maxima’s decreases with
P the increase in the order n of the maxima. ½

{
A
(b) When the width of the slit is made double:
{
y1
θ
d θ O 2λD
C N As bCentral = , 1
d
B ½ d is slit width
D If d is made double, the width of central maxima
Let us now consider the secondary wavelets travelling will become half of original value.
in a direction making an angle q with CO. Let these Intensity ∝ (slit width)2 1
waves meet at a point P. The intensity at P will
So intensity will become four times. ½
depend on path difference between the secondary
wavelets emitted from the corresponding points of 34. (a) See Answer 21. 1+1
the wavefront. (b) (i) l = 500 nm = 500 × 10 m, –9

Path difference = BN = d sin q a = 0.2 mm = 0.2 × 10–3 m


If d sin q = l Angular width of central maxima
Then the point P will be of minimum intensity. It is wo = 2λ  ½
because, if the slit is assumed to be divided into two a
equal parts then, wavelets from the corresponding 2 × 500 × 10 −9
points of the two halves of the slit will have a = −3
= 5 × 10–3 radian 1
λ 0.2 × 10
path difference of and hence they will produce
d 2λD
minima. (ii) In diffraction pattern b0 = (Central maxima)
a
For first minima,d sin q1 = l
λD
λ In interference b =  ½
sin q1 =  ½ d
d
Wave Optics 369
D:\EG_Physics-12_(26-06-2022)\Open_Files\Ch-10\Ch-10
\ 27-Jul-2022 Ved_Goswami Proof-4 Reader’s Sign _______________________ Date __________

Let n be the number of interference fringe which nλD 2λD 2d


can be accommodated in the central maxima then = or n= = 2 1
d a a
n b = b0

Case Based Questions


I. When the light from a monochromatic source is II. The phenomenon of bending of light around the
incident on a single monochromatic slit, it gets sharp corners and the spreading of light within the
diffracted and a pattern of alternate bright and geometrical shadow of opaque obstacles is called
dark fringes is obtained on screen, called diffraction diffraction of light. The light thus deviates from
pattern of single slit. In diffraction pattern of single its linear path. The deviation becomes much more
slit, it is found that.
pronounced, when the dimensions of the aperture
(i) Central bright fringe is of maximum intensity or the obstacle are comparable to the wavelength of
and the intensity of any secondary bright fringe light.
decreases with increase in its order. Incident
(ii) Central bright fringe is twice as wide as any other wave
secondary bright or dark fringe. Diffracted wave

Screen
1. Why does light seem to propagate in rectilinear
path?
2. On which factors, in diffraction from a single
Incident
light Slit slit, does the angular width of the central maxima
depend?
Viewing screen
3. In a single diffraction pattern observed on a screen
1. A diffraction pattern is obtained by using a beam placed at D metre distance from the slit of width
of red light. What will happen if the red light is d metre, what will be the ratio of width of central
replaced by the blue light? maxima to the width of the other secondary
2. A single slit of width 0.1 mm is illuminated by a maxima.
parallel beam of light of wavelength 6000 Å and Ans. 1. The wavelength of visible light is very small, that
diffraction bands are observed on a screen 0.5 m hardly shows diffraction, so it seems to propagate
from the slit. What is the distance of third dark in rectilinear path.
band from the central bright band? 2. In diffraction from a single slit, the angular width
3. Write condition to observe the diffraction. of the central maxima depends on
Ans. 1. The diffraction pattern bands will become narrow (i) Wavelength of light used and
and crowded together. (ii) Width of slit 2λD
2. For third dark band. d sin q = 3l 3. Width of central maxima =
d
3λ y λD
\ sin q = =   Width of other secondary maxima =
d D d
3Dλ 3 × 0.5 × 6 × 10 −7 ... Width of central maxima : Width of other
= y=
d 0.1 × 10 −3 secondary maxima = 2:1
= 9 × 10–3 m = 9 mm III. The phenomenon of interference is a strong evidence
3. To observe diffraction the size of obstacles should in support of wave nature of light. Interference results
be of the order of wavelength. due to super position of light waves from two coherent

370 Physics–12
D:\EG_Physics-12_(26-06-2022)\Open_Files\Ch-10\Ch-10
\ 27-Jul-2022 Ved_Goswami Proof-4 Reader’s Sign _______________________ Date __________

sources. Coherent sources continuously emit light S1 and S2 is 12500 Å. The path difference is zero
of same frequency with constant phase difference. at O and a central bright fringe of intensity ʻI0ʼ is
Energy get redistributed and intensity (I) is maximum obtained at O. Q is another point 2.0 mm from O.
at some points and minimum at other points. If the screen is now shifted to a new position (1′)
[not mention in the Fig.] so that D changes, the fringe
width is found to be 50% more than its previous value
and the angular fringe width become 0.1°.
[Assume D to be large and sin q ≈ tan q ≈ q as ʻqʼ is
very small].
1. When the screen is at position (1), phase difference
of waves from coherent sources S1 and S2 at point
P is effectively
(a) 90° (b) 135° (c) 180° (d) 45°
2. Intensity at point P when the screen is at position
In fact, intensity at any point that receives light
(1), is
waves from both sources depends on the phase
I
or path difference. On such points at which path (a) 20 I0 (b) 7 I0 (c) zero (d) 80
difference Dx = nl, where n is an integer, On 3. Value of fringe width when screen is at position
intensity is maximum while, at points where (1) will be
(2n + 1) λ
Dx = , intensity is minimum. Interference (a) 1.00 mm (b) 0.98 mm
2
was discovered by Thomas Young. Consider that a (c) 0.44 mm (d) 0.72 mm
source of monochromatic light of wavelength 5000 4. The intensity at point Q when the screen is at
Å is placed at S. Two slits, S1 and S2 are equidistant position (1), will be
from S and have equal widths. A wavefront originates (a) 2.5 I0 (b) 0.65 I0
from S, and S1 and S2, being points on this wavefront, (c) 3.35 I0 (d) 0.02 I0
emit light waves, which superimpose to result in an 5. Value of distance ʻdʼ between S1 and S2 is
interference pattern on the screen kept at the position
(a) 0.29 mm (b) 0.69 mm
(1). O is a point on the screen equidistant from slits
S1 and S2. P is a point on the screen 1.1 mm from O (c) 0.79 mm (d) 0.86 mm
at which path difference between waves from slits Ans. 1. (c) 2. (c) 3. (c) 4. (d) 5. (a)

IMPORTANT FORMULAE
Formulae Symbols Application
1. A = a1 + a2 + 2a1a2 cos f
2 2 2
A = Resultant amplitude To determine ratio of the intensities
Or I = Intensity or amplitude of the resultant wave.
I = I1 + I2 + 2 I1 I 2 cos f
2. 2π To determine wavelength
Df = ∆x Df = Phase difference
λ Dx = path difference
3. I max (a + b) 2 Imax = Max. intensity To find ratio of max. and min.
=
I min (a – b) 2 Imin = Min. intensity intensity
a,b = respective amplitudes
4. (i) Condition for max. intensity at a To determine the position of bright
point in interference and dark fringes.
Df = 2np; Dx = nl
where n = 0, 1, 2, ...
(ii) Condition for min. intensity at a
point in interference
λ
Df = (2n –1) p; Dx = (2n –1)
2

Wave Optics 371


D:\EG_Physics-12_(26-06-2022)\Open_Files\Ch-10\Ch-10
\ 27-Jul-2022 Ved_Goswami Proof-4 Reader’s Sign _______________________ Date __________

5. In YDSE: For Bright fringe yn = distance of nth bright fringe To determine the wavelength of a
λD from the center monochromatic light
(i) yn = n
d qn = Angular position of nth bright To determine the angular width of
yn nλ fringe a fringe.
(ii) qn = =
D d
For dark fringe: n = order of fringe To find l and q
λD
(i) yn = (2n –1)
2d
yn ( 2n − 1) λ
(ii) qn = D = 2d
6. λD b = fringe width To find fringe width for interference
β= fringes
d l = wavelength of light
D = distance between the slits and
the screen
d = distance between the slits
7. Diffraction of light: y = Distance of nth secondary In study of bending of light ray
n
λD max. from the centre of the
(i) yn = (2n + 1)
2a screen
( 2n + 1) λ qn = 
Angular position of nth
(ii) qn = secondary maximum
2a

(iii) Width of central maxima a = Slit width In study of spreading of light


2λD
=
a
(iv) Width of secondary maxima or
minima
λD
=
a

COMMON ERRORS
S No. Errors Corrections
1. Differences between Interference and Differences should be learnt with the help of graphs or patterns.
Diffraction
2. Forgot to recognised the fringe type i.e. Highlight the type of fringe in the exam question paper with
bright or dark, and always applied the slight pencil mark.
formula for bright fringes
3. Student get confussed between linear width Central max. of a diffraction battern is twice the secondary max.
of central max. (b0) and linear width of b0 = 2b
secondary max. (b)

372 Physics–12
D:\EG_Physics-12_(26-06-2022)\Open_Files\Ch-10\Ch-10
\ 27-Jul-2022 Ved_Goswami Proof-4 Reader’s Sign _______________________ Date __________

REVISION CHART

Wavefronts and Huygen’s Principle


Wavefronts: It is a continuous locus of all the points in the same phase of vibrations. Wavefronts are perpendicular to the direction
of propagation of energy.
Huygen’s Principle: Each point on a wavefront act as fresh a source of secondary waves, which add up to give a wavefront at
any later time.

Spherical Wavefront Cylindrical Wavefront Plane wavefront


Wavefronts are spherical when source When the source of light is linear in Plane wavefront is a limited or a
of light is a point source. shape, for example fine rectangular slit small part of a large spherical or
then wavefront is cylindrical in shape. cylindrical wavefront.

Interference
The phenomenon of redistribution of energy in the region when two waves of same frequency travelling in the same direction with
zero or constant initial phase difference superimpose is called interference.

Constructive interference Destructive interference


When two waves of same frequency travelling in same direction When two waves superimpose out of phase.
superimpose in phase.

Diffraction at Single Slit


Diffraction is the phenomenon of bending of light around the corners of an obstacle or an opening in its path. Due to diffraction
light spreads into the regions of geometrical shadow.

Graph for Variation of Intensity


Broad central maximum is in direction q = 0° of incident light. On,
either side, it has secondary maxima of decreasing intensity.

Wave Optics 373


D:\EG_Physics-12_(26-06-2022)\Open_Files\Ch-10\Ch-10
\ 27-Jul-2022 Ved_Goswami Proof-4 Reader’s Sign _______________________ Date __________

IMPORTANCE OF EACH TOPIC AND FREQUENTLY ASKED TYPES OF QUESTIONS

☞ Important Topics
1. Questions related to Y.D.S.E., Single slit. 2. Questions on Huygen’s principle and interference.
3. Questions based Fringe width.

1. Write two difference between interference and diffraction.


2. Red light of wavelength 750 nm enters a glass plate of refractive index 1.5. If the velocity of light in vacuum is 3
× 108 ms–1, calculate in the glass (i) frequency (ii) velocity and (iii) wavelength of light.[Ans. (i) 4 × 1014 Hz, (ii)
2 × 108 ms–1, (iii) 500 nm]
3. The two slits in Young’s double slit experiment are separated by a distance of 0.03 mm. An interference pattern
is produced on a screen 1.5 m away. The 4th bright fringe is at a distance of 1 cm from the central maximum.
Calculate the wavelength of light used.  [Ans. 500 Å]
4. Two wavelengths of sodium light 590 nm, 596 nm are used, in turn, to study the diffraction taking place at slit
and the screen is 1.5 m. Calculate the separation between the positions of first maximum of the diffraction pattern
obtained in the two cases.  [Ans. 6.77 mm]

374 Physics–12
D:\EG_Physics-12_(26-06-2022)\Open_Files\Ch-10\Ch-10
\ 27-Jul-2022 Ved_Goswami Proof-4 Reader’s Sign _______________________ Date __________

ASSIGNMENT
I. Objective Type Questions (1 Mark)
1. Multiple choice questions:
(i) Two waves having intensity in the ratio 25 : 4 produce interference. The ratio of the maximum to minimum
intensity is
(a) 5 : 2 (b) 7 : 3 (c) 49 : 9 (d) 9 : 49
(ii) The condition for observing Fraunhoffer diffraction from a single slit is that the light wavefront incident on the
slit should be
(a) spherical (b) cylindrical (c) plane (d) none of these
2. Fill in the blanks:
(i) Two sources with zero or constant phase difference between them are called ............. sources.
(ii) The penetration of light into the region of geometrical shadow is called .................... .
II. Very Short Answer Type Questions (1 Mark)
3. State the essential conditions for two light waves to be coherent.
4. How does the angular separation between fringes in single slit diffraction experiment change when the distance
of separation between the slit and screen is doubled?
III. Short Answer Type Questions-I (2 Marks)
5. A slit of width ‘a’ is illuminated by white light. For what value of ‘a’ is the first minima, for red light of l = 650
nm, located at point P? For what value of wavelength of light will the first diffraction maxima also fall at P?
6. Find the ratio of intensities at two points P and Q on a screen in a Young’s double slit experiment when waves
p p
from S1 and S2 have a phase difference of : (i ) and (ii ) respectively.
3 2
7. Draw reflected/refracted wavefronts when an incident wavefront is plane wavefront from (i) concave mirror (ii)
convex lens
IV. Short Answer Type Questions-II (3 Marks)
8. Consider two coherent sources S1 and S2 producing monochromatic waves to produce interference pattern. Let the
displacement of the wave produced by S1 be given by
y1 = a cos wt. and the displacement by S2 be y2 = a cos (wt + f).
Find out the expression for the amplitude of the resultant displacement at a point and show that the intensity at
φ
that point will be I = 4a 2 cos 2 . Hence establish the conditions for constructive and destructive interference.
2
How will the intensity of transmitted light vary on further rotating the third polaroid?
9. A slit of width 0.6 mm is illuminated by a beam of light consisting of two wavelength 600 nm and 480 nm. The
diffraction pattern is observed on a screen 1.0 m from the slit find:
(i) The distance of the second bright fringe from the central maximum pertaining to light of 600 nm
[CBSE 2022]
(ii) The least distance from the central maximum at which bright fringes due to both the wavelengths coincide.
V. Long Answer Type Question (5 Marks)
10. (a) Use Huygens’ principle to show the propagation of a plane wavefront from a denser medium to a rarer medium.
Hence find the ratio of the speeds of wavefronts in the two media.
(b) State the essential condition for diffraction to occur. 



Wave Optics 375


D:\EG_Physics-12_(26-06-2022)\Open_Files\Ch-11\Ch-11
\ 27-Jul-2022 Ved_Goswami Proof-4 Reader’s Sign _______________________ Date __________

Topics Covered
11 Dual Nature of Radiation and Matter

11.1 Photo-electric Effect and Electron Emission 11.2 Wave Nature of Matter

C hapter map
Dual Nature of
Radiation and Matter

Wave nature of matter Particle nature of


radiations

de-Broglie waves or
matter waves Photoelectric effect

h h
λ= ,λ =
p 2mK Based on Work Graphs Governed by Laws
h 12.27
function and 1. P h o t o e l e c t r i c c u r r e n t v s of photoelectric
λ= = Å threshold frequency potential at varying intensity. effect
2me V V
2.  P h o t o e l e c t r i c c u r r e n t v s
potential at varying frequency.
3. Stopping potential vs frequency Einstein’s equation
of radiation of photoelectric
emission
1 2
hv = φ0 + mVmax
2

Topic 1. Photo-electric Effect and Electron Emission


•• Photon Picture of Electromagnetic Radiation: hc
According to quantum theory, energy of electromagnetic E = hv =
λ
wave is carried by discrete packets or bundles of where h = Planck’s constant
energy,called quantums.
value of h = 6.6 × 10–34 Js.
1 quantum of radiation of light is called a photon, which
(iii) Frequency of photon does not change when it travels
travels with speed of light.
from one medium to another.
•• Properties of Photons:
(iv) Speed of photon changes as it travels through different
(i) All photons emitted by any source travel through free media due to change in its wavelength.
space with speed of light i.e., 3 × 108 m/s.
(v) Photon are electrically neutral and are not deflected
(ii) Each photon has a definite energy depending on by electric or magnetic field.
frequency of radiation.

376
D:\EG_Physics-12_(26-06-2022)\Open_Files\Ch-11\Ch-11
\ 27-Jul-2022 Ved_Goswami Proof-4 Reader’s Sign _______________________ Date __________

(vi) A photon may collide with a material particle. Total Secondary Emission: When fast moving electrons strike
energy and total momentum remain conserved in a metal surface they transfer some of their energy to
such collisions. But number of photons may not be the electrons of the metal. As a result the energy of the
conserved. electrons increases and when it becomes more than the
(vii) If the intensity of light of given wavelength is increased, work function, electron leaves the metal surface.
there is an increase in number of photon incident on a Photoelectric Emission: It is the process in which
given area in a given time. But energy of each photon electrons are emitted from a metal surface when
remains the same. electromagnetic radiation of suitable frequency are
(viii) As mass of particle varies according to the relation incident on it. The electrons so emitted are called photo
mo electrons.
m=
v2 •• Photoelectric Effect: It is the phenomenon of emission
1− 2 of electrons from surface of a metal when radiations of
c
where mo = rest mass of particle (mass of particle suitable frequency fall on it.
at rest) •• Hertz and Lenard’s Experiment: The phenomenon
c = speed of light of photoelectric emission was discovered in 1887 by
For a photon moving with speed of light Heinrich Hertz (1857-1894), during his electromagnetic
wave experiments. In his experimental investigation
mo = 0
on the production of electromagnetic waves by means
Rest mass of photon is 0. of a spark discharge. Hertz observed that high voltage
(ix) Moving mass ‘m’ of a photon can be determined by sparks across the detector loop were enhanced when the
Einstein’s mass-energy relationship emitter plate was illuminated by ultraviolet light from
E = mc2 = hv an arc lamp.
m = hv
Ultraviolet
Electrons light
c2 Quartz
bulb
(x) Linear momentum of photon p = mc – +

c = vl C A

E = hv = mc2
hv h  c  mA
mc = = c = vλ ⇒ v = λ 
c λ – +

Momentum of photon Battery

h •• Light shining on the metal surface somehow facilitated


p = mc = the escape of free, charged particles which we now
λ
know as electrons. When light falls on a metal surface,
•• Work Function: The minimum amount of energy some electrons near the surface absorb enough energy
required by an electron to just escape the metal surface from the incident radiation to overcome the attraction
is called its work function. of the positive ions in the material of the surface. After
It is represented by Wo and its S.I. Unit = eV (electron gaining sufficient energy from the incident light, the
volt) electrons escape from the surface of the metal into the
1 eV is the kinetic energy gained by an electron when it surrounding space.
is accelerated through a potential difference of 1V. •• Wilhelm Hallwachs and Philipp Lenard investigated the
1 eV = 1.6 × 10–19 J phenomenon of photoelectric emission in detail during
•• Electron Emission: Thermionic emission: When a 1886-1902.
metal is heated, its electrons get sufficient thermal energy •• Lenard (1862-1947) observed that when ultraviolet
and they can overcome the surface barrier. radiations were allowed to fall on the emitter plate
This method is called thermionic emission. of an evacuated glass tube enclosing two electrodes
Field emission /cold cathode emissions: When metal (metal plates), current flows in the circuit. As soon as
surface is subjected to very high electric field, electrons the ultraviolet radiations were stopped, the current flow
are emitted. also stopped. These observations indicate that when

Dual Nature of Radiation and Matter 377


D:\EG_Physics-12_(26-06-2022)\Open_Files\Ch-11\Ch-11
\ 27-Jul-2022 Ved_Goswami Proof-4 Reader’s Sign _______________________ Date __________

U.V. radiation is incident on emitter plate, electrons – In 1900, Lenard argued that when UV light is
are ejected. The electrons flow through the glass tube incident on emitter plate, it cause emission of
resulting in flow of current. electrons from its surface.
•• Hallwachs’ and Lenard Observations: – These electrons are attracted by +ve collector plate
– A zinc plate was connected to an electroscope. so that circuit is completed and current flows.
This current was called photoelectric current.
– Ultraviolet light was made to fall on a zinc plate.

Hallwach and Lenard also observed that when
– It was observed that the zinc plate became : (i)
frequency of incident light were less than certain
uncharged if initially negatively charged and (ii)
min. value, called threshold frequency, no
positively charged if initially uncharged and (iii)
photoelectrons were emitted at all.
more positively charged if initially positively
charged. •• Experimental Study of Photoelectric Effect: It
Ultraviolet consists of evaluated glass or quartz tube enclosing two
light
Zinc plate electrodes, cathode and anode.

C A
Photo-electrons

µA
V

Electroscope

– It was concluded that some negatively charged


particles were emitted by the zinc plate when
exposed to UV light.
Lenard Observations:
– When UV radiations of suitable frequency are ( )
allowed to fall on the emitter plate of an evacuated
glass tube, enclosing 2 electrodes, a current flows in Radiations are made to fall on plate C through window.
circuit. Potential divider unit is used to vary the potential
Ultraviolet difference between C and A.
light
Quartz (a) Effect of Intensity of Light on Photoelectric
– +
bulb Current:
C Electrons A

µA
– +
Photo
Battery electric
current
– As soon as UV radiations are stopped, current also
stops.
– These observations indicate that UV radiations
incident on emitter plate (eject out some –vely charged
particles from it.) Intensity of light

– These particles are attracted by collector plate A,


The number of photo electrons emitted per second
setting up a current through evacuated glass tube.
is proportional to intensity of light when potential
– After discovery of electrons by J.J. Thomson in difference between the electrodes and frequency of
1897, it was established that these particle are incident radiation is kept constant.
electrons and were called photoelectrons.

378 Physics–12
D:\EG_Physics-12_(26-06-2022)\Open_Files\Ch-11\Ch-11
\ 27-Jul-2022 Ved_Goswami Proof-4 Reader’s Sign _______________________ Date __________

(b) Effect of Potential (n = constant) radiation energy continuously. Greater the intensity
I1 , I2 are intensity of of radiation, grater are the amplitudes of electric and
Photo
incident light magnetic field and hence greater is the energy density
electric
I2
of the wave. So higher intensity should liberate
current
I2 > I1 photoelectrons with greater kinetic energy but this
I1
contradicts the experiment observations.
(ii) No matter what the frequency of incident radiation
Stopping light of sufficiently high intensity should be able
potential to eject the electrons from the metal surface. So
wave theory fails to explain existence of threshold
Potential difference frequency.
(c) Effect of potential (I = Constant): (I — Intensity) (iii) As the energy of light wave is uniformly distributed
over its wavefront, the electron intercepts a very small
Photo amount of this energy and hence it should require
electric finite time to escape from the metal surface. But
current
actually, the emission is almost instantaneous.
v2 > v1 Note: Photoelectric effect confirms particle nature of
v2
light.
v1
•• Einstein’s Theory of Photoelectric Effect:
(i) Photoelectric emission is result of interaction of one
photon with one electron of metal.
V02 V01 Potential difference
(Photon is of incident radiation).
(d) Effect of Frequency on Stopping Potential
(ii) As the electrons are bound with metal, so minimum
amount of energy is required to liberate an electron
Metal1 Metal2
Stopping from the metal surface i.e., wave function.
potential (iii) When a photon interacts with one electron, part of
its energy is used in liberating electron, (i.e., work
function) while remaining energy is carried by
ν01 ν02 electron as kinetic energy.
Frequency of incident radiation E = Wo + (KEmax)
1
•• Laws of Photoelectric Emission: hv = hvo + mVmax 2
2
(i) Photoelectric emission is an instantaneous process.
1 2
(ii) For a given photosensitive material, there exists a mVmax = hv – hv0 = h(v – vo)
2
certain minimum, cut off frequency below which
photoelectrons are not emitted. This frequency is If v < vo
called threshold frequency. KE = –ve which is not possible.
(iii) Above threshold frequency, maximum kinetic energy So there has to be a minimum frequency required to
of the photoelectrons is directly proportional to emit the electrons.
frequency of incident radiation but is independent of •• Explanation of Kinetic Energy: For v > vo, (Kinetic
intensity. Energy)max = +ve
(iv) The photoelectric current is directly proportional to As the frequency of incident radiation increases, kinetic
the intensity of light for a given frequency of incident energy carried by photoelectrons will also increase.
radiation. Hence, more stopping potential is required to stop the
Saturation current ∝ Intensity of incident radiation. photoelectric current.
•• Failure of Classical Wave Theory: •• Explanation of Intensity: Increase in intensity means
increase in no. of photons striking per unit time i.e.,
(i) According to wave theory, when a wavefront strikes
increase in no. of e– being emitted.
metal surface, the electrons in metal surface absorb
Dual Nature of Radiation and Matter 379
D:\EG_Physics-12_(26-06-2022)\Open_Files\Ch-11\Ch-11
\ 27-Jul-2022 Ved_Goswami Proof-4 Reader’s Sign _______________________ Date __________

As each photon eject only one electron, the no. of If Vo is stopping potential
photoelectrons emitted will also increase with increase KEmax = eVo
in intensity.
So saturation current depends on intensity.
•• Determination of Planck’s Constant: Stopping
potential
(KEmax) = hv – hvo
At A, v=0
O
(KEmax) = 0 – hvo = – hvo = – OA. v0 Frequency
v

KEmax KE max
KEmax Vo =
e
O v0
KEmax = hv – hvo
v
eVo = hv – hvo
h h
Vo = v − vo
A e e
h
Work function = – (intercept on y-axis) Now slope becomes
e
KE max
h= OR h = e × slope of V0 vs v curve
v − vo
 Work function 
Slope of the line gives Planck’s constant (h) Intercept = −  e


EXERCISE 11.1
I. Objective Type Questions (1 Mark) (a) decreases with increasing n with n fixed.
1. Choose the correct answers from the given options (b) decreases with n fixed n increasing.
(MCQs). (c) remains constant with n and n changing such
(i) When ultraviolet rays incident on metal plate then that nn is a constant.
photoelectric effect does not occur, it occurs by (d) increases when the product nn increases.
incidence of (v) Photoelectric emission from a given surface of
(a) Infrared rays (b) Radio waves metal can take place when the value of a ‘physical
(c) X-rays (d) Light waves quantity’ is less than the energy of incident photon.
(ii) Photoelectric effect shows The physical quantity is: [SP 2019-20]
(a) Wave nature of electrons (a) Threshold frequency
(b) Particle nature of light (b) Work function of surface
(c) Both (a) and (b) (c) Threshold wavelength
(d) None of these (d) Stopping Potential
(iii) The lowest frequency of light that will cause the (vi) Photons of energies 1 eV and 2eV are successively
emission of photoelectrons from the surface of a incident on a metallic surface of work function 0.5
metal (for which work function is 1.65 eV) will eV. The ratio of kinetic energy of most energetic
be photoelectrons in the two cases will be
(a) 4 × 1010 Hz (b) 4 × 1011 Hz [Delhi, AI 2020]
(c) 4 × 10 Hz
14
(d) 4 × 10–10 Hz (a) 1 : 2 (b) 1 : 1 (c) 1 : 3 (d) 1 : 4
(iv) Photons absorbed in matter are converted to (vii) If photons of frequency v are incident on the
heat. A source emitting n photons per-second of surfaces of metals. A & B of threshold frequencies
frequency n is used to convert 1 kg of ice at 0°C v/2 and v/3 respectively, the ratio of the maximum
to water at 0°C. Then the time T taken for the kinetic energy of electrons emitted from A to that
conversion from B is [Delhi 2020]

380 Physics–12
D:\EG_Physics-12_(26-06-2022)\Open_Files\Ch-11\Ch-11
\ 27-Jul-2022 Ved_Goswami Proof-4 Reader’s Sign _______________________ Date __________

II. Very Short Answer Type Questions (1 Mark)


(a) 2 : 3 (b) 3 : 4 (c) 1 : 3 (d) 3 : 2
2. Fill in the blanks. 1. Define the following terms: (i) Work function
(ii) Threshold frequency [Delhi 2019]
(i) One quantum of radiation of light is called a
2. What is meant by the term stopping potential?
..................... .
[Delhi 2019]
(ii) Energy of emw is carried by discrete packets or 3. Two metals A and B have work functions 4eV and
bundles of energy called ................. . 10eV respectively. Which metal has higher threshold
(iii) The minimum amount of energy required by an wavelength?
electron to just escape the metal surface is called 4. If the wavelength of an e.m. radiation is doubled,
its .......................... . what will happen to the energy of the photon?
3. State True or False 5. Is photoelectric emission possible at all frequencies?
(i) The stopping potential depends upon frequency Give reason for your answer.
of incident light and work function of metal. 6. Work function of aluminium is 4.2 eV. If two photons
each of energy 2.5 eV are incident on its surface, will
(ii) Light of frequency 1.5 times the threshold the emission of electrons take place? Justify your
frequency is incident on a photo sensitive material. answer.
If the frequency of incident light is halved and the
7. The stopping potential to an experiment on a photo
intensity is doubled, the photo current becomes
electric effect is1.5 V. What is the maximum kinetic
one.
energy of the photoelectrons emitted? [Delhi 2009]
4. Match the Columns 8. A source of light is placed at a distance of 1m from
Column-I Column-II a photocell and the cut off potential is found to be
(i) Saturation current (a) very high electric V0. If the distance is doubled what will be the cut off
related to field potential?[Delhi 2008]
(ii) Cold cathode emissions (b) Intensity 9. Plot a graph showing the variation of photoelectric
hc current with anode potential for two light beam of
(iii) Work function (c) same wavelength but different intensity.
λ0
Or
P Plot a graph of photocurrent versus anode potential
(iv) Number of photons (d)
E for a radiation of frequency n and intensities I1 and
emitted per seconds I2 (I1 < I2).
For Q. 5 and 6 there are two statements labelled as 10. Does the stopping potential in photoelectric emission
assertion (A) and reason (R). Select the correct answer depend upon the intensity of the incident radiation in
to these questions from the codes (a), (b), (c) and (d) as a photo cell? Comment on it. [Delhi 2005]
given below. 11. The figure shows a plot of three curves a, b, c showing
(a) Both A and R are true and R is the correct the variation of photo-current versus collector plate
explanation of A. potential for three different intensities I1, I2 and I3
(b) Both A and R are true but R is not the correct having frequency v1, v2 and v3 respectively. Incident
explanation of A. on a photosensitive surface. Point out the two curves
for which the incident radiation have same frequency
(c) A is true but R is false.
but different intensities. [Delhi 2009]
(d) Both A and R are false.
5. Assertion (A): Photoelectric effect demonstrates
the wave nature of light. I
I1
Photo
Reason (R): The number of photoelectrons is electric I2
I3
proportional to the frequency of light. current b

6. Assertion (A): Photosensitivity of a metal is high


if its work function is small.
c
Reason (R): Work function = hvo, where vo is a

threshold frequency. Collector plate potential

Dual Nature of Radiation and Matter 381


D:\EG_Physics-12_(26-06-2022)\Open_Files\Ch-11\Ch-11
\ 27-Jul-2022 Ved_Goswami Proof-4 Reader’s Sign _______________________ Date __________

12. Draw a plot showing the variation of photoelectric Mark the graph for the radiation of higher intensity.
current versus the intensity of incident radiation on [Delhi 2018]
a given photosensitive surface. [Delhi 2014] 18. Work function of Sodium is 2.75eV. What will be KE
13. The given graph shows the variation of photo-electric of emitted electron when photon of energy 3.54eV is
current (I) with the applied voltage (V) for two incident on the surface of sodium?
different materials and for two different intensities [CBSE S.P. 2019-20]
of the incident radiations. Identify and explain using 19. The variation of the stopping potential (V0) with the
Einstein’s photo electric equation the pair of curves frequency (v) of the light incident on two different
that correspond to (i) different materials but same photosensitive surfaces M1 and M2 is shown in the
intensity of incident radiation, (ii) different intensities figure. Identify the surface which has greater value
but same materials. [AI 2016] of the work function.[AI 2020]
V0
I
M1 M2

O v
2 3 20. Define the term ‘threshold frequency’ in photoelectric
4 emission.[AI 2020]
21. The threshold wavelength for two photosensitive
V
surfaces A and B are l1 and l2 respectively. What is
14. Using the graph shown in the figure for stopping the ratio of the work functions of the two surfaces?
potential V/s the incident frequency of photons.  [AI 2020]
Calculate Planck’s constant. [Delhi 2015]
22. In a photoelectric experiment, the potential required
1.8 to stop the ejection of electrons from cathode is 4V.
What is the value of maximum kinetic energy of
emitted photoelectrons?
V (Volts)

1.4
1.23  [CBSE S.P. 2020-21]
III. Short Answer Type Questions-I (2 Marks)
0.8 23. The work function of caesium is 2.14 eV. Find (i)
the threshold frequency for caesium and (ii) the
0.4
5 × 1014 Hz
wavelength of the incident light if the photocurrent
is brought to zero by a stopping potential of 0.60 V.
0 2 4 6 8 10 Given h = 6.63 × 10–34 Js. [NCERT]
v (1014 Hz) 24. Monochromatic light of frequency 6.0 × 1014 Hz is
15. Does the stopping potential in photoelectric emission produced by a laser. The power emitted is 2.0 × 10–3
depend upon W.
(i) the intensity of the incident radiation in a (i) What is the energy of a photon in the light beam?
photocell? [CBSE S.P. 2018-19]
(ii) the frequency of the incident radiation? (ii) How many photons per second, on the
16. Define intensity of radiation on the basis of photon average, are emitted by the source? Given
picture of light. Write its S.I. Unit. [AI 2014] h = 6.63 × 10–34 Js. [Delhi 2014]
Or 25. A monochromatic source, emitting light of wavelength,
600 nm, has a power output of 66W. Calculate the
Define the term “Intensity” in photon picture of
number of photons emitted by this source in 2
electromagnetic radiation. [Delhi 2019]
minutes. Use h = 6.6 × 10–34 Js. [S.P. 2013]
17. Draw graphs showing variation of photoelectric
26. Sketch a graph between frequency of incident
current with applied voltage for two incident
radiations and stopping potential for a given
radiations of equal frequency and different intensities.
photosensitive material. What information can

382 Physics–12
D:\EG_Physics-12_(26-06-2022)\Open_Files\Ch-11\Ch-11
\ 27-Jul-2022 Ved_Goswami Proof-4 Reader’s Sign _______________________ Date __________

be obtained from the value of the intercept on the 32. Using Einstein’s photoelectric equation show how the
potential axis? [Delhi 2010] cut-off voltage and threshold frequency for a given
27. Draw a plot showing the variation of photoelectric photosensitive material can be determined with the
current with collector plate potential for two different help of a suitable plot/graph. [Delhi 2012]
frequencies, v1 > v2 of incident radiation having 33. For a photosensitive surface, threshold wavelength is
the same intensity. In which case will the stopping l0. Does photoemission occur if the wavelength (l)
potential be higher? Justify your answer. of the incident radiation is (i) more than l0 (ii) less
[Delhi 2011] than l0? Justify your answer. [Delhi 2011 C]
28. Figure shows variation of stopping potential (V0) with 34. For photoelectric effect in sodium, figure shows the
frequency (v) for two photosensitive material M1 and plot of cut-off voltage versus frequency of incident
M2. radiation. Calculate :
(i) Why is the slope same for both lines? (i) the threshold frequency.
(ii) For which material will the emitted electrons have (ii) the work function for sodium.
greater kinetic energy for the incident radiations
of the same frequency? Justify your answer.
M1

Cut-off voltage
M2 3

(in volts)
2
V0
1

0
1 2 3 4 5 6 7 8 9 10
v Frequency (in Hz) × 1014
29. Two beams one of red light and other of blue light of 35. When light of frequency 2.4 × 1015 Hz, falls on a photo
the same intensity are incident on a metallic surface sensitive surface, the retarding potential needed to
to emit photoelectrons. Which one of the two beam completely stop the emitted photoelectrons is found
emits electrons of greater kinetic energy? to be 6.8 V. What is the work function (in eV) of the
30. By how much would the stopping potential for a given given photosensitive surface? [AI 2007 C]
photosensitive surface go up if the frequency of the 36. There are two sources of light, each emitting with a
incident radiation were to be increased from 4 × 1015 power 100W. One emits X-rays of wavelength, 1 nm
Hz to 8 × 1015 Hz? Given h = 6.4 × 10–34 Js, e = 1.6 and the other visible light at 500 nm. Find the ratio of
× 10–19 C. [S.P. 2008) number of photons of X-rays to the photon of visible
31. The graph of figure shows the variation of photoelectric light of the given wavelength. [NCERT Exemplar]
current with collector plate potential for different 37. A monochromatic light source of power 5mW
frequencies of incident radiations. emits 8 × 1015 photons per second. This light eject
photoelectrons from a metal surface. The stopping
(i) Which physical parameter is kept constant for the
potential for this set up is 2V. Calculate the work
three curves?
function of the metal. [S.P. 2016]
(ii) Which frequency (v1, v2 or v3) is the highest?
38. If light of wavelength 412.5 nm is incident on each
[Delhi 2009]
of the metals given below, which ones will show
photoelectric emission and why? [Delhi 2018]
Photo Metal Work Function (eV)
electric
current
Na 1.92
K 2.15
Ca 3.20
Mo 4.17
v1
v2 39. Why is wave theory of electromagnetic radiation not
v3
able to explain photo electric effect? How does photon
Collector plate potential picture resolve this problem? [Delhi 2019]

Dual Nature of Radiation and Matter 383


D:\EG_Physics-12_(26-06-2022)\Open_Files\Ch-11\Ch-11
\ 27-Jul-2022 Ved_Goswami Proof-4 Reader’s Sign _______________________ Date __________

40. The work function of Cs is 2.14 eV. Find photosensitive material, in terms of the parameter l,
(i) threshold frequency for Cs n and the mass m of the electron. [S.P. 2013]
(ii) Wavelength of incident light if the photo current
is brought to zero by stopping potential of 0.6 V. 2
V max
[CBSE S.P. 2019-20]
41. Figure shows the stopping potential (V0) for the photo
electron versus (1/l) graph, for two metals A and B, n
l being the wavelength of incident light. θ Frequency (v)
(i) How is the value of Planck’s constant determined l
from the graph?
C
V0 A B

45. The threshold frequency of a metal is f0. When the


light of frequency 2f0 is incident the metal plate, the
O maximum velocity of electron emitted is V1. When
1/l
the frequency of the incident radiation is increased
(ii) If the distance between the light source and the to 5f0, the maximum velocity of electrons emitted is
surface of metal A is increased, how will the V2. Find the ratio of V1 and V2.
stopping potential for the electrons emitted from 46. In a plot of photoelectric current versus anode
it be effected? Justify your answer. [Delhi 2020] potential, how does:
42. Write two main observations of photo electric (i) The saturation current vary with anode potential
effect experiment which could only be explained by for incident radiations of different frequency but
Einstein's photo electric equation. same intensity?
IV. Short Answer Type Questions-II (3 Marks) (ii) The stopping potential vary for incident radiations
43. Figure shows the variation of stopping potential V0 of different intensities but same frequency?
with the frequency v of the incident radiation for two (iii) Photoelectric current vary for different intensities
photosensitive metals P and Q: but same frequency of incident radiations? Justify
(i) Explain which metal has smaller threshold your answer is each case.
wavelength. 47. Radiations of frequency v1 and v2 are made to fall
(ii) Explain giving reason, which metal emits in turn, on a photosensitive surface. The stopping
photoelectrons having smaller kinetic energy, for potentials required for stopping the most energetic
the same wavelength of incident radiation. photoelectrons in the two cases are V 1 and V 2
(iii) If the distance between the light source and metal respectively. Obtain a formula for determining the
P is doubled, how will the stopping potential threshold frequency in terms of these parameters.
change.[Delhi 2008] 48. Write Einstein’s photoelectric equation. State clearly
P Q the three salient features observed in photoelectric
effect, which can be explained on the basis of the
above equation. [Delhi 2010]
V0
Or
Write two characteristic feature observed in
photoelectric effect, which support the photon picture
of electromagnetic radiation. [Delhi 2012]
v
49. State how in a photo cell, the work function of the
44. When a given photosensitive material is irradiated metal influence the kinetic energy of the emitted
with light of frequency v, the maximum speed of electrons.
the emitted photo-electrons equals Vmax. The square (i) If the intensity of the incident radiation is doubled,
of Vmax is observed to vary with n, as per the graph what changes occur in (a) the stopping potential
shown in figure. obtain expression for (i) Planck’s and (b) in photoelectric current?
constant and (ii) the work function the given
384 Physics–12
D:\EG_Physics-12_(26-06-2022)\Open_Files\Ch-11\Ch-11
\ 27-Jul-2022 Ved_Goswami Proof-4 Reader’s Sign _______________________ Date __________

(ii) If the frequency of the incident radiation is (ii) Radiation of frequency 1015 Hz is incident on
doubled, what changes occur in (a) the stopping two photosensitive surface P and Q. There is no
potential and (b) photoelectric current. photoemission from surface P. Photoemission
50. Green light ejects photoelectrons from a given occurs from surface Q but photoelectrons have
photosensitive surface where as yellow light does zero kinetic energy. Explain these observations
not. What will happen in case of violet and red light? and find the value of work function for surface
Give reason for your answer. Q.[Delhi 2017]
51. Write the basic features of photon pictures of 57. (i) In photoelectric effect, do all the electrons that
electromagnetic radiation on which Einstein’s absorb a photon come out as photoelectrons
photoelectric equation is based. [Delhi 2013] irrespective of their location? Explain.
52. State laws of photo-electric emission. (ii) A source of light, of frequency greater than the
53. Sketch the graphs, showing the variation of threshold frequency, is placed at a distance ‘d’
stopping potential V S with frequency v of the from the cathode of a photocell. The stopping
incident radiations for two photosensitive potential is found to be V. If the distance of the
materials A and B having threshold frequencies light source is reduced to d/n (where n > 1),
v0 > v′0 respectively. explain the changes that are likely to be observed
(i) Which of the two metals A or B has higher work in the (i) photoelectric current and (ii) stopping
function? potential.  [CBSE S.P. 2018-19]
(ii) What information do you get from the slope of 58. Plot a graph showing the variation of photo current
the graphs? vs collector potential for three different intensities
(iii) What does the value of the intercept of graph A I1 > I2 > I3, two of which (I1 and I2) have the same
on the potential axis represent? [Delhi 2010] frequency v and the third has frequency v1 > v.
54. Light of intensity ‘I’ and frequency ‘v’ is incident Explain the nature of the curves on the basis of
on a photosensitive surface and causes photoelectric Einstein’s equation. [AI 2016]
emission. What will be the effect on anode current 59. State the main implications of observations obtained
when (i) the intensity of light is gradually increased, from various photoelectric experiments. Can these
(ii) the frequency of incident radiation is increased implications be explained by wave nature of light?
and (iii) the anode potential is increased? In each case, Justify your answer.  [CBSE S.P. 2020-21]
all other factors remain the same. Explain, giving
justification in each case. [AI 2015] V. Long Answer Type Questions (5 Marks)
55. (i) How does one explain the emission of electrons 60. (i) Describe briefly the experimentally observed
from a photosensitive surface with the help of features in the phenomenon of photo-electric
Einstein’s photoelectric equation? effect.
(ii) The work function of the following metals is (ii) Why photoelectric effect cannot be explained on
given: Na = 2.75 eV, K = 2.3 eV, Mo = 4.17 eV the basis of wave nature of light?
and Ni = 5.15 eV. Which of these metals will [Delhi 2013, AI 2015]
not cause photoelectric emission for radiation of 61. The maximum kinetic energy of the photoelectrons
wavelength 3300 Å from a laser source? gets doubled when the wavelength of light incident
[Delhi 2017] on the surface changes from l1 to l2. Derive the
56. (i) State two important features of Einstein’s expressions for the threshold wavelength l0 and work
photoelectric equation. function for the metal surface. [AI 2020]

Answers 11.1
I. Objective Type Questions II. Very Short Answer Type Questions
1. (i) (c) (ii) (b) (iii) (c) 1. (i) Work function: It is equal to the minimum amount
(iv) (a), (b) and (c) (v) (b) (vi) (c) of energy required for an electron to just escape from
(vii) (b) the surface of metal.
2. (i) Photon (ii) quantum (iii) work function (ii) Threshold frequency: It is equal to the minimum
3. (i) True (ii) False value of the frequency of the incident radiations
required for photoelectrons to be just emitted.
4. (i)-(b) (ii)-(a) (iii)-(c) (iv)-(d) 5. (d) 6. (b)
Dual Nature of Radiation and Matter 385
D:\EG_Physics-12_(26-06-2022)\Open_Files\Ch-11\Ch-11
\ 27-Jul-2022 Ved_Goswami Proof-4 Reader’s Sign _______________________ Date __________

Relation between work function f0 and threshold 11. Since the value of stopping potential is same for curves
frequency vo is f0 = hvo a and b, hence frequency v1 and v2 are the same but their
h is planck’s constant. intensities are different.
2. Stopping potential: It is the minimum negative 12.
potential given to the anode in a photocell for which A
the photoelectric current becomes zero. Photo
electric
If Vo is stopping potential, then the maximum kinetic current
energy of the emitted photo electrons is (KE)max. = eV0
hc hc
3. As work function φo = hvo = or λ o =
λo φo
So higher the work function, lesser is threshold O Intensity
wavelength.
13. (i) (1, 3) and (2, 4) corresponds to different materials
So metal A with fo = 4eV will have higher threshold but same intensity.
wavelength.
(ii) (1, 2) and (3, 4) corresponds to different intensities
hc but same materials.
4. As energy of a photon E = hv =
λ 14. According to photoelectric equation
If l is doubled, energy of the photon will become half
eV0 = hv – hv0,
of its earlier value.
5. No, photoelectric emission is possible only if the h h
V0 = v − v0
frequency of the incident radiation is greater than the
e e
work function of the metal. h 1.23 − 0
6. No photoelectric emission will take place. As the Slope of the line = =
e (8 − 5) × 1014
energy of a single photon must be greater than the work
function of the metal for the emission of photoelectrons. 1.23
So h= × 1.6 × 10 −19
7. Max. K.E. = Kmax = eV0 = = e × 1.5V = 1.6 × 10–19 × 3 × 1014
1.5 J = 2.4 × 10–19 J −34
8. V0; When distance is doubled, the intensity of incident = 6.56 × 10 Js
light becomes one fourth  I ∝ 1  15. (i) No, the stopping potential does not depend upon the
  intensity of the incident radiation.
r2 
but the frequency of incident light remains unchanged. (ii) Yes, the stopping potential depends on the frequency
of the incident radiation (if frequency is more than
The value of cut off potential of a material only
threshold frequency).
depends upon the frequency of the incident light and
16. Intensity of radiation on the basis of photon picture of
is independent of the intensity of incident light.
light:
9. On the basis of photon picture of light, the intensity of
radiation depends upon the number of photons falling
Photo on the surface per second per unit area.
electric Intensity I1
current Intensity I2
Definition: It is equal to the energy of all the photons
(I) incident normally per unit area per units time
It’s SI unit is Js–1 m–2
17. I2 > I1

Anode potential
As wavelength is same so the stopping potential will
be same.
10. No, the stopping potential in photoelectric emission does
not depend upon the intensity of the incident radiation.
It depends upon the nature of the photosensitive surface
and the frequency of the incident radiation.
386 Physics–12
D:\EG_Physics-12_(26-06-2022)\Open_Files\Ch-11\Ch-11
\ 27-Jul-2022 Ved_Goswami Proof-4 Reader’s Sign _______________________ Date __________

18. 0.79 eV. hc (6.6 × 10 −34 ) × (3 × 108 )


19. Photosensitive surface M2 has greater value of work E= λ =
6 × 10 −7
function (W) due to higher value of threshold frequency
(n) as W = hn0. = 3.3 × 10–19 J ½
Energy emitted by source in time t,
20. Threshold frequency is defined as the minimum
frequency of incident light which can cause photoelectric ES = Pt = 66 × 2 × 60 J ½
emission i.e. this frequency is just able to eject electrons \ No. of photons emitted by the source in 2 minutes
without giving them additional energy. ES Pt 66 × 2 × 60
N= = =
hc E E 3.3 × 10 −19
21. Work function of surface A, fA = …(1)
λ1 = 2.4 × 1022 photons. 1
hc 26. As hv = (KE)max + hv0, (KE)max = hv – hv0,
Work function of surface B, fB = …(2) eV0 = hv – hv0
λ2
h h
φA hc / λ1 λ 2 V0 = v − v0  1
on dividing (1) by (2), we get = = e e
φB hc / λ 2 λ1
22. Given V = 4V, Kmax = eV
\ Kmax = e × 4V = 4eV
V0
23. f0 = 2.14 eV = 2.14 × 1.6 × 10–19 J
φ    ½
(i) Threshold freq. v0 = 0  ½
h
2.14 × 1.6 × 10 −19
= = 5.16 × 1014 Hz ½ O v0 Frequency in Hz
6.63 × 10 −34
V0 is stopping potential
hc − hv0
(ii) hv = f0 + (KE)max or = f0 + eV0 ½ Intercept on the potential axis =  ½
λ e
hc It helps us to find the work function of the metal.
l= 27.
eV0 + φ0 I

6.63 × 10 −34 × 3 × 108


  =
(1.6 × 10 −19
× 0.6 + 2.14 + 1.6 × 10 −19 ) v1 > v2
v2
v1 Saturation
19.89 × 10 −26 current
= −19
= 454 × 10 −9 m
2.74 × 1.6 × 10
V 1
= 454 nm ½ – Vo1 – Vo2
24. (i) Energy carried by a photon = E = hv Stopping potential will be higher for v 1 (higher
E = 6.63 10–34 × 6.0 × 1014 ½ frequency)
= 3.98 × 10–19 J  ½ As eV0 = hv – hv0  1
[CBSE Marking Scheme, 2018-19] 28. (i) From Einstein’s photoelectric equation
(ii) No. of photons emitted per second h φ0
eV0 = hv – f0 or V0 = e v − e  ½
P 2.0 × 10 −3
n= = The slope of stopping potential vs frequency graph is
E 3.98 × 10 −19 J
h
= 5.0 ×1015 photons per second ½ a constant equal to h , and in same for all metals.
e e
25. Here, l = 600 nm = 6 × 10–7 m; P = 66 W,
So the slope is same for both lines. ½
    N = ?, T = 2 × 60s
(ii) As (K.E.)max = hv – hv0 ½
Energy of each photon,

Dual Nature of Radiation and Matter 387


D:\EG_Physics-12_(26-06-2022)\Open_Files\Ch-11\Ch-11
\ 27-Jul-2022 Ved_Goswami Proof-4 Reader’s Sign _______________________ Date __________

For metal M1, the threshold frequency v0 is lesser. From the graph threshold frequency v0 = OB Also in
So the (K.E.)max is larger for metal M1 so metal M1 magnitude
will emit photoelectrons of higher K.E. ½ φ0
29. lb < lR so vb > vR OA = or f0 = e × OA ½
e
Energy of blue light (hv)blue is greater than energy of 33. Maximum kinetic energy of photoelectrons is given by
red light (hv)red. 1 (KE)max = hv – hv0, ½
In photoelectric emission, max K.E. of emitted v0 → threshold freq.
electron = hv – f0, 1
hc hc  λ − λ
So, K.E. of emitted electrons is more with blue light = − = hc  0  ½
than that of red light. λ λ0  λλ 0 
30. n1 = 4 × 1015 Hz, n2 = 8 × 1015 Hz (i) If l > l0, Then (KE)max is negative which is not
eV0 = hv – f0 possible so no photoelectric emission will take place.
eVo1 = hv1 – f0  ½
(ii) If l < l0, Then (KE)max is positive. So photoelectric
eVo2 = hv2 –f0
emission will take place. ½
e(Vo2 –Vo1) = h(v2 – v1) 1
34. From the graph
h
or Vo 2 − Vo1 = (v2 − v1 ) (i) Threshold frequency v0 = 4.5 × 1014 Hz 1
e
(ii) Work function f0 = hv0
6.4 ×10 −34 = 6.6 × 10–34 × 4.5 × 1014
= −19
(8 × 1015 − 4 × 1015 )
1.6 × 10 = 2.97 × 10–19 J 1
6.4 ×10 −34 × 4 × 1015 35. v = 2.4 × 10 Hz, V0 = 6.8V
15
= = 16 V  1
1.6 × 10 −19 As hv = (KE)max + f0, (KE)max = eV0
31. (i) As the saturation current is same for all the three f0 = hv – eV0
curves. So the intensity of incident radiation is kept   = (6.6 × 10–34 × 2.4 × 1015 – 1.6 × 10–19 × 6.8) J 1
constant. 1
 6.6 × 10−34 × 2.4 × 1015 1.6 × 10−19 × 6.8 
(ii) As stopping potential is maximum for radiation of   =  −  eV
frequency v1, so v1 is the highest frequency. 1  1.6 × 10−19 1.6 × 10−19 
32. According to Einstein’s equation of photoelectric   = (9.9 – 6.8) eV = 3.1 eV 1
emission 36. P1 = P2 = 100 W, l1 = 1 nm = 1 × 10 m,
–9

KE)max = hv – f0, f0 is work function n1


or  eV0 = hv – f0, V0 is cut-off voltage l2 = 500 nm = 500 × 10–9 m, =?
n2
hv φ0 As P1 = P2 So n1E1 = n2E2 1
V0 =
−  1
e e hc hc n1 λ1 1
If we plot a graph between v and V0, it will be a straight    n1 = n2 or = =  1
λ1 λ2 n2 λ 2 500
h − φ0
line with slope = and intercept on y-axis = 37. P = 5 mW = 5 × 10–3 W
e e
   n = 8 × 1015 Photon/sec
V0 = 2V
C
V0 P 5 × 10 −3
n= ,E= = 6.25 × 10 −19 J  1
E 8 × 1015
6.25 × 10 −19
B E= = 3.9 eV  ½
v0 v    ½ 1.6 × 10 −19
O
f0 = E – eV0 = (3.9 – 2) eV = 1.9 eV ½
38. We know
A
hc
E=
λ

388 Physics–12
D:\EG_Physics-12_(26-06-2022)\Open_Files\Ch-11\Ch-11
\ 27-Jul-2022 Ved_Goswami Proof-4 Reader’s Sign _______________________ Date __________

Given l = 412.5 nm (ii) As Kmax= eV0 = 0.6eV


    = 412.5 × 10–9 m Energy of photon
   c = 3 × 108 m/s E = Kmax+ W = 0.6 eV + 2.14 eV
   h = 6.67 × 10–34 Js = 2.74eV
6.67 × 10 × 3 × 10 −34 8 Wavelength of photon

\ E=
412.5 × 10 −9 λ=
hc 6.62 × 10 −34 × 3 × 108
=
or E = 0.048 × 10–17 J 1 E 2.74 × ×1.6 × 10 −19
1 eV = 1.6 × 10–19 J = 4530Å 1
1eV 41. (i) By Einstein’s theory of Photoelectric Effect:
\ 1J =
1.6 × 10 –19 hc
0.048 × 10 –17 eV0 = – W0
\ E = eV λ
1.6 × 10 –19
or ½ hc  1  W0
E = 3eV  or V0 =  − ...(1)
e λ e
Energy of incident light wave is 3eV. So photo electric
emission will be exhibited by those metals for which where, W0 = work function of metal surface.
work function is lesser than this energy. So here only on comparing (1) with y = mx + c,
Na and K will exhibit- photo electric emission. ½ hc
we get, m =
39. There are two main aspects of experimental results e
which cannot be explained by wave theory. If light Equ. (1) shows the equation of a straight line as
is wave then its energy increases as we increase the shown in the figure here.
intensity of the wave, but this increases only the number So, slope (m) of the line
of electrons emitted but does not increase the energy of hc
= tan q =
the electron. Secondly if electrons absorbs the energy of e
the wave, the emission of the electron must be delayed
with respect to the instance light falls on the substance, e tan θ
or h= ...(2)
but it is immediate. ½ c
Lastly, photon theory of light is just not what you state, Planck’s constant (h) can be determined by putting
the values of the slope of the graph, speed of light (=
“And a ‘photon’ is simply a measure of energy of an
3 × 108 m/s) and the electronic charge (e) in equation
electromagnetic wave with a certain wavelength or (2).
frequency.” the model is completely different. ½
V0 A B
In a wave model electron has to oscillate as per the
oscillating electric field and gain enough energy and
Stopping
potential

then get out. ½


In a photon model which is not just another particle q q
model but it is a quantum model emphasizing duality O 1/l
at the fundamental level. According to it light consists (ii) Stopping potential (V0) only depends on frequency
of particles each with energy ‘hf ’. [Note that energy of incident light. So if the distance is increased the
is particle aspect and frequency is wave aspect. This stopping potential will not change.
particle meets the electron and itself gets vanished, 42. (i) Instantaneous process i.e. no time lagging.
giving all its energy in one shot process to the electron]. (ii) Saturation current ∝ Intensity of incident radiation.
 ½ 43. (i) From the graph we see that threshold frequency is
40.Work function W = 2.14 eV less for metal P.
(i)Threshold frequency W = hν0 hc
f0 = hv0 =  ½
W 2.14 × 1.6 × 10 −19 λ0
ν0 = =
h 6.62 × 10 −34 For metal Q, v0 is large so l0 the threshold wavelength
= 5.17 × 1014 Hz 1 is smaller. ½

Dual Nature of Radiation and Matter 389


D:\EG_Physics-12_(26-06-2022)\Open_Files\Ch-11\Ch-11
\ 27-Jul-2022 Ved_Goswami Proof-4 Reader’s Sign _______________________ Date __________

(ii) As energy of incident radiation 46. (i) The saturation current for a given metal depends on
hv = (KE)max + f0 the intensity of the incident radiation and not on its
hc hc frequency. So the saturation current will be same for
= (KE ) max +
    λ λ0  ½ incident radiations of same intensity but different
frequency. 1
hc hc
(KE ) max = − (ii) The stopping potential for a given metal depends on
    λ λ0
As voP < voQ the frequency of the incident radiation but not on its
hc hc intensity. ½
So λ oP > λ oQ , > So the value of stopping potential will be same
λ oQ λ o P
for radiations of same frequency but of different
So (KE)max is smaller for metal Q. ½
intensity. ½
(iii) If the distance between the light source and metal
(iii) As the intensity of incident radiation is changed, the
P is doubled, the intensity of radiations falling on
number of photons falling on the metal will change
1
the metal will become one-fourth as I ∝ 2 , but the thus changing the number of photo-electron emitted
r (if v > v0). ½
frequency will remain same. ½
As stopping potential is independent of intensity of So as the intensity of incident radiations in changed
incident radiations so stopping potential for metal keeping same frequency then photoelectric current
P will remain unchanged. ½ will also change. ½
44. According to Einstein’s photoelectric equation 47. If v0 is the threshold frequency, then from photoelectric
1 equation, we have
(KE ) max = mVmax 2
= hv − φ0
2 eV1 = hv1 – f0 and eV2 =hv2 – f0
 2h  2φ 0 \ e(V2 – V1) = h(v2 – v1)
=   v −
2
or Vmax  ½
m m e (V2 − V1 )
or h =  1
Thus the graph between V2max and v will be a straight (v2 − v1 )
2h l
line. Slope of st. line = =  ½ Now eV1 = hv1 – f0 = hv1– hv0
m n
eV1  v −v 
Intercept of V 2 axis, 2φ0 = l [In magnitude] Or v0 = v1 − = v1 − eV1  2 1   1
max
m h  e(V2 − V1 ) 
ml V1 (v2 − v1 )
(i) \ Planck’s constant, h =  1 = v1 −
2n (V2 − V1 )
(ii) Work function, f0 = ml  1 ν1V2 − ν1V1 − ν2 V1 + ν1V1
2 =
45. As f0 is threshold frequency (V2 − V1 )
So f0 = hf0 ½ ( ν1V2 − ν2 V1 )
=  1
According to Einstein’s photo electric equation (V2 − V1 )
1 2 48. Let v be the frequency of the incident radiation and f0
hv = φ0 + mVmax  ½
2 is the work function of the metal.
1 According to law of conservation of energy
So, when v = 2f0 then mV12
2 Energy of incident radiation
= h × 2f0 – f0 = 2hf0 – hf0 = hf0 1 = Work function + (KE)max of photo-electron
When v = 5f0 1 2 1 2
hv = φ0 + mVmax = hv0 + mVmax
1 2 2
Then mV22 = h × 5 f 0 − hf 0 = 4hf 0  ½
2 The salient features observed in photoelectric effect:
1 (i) If v < v0, then (KE)max becomes negative. Negative
mV12 hf 0
1
2 V1  1  2 1 K.E. has no physical meaning. So no photo electric
   = or =  =  ½
1 emission is possible below the threshold frequency
mV2 4hf 0
2 V2  4  2
2 v0. 1

390 Physics–12
D:\EG_Physics-12_(26-06-2022)\Open_Files\Ch-11\Ch-11
\ 27-Jul-2022 Ved_Goswami Proof-4 Reader’s Sign _______________________ Date __________

(ii) As (KE)max = hv – hv0 = h(v – v0) 3. For radiations of a given frequency of wavelength,
Above threshold frequency, Max. K.E. of the increase of photons falling per second on the surface,
photoelectrons depends linearly on the frequency but the energy of each photon is same i.e., hv. This
of incident radiations. 1 means the energy of photon is independent of the
(iii) The maximum K.E. of photoelectrons is independent intensity of radiations. ½
of the intensity of incident radiations. 1 4. Velocity of photons is different in different media
49. As (KE)max = hv – f0 which is due to change in its wavelength. ½
Where f0 is work function of the metal. 5. In a photon-particle collision, the energy and
momentum are conserved. ½
Higher is the value of work function, lesser will be the
52. 1. The Photoelectric emission is an instantaneous
kinetic energy of emitted electrons for a given frequency
process. ½
of incident radiations.
2. For a given frequency of incident radiation and a
(i) If intensity of the incident radiation is doubled
given metal, the number of photoelectrons emitted
(a) there will be no change in stopping potential
per second is directly proportional to the intensity
as stopping potential depends on frequency not
of incident radiations. ½
intensity of incident radiation. (b) Photoelectric
current will get doubled. 1 3. For a given metal, there is a minimum frequency
(ii) If the frequency of the incident radiation is doubled of incident radiation below which emission of
(a) then the stopping potential becomes more than photo electrons does not take place. That minimum
double ½ frequency of incident radiation below which no
As  eV0 = hv – f0, hv = eV0 + f0 photo electrons are emitted is called threshold
eV′0 = h2v – f0 = 2hv – f0 frequency. 1
= 2(eV0 + f0) – f0 4. Above threshold frequency, the maximum kinetic
energy of emitted photoelectrons is independent of
eV′0 = 2eV0 + f0
the intensity of incident radiations but depends only
φ0 upon the frequency of the incident radiations. 1
V′0 = v10 = 2V0 +
 1
e 53. The variation of stopping potential (VS) with frequency
(b) Photoelectric current will remain same. ½ (v) of the incident radiations for two photosensitive
50. According to Einstein’s photo electric eqn. materials is shown in figure.
(KE)max = hv – hv0 ½
B
As green light ejects photo-electron so the frequency VS
A
of green light is more than threshold frequency. ½
vViolet > vGreen
So vViolet > v0 thus photoelectrons will be emitted. 1
v′0
As no photo electrons are emitted for yellow colour so v0 v
vYellow < v0 ½ – φ′0
vRed < vYellow e
1
– φ0
So vRed < v0 so red colour will not eject e
photoelectrons. ½
51. Basic features of photon pictures of e.m. radiation on
which Einstein’s photo-electric emission in based:
(i) As v0 < v′0 and f0 = hv0, f′0 = hv′0
1. Each photon carries energy E = hv = hc and So f0 > f′0 i.e., metal A has higher work function½
λ h
hv h (ii) Slope of the graphs gives  ½
momentum p = =  ½ e
c λ (iii) Intercept of graph ‘A’ on the potential axis represents
where h is Planck’s constant, v is frequency, l is − φ0 − hv0
wavelength of radiation and c is speed of light. ½ =  1
e e
2. All photons of a particular frequency v have same 54. (i) Anode current will increase with increase in intensity
energy E = hv, irrespective of the intensity of of the incident light. ½
radiation. ½

Dual Nature of Radiation and Matter 391


D:\EG_Physics-12_(26-06-2022)\Open_Files\Ch-11\Ch-11
\ 27-Jul-2022 Ved_Goswami Proof-4 Reader’s Sign _______________________ Date __________

Reason: Increase in intensity means more number come out of the surface of the metal for whom the
of photons are striking the metal per second so more incident energy is greater than the work function of
number of electrons will be emitted,there by increase the metal. 1
the anode current. ½ (b) on reducing the distance, intensity increases. ½
(ii) There will be no change in anode current with Photoelectric current increases with the increase
increase is frequency of incident radiation. ½ in intensity. Stopping potential is independent of
Reason: No. of electrons emitted per second intensity, and therefore remains unchanged. ½
depends upon the intensity, not on frequency of [CBSE Marking Scheme 2018-19]
incident radiation. ½
58.
(iii) With increase in anode potential the anode current
will increase initially (for v > threshold frequency) Photo
and attain saturation later on. 1 current I1
55. (i) According to Einstein’s photoelectric equation I2
hn = f0 + (KE)max  ½ I3
So when a photon of frequency n strikes a metal 1

surface. Some of its energy i.e. f0 is used up in
removing electron and rest of it is carried by it in v1
v v
the form of kinetic energy. 1
Collector potential
hc −34 8
(ii) E = hn = = 6.6 × 10 × 3 × 10
λ 3300 × 10 −10 × 1.6 × 10 −19 Explanation: As per Einstein’s equation:
= 3.77eV ½ (i) Stopping potential is same for I1 and I2 as stopping
work functions of Mo and Ni are more than 3.77 eV. potential depends upon the frequency of the incident
So this incident photon will not cause photoelectric photon and not the intensity. More is the intensity of
emission in Mo and Ni. 1 incident radiation more will be the saturation current.
56. (i) According to Einstein’s photoelectric equation  1
(KE)max = hn – hn0 ½ As v1 > v, so cut off voltage will be more for
(a) If frequency of incident photon n is less than the frequency v1 because more in the energy of the
threshold freq. n0, no emission of photo-electrons incident photon more will be (KE)max or stopping
is possible because for n < n0 (KE)max is negative,
potential for photoelectrons. 1
which is not possible ½
59. Main implications of observation obtained from
(b) With increase in frequency of incident radiation
there will be increase in (KE) of photoelectrons but various photoelectric experiments:
no change in photo-electric current. ½ (i) The number of photo-electrons emitted per second
(ii) n = 1015 Hz is directly proportional to the intensity of incident
As there in no photoemission from surface P, so radiation.
threshold freq. of P is more than 1015 Hz (ii) For a given frequency of the incident radiation, the
For metal Q, (KE) = 0 ½ stopping potential (or the maximum kinetic energy)
So hn = f0 + KE is independent of its intensity.
\ hn = f0 (iii) The maximum kinetic energy of the photo-electrons
Work function of Q = hn varies linearly with the frequency of incident
radiation, but is independent of its intensity.
= 6.6 × 10–34 × 1015 = 6.6 × 10–19 J ½
(iv) for a frequency v of incident radiation, lower than
6.6 × 10 −19
= = 4.13 eV ½ the cut-off frequency v0, no photoelectric emission
1.6 × 10 −19 is possible even if the intensity is large.
57. (a) No, it is not necessary that if the energy supplied to
an electron is more than the work function, it will (v) The photoelectric emission is an instan taneous
process without any apparent time lag (~10–9s or less).
come out. 1
Wave nature of light emphasizes that the greater the
The electron after receiving energy, may lose energy intensity of radiation, the greater are the amplitude
to the metal due to collisions with the atoms of the of electric and magnetic fields, consequently, greater
metal. Therefore, most electrons get scattered into should be the energy absorbed by each electron and
the metal. Only a few electrons near the surface may
392 Physics–12
D:\EG_Physics-12_(26-06-2022)\Open_Files\Ch-11\Ch-11
\ 27-Jul-2022 Ved_Goswami Proof-4 Reader’s Sign _______________________ Date __________

hence kinetic energy is expected to increase. Also, Reason: Greater the intensity of incident radiation,
threshold frequency should not exist as wave picture greater the amplitude of electric and magnetic field.
says in terms beam should be able to impact enough Hence higher intensity should liberate photoelectrons
energy to electrons to escape from metal surface. All with greater kinetic energy. According to wave
the above expectation of wave theory contradicts theory the energy of photo electrons should depend
observation (ii) and (iii).
upon the intensity of incident radiations. 1
60. (i) 1. The photoelectric emission is an instantaneous
3. There exists a minimum frequency of incident
process.
radiations below which no photoelectric emission
2. No emission of photo electrons is possible if the is possible.
frequency of incident radiation is less than a certain
Reason: According to wave theory, whatever be
frequency called threshold frequency however high
the intensity of incident. the frequency of incident radiations, a light wave of
sufficient intensity should be able to give the energy
3. The photoelectric current become zero at a certain
required to eject electrons from the metal surface.1
value of negative potential called stopping potential
applied at the anode. 1 61. As hv = hv0 + (KE)maxLet (KE)max = EK
4. The value of the stopping potential increases with hc hc
For l1, = + E K ...(1) 1
the increase in the frequency of the incident radiation λ1 λ0
and the maximum kinetic energy of photoelectrons
in directly proportional to the frequency of incident hc hc
For l2, = + 2E K ...(2) 1
radiation. λ2 λ0
5. The maximum kinetic energy of photo-electron is Multiply eqn. (1) by 2,
independent of the intensity of incident radiations. 2hc 2hc
6. The photoelectric current is directly proportional to = + 2E K ...(3) 1
λ1 λ0
the intensity of incident radiations. 1
Subtract eqn. (2) from eqn. (3)
(ii) Wave nature of light could not explain:
1. That photo electric emission is an instantaneous 2hc hc hc 2 1 1
− = or − =
process. λ1 λ2 λ0 λ1 λ 2 λ 0
Reason: According to wave nature of light, energy λ1λ 2
of light wave is uniformly distributed across its or l0 =  1
wavefront. Each electron gets a very small amount 2λ 2 − λ1
of this energy and so it should require a finite time Work function
to escape the metal surface. 1 hc hc (2λ 2 − λ1 )
f0 = =  1
2. That maximum K.E. of photoelectrons is independent λ0 λ1λ 2
of the intensity of incident radiations.

Topic 2. Wave Nature of Matter


•• Dual Nature of Matter/de Broglie Hypothesis: As If v = 0 (particle is at rest)
radiation has dual nature i.e., wave as well as particle as
l=∞
energy and mass are interconvertible according to the
relation E = mc2 so de Broglie put the hypothesis that This implies that waves are associated with material
the material particle in motion should display wave like particles only when they are in motion.
properties because nature loves symmetry. To be associated with matter waves, the particle need not
•• de Broglie Wave and Wavelength: The wave associated have a charge.
with material particles in motion are called matter waves
de Broglie waves are not electromagnetic in nature
or de Broglie Waves. Their wavelength is called de
because EM waves are associated with accelerated
Broglie wavelength.
charged particles.
h h
l= =
p mv

Dual Nature of Radiation and Matter 393


D:\EG_Physics-12_(26-06-2022)\Open_Files\Ch-11\Ch-11
\ 27-Jul-2022 Ved_Goswami Proof-4 Reader’s Sign _______________________ Date __________

EXERCISE 11.2
I. Objective Type Questions (1 Mark) 2. Fill in the blanks.
1. Choose the correct answers from the given options (i) Out of proton, neutron, b-particle and a-particle
(MCQs). ................ will have the maximum de-Broglie
(i) The de-Broglie wavelength of an electron in the wavelength.
first Bohr orbit is (ii) If a photon and an electron are considered to be
(a) equal to the circumference of the first orbit. of same de-Broglie wavelength, then the velocity
(b) equal to twice the circumference of the first
of photon is .................. .
orbit.
(c) equal to half the circumference of the first (iii) An electron of mass m and charge e is accelerated
orbit. from rest through a potential difference of V in
(d) equal to one-fourth the circumference of two vacuum. Its final velocity will be ................. .
first orbit. (iv) When a proton is accelerated through a potential
(ii) If the kinetic energy of a free electron doubles, its difference of one volt, the kinetic energy gained
de-Broglie wavelength changes by the factor by it is roughly equal to ................... .
1 1 (v) An electron is accelerated through a potential
(a) 2 (b) (c) 2 (d)
2 2 difference of 100 V, then de-Broglie wavelength
(iii) A particle is dropped from a height H. The de- associated with it is approximately ....................
Broglie wavelength of the particle as a function Å. [S.P. 2019-20]
of height is proportional to 3. State True or False
(a) H (b) H1/2 (c) H0 (d) H–1/2 (i) An electron, photon, a neutron are accelerated
(iv) Relativistic corrections become necessary through the same potential difference. The kinetic
when the expression for the kinetic energy energies acquired by them will be in the ratio 1 :
1 2 2 : 4.
mv becomes comparable with mc2, where
2 (ii) An electron is accelerated through a potential
m is the mass of the particle. At what de Broglie differences of 100 V. The wavelength associated
wavelength, will relativistic corrections becomes with it is 1.05 Å.
important for an electron?
(iii) An electron is accelerated through a potential
(a) l = 10 nm (b) l = 10–1 nm
differences of 104 V. The energy acqiured by the
(c) l = 10 nm
–4
(d) l = 10–6 nm
electron is 1.6 × 10–15 J.
(v) Two particles A1 and A2 of masses m1 and m2
(m1 > m2) have the same de-Broglie wavelength. (iv) If electrons, photons are considered to be of same
Then de-Broglie wavelength, then they will have the
(a) their momenta are the same. same frequency.
(b) their energies are the same. 4. Match the Columns
(c) energy of A1 is less than the energy of A2. Column-I Column-II
(d) energy of A1 is more than the energy of A2. h
(vi) The graph showing the correct variation of linear (i) de-Broglie wavelength (a)
momentum (p) of a charge particle with its de- 2meV
Broglie wavelength (l) is [Delhi 2020] (ii) Sharp diffraction (b) 2d sin q = l
p p occurs due to
(iii) 1st order diffraction (c) constructive
formula interference
l l For Q. 5 there are two statements labelled as assertion
(a) (b)
(A) and reason (R). Select the correct answer to these
p p
questions from the codes (a), (b), (c) and (d) as given
below.
(a) Both A and R are true and R is the correct
l l explanation of A.
(c) (d)

394 Physics–12
D:\EG_Physics-12_(26-06-2022)\Open_Files\Ch-11\Ch-11
\ 27-Jul-2022 Ved_Goswami Proof-4 Reader’s Sign _______________________ Date __________

(b) Both A and R are true but R is not the correct 12. A proton and an alpha particle are accelerated through
explanation of A. the same potential. Which one of the two has (i)
(c) A is true but R is false. greater value of de-Broglie wavelength associated
(d) Both A and R are false. with it and (ii) less kinetic energy? Justify your
answers.[Delhi 2009, 2010]
5. Assertion (A): The de-Broglie wavelength of particle
having kinetic energy K is l. If its kinetic energy 13. A proton and an electron have same velocity. Which
λ one has greater de-Broglie wavelength and why?
becomes 4K then its new wavelength would be . [Delhi 2012]
2
14. Show that de-Broglie hypothesis of matter wave
Reason (R): The de-Broglie wavelength is inversely
supports the Bohr’s concept of stationary orbit.
proportional to the square root of kinetic energy.
[Delhi 2011]
II. Very Short Answer Type Questions (1 Mark) 15. The kinetic energy of electrons orbiting in the first
1. Show graphically, the variation of the de-Broglie excited state of hydrogen atom is 3.4 eV. Determine
wavelength (l) with the potential (V) through which the de-Broglie wavelength associated with it.
can electron is accelerated from rest. [Delhi 2011] [Foreign 2015]
2. Draw a plot showing the variation of de Broglie 16. A proton and an a-particle have the same de-
wavelength of electron as a function of its K.E. Broglie wavelength. Determine the ratio of (i) their
[Delhi 2015] accelerating potentials (ii) their speeds.
3. Why are de-Broglie wave associated with a moving [Delhi 2015]
football not visible? 17. The equivalent wavelength of a moving electron has
4. State de-Broglie hypothesis. [Delhi 2012] the same value as that of a photon having an energy of
5. What are the consideration that led de-Broglie to 6 × 10–17 J. Calculate the momentum of the electron.
say that material particles can also exhibit wave like [S.P. 2015]
properties? 18. An electron is revolving around the nucleus with a
6. Two particles have equal momenta. What is the ratio constant speed of 2.5 × 108 m/s. Find the de-Broglie
of their de-Broglie wavelengths? wavelength associated with it. [AI 2014 C]
[CBSE S.P. 2018-19] 19. Why are de-Broglie waves with a moving football
7. A proton and an electron have equal speeds. Find the not visible?
ratio of de-Broglie wavelengths associated with them. 20. Plot a graph showing variation of de-Broglie
[AI 2020] 1
wavelength l versus , where V is accelerating
III. Short Answer Type Questions-I (2 Marks) V
8. For what kinetic energy of a proton, will the associated potential for two particles A and B carrying same
de-Broglie wavelength be 16.5 nm? Mass of proton charge but of masses m1, m2 (m1 > m2). Which one
= 1.675 × 10–27 kg, h = 6.63 × 10–34 Js. of the two represents a particles of smaller mass and
[Delhi 2008 C] why?
9. An a-particle and a proton are accelerated from rest 21. Figure shows the variation of de-Broglie wavelength
through the same potential difference V. Find the ratio 1
(l) versus (where V is the accelerating potential)
of de-Broglie wavelength associated with them. V
[Delhi 2010] for two charged particles A and B. Which particles
10. An electron is accelerated through a potential have the havier mass? Explain.
difference of 64 volts. What is the de-Broglie
wavelength associated with it? To which part of A
the electromagnetic spectrum does this value of
λ
wavelength correspond?  [Delhi 2010] B
11. Calculate the ratio of the accelerating potential
required to accelerate (I) a proton and (II) an
a-particle to have the same de-Broglie wavelength
associated with them. [Delhi 2009 C]
O 1
V

Dual Nature of Radiation and Matter 395


D:\EG_Physics-12_(26-06-2022)\Open_Files\Ch-11\Ch-11
\ 27-Jul-2022 Ved_Goswami Proof-4 Reader’s Sign _______________________ Date __________

22. Calculate de-Broglie wavelength of an electron this surface have the de-Broglie wavelength l1, prove
accelerated through potential difference of V. volt. 2mc  2
that l =  λ .[Delhi 2008]
23. An α-particle and a proton are accelerated through same  h  1
potential difference. Find the ratio (vα/ vp) of velocities 28. An electron and photon each have a wavelength 1.00
nm. Find (i) their momenta (ii) the energy of the
acquired by two particles.
photon and (iii) the kinetic energy of electron. Given
[CBSE S.P. 2019-20] h = 6.6 × 10–34 Js, c = 3 × 108 m/s. [Delhi 2011]
IV. Short Answer Type Questions-II (3 Marks) 29. A proton and a deuteron are accelerated through the
24. X-rays fall on a photosensitive surface to cause same accelerating potential. Which one of the two has
photoelectric emission. Assuming that the work (a) greater value of de-Broglie wavelength associated
function of the surface can be neglected, find the with it, and (b) less momentum? Give reasons to
relation between the de-Broglie. Wavelength (l) of justify your answer. [Delhi 2014]
the electrons emitted to the energy (Ev) of the incident 30. Ultraviolet light of wavelength 2270 Å from 100W
photons. Draw the graph for l as a function of Ev. mercury source irradiates a photo cell made of given
[Delhi 2014 C] metal. If the stopping potential is – 1.3V, estimate the
25. A particle is moving three times as fast as an electron. work function of the metal. How would the photo cell
The ratio of the de-Brogile wavelength of the particle respond to a high intensity (~ 105 Wm–2) red light of
to that of the electron is 1.813 × 10–4. Calculate the wavelength 6300Å produced by a laser? [AI 2015]
particle’s mass and identify the particle. Mass of 31. Calculate the de-Broglie wavelength associated with
electron = 9.11 × 10–31 kg. [NCERT] the electron in the 2nd excited state of hydrogen atom.
The ground state energy of the hydrogen atom is 13.6
26. An electron, an a-particle, and a proton have the same eV.[Delhi 2020]
kinetic energy. Which of these particles has the (i) 32. (a) Explain de-Broglie argument to propose his
shortest de-Broglie wavelength? hypothesis. Show that de-Broglie wavelength of
(ii) longest de-Broglie wavelength? [Delhi 2007] photon equals electromagnetic radiation.
27. If an electromagnetic wave of wavelength l is (b)  If, deuterons and alpha particle are accelerated
incident on a photosensitive surface of negligible through same potential, find the ratio of the
work function. If the photo-electrons emitted from associated de-Broglie wavelengths of two.
[CBSE S.P. 2020–21]

Answers 11.2
I. Objective Type Questions λ

1. (i) (a) (ii) (b) (iii) (d)


(iv) (c) and (d) (v) (a) and (c) (vi) (b)
2. (i) b-particle
(ii) greater than that of the electron
V
(iii)
2eV (iv) 1eV (v) 1.227 Å V is potential difference through which electron is
m
accelerates.
3. (i) False (ii) False (iii) True (iv) False 1
4. (i)-(a) (ii)-(c) (iii)-(b) So l∝
V
5. (a) 2. de-Broglie wavelength of an electron
II. Very Short Answer Type Questions h h
l= =
h p 2mE K
1. l = , h → Planck’s constant
2meV 1
or     λ ∝
m, e are mass and charge of an electron resp. EK

396 Physics–12
D:\EG_Physics-12_(26-06-2022)\Open_Files\Ch-11\Ch-11
\ 27-Jul-2022 Ved_Goswami Proof-4 Reader’s Sign _______________________ Date __________

(6.63 × 10 −34 ) 2
=
2 × 1.675 × 10 −27 m × (16.5 × 10 −9 ) 2
λ = 4.8 × 10–25 J 1
λα
9. ua = up = 0, accelerating potential = V, λ = ?
p
Ek h
As l= 1  ½
h
3. As l = (2mqV) 2
mv
1
As mass of a football is large so the de-Broglie ( 2 m p q p Vp )2
λα h
wavelength associated with a moving football is very = 1
×
small. Hence its wave nature is not noticeable.       λ p h  ½
(2mα qα Vα )2
4. According to de-Broglie hypothesis, as radiations show 1 1
particle nature, the material particles in motion display  mp qp  2  1 1  2 1 1
wave-like properties. =  ×  = ×  = =
 mα qα  4 2 8 2 2
5. As radiation have dual nature i.e., they show the
properties of a particle as well as wave and energy λα : λ p = 1: 2 2  1
and matter are inter convertible. He said nature loves 12.27 12.27 12.27
symmetry so material particles also like radiations 10. l = Å= Å= Å = 1.53Å 1
V 64 8
exhibit dual nature.
They belong to X-rays. 1
6. p1 = p2 ½
11. If a charged particle of mass m, charge q is accelerated
h h p under a pot diff. V, the velocity acquired by particle is

Ratio l1/ l2= = 2 = 1: 1  ½
p1 p2 p1 v. Then where p is momentum of particle
7. Given vP = ve using de-Broglie equation, or p = 2mqV
h h De-Broglie wavelength,
l= = , we get h h
p mv l= =  1
p 2mqV
h
λp mpvp mp vp mp h h
    = = × = Now, lp = and λ α = ½
λe h me ve me 2 m p q p Vp 2mα qα Vα
me ve
Given, lp = la
−27
1.67 ×10 h h
= = 1.8 × 103 So =
9.1×10−31 2 m p q p Vp 2mα qα Vα
[CBSE Marking Scheme 2018-19] Vp mα qα 4m × 2e
or = = =8 ½
8. EK = ?, l = 16.5 nm = 16.5 × 10 m –9
Vα mp q p m×e
mp = 1.675 × 10–27 kg, h = 6.63 × 10–34 Js 12. (i) When a charged particle of charge q, mass m is
1
h p 2 accelerated under a pot. Diff. V, let v be the velocity
As l= and = E K So p = (2mE K )2
acquired by the particle, then
p 2m
1
1
h qV = mv 2 or mv = [2mqV] 2

\ l= 1  1 2
(2mE K )2 h h 1
  \ l= = or λ ∝  ½
On squaring both side and solving for EK mv 2mqV mq
Hence,
h2 λp mα qα 4m 2e
EK = = = × = 2 2 >1 ½
2 mλ 2 λα mp q p m e

Dual Nature of Radiation and Matter 397


D:\EG_Physics-12_(26-06-2022)\Open_Files\Ch-11\Ch-11
\ 27-Jul-2022 Ved_Goswami Proof-4 Reader’s Sign _______________________ Date __________

So lp > la, i.e., de-Broglie wavelength associated h


with proton is greater than that of alpha particle. l= , V is accelerating potential
2mqV
(ii) Kinetic energy of charged particle
EK = qV, i.e., EK ∝ q ½ λp mα qα Vα
(i) =
EK p qp λα m p q p Vp
e 1
\ = = = < 1 or E K p < E K α  ½
E Kα qα 2e 2 4m p 2e Vα
  So  1= × ×  ½
13. de Broglie wavelength mp e Vp
h 8Vα Vp
l= , v → velocity ½ 1= or = 8 ½
mv Vp Vα
λp me ve mv m
= = e = e < 1 , as me < mp 1 h h
λe mP v p m p v m p (ii) lp = , λα =
mpvp mα vα
lp < le so de-Broglie wavelength of electron is greater. λp mα vα
 ½ then =  ½
λα mpvp
14. According to de-Broglie hypothesis, the wavelength of
the wave associated with electron while moving with 4 mp vα
v → velocity 1 = ×
velocity v is given by mp vp
h vp
l= ...(i) ½ = 4 ½
mv vα
According to de-Broglie, stationary orbit is that
hc
orbit whose circumference is an integral multiple of 17. For a photon E =
λ
wavelength of wave associated with electron in that
orbit. hc
So l=  ½
If l is the de Broglie wavelength of electron while E
revolving in nth orbit of radius r, then h
For an electron l =  ½
2πr p
2pr = nλ or λ = ...(ii) ½
n h hc
So l= =
From (i) and (ii),        p E
2pr h nh E 6 × 10 − 17
= or mvr =  ½ or p= = = 2 × 10 −25 kgm/s 1
n mv 2π c 3 × 108
 h h
i.e., Total angular momentum = n   18. l=  1
2π mv
This is one of the postulate in Bohr’s atomic model.½ 6.6 × 10 −34
=
15. EK = 3.4 eV = 3.4 × 1.6 × 10–19 J ½ 9.1 × 10 −31 × 2.5 ×108
 p 2
 = 2.9 × 10–12 m 1
 Use = EK 
 2m  19. de-Broglie wavelength associated with a body of mass
l = ? h
h h m, moving with velocity v is given by λ =  1
l= =  ½ mv
p 2mE K Since, the mass of football is quite large, hence the
de-Broglie wavelength associated with it is quite small
6.6 × 10 −34 hence it is not visible. 1
= 1 ½
−31 20. de-Broglie wavelength when a charged particle is
[2 × 9.1 × 10 × 3.4 × 1.6 × 10 −19 ] 2
accelerated through a potential difference of V volt is
= 6.6 × 10–10 m ½ 1
16. la = lp (given) l=  ½
2mqV

398 Physics–12
D:\EG_Physics-12_(26-06-2022)\Open_Files\Ch-11\Ch-11
\ 27-Jul-2022 Ved_Goswami Proof-4 Reader’s Sign _______________________ Date __________

1
m v 2 = Ka = qaVa  (ii) ½
A
2 a a
λ
B Divide (ii) by (i)
       ½ mα vα2 q 2
= α =
mpvp 2
qp 1
vα2 mp × 2 2m p 1
= =
O
=
1
V
vp2
mα × 1 4m p × 1 2
As m1 > m2
So for particle of mass m1, the slope of the line will be va : vp = 1 : 2  1
less as m1 > m2. 24. As work function is negligible so f0 = 0
So the line B is for m1 and the line A is for m2. 1 According to Einstein’s eqn. of photoelectric emission
21. de-Broglie wavelength associated with a charge q, hv = (KE)max + f0 ½
accelerated through a P.D. V is given by 1 2
So mvmax = hv
h h 2
l= =  1
2mqV 2mq V hv = Ev given
1 2
1 So mvmax = Ev ½
For same value of , l > lB so mA < mB 2
V A
p2
So particle A is lighter. 1 According to the formula = K.E.
2m
h
22. l= ½ p= ½
mv 2m K.E. = 2mE V 
m = mass of electron
So the de-Broglie wavelength
e = charge of electron
h h h
The electron is accelerated through pot. of V volt. l== =  ½
p mvmax 2mE V
KE = eV
p2 1
KE = So l∝  ½
2m EV
p2 = 2m (KE)
p = 2mKE  ½ Energy of
the incident
h h photon E V

l= =
2m(KE ) 2meV λ
    ½
12.27 λ
l= Å ½ 25. n = 3ve, = 1.813 × 10 −4 , m = ?,
V λe
me = 9.11 × 10–31 kg
h h
λ As l= , λe =
mv me ve
     ½ λ h mv m v
= × e e = e × e 1
λe mv h m v
O 1
9.11 × 10 −31 1
V 1.813 × 10–4 = ×  1
23. Gain in KE of particle = qV m 3
1 9.11 × 10 −31
m v 2 = Kp = qpVp  (i) ½ So m=
2 p p 3 × 1.813 × 10 −4
Vp = Va = V = 1.675 × 10–27 kg 1

Dual Nature of Radiation and Matter 399


D:\EG_Physics-12_(26-06-2022)\Open_Files\Ch-11\Ch-11
\ 27-Jul-2022 Ved_Goswami Proof-4 Reader’s Sign _______________________ Date __________

26. (i) ( E K )e = ( E K )α = ( E K ) p 6.63 ×10 −34


= = 6.6 × 10 −25 kgm/s  1
h p 2 10 −9
l= = E=
and K so p 2mE K  1
p 2m hc
(ii) K.E. of photon = E K = hv =
h λ
1
Thus l= or l∝  ½
2mE K m 6.6 × 10 −34 × 3 × 108
= −9
= 19.8 × 10 −16 J  ½
The particle having highest mass will have the shortest 10
wavelength. pe2
(iii) K.E. of electron = E K =
1 2me
(ii) ma = 4m p , me = mp  ½
1836
(6.6 × 10 −25 ) 2
So electron has the least mass and an a particle has the = −31
= 2.39 × 10 −19 J  1
highest mass. ½ 2 × 9.1 × 10
So a particle will have shortest wavelength. De-Broglie 29. (a) de-Broglie wavelength is given by
wavelength is maximum for electron. ½ h 1
l = or λ ∝  ½
27. As work function is negligible so f0 = 0 2qVm qm
1 hc
So 2
mVmax = hν = ...(1) ½ λp qd md md
2 λ
\ = =  1 ( qd = qp)
λd q p mp mp
de-Broglie wavelength
h h As md > mp; so lp > ld ½
l1 = =  ½
p mV (b) Momentum of a particle,
h 1 pp λd
p2 p=i.e., p ∝ ∴ = <1 1
To find mv = p we use = EK,EK is K.E. λ λ pd λ p
2m
or pp < pd
p2 hc 30. V0 = 1.3 V, l = 2270 Å = 2270 × 10–10 m
=  ½
2m λ hc
[Using eqn. (1)] eV0 = − φ0 , f0 is work function
1
λ
p =  2mch   2
½  f0 =
hc
− eV0  ½
 
λ  λ
l is wavelength of incident photon  6.6 × 10 −34 × 3 × 108 
Thus the wavelength of emitted electron =  −10
− 1.6 × 10 −19 × 1.3 J
 2270 × 10 
1 1
h hλ 2  hλ  2  1980 × 10 −19 
l1 = = =  ½ =  − 1.6 × 10 −19 × 1.3 J
p 1  2mc   227 
(2mch) 2
= 8.7 × 10–19 – 1.6 × 10–19 × 1.3 J ½
hλ 2mc 2
\ l1 = 2
or l = λ1  ½  8.7 × 10 −19 
2mc h =
 1.6 × 10 −19 − 1.3 eV
28. le = lPhoton = 1.0 nm = 10–9 m, p = ?,
EK of photon = ?, EK of electron = ? = 5.4 –1.3 = 4.1 eV ½
(i) Momenta of photon If l is increased to 6300 Å Energy of incident photon
h 6.63 × 10 −34 hc 6.6 × 10 −34 × 3 × 108
= pph = = = =
λ Ph 10 −9 λ 6300 × 10 −10
= 6.6 × 10–25 kgm/s ½ 198
= × 10 −19 = 3.14 × 10 −19 J 
Momenta of electron 63
h 3.14 × 10 −19
= pe = = eV −~ 2 eV  ½
λe
1.6 × 10 −19

400 Physics–12
D:\EG_Physics-12_(26-06-2022)\Open_Files\Ch-11\Ch-11
\ 27-Jul-2022 Ved_Goswami Proof-4 Reader’s Sign _______________________ Date __________

putting the values in formula for wavelength we get,


q h 6.63 × 10−34
l= =
50°
q 2m ( KE ) 2 × 9.1× 10−31 × 2.42 × 10−19
  = 1.001 × 10–9 m ~ 10Å
  ½ 32. (a) D e-Broglie put forward the bold hypothesis
that moving particles of matter should display
wave-like properties under suitable conditions.
As the wavelength of incident photon is increased to
If radiation shows dual aspects, so should matter.
6300 Å. No photo-emission will take place as energy De-Broglie proposed that the wave length l
of incident photon is less than work function. associated with a particle of momentum ‘P’is given
31. de-Broglie wavelength is given by as:
h h h h
l= = ...(1) l= = ;
nv p p mv
we know that, where,
m = mass of the particle
p2 v = particle speed
KE =
2m For a photon: p =
hv

⇒ p= 2m ( KE ) c

h Therefore, h = c = λ

⇒ l=  [using (1)] p v
2m ( KE ) Thus, De-Broglie equation equals the wavelength
Bohr’s model, Kinetic Energy of e– of em radiation of which the photon is a quantum
of energy and momentum.
13.6 × Z2
   = | Total Energy of e | = − – eV (b) De-broglie wavelength is given by:
n2 h
for Hydrogen (Z = 1) and for 2nd excited state, n = 3 l=
p
13.6 × 12 h

\ KE = 2
eV = 1.51 eV l=
3 2mqv
       = 1.51 × 1.6 × 10–19 J
λd mα qα 4×2
[ 1 eV = 1.6 × 10–19 J] = = =2
      = 2.42 × 10–19 J λα md qd 2 ×1

Case Based Questions


I. When light of a sufficiently high frequency is incident on 2. If frequency (n > n0) of incident light becomes n times
a metallic surface, electrons are emitted from the metallic the initial frequency (n), then what will be the kinetic
surface. This phenomenon is called photoelectric emission. energy of the emitted photo electrons?
Kinetic energy of the emitted photo electrons depends on 3. When light is incident on a metallic surface, how
the wavelength of incident light and is independent of the does the maximum kinetic energy of the emitted
intensity of light. The number of emitted photoelectrons
electrons vary?
depends on intensity. (hn – f) is the maximum kinetic
Ans. 1. Emax = hn – f
energy of emitted photo-electrons (where f is the
2. It will be more than the n times of initial kinetic
work function of metallic surface). Reverse effect of
photoemission produces X-ray. X-ray is not deflected by
energy.
magnetic and electric fields. Wavelength of continuous 3. It varies with frequency of incident light.
X-ray depends on potential difference across the tube. II. According to de-Broglie, a moving material particle
Wavelength of characteristic X-ray depends on the atomic sometimes acts as a wave and sometimes as a particle or a
number. wave associated with moving particle which controls the
1. Write Einstein’s photoelectric equation. particle in every respect. The wave associated with moving
particle is called matter wave or de-Broglie wave whereas

Dual Nature of Radiation and Matter 401


D:\EG_Physics-12_(26-06-2022)\Open_Files\Ch-11\Ch-11
\ 27-Jul-2022 Ved_Goswami Proof-4 Reader’s Sign _______________________ Date __________

wavelength is called de-Broglie wavelength, is given by an electron from the metal surface i.e., wave
h function.
mv (iii) When a photon interacts with one electron, part
of its energy is used in liberating electron, (i.e.,
work function) while remaining energy is carried
by electron as kinetic energy.
E = W0 + (KEmax)
1 2
E = W0 + mvmax
 2
1. If the velocity of an electron increases, what will 1 2
happen to its de-Broglie wavelength? hn = hn0 + mvmax
2
2. What is the de-Broglie wavelength of a body of 1 2
mass 150g moving at a speed of 50m/s? h(n – n0) = mvmax
2
3. According to de-Broglie's relation, if the velocity of a
particle is infinite, then what will be its wavelength? If n < n0L.H.S. is –ve, no photoemission
h n > n0L.H.S. is +ve, photoemission
Ans. 1. As λ = , therefore the de-Broglie wavelength n = n0L.H.S. is zero, no photoemission

will decrease on increasing the velocity. 1 2
but mvmax = eV0where V 0 is accelerating
h 6.6 × 10 −34 2
2. λ = = = 8.8 × 10–35 m potential
mν 150 × 10 −3 × 50
hn = hn0 + eV0
3. The wavelength will be zero.
hn = W0 + eV0
III. Einstein’s Equation of Photoelectric Effect:
Photoelectric effect does not takes place by continuous h W
V0 = ν− 0
absorption of energy from radiation. Radiation energy e e
is built up of discrete units—the so called quanta of
energy radiation. Each quanta of radiant energy has
energy hn where h is Planck’s constant and n is the
frequency of light.
Kmax
h

W0
Compare it with equation of a straight line
y = mx + c
The slope of the curve between stopping potential
If a quanta of radiant energy (hn) called photon falls h
on a metallic surface, this energy in totally imported and frequency gives and intercept on y-axis is
−W0 e
to a single electron and some part of this energy is
.
used by electron just to overcome the surface barrier e
(W0) and rest part remains in the form of kinetic ● Explanation of Kinetic Energy:
energy with the electron. If energy of photon is less For v > vo, (Kinetic Energy)max = +ve
than the whole function of the electron, no electron As the frequency of incident radiation increases,
will emitted out, thus for photoelectric emission a kinetic energy carried by photoelectrons will
minimum amount of energy i.e., threshold energy at also increase. Hence, more stopping potential is
least equal to the work function is needed (W0 = hn0, required to stop the photoelectric current.
where n0 is threshold energy). ● Explanation of Intensity: Increase in intensity
(i) Photoelectric emission is result of interaction of means increase in no. of photons striking per unit
one photon with one electron of metal. time i.e., increase in no. of e– being emitted.
(Photon is of incident radiation). As each photon eject only one electron, the no.
(ii) As the electrons are bound with metal, so of photoelectrons emitted will also increase with
minimum amount of energy is required to liberate increase in intensity.
402 Physics–12
D:\EG_Physics-12_(26-06-2022)\Open_Files\Ch-11\Ch-11
\ 27-Jul-2022 Ved_Goswami Proof-4 Reader’s Sign _______________________ Date __________

So saturation current depends on intensity.


1. Photoelectrons stopping potential depends on
(a) frequency of incident light and nature of the
4.
cathode material.
(b) the intensity of the incident light.
(c) the frequency of the incident light.
(d) nature of cathode material.
2. What is the value of the maximum kinetic
energy acquired by electron due to radiation of Which metal has smaller threshold wavelength?
wavelength 100 nm? (a) Metal ‘A’ (b) Metal ‘B’
(a) 12 eV (b) 6.2 eV (c) Metal-A and Metal-B have equal wavelength
(c) 100 eV (d) 300 eV (d) None of these.
3. If the work function for a certain metal is 3.2 5. Light of wavelength 4000 Å is incident on
× 10–19 joule and it is illuminated with light of a sodium surface for which the threshold
frequency 8 × 1014 Hz. The maximum kinetic wavelength of photo-electrons is 5420 Å. The
energy of the photo-electrons would be (h = 6.63 work function of sodium is
× 10–34 Js). (a) 5 eV (b) 3 eV (c) 2.29 eV (d) 0.57 eV
(a) 2.1 × 10–19 J (b) 8.5 × 10–19 J Ans. 1. (a) 2. (b) 3. (a) 4. (b) 5. (c)
(c) 5.3 × 10 J
–19
(d) 3.2 × 10–19 J

IMPORTANT FORMULAE
Formula Symbols Application
1. c E = energy of photon, To find the energy of photon
E = hv = h
λ h = Planck’s constant,
v = frequency
2. W0 = hv0 = hc/l0 W0 = work function Relation between work function and v0, l0
v0 = threshold frequency
l0 = threshold wavelength
3. 1 Kmax = Maximum kinetic Einstein's photoelectric equation.
Kmax = mv 2 max energy of emitted
2
electrons
  = hn – W0 = h (n –n0)
4. 1 e = charge of electron Relation between maximum kinetic energy and
Kmax = mv 2 max = eV0
2 stopping potential.
5. h h l = wavelength of matter de Broglie wavelength for matter wave.
l= or l =
mv p wave
p = Momentum
6. h h E = Kinetic energy
λ= = Relation between l and E, V
2mE 2mqV V = accelerating potential
7. l = h 12.27
,l= Å V = accelerating potential de Broglie wavelength for electron when accelerated
2meV V through a potential difference of V volt.

COMMON ERRORS
S.No. Errors Corrections
1. Explanation of laws of Photo Electric Graphical treatment should be explained with different factors
Effect using Einstein equation and drawing the conclusions
2. Graph between stopping potential and Calculation of slope of the graph
frequency

Dual Nature of Radiation and Matter 403


D:\EG_Physics-12_(26-06-2022)\Open_Files\Ch-11\Ch-11
\ 27-Jul-2022 Ved_Goswami Proof-4 Reader’s Sign _______________________ Date __________

REVISION CHART

Photoelectric Effect and Electron Emission


Photon Picture of Electromagnetic Radiation: According to quantum theory, energy of electromagnetic wave is carried by
discrete packets or bundles of energy,called quantums.
1 quantum of radiation of light is called a photon, which travels with speed of light.

Electron Emission: Thermionic emission Photoelectric Emission


When a metal is heated, its electrons get sufficient It is the process in which electrons are emitted from a
thermal energy and they can overcome the surface barrier. metal surface when electromagnetic radiation of suitable
frequency are incident on it. The electrons so emitted are
called photo electrons.

Wave Nature of Matter


Dual Nature of Matter/de Broglie Hypothesis: As radiation has dual nature i.e., wave as well as particle and energy and mass
are interconvertible according to the relation E = mc2 so de Broglie put the hypothesis that the material particle in motion should
display wave like properties because nature loves symmetry.

de Broglie Wave and Wavelength


The wave associated with material particles in motion are called matter waves
or de Broglie waves. Their wavelength is called de Broglie wavelength.
h h
l= =
p mv
If v = 0 (particle is at rest) l = ∞

404 Physics–12
D:\EG_Physics-12_(26-06-2022)\Open_Files\Ch-11\Ch-11
\ 27-Jul-2022 Ved_Goswami Proof-4 Reader’s Sign _______________________ Date __________

IMPORTANCE OF EACH TOPIC AND FREQUENTLY ASKED TYPES OF QUESTIONS

☞ Important Topics
1. Questions on Einstein equation of photoelectric emission.
2. Questions based on de Broglie waves, stopping potential.
* Maximum weightage is of Experimental study of Photoelectric Effect.

1. Ultraviolet light of wavelength 350 nm and intensity 1 W/m2 is directed at a potassium surface having work function
2.2 eV.
(i) Find the maximum kinetic energy of the photoelectron.
(ii) If 0.5 percent of the incident photons produce photoelectric effect, how many photoelectrons per second are
emitted from the potassium surface that has an area 1 cm2 .
photoelectron
[Ans. (i) KEmax. = 1.3 eV; (ii) n = 8.8 × 1011 ]
second
2. A metal surface illuminated by 8.5 × 1014 Hz light emits electrons whose maximum energy is 0.52 eV the same
surface is illuminated by 12.0 × 1014 Hz light emits electrons whose maximum energy is 1.97eV. From these data
find work function of the surface and value of Planck's constant. [Ans. Work Function = 3eV]
3. An electron and photon each have a wavelength of 0.2 nm. Calculate their momentum and energy.
[Ans. (i) 3.3 x 10–24 kgm/s; (ii) 6.2 k eV for photon; (iii) 38 eV for electron]
4. What is the (i) Speed (ii) Momentum (iii) de Broglie wavelength of an electron having kinetic energy of 120 eV?
[Ans. (i) 6.5 × 106 m/s; (ii) 5.92 × 10–24 kg m/s; (iii) 0.112 nm.]
5. If the frequency of incident light in photoelectric experiment is doubled then does the stopping potential become
double or more than double, justify? [Ans. More than double]
6. A proton is accelerated through a potential difference V. Find the percentage increase or decrease in its de Broglie
wavelength if potential difference is increased by 21 %. [Ans. 9.1%]
7. For what Kinetic energy of a neutron will the associated de Broglie wavelength be 5.6 ×10 m?–10

2 2
h  h 1  6.6251 × 10 −34  1
[Ans. 2mn × K.E. = ⇒ K.E. =   × =  = 3.351 × 10–21 J]
λ  λ 2mn −10 
 5.61 × 10  2 × 1.675 × 10 −27

M 2M
8. A nucleus of mass M initially at rest splits into two fragments of masses and . Find the ratio of de Broglie
3 3
wavelength of the fragments.
[Ans. Following the law of conservation of momentum,
2M
M 2M ν2
M 2M h λ1 λ 2ν
ν1 + ν2 = 0 or ν1 = ν2 ; λ = ⇒ = 3 ⇒ 1 = 2
3 3 3 3 mν λ2 M λ2 ν1
ν
3 1
9. An electron and a proton are possessing same amount of K.E., which of the two have greater de-Broglie, wavelength?
Justify your answer.
Ans. E e = 1 me νe2 and E p = 1 m p ν2p ⇒ ν m = 2E m and m ν = 2E m
e e e e p p p p
2 2

λe mp
But. Ee = Ep ⇒ = >1 \ le > lp.
λp me

Dual Nature of Radiation and Matter 405


D:\EG_Physics-12_(26-06-2022)\Open_Files\Ch-11\Ch-11
\ 27-Jul-2022 Ved_Goswami Proof-4 Reader’s Sign _______________________ Date __________

ASSIGNMENT
I. Objective Type Questions (1 Mark)
1. Multiple choice questions:
(i) When ultraviolet rays incident on metal plate then photoelectric effect does not occur, it occurs by incidence
of
(a) Infrared rays (b) Radio waves (c) X-rays (d) Light waves
(ii) A particle is dropped from a height H. The de-Broglie wavelength of the particle as a function of height is
proportional to
(a) H (b) H1/2 (c) H0 (d) H–1/2
2. Fill in the blanks:
(i) One quantum of radiation of light is called a ..................... .
(ii) Out of proton, neutron, b-particle and a-particle ................ will have the maximum de-Broglie wavelength.
II. Very Short Answer Type Questions (1 Mark)
3. Show graphically, the variation of the de-Broglie wavelength (l) with the potential (V) through which can electron
is accelerated from rest.
4. Define intensity of radiation on the basis of photon picture of light. Write its SI Unit.
III. Short Answer Type Questions-I (2 Marks)
5. The kinetic energy of electrons orbiting in the first excited state of hydrogen atom is 3.4 eV. Determine the de-
Broglie wavelength associated with it.
6. Figure shows variation of stopping potential (V0) with frequency (n) for two photosensitive material M1 and M2.
(i) Why is the slope same for both lines?
(ii) For which material will the emitted electrons have greater kinetic energy for the incident radiations of the same
frequency? Justify your answer. M1 M2
7. Monochromatic light of frequency 6.0 × 1014 Hz is produced by a laser. The power
V
emitted is 2.0 × 10–3 W. (a) What is the energy of a photon in the light beam? (b) How 0
many photons per second, on the average, are emitted by the source? Given h = 6.63
× 10–34 Js. v
8. Why it is the frequency and not the intensity of light source that determines whether
emission of photoelectrons will occur or not? Explain. [CBSE 2022]
IV. Short Answer Type Question-II (3 Marks)
9. State laws of photo-electric emission.
10. (a) Give an example each of a metal from which photoelectric emission takes place when irradiated by (i) UV light
(ii) visible light.
(b) The work function of a metal is 4.50 eV. Find the frequency of light to be used to eject electrons from the metal
surface with a maximum kinetic energy of 6.06 × 10–19 J. [CBSE 2022]
V. Long Answer Type Question (5 Marks)
11. (a) An electron is accelerated from rest through a potential V. Obtain the expression for the de-Broglie wavelength
associated with it. 2
1
(b) Plot a graph showing variation of de-Broglie wavelength l versus , where V is accelerating. Potential for
V
two particles A and B carrying same charge but of masses m1, m2 (m1 > m2). Which one of the two represents
a particles of smaller mass and why?  1

(c) Show that de-Broglie hypothesis of matter wave supports the Bohr’s concept of stationary orbit. 2


406 Physics–12
D:\EG_Physics-12_(26-06-2022)\Open_Files\Ch-12\Ch-12
\ 27-Jul-2022 Ved_Goswami Proof-4 Reader’s Sign _______________________ Date __________

Topics Covered
12 Atoms

12.1 Alpha-particle Scattering Experiment and Rutherford’s Nuclear Model of Atom


12.2 Bohr ’s Model of Hydrogen Atom and Hydrogen Spectra

C hapter map
Atoms

Alpha particle scattering Bohr’s postulates


experiment

Bohr’s Model of
Rutherford’s Nuclear Hydrogen Atom &
Model of an Atom Spectra

Hydrogen Radius of nth orbit of


Spectrum an electron and Energy
level diagram

Spectral series
De Broglie’s
explanation

Lyman series Balmer series Paschen series Brackett series Pfund series

Topic 1. Alpha-particle Scattering Experiment and Rutherford’s Nuclear


Model of Atom
•• A narrow beam of alpha-particles is made to strike on a
thin foil of gold in an evacuated chamber. 107
Observations: Theoretical
106
Number of particles

prediction
(i) Most a particles passed straight, through the foil or
scattered, N (θ)

suffer only small deflection. 105


Experimental data
Conclusion: Most space in an atom is empty. 104
(ii) A few particles, about 1 in 8000, get deflected through 103
90° or more.
102
(iii) Occasionally, an a-particle gets rebounded from the
gold foil suffering a reflection of nearly 180°. 10
0 20 40 60 80 100 120 140
Scattering angle θ (in degrees)

407
D:\EG_Physics-12_(26-06-2022)\Open_Files\Ch-12\Ch-12
\ 27-Jul-2022 Ved_Goswami Proof-4 Reader’s Sign _______________________ Date __________

Conclusion: All charge and most of the mass is


concentrated in a very small region called the nucleus.
Conclusions: He concluded that nucleus is
surrounded by electrons whose negative charge is
equal to the positive charge of the nucleus. Electrons
are revolving around nucleus.
•• Rutherford’s Model of an Atom: An atom consists of
a small and massive core in which entire +ve charge and 1 2 KZe 2
K.E. = mv = ...(1)
almost the whole of an atom is concentrated. This core 2 2r
is called the nucleus. − KZe × e − KZe 2
Size of the nucleus is very small as compared to size of P.E. = = ...(2)
r r
atom.
1
+ Total energy E=  [Using (1) and (2)]
4πε 0
− e2
For Hydrogen Z = 1, so E =
+ 8πε 0 r
Negative sign shows that it is a bound system. Energy
needs to be supplied to remove the electrons.
•• Estimate of Nuclear Size or distance of closest
Nucleus is surrounded by a suitable number of electrons approach: Consider an a-particle of mass m moving
so that atom as a whole is neutral. Electrons revolve with initial velocity v directly towards centre of nucleus
around the nucleus in various orbits. The centripetal of atom.
force required for this revolution is provided by the
electrostatic attraction between the electrons and the α-particle B
nucleus. A
1 2
KE = mv
•• Limitations of Rutherford’s Atomic Model: 2
(i) An electron revolving around nucleus is under
r0
continuous acceleration so it should continuously
lose energy and the radii of the orbits will continue
As the a-particle approaches nucleus, its kinetic energy
to decrease. So, the electron will follow a spiral path
continues to decrease, but the potential energy increases.
and finally collapse into the nucleus.
At a certain distance r0, the alpha particle begins to
So, Rutherford’s model cannot explain the stability
retrace its path. This r0 is called the distance of closest
of an atom.
approach.
(ii) As electrons can revolve in orbits of all possible radii,
Let Z be the atomic number of the nucleus.
so it should emit a continuous spectrum. But, an atom
Charge on nucleus = + Ze
like hydrogen always gives a line spectrum.
Charge on a-particle = + 2e
So, the model could not explain line spectrum of
hydrogen. 2e Ze 1 2e Ze
PE = K =
•• Total energy of electron in a hydrogen atom on the r0 4π ∈0 r0
basis of Rutherford’s Model: According to Rutherford’s KE at A = PE at B.
model, electron is revolving around nucleus in a circular 2 Ze 2
1
orbit of radius r. mv 2 =
2 4π ∈0 r0
Let mass of electron be m.
Let Z be the charge number so charge on the nucleus Ze 2 Ze 2
r0 =
mv 2 KZe e 4π ∈0 (KE )
Centripetal force = =
r r2 2 Ze 2
⇒ r0 =
KZe 2 4π ∈0 (KE )
or mv2 =
2r r0 was the upper limit on the size of a nucleus.

408 Physics–12
D:\EG_Physics-12_(26-06-2022)\Open_Files\Ch-12\Ch-12
\ 27-Jul-2022 Ved_Goswami Proof-4 Reader’s Sign _______________________ Date __________

•• Impact Parameter: It is the perpendicular distance of In the given diagram, distance between particle 1 and
the velocity vector of the alpha particle from the central 4 is impact parameter for particle 1. Distance between
line of the nucleus, when the particle is far away from particle 3 and 4 is impact parameter for particle 3.
the atom.
θ
Ze 2 cot
Impact parameter
1
1 2
for particle 1  b 2
Impact parameter = b =
3
4π ∈0 E

++++
4 + ++ Nucleus
+ +
+++
3′
2′ q is angle of scattering,
1′

E → KE of a-particle
r0
r0 is the distance of closest approach

Relation between impact parameter and angle of Z → atomic number of gold nucleus.
scattering: More is the impact parameters lesser is the
angle of scattering.

EXERCISE 12.1
I. Objective Type Questions (1 Mark) (d) The stability of atom was established by the
1. Choose the correct answers from the given options model.
(MCQs). 2. Fill in the blanks.
(i) A set of atoms in an excited state decays (i) ............... of the electron in the orbit signifies that
the electron and nucleus is a bound system.
(a) in general to any of the states with lower
(ii) Rutherford’s a-particle scattering experiment
energy.
shows the existence of a ................. charged nucleus
(b) into a lower state only when excited by an
of ............. size located at the ......................... .
external electric field.
(iii) The kinetic energy associated with an electron
(c) all together simultaneously into a lower state. decreases with an ................. in the radii of the
(d) to emit photons only when they collide. orbits.
(ii) In 88Ra226 nucleus, there are 3. State True or False
(a) 138 protons and 88 neutrons (i) For a given projectile and target, the distance of
(b) 138 neutrons and 88 protons closest approach increase with increase in K.E.
(c) 226 protons and 88 electrons of the projectile.
(d) 226 neutrons and 138 electrons (ii) In scattering of a-particles by nucleus, the distance
(iii) A nucleus represented by the symbol ZXA has of closest approach depends upon the charges of
(a) Z protons and A neutrons projectile and target nucleus as well as kinetic-
energy of a-particle.
(b) A protons and (Z – A) neutrons
(iii) A few particles, about 1 in 8000, get deflected
(c) Z neutrons and (A – Z) protons
through 90° or more.
(d) Z protons and (A – Z) neutrons
4. Match the Columns
(iv) According to classical theory, Rutherford atoms
is: Column-I Column-II
(a) stable (b) unstable (i) From a-particle (a) the size of nucleus
(c) metastable (d) semistable scattering, it can be
(v) Which of the following statements in not correct concluded that
according to Rutherford model? (ii) Distance of closest (b) the stability of the
[Delhi, AI 2020] approach gives idea atom
(a) Most of the space inside an atom is empty. about
(b) The electrons revolve around the nucleus (iii) Rutherford’s atomic (c) the atom is mostly
under the influence of coulomb force acting model could not neutral
on them. explain
(c) Most part of the mass of the atom and its (iv) Rutherford’s (d) electrons are not
positive charge are concentrated at its centre. concluded that stationary

Atoms 409
D:\EG_Physics-12_(26-06-2022)\Open_Files\Ch-12\Ch-12
\ 27-Jul-2022 Ved_Goswami Proof-4 Reader’s Sign _______________________ Date __________

For Q. 5 and 6 there are two statements labelled as 5. The kinetic energy of a-particle incident on gold foil
assertion (A) and reason (R). Select the correct answer is doubled. How does the distance of closest approach
to these questions from the codes (a), (b), (c) and (d) as change?
given below. III. Short Answer Type Questions-I (2 Marks)
(a) Both A and R are true and R is the correct 6. In Rutherford scattering experiment, if a proton is
explanation of A. taken instead of an alpha particle, then for same
(b) Both A and R are true but R is not the correct distance of closest approach, how much K.E. in
explanation of A. comparison to K.E. of a-particle will be required?
(c) A is true but R is false. [Delhi 2009]
(d) Both A and R are false. 7. Explain Rutherford’s model of an atom.
5. Assertion (A): The force of repulsion between atomic 8. What are the limitations of Rutherford’s atomic
nucleus and a-particle varies with distance according models?
to inverse square law. 9. Define the distance of closest approach. An a-particle
Reason (R): Rutherford did a-particle scattering of kinetic energy ‘K’ is bombarded on a thin gold foil.
experiment. The distance of the closest approach is ‘r’. What will
6. Assertion (A): For the scattering of a-particles be the distance of closest approach for an a-particle
at a large angle, only the nucleus of the atom is of double the kinetic energy?
responsible. Or
Reason (R): Nucleus is very heavy in comparison to Write two important limitation of Rutherford nuclear
electrons model of the atom. [Delhi 2017]
II. Very Short Answer Type Questions (1 Mark) 10. Calculate the orbital period of the electron in the first
1. Write two important inferences drawn from excited state of hydrogen atom. [Delhi 2019]
Rutherford’s a-particle scattering experiment. 11. Write shortcomings of Rutherford atomic model.
[AI 2005 C] Explain how these were overcome by the postulates
Or of Bohr’s atomic model. [Delhi 2020]
Explain briefly how Rutherford scattering of 12. What results do you expect if alpha particle scattering
a-particle by a target nucleus can provide information experiment is repeated using a thin sheet of hydrogen
on the size of the nucleus. [Delhi 2019] in place of a gold foil? Explain. (Hydrogen is a solid
2. In Rutherford’s alpha ray scattering experiment, at temperature below 14 K.) [CBSE 2022]
why most of alpha-particles pass through almost
unscattered? IV. Short Answer Type Questions-II (3 Marks)
3. Define distance of closest approach and impact 13. Using Rutherford’s model of an atom, derive the
parameter. expression for the total energy of the electron in
4. What is the impact parameter for scattering of hydrogen atom what is the significance of total
a-particle by 180°? negative energy possessed by the electrons.

Answers 12.1
I. Objective Type Questions II. Very Short Answer Type Questions
1. (i) (a) (ii) (b) (iii) (d) (iv) (b) 1. (i) Most of the mass and entire positive charge are
(v) (d) concentrated in a very small volume of the atom,
called nucleus.
2. (i) Negative energy
(ii) Most of the space in an atom is empty and nuclear
(ii) positively, very small, centre of the atom radius is about 1/10,000 of the atomic radius.
(iii) increase 2. This is because there is lot of empty space between the
3. (i) False (ii) True (iii) True nucleus and revolving electrons in the atom.
4. (i)-(c) (ii)-(a) (iii)-(b)(iv)-(d) 3. Distance of closest approach is the distance between the
centre of nucleus and the point from which the alpha
5. (b) 6. (a)
particle approaching directly to the nucleus returns.

410 Physics–12
D:\EG_Physics-12_(26-06-2022)\Open_Files\Ch-12\Ch-12
\ 27-Jul-2022 Ved_Goswami Proof-4 Reader’s Sign _______________________ Date __________

Impact parameter is the perpendicular distance of the So, the model could not explain line spectrum of
velocity vector of the alpha particle from the central hydrogen. 1
line of the nucleus, when the particle is far away from +
the atom.
4. Zero.
5. The distance of closest approach is given by
+
2 KZe 2 1 1
r0 = ,K= , E K = mV 2
EK 4π ∈0 2
1
So, r0 ∝ 9. Distance of closest approach: It is the distance between
EK
the centre of the nucleus and the point from which
If kinetic energy is doubled, the distance of closest the alpha particle approaching directly to the nucleus
approach r0 will become half of its previous value. returns
6. Let the distance of closest approach be do Distance of closest approach
K Ze × q 1  2 Ze 2 
EK =
d0   r=  K  1
4πε 0  
So, a-particles q = 2e Where K is kinetic energy of a-particle
KZe × 2e 1
(EK)a =  ½ r∝
d0 K
For proton q=e As the kinetic energy K of a-particle is doubled,
KZee distance of closest approach will become half of the
(EK)P =  ½ r
d0 previous value i.e.  1
2
( E K )α Or
= 2 1
(E K ) P Limitations of Rutherford nuclear model:
Thus, a proton would require half the initial K.E. of that (See Answer 8)
2
of the a-particle for the same value of d0. n
10. r = 0.53 × 10–10 m
7. • An atom consists of a small and massive core in Z
which entire +ve charge and almost the whole of an For first excited state n = 2
atom is concentrated. This core is called the nucleus.
22
 ½ r = 0.53 × 10–10 = 2.12 × 10–10 m ½
1
• Size of the nucleus is very small as compared to size
of atom. ½ Z Z
v = v0 × = 2.18 × 106 × m/s ½
• Nucleus is surrounded by a suitable number of n n
electrons so that atom as a whole is neutral. ½ For first excited state n = 2, Z = 1 for hydrogen atom
• Electrons revolve around the nucleus in various 1
orbits. The centripetal force required for this \ v = 2.18 × 106 × m/s
2
revolution is provided by the electrostatic attraction
fi v = 1.094 × 106 m/s ½
between the electrons and the nucleus. ½
8. (i) An electron revolving around nucleus is under 2πr 2 × 3.14 × 2.12 × 10 −10

Orbital period = =
continuous acceleration so it should continuously v 1.094 × 106
lose energy and the radii of the orbits will continue = 1.22 × 10–15 sec ½
to decrease. So, the electron will follow a spiral path
11. To resolve this problem, Bohr modified the Rutherford
and finally collapse into the nucleus. So Rutherford’s
model by proposing that the electrons move in orbits of
model cannot explain stability of atom. 1
fixed size and energy. The energy of an electron depends
(ii) As electron can revolve in orbits of all possible radii on the size of the orbit and is lower for smaller orbits.
so it should emit a continuous spectrum. But an atom
Radiation can occur only when the electron jumps from
like hydrogen always gives a line spectrum.
one orbit to another.

Atoms 411
D:\EG_Physics-12_(26-06-2022)\Open_Files\Ch-12\Ch-12
\ 27-Jul-2022 Ved_Goswami Proof-4 Reader’s Sign _______________________ Date __________

12. In the alpha-particle scattering experiment, if a thin mv 2 KZe e


sheet of solid hydrogen is used in place of a gold foil, Centripetal force = =  ½
r r2
then the scattering angle would not be large enough.
2
This is because the mass of hydrogen (1.67 × 10–27 kg) or mv2 = KZe
is less than the mass of incident a-particles (6.64 × 2r
10–27 kg). Thus, the mass of the scattering particle is 1 2 KZe 2
more than the target nucleus (hydrogen). As a result, the K.E. = mv = ...(1) ½
a-particles would not bounce back if solid hydrogen is 2 2r
used in the a-particle scattering experiment. − KZe × e − KZe 2
P.E. = =  ½
13. According to Rutherford’s model, electron is r r
revolving around nucleus in a circular orbit of
KZe 2
radius r. Total energy = E =K.E. + P.E. = –  ½
2r
v
1
K=
          ½ 4πε 0
r
e
+ Ze For Hydrogen Z = 1
− e2
So E=
8πε 0 r
Let mass of electron be m Negative sign shows that it is a bound system. Energy
Let Z be the atomic number. So charge on the nucleus needs to be supplied to remove the electron. ½
Ze

Topic 2. Bohr’s Model of Hydrogen Atom and Hydrogen Spectra


•• Electrons are revolving around nucleus in certain KZe 2
permitted orbits called stationary orbits. Which satisfy mv2 = …(1)
r
the condition.
nh KZe 2
L= , L is angular momentum r= ...(2)
2π mv 2
where n = 1, 2, 3, 4...
n is called quantum number. v
While revolving in these orbits, electrons do not radiate
energy. +Ze e

•• An atom can emit or absorb radiation in the form of


discrete packets called photons only when an electron
jumps from higher to lower orbit or from lower to higher
orbit. According to Bohr’s quantisation condition
Bohr’s theory of hydrogen atom nh
mvr =
Let Z be the atomic number. 2π
So the charge on the nucleus is + Ze. An electron of mass nh
r= ...(3)
m is moving with velocity v in a circular orbit of radius 2πmv
r.
KZe 2 nh
mv 2 Ke Ze From (2) and (3) 2 =
= mv 2πmv
r r2
mv 2
KZe 2 2πKZe 2 Z
= v= ,n∝
r r2 nh n

412 Physics–12
D:\EG_Physics-12_(26-06-2022)\Open_Files\Ch-12\Ch-12
\ 27-Jul-2022 Ved_Goswami Proof-4 Reader’s Sign _______________________ Date __________

Substitute value of v in (3) 1


2π 2 m k 2 e 4 1
nh nh =  2 − 2
r= × h2  n1 n2 
2πm 2πKZe 2
n2 h2 2π 2 m k 2 e 4  1 1
= ...(4) v= 3  2 − 2
4π2 mKZe 2 h  n1 n2 

n2 1 2π 2 m k 2 Ze 2 1 1  c 
i.e., r ∝ n2 or in general r α λ
=  2 − 2 Q λ = v 
Z ch3  n1 n2 
Energy of orbital electron:
1 1 1
− KZee − KZe 2 = R  n 2 − n 2  …(i)
P.E. = = λ  1 2 
r r
From eqn. (1) where R = Rydberg’s constant.
2
1 KZe
K.E. = mv 2 = , For Lyman Series:
2 2r
1 1 1
Total energy = R  2 − 2  where n2 = 2, 3, 4, 5, ...
E = P.E. + K.E. λ 1 n2 

− KZe 2 KZe 2 − KZe 2 This belongs to ultraviolet region of the electromagnetic


= + = spectrum.
r 2r 2r
Bohr ’s Model of the Hydrogen Atom and Hydrogen
Substituting the value of r from eqn. (4)
Spectra:
− KZe 2 1
E= 4π 2 mKZe 2 1 1
2 2
2n h For Lyman Series: = R 2 − 2
λ 1 n2 
− 2π 2 m K 2 Z 2 e 4 Z2 where n = 2, 3, 4, ...
= , E ∝
n2 h2 n2 This falls in ultraviolet region.
For Hydrogen Z = 1, For Balmer Series:
− 2π 2 m K 2 e 4 1 1 1
So E= , = R  2 − 2  where n = 3, 4, 5, 6, ...
n2 h2 λ 2 n2 
for Hydrogen atom This falls in visible region
− 13.6
En = eV For Paschen series:
n2
Origin of different spectral lines of hydrogen spectrum 1 1 1
= R  2 − 2  where n = 4, 5, 6, ...
on the basis of Bohr’s theory. λ 3 n2 
•• According to Bohr’s theory, the energy of an electron in This lies in infrared region.
Hydrogen atom in an orbit is given by
For Brackett series:
− 2π 2 mk 2 e 4
En = , n is quantum number. 1 1 1
n2 h2 = R  2 − 2  where n = 5, 6, 7, ...
λ 4 n2 
When an electron makes a transition from higher energy
level (n2) to lower energy level (n1), the difference of This also falls in infrared region.
energy appears in form of a photon. For Pfund series:
so the energy of emitted photon is given by
1 1 1
hv = E n − E n = R  2 − 2  where n = 6, 7, 8, ...
2 1
λ 5 n2 
2π m k 2 e 4 2π 2 mk 2 e 4
2
Falls in infrared region.
= − +
n22 h 2 n12 h 2

Atoms 413
D:\EG_Physics-12_(26-06-2022)\Open_Files\Ch-12\Ch-12
\ 27-Jul-2022 Ved_Goswami Proof-4 Reader’s Sign _______________________ Date __________

•• Energy level diagram for hydrogen atom:


0 eV n=∞

n=8
n=7
n=6
– 0.54 eV n=5
Pfund series
– 0.85 eV n=4
Brackett
series
–1.5 eV n=3
Paschen
series
–3.4 eV n=2
Balmer
series
–13.6 eV n=1
Lyman
series

EXERCISE 12.2
I. Objective Type Questions (1 Mark) (b) of the electrons colliding with each other.
1. Choose the correct answers from the given options (c) of screening effects.
(MCQs). (d) the force between the nucleus and an electron
(i) Taking Bohr’s radius as a0 = 53 pm, the radius of will no longer be given by Coulomb’s law.
Li++ ion in its ground state, on the basis of Bohr’s (iv) An ionised H-molecule consists of an electron and
model, will be about two protons. The protons are separated by a small
(a) 53 pm (b) 27 pm distance of the order of Å. In the ground state,
(c) 18 pm (d) 13 pm (a) the electron would not move in circular orbits.
(ii) The binding energy of a Hydrogen atom, (b) the energy would be (2)4 times that of h-atom.
considering an electron moving around a fixed (c) the electrons, orbit would go around the
nuclei (proton) is protons.
me 4 (d) the molecule will soon decay in a proton and
B= −
(m = mass of electron) a H-atom.
8n 2 ε 02 h 2
(v) Consider aiming a beam of free electrons towards
If one decides to work in a frame of reference, free protons. When they scatter, an electron and a
where the electron is at rest, the proton would proton cannot combine to produce a H-atom,
be moving around it. By similar arguments, the
(a) because of energy conservation.
binding energy would be
(b) without simultaneously releasing energy in
− Me 4 the form of radiation.
B=  (M = mass of proton)
8n 2 ε 02 h 2 (c) because of momentum conservation.
The last expression is not correct because (d) because of angular momentum conservation.
(a) n would not be integral. (vi) A photon beam of energy 12.1eV is incident on
(b) Bohr-quantisation applies only to electron. a hydrogen atom. The orbit to which electron of
(c) the frame in which the electron is at rest is H-atom be excited is [CBSE S.P. 2019-20]
not inertial. (a) 2nd (b) 3rd (c) 4th (d) 5th
(d) the motion of the proton would not be in 2. Fill in the blanks.
circular orbits, even approximately. (i) From Bohr’s theory, when an electron jumps from
(iii) The simple Bohr’s model cannot be directly higher energy orbit to second orbit, the spectral
applied to calculate the energy levels of an atom lines that occur belong to .................. series.
with many electrons. This is because (ii) When a hydrogen atom is raised from the ground
(a) of the electrons not being subject to central state to an excited state, them P.E. ............... and
force. kinetic energy .................... .

414 Physics–12
D:\EG_Physics-12_(26-06-2022)\Open_Files\Ch-12\Ch-12
\ 27-Jul-2022 Ved_Goswami Proof-4 Reader’s Sign _______________________ Date __________

(iii) The ................. lies in the infrared region of 2. What is the ratio of radii of orbits corresponding to
spectrum. first excited state and ground state in a hydrogen
(iv) According to Bohr ’s atomic model, the atom?[Delhi 2010]
circumference of the electron orbit is always an 3. What is the ionization potential of hydrogen atom?
................ multiple of de-Broglie wavelength. [Delhi 2011]
[AI 2020] 4. When is Ha line of Balmer series in the emission
3. State True or False spectrum of hydrogen atom obtained?[Delhi 2013C]
(i) Band spectrum is produced by the substance in 5. Name the spectral series of hydrogen atom, which be
molecular state. in infrared region.
(ii) The ionisation energy of hydrogen atom is E. 6. What is the angular momentum of an electron in the
When the electron in a hydrogen atom jumps third orbit of an atom?
from the state n = 1 to the state n = 2, the energy 7. When electron in hydrogen atom jumps from energy
5E state ni = 4 to nf = 3, 2, identify the spectral lines to
absorbed by it is . which the emission lines belong. [AI 2013]
4
(iii) The energy of the atom goes on increasing as we 8. The ground state energy of hydrogen atom is –13.6
go to higher excited states. eV. What are P.E. and K.E. of electron in this state?
[Delhi 2010, 2011]
4. Match the Columns
9. What is the value of angular momentum of electron in
Column-I Column-II
the second orbit of Bohr’s model of hydrogen atom?
(i) Speed of an electron (a) Infrared region [CBSE S.P. 2020-21]
revolving in nth orbit
2πKZe 2 III. Short Answer Type Questions-I (2 Marks)
(ii) Angular momentum (b) 10. Write an expression for Bohr’s radius in hydrogen
nh
of the electron atom. [Delhi 2010]
(iii) Paschen series (c) Visible region 11. State postulates for Bohr’s theory of hydrogen atom.
nh 12. The energy of electron in ground state of hydrogen
(iv) Balmer series (d)
2π atom is –13.6 eV. How much energy is required to
For Q. 5 there are two statements labelled as assertion take an electron in this atom from the ground state to
(A) and reason (R). Select the correct answer to these first excited state. [Delhi 2009]
questions from the codes (a), (b), (c) and (d) as given 13. Show that Bohr’s second postulate “The electron
below. revolves around the nucleus only in certain fixed
(a) Both A and R are true and R is the correct orbits without radiating energy can be explained on
explanation of A. the basis of de-Broglie hypothesis of wave nature of
(b) Both A and R are true but R is not the correct electron.Or
explanation of A. Show mathematically how Bohr ’s postulate
(c) A is true but R is false. of quantization of orbital angular momentum
in hydrogen atom is explained by de-Broglie’s
(d) Both A and R are false.
hypothesis. [AI 2016]
5. Assertion (A): Hydrogen atom consists of only one
14. The ground state energy of hydrogen atom is – 13.6
electron but its emission spectrum has many lines.
eV:
Reason (R): Only Lyman series is found in the
(i) What are the potential energy and K.E. of electron
absorption spectrum of hydrogen atom whereas in
is 3rd excited state?
the emission spectrum all the series are found.
(ii) If the electron jumps to the ground state from
II. Very Short Answer Type Questions (1 Mark) the third excited state, calculate the frequency of
1. Consider two different atoms. The electron in each photon emitted. [S.P. 2011]
atom is in an excited state. Is it possible for the 15. Determine the speed of the electron in n = 3 orbit of
electrons to have different energies, but same orbital He+ ion. [NCERT]
angular momentum according to the Bohr’s model? 16. Which level of the doubly ionised lithium Li2+
[NCERT Exemplar] has same energy as the ground state energy of the

Atoms 415
D:\EG_Physics-12_(26-06-2022)\Open_Files\Ch-12\Ch-12
\ 27-Jul-2022 Ved_Goswami Proof-4 Reader’s Sign _______________________ Date __________

hydrogen atom? Compare the orbital radius of the 28. Find out the wavelength of the electron orbiting in
two levels. [NCERT] the first excited state of hydrogen atom.
17. Which state of the triply ionised beryllium Be3+ has [Delhi 2017]
the same orbital radius as that of the ground state of 29. Obtain the expression for the ratio of the de-Broglie
hydrogen?[NCERT] wavelengths associated with the electron orbiting in
18. Calculate the shortest wavelength in the Balmer series the second and third excited states of hydrogen atom.
of hydrogen atom. In which region (IR, visible, UV) [Delhi 2019]
of hydrogen spectrum does this wavelength lie? 30. State Bohr’s quantization condition of angular
[AI 2015, 2016] momentum. Calculate the shortest wavelength of
19. Using Rydberg formula, calculate the longest the Brackett series and state to which part of the
wavelength belonging to Lyman and Balmer series. electromagnetic spectrum does it belong.
In which region of hydrogen spectrum do these [Delhi 2019]
transitions lie? Given R = 1.1 × 107 m–1 31. Derive an expression for the radius of nth Bohr’s orbit
[Foreign 2015] in Hydrogen atom.[CBSE S.P. 2019-20]
20. The shortest wave-length limit for the Lyman series [AI 2020]
of hydrogen spectrum is 913.4 Å. Calculate short 32. Energy of electron in first excited state in Hydrogen
wavelength limit for Balmer series of hydrogen atom is -3.4eV. Find KE and PE of electron in the
spectrum.[S.P. 2015] ground state.[CBSE S.P. 2019-20]
21. Using the Rydberg formula, calculate the wavelengths 33. Using Bohr’s atomic model, derive the expression for
of the first four lines in the Lyman series of the the velocity of electron revolving in the nth orbit of
hydrogen spectrum. [NCERT] hydrogen atom.[AI 2020]
22. In the ground state of hydrogen atom, its Bohr radius IV. Short Answer Type Questions-II (3 Marks)
is 5.3 × 10–11 m. The atom is excited such that the 34. Explain the origin of different spectral lines of
radius becomes 21.2 × 10–11 m. Find the value of hydrogen spectrum on the basis of Bohr’s theory.
principal quantum number and total energy of the [Delhi 2015]
atom in excited state. [Delhi 2013C] 35. Use Bohr model of hydrogen atom to calculate the
23. The energy levels of an atom are shown in figure. speed of the electron in the first excited state.
Which transition corresponds to emission of [AI 2016]
radiation of (i) maximum wavelength (ii) minimum 36. Find the relation between the three wavelengths l1, l2
wavelength?[Delhi 2013] and l3 from the energy level diagram shown below.
0 eV [AI 2016]
A
– 2 eV C
B C
– 4.5 eV

D
– 10 eV
λ1 λ3
24. The electron in hydrogen atom is initially in the B
third excited state. What is the maximum number of
λ2
spectral lines which can be emitted, when it finally
A
moves to the ground state? [Delhi 2013 C]
37. (a) The energy levels of an atom are shown in figure.
25. Calculate the de-Broglie wavelength of the electron
Which of them will result in the emission of a
orbiting in the n = 2 state of hydrogen atom.
photon of wavelength 275 nm?
[AI 2016]
26. Define ionization energy. How would the ionization (b) Which transition corresponds to emission of
energy change when electron in hydrogen atom is radiation of maximum wavelength?[Delhi 2011]
replaced by a particle of mass 200 times that of the A
0 eV
electron but having the same charge? [AI 2016] B C
– 2 eV
– 4.5 eV
27. Find out the wavelength of the electron orbiting in
the ground state of hydrogen atom. [Delhi 2017] D
– 10 eV

416 Physics–12
D:\EG_Physics-12_(26-06-2022)\Open_Files\Ch-12\Ch-12
\ 27-Jul-2022 Ved_Goswami Proof-4 Reader’s Sign _______________________ Date __________

38. Ground state energy of hydrogen atom is – 13.6 eV Calculate the wavelength of the first member of
(i) What is the K.E. of an electron in the second Paschen series and first member of Balmer series.
excited state? [Delhi 2014]
(ii) What is the P.E. of an electron in the third excited 44. It is found experimentally that 13.6 eV energy is
state? required to separate a hydrogen atom into a proton and
(iii) If the electron jumps to the ground state from the an electron. Compute the orbital radius and velocity
third excited state, calculate the wavelength of the of electron in a hydrogen atom. [NCERT]
photon emitted. [AI 2008] 45. (a) State Bohr’s postulate to define stable orbits in
39. A hydrogen atom initially in the ground level absorbs hydrogen atom. How does de Broglie’s hypothesis
a photon, which excites it to the n = 4 level. Determine explain the stability of these orbits?
the wavelength and frequency of photon. [NCERT] (b) A hydrogen atom initially in the ground
[AI 2014 C] state absorbs a photon which excites it to the
n = 4 level. Estimate the frequency of the photon.
40. When an electron in hydrogen atom jumps from the [Delhi 2018]
third excited state to the ground state, how would the 46. A monochromatic radiation of wavelength 975 Å
de-Broglie wavelength associated with the electron excites the hydrogen atom from its ground state to a
change? Justify your answer. [AI 2015] higher state. How many different spectral lines are
41. Given the ground state energy E0 = – 13.6 eV and possible in the resulting spectrum? Which transition
Bohr radius a0 = 0.53 Å. Find out how de-Broglie corresponds to the longest wavelength amongst them?
wavelength associated with the electron orbiting in  3
the ground state would change when it jumps into the [CBSE S.P. 2018-19]
first excited state. [AI 2015] 47. Derive an expression for the frequency of radiation
emitted when a hydrogen atom de-excites from level
42. The ground state energy of hydrogen atom is – 13.6
n to level (n – 1). Also show that for large values of
eV. If an electron makes a transition from an energy
n, this frequency equals to classical frequency of
level – 0.85 eV to –3.4 eV, calculate the wavelength revolution of an electron. [CBSE S.P. 2020-21]
of spectral line emitted. To which series of hydrogen
V. Long Answer Type Questions (5 Marks)
spectrum does this wavelength belong?[Delhi 2012]
48. State the basic postulates of Bohr’s theory of atomic
43. A 12.5 eV beam of electrons is used to bombard spectra. Hence obtain an expression for radius of orbit
gaseous hydrogen at room temperature. Upto which and the energy of orbital electron in hydrogen atom.
energy level the hydrogen atoms would be excited? [Delhi 2013, 2015 C]

Answers 12.2
I. Objective Type Questions 2. As r ∝ n2, for ground state n = 1
1. (i) (c) (ii) (c) (iii) (a) (iv) (a) and (c) For first excited state n = 2,
(v) (a) and (b) (vi) (b) 2 2
2. (i) Balmer (ii) increase, decrease r2 n22  n2   2 4
So = 2 =  =  =
(iii) Paschen series (iv) integral r1 n1  n1   1  1
3. (i) True (ii) False (iii) True 3. 13.6 V.
4. (i)-(b) (ii)-(d) (iii)-(a) (iv)-(c)
4. Ha line of Balmer series is obtained when an electron
5. (b)
jumps to 2nd orbit from 3rd orbit of hydrogen atom.
II. Very Short Answer Type Questions
5. Paschen Series, Brackett Series and Pfund Series lie in
− 13.6 infrared region.
1. E n = eV
n2 6. n = 3
So the electrons which have different energies will be
in different levels so the value of n will be different. As nh
Angular momentum L = mvr =
angular momentum 2π
nh 3 × 6.6 × 10 −34 × 7
L = mvr = So L= =3.15 ×10 −34 Js
2π 2 × 22
So L will also be different.
Atoms 417
D:\EG_Physics-12_(26-06-2022)\Open_Files\Ch-12\Ch-12
\ 27-Jul-2022 Ved_Goswami Proof-4 Reader’s Sign _______________________ Date __________

7. ni = 4 to nf = 3 Paschen series 13. According to de-Broglie hypothesis, an electron in


ni = 4 to nf = 2 Balmer series motion is associated with a wave character. A circular
8. Total energy = E = – 13.6 eV orbit of an electron can be taken to be stationary energy
Kinetic energy EK = 13.6 eV state only if the circumference of the orbit is an integral
Potential energy EP = – 2EK = – 2 × 13.6 = –27.2 eV multiple of de-Broglie wavelength. ½
h 2pr = nl
9. L = n
2π       2pr = n h  ½
Here given n = 2 mv
 h h nh
\ L = 2  = mvr =
 2π  π 2π
nh
       L =  ½
n2 h2  2π
10.  r = 2 , 1
 4π mKZe 2 
n is quantum number
h is Planck’s constant
m is mass of an electron Nucleus
Z = 1 for hydrogen λ    ½
e is charge on an electron
r
1
K=  1
4πε 0
11. (i) Bohr’s Quantisation Condition: The electrons can
revolve without radiating energy in the orbit which − 13.6
14. (i) En = eV
satisfies the condition that angular momentum of all n2
electron is an integral multiple of In third excited state n = 4
− 13.6 eV
nh So E4 = = − 0.85 eV  ½
L=
 1 42

KE = – (T.E.) = 0.85 eV
where n = 1, 2, 3, ...
PE = 2TE = 2 × (– 0.85) = –1.7 eV ½
These orbits are called stationary orbits.
(ii) hv = E4 – E1 ½
(ii) Frequency conditions: An atom can emit or absorb
E 4 − E1
radiation in the form of discrete packets called v=
photons, only when an electron jumps from higher h
to lower orbit or from lower to higher orbit. ½ [ − 0.85 − ( − 13.6)]
= −34
× 1.6 × 10 −19
So if E1 and E2 are the energies associated with the 6.6 × 10
permitted/stationary orbits then energy emitted or = 3 × 1015 Hz. ½
absorbed is given by 15. n = 3, Z = 2
hv = E2 – E1, E1 is the energy in the initial level, E2 2π KZe 2
is the energy in final level. ½ Using v=  1
nh
− 13.6
12. As En = eV  ½ 2 × 3.14 × 9 × 109 × 2 × (1.6 × 10 −19 ) 2
n2 =
3 × 6.6 × 10 −34
Energy in ground state E1 = – 13.6 eV. = 1.46 × 106 m/s 1
Energy is first excited state, i.e., n = 2 is
2π 2 mK 2 e 4 Z 2
− 13.6 16. En = −  ½
    E2 = = − 3.4 eV  ½ h2 n2
22
Z2
\ Required energy = E2 – E1
\ En ∝ Given En(Li2+) = E1(H) ½
n2

    = – 3.4 – (– 13.6) = 10.2 eV 1

418 Physics–12
D:\EG_Physics-12_(26-06-2022)\Open_Files\Ch-12\Ch-12
\ 27-Jul-2022 Ved_Goswami Proof-4 Reader’s Sign _______________________ Date __________

 Z2   Z2  1 1 1 1 1 5
    2  =  2  ½ = R 2 − 2 = R −  = R ½
λ 2 3   4 9  36
 n  Li2+  n H
36 36
32 12 l= = = 6.545 × 10 −7 m
or = 5R 5 × 1.1 × 107
n2 12
= 655 nm

\ n = 3 ½
This lies in the visible region. ½
n2 h2 20. Lyman series for shortest l, nf = 1, ni = ∞ ½
17.   rn =  1
4π2 m KZe 2
1 1 1 1 1 
2 = R 2 − 2 = R  2 − 2 = R ½
or rn ∝ n  λ LS  n f ni  1 ∞ 
Z
1
  ( rn )Be3+ = r1(H) lLS = For Balmer series nf = 2, ni = ∞ ½
R
 n2   n2  2 2
1
  =   or n = 1  ½
1 1  R
= R  2 − 2  = , λ BS =
4
 Z  Be3+  Z H 4 12 λ BS 2 ∞  4 R
\ n = 2 ½ λ LS 1 R 1
hc = × = ⇒ λ BS = 4 λ LS
18. As E = hv =  ½ λ BS R 4 4
λ
lBS = 4 × 913.4 = 3653.6Å ½
So the shortest wavelength will correspond to maximum
energy difference and that will happen when the 1 1 1
electron jumps from n1 = ∞ to nf = 2 ½ 21. = R 2 − 2
λ  n f ni 
1 1 1 For Lyman series nf = 1, ni = 2, 3, 4, 5, ...
Using = R  2 − 2 ½
λ  n f ni  1 1 1
So = R 2 − 2 ½
1 1 1 R λ 1 ni 
= R  2 − 2=
λS  2 
∞  4 1  ni2 − 1 ni2
= R   or λ =  ½
λ
( )
2
4 4  ni  R ni2 − 1
lS = = = 3.637 × 10 −7
R 1.1 × 107
22
= 3637 Å ½ Let ni = 2, λ1 =
1.1 × 107 (22 − 1)
This wavelength falls is ultraviolet region.
4
1 1 1 = = 1216 Å 
19. As = R 2 − 2 1.1 × 107 × 3
λ  n f ni 
32
Let ni = 3, λ 2 =
hc 1.1 × 107 (32 − 1)
  E = hv =
λ 9
The longest wavelength corresponds to a transition with = = 1026 Å  ½
8 × 1.1 × 107
minimum energy difference.
For Lyman series : nf = 1, ni = 2 42
ni = 4, λ 3 =
1 1 1 3 1.1 × 107 (42 − 1)
= R  2 − 2= R ½
λ 1 2  4 16
= = 972.8 Å
4 4 15 × 1.1 × 107
l= = = 1.21 ×10 −7 m
3R 3 × 1.1 × 107 52
ni = 5, λ 4 =
= 121 nm 1.1 × 107 (52 − 1)
This falls in ultraviolet region ½ 25
= = 950 Å  ½
For Balmer series nf = 2, ni = 3 24 × (1.1 × 107 )

Atoms 419
D:\EG_Physics-12_(26-06-2022)\Open_Files\Ch-12\Ch-12
\ 27-Jul-2022 Ved_Goswami Proof-4 Reader’s Sign _______________________ Date __________

22. Here, r1 = 5.3 × 10–11 m; n1 = 1 26. Ionization energy: The amount of energy required to
As r ∝ n2 completely knock out an electron out of the atom.
2 Ionization energy = E∞ – E1
 n2  r2 21.2 × 10 −11

\  n  = r = = 4 1 For Hydrogen atom
1 1 5.3 × 10 −11 Ionisation energy = 0 – (– 13.6)
n2 = 2n1 = 2 × 1 = 2 = 13.6 eV ½
E1 − 13.6 As E∞ = 0 so ionisation energy = – E1

E2 = 2
= = − 3.4 eV  1
 (n2 ) 22 − 2π 2 mK 2 e 4
E1 =  ½
23. For transition A, Energy of emitted photon h2
= 0 – (– 2) = 2 eV i.e., Ionisation energy ∝ m
For transition B, Energy of emitted photon Let New ionisation energy = E′
= 0 – ( – 4.5) = 4.5 eV ½ Original ionisation energy = E
For transition C, Energy of emitted photon El ml 200 me
= = = 200 ½
= – 2 – (– 4.5) = 2.5 eV E m me
For transition D, Energy of emitted photon E′ = 200E = 200 × 13.6 = 272.00 eV ½
– 2 – (– 10) = 8 eV ½ 27. According to de Broglie hypothesis
hc 1 The circumference of the orbit must be an integral
As energy emitted, E = ∝ multiple of de Broglie wavelength l,
λ λ
\ nl = 2pr ½
\ for maximum wavelength, E should be minimum.
2πr
Transition A for which energy difference E is l= , for ground state n = 1
minimum corresponds to emission of radiation of max. n
wavelength. ½ l = 2p × r = 2 × 3.14 × 0.53 × 10–10 ½
Similarly, transition D for which energy diff. E is = 3.32 × 10 m = 3.32Å
–10
1
minimum, corresponds to emission of radiation of 28. According to de Broglie hypothesis
minimum wavelength. ½ nl = 2pr ½
24. For third excited state, n = 4 and for ground state n = 1 2πr
l=  ...(1),
Hence, the possible transitions are n
ni = 4 to nf = 3, 2, 1 i.e., three (3) ½ for first excited state n = 2
ni = 3 to nf = 2, 1 i.e., two (2) ½  n2 
rn = n2r1  ½ Q rn ∝ 
ni = 2 to nf = 1 i.e., one (1) ½  Z
Hence, total number of transitions 2×3.14×2 ×0.53×102 –10

= 3 + 2 + 1 = 6 ½ l=  [Using (1)]
2
25. Energy of an electron in Hydrogen atom = 6.64 × 10–10 m = 6.64Å ½
− 13.6 (Alternative method)
En = eV
n2 In the first excited state n = 2
− 13.6 eV − 13.6 −13.6
E2 = = eV = − 3.4 eV  ½ En =  ½
22 4 n2
h –13.6
De Broglie wavelength l =  ½ E2 = = – 3.4 eV
2mE 4
h
6.6 ×10 − 34 l=  ½
= 2mE
2 × 9.1 × 10 −31 × 3.4 × 1.6 × 10 −19 6.6 × 10 –34
=
6.6 × 10 −34 2×9.1×10 –31 ×3.4×1.6×10 –19
= −25
= .663 × 10 −9
9.95 × 10 6.6×10 –34
= = 6.64 × 10–10 m 1
= 6.63 × 10–10 m 1 9.95×10 –25

420 Physics–12
D:\EG_Physics-12_(26-06-2022)\Open_Files\Ch-12\Ch-12
\ 27-Jul-2022 Ved_Goswami Proof-4 Reader’s Sign _______________________ Date __________

 1 h
1 1 As mvnrn = n ⋅ 
½
29. = RZ2  2 − 2   ½ 2π
λ  n1 n2 
nh
1  1 1

v= put in (i)
i.e. = R  2 − 2  [Q Z = 1 for hydrogen] 2πmrn
λ  n1 n2 
n2 h2 1 e2
When; n1 = 1, n2 = 3 (second excited state)
m· 2 2 2 =
4π m rn 4πε 0 rn
1 1 1
= R  2 − 2  ε0 n2 h2
λ1 1 3  fi
rn = 
½
πme 2
1  1  8R

fi = R 1 −  = ...(1) ½ 32. Energy of electron in n = 2 is –3.4 eV
λ1  9 9
x x
When; n1 = 1, n2 = 4 (Third excited state) En = 2 fi –3.4 eV = 2
n 2
1
= R  1 − 1  fi energy in ground state = –13.6 eV 1
λ2  12 42  \ energy in ground state = –13.6 eV
1  1  15R KE = –TE = +13.6 eV ½

fi = R 1 −  = ...(2) ½
λ2  16  16 PE = 2TE = – 2 × 13.6 eV = –27.2 eV 
½
Divide (2) by (1), we get 33. Bohr’s theory of hydrogen atom
λ1 135 Let Z be the atomic number.
=  ½
λ2 128 So the charge on the nucleus is + Ze. An electron of
mass m is moving with velocity v in a circular orbit of
30. According to Bohr’s quantisation, the electrons revolve
radius r.
around the nucleus only in those orbits for which the
h mv 2 Ke Ze
angular momentum is the integral multiple of =  ½
2π r r2
nh mv 2 KZe 2
L=  ½ =
2π r r2
In Brackett series n2 = ∞, (for Shortest wavelength) KZe 2
1 mv2 = …(1)
 1 1 R r
= RHZ2  2 −  = H  ½
λ 4 ∞ 16
KZe 2
r= ...(2)
16 mv 2

fi l= = 14.58 × 10–7m 1
RH According to Bohr’s quantisation condition
This wavelength belongs to the infra-red region. nh
mvr =  ½
31. Vn 2π
electron
nh
r= ...(3)
2πmv
+e rn From (2) and (3)
KZe 2 nh
=
mv 2 2πmv
centripetal force = electrostatic attraction 2πKZe 2 Z
v= ,n∝  1
mvn2 1 e2 nh n
=  ½
rn 4πε 0 rn2 34. Refer theory topic No. 2 under the heading ‘Origin of
1 e 2 different spectral lines of hydrogen spectrum on the
mvn2 = ...( )
i½ basis of Bohr’s theory’
4πε 0 rn

Atoms 421
D:\EG_Physics-12_(26-06-2022)\Open_Files\Ch-12\Ch-12
\ 27-Jul-2022 Ved_Goswami Proof-4 Reader’s Sign _______________________ Date __________

35. According to Bohr’s model of hydrogen atom For the first excited state n = 2
(i) Entire positive charge and almost entire mass of 2πKe 2 v1
an atom is concentrated in a small volume called v2 = =
2h 2
nucleus.
v2 1 v1 c 
(ii) Electrons are revolving around nucleus in an orbit    = ; v2 = ; v1 = ½
called stationary orbit, which satisfy the condition v1 2 2 274
that angular momentum L is an integral multiple of hc
h nh 36. EC – EB = ...(1) ½
i.e.; L = mvr = λ1
2π 2π
hc
(iii) Electron can jump from outer orbit to inner orbit EB – EA = ...(2) ½
or vice versa by emitting or absorbing a packet of λ2
energy hc
E = hv = Ef – Ei ½ EC – EA = ...(3) ½
λ3
Expression for speed of the electron:
Adding eqn. (1) and (2),
For hydrogen atom Z = 1
 1 1
The necessary centripetal force for the electron to move EC – EA = hc  +  ...(4) ½
in a circular path around nucleus is provided by the  λ1 λ 2 
Coulombian force between nucleus and electron
hc
Comparing equ. (3) and (4),
λ3
 1 1 1 1 1
= hc  +  or = + 
1
 λ1 λ 2  λ 3 λ1 λ 2
37. (a) Energy of the emitted photon
hc 6.6 × 10 −34 × 3 × 108 
E = hv = = J ½
λ 275 ×10 −9

mv 2 Ke 2 6.6 × 10 −34 × 3 × 108


  = eV = 4.5 eV  ½
    r = r 2  ½ 275 ×10 −9 × 1.6 × 10 −19
Ke 2 Transition B corresponds to photon of energy

or mv2 = ...(1) = 0 –(–4.5) = 4.5 eV ½
r
So transition B will result in the emission of a photon
Ke 2 of l = 275 nm. ½
or r= ... (2)
mv 2 hc 1
According to Bohr’s quantisation condition (b) E = hv = i.e., E ∝  ½
λ λ
nh
mvr = , n is principal quantum number So minimum energy will corresponds to radiation
2p of maximum wavelength.
nh For transition A, energy is minimum thus wavelength
r= ...(3) ½
mv2p is maximum. ½
Comparing eqn. (2) and (3) E
38. E1 = − 13.6 eV, E n = 21  ½
Ke 2 nh n
2 = (i) For second excited state n = 3
mv 2pmv
2pKe 2  E − 13.6
So v= 1 So E3 = 21 = = − 1.51 eV  ½
nh 3 9
1 Kinetic energy in the second excited state
or v ∝ , for ground state n = 1
n = – E3 = 1.51 eV ½
2pKe 2 E − 13.6
v1 = (ii) E4 = 21 = = − 0.85 eV
h 4 16

422 Physics–12
D:\EG_Physics-12_(26-06-2022)\Open_Files\Ch-12\Ch-12
\ 27-Jul-2022 Ved_Goswami Proof-4 Reader’s Sign _______________________ Date __________

P.E. in 3rd excited state = 2E4 For ground state n = 1


= 2 × (– 0.85) = – 1.7 eV ½ For First excited state n = 2
hc 13.6
(iii) E = hv = = E 4 − E1 En = − eV  ½
λ n2
hc 13.6
l= E1 = − 2 eV = –13.6 eV ½
E 4 − E1 1
[ − 0.85 − ( − 13.6)] − 13.6 E2 1
= × 1.6 × 10 −19 E2 = eV , So =  ½
6.6 × 10 −34
4 E1 4

6.6 × 10 −34 × 3 × 108 λ1 E2 1


= = 970 Å  1 = = ⇒ l2 =2l1 1
12.75 × 1.6 × 10 −19 λ2 E1 2
− 13.6 42. As energy of the photon emitted is given by
39. As En = eV hc
n2 E = hv = = Ei − E f
λ
For ground state n = 1, E1 = –13.6 eV
= – 0.85 – (– 3.4)
− 13.6   = 2.55 eV ½
For n = 4, E4 = = − 0.85 eV  ½
42 −34 8
hc 6.6 × 10 × 3 × 10
Ei + hv = Ef l= =
Ei − E f 2.55 × 1.6 × 10−19
hc
= hv = E f − E i = – 0.85 – (– 13.6)   = 4.852 × 10–7 m ½
λ
= 12.75 eV ½ − 13.6
As En = eV  ½
−34 8 n2
hc 6.6 × 10 × 3 × 10
l= = For ground state n = 1
E f − E i 12.75 × 1.6 × 10 −19
− 13.6 − 13.6
= 0.975 × 10–7 m = 975 Å 1
–0.85 = 2
⇒ ni2 = = 16
ni 0.85
8
c 3 × 10   ni = 4 ½
v= =
λ 0.975 × 10 −7 − 13.6 13.6
and –3.4 = 2
⇒ n 2f = =4
= 3.077 × 1015 Hz 1 nf 3.4
40. For third excited state n = 4 nf = 2 ½
For ground state n = 1 The electron transited from n = 4 to n = 2
13.6 So, it belongs to second line of Balmer series.
En = − 2 eV  ½
n It lies in visible region. ½
So E1 = –13.6 eV 43. Energy in the ground state = – 13.6 eV
13.6 Total energy on bombardment = –13.6 + 12.5
E4 = − 2
4 = – 1.1 eV ½
E4 1 h h − 13.6
= 2 ,λ= = 
½ As En = eV
E1 4 p 2mE n2
λ1 − 13.6
E4 –1.1 = eV
So = 
1 n2
λ4 E1
13.6
n2 = = 12.36
λ1 E4 1 1.1
So = = or λ 4 = 4λ1  1
λ4 E1 4 1
n = (12.36) 2 = 3.51  1
41. As de-Broglie wavelength
So electron will be excited to n = 3 i.e., 3rd level of
h h
l= = 
½ energy.
p 2mE
Paschen Series : nf = 3, ni = 4, 5, 6, ...

Atoms 423
D:\EG_Physics-12_(26-06-2022)\Open_Files\Ch-12\Ch-12
\ 27-Jul-2022 Ved_Goswami Proof-4 Reader’s Sign _______________________ Date __________

So the first member of Paschen series of radius r, total distance covered by electron
ni = 4, nf = 3 = circumference = 2pr.
hc For the permissible orbit
−1 1 
hv = = E 4 − E 3 = + 13.6  2 + 2  2pr = nl ½
λ 4 3 
2πr
= 0.66 eV 
½ or = n
−34 8 λ
hc 6.6 × 10 × 3 × 10 h
l= −19
= l=
0.66 × 1.6 × 10 0.66 × 1.6 × 10 −19 p
= 18.75 × 10–7 m. ½ p = mv
For Balmer series : nf = 2, ni = 3, 4, 5, ... 2prp
\ = n
For the first member of Balmer series h
nf = 2, ni = 3 nh
or pr =
− 13.6 − 13.6 2p
E3 – E2 = − = – 1.51 + 3.4 nh
32 22 or mvr =
= 1.89 eV 2p
nh 
hc 6.6 × 10 − 34 × 3 × 108 or L= 1
l= = 2p
E3 − E 2 1.89 × 1.6 × 10 −19
This is the expression given by Bohr.
= 6.547 × 10–7 m ½ 1  1 1
44. Here, total energy of electron (b) = R 2 − 2
λ  n1 n2 
E = – 13.6 eV = – 13.6 × 1.6 × 10–19 J Here given n1 = 1, n2 = 4

= – 2.2 × 10–18 J ½ 1 1 1 
\ = R 2 − 2
− e2 λ 1 4 

From E = 
½
8π ∈0 r 1  1
or = R 1 − 
−e 2
− 9 × 10 (1.6 × 10 ) 9 −19 2 λ  16 
r= =
2(4π ∈0 )E 2( − 2.2 × 10 −18 ) 1 15
or = R×  ½
= 5.3 × 10–11 m ½ λ 16
e c 15
Velocity, v = 
½ But = cR ×
4π ∈0 mr λ 16
Where m = 9.1 × 10–31 kg c
As c = nl or =n
1.6 × 10 −19 λ

\ v=
9.1 × 10 −31 × 5.3 × 10 −11 15

\ n = cR ×
9 × 109 16
15
= 2.2 × 106 m/s 1 or n = 3 × 108 × 1.09 × 107 ×
16
45. (a) Bohr’s second postulates define these stable orbits.
or    ν = 3 × 1015 Hz

1
According to this postulate an electron can revolve
around the nucleus in certain discrete, non radiating Alternate (as per NCERT)
orbits in which the angular momentum of an electron −13.6
En = eV
h nh n2
is an integral multiple of i.e. L = .
2p 2p for n1 = 1, E1 = –13.6 eV ½
De Broglie’s hypothesis explains the stability of −13.6
these orbits that the quantised electron orbits and for n2 = 4, E2 = eV ½
16
energy states are due to the wave nature of the The amount of energy absorbed by the photon is
electron and only resonant standing wave can persist.  15 
For an electron revolving in nth circular orbit E = E2 – E1 = 13.6 ×   eV
 16 

424 Physics–12
D:\EG_Physics-12_(26-06-2022)\Open_Files\Ch-12\Ch-12
\ 27-Jul-2022 Ved_Goswami Proof-4 Reader’s Sign _______________________ Date __________


15
= 13.6 × × 1.6 × 10–19 J me 4 ( 2n − 1)
16 = 3
h
or E = 2.04 × 10–18 J 1 ( 4π )3 ∈02   n2 ( n − 1)2

hn = 2.04 × 10–18
For large n, we can write (2n – 1) = 2n and (n – 1) = n.
2.04 × 10 −18
    n= me 4
6.6 × 10 −34 \ v= 3 
…(iv)
3  h 3
15 32π ∈02   n
   ν = 3.1 × 10 Hz  1 2π
46. Energy corresponding to the given wavelength: 48. Basic postulates of Bohr’s Theory of Atomic Spectra
12400 12400 1. Electrons are revolving around nucleus in certain
E (in eV) = = = 12.71 eV 1

975 permitted orbits called stationary orbits. Which
λ(in A) satisfy the conditions
The excited state:
nh
En – E1 = 12. 71 L= , L is angular momentum
2p
−13.6 where n = 1, 2, 3, 4, ....
+ 13.6 = 12.71 ½
n2 n is called quantum number.

∴ n = 3.9 ≈ 4 ½
While revolving in there orbits, electrons do not
n(n − 1)
Total no. of spectral lines emitted: =6 ½ radiate energy. 1
2 2. An atom can emit or absorb radiation in the form
Longest wavelength will correspond to the transition of discrete packets called photons only when an
n = 4 to n = 3 ½ electron jumps from higher to lower orbit or from
[CBSE Marking Scheme 2018-19] lower to higher orbit. 1
47. It is given that a hydrogen atom de-excites from an Bohr’s Theory of Hydrogen Atom
upper level (n) to a lower level (n–1). Let Z be the atomic number.
We have the relation for energy (E1) of radiation at level So the charge on the nucleus is +Ze
n as: v
4
hme
 1
E1 = hv1 = ×  2  …(i) e
h n  3

( 4π )3 ∈02   + Ze



Now, the relation for energy (E2) of radiation at level
An electron of mass m is moving with velocity v in
(n – 1) is given as:
a circular orbit of radius r.
hme 4 1
E2 = hv2 = × …(ii) mv 2 KeZe KZe 2
( 4π )3 ∈02 
h
3
( n − 1) 2

r
= =
 r2 r2
2π 
Where, 2
mv2 = KZe ...(1)
v2 = Frequency of radiation at level (n–1) r
Energy (E) released as a result of de-excitation:
KZe 2
E = E2 – E1 ⇒ hv = E2 – E1 …(iii) r =  ...(2)
Where, mv 2
v = Frequency of radiation emitted According to Bohr’s quantisation condition
Putting values from equations (i) and (ii) in equation nh nh
mvr = ⇒r= ...(3) 1
(iii), we get: 2p 2pmv
me 4  1 1 From (2) and (3)
v=  − 
3
3 2  h   ( n − 1)
2
n 2  KZe 2 nh
( 4π ) ∈0    2
=
2π      mv 2πmv

Atoms 425
D:\EG_Physics-12_(26-06-2022)\Open_Files\Ch-12\Ch-12
\ 27-Jul-2022 Ved_Goswami Proof-4 Reader’s Sign _______________________ Date __________

KZe 2 2p 2
Total energy
= 2 pKZe
v = E = P.E. + K.E
nh nh
− KZe 2 KZe 2 − KZe 2
Substitute value of v in (2) = + =  ½
r 2r 2r
nh nh Substituting the value of r from eqn. (4)
r= ×
2 pm 2 pKZe 2 − KZe 2
E= 2 2
4π 2 mKZe 2
n2 h2 2n h
= ...(4) 1
4p 2 mKZe 2 − 2π 2 m K 2 Z 2 e 4
=
i.e., r ∝ n2 n2 h2
Energy of orbital electron: For Hydrogen Z = 1
− 2π 2 m K 2 e 4
− KZee − KZe 2 So E=  ½
P.E. = = n2 h2
r r For Hydrogen atom
2
− 13.6
From eqn. (1), K.E. = 1 mv 2 = KZe En = eV
2 2r n2

Case Based Questions


I. Bohr's model explains the spectral lines of hydrogen Ans. 1. In transition-II
atomic emission spectrum. While the electron of E2 = –3.4 eV, E4 = – 0.85 eV
the atom remains in the ground state, its energy is \ DE = (– 0.85) – (– 3.4) = 2.55 eV
unchanged. When the atom absorbs one or more
quanta of energy, the electrons move from the ground hc
Since DE =
state orbit to an excited state orbit that is further away. λ
hc 6.6 × 10 −34 × 3 × 108

\ l= = = 487 nm

∆E 2.55
2. In transition VI, photon of Balmer series is
Principal quantum number

4
III absorbed.
3 3. Transition V will occur.
II. Hydrogen spectrum consists of discrete bright lines
II IV V VI in a dark background and it is specifically known
2 as hydrogen emission spectrum. There is one more
type of hydrogen spectrum that exists where we get
dark lines on the bright background, it is known as
absorption spectrum. Balmer found an empirical
I formula by the observation of a small part of their
1 1  1 1
spectrum and it is represented by = R  2 − 2 
λ 2 n 
The given figure shows an energy level diagram of where n = 3, 4, 5, .......
the hydrogen atom. Several transitions are marked For Lyman series, the emission is from first state to
as I, II, III and so on. The diagram is only indicative nth state, for Paschen series, it is from third state to
not to scale. nth state, for Brackett series, it is from fourth state to
1. What is the wavelength of radiation involved in nth state and for Pfund series, it is from fifth state to
transition II? nth state.
2. In which transition is photon of Balmer series 1. How many spectral lines are there in hydrogen
absorbed? atom?
3. Which transition will occur when a hydrogen 2. Which series of hydrogen spectrum corresponds
atom is irradiated with radiation of wavelength to ultra violet region?
103 mm?
426 Physics–12
D:\EG_Physics-12_(26-06-2022)\Open_Files\Ch-12\Ch-12
\ 27-Jul-2022 Ved_Goswami Proof-4 Reader’s Sign _______________________ Date __________

3. If hydrogen atom is excited from ground state 1


to another state with principal quantum number where R is Rydberg’s constant and is wave number.
λ
equal to 4, then what will be the number of spectral
• When electron jumps from any higher energy level to
lines in the emission spectra?
that lower orbit, the distribution of energy wavelength
Ans. 1. Number of spectral lines in hydrogen atom is
wise is called spectrum.
infinity (∞).
2. Lyman series lies in ultraviolet region. • When electron jumps from any higher energy level
3. The number of spectral lines will be 6. n2 = 2, 3, 4, ... to first orbit n1 = 1, the sequence of
spectral lines obtained Lyman series.
III. Spectral Lines of Hydrogen: Bohr’s model explains
For Lyman series,
the spectral lines of the hydrogen atomic emission
spectrum. While the electron of the atom remains 1 1 1 
= R − 2
in the ground state, its energy is unchanged. When λ 1 n2 
the atom absorbs any quanta of energy, the electron • When electron jumps from any higher energy level n2
moves from the ground state orbit to an excited state = 3, 4, 5... to lower energy level n1 = 2, the sequence
orbit that is further away. Energy levels are denoted of spectral lines obtained is called Balmer series.
with the variable n. The ground state is n = 1, the first
excited state is n = 2, and so on. The energy that is 1 1 1 
= R  − 2
gained by the atom is equal to the difference in energy λ  4 n2 
(DE) between the two energy levels. When the atom • When electron jumps from any higher energy level
relaxes back to a lower energy state, it releases energy n2 = 4, 5, 6 ... to lower energy n1 = 3, the sequence of
(DE) that is again equal to the difference in energy of spectral lines obtained is called Paschen series.
the two orbits. For Paschen series,
n=2 n=3 1 1 1 
= R  − 2
λ  9 n2 
n=1
• When electron jumps from any higher energy level n2
= 5, 6, 7... to lower energy level n1 = 4, the sequence
of spectral lines obtained is called Brackett series.
E = h
For Brackett series
The change in energy, DE, then translates to light of 1 1 1
= R  − 2
a particular frequency being emitted according to the λ 16 n2 
equation E = hn. • When electron jumps from any higher energy level n2
• Let E1 and E2 be the energies of an electron in orbit n1 = 6, 7, 8 ... to lower energy level n1 = 5, the sequence
and n2 respectively. When electron jumps from orbit of spectral lines obtained is called Pfund series.
of higher energy (n2) to orbit of lower energy (n1) it For Pfund series,
emits energy of frequency ν in the form of photon.
1 1 1
hν = E2 – E1 = R  − 2
λ  25 n2 
c 2 p 2 k 2 Z2 e 4 m
where, ν= E2 = – Above series are shown in figure.
λ n22 h 2 n =∞
n=7
2 p 2 k 2 Z2 e 4 m n=6
and E1 = – n=5
n12 h 2 Pfund
series n=4
Brackett
2p k Z e m  1
2 2 2 4
1 series
∴ hν = 2  2 − 2 n=3
h  n1 n2  Paschen
series
n=2
On solving, we get Balmer
1 1 1
series
= R 2 − 2 n=1
λ  n1 n2  Lyman
series

Atoms 427
D:\EG_Physics-12_(26-06-2022)\Open_Files\Ch-12\Ch-12
\ 27-Jul-2022 Ved_Goswami Proof-4 Reader’s Sign _______________________ Date __________

1. The wave-numbers decrease from 4. The wave number of first line of Balmer series of
(a) Lyman to Pfund series hydrogen atom is 15200 cm–1. What is the wave
(b) Pfund to Lyman series number of the first Line of Balmer series of Li2+
(c) Balmer series to Brackett series ion?
(d) None of above. (a) 15200 cm–1 (b) 136800 cm–1
2. When an electron jumps in n1 orbit the series of
(c) 76000 cm–1 (d) 13680 cm–1
spectral lines obtained is called
(a) Balmer series. (b) Pfund series 5. Shortest wavelength present in the Paschen series
(c) Brackett series (d) Lyman series is given by
3. The wavelength of the second line of the Balmer (a) 8204 Å (b) 8600 Å
series in the hydrogen spectrum is 4861 Å. Then
value of wavelength of the first line will be (c) 6600 Å (d) 8330 Å
(a) 6562 Å (b) 6200 Å Ans. 1. (a) 2. (d) 3. (a) 4. (b) 5. (a)
(c) 5660 Å (d) 1238 Å

IMPORTANT FORMULAE
Formula Symbols Application

1. k4Ze 2 To find the distance of closest approach r0.


r0 = 1 , Z = Atomic number of element.
mv 2
k=
4πε0
m = mass of electron,
v = velocity of electron.

2. k Ze 2 cot θ/2 b = impact parameter To find the impact parameter


b= q = scattering angle
1
mv 2
2

3. n2 h2 rn = Radius of nth orbit Bohr’s radius (Z =1 n =1)


rn = r0 = 0.53 Å
4π 2 mk Ze 2

n2
r∝
Z

4. 2πkZe 2 En = Total energy of an electron in nth orbit.


v=
nh
c
v ∝ Z/n, v =
137n

5. 2π 2 mk 2 Z 2 e 4 En = Total energy of electron in nth orbit.


En = – 13.6
n2 h2 En = − Total energy of
n2
E ∝ Z2/n2 electron in nth orbit for hydrogen.

6. kZe 2 1 Kinetic energy of electron Relation between K.E. and total


K.E = k= , Z = Atomic number of element energy
2r 4πε0
K.E = – En

428 Physics–12
D:\EG_Physics-12_(26-06-2022)\Open_Files\Ch-12\Ch-12
\ 27-Jul-2022 Ved_Goswami Proof-4 Reader’s Sign _______________________ Date __________

7. kZe 2 En = Total Energy Potential energy of electron relation between P.E. and total
P.E = – energy
r
P. E. = 2 En

8. l = Wavelength of emitted radiation. 1


1 1 1 = Wave number
=R 2 − 2 R = Rydberg's constant λ
λ  n1 n2  Lyman series : n1 = 1, n2 = 2, 3, 4...
Balmer series : n1 = 2, n2 = 3, 4, 5...

9. (i) Kinetic energy (K.E) K.E = Kinetic energy Relation among K.E., P.E and T.E.
T.E = Total energy
= – Total Energy (T.E) P.E. = Potential energy

(ii) Potential energy (P.E)


= 2 × Total Energy

COMMON ERRORS
S No. Errors Corrections
1. Student forget to convert from eV to Joule Use 1eV = 1.6 × 10–19 J
2. Students get confused in Ground state and first For ground state n = 1
excited state. For first excited state n = 2

Atoms 429
D:\EG_Physics-12_(26-06-2022)\Open_Files\Ch-12\Ch-12
\ 27-Jul-2022 Ved_Goswami Proof-4 Reader’s Sign _______________________ Date __________

REVISION CHART

1. Every atom consists of a tiny central core, called


the atomic nucleus, in which the entire positive 2. The size of Nucleus is of the order of 10–15m , which
charge and almost entire mass of the atoms are is very small as compared to the size of the atom
which is of the order of 10–10 m
concentrated.

Rutherford’s Model of Atom

3. 
The atomic nucleus is surrounded by certain 4. These electrons revolve around the nucleus in
number of electrons. various circular orbits as do the planets around
the sun.

Bohr’s Model of Hydrogen

5. It is based on the non-classical assumption that 6. Bohr’s model calculated the following energies for
electrons travel in specific shells or orbits, an electron in the shell, (n):
around the nucleus. Without radiating energy, –13.6
nh E (n) = eV
when angular momentum = n2

430 Physics–12
D:\EG_Physics-12_(26-06-2022)\Open_Files\Ch-12\Ch-12
\ 27-Jul-2022 Ved_Goswami Proof-4 Reader’s Sign _______________________ Date __________

IMPORTANCE OF EACH TOPIC AND FREQUENTLY ASKED TYPES OF QUESTIONS

☞ Important Topics
1. Questions related to Total energy of an electron, radius of hydrogen atom.

1  1 1
2. Questions on Rydberg formula =R 2 − 2
λ  n1 n2 
3. Questions based on Energy level diagram.
* Maximum weightage is of Bohr Model of Hydrogen Atom.

1. Calculate the radius of the third Bohr orbit of hydrogen atom and energy of electron in that orbit.
[Ans. r3 = 4775 Å and E3 = – 2.43 × 10–19 J]
2. Calculate the longest and shortest wavelength in the Balmer series of Hydrogen atom. Rydberg constant = 1.098
× 107 m–1.[Ans. l1 = 6563 Å, lS = 3646 Å]
3. What will be the distance of closest approach of a 5 MeV proton as it approaches a gold nucleus?
[Ans. 4.55 × 10–14 m]
4. A 12.5 MeV alpha-particle approaching a gold nucleus is deflected by 180°. What is the closest distance to which
it approaches the nucleus? [Ans. 1.82 × 10–14 m]
5. Determine the speed of the electron in n = 3 orbit of hydrogen atom. [Ans. 7.29 × 105 ms–1]

Atoms 431
D:\EG_Physics-12_(26-06-2022)\Open_Files\Ch-12\Ch-12
\ 27-Jul-2022 Ved_Goswami Proof-4 Reader’s Sign _______________________ Date __________

ASSIGNMENT
I. Objective Type Questions (1 Mark)
1. Multiple choice questions:
(i) According to classical theory, Rutherford atoms is:
(a) stable (b) unstable (c) metastable (d) semistable
(ii) Taking Bohr’s radius as a0 = 53 pm, the radius of Li ion in its ground state, on the basis of Bohr’s model, will
++

be about
(a) 53 pm (b) 27 pm (c) 18 pm (d) 13 pm
2. Fill in the blanks:
(i) ............... of the electron in the orbit signifies that the electron and nucleus is a bound system.
(ii) The ................. lies in the infrared region of spectrum.
II. Very Short Answer Type Questions (1 Mark)
3. The radius of innermost electron orbit of a hydrogen atom is 5.3 × 10 m. What is the radius of orbit in second
–11

excited state?
4. What is the angular momentum of an electron in the third orbit of an atom?
III. Short Answer Type Questions-I (2 Marks)
5. Calculate the frequency of the photon, which can excite the electron to –3.4eV from – 13.6 eV.
−13.6
6. The energy of electron in hydrogen atom is En = eV, where n = 1, 2, 3, ... Show that:
n2
(i) The electron in hydrogen atom cannot have an energy of – 6.8 eV.
(ii) Spacing between the lines (consecutive energy levels) within the given set of observed hydrogen spectrum
decreases as n increases.
7. The ground state energy of hydrogen atom is – 13.6 eV:
(i) What are the potential energy and K.E. of electron is 3rd excited state?
(ii) If the electron jumps to the ground state from the third excited state, calculate the frequency of photon
emitted.
IV. Short Answer Type Question-II (3 Marks)
8. Show mathematically how Bohr’s postulate of quantisation of angular momentum in hydrogen atom is explained
by de-Broglie hypothesis.
9. An electron in a hydrogen atom makes transition from orbits of higher energies to orbits of lower energies.
(i) When will such transitions result in (a) Lyman (b) Balmer series? [CBSE 2022]
(ii) Find the ratio of the longest wavelength in Lyman series to the shortest wavelength in Balmer series.
V. Long Answer Type Question (5 Marks)
10. (a) State the basic postulates of Bohr’s theory of atomic spectra. Hence obtain an expression for radius of orbit
and the energy of orbital electron in hydrogen atom. 2
(b) The energy levels of an atom are shown in figure below. Which of them will result in the emission of a photon
of wavelength 275 nm. 3
0 eV
A
– 2 eV
B C
– 4.5 eV

D
– 10 eV


432 Physics–12
D:\EG_Physics-12_(26-06-2022)\Open_Files\Ch-13\Ch-13
\ 27-Jul-2022 Ved_Goswami Proof-4 Reader’s Sign _______________________ Date __________

Topics Covered
13 Nuclei

13.1 Atomic Mass, Composition and Size of a Nucleus 13.2 Binding Energy
13.3 Nuclear Reactions

C hapter map
Nuclei

Atomic Protons and Composition Isotopes, Nuclear


Number Neutrons of nuclei isobars, Reactions
isotones

Nuclear Binding
Energy
Nuclear Effects of
Nuclear
Fission and Nuclear
Reactor
Binding Mass defect Fusion reaction
Nuclear
Energy and its Forces and packing
curve fraction

Topic 1. Atomic Mass, Composition and Size of a Nucleus


A – Being neutral, it does not interact with electrons so
•• Nucleus: Representation : Z X
it has a low ionising power.
Z = atomic no. = no. of protons = no. of electrons
– Being neutral, they are not attracted or repelled
A = atomic mass no. = no. of nucleons by the nucleus so they can easily penetrate heavy
No. of neutrons = A – Z. nuclei and induce nuclear reactions.
Nucleus consists of protons and neutrons. •• Nucleon: The protons and neutrons are collectively
Proton is a fundamental particle with a charge of + 1.6 called nucleon.
× 10–19 C and a mass of 1.6726 × 10–27 kg. •• Atomic Number (Z): It is no. of protons in the nucleus.
A proton has an intrinsic spin angular momentum equal •• Mass Number (A): It is total no. of protons and neutrons
to 1/2 and also possess magnetic moment. present in a nucleus.
•• Properties of Neutron: Neutron is a charge-less •• Nuclide: It is specific nucleus of an atom characterised
fundamental particle having mass slightly greater than by its atomic number and mass number.
that of proton. Represented by A
ZX
Inside a nucleus, neutron is stable but outside the nucleus,
it is unstable. A free neutron spontaneously decays into A → Mass number Z → atomic number.
a proton and an electron and a particle called anti- •• Isotopes: They are atoms of an element which have the
neutrino. same atomic number but different mass number.
1 1 0 They differ in number of neutrons but have the same
0 n → 1H + −1e + v + Q ,where Q is energy.
number of protons.
– A neutron has a mean life of 1000 seconds.
433
D:\EG_Physics-12_(26-06-2022)\Open_Files\Ch-13\Ch-13
\ 27-Jul-2022 Ved_Goswami Proof-4 Reader’s Sign _______________________ Date __________

1
e.g., 12 14 ; 1
6 C, 6 C
2 3
1 H, 1 H, 1 H
or R ∝ A3
The different isotopes of an element are found to have 1
different relative abundances. So the weighted average i.e., R = R0 A 3
of the atomic mass of all isotopes of an element is taken R0 is a constant and
as its average atomic mass. e.g. 75% of Chlorine is 17
35
Cl
R0 = 1.2 × 10–15 m.
37
25% of Chlorine is 17 Cl •• Nuclear Density: The density of nuclear matter is the
ratio of mass of a nucleus to its volume
75 × 35 + 25 × 37
Weighted average = = 35.5 Mass
75 + 25 D = Volume
•• Isobars: Atoms of different elements having the same Let A = Mass number of atom
mass number but different atomic number are called R = Its radius
isobars. They contain different number of protons and
The average mass of a nucleon is m
electrons.
4
e.g., 13 H and 32 He Volume of nucleus = pR 3
3
14
6 C and 7 N
14 Mass of nucleus = Am

Am 3 Am
•• Isotones: Nuclei having same number of neutrons are Density = =
4 3 4πR 3
37
called isotones. E.g. 17 39
Cl, 19 K; 12 H, 32 He πR
3
1 3 Am 3 Am 3m
•• Atomic mass unit (amu): It is th of actual mass of D= = =
( ) 12 3
carbon atom 6 C
12  1 4π R 0 A 4πR 30
3

4π  R 0 A 
3

Mass of 6 × 1023 atoms of C = 12 g  


12 Density is independent of mass number.
Mass of 1 atom of C = 23
g
6 × 10 Note : Nucleus density is not uniform throughout
the nucleus. It is max. at centre and gradually
1 10 −3
1 amu = g= kg = 1.66 × 10 −27 kg decrease to 0 as we move away from the
6 × 1023 6 × 1023 centre. So the effective value of nucleus
1 amu = 1.66 × 10 −27 kg
radius is taken upto a point where its density
    reduces to half of its value at the centre.
•• Energy Equivalent of 1 amu: •• Nuclear Force: It is the force existing between two
E= mc2 nucleons.
E = 1.66 × 10–27 × 3 × 108 × 3 × 108 – Properties of Nuclear force:
= 1.66 × 9 × 10–27 × 1016 1. It is strongest interaction in nature: This holds
= 14.94 × 10–11 J the nucleus together despite Coulombian repulsion
between protons. e.g., Fg : Fe : Fn = 1 : 1036 : 1038
14.94 × 10 −11
= eV
1.6 × 10 −19
14.94
= × 108 = 9.33 × 108 eV
1.6
= 933 MeV
•• Nucleus Size: The volume of a nucleus is directly
proportional to its mass number.
Let R be the radius of the nucleus
A is mass number of atom, 2. It is short range force: If operates only up to a very
4 3 short distance of order of 2-3 fm (fermi).
pR ∝ A
3 3. When r > r0, PE gradually decreases to 0 with
increase in r. It indicates attractive nuclear force.
434 Physics–12
D:\EG_Physics-12_(26-06-2022)\Open_Files\Ch-13\Ch-13
\ 27-Jul-2022 Ved_Goswami Proof-4 Reader’s Sign _______________________ Date __________

This attraction is maximum r0 = 0.8 fm and does not 5. Nuclear force is saturated force. This means a
follow the inverse square law. nucleon interacts only with its nearest neighbour.
When r < r0, PE increases rapidly indicating strong 6. It is non-central. Does not act along line joining
repulsive nuclear force. two charges.
4. Nuclear force is charge independent. This mean 7. Spin dependent force i.e., nuclear force between
that force between two protons is nearly equal to two nucleons having parallel spin (same direction
the force between two neutrons or between neutron spin) is stronger than the force between two anti-
and a proton. parallel spins.

EXERCISE 13.1
I. Objective Type Questions (1 Mark) 4. Match the Columns
1. Choose the correct answers from the given options Column-I Column-II
(MCQs). (i) It is number of protons in (a) Mass number
(i) Heavy stable nuclei have more neutrons than the nucleus
protons. This is because of the fact that (ii) The protons and neutrons (b) Atomic number
(a) neutrons are heavier than protons. are collectively called
(b) electrostatic force between protons are (iii) It is total number of protons (c) Isotopes
repulsive. and neutrons present in
(c) neutrons decay into protons through beta nucleus
decay. (iv) Atoms having same atomic (d) nucleon
number but different mass
(d) nuclear forces between neutrons are weaker
number
than that between protons.
(ii) Nucleus of an atom of mass no. 24 and charge no. For Q. 5 there are two statements labelled as assertion
11 consists of (A) and reason (R). Select the correct answer to these
questions from the codes (a), (b), (c) and (d) as given
(a) 11 electrons 11 protons and 13 neutrons below.
(b) 11 electrons, 11 protons and 11 neutrons (a) Both A and R are true and R is the correct
(c) 11 protons and 13 neutrons explanation of A.
(d) 11 protons and 13 electrons (b) Both A and R are true but R is not the correct
(iii) Which of the following particles has similar mass explanation of A.
to that of the protons? (c) A is true but R is false.
(a) Proton (b) Neutron (d) Both A and R are false.
(c) Positron (d) Neutrino 5. Assertion (A): Nuclear density is extremely higher
2. Fill in the blanks. than atomic density.
(i) Proton was discovered by ................... and neutron Reason (R): Most of the mass of the atom is
was discovered by ................. . concentrated in the nucleus.
(ii) An element A decays into element C by a two step I. Very Short Answer Type Questions (1 Mark)
process: 1. What is an isotope? Give two example.
A → B + 42He; B → C + 2e– 2. What is an isobar? Give two examples.
Then A and C are ................... 3. What is an isotone? Give two example.
3. State True or False 4. Two nuclei have mass numbers in the ratio 1 : 8. What
(i) Atoms having the same atomic number but is the ratio of their nuclear radii? [Delhi 2009]
different mas number are called isotopes. 5. What holds nucleons together in a nucleus?
1 6. Two nuclei have mass numbers in the ratio 1 : 2. What
(ii) The size of the nucleus varies as power of mass is the ratio of their nuclear densities? [Delhi 2009]
number. 5
7. What is one electron volt?
(iii) The order of magnitude of the density of nuclear 8. If one a.m.u. = 1.66 × 10–27 kg, what is the mass of
matter is 1017 kg m–3. one atom of 12C?

Nuclei 435
D:\EG_Physics-12_(26-06-2022)\Open_Files\Ch-13\Ch-13
\ 27-Jul-2022 Ved_Goswami Proof-4 Reader’s Sign _______________________ Date __________

9. How is nuclear size related to its mass number? 18. Find energy equivalence of 1 amu.
10. Electrons cannot be a part of nucleus but protons can 19. Neon has two isotopes of masses 20 and 22 which
be. Why? occur is the ratio 9 : 1. Find the atomic mass of neon.
11. Why is nuclear density same for all nuclei? 20. You are given two nuclei 3X7 and 3Y4. Explain giving
[Delhi 2013] reasons, as to which one of the two nuclei is likely to
12. Two nuclei have mass numbers in the ratio 2 : 5. What be more stable?
is the ratio of their nuclear densities? [Delhi 2009] 21. Assuming the nuclei to be spherical in shape, how
13. Draw a graph showing variation of potential energy does the surface area of a nucleus of mass number
of a pair of nucleons as a function of their separation. A1 compare with that of a nucleus of mass number
Indicate the region in which nuclear force is A2? [Delhi 2008 C][HOTS]
(a) attractive (b) repulsive. [Delhi 2009, 2012] III. Short Answer Type Questions-II (3 Marks)
14. Is free neutron a stable particle? If not, what is its 22. (a) Write three characteristic properties of nuclear
mode of decay? force.[AI 2015]
II. Short Answer Type Questions-I (2 Marks) (b) Draw a plot of potential energy of a pair of
15. Define the term atomic mass unit (1 u). Find out its nucleons as a function of their separation. Write
value in kilogram. two important conclusions that can be drawn from
16. What is the nuclear radius of Fe125, if that of Al27 is the graph. [AI 2010, 2011]
3.6 fermi. [Delhi 2008] 23. Group the following six nuclides into three pairs of
17. Show that nuclear density is same for all the nuclei. (i) isotones (ii) isotopes and (iii) isobars. 6C12, 2He3,
198, H3, Au197, C14
[Delhi 2013] 80Hg 1 79 6 [Delhi 2013]
Or 24. Explain the concept of nuclear forces. Discuss their
Show that density of nucleus is independent of its characteristic properties. Which properties distinguish
mass number A. [Delhi 2019] them from electrostatic forces? [Delhi 2011]

Answers 13.1
I. Objective Type Questions 1 1
1. (i) (b) (ii) (c) (iii) (c) R1  A1  3  1  3 1
=  =  8  = 2
2. (i) Rutherford, Chadwick  (ii) Isotopes       R 2  A 2 
3. (i) True (ii) False (iii) True 5. Nuclear forces amongst the nucleons in a nucleus.
4. (i)-(b) (ii)-(d) (iii)-(a) (iv)-(c) 6. The ratio of nuclear densities is 1. This is because
5. (a) nuclear density does not depend upon mass number.
II. Very Short Answer Type Questions 7. One electron volt is defined as the amount of energy
acquired by an electron when it is accelerated under a
1. Isotope: Isotopes are the atoms of an element that have
potential difference of 1 volt.
the same atomic number but different atomic masses.
Isotopes have same number of protons but different 1
8. 1 amu = ofmass of one 12
6 C atom
number of neutrons. 12
12 14 35 37 Mass of one atom of 12
6 C is 12 amu
For example 6 C and 6 C, 17 Cl and 17 Cl
= 12 × 1.66 × 10–27 kg
2. Isobars: The atoms which have same mass numbers
but different atomic numbers are called isobars. = 1.992 × 10–26 kg
9. The radius R of atomic nucleus is related to mass
For example, 13 H and 32 He are isobars 1

14 14 number A of the nucleus as R = R 0 A 3 where R0 = 1.2


6 C and 7 N are isobars
× 10–15 m; an. empirical constant.
3. Isotones: Isotones have same number of neutrons. 10. The de-Broglie wavelength of electrons is larger
39 37
than the nucleus, whereas the de-Broglie wavelength
For example : 19 K and 17 Cl
1 associated with protons is smaller than the size of the
4. Using R = R 0 A 3 nucleus.

436 Physics–12
D:\EG_Physics-12_(26-06-2022)\Open_Files\Ch-13\Ch-13
\ 27-Jul-2022 Ved_Goswami Proof-4 Reader’s Sign _______________________ Date __________

Mass of nucleus In case of 3Y4 ,


11. As nuclear density =
Volume of nucleus No. of neutron 4 − 3 1
= = = 0.33
So, volume of a nucleus ∝ A (mass number)    No. of proton 3 3
So nuclear density is independent of mass number. For stability, this ratio has to be close to one. Obviously,
12. One. As nuclear density is independent of mass number. nucleus 3X7 is more stable than the nucleus 3Y4.
13. 1

21. Surface Area = 4pR2, R = R0A 3


1 2

 0 13 
R A 2
(S. Area)1 4πR12  
 A 3
     = 2
= 2 =  1
(S. Area) 2 4πR 2  1  A2 
 R 0 A 23 
 
14. No, free neutron is not a stable particle. It decays as
1
0n → 11H + 0
−1e + v , v is antineutrino 22. Refer theory topic no. 1 under the heading Nuclear force.
23. (i) Isotones 80Hg198, 79Au197 same no. of neutrons i.e.,
1
15. One atomic mass unit is defined as th of the mass of A–Z
12
carbon -12 atom (ii) Isotopes 6C12, 6C14 same Z, different A
1
1u = mass of C-12 atom (iii) Isobars 2He3, 1H3 same A, different Z
12
24. Nuclear force : Inside the nucleus large number of
1 12 protons are held along with the neutrons in a very small
= × g = 1.66 × 10 −27 kg
12 6.023 × 1023 space. The Coulombian force of repulsion between
1 protons (due to like charges) is nullified by a strong
16. Using R = R 0 A 3 attractive force called nuclear force.
1 1
So Nuclear forces are the strong forces of attraction
R1  A1  3  125  3 5 which hold together the nucleus in a small space called
So = = =
R 2  A 2   27  3 nucleus, inspite of strong electrostatic force of repulsion
between protons.
5 5
R1 = R 2 = × 3.6 = 6.0 fermi Characteristic/properties of nuclear forces:
    3 3
17. Refer theory topic no. 1 under the heading Nuclear Refer theory topic no. 1 under the heading Nuclear
density. force.
18. 1 amu = 1.66 × 10–27 kg Comparison between electrostatic and nuclear force:
E = mc2 = 1.66 × 10–27 × (3 × 108)2 Electrostatic Nuclear Force
= 1.49 × 10–10 J 1. These are weak forces. 1. These are the
2. Depending upon the charge strongest attractive
1.49 × 10 −10
= MeV = 931.25 MeV it could be attractive or forces
1.6 × 10 −13
repulsive and magnitude 2. It is independent of
19. Atomic mass is the weighted average mass of all depends upon the amount charge.
isotopes. of charge.
So average atomic mass
3. It is a long range force. 3. It is a short range
= 20 × 9 + 22 × 1 = 180 + 22 = 202 = 20.2 force.
9 +1 10 10 4. It is a central force. 4. It is a non central
20. In case of 3X7, force.
No. of neutron 7 − 3 4 5. It obeys inverse square 5. It does not obey
= = = 1.33
   No. of proton 3 3 law. inverse square law.

Nuclei 437
D:\EG_Physics-12_(26-06-2022)\Open_Files\Ch-13\Ch-13
\ 27-Jul-2022 Ved_Goswami Proof-4 Reader’s Sign _______________________ Date __________

Topic 2. Binding Energy


•• Mass Defect: It is difference between mass of the 1. Except for small nuclei like He, C, O, binding
nucleons and mass of nucleus. energy per nucleon lie on or near a smooth curve.
A
ZX → ∆m = Zm p + (A − Z) mn − M N 2. Binding energy per nucleon is small for lighter
mP = Mass of proton; mN = Mass of neutron; 1 2 3
nuclei like 1 H, 1 H, 1 H
MN = Mass of nucleus
3. Upto mass number 20, there are some well defined
Z = Atomic number; A = Mass number
•• Binding Energy: Binding energy of a nucleus is defined peaks indicating the higher stability of these nuclei
as the energy required to break up a nucleus into its than their neighbours.
constituent neutrons and protons and to separate them 4. The curve has a broad maxima close to 8.5 MeV,
large distances apart so that they don’t interact. for mass numbers 40 to 120 and 56Fe has maximum
It can be defined as the surplus energy which the B.E. for nucleons 8.8 MeV.
nucleons give up by virtue of their attractions when
they become bound together to form a nucleus 5. As mass number increases further there is gradual
decrease in B.E. per nucleon and this decrease is
B.E. = ∆mc 2
due to Coulombian repulsion between protons.
2
•• Binding Energy per Nucleon = Dmc •• Importance of Binding Curve:
A
1. The lighter nuclei are suitable for fusion. BE per
It is the average energy required to extract one nucleon
from nucleus. nucleon is smaller for lighter nuclei i.e., They are
•• B.E./ Nucleon Vs Mass Number curve less stable. When lighter nuclei combine to form
a heavy nucleus, the higher BE per nucleon of the
8.8 latter results in the release of energy.
56
MeV Fe 2. BE per nucleon is smaller for heavy nuclei than
16
8O for the middle nuclei. Heavy nuclei are less stable
12
6C than the middle ones. When heavy nucleus splits
4
2 He into lighter nucleus, the BE per nucleon increases
BE resulting in liberation of energy.
per 3
nucleon 1H 3. The constancy of BE curve in the range of mass
2 number from 30-170 is a consequence of the fact
1H
the nuclear force is a short range force.
Mass number

EXERCISE 13.2
I. Objective Type Questions (1 Mark) (c) Q1 = (Mx – My – 2me) c2 and Q2 = (Mx – My
1. Choose the correct answers from the given options + 2me) c2
(MCQs). (d) Q1 = (Mx – My + 2me) c2 and Q2 = (Mx – My
(i) Mx and My denote the atomic masses of parent and + 2me) c2
the daughter nuclei respectively in a radioactive (ii) Tritium is an isotope of hydrogen whose nucleus
decay. The Q1-value for a b– decay is Q1 and for triton contains 2 neutrons and 1 proton. Free
a b+ decay is Q2. If me denotes the mass of an neutrons decay into p + e + n, if one of the
electron, then which of the following statements neutrons in triton decays, it would transform
is correct? into 2He3 nucleus. This does not happen. This is
(a) Q1 = (Mx – My) c2 and Q2 = (Mx – My – 2me) because
c2 (a) Triton energy is less than that of He3 nucleus.
(b) Q1 = (Mx – My) c2 and Q2 = (Mx – My) c2
438 Physics–12
D:\EG_Physics-12_(26-06-2022)\Open_Files\Ch-13\Ch-13
\ 27-Jul-2022 Ved_Goswami Proof-4 Reader’s Sign _______________________ Date __________

(b) yhe electron created in the beta decay process Reason (R): It is the total energy required to extract
cannot remain in the nucleus. electrons from the nucleus.
(c) both the neutrons in triton have to decay II. Very Short Answer Type Questions (1 Mark)
simultaneously, resulting in a nucleus with 3 1. What do you mean by mass defect?
protons which is not a He3 nucleus. 2. Define the term binding energy.
(d) free neutrons decay due to external perturbation 3. The mass of the nucleus is less than the sum of the
which is absent in triton nucleus. masses of the nucleons forming it. Why?
(iii) A nucleus nXm emits one alpha and two beta [Delhi 2009]
particles (b). The resulting nucleus is 4. If the nucleons bound in a nucleus are separated apart
(a) nXm–4 (b) n–2Xm–4 from each other, the sum of their masses is greater
(c) n–4Xm–4 (d) nXm–5 than the mass of the nucleus. Where does this mass
(iv) The mass of neutron and proton are 1.0087 and difference come from? Explain briefly.
1.0073 u respectively. If the neutrons and protons 5. A nucleus of mass number A has mass defect (Dm)
combine to form a helium nucleus of mass 4.0015 what is BE per nucleon of this nucleus?
u, the binding energy of the helium nucleus will 6. For a given nuclear reaction the B.E./nucleon of
be the product nucleus/nuclei is more than that for
(a) 28.4 MeV (b) 20.8 MeV the original nucleus/nuclei. Is this nuclear reaction
(c) 27.3 MeV (d) 14.2 MeV exothermic or endothermic in nature? Justify your
2. Fill in the blanks. choice.[S.P. 2013][HOTS]
(i) The average binding energy per nucleon for the 7. Four nuclei of an element fuse together to form a
nuclei lying in the middle of periodic table is heavier nucleus. If the process is accompanied by
nearly ................. release of energy, which of the two-the parent or the
(ii) Neutrino is a particle with ........................ . daughter nucleus would have higher binding energy
3. State True or False per nucleon? [CBSE 2018][HOTS]
(i) The B.E. per nucleon is maximum for 235 92U. OR
A A A−4 A−4
(ii) In a given reaction: Z X → Z+1Y → Z −1 K → Z −1 K Four nuclei of an element undergo fusion to form a
the radioactive radiations are emitted in the heavier nucleus, with release of energy. Which of the
sequence of a, g, b. two–the parent or the daughter nucleus–would have
(iii) A positron has the same mass as electron. higher binding energy per nucleon?
4. Match the Columns 8. A nucleon makes a transition from one permitted
Column-I Column-II energy level to another level of lower energy. Name
(i) Positively charge particles (a) g-rays the region of e.m. spectrum to which the emitted
(ii) Negatively charge particles (b) mass defect photon belongs.
(iii) Travel with speed of light (c) a-rays 9. Name the process responsible for energy production
in the sun. [Delhi 2004]
(iv) Difference between mass (d) b-rays
10. Draw the graph showing the distribution of kinetic
of nucleons and mass of
energy of electrons emitted during beta decay.
nucleus
[Delhi 2006]
For Q. 5 there are two statements labelled as assertion 11. If both the number of protons and neutrons in a
(A) and reason (R). Select the correct answer to these nuclear reaction is conserved, in what way is mass
questions from the codes (a), (b), (c) and (d) as given converted into energy (or vice versa)? Explain giving
below. one example. [Delhi 2015C]
(a) Both A and R are true and R is the correct OR
explanation of A. If the total number of neutrons and protons in a
(b) Both A and R are true but R is not the correct nuclear reaction is conserved, how than is the energy
explanation of A. absorbed or evolved in the reaction? Explain.
(c) A is true but R is false. 12. Complete the following nuclear reactions:
(d) Both A and R are false. 10 1
(a) 5 B + 0 n  → 42 He + .....
Dmc 2
5. Assertion (A): Binding Energy per Nucleon = (b) 94
42 Mo + 12 H 
→ 95
43 Te + .....
A

Nuclei 439
D:\EG_Physics-12_(26-06-2022)\Open_Files\Ch-13\Ch-13
\ 27-Jul-2022 Ved_Goswami Proof-4 Reader’s Sign _______________________ Date __________

13. In a tropical nuclear reaction e.g., 21H + 12H → 23He 20. Suppose, we think of fission of a 56
nucleus into
26 Fe
+ 10n + 3.27 MeV, although number of nucleons is
28
conserved, yet energy is released. How? Explain. two equal fragments, 13 Al. Is the fission energetically
[Delhi 2013] possible? m 56 28
26 Fe = 55.93494 amu, m 13 Al = 27.98191
14. 32 He and 13 H nuclei have same mass number. Do they amu. [NCERT][HOTS]
have the same binding energy?
21. Calculate the energy release in MeV in the deuterium-
III. Short Answer Type Questions-I (2 Marks) tritium fusion reaction:
15. The figure shows the plot of binding energy (BE) per 2
+ 13 H 
→ 42 He + 10 n
1H
nucleon as a function of mass number A. The letters
Using the data:
A, B, C, D and E represent the positions of typical
nuclei on the curve. Point out, giving reasons, the 1
m ( H ) = 2.014102 u
2
1 m ( H ) = 3.016049 u
3

two processes (in terms of A, B, C, D and E), one of


which can occur due to nuclear fission and the other
( )
m 42 He = 4.002603 u    mn = 1.008665 u
   1u = 931.5 MeV/c2
due to nuclear fusion. [AI 2015]
22. A given coin has a mass of 3.0 g. Calculate the nuclear
energy that would be required to separated all the
neutrons and protons from each other. For simplicity,
assume that the coin is entirely made of 29Cu63 atoms
(of mass 62.92960 u).[NCERT][HOTS]
IV. Short Answer Type Questions-II (3 Marks)
23. Binding energy per nucleon versus mass number
A A1 A2 A3
curve is as shown. Z S, Z1 W, Z2 X and Z3 Y are four
16. Calculate the binding energy per nucleon of 26Fe56 nuclei indicated on the curve.
nucleus. Given that mass of 26Fe56 = 55.934939 u, mass
of proton = 1.007825 u, mass of neutron = 1.008665
u and 1u = 931 MeV.
17. (a) Plot a graph showing the variation of binding
energy per nucleon as a function of mass number.
Which property of nuclear force explains the
approximate constancy of binding energy in the
range 30 < A < 170? [AI 2015]
(b) (i) Plot a graph showing the variation of binding
energy per nucleon as a function of mass
number.
(ii) Which property of nuclear force explains the Based on the graph:
approximate constancy of binding energy in the (a) Arrange X, W and S in the increasing order of
range 30 < A < 170? stability.
(iii) How does one explain the release of energy in (b) Write the relation between the relevant A and Z
both the processes of nuclear fission and fusion values for the following nuclear reaction.
from the graph? S → X + W
18. Calculate the B.E/ nucleon of 17C35 nucleus. Given (c) Explain why binding energy for heavy nuclei is
that mass of proton = 1.007825 u. mass of neutron = low.
1.008665 u, mass of 17Cl35 = 34.980000u; 1u = 931 OR
MeV. How are protons, which are positively charged, held
19. A heavy nucleus X of mass number A = 240 and
together inside a nucleus? Explain the variation of
B.E./A = 7.6 MeV is split into two nearly equal
potential energy of a pair of nucleons as a function
fragments Y and Z of mass numbers A1 = 110 and
of their separation. State the significance of negative
A2 = 130. The binding energy of each one of these
potential energy in this region. [CBSE S.P. 2018-19]
nuclei is 8.5 MeV per nucleon. Calculate the total
binding energy of each of the nuclei X, Y and Z and 24. Explain the processes of nuclear fission and nuclear
hence the energy Q released per fission in MeV. fusion by using the plot of binding energy per nucleon
[Delhi 2010][HOTS] (BE/A) versus the mass number A. [Delhi 2018]

440 Physics–12
D:\EG_Physics-12_(26-06-2022)\Open_Files\Ch-13\Ch-13
\ 27-Jul-2022 Ved_Goswami Proof-4 Reader’s Sign _______________________ Date __________

Answers 13.2
I. Objective Type Questions 10. Y

1. (i) (a) (ii) (a) (iii) (a) (iv) (a)


2. (i) 8.5 MeV

Number of β-particles
  (ii) Zero charge and spin property
3. (i) False
End point energy
(ii) False (iii) True
4. (i)-(c) (ii)-(d) (iii)-(a) (iv)-(b)
5. (c)
II. Very Short Answer Type Questions 0 0.2 0.4
(Max)
0.6
X

1. Mass defect is the difference between the sum of the   β-particle energy in MeV
masses of neutrons and protons forming a nucleus and 11. As the number of nucleons is conserved in a nuclear
mass of the nucleus. reaction, the total rest mass of protons and neutron on
If mp is mass of a proton either side of the reaction remains same. But the binding
mn is mass of a neutron.
energies of nuclei on the two sides of the reaction are
different. It is this difference in B.E. that appears as the
M is mass of the nucleus
energy released in the nuclear reactions.
Then mass defect = [Zmp + (A – Z) mn –M] 2
1H + 12 H 
→ 32 He + 10 n + Energy
2. Binding energy is defined as the energy required to
10
break up a nucleus into its constituent protons and 12. (a) 5 B + 10 n 
→ 42 He + 37 Li
neutrons and to separate them to such a large distance 94
that they do not interact with each other. (b) 42 Mo + 12 H 
→ 95 1
43 Te + 0 n

3. Size of a nucleus is about 10–15 m. Neutrons and protons 13. Although the number of nucleons is conserved, but the
have to collect in a very small place. The energy required binding energy of nuclei on the two sides of the reaction
for this i.e., to bind the nucleons together comes at the are different. It is the difference in binding energy that
cost of their masses. So the mass of the nucleus is less is released.
than the total mass of all nucleons forming it. 14. No, because 32 He has 2 protons and 1 neutron whereas
3
4. When nucleons are bound in a nucleus, some of their 1 H has 1 proton and 2 neutron. There is no repulsive

mass is converted into energy that binds them together force between protons in 13 H . So B.E. of 13 H is greater.
in the nucleus. The energy equal to binding energy is 15. (i) B.E./nucleon of lighter nuclei is less i.e.,
spend to separate these nucleons apart from one another.
It is this energy which appears in the form of increased A A (B.E. ) < (B.E. )
AB
mass Dm = B.E./c2.
when two lighter nuclei say two A nuclei combine
( ∆m)c 2 to form a nucleus B then energy will be released.
5. BE / nucleon = As B is more stable than A.
A
where c = velocity of light in vacuum. Lower mass number nuclei when fuse to form a
heavy nucleus energy will be released.
6. As BE/nucleon of the product nuclei is more than that
(ii) When nucleus E breaks up into two nuclei of
of original nuclei. So more mass has been converted
comparable masses, the B.E/A. of the product will
into energy. This would result in release of energy so lie in between C and D. i.e., the B.E./A of the product
it is exothermic. is more than the B.E./A of the parent nucleus. Thus
7. As the process is accompanied by release of energy so energy will be released.
the B.E./A of daughter nuclei is more than the B.E./A 16. For 26Fe56, no. of protons = 26 = Z
of parent nuclei. (The lighter nuclei are less stable than No. of neutrons = A – Z = 56 –26 = 30
intermediate nuclei).
Dm = [Zmp + (A – Z) mn – MN]
8. g-ray are emitted. \ Mass defect = 26 mp + 30 mn – MFe
The emitted photons have energy of the order of MeV. = 26 × 1.007825 + 30

9. Fusion of four hydrogen nuclei into helium nucleus. × 1.008665 – 55.934939

Nuclei 441
D:\EG_Physics-12_(26-06-2022)\Open_Files\Ch-13\Ch-13
\ 27-Jul-2022 Ved_Goswami Proof-4 Reader’s Sign _______________________ Date __________

   = 26.20345 + 30.25995 – 55.934939 For nucleus Y, A =110, B.E./A = 8.5 MeV


= 0.528461 u. Total B.E. = 110 × 8.5 = 935 MeV
931 For nucleus Z, A = 130, B.E./A = 8.5 MeV
  B.E./nucleon = 0.528461 × = 8.79 MeV.
56 Total B.E. = 8.5 × 130 = 1105 MeV
17. (a) In the range 30 < A < 170 Energy released per fission
Q = B.E. of Y + B.E. of Z – B.E. of X
= 935 + 1105 – 1824 = 216 MeV
20. According to question
56 28 28
26 Fe 
→ 13 Al + 13 Al + Q

Q =  m 56 28 
26 Fe − 2m 13 Al  × 931.5 MeV

= [55.93494 – 2 × 27.98191] × 931.5 MeV


  = (– 0.02888) × 931.5
= – 26.9 MeV
As Q value is negative, the fission is not possible
The approximate constancy of binding energy in the
energetically.
range 30 < A < 170 because the nuclear force is a
short range force. A nucleon well inside a nucleus 21. Dm = mass defect
will be under the influence of only those nucleons = (2.014102 + 3.016049 – 4.002603
which come within the range of nuclear force. As – 1.008665)u
nuclear force is a short range force so it will be = 0.018883 u
influenced only by a small number of nucleons, even
Energy released
if a large no. of nucleons are surrounding it. It also
MeV 2
shows that nuclear force has saturation property. = Dm × c2 = 0.018883 × 931.5 c
(b) For the first two parts the answer is same as for the c2
= 17.5895 MeV
part (a).
22. Number of atoms in 3 g coin
(iii) Nuclear fusion : The binding energy per nucleon
23
for very light (i.e., A < 10) nuclei is less i.e., they = 6.023 × 10 × 3 = 2.868 × 1022
are less stable. When two lighter nuclei combine to 63
form a heavier nucleus, the higher binding energy Each atom of copper contains 29 protons and 34
per nucleon of the formed nucleus results in release neutrons. Therefore, mass defect of each atom
of energy in fusion process.
= [29 × 10.00783 + 34 × 1.00867]
Nuclear fission : When a heavy nucleus breaks up
into two lighter nuclei of comparable masses, the – 62.92960
binding energy per nucleon of the lighter nuclei is = 0.59225 u
more than the B.E./nucleon of the heavy nucleus. Total mass defect for all the atoms
The lighter nucleus are more stable and it results in = 0.59225 × 2.868 × 1022 u
release of energy.
Dm = 1.6985 × 1022 u
18. In 17Cl35, no. of proton = 17
No. of neutrons = 35 – 17 = 18 As 1u = 931 MeV.
Mass defect = 17mp + 18mn – MCl \ Nuclear energy required
= 17 × 1.007825 + 18 × 1.008665 = 1.6985 × 1022 × 931 MeV
– 34.980000 = 1.58 × 1025 MeV
= 17.133025 + 18.155970 – 34.980000 23. (a) S, W, X 1
= 0.308995u. (b) Z = Z1 + Z2 ½
0.308995 × 931 A = A1 + A2 ½
B.E./Nucleon = = 8.22 MeV
35 (c) Reason for low binding energy:
19. For nucleus X: A = 240, B.E./A = 7.6 MeV In heavier nuclei, the Coulombian repulsive effects
Total B.E. = 7.6 × 240 = 1824 MeV can increase considerably and can match/ offset

442 Physics–12
D:\EG_Physics-12_(26-06-2022)\Open_Files\Ch-13\Ch-13
\ 27-Jul-2022 Ved_Goswami Proof-4 Reader’s Sign _______________________ Date __________

the attractive effects of the nuclear forces. This can (iii) The nuclear force between neutron-neutron, proton-
result in such nuclei being unstable. 1 neutron and proton-proton is approximately the
OR same. The nuclear force does not depend on the
Nuclear force binds the protons inside the nucleus. electric charge. 2
½ Significance of negative potential energy: Force is
For Graph and explanation, attractive in nature ½
[CBSE Marking Scheme, 2018-19]
(i) The nuclear force is much stronger than the Coulomb
force acting between charges or the gravitational 24. B.E. per nucleon v/s mass number (A) curve can be
used to explain the nuclear fission and nuclear fusion
forces between masses. The nuclear binding force
processes.
has to dominate over the Coulomb repulsive force
between protons inside the nucleus. This happens
only because the nuclear force is much stronger than
the coulomb force. The gravitational force is much
weaker than even Coulomb force.
(ii) The nuclear force between two nucleons falls
rapidly to zero as their distance is more than a few
femtometres. This leads to saturation of forces in a
medium or a large-sized nucleus, which is the reason
Nuclear fission: From the above curve it is clear that a

for the constancy of the binding energy per nucleon.
heavy nucleus say A = 240, has lower B.E. per nucleon,
A rough plot of the potential energy between two compared to that of a nucleus with A = 120, i.e. it is less
nucleons as a function of distance is shown in the stable, thus if a nucleus of atomic mass A = 240 breaks
figure. The potential energy is a minimum at a into two lighter nuclei of A = 120, nucleons get more
distance r0 of about 0.8 fm. This means that the tightly bound. This implies energy would be released
force is attractive for distances larger than 0.8 fm in the process i.e. splitting of a heavy nucleus into two
and repulsive if they are separated by distances less lighter nuclei i.e. nuclear fission.
than 0.8 fm Nuclear fusion: The B.E. per nucleon is small for

lighter nuclei. These are less stable so when two light
nuclei combine to form a heavier nucleus; the higher
B.E./nucleon of the later result in the release of energy.
This is what happens in a nuclear fusion.
Nuclear fission and nuclear fusion are the basis of atom
bomb or hydrogen bomb respectively.

Topic 3. Nuclear Reactions


•• Fission Reaction: When a heavy nuclear breaks up into (a) Uncontrolled Chain Reaction: In these, each
two nuclei of comparable masses, then phenomenon is neutron released in fission reaction is allowed to
called nuclear fission. carry on the fission, thus reaction accelerates at
1 235
→ 141 92
+ 3 10 n + Q (≈ 200 MeV) a very rapid rate and entire material will explode
0n + 92 U  56 Ba + 36 Kr
within a microsecond liberating lot of energy.
•• Thermal Neutron: The slow moving neutron of They forms principle of atomic bombs.
energies 0.0235 eV are called thermal neutron. They (b) Controlled Chain Reactions: The chain reactions
have velocities of 2200 m/s, which is equal to random can be controlled and made to proceed at a steady
velocities of atoms and nucleus at room temperature. rate by absorbing suitable number neutron at each
•• Chain Reaction It is the one in which the particle needed stage of the reaction so that on an average, one
for causing the fission is also one of the products so the neutron remains available for exciting causing
equation once started will continue on its own if we have further fission.
the sufficient mass.

Nuclei 443
D:\EG_Physics-12_(26-06-2022)\Open_Files\Ch-13\Ch-13
\ 27-Jul-2022 Ved_Goswami Proof-4 Reader’s Sign _______________________ Date __________

•• Reproduction/Multiplication Factor (k): Ratio of and pressure,they are also called thermonuclear reaction.
number of neutrons present at the beginning 1 generation Nuclear fusion is the source of energy in the sun.
to the number of neutrons present at beginning of Due to the need of very high temperature and pressure
previous generation. these reaction cannot be carried out in laboratory.
If k = 1 steady rate (critical reaction) The fusion reaction in the sun is a multi step process.
k > 1 Uncontrolled (accelerate) (super critical reaction) The proton-proton cycle is :
k < 1 highly controlled (reaction will gradually die out) 1
1H → 12 H + e + + v + 0.42 MeV ...(i)
+ 11H 
(sub critical)
•• Nuclear Fusion: It is the process in which two lighter e + + e − 
→ γ + γ + 1.02 MeV ...(ii)
nuclei combine to form a heavier nucleus with liberation 2
1H + 11H 
→ 3
He + γ + 5.49 MeV ...(iii)
of large a mount of energy. 2

The energy released per unit mass is much more than 3


+ 32 He 
→ 42 He + 11H + 11 H + 12.86 MeV 
2 He ... (iv)
the energy released per unit mass in fission. But these
reactions require extremely high temperature ~ 106 K for If we consider 2 × (i) + 2(ii) + 2(iii) + (iv)
the fusing nuclei to overcome the Coulombian force of
411 H + 2e − 
→ 42 He + 2v + 6 γ + 26.7 MeV
repulsion. As these reaction require very high temperature

EXERCISE 13.3
I. Objective Type Questions (1 Mark) (b) Both A and R are true but R is not the correct
1. Choose the correct answers from the given options explanation of A.
(MCQs). (c) A is true but R is false.
(i) Nuclear fission is possible (d) Both A and R are false.
(a) only between light nuclei 4. Assertion (A): A fission reaction can be more easily
(b) only between heavy nuclei controlled than a fusion reaction.
(c) between both light and heavy nuclei Reason (R): The percentage of mass converted to
(d) only between nuclei which are stable against energy in a fission reaction is 0.1% whereas in fusion
decay reaction is 0.4%.
(ii) In nuclear reactor, controlling rods are made of 5. Assertion (A): Cadmium rods, used in nuclear
(a) cadmium (b) graphite reactor, control the rate of fission.
(c) plutonium (d) stainless steel Reason (R): Cadmium rod speeds up the slow
2. Fill in the blanks. neutrons.
(i) In the fission of 235U nucleus on an average II. Very Short Answer Type Questions (1 Mark)
................. neutrons are released. 1. State with reason why heavy water is generally used
(ii) In a nuclear reactor, heavy water is used as a as moderator in a nuclear reactor. [Delhi 2008]
..................... which slows down the neutrons.
2. Name the absorbing material used to control the
(iii) The sun obtains its radiant energy from reaction rate of neutrons in a nuclear reactor.
......................... .
[Delhi 2008]
3. State True or False
(i) Nuclear fission is the process in which light nuclei OR
fuse together to form a heavy nucleus. Four nuclei of an element undergo fusion to form
(ii) The average energy released per fission of 235 a heavier nucleus, with release of energy. Which of
92U is
approximately 100 MeV. the two – the parent or the daughter nucleus – would
(iii) Plutonium was the fissionable material used in the have higher binding energy per nucleon?
bomb dropped at Nagasaki in 1945. 3. Why is nuclear fusion difficult to carry out?
For Q. 4 and 5 there are two statements labelled as 4. Out of nuclear fusion and fission, which reaction is
assertion (A) and reason (R). Select the correct answer more energetic?
to these questions from the codes (a), (b), (c) and (d) as 5. Name the reaction which takes place when a slow
given below. neutron beam strikes 92235
U nuclei. Write the nuclear
(a) Both A and R are true and R is the correct reaction involved.
explanation of A.
444 Physics–12
D:\EG_Physics-12_(26-06-2022)\Open_Files\Ch-13\Ch-13
\ 27-Jul-2022 Ved_Goswami Proof-4 Reader’s Sign _______________________ Date __________

6. Four nuclei of an element undergo fusion to form 8. What is nuclear fusion? Why is it called thermonuclear
a heavier nucleus, with release of energy. Which of reaction?
the two – the parent or the daughter nucleus – would 9. What do you mean by uncontrolled chain reaction?
have higher binding energy per nucleon? Give its one application.
[Delhi 2018] IV. Short Answer Type Questions-II (3 Marks)
III. Short Answer Type Questions-I (2 Marks) 10. Calculate and compare the energy released by
7. Distinguish between nuclear fission and nuclear (a) fusion of 1 kg of hydrogen deep within the sun
fusion. Explain how the energy is released in both and
the processes. (b) the fission of 1 kg of 235U in a fission reactor.

Answers 13.3
I. Objective Type Questions 8. Nuclear fusion is a process in which two light nuclei
1. (i) (a) (ii) (a) combine to form a heavier nucleus with liberation of
2. (i) 2.5  (ii) Moderator (iii) Fusion process huge amount of energy. Since nuclear fusion requires
3. (i) False (ii) False (iii) True very high temperature and pressure, it is also called
4. (b) 5. (c) thermonuclear reaction.
9. This is a chain reaction in which each neutron released
II. Very Short Answer Type Questions
in fission reaction is allowed to carry on the fission,
1. Heavy water contains protons and mass of a proton is thus reaction accelerates at a very rapid rate and entire
nearly equal to mass of a neutron. Fast moving neutrons material explodes within a microsecond liberating very
suffer elastic collision with these slow moving neutrons large amount of energy.
and thus get slowed down. Thus heavy water is used as
a moderator. Application: Atom bombs
2. Cadmium or Boron are absorbers of neutrons. They 10. (a) In sun 4 hydrogen nuclei combine to form a helium
nucleus and 26 MeV of energy is released
serve as controllers of reaction in the nuclear reactor.
3. Because nuclear fusion requires very high temperature 1 → 4 He + 2e + + 26 MeV
4 1H 2
~
− 106K. This temperature can be attained by causing a
nuclear fission only. No. of atom in 1g of hydrogen = 6.023 × 1023
4. Energy released per unit mass is more in fusion reaction No. of atom in 1kg of hydrogen = 6023 × 1023
as compared to fission. So fusion reaction are more Energy released by 1 kg of hydrogen
energetic. 26 × 6023 × 1023  39 × 1026 MeV
5. It is a nuclear fission reaction =
4
235
92 0 U + 1 n 
56 → 141Ba +
36 0
92
Kr + 3 1 n + Energy (b) No. of atom in 235g of

235U = 6.023 × 1023
6. The binding energy per nucleon of the heavier nuclei
is more than the binding energy per nucleon of the No. of atom in 1 kg of
lighter nuclei. So binding energy is more per nucleon 23 1000
in daughter nuclei. 235U = 6.023 × 10 ×
235
7. In nuclear fission a heavy nucleus breaks up into smaller Energy released in per fission of
nuclei accompanied by release of energy whereas in 235U = 200 MeV
nuclear fusion two light nuclei combine to form a Energy released in per fission of 1 kg of 235U
heavier nucleus accompanied by release of energy. In
both cases, some mass (mass defect) gets converted 6.023 × 1023 × 1000 × 200  5.1 × 1026 MeV
  =
into energy as per the relation E = Dmc2. 235

Case Based Questions


I. The density of nuclear matter is the ratio of the mass density of nuclear matter is independent of the size of
of a nucleus to its volume. As the volume of a nucleus the nucleus. This nuclear matter behaves like a liquid
is directly proportional to its mass number A, so the of a constant density. Different nuclei are like drops

Nuclei 445
D:\EG_Physics-12_(26-06-2022)\Open_Files\Ch-13\Ch-13
\ 27-Jul-2022 Ved_Goswami Proof-4 Reader’s Sign _______________________ Date __________

of this liquid of different sizes but of same density. order of 10–14m and is thus 10000 times smaller than
Let A be mass number and R be radius of a nucleus. the size of atom.
If m is the average mass of a nucleon then mass of 1. What is the ratio of mass of nucleus to the mass
nucleus = mA of atom?
Volume of nucleus
2. What are the atoms with the same number of
4 3 4
( ) 4
3
= πR = π R 0 A
1/3
= πR 30 A neutron but different atomic number called?
3 3 3
3. If R is the radius and A is mass number then what
\ Nuclear density

will be the nature of log R vs log A graph?
Mass of nucleus
rnu = Ans. 1. Ratio of mass of nucleus to mass of atom
Volume of nucleus
99.9
mA 3m = = 0.999  1
or rnu = 4 =     100
πR 30 A 4πR 30
3 2. Atoms with same number of neutron but different
Clearly, nuclear density is independent of mass atomic number are called isotones.
number A or the size of the nucleus. 3. The graph log R vs log A will be a straight line.
The nuclear mass density is of the order 1017 kg m–3. III. Nuclear Fission: When a heavy nucleus splits into
This density is very large as compared to the density two or more medium intermediate mass nuclei the
of ordinary matter, say water, for which r = 1.0 × 103 phenomenon is called nuclear fission. For example
kg m– 3. when Uranium-235 is bombarded by a thermal
1. What is the density of hydrogen nucleus? neutron, it splits Krypton and Barium with 2 or 3
56
2. The nuclear mass of 26 Fe is 55.85 amu. What is neutrons and large amount of energy.
its nuclear density?
16
3. The nuclear radius of 8 O is 3 × 10– 15 m. What
is the density of nuclear matter?
3m p
Ans. 1. Density r =
4 pR 30
− 27
= 3 × 1.007825 × 1.66 × 10
22
4× × (1.1 × 10− 15 )3
7
= 2.98 × 1017 kg m–3
Hence the density of H-nucleus is 2.98 × 1017 kg 235 U + 10 n → 92 Kr + 141
56 Ba
+ 310 n + 179.4 MeV
m–3. 92 36

mFe 9.27 × 10− 26 Fission fragments get very large kinetic energies

2. rnu = = due to coulomb force of repulsion. As a result of
4 3 4π
πR 0 × (1.2 × 10−15 )3 collisions, this energy appears as thermal energy. In
3 3 figure a graph between mass difference per nucleon
   = 128.13 × 1017 kg m– 3 [M – Zmp – (A – Z)mn]/A versus A in units of MeV/
16 × 1.66 × 10−27 c2 is shown in figure.
3. rnu = 4 = 2.4 × 1017 kg m–3
π (3 × 10 −15 3
) ● This curve is just the negative of the binding energy
3 curve. Figure shows that the rest energy per particle
II. The nucleus was first discovered in 1911 by Lord of heavy nuclides (A ≥ 200) is more as compared to
Rutherford and his associates by experiments on nuclides of intermidiate mass.
scattering of a-particles by atoms. He found that ● Similarly the rest energy per particle of light
the scattering results could be explained, if atoms nuclides (A ≤ 20) is more as compared to nuclides
consist of a small central massive and positive core of intermediate mass.
surrounded by orbiting electrons. The experimental
● In fission the total mass decreases and energy is
results indicated that the size of the nucleus is of the
released. For A = 200 the rest energy is about 1

446 Physics–12
D:\EG_Physics-12_(26-06-2022)\Open_Files\Ch-13\Ch-13
\ 27-Jul-2022 Ved_Goswami Proof-4 Reader’s Sign _______________________ Date __________

MeV per nucleus greater than for A = 100. Hence The energy released per unit mass is much more than
in a nuclear fission where approximately equal size the energy released per unit mass in fission. But these
fragments are formed about 200 MeV is released. reactions require extremely high temperature ~ 106
● Several neutrons are emitted in the fission process K for the fusing nuclei to overcome the Coulombian
which can cause further fission, thereby producing a force of repulsion. As these reaction require very
chain reaction. When 235U captures a thermal neutron high temperature and pressure, they are also called
the probability resulting 236U nucleus undergoes thermonuclear reaction.
fission is about 85% and of emitting gamma rays as Nuclear fusion is the source of energy in the sun.
it de-excites to the ground state is about 15%. Due to the need of very high temperature and pressure
these reaction cannot be carried out in laboratory.
1. A neutron most effective as a bullet in nuclear The fusion reaction in the sun is a multi step process.
reaction because The proton-proton cycle is :
(a) neutron are lighter than proton 1 1
→ 12 H + e+ + v + 0.42 MeV ...(i)
1 H + 1H 

(b) neutron carries no charge
(c) neutron have high penetration power e + + e − 
→ γ + γ + 1.02 MeV ...(ii)
(d) neutron are more active than other particles 2 1
→ 32 He + γ + 5.49 MeV ...(iii)
1 H + 1H 

2. It is necessary to slow down the neutrons,
produced through the fission of 235 92
U nuclei 3

3
→ 42 He + 11H + 11 H + 12.86 MeV
2 He + 2 He 
(by neutrons), to sustain a chain reaction because ...(iv)
(a) slow neutrons have less fission capacity
If we consider 2 × (i) + 2(ii) + 2(iii) + (iv)
(b) slow neutrons have much higher fission
capacity 411 H + 2e − 
→ 42 He + 2v + 6γ + 26.7 MeV

(c) slow neutrons have high potential energy
1. Given:
(d) slow neutrons are unstable
m( 12 H ) = 2.014102 u, m( 13 H ) = 3.0161049 u

3. The nuclei preferably needed for slowing down
fast neutrons are called m( 24 He) = 4.002603 u, m( 10 n ) = 1.008665 u and

(a) moderators (b) control rods 1u = 931 MeV/c2
(c) heavy element (d) Anti-moderators
Energy released for the following Deuteron-
4. A neutrons is absorbed by a 63 Li nucleus with the Tritium (D-T) reaction
subsequent emission of an alpha particle. Nuclear 12 H + 13 H " 24 He + 10 n will be
reaction is given as 63 Li + 10 n " 24 He + 13 H + Q .
Given: Mass 63 Li = 6.015126 u, mass (neutron) (a) 25.12 MeV (b) 13.2 MeV
= 1.0086654 u, mass (alpha particle) = 4.002604 (c) 15.75 MeV (d) 17.56 MeV
u, and mass (triton) = 3.0100000 u. Take 1 u = 2. In the deuterium-tritium fusion reaction
931 MeV/c2. The energy released in MeV in this the Q-value is 17.6 MeV. Rate at which
reaction will be deuterium is consumed to produce 1 MW, will be
(a) 9.071 MeV (b) 12.23 MeV (a) 5.25 × 10–6 kg/s (b) 1.179 × 10–9 kg/s
(c) 11.145 MeV (d) 10.415 MeV (c) 2.09 × 10–9 kg/s (d) 3.25 × 10–8 kg/s
5. The force between two fission fragments of 3. It is proposed to use the nuclear fusion reaction
equal masses and sizes that are produced in
2 H + 2 H " 4 H e, then the energy released in this
the fission of 23994
Pu (by a thermal neutron) 1 1 2
in which 4 neutrons are emitted will be reaction will be
[Given R0 = 1.2 fm.] (Given the masses of 12 H and 24 He are 2.0141u
(a) 3.76 × 103 N (b) 4.575 × 102 N and 4.0026u respectively.)
(c) 5.676 × 103 N (d) 6.334 × 103 N (a) 4.21 u (b) 0.357 u
Ans. 1. (b) 2. (b) 3. (a) 4. (d) 5. (a) (c) 0.0256 u (d) 5.25 u
IV. Nuclear Fusion: It is the process in which two 4. Given: Mass of 1 H = 1.007825 u and
1

lighter nuclei combine to form a heavier nucleus with Mass of helium nucleus = 4.002603 u, 1 u = 931
liberation of large amount of energy. MeV/c2.
Nuclei 447
D:\EG_Physics-12_(26-06-2022)\Open_Files\Ch-13\Ch-13
\ 27-Jul-2022 Ved_Goswami Proof-4 Reader’s Sign _______________________ Date __________

When four hydrogen nuclei combine to form a 5. The approximate value of energy released by
helium nucleus the amount of energy in MeV fusion of 1 kg of hydrogen deep within the sun is
released in this process of fusion will be given by (a) 28 × 1024 MeV (b) 39 × 1026 MeV
(a) 21.2 MeV (b) 25.71 MeV
(c) 22 × 1024 MeV (d) 21.5 × 1026 MeV
(c) 25.7 MeV (d) 32.7 MeV
Ans. 1. (d) 2. (b) 3. (c) 4. (b) 5. (b)

IMPORTANT FORMULAE
S.NO. Formula Symbols Application
1. R = R0 A1/3 R0 = 1.2 × 10–15 m Relation between Radius of
nucleus and mass number
2. m Density of a nucleus
ρ= r = Nuclear density,
4
π R 03 m = average
3 mass of a nucleon.
3. ∆m = [Zmp + (A–Z)mn]– MN MN = Mass of nucleus • To find Q-value
• In nuclear reactor
4. (i) B.E. = D m c2 B.E. = Binding Energy Relation between binding energy
B.E. Dm = mass defect and mass defect.
(ii) B= 1. a.m.u = 931.5 MeV
A
B = Binding energy per nucleon • To study nuclear forces

COMMON ERRORS
S No. Errors Corrections
1. Students get confused in use of the term nucleons. Nucleons indicates both protons and neutrons.

2. In deciding the value of N. N denotes the numer of the remaining nuclei at any
instant t.

448 Physics–12
D:\EG_Physics-12_(26-06-2022)\Open_Files\Ch-13\Ch-13
\ 27-Jul-2022 Ved_Goswami Proof-4 Reader’s Sign _______________________ Date __________

REVISION CHART
Atomic Number Mass Number
The number of protons in the nucleus of an atom of the element The total number of protons and neutrons present inside the
is called atomic number (Z) of the element. nucleus of an atom of the element is called mass number (A)
of the element.

Nucleus
The entire positive charge and nearly the entire mass of atom is concentrated in a very small space called the nucleus of an atom.

Nuclear Size Nuclear Density


The radius of the nucleus It is independent of mass number and therefore same for all
R  A1/3 ⇒ R = R0A1/3. Nuclei.

Atomic mass unit (a.m.u) Isotopes


It is defined as 1/12th the mass of a carbon atom. The atom of an element having same atomic number but
different mass numbers are called isotopes.

Isobars Isotones
The atoms of different elements having same mass number but The atoms of different element having different atomic
different atomic numbers are called isobars. numbers and different mass numbers but having same number
of neutrons, are called isotones.

Mass Defect
Nuclear Force
The difference between the sum of masses of all nucleons (m)
The force acting inside the nucleus or acting between nucleons and mass of the nucleus (M) is called mass defect.
is called nuclear force.

Nuclear binding energy


Nuclear Chain Reaction The minimum energy required to separate the nucleons upto
If the particle starting the nuclear fission reaction is produced an infinite distance from the nucleus, is called nuclear binding
as a product and further take part in the nuclear fission reaction, energy.
then a chain of fission reaction started, which is called nuclear
chain reaction.
Nuclear Fission
The process of the splitting of a heavy nucleus into two or more
lighter nuclei is called nuclear fission.
Nuclear Fusion
The process of combining of two lighter nuclei to form one
heavy nucleus, is called nuclear fusion. Nuclear Reactor
The main parts of a nuclear reactor are following:
(i) Fuel Fissionable materials like 92U235, 92U238, 94U239 are
used as fuel.
Thermonuclear Energy (ii) Moderator Heavy water, graphite and beryllium oxide
The energy released during nuclear fusion is know as are used to slower down fast moving neutrons.
thermonuclear energy. Protons are needed for fusion while (iii) Coolant the cold water, liquid oxygen, etc. are used to
neutrons are needed for fission process. remove heat generated in the fission process.
(iv) Control rods Cadmium or Boron rods are good absorber
of neutrons and therefore used to control the fission
reaction.

Nuclei 449
D:\EG_Physics-12_(26-06-2022)\Open_Files\Ch-13\Ch-13
\ 27-Jul-2022 Ved_Goswami Proof-4 Reader’s Sign _______________________ Date __________

IMPORTANCE OF EACH TOPIC AND FREQUENTLY ASKED TYPES OF QUESTIONS

☞ Important Topics
1. Binding Energy
2. Q-value

1. Identify the nuclides X and Y in the nuclear reactions:


1 8 14
5 B + 1H
      11 4 Be + X; 6 C Y + –01 e [Ans. X = 24 He and Y = 14
7 N]

2. What do you mean by Q-value of a nuclear reaction?


[Ans. The amount of energy released or absorbed in a nuclear reaction is called its Q-value.
a B b Q
A
Nucleus + Elementary Nucelus + Elementary + Energy ]
particle particle

3. Binding energy of 2 He4 and 3 Li7 nuclei are 27.37 MeV and 39.4 MeV respectively. Which of the two nuclei is
more stable? Why? [Ans. 2He4, because its BE/nucleon is greater]

4. Find the binding energy and binding energy per nucleon of nucleus 83Bi209. (Given: mass of proton = 1.0078254
u, mass of neutron = 1.008665u, mass of 83Bi209 = 208.980388u.)  [Ans. 1639.38 MeV and 7.84 MeV/Nucleon]

5. Is the fission of iron (26Fe56) into (13Al28) as given below possible?


56 28 28
         26Fe → 13Al + 13A1 + Q
Given mass of 26Fe56 = 55.934940 and 13AI28 = 27.98191 u
[Ans. Since Q value comes out negative, so this fission is not possible]

450 Physics–12
D:\EG_Physics-12_(26-06-2022)\Open_Files\Ch-13\Ch-13
\ 27-Jul-2022 Ved_Goswami Proof-4 Reader’s Sign _______________________ Date __________

ASSIGNMENT
I. Objective Type Questions (1 Mark)
1. Multiple choice questions:
(i) Which of the following particles has similar mass to that of the protons?
(a) Proton     (b) Neutron     (c) Positron     (d) Neutrino
(ii) The mass of neutron and proton are 1.0087 and 1.0073 u respectively. If the neutrons and protons combine to
form a helium nucleus of mass 4.0015 u, the binding energy of the helium nucleus will be
(a) 28.4 MeV   (b) 20.8 MeV    (c) 27.3 MeV    (d) 14.2 MeV
2. Fill in the blanks:
(i) Proton was discovered by ................... and neutron was discovered by ................. .
(ii) Neutrino is a particle with ........................ .
II. Very Short Answer Type Questions (1 Mark)
3. Electrons cannot be a part of nucleus but protons can be. Why?
4. You are given two nuclei 3X7 and 3Y4. Explain giving reasons, as to which one of the two nuclei is likely to be
more stable?
III. Short Answer Type Questions-I (2 Marks)
5. The figure shows the plot of binding energy (BE) per nucleon as a function of mass number A. The letters A, B,
C, D and E represent the positions of typical nuclei on the curve. Point out, giving reasons, the two processes (in
terms of A, B, C, D and E), one of which can occur due to nuclear fission and the other due to nuclear fusion.

B.E
A C D
E
B
A
Mass Number A
6. A heavy nucleus X of mass number A = 240 and B.E./A = 7.6 MeV is split into two nearly equal fragments Y and
Z of mass numbers A1 = 110 and A2 = 130. The binding energy of each one of these nuclei is 8.5 MeV per nucleon.
Calculate the total binding energy of each of the nuclei X, Y and Z and hence the energy Q released per fission in
MeV.
IV. Short Answer Type Questions-II (3 Marks)
7. Explain the processes of nuclear fission and nuclear fusion by using the plot of binding energy per nucleon (BE/A)
versus the mass number A.
238
8. In a fission event of 92 U by fast moving neutrons, no neutrons are emitted and final products after the beta decay
of the primary fragments, are 140 99
58 Ce and 44 Ru . Calculate Q for this process. Neglect the masses of electrons,
positrons emitted during the intermediate steps
Given: m( 238
92 U ) = 238.05079 u m ( Ce) = 139.90543 u
140
58 [CBSE 2022]

m( Ru ) = 98.90594 u m ( n ) = 1.008665 u
99 1
44 0



Nuclei 451
D:\EG_Physics-12_(26-06-2022)\Open_Files\Ch-14\Ch-14
\ 27-Jul-2022 Ved_Goswami Proof-4 Reader’s Sign _______________________ Date __________

Topics Covered
14 Semiconductor Devices

14.1 Semiconductors 14.2 pn-Junction diode

C hapter map
Semiconductor
Devices

Semiconductors Junction Diodes

Intrinsic Extrinsic PN-Nunction


Diodes

Pure Semi- p-type and n-type Potential barrier


conductor Semi-conductor and Field

Depletion Region

Valence Band Conduction


Band Rectifier (half wave
& full wave)

Energy Gap Filter

Topic 1. Semiconductors
•• Definition: They are the substances whose conductivity (b) On the basis of source and nature of charge carriers:
is more than that of insulators, but less than that of (i) Intrinsic (ii) Extrinsic
conductors. •• Formation of Energy Bands is Solids: Electrons have well
Conductivity of : defined energy levels in an isolated atom but as the atoms
(i) Metals : 102 to 108 S m–1 come closer to form a crystal, then there is maximum
(ii) Insulators : 10–8 S m–1 interaction between valence electrons of different atoms.
(iii) Semi conductors : 10–5 to 10° S m–1 So the electrons of outer shells are forced to have energies
different from those of isolated atoms. So each energy
•• Classification of Semiconductors:
level splits into a number of energy levels, very closely
(a) On the basis of chemical composition :
spaced and these set of closely spaced energy levels are
(i) Elementary semiconductors : Eg – Si, Ge called energy bands.
(ii) Compound semiconductors : They can be organic •• Valence Band: The highest energy bands filled with
or inorganic valence electrons bound in the bond are valence bands.
Organic – Polypyrrole, polyaniline, polythiophene •• Conduction Band: The lowest unfilled allowed energy
Inorganic – CdS, GaAs, InP, CdSe band next to the valence band is conduction band.
452
D:\EG_Physics-12_(26-06-2022)\Open_Files\Ch-14\Ch-14
\ 27-Jul-2022 Ved_Goswami Proof-4 Reader’s Sign _______________________ Date __________

•• Energy Gap (Eg): The gap between top of valence band Conduction
and bottom of conduction band in which no allowed band
energy levels of electrons can exist is called energy band Valence band
gap/energy gap.
Here, the VB is completely filled but due to overlap
CB between CB and VB, energy gap is 0.
} Eg •• Fermi Level: The highest energy level in conduction band
filled up with electrons at absolute zero of temperature
VB is called fermi level.
•• Distinguish between metal, semiconductor and insulator •• Types of semiconductors: Intrinsic and extrinsic
on basis of band theory Intrinsic Semiconductors: Pure semiconductors are
(i) Insulators: called intrinsic semiconductors. The number density of
At 0°K (absolute zero) free electrons is equal to number density of holes.
ne = nh = ni
CB (Completely empty) where ni = no. density of intrinsic semiconductor.
} Eg [Band gap is high] Eg > 3eV
For any pure semiconductor, ne nh = ni2,
VB (Completely filled)
Electrical conductivity is low and it depends on
temperature. Conductivity will increase with increase
`Even at room temperature gap is very high, so in temperature, as more and more bonds will break and
conduction cannot occur. electrons will become free to conduct electricity.
(ii) Semiconductors I = Ie + Ih
At 0°K : At 0°K, all semiconductors are insulators. •• Mechanism of conduction in pure semiconductors on the
basis of valence band model
CB Consider crystal of semiconductor Ge. Each Ge atoms is
} Eg < 3eV
tetrahedrally bonded to 4 neighbouring Ge atoms. Such
a crystal structure is called diamond like structure.
VB
The shared pair of electrons oscillates between the only
At room temperature: Some of the electrons gain
two associated atoms. But such a structure with all bonds
sufficient energy to come out of the bond and
intact (none broken) exists only at low temperature.
become free.
Electron
CB Ge Ge Ge
} Eg
Hole
VB
(iii) Metals/conductors Case I: At 0°K as well as at room
temperature : The energy gap between completely Ge Ge Ge

filled valence band and partially filled conduction


band is very less, i.e. CB is partially filled.

Ge Ge Ge
CB (partially filled)

As temperature increases, thermal energy of valence


VB (filled)
electrons increases. An electron may break away from the
This structure is found in alkali metals, e.g. Li, Na, covalent bond and becomes free to conduct electricity.
K. This electron leaves behind a vacancy of an electron with
Case- II. The conduction band and valence band a effective + ve electron charge, called a hole. It behave
may also partially overlap. as an apparent free particle with charge + e.

Semiconductor Devices 453


D:\EG_Physics-12_(26-06-2022)\Open_Files\Ch-14\Ch-14
\ 27-Jul-2022 Ved_Goswami Proof-4 Reader’s Sign _______________________ Date __________

(ii) By heating crystalline semiconductor in an


atmosphere containing dopant atoms.
•• Extrinsic Semiconductors: A semiconductor doped with
Ge Ge Ge

some suitable impurity atoms so as to increase its number


of charge carriers are called extrinsic semiconductors.
Hole They are of two types : n-type and p-type.
Ge Ge Ge
•• n-type semiconductors: They are obtained by doping
the semiconductor with pentavalent impurities such as
P, As, Sb (elements of group 15).
Ge Ge Ge
At 0°K

CB
Thermally generated free electron
0.1 eV Donor energy level
The process of setting free an electron from a covalent
bond and simultaneous creation of a hole requires a kind
of ionisation energy (Eg). The number of electrons, ne VB (filled)
set free at absolute temperature T is given by
ne = Ce–Eg/2kT At room temperature
where k = Boltzman constant
For given Eg, ne increases as temperature increases.
CB
•• Holes and Electrons: Electrons move from lower to
higher energy level, whereas holes move from higher to
lower energy levels in the valence band.
As the conventional direction of current is the direction
in which +ve charge is moving, so though electrons and VB
holes move opposite to each other, they carry current in
the same direction.
When pentavalent impurity is added, 4 of its valence
I = Ie + Ih electrons form bonds with the neighbouring silicon atoms
Holes are the vacancy of an electron in the band. They while the 5th electron is loosely bound to the impurity
can move only in the valence band, whereas electrons atom.
can move in conduction band also.
•• The mobility of holes is much less than that of electrons
1. Doping: It is the deliberate addition of an impurity
in a pure semiconductor, so as to increase its
conductivity.
2. Essential requirements of doping
(i) Dopant atom should nearly replace the
semiconductor atom.
(ii) Size of dopant atom should be almost same as that
of semiconductor atom.
For this reason, atoms of 13th and 15th group are
most suitable.
(iii) Concentration of dopant atom should be small i.e.
1 ppm.
(iv) Dopant atom should not distort crystal lattice. A very small amount of energy is required to detach this
•• Methods of doping: electron. At room temperature, the thermal energy is
(i) By adding impurity atoms to an extremely pure enough to make this electron free. These semiconductors
sample of a molten semiconductor. have free electrons contributed by donors and generated

454 Physics–12
D:\EG_Physics-12_(26-06-2022)\Open_Files\Ch-14\Ch-14
\ 27-Jul-2022 Ved_Goswami Proof-4 Reader’s Sign _______________________ Date __________

by the thermal process, while the holes are only due to There are holes created by acceptor atoms in addition to
thermal generation. So the number density of electrons the thermally generated holes, while the free electrons are
is much more than the number density of holes. only due to thermal generation. Hence, holes are majority
charge carrier and electrons are minority charge carriers
ne >>> nh
and electrons are minority charge carriers.
But still, nenh = ni2 Most of the current here is carried by holes, which have
The electrons contributed by donor atom will occupy the effective positive charge.
energy level just below the conduction band. This energy nh >>> ne
level is called donor energy level.
But still, nhne = ni2
•• p-type Semiconductors: It is obtained by doping the The holes created by the acceptor atom will occupy a
semiconductor with trivalent impurities like Al or Ga. energy level just above the valence band. This energy
At 0°K level is called acceptor energy level.
•• Resistivity of a semiconductor: Consider a semiconductor
of length l and area of cross-section ‘A’. A potential
difference V is applied across it.
Let ne = number density of electrons
nh = number density of holes
At room temperature V
E=
l
CB The electrons will drift in a direction opposite to that
of holes, but they both will carry current in the same
direction.
VB E

The impurity atom forms covalent bonds with the three


neighbour Si atoms and one covalent bond is left due to
deficiency of one electron.

V
Si Si Si
I = Ie + Ih = nceAve + nheAvh
= eA(neve + nhvh)
V
= eA(neve + nhvh)
Si Al Si R
VA
= eA(neve + nhvh)
ρl

E
= e(neve + nhvh)
Si Si Si ρ
1 e
Hole = (ne ve + nh vh )
ρ E
An electron can slide into this vacant bond, creating a
vacancy/hole in that bond. 1
= e ( ne µ e + nh µ h ) where µ = mobility
The trivalent impurity atom is called acceptor because ρ
it creates a hole which can accept an electron from the
\ Conductivity s = e(neme + nhmh)
neighbouring bond.

Semiconductor Devices 455


D:\EG_Physics-12_(26-06-2022)\Open_Files\Ch-14\Ch-14
\ 27-Jul-2022 Ved_Goswami Proof-4 Reader’s Sign _______________________ Date __________

EXERCISES 14.1
I. Objective Type Questions (1 Mark) For Q. 5 there are two statements labelled as assertion
1. Choose the correct answers from the given options (A) and reason (R). Select the correct answer to these
(MCQs). questions from the codes (a), (b), (c) and (d) as given
(i) The conductivity of a semiconductor increases below.
with increase in temperature, because (a) Both A and R are true and R is the correct
explanation of A.
(a) number density of free current carriers
increases. (b) Both A and R are true but R is not the correct
explanation of A.
(b) relaxation time increases.
(c) A is true but R is false.
(c) both number density of free current carriers
(d) Both A and R are false.
and relaxation time increases.
5. Assertion (A): Silicon is preferred over germanium
(d) number density of current carriers increases
for making semi-conductor devices.
and relaxation time decreases, but effect of
decrease in relaxation time is much less than Reason (R): The energy band for germanium is more
increase in number density. than the energy band of silicon.
(ii) Hole in semiconductor is II. Very Short Answer Type Questions (1 Mark)
(a) an antiparticle of electron. 1. A semiconductor has equal electron and hole
concentration of 6 × 104 m–3. On doping with a certain
(b) a vacancy created when an electron leaves a
impurity, electron concentration increases to 8 × 1012
covalent bond.
m–3. Identify the type of semiconductor.
(c) absence of free electrons.
2. Name the factors on which electrical conductivity of a
(d) an artificially created particle. pure semiconductor at a given temperature depends?
2. Fill in the blanks. 3. How does the conductivity of a semiconductor change
(i) In insulator, the valence electrons form a band with the rise in its temperature?
that is ......................... . 4. Give the ratio of number of holes and the number
(ii) In the band structure of an intrinsic semiconductor of conduction electrons in a (i) pure semiconductor,
is located ........................ . (ii) n-type semiconductor and (iii) p-type
(iii) A small impurity is added to Ge to get a p-type semiconductor.
semiconductor and this impurity is called 5. Carbon and silicon are known to have similar lattice
............................. . structures. However, the four bonding electrons of
carbon are present in second orbit while those of
3. State True or False
silicon are present in its third orbit. How does this
(i) In a semiconductor, the forbidden energy gap difference result in difference in their electrical
between the valence band and the conduction conductivities? [S.P. 2008]
band is of the order of 1 eV.
6. An n-type semiconductor has large number of
(ii) If the conductivity of a semiconductor is only due electrons but still it is electrically neutral. Explain.
to break of the covalent band due to the thermal [Delhi 2008]
excitation, then the semiconductor is called
7. How does the energy gap of an intrinsic semiconductor
intrinsic.
vary, when doped with a trivalent impurity?
(iii) A hole in a p-type semiconductor is a missing
8. C, Si and Ge have same lattice structure. Why is C
atom.
insulator while Si and Ge intrinsic semiconductor?
4. Match the Columns 9. Name majority current carriers in n-type
Column-I Column-II semiconductor.
(i) Energy gap Eg = 0 (a) semiconductor 10. Name majority current carriers in p-type
(ii) Eg < 3 eV (b) Insulator semiconductor.
(iii) Eg > 3 eV (c) ne >> nh 11. Why is Ge preferred over Si for making semiconductor
(iv) n-type (d) Conductors devices?

456 Physics–12
D:\EG_Physics-12_(26-06-2022)\Open_Files\Ch-14\Ch-14
\ 27-Jul-2022 Ved_Goswami Proof-4 Reader’s Sign _______________________ Date __________

12. Why is the conductivity of n-type semi-conductor 24. Distinguish between intrinsic and extrinsic semi-
greater than that of the p-type semiconductor even conductors on the basis of energy band diagram.
when both of these have same level of doping? [Delhi 2015]
13. At what temperature would an intrinsic semiconductor IV. Short Answer Type Questions-II (3 Marks)
behave like an insulator? [Delhi 2008 C]
25. (a) Draw the necessary energy band diagrams to
14. Which has higher mobility — electrons or holes?
distinguish between conductors, semiconductors
15. Name the current carriers in a semiconductor. and insulators.
16. Is the current due to electrons and holes in the same
(b) How does the change in temperature affect the
directions?
behaviour of these materials? Explain briefly.
17. What type of extrinsic semiconductor is formed when:
[AI 2015]
(i) germanium is doped with indium?
(ii) silicon is doped with bismuth? 26. Distinguish between n-type and p-type of
18. The energy gaps in energy band diagrams of a semiconductors on the basis of energy band diagrams.
conductor, semiconductor and insulator are E1, E2 Compare their conductivities at absolute zero
and E3. Arrange them in increasing order. temperature and at room temperature.
III. Short Answer Type Questions-I (2 Marks) [Delhi 2015 C]
19. How does the energy gap in an intrinsic semiconductor 27. Write any two distinguishing features between
conductors, semiconductors and insulators on the
vary, when doped with (a) a pentavalent impurity and
basis of energy band diagram.
(b) a trivalent impurity? Draw their energy band
28. D i ff e r e n t i a t e b e t w e e n n - t y p e a n d p - t y p e
diagrams.
semiconductors on the basis of energy band diagram.
20. Distinguish between n-type and p-type semiconductor. Explain the process of conduction in both type of
21. A semiconductor has equal electron and hole materials.
concentration of 2 × 108 m–3, concentration increases 29. Two material bars A and B of equal area of cross-
to 4 × 1010 m–3. (i) What type of semiconductor is section, are connected in series to a DC supply. A
obtained on doping? (ii) Calculate the new electron is made of usual resistance wire and B of an n-type
hole concentration of the semiconductor. (iii) How semiconductor.
does the energy gap vary with doping?
(a) In which bar is drift speed of free electrons
22. Why are elemental dopants for silicon or germanium
greater?
usually chosen from group 13 or group 15?
[NCERT Exemplar] (b) If the same constant current continues to flow for
23. Sn, C, Si and Ge are all group 14 elements. Yet Sn a long time, how will the voltage drop across A
is conductor, C is an insulator and Si and Ge are and B be affected? [CBSE S.P. 2018-19]
semiconductor. Why? [NCERT Exemplar][HOTS] Justify each answer.

Answers 14.1
I. Objective Type Questions 3. Conductivity of a semiconductor increases with increase
1. (i) (d) (ii) (b) in temperature.
2. (i) valence band which is completely filled nh nh nh
4. (i) =1 (ii) < 1 (iii) >1
(ii) middle of the valence band and conduction band ne ne ne
(iii) Trivalent 5. As the binding energy of electrons is less in the higher orbit
3. (i) True (ii) True (iii) False and more in the lower (inner) orbits. So the energy required
4. (i)-(d) (ii)-(a) (iii)-(b) (iv)-(c) to take out an electron from Si atom is much less than that
5. (d) in case of C atom. So the number of free electrons in Si is
much more than in C. So conductivity of Si is much larger
II. Very Short Answer Type Questions than that of C.
1. n-type, as ne >> nh 6. In n-type semiconductor, the tetravalent impurity atom
2. (i) The number density of free current carriers. shares four of its valence electrons with four valence
(ii) Energy gap. electrons of the host atom, while the fifth electron

Semiconductor Devices 457


D:\EG_Physics-12_(26-06-2022)\Open_Files\Ch-14\Ch-14
\ 27-Jul-2022 Ved_Goswami Proof-4 Reader’s Sign _______________________ Date __________

remains almost free. This impurity atom as a whole is


Conduction
neutral. So the semiconductor is neutral.
Band
7. When an intrinsic semiconductor is doped with the
impurity atoms of valency three like indium or boron, Acceptor
some allowed energy levels are produced, situated in energy level
the energy gap slightly above the valence band. These Valence
levels are called acceptor energy levels. So the energy Band
gap decreases.
8. In C, Si and Ge, their valence electrons are present in 20. n-type p-type
second, third and fourth orbit respectively. The energy
required to take an electron will be maximum in case 1. n-type semiconduc- It is obtained by doping
of C and least in case of Ge. Due to this, the number tor is obtained by the semiconductor
of free electrons will be significant in case of Ge and doping with pentava- with trivalent impurity.
Si, but will be negligibly small in case of C. So C is an lent impurity.
insulator, while Si and Ge are intrinsic semiconductor.
2. An extra energy level An extra energy level
9. Electron, ne >>> nh
called donor energy called acceptor energy
10. Holes, nh >>> ne level is created in level is created in energy
11. Energy gap in Ge is less than the energy gap for Si. the energy gap, just gap, just above the
12. In n-type semiconductor ne >> nh. below the the bottom valence band.
Under a given electric field the mobility of free electrons of conduction band.
is more than of holes. Conduction Conduction
13. At 0°K band band
14. An electron has higher mobility than hole in a Donor
energy level
semiconductor.
15. Electrons and holes are the current carriers in a Valence
band
semiconductor.
16. Yes, the current due to electrons and holes in a
semiconductor is in the same direction. 3. The majority current The majority current
carriers are electrons. carriers are holes. nh >>>
17. (i) As indium is trivalent, so it will make p-type
ne >>> nh ne
semiconductor.
(ii) As bismuth is pentavalent, so it will make n-type 21. ni = 2 × 108 m–3
semiconductor. On doping nh = 4 × 1010 m–3
18. E1 < E2 < E3 (i) As on doping the number density of holes have
19. (a) When an intrinsic semiconductor is doped with a increased so it is a p-type semiconductor.
pentavalent impurity, an extra energy level called
(ii) ni2 = nenh
donor energy level is created in the energy gap just
below the bottom of conduction band. ni2 (2 × 108 ) 2
ne =
= = 106 m −3
Conduction nh 4 × 1010
Band
(iii) Energy gap decreases with doping due to the creation
Donor energy
of acceptor energy level.
level
22. The size of the dopant atom should be almost same as
Valence that of the pure semiconductor so that it does not distort
band the pure semiconductor lattice structure and yet easily
contribute to current carrier.
(b) When an intrinsic semiconductor is doped with
a trivalent impurity, an extra energy level called 23. In case of Sn, energy gap is zero as valence band
acceptor energy level is created in the energy gap and conduction band are overlapping so it acts as a
just above the valence band. conductor.

458 Physics–12
D:\EG_Physics-12_(26-06-2022)\Open_Files\Ch-14\Ch-14
\ 27-Jul-2022 Ved_Goswami Proof-4 Reader’s Sign _______________________ Date __________

In case of C, the valence electrons is in the inner orbit 27. Conductors Semiconduc- Insulators
as compared to Si and Ge. So energy gap is large  tors
5.4 eV in case of C, 1.1 eV for Si and 0.7 eV for Ge. 1. Either there Valence band Valence band
So C is insulator and Si and Ge are semiconductors. is overlap- and conduc- and conduction
Extrinsic ping of va- tion band are band are sepa-
24. Intrinsic semiconductor lence band separated by rated by a large
semiconductor
and conduc- a small energy energy gap i.e.,
1. They are pure semicon- They are doped with
tion band or gap. Eg >3eV
ductors, not doped with trivalent or pentava-
conduction i.e., Eg < 3 eV
any impurity. lent impurity.
band is par-
2. Number density of free ne ≠ nh
tially filled
electrons is equal to num-
i.e., Eg = 0
ber density of holes. i.e.,
ne = nh 2. They have T h e y h a v e They have no
large num- v e r y s m a l l free electrons in
Electron
ber of free n u m b e r o f the conduction
CB
3. } Eg electrons in free electrons band to conduct
Hole conduction in conduction electricity.
VB b a n d f o r band for con-
conduction. duction
25. (a) 28. For difference between n-type and p-type on the basis
Insulator Semiconductor of energy band diagram, refer to the answer of Q. No.
20.
Conduction in n-type of semiconductor : In n-type of
Eg > 3eV } Energy
gap (Eg) { } Eg < 3eV semiconductors, donor energy level lies just below the
bottom of conduction band. The electrons from the donor
Filled
valence
atoms which occupy this level jumps to empty energy
band levels of conduction band, with a very small supply of
energy. So the conduction band has large number of
free electrons. When an electric field is applied, there
electrons drift opposite to the applied field.
Conduction is p-type of semiconductors : In p-type

of semiconductors, acceptor energy level is just above
Overlapped conduction and valence band the top of valence band. When small amount of energy
(b) With increase in temperature, resistivity of a is supplied, the electrons from the valence band jump to
conductor increases because of increase in no. acceptor energy level, leaving behind holes in valence
of collisions. So relaxation time decreases which band. When an electric field is applied, the electrons
increases resistivity. from the neighbouring covalent bonds move into these
holes, creating new holes. So the holes move in the
With increase in temperature, resistivity of a
semiconductor decreases. Because of increase in direction of the field. Thus the holes act as positive
temperature more number of electrons gain energy charge carriers in the valence band.
to jump to conduction band and are available for 29. (a) Drift speed in B (n-type semiconductor) is higher
conducting current. In case of insulator for moderate  ½+½
change of temperature, there is no change in Reason: I = neAvd is same for both
behaviour. n is much lower in semiconductors. ½
26. Refer to the solution 20. (b) Voltage drop across A will increase as the resistance
At absolute zero temperature conductivities of both of A increases with increase in temperature. ½
n-type and p-type semiconductors are zero. At room Voltage drop across B will decrease as resistance of
temperature, an equally doped n-type semiconductor will B will decrease with increase in temperature.
have more conductivity than a p-type semiconductor. ½ + ½ [CBSE Marking Scheme, 2018-19]

Semiconductor Devices 459


D:\EG_Physics-12_(26-06-2022)\Open_Files\Ch-14\Ch-14
\ 27-Jul-2022 Ved_Goswami Proof-4 Reader’s Sign _______________________ Date __________

Topic 2. pn-Junction diode


•• pn-Junction: Formation of depletion region and potential this current is called reverse current/leakage current.
barriers. When the reverse voltage is high, bonds suddenly start
(a) When a p-n junction is formed, due to the difference breaking so the reverse current increases sharply. That
in their densities, electrons diffuse from n-type to reverse bias at which the current increases sharply is
p-type and holes diffuse from p-type to n-type of called the breakdown voltage.
semiconductor. Thus the diffusion current is set up •• Circuit diagram to study characteristic curves:
from p-type to n-type. Electrons diffuse from n-type (i) For Forward Bias   
to p-type leaving behind positive immobile ion
and similarly negative immobile ion are left in the
p-side of the junction. This sets up an electric field
near the junction from n → p side. So a drift current
flows from n-type to p-type of semiconductor.
Slowly the diffusion current will decrease but drift
current will increase. In equilibrium, the diffusion
current will become equal to drift current and there
is no net flow of charge across the junction. The
difference of potential created across the junction
(ii) For Reverse Bias
when Idiffusion = Idrift is called potential barrier.
The region around the junction, which is devoid of
free electrons or holes is called depletion region.
Potential barrier: The maximum potential difference
created across an unbiased pn-junction is called potential
barrier.
VB
– – + +
– – + +
– – + + •• Characteristic Curve of a pn Junction Diode:
p – – + + n
– – + +
– – + +
– – + + I
– – + + (in mA)
A
I diffusion
I drift C B
Depletion region: Small region in vicinity of junction
which is depleted of free charge carriers and has only
immobile ions. Knee voltage
Barrier field: Electric field set up across the junction Breakdown PD (in volt)
due to potential barrier. voltage
Reverse biasing of Pn Junction: Reverse bias voltage
(V) supports the potential barrier (VB). As a result of
this, the barrier height increases, and width of depletion
region increases. Effective barrier potential in reverse
bias is |V + VB|. I
(in µA)
Majority charge carriers move away from junction,
increasing width of depletion region. There is no
conduction across the junction due to majority carriers.
•• Important Point:
However, a few minority carriers cross the junction after
being accelerated by high reverse bias voltage. They – In forward bias, initially the current increases very
constitute a current that flows in opposite direction and slowly, almost negligibly till the voltage across the

460 Physics–12
D:\EG_Physics-12_(26-06-2022)\Open_Files\Ch-14\Ch-14
\ 27-Jul-2022 Ved_Goswami Proof-4 Reader’s Sign _______________________ Date __________

diode crosses the threshold voltage/cut off voltage. PP′ → Primary coil of transformer
This voltage for Ge diode is around 0.2 V and for Si SS′ → Secondary coil of transformer
diode, it is 0.7V. (b) Frequency of input = freq. of output
After the cut off voltage, the diode current increases pn-junction diode as full wave rectifier:
rapidly. Here, the majority carriers feel negligible
resistance at the junction.
– From forward characteristic curve, we can find the
dynamic resistance and static resistance of junction
diode.
∆V BC
Dynamic resistance = Ra.c. = =
∆I AB
V
Static resistance = Rd.c. =
I P S
– Junction diode is a non-ohmic device. The
resistance of the diode is very low in forward bias
and above knee voltage. In reverse bias, resistance
is very high.
– For an ideal diode, resistance of forward biased
junction diode is 0 and resistance of reverse bias
junction diode is very high (∞). So, a forward biased
junction diode will conduct electricity but a reverse
biased junction diode will not conduct electricity.
•• Use of Junction Diode as a Rectifier
Rectifier is a device which is used to convert alternating
current into a pulsating direct current. (c) Frequency of output = 2 frequency of input
Principle: An ideal diode offers zero resistance to current Filter: Output of a rectifier is pulsating. In order to make
when forward biased and offers infinite resistance when d.c. with constant value, a capacitor of suitable value is
reverse biased. connected in parallel with load resistance. The capacitor
Half Wave Rectifier: Circuit diagram: allows varying component to pass through it but not the
(a) 1
P S d.c. component as X C = is ∞ for d.c. Thus the P.D.
ωC
across load resistance is constant:
dc component
X
R Output
A.C.
input
~ IL
Rectifier

ac +
– C RL dc

P′ S′
(b) Y
(a)
Output with

input filter
capacitor

t
(b)

Semiconductor Devices 461


D:\EG_Physics-12_(26-06-2022)\Open_Files\Ch-14\Ch-14
\ 27-Jul-2022 Ved_Goswami Proof-4 Reader’s Sign _______________________ Date __________

EXERCISE 14.2
I. Objective Type Questions (1 Mark) (ii) The width of the depletion layer and the potential
1. Choose the correct answers from the given options barrier across the junction .................... due to
(MCQs). reverse biasing.
(i) In given figure, V0 is the potential barrier across a (iii) when a p-n junction is forward biased, then
p-n junction, when no battery is connected across the motion of charge carriers across the barrier
the junction is due to ................... and when it is reversed
biased, then the motion of charge carriers is due
1 to ......................... .
V0
2 3. State True or False
3
(i) The thickness of the depletion layer is of the order
of 10–3 m.
h
(ii) In insulator, the valence electrons form a band
that is valence band which is filled completely.
(a) 1 and 3 both corresponds to forward bias of (iii) In a digital circuit, the diodes and transistors are
junction. operated by a 5V supply. The states 1 and 0 will
(b) 3 corresponds to forward bias of junction and correspond to 10 V and 5 V respectively.
1 corresponds to reverse bias of junction. 4. Match the Columns
(c) 1 corresponds to forward bias and 3
Column-I Column-II
corresponds to reverse bias of junction.
(i) Dynamic resistance (a) half wave rectifier
(d) 3 and 1 both corresponds to reverse bias of
DV
junction. (ii) Static resistance (b)
(ii) In given figure below, assuming the diodes to be DI
ideal, (iii) Frequency of input (c) Full wave rectifier
–10V A R = Frequency of
output
D1 V
(iv) Frequency of output (d)
I
D2 = 2 × frequency of input
B For Q. 5 there are two statements labelled as assertion
(A) and reason (R). Select the correct answer to these
questions from the codes (a), (b), (c) and (d) as given
(a) D1 is forward biased and D2 in reverse biased below.
and hence current flows from A to B. (a) Both A and R are true and R is the correct
(b) D2 is forward biased and D1 is reverse biased explanation of A.
and hence no current flows from B to A and vice (b) Both A and R are true but R is not the correct
versa. explanation of A.
(c) D1 and D2 both are forward biased and hence (c) A is true but R is false.
current flows from A to B.
(d) Both A and R are false.
(d) D1 and D2 are both in reverse bias hence no
5. Assertion (A): V-I characteristic of p-n diode is same
current flows from A to B and vice-versa.
as that of any other conductor.
(iii) In the depletion region of a diode
Reason (R): p-n. diodes behave as conductor at room
(a) there are many mobile charges.
temperature.
(b) unequal number of holes and electrons exist.
(c) the recombination of holes and electrons has II. Very Short Answer Type Questions (1 Mark)
taken place. 1. What is the direction of diffusion current in a junction
(d) mobile charged ions exist. diode?
2. Fill in the blanks. 2. Can we take one slab of p-type semiconductor and
(i) ........................ is called as junction in p-n junction physically join it to another n-type semiconductor to
diode. get p-n junction? [NCERT]

462 Physics–12
D:\EG_Physics-12_(26-06-2022)\Open_Files\Ch-14\Ch-14
\ 27-Jul-2022 Ved_Goswami Proof-4 Reader’s Sign _______________________ Date __________

3. What is depletion region in a p-n junction? III. Short Answer Type Questions-I (2 Marks)
4. Which process causes depletion region in a p-n 17. What is meant by depletion region in a junction diode?
junction? How is this region formed? [Delhi 2011, 2014]
5. Name the type of biasing of a p-n junction diode so 18. Distinguish clearly between forward biasing and
that the junction offers very high resistance. reverse biasing of a p-n-junction. [Delhi 2011]
6. What happens when a forward bias is applied to a p-n 19. In V-I characteristics of a p-n junction diode:
junction? [Delhi 2015] (a) Why is the current under reverse bias almost
7. How does the width of the depletion region of a independent of the applied potential upto a critical
p-n junction vary, if the reverse bias applied to it voltage? [Delhi 2013]
decreases? [AI 2008]
(b) Why does the reverse current show a sudden
8. In the figure given below, is the diode D forward or increase at critical voltage?
reverse biased?
20. In the following diagrams, indicate which of the
–10 V diodes are forward biased and which are reverse
R D biased.
9. What happens when reverse bias is applied to p-n
junction?
10. Name two important processes that occur during the
formation of a p-n junction. [AI 2016]
11. Draw diagram to show forward biasing of a p-n – 10V – 12V
junction.
12. Draw diagram to show reverse biasing of a p-n- (c) (d) – 5V
junction.
13. Can the potential barrier across the p-n junction be 21. In th e circuit diagram given below, find the current
measured by simply connecting a voltmeter across through 15W and 5W resistor. [HOTS]
the junction? [HOTS] D1 5Ω
14. The graph of potential barrier versus width of
depletion region for an unbiased diode is shown in
D2 15Ω
A. In comparison to A, graphs B and C are obtained
after biasing the diode in different ways. Identify the
type of biasing in B and C and justify your answer.
[S.P. 2010] [HOTS] 3V
A B C
Assume diodes to be ideal.
22. For the circuit shown in figure, find the current
V(x)

V(x)

V(x)

n
flowing through the 1W resistor. Assume that the two
x x x diodes are ideal diodes. [Delhi 2013 C] [HOTS]
15. In half wave rectifier, what is the output frequency D1 2Ω
if the input frequency is 50 Hz? What is the output
frequency of a full wave rectifier for the same input
frequency? [NCERT] D2 2Ω
16. If V0 is the potential barrier across a p-n junction
which one of the curves represents (i) forward bias
(ii) reverse bias. [NCERT Exemplar] 6V 1Ω

23. What is an ideal diode? Draw the output waveform


across R for the input waveform given below.

Semiconductor Devices 463


D:\EG_Physics-12_(26-06-2022)\Open_Files\Ch-14\Ch-14
\ 27-Jul-2022 Ved_Goswami Proof-4 Reader’s Sign _______________________ Date __________

24. Explain how the width of depletion layer in a 30. A student wants to use two p-n junction diodes to
P-n junction diode changes when the junction is convert alternating current into direct current. Draw
(i) forward biased (ii) reverse biased.[Delhi 2013 C] the labelled circuit diagram she would use and explain
25. Describe briefly with the help of a diagram, the role of how it works. [Delhi 2018]
the two important processes involved in the formation IV. Short Answer Type Questions-II (3 Marks)
of a p-n junction. [AI 2012] 31. (a) Explain with the help of a diagram, how depletion
26. The following table provides the set of values V region and potential barrier are formed in a
and I obtained for a given diode. Assuming the junction diode.
characteristics to be nearly linear, over this range,
(b) If a small voltage is applied to a p-n junction diode
calculate the forward and reverse bias resistance of
how will the barrier potential be affected when it
the given diode. [Delhi 2007 C]
is (i) forward biased, and (ii) reverse biased?
V I [AI 2015, 2010]
Forward 2.0 V 60 mA 32. Draw the circuit arrangement for studying the V - I
biasing 2.4 V 80 mA characteristics of a p-n junction diode (i) in forward
Reverse 0V 0 µA bias and (ii) in reverse bias. Draw the typical V - I
biasing –2V – 0.25 µA characteristics of a silicon diode. [AI 2015]
33. Draw a circuit diagram of full wave rectifier. Explain
27. Suppose a pure Si crystal has 5 × 1028 atoms m–3. It its working. Draw the input/output, waveforms
is doped by 1 ppm concentration of pentavalent As. indicating clearly the function of the two diodes.
Calculate the number of electrons and holes. Given
that ni = 1.5 × 1016 m–3. [NCERT] [AI 2008, 2011, Delhi 2014C, 2015C,
2015, 2019, AI 2016]
28. The figure shows diode connected to an external
resistance and an e.m.f. source. Assuming that the 34. Explain, with the help of a circuit diagram, the
barrier potential developed in diode is 0.5V, obtain working of a p-n junction diode as a half wave
the value of current in the circuit. [HOTS] rectifier. [AI 2014]
150 Ω 35. An a.c. signal is fed into two circuits X and Y end
the corresponding output in the two cases have the
waveforms shown in the figure below. Name the
circuits X and Y. Also draw their detailed circuit
mA

diagrams.
4V
29. A 220V, A.C. supply is connected between points X
A and B as shown in figure below. What will be the
potential difference V across the capacitor?
[NCERT Exemplar] [HOTS] Y

36. Draw a labelled circuit diagram of a full wave rectifier.


Explain its underlying principle and working. Depict
the input and output waveforms. Describe briefly the
role of a capacitor in filtering. [AI 2015]

Answers 14.2
I. Objective Type Questions II. Very Short Answer Type Questions
1. (i) (b) (ii) (b) (iii) (c) 1. From p-type to n-type of semiconductor.
2. (i) The area of contact of p-type and n-type crystal 2. No. Any surface, however smooth, will have roughness
(ii) Increase (iii) diffusion, drift which will be larger than inter atomic spacing and hence
3. (i) False (ii) True (iii) False there will not be continuous contact at atomic level. So
4. (i)-(b) (ii)-(d) (iii)-(a) (iv)-(c)] a discontinuity will be there for moving charge carriers.
5. (d)

464 Physics–12
D:\EG_Physics-12_(26-06-2022)\Open_Files\Ch-14\Ch-14
\ 27-Jul-2022 Ved_Goswami Proof-4 Reader’s Sign _______________________ Date __________

3. It is a thin region around the junction which has been 16. (i) Forward Bias : Curve 3, as in forward bias potential
depleted of its free current carriers. barrier decreases.
4. Diffusion of electrons and holes across the junction (ii) Reverse Bias : Curve 1, as in reverse bias, potential
causes depletion region. barrier increases.
5. Reverse Biasing. 17. Depletion Region : The region around the junction
6. (i) Potential barrier decreases (ii) depletion region which is devoid of free electrons and holes and has
reduces in width (iii) Junction offers very less immobile ions is called depletion region.
resistance i.e., the junction conducts electricity. When a junction of p-type and n-type of semiconductor
7. The width of depletion region increases if the reverse is formed, due to difference in density of electrons and
bias voltage is decreased. holes, the electrons from n side diffuse towards p-type
8. Reverse biased. and the holes diffuse from p to n side.
9. (i) Potential barrier increases Near the junction, there is recombination of electrons
(ii) Depletion region increases. and holes, thus depleting the region of its free electrons
(iii) Resistance of the junction becomes very high. and holes. There are positive and negative immobile
ions in this region.
10. Diffusion and drift of electrons and holes across the
junction. 18. Forward biasing Reverse biasing
VB 1. p-type of semiconductor p-type of semiconductor
11.        V
p A A′ n is connected to positive is connected to connected
terminal of battery and to negative terminal of
n to negative terminal. battery and n to positive
terminal.
B B′

AA′ → depletion region before biasing.


BB′ → depletion region after biasing.
VB
12. V
p A A′ n 2. Potential barrier Potential barrier increases.
decreases
3. Depletion region Depletion region increases
decreases.
B B′
4. Resistance of the junc- Resistance of the junction
tion is very-very low is very high (infinite).
AA′ → depletion region before biasing.
19. (a) When a p-n junction is reverse biased, the minority
BB′ → depletion region after reverse biasing. charge carriers i.e., electrons from p side and holes
13. No. In the depletion region, there are no free charge from n side drift towards the junction. The number
carriers. The voltmeter which can be used to measure density of minority charge carrier remains almost
P.D. across the p-n junction and have a very high constant upto a critical voltage. So the current under
resistance as compared to the junction resistance, which the reverse bias is independent of applied voltage
is nearly infinite, if not biased. upto a critical voltage.
14. B is reverse biased, because on reverse biasing potential (b) When the reverse biased voltage is increased beyond
barrier increases. C is forward biased, because on critical voltage, the electric field at the junction
forward biasing potential barrier decreases. becomes strong enough to pull out electrons from the
15. In half wave rectifier host atoms on p-side and accelerates them towards
n-side. Due to the movement of these electrons
Frequency of output = frequency of input = 50 Hz. across the junction, the current increases suddenly
In full wave rectifier at critical voltage.
Frequency of output = 2 × frequency of input = 2 × 50 20. In forward biased p-side should be at high potential
= 100 Hz. and n-side should be at low potential & vice-versa in
reverse biased.
Semiconductor Devices 465
D:\EG_Physics-12_(26-06-2022)\Open_Files\Ch-14\Ch-14
\ 27-Jul-2022 Ved_Goswami Proof-4 Reader’s Sign _______________________ Date __________

(a) Forward biased (b) reverse biased (c) reverse biased Reverse Biased
(d) forward biased. A A′
21. Diode D1 is forward biased, D2 is reverse biased. p n
As D2 is reverse biased so no current will flow through B B′
15W resistor.
AA′ → Original depletion layer
BB′ → Depletion layer after reverse biasing
25. Two main processes that occur during the formation of
a p-n junction are (i) diffusion (ii) drift.
Current through 5W resistor (i) Diffusion: The number density of electrons in n-type
of semiconductor is much more than the number
V 3 density of electrons in p-type whereas the number
I= = = 0.6A
R 5 density of holes in p-type is much more than number
22. D2 is reverse biased so no current will flow through D2 density of holes in n type. So due to concentration
Thus Rnet = 2 + 1 = 3W difference electrons diffuse from n-side to p-side
and holes diffuse from n-side to p-side.
V 6
I= = = 2A Electron diffusion
R 3 p n
23. Ideal diode offers zero resistance when forward biased – +
and infinite resistance when reverse biased. – +
– +
Output waveform across R:
Hole diffusion
+5
(ii) Drift: Due to diffusion across the junction, near
the junction p-side acquires negative potential and
n-side acquires positive potential. So an electric field
is setup across the junction from n-side to p-side.
O Due to this electric field, electrons drift from p-side
24. (i) In forward bias, positive terminal of the battery is to n-side and holes from n-side to p-side. Thus a drift
connected to p side and negative terminal to n-side current starts. Drift current is opposite to diffusion
of the p-n junction. So the holes from p side which current.
are majority carrier are pushed towards the junction p n
and electron from n-side are pushed towards the – +
junction. This results in reduction of the width of – +
depletion layer. – +
(ii) in reverse bias, n-side is connected with positive Electron drift
Hole drift
terminal of the battery and p-side to negative
terminal of the battery. The minority carriers i.e., DV
26. In forward biasing r =
holes from n-side are pushed towards the junction DI
whereas electrons which are minority carriers in DV = 2.4 – 2.0 = 0.4 V
p-side are pushed towards the junction. The majority DI = 80 – 60 = 20 mA = 20 × 10–3 A
carrier from p as well as n-side move away from the Forward resistance of diode,
junction. This increases the width of depletion layer. 0.4
rfb = = 20Ω
Forward Biased: 20 × 10 −3
A A′ In reverse biasing:
p n DV = – 2 – 0 = –2V
DI = – 0.25 – 0 = – 0.25 µA
B B′
= – 0.25 × 10–6 A
Reverse resistance of diode,
AA′ → Original depletion layer −2
rrb = = 8 × 106 Ω
BB′ → Depletion layer after forward biasing − .25 × 10 −6

466 Physics–12
D:\EG_Physics-12_(26-06-2022)\Open_Files\Ch-14\Ch-14
\ 27-Jul-2022 Ved_Goswami Proof-4 Reader’s Sign _______________________ Date __________

27. ni = 1.5 × 1016 m–3 31. (a) See theory


5 × 1028
(b) (i) When forward biased, potential barrier will
ne = = 5 × 1022 decrease.
106
(as concentration of dopant is 1 ppm) (ii) When reverse biased, potential barrier will
increase.
Using nenh = ni2,
32. Refer theory topic No. 2 under the heading ‘Circuit
ni2 (1.5 × 1016 ) 2 diagram to study characteristic curves’ and characteristics
nh = = = 4.5 × 109 m −3
ne 5 × 1022 curve of a pn-junction diode.
28. Emf of cell = 4V 33. Primary coil of the transformer is connected to AC input
source.
Potential barrier = 0.5V P –S+
Forward bias opposes potential barrier
So net voltage = 4 – 0.5 = 3.5 V V0
V 3.5 7 T A
I == = A ~ B
R 150 300 RL

29. In positive half cycle of A.C. diode will conduct


electricity. So, the capacitor will get charged to its peak
value, and will not get discharged as their is no load P′ +S′
resistance. The two sides S and S′ of the secondary coil are
PD. across capacitor = peak voltage of given a.c. connected to the p-side of the two diodes.
                 = 220 2 V The secondary coil is centrally tapped at T and it is
30. Circuit diagram to use two p-n junction diodes to convert connected to n-sides of the diodes through a load
ac into dc (circuit diagram of full wave rectifier). resistance.
A.C. Input at S
Waveform at S

t
Waveform at S

A.C. Input at S

Working of full wave rectifier – circuit diagram


Output voltage

shown in the figure gives rectified output voltage


Due to Due to Due to Due to
Output waveform

D1 D2 D1 D2
corresponding to both the positive as well as negative
across RL

t
half cycles of ac. So it is known as full wave rectifier.
p type semiconductors of both the junction diodes D1
and D2 are connected to the ends of secondary of the At the instant when the voltage at S with respect to
centre tape transformer and n type semiconductors of centre tap T is +ve, then S′ will be negative. Diode D1
both the junction diodes are connected together and will be forward biased but D2 will be reverse biased.
rectified output is taken between the common point of Diode D1 will conduct electricity and the current
diodes and the midpoint of secondary of transformer. through load resistance will be from A to B.
So for a full wave rectifier the secondary of the When the voltage is –ve with respect to center tap, then
transformer is provided with a centre tapping, and due S′ will be +ve. In this case, D1 will be reverse biased
to this transformer is called centre tap transformer. At and D2 will be forward biased. D1 will not conduct
a time either D1 or D2 is forward biased, and rectifies electricity but D2 will. Current in load resistance will
only half the cycle, and both do so alternate half cycle. again be from A to B.
Thus the output between their common terminals and Both –ve and +ve halves of the AC inputs will be
the centre tap of the transformer becomes a full wave produced across the load resistance and the current
rectified output, i.e. we get full wave rectified output through the load resistance is always from A to B.
voltage during both, the positive as well as negative 34. Half wave rectifier consists of a stepdown transformer,
half of the cycle. a junction diode and a load resistance.

Semiconductor Devices 467


D:\EG_Physics-12_(26-06-2022)\Open_Files\Ch-14\Ch-14
\ 27-Jul-2022 Ved_Goswami Proof-4 Reader’s Sign _______________________ Date __________

The primary coil of the transformer is connected to AC RL → Load resistance


mains and the secondary coil is connected in series with P –S+ D1
junction diode and load resistance.
V0
T A
~ B
RL

D2

P +S
36. (i) Principle : An ideal diode when forward biased will
Working: We assume that the diode is ideal. It offers conduct electricity but will not conduct when reverse
biased.
zero resistance when forward biased and infinite
resistance when reverse biased. (ii) For circuit diagram, working, input and output
waveforms, refer to Q No. 30 of short answer
During +ve half cycle of voltage at S, the diode is
question for 2 Marks.
forward biased and the current I flows through the
Role of a capacitor in filtering:
load resistance and there is potential drop across load
resistance = IRL.
As voltage at S increases or decreases, the current also
changes accordingly, so does the output voltage.
During –ve half cycle at the end S, diode is reverse bias
and no current flows, so there is no voltage drop across
the load resistance.

The output of a rectifier is in the form of pulses,


though it is unidirectional but does not have constant
value. To get steady d.c. a capacitor of suitable value
is connected across the output terminals in parallel
with load resistance. When the voltage across the
capacitor is rising, the capacitor gets charged. As it
is connected with load resistance, the capacitor gets
35. Device X is a half wave rectifier discharged through the load and the voltage across
Circuit diagram: it begins to fall. In the next half cycle of rectified
PP′ → Primary coil of transformer output, it again gets charged to the peak value. The
SS′ → Secondary coil of transformer rate of fall of voltage across capacitor depends upon
D → Diode, RL → Load resistance the product of C and RL used in the circuit. The
Circuit diagram: time constant is kept large so that the output voltage
obtained is nearer to the peak voltage of the rectified
voltage.
ac input

Device Y is a full wave rectifier


Output with

PP′ → Primary coil of transformer


input filter
capacitor

SS′ → Secondary coil of transformer t

D1, D2 → Diodes

468 Physics–12
D:\EG_Physics-12_(26-06-2022)\Open_Files\Ch-14\Ch-14
\ 27-Jul-2022 Ved_Goswami Proof-4 Reader’s Sign _______________________ Date __________

Case Based Questions


I. Diode as a Half-Wave rectifier: Diode conducts
corresponding to positive half cycle and does not
conduct during negative half cycle. Hence, ac is
converted by diode into unidirectional pulsating dc.
This action is known as half-wave rectification.

The rectified voltage is in the form of pulses of the


shape of half sinusoidals. Although it is unidirectional
but not steady. To get steady dc output from the
pulsating voltage, a capacitor is connected across the
output terminals, parallel to load resistance which will
filter out the ac ripple and provide a pure dc steady
output.

Diode as a Full-Wave rectifier: In the full-wave


rectifier, two p-n junction (Here, D1 and D2) diodes,
are used. Its principle is that junction diode puts
very low resistance in forward bias while very high
resistance in reverse bias condition. 1. The a.c. to be rectified is applied to the
(a) primary of a transformer
(b) secondary of a transformer
(c) between primary and secondary
(d) anywhere on the diode
2. The secondary coil of the transformer is tapped
in the
(a) corner (b) middle
(c) upper half (d) lower half
3. During positive half of a.c. cycle diode D1, is in
forward biasing and it conducts. At this point the
diode D2 will not conduct because
(a) being in reverse biasing
(b) being in forward biasing.
(c) as in high biasing
(d) as in low biasing
4. The output of full wave rectifier is
(a) pulsating a.c. (b) pulsating d.c.
(c) constant a.c. (d) constant d.c.
5. Pulsations can be minimised by using suitable
(a) filter circuit (b) CL circuit
(c) RC circuit (d) CR circuit
Semiconductor Devices 469
D:\EG_Physics-12_(26-06-2022)\Open_Files\Ch-14\Ch-14
\ 27-Jul-2022 Ved_Goswami Proof-4 Reader’s Sign _______________________ Date __________

Ans. 1. (a) 2. (b) 3. (a) 4. (b) 5. (a) Ans. 1. Potential difference across R = 3 – 0.4 = 2.6V
II. The potential barrier in p-n junction diode is the Potential difference
Resistance R =
barrier in which the charge requires additional force Current
for crossing the region. In the other words, the barrier 2.6
= = 130 Ω
in which charge carrier stopped by the obstructive 20 × 10−3
          
forces is known as the potential barrier.
VB 0.4
When a p-type semiconductor is brought into a close 2. E = = =1.0 × 106 Vm–1
d 4.0 × 10−7
contact with n-type semi-conductor, we get a p-n
junction with a barrier potential 0.4V and width of 3. Concentration of positive and negative charges
depletion region is 4.0 × 10–7m. This p-n junction near the junction is the causes of the potential
is forward biased with a battery of voltage 3V and barrier in a p-n junction.
negligible internal resistance in series with a resistor III. p-n junction is a single crystal of Ge or Si dopped in
of resistance R, ideal milliammeter and key K as such a manner that one half portion of it acts as p-type
shown in figure. semiconductor and other half functions as n-type
When key is pressed a current of 20 mA passes semiconductor. As soon as a p-n junction is formed,
through the diode. the holes from p-region diffuse into n-region and
electrons from n-region diffuse into p-region. This
results in the development of VB across the junction
+
which opposes the further diffusion of electrons and
mA
– holes through the junction.
1. Why do electrons in an unbiased p-n junction
+ – diffuse from n-region to p-region?
3V K ‘R’ 2. What causes the potential of depletion layer?
Ans. 1. Electron concentration in n-region is more as
1. What is the resistance of R ?
compared to that in p-region. So electrons diffuse
2. What is the intensity of electric field in the from n-side to p-side.
depletion region when p-n junction is unbiased?
2. The potential of depletion layer is due to ions.
3. What is the cause of the potential barrier in a p-n.
junction diode?

IMPORTANT FORMULAE
Formulae Symbols Application
1. ne = nh ne = free electron density Intrinsic semiconductors
nh = hole density
2. ne < nh p - type semiconductors
3. ne > nh n - type semiconductors
4. ne.nh = ni2 ni = density of Intrinsic carriers Relation between charge carriers
5. I = Ie + Ih Ie = current through electrons Current through a semiconductor
I = neeAVde + nheAVdh Ih = current through holes
Vde = Drift velocity of electrons
Vdh = Drift velocity of holes
6. s = e(neme + nhmh) m = mobility To determine mobility and conductivity
s = conductivity of a semiconductor

COMMON ERROR
S.No. Error Correction
1. Energy level diagram for Extrinsic and intrinsic Labelling of acceptor and donor level in the energy band
semiconductor diagrams.

470 Physics–12
D:\EG_Physics-12_(26-06-2022)\Open_Files\Ch-14\Ch-14
\ 27-Jul-2022 Ved_Goswami Proof-4 Reader’s Sign _______________________ Date __________

REVISION CHART

Valence Band Conduction Band


The highest energy bands filled with valence electrons The lowest unfilled allowed energy band next to the valence
bound in the bond are valence bands. band is conduction band.

Semiconductor
They are the substances whose conductivity is more than that of insulators, but less than that of conductors.

Intrinsic Semiconductors Extrinsic Semiconductors


Pure semiconductors are called intrinsic semiconductors. The A semiconductor doped with some suitable impurity atoms so
number density of free electrons is equal to number density as to increase its number of charge carriers are called extrinsic
of holes. semiconductors.
ne = nh = ni
where ni = no. density of intrinsic semiconductor.
For any pure semiconductor, ne nh = ni2,
n-type semiconductors
Electrical conductivity is low and it depends on temperature. They are obtained by doping the semiconductor with
I = Ie + Ih, pentavalent impurities such as P, As, Sb (elements of group
Ie Æ electron current Ih Æ hole current 15). The number density of electrons ne >>> number density
of holes (nn)
But nenh = ni2

p-type semiconductors.
They are obtained by doping the semiconductor with trivalent
impurities. nh >>> ne nenh = ni2

Semiconductor Devices 471


D:\EG_Physics-12_(26-06-2022)\Open_Files\Ch-14\Ch-14
\ 27-Jul-2022 Ved_Goswami Proof-4 Reader’s Sign _______________________ Date __________

REVISION CHART

Junction Diodes

Forward Biasing of pn-Junction


p-type of semiconductor is connected with positive terminal and n-type is
connected to negative terminal of battery. Forward bias voltage V opposes
the potential barrier Vb. As a result barrier height decreases and the width
of is | V – VB |. For an ideal diode resistances in forward bias is zero.

Factors related to Semi-conductor

Depletion region Barrier field Potential barrier (VB)


Small region in vicinity of junction which Electric field set up across the The maximum potential difference
is depleted of free charge carriers and has junction due to potential barrier. created across an unbiased n-junction
only immobile ions. is called potential barrier.

Rectifier

Half wave rectifier Full wave rectifier


It is a rectifier, which rectifies only one halves of each It is a rectifier, which rectifies both halves of each a.c.
a.c. input. input. It’s output is continuous without any pulsating d.c.
It's output is discontinuous and pulsating d.c. As nothing goes waste, so it’s efficiency is very high.
As half cycle of A.C. input goes waste, its efficiency is
very low.

472 Physics–12
D:\EG_Physics-12_(26-06-2022)\Open_Files\Ch-14\Ch-14
\ 27-Jul-2022 Ved_Goswami Proof-4 Reader’s Sign _______________________ Date __________

IMPORTANCE OF EACH TOPIC AND FREQUENTLY ASKED TYPES OF QUESTIONS

☞ Important Topics
1. Questions based on diodes and rectifiers.

1. Mobilities of electrons and holes in a sample of intrinsic germanium at room temperature are 0.54 m2 V–1 s–1 and
0.18 m2V–1s–1 respectively. If the electron and hole densities are equal to 3.6 × 1019 m–3, calculate the germanium
conductivity.  [Ans. 4.147 Sm–1]
2. The V–I characteristic of a silicon diode is given in figure. Calculate the diode resistance in:
(a) forward bias at V = ± 2 V and V = + 1V, and (b) reverse bias V = – 1V and –2V.
Ge-diode
Si-diode
80 ∆I
70

Milliampere
60 ∆V

(mA)
50
40
30
V (Reverse bias) 20
–3 –2 –1 10
1 2 3
– 0.5 Volts
– 1.0 V (Forward bias)
Cut-in voltage
– 1.5
Microampere
(µA)
Breakdown

[Ans. (a) rrb (at + 2V) = 20W,rrb = (at + 1V) = 40W, (b) rrb (– 2V) = 8 × 106W, rrb = (– 1V) = 8 × 106W]
3. Explained the working of ‘Full wave rectifier’ with circuit diagram.
4. Differentiate between the following:
(i) Conduction Band and Valence Band
(ii) Intrinsic and Extrinsic Semiconductor

Semiconductor Devices 473


D:\EG_Physics-12_(26-06-2022)\Open_Files\Ch-14\Ch-14
\ 27-Jul-2022 Ved_Goswami Proof-4 Reader’s Sign _______________________ Date __________

ASSIGNMENT
I. Objective Type Questions (1 Mark)
1. Multiple choice questions:
(i) In depletion region of a diode
(a) there are no mobile charges
(b) equal number of holes and electrons exist making the region neutral
(c) the recombination of holes and electrons has taken place
(d) immobile charged ions exist
(ii) On recombination the energy is released in the form of
(a) 24 dB (b) 41 dB (c) 250 dB (d) 1250 dB
2. Fill in the blanks:
(i) In insulator, the valence electrons form a band that is ......................... .
(ii) ........................ is called as junction in p-n junction diode.
II. Very Short Answer Type Questions (1 Mark)
3. Why is the conductivity of n-type semi-conductor greater than that of the p-type semiconductor even when both
of these have same level of doping?
4. Define the term ‘Doping’
III. Short Answer Type Questions-I (2 Marks)
5. Explain the working of ‘Forward Biasing’ of pn-Junction.
6. Give two differences between a half wave rectifier and a full wave rectifier. [CBSE 2022]
IV. Short Answer Type Questions-II (3 Marks)
7. Explain, with the help of a circuit diagram, the working of a p-n junction diode as a half wave rectifier.
8. (i) Draw V-I characteristics of a p-n Junction diode.

(ii) Differentiate between the threshold voltage and the breakdown voltage for a diode.

(iii) Write the property of a junction diode which makes it suitable for rectification of ac voltage. [CBSE 2022]



474 Physics–12
D:\EG_Physics-12_(26-06-2022)\Open_Files\Sample Paper_1_(Solved)\Sample Paper_1
\ 27-Jul-2022 Ved_Goswami Proof-4 Reader’s Sign _______________________ Date __________

Sample Paper– 1 (Solved)


Time Allowed: 3 hours Maximum Marks: 70
General Instructions:
(i) All questions are compulsory. There are 33 questions in all.
(ii) This question paper has five sections: Section A, Section B, Section C, Section D and Section E.
(iii) Section A contains four objective and six very short answer questions and four assertion reasoning MCQs of 1
mark each, Section B has two case based questions of 4 marks each, Section C has nine short answer questions
of 2 marks each, Section D contains five short answer questions of 3 marks each and section E contains three
long answer questions of 5 marks each.
(iv) There is no overall choice. However, internal choice is provided. You have to attempt only one of the choices
in such questions.

Section-A
All questions are compulsory. In case of internal choices, attempt any one of them.
I. Objective Type Questions (1 Mark)
Q1. A circular of radius R carries a current I. The magnetic field at its centre is B. At what distance from the centre on
B
the axis of the coil, the magnetic field will be ? 1
8
(a) 2R (b) 3R (c) 2R (d) 3R
Ans. (b)
Q2. These are 3 wavelengths 107 m, 10–10 m, 10–7 m. Their respective names will be 1
(a) Radiowaves, X-rays, Visible rays (b) X-rays, Visible rays, Radiowaves
(c) X-rays, g-rays, Visible rays (d) Visible rays, g-rays, X-rays
Ans. (a)
Q3. Which of the following gives the direction of the induced e.m.f.? 1
(a) Faraday’s law (b) Lenz’s law (c) Ampere (d) Biot-Savart’s law
Ans. (b)
Q4. Consider aiming a beam of free electrons towards free protons. When they scatter, an electron and a proton cannot
combine to produce a H-atom, 1
(a) because of energy conservation.
(b) with simultaneously releasing energy in the form of radiation.
(c) because of momentum conservation.
(d) because of angular momentum conservation.
Ans. (a)
II. Very Short Answer Type Questions (1 Mark)

Q5. What is the direction of the force acting on a charged particle q, moving with a velocity v in a uniform magnetic

field B ? 1
OR

Define magnetic flux. Give its SI unit.
→ → →
Ans. Q F = q ( v × B)

475
D:\EG_Physics-12_(26-06-2022)\Open_Files\Sample Paper_1_(Solved)\Sample Paper_1
\ 27-Jul-2022 Ved_Goswami Proof-4 Reader’s Sign _______________________ Date __________

→ →
Magnetic force is always normal to plane of v and B .
OR

The total number of magnetic lines of force crossing the surface A in a magnetic field B is termed as magnetic
flux.
f = BA cos θ
Its SI unit is Weber. It is a scalar quantity.
Q6. Biot-Savart law indicates that the moving electrons (with velocity v) produce a magnetic field B then write the
direction of Magnetic field (B) with respect to velocity (v). 1
OR
Draw the magnetic field lines due to a current carrying loop.
Ans. B is perpendicular to velocity v.
OR

Q7. How we will reduce the resonant frequency in an LCR series circuit with a generator. 1
OR
The peak value of emf in AC is E0. Write its (i) rms (ii) average value over a complete cycle.
Ans. Another capacitor should be added in parallel to the first.
OR
E0
(i) Erms = = 0.707 E0 (ii) Em = 0
2
Q8. A convex lens is dipped in a liquid whose refractive index is equal to the refractive index of the lens. Then how
its focal length will change. 1
OR
Write the relationship between angle of incidence ‘i’, angle of prism ‘A’ and angle of minimum deviations ∆m for
a triangular prism.
Ans. Focal length become infinite.
OR
The relation between the angle of incidence i, angle of prism A, and the angle of minimum deviation ∆m, for a
triangular prism is given by i = (A + ∆m)/2
Q9. The ground state energy of hydrogen atom is – 13.6 eV. What are the kinetic and potential energies of electron in
this state? 1
Ans. As in ground state of hydrogen atom (Z = 1, n = 1)
1 KZ2 KZ2
K.E. = and P.E. = – = – 2K.E
2 r r
KZ2
TE = KE + PE = – 1
2 r

\ TE = – E, PE = – 2E, KE = E
Hence KE = 13.6 eV, PE = – 27.2 eV

476 Physics–12
D:\EG_Physics-12_(26-06-2022)\Open_Files\Sample Paper_1_(Solved)\Sample Paper_1
\ 27-Jul-2022 Ved_Goswami Proof-4 Reader’s Sign _______________________ Date __________

Q10. Name one impurity each, which when added to pure Si, produces (a) n-type and (b) p-type semiconductor. 1
Ans. (a) As (Arsenic) (b) In (Indium).
For questions numbers 11, 12, 13 and 14, two statements are given-one labelled Assertion (A) and the other labelled
Reason (R). Select the correct answer to these questions from the codes (a), (b), (c) and (d) as given below.
(a) Both A and R are true and R is the correct explanation of A
(b) Both A and R are true but R is NOT the correct explanation of A
(c) A is true but R is false
(d) A is false and R is also false
Q11. Assertion (A): In Young's experiment, the fringe width for dark fringes is different from that for white fringes.1
Reason (R): In Young's double slit experiment the fringes are performed with a source of white light, then only
black and bright fringes are observed.
Ans. (d) A is false and R is also false
Young show experimentally the concept of interference of light. For S1 and S2 being two coherent sources, ʻdʼ is
the distance between two sources and distance between source and screen is D, then interference pattern consists
of dark and white fringes. Fringe width for both bright and black is same.



W= d ,
where l is wavelength. Also for white light source, coloured fringes are formed.
Q12. Assertion (A): Microwave communication is preferred over optical communication.
Reason (R): Microwaves provide large number of channels and band width compared to optical signals.
1
Ans. (a) Both A and R are true and R is the correct explanation of A
Microwaves are electromagnetic waves (EMW) with wavelength as longer than those of infrared (I.R.) light
but relatively short from radio waves. The microwave range includes ultra high frequency (UHF), super high
frequency (SHF) and extremely high frequency (30 – 300 GHz) signals. Optical communication is any form of
telecommunication that uses light as the transmission medium. Hence, its channels and bandwidth (w) are lesser
than microwave communication.
Q13. Assertion (A): A convex mirror cannot form real images. 1
Reason (R): Convex mirror converges the parallel rays that are incident on it.

Ans. (c) A is true but R is false
Q14. Assertion (A): Diamond glitters brilliantly.
Reason (R): Diamond does not absorb sunlight.
1
Ans. (b) Both A and R are true but R is NOT the correct explanation of A
The faces of diamond are so cut that the light ray entered inside diamond fall on the diamond-air interface at an
angle greater than critical angle (i > ic) and so suffer repeated total internal reflections (T.I.R) and comes out of its
faces only in some specific directions, because of which it shines brilliantly. All the light-entering in it comes-out
of diamond after a number of reflections and no light is get absorbed by it.

Sample Paper–1 (Solved) 477


D:\EG_Physics-12_(26-06-2022)\Open_Files\Sample Paper_1_(Solved)\Sample Paper_1
\ 27-Jul-2022 Ved_Goswami Proof-4 Reader’s Sign _______________________ Date __________

Section-B
Questions 15 and 16 are Case Study based questions and are compulsory. Attempt any 4 sub parts from each
question. Each sub part carries 1 mark.
Q15. The phenomenon of interference is a strong evidence in support of wave
nature of light. Interference results due to super position of light waves
from two coherent sources. Coherent sources continuously emit light of
same frequency with constant phase difference. Energy get redistributed
and intensity (I) is maximum at some points and minimum at other
points. 4
In fact, intensity at any point that receives light waves from both
sources depends on the phase or path difference. On such points at
which path difference Dx = nl, where n is an integer, On intensity
(2n + 1) λ
is maximum while, at points where Dx = 2 , intensity is
minimum. Interference was discovered by Thomas Young. Consider that a source of monochromatic light
of wavelength 5000 Å is placed at S. Two slits, S1 and S2 are equidistant from S and have equal widths.
A wavefront originates from S, and S1 and S2, being points on this wavefront, emit light waves, which
superimpose to result in an interference pattern on the screen kept at the position (1). O is a point on the screen
equidistant from slits S1 and S2. P is a point on the screen 1.1 mm from O at which path difference between
waves from slits S1 and S2 is 12500 Å. The path difference is zero at O and a central bright fringe of intensity
ʻI0ʼ is obtained at O. Q is another point 2.0 mm from O. If the screen is now shifted to a new position (1′)
[not mention in the Fig.] so that D changes, the fringe width is found to be 50% more than its previous value and
the angular fringe width become 0.1°.
[Assume D to be large and sin q ≈ tan q ≈ q as ʻqʼ is very small].
1. When the screen is at position (1), phase difference of waves from coherent sources S1 and S2 at point P is effectively
(a) 90° (b) 135° (c) 180° (d) 45°
2. Intensity at point P when the screen is at position (1), is
I
(a) 20 I0 (b) 7 I0 (c) zero (d) 80
3. Value of fringe width when screen is at position (1) will be
(a) 1.00 mm (b) 0.98 mm (c) 0.44 mm (d) 0.72 mm
4. The intensity at point Q when the screen is at position (1), will be
(a) 2.5 I0 (b) 0.65 I0 (c) 3.35 I0 (d) 0.02 I0
5. Value of distance ʻdʼ between S1 and S2 is
(a) 0.29 mm (b) 0.69 mm (c) 0.79 mm (d) 0.86 mm
Ans. Given that l = 5000 Å
Distance of P from O = 1.1 mm
Path difference (Dx) at point P when screen is in position (1) = 12500 Å
Distance of point Q from O = 2.0 mm
Angular fringe width when screen is in position (1′) = 0.1°
1. Path difference (Dx) at point P = 12500 Å
= 2.5l (l = 5000 Å)

Phase difference Dx = λ × Path difference (Dx)

fi Df = λ × 2.5l
fi Df = 5p
Phase difference Df enters the expression of resultant intensity (I) as cos f (I = I1 + I2 + 2 I1 I 2 cos f).

478 Physics–12
D:\EG_Physics-12_(26-06-2022)\Open_Files\Sample Paper_1_(Solved)\Sample Paper_1
\ 27-Jul-2022 Ved_Goswami Proof-4 Reader’s Sign _______________________ Date __________

\ cos (5p) = cos (4p + p) = cos p


Hence, phase difference Df is effectively = p = 180°)
Hence, option (c) is correct.
2. Let I is the intensity of light emitted by each slit. Resultant intensity (IR) can be given as,
IR = I1 + I2 + 2 I1 I 2 cos f = I + I + 2I cos f = 2I (1 + cos f)
= 2I(2 cos2 f/2) = 4I cos2 f/2  ...(1)
Intensity of a bright fringe, say, the central maximum formed at centre O is I0. At O, f = 0°.
\ I0 = 4I(1)2 = 4I ...(2)
So, we can say that at any point,
φ
IR = I0 cos2 2  [using (1) and (2)]
At point P, as found in Sol. 1,
f = 5p or effectively = p
π
\ IR = I0 cos2 2 = 0
Hence, the correct option is (c).
3. Point P is at distance y = 1.1 mm from centre O. Therefore, path difference at P,
yd 1.1×10 –3 d
Dx = D = D
but Dx = 12500 Å = 2.5l
–3
1.1×10 d
\ D = 2.5l
–3
λD 1.1×10
or d = 2.5 m = 0.44 × 10–3 m
λD
\ d = 0.44 mm
λD
Since w= d
\ fringe width w = 0.44 mm
Hence (c) is the correct answer.
4. Point Q is y = 2.0 mm from centre O. Hence path difference
2.0 # 10 –3 d
Dx = D
–3
1.1×10 d
but D = 2.5l
2.5λ
\ Dx = 2.0 × 1.1
Dx = 4.545l
2π 2π
f = λ (Dx) = λ × 4.545l = 9.09p

IR = I0 cos2 b 2 l = 0.02I0
9.09π

Hence (d) is the correct answer.
λ
5. Angular fringe width (q), when screen is at position (1′), is 0.1°. Angular width of a fringe q = d and does not depend
on D. (angular fringe width, when screen is at position (1), is also (0.1°).
but value of ʻqʼ is taken in radian, so change 0.1° into radian
λ
d = 0.1° = 0.001744 radian
–10
λ 5000 # 10
⇒ d = 0.001744 = 0.001744 m = 2.87 × 10–4 m
\ d = 0.29 mm
Hence (a) is the correct answer.
Sample Paper–1 (Solved) 479
D:\EG_Physics-12_(26-06-2022)\Open_Files\Sample Paper_1_(Solved)\Sample Paper_1
\ 27-Jul-2022 Ved_Goswami Proof-4 Reader’s Sign _______________________ Date __________

Q16. Equipotential Surface: The surface at which potential at all points is same called equipotential surface. The
work done across any two points on equipotential surface is always zero. The electric field is always normal to the
equipotential surface.
Some equipotential surfaces are given below: 4

Properties:
(i) No work is done in moving a test charge over an equipotential surface.
(ii) Electric field is always normal to the equipotential surface at every point.
(iii) Equipotential surface are closer together in the region of strong field and father apart in the region of weak
field.
1. What is the shape of the equipotential surfaces for a uniform electric field?
" "
(a) Parallel to the direction of E (b) Anti-parallel to the direction of E
"
(c) Perpendicular to the direction of E (d) Can have any direction

2. What is the shape of the equipotential surfaces for an isolated point charge?
(a) square (b) rectangular (c) oval (d) concentric spheres
"

3. A uniform electric field E of 300 NC–1
is directed along PQ. A, B and C are three points in the field having x
and y coordinates (in metres) as shown in the figure. Potential difference between the points B and C will be

(a) 1500 V (b) – 2100 V (c) – 1100 V (d) 2000 V



4. Two uniformly charged parallel plates having densities + s and – s are placed in the XZ plane at a distance
ʻdʼ apart, sketch of the equipotential surface due to electric field between the plates will be

(a) (b)

480 Physics–12
D:\EG_Physics-12_(26-06-2022)\Open_Files\Sample Paper_1_(Solved)\Sample Paper_1
\ 27-Jul-2022 Ved_Goswami Proof-4 Reader’s Sign _______________________ Date __________

(c) (d)

5. The separation between successive equipotential surfaces get wider as the distance from the charges increases
because
1 1
(a) dr µ E (b) dr µ E (c) dr µ 2 (d) dr µ E2
E
Ans. 1. The equipotential surfaces are perpendicular to the direction of electric field.
Hence, (c) is the correct answer.
2. The equipotential surfaces are concentric spheres whose centres are located at the given point charge.
Hence, (d) is the correct answer.
3V
3. Since E = – ∆x
Potential difference between B and C
DV = – E Dx = – 300 × 7 = – 2100 V
Hence, (b) is the correct answer.
d
4. Equipotential surface is at a distance 2 from either plate and parallel to XZ plane.

Hence, (a) is the correct answer.


5. Equipotential surfaces are closer in region of strong field and farther apart in region of weak field.
– dV
E = dr
dV
| E | = dr
For constant P.D.
1 1
E µ dr fi dr µ E
Hence, (b) is the correct answer.

Section-C
All questions are compulsory. In case of internal choices, attempt any one of them.
Q17. I-V graph for a metallic wire at two different temperatures, T1 and T2 is as shown in the figure. Which of the two
temperature is lower and why? 2
T1

T2
I

I 1
Ans. For the given figure the slope of a V-I graph is = . Here the smaller the slope larger is the resistance. As the
V R
resistance of a metal increases with the increase in temperature, so resistance at T2 is more and T1 is lower.

Sample Paper–1 (Solved) 481


D:\EG_Physics-12_(26-06-2022)\Open_Files\Sample Paper_1_(Solved)\Sample Paper_1
\ 27-Jul-2022 Ved_Goswami Proof-4 Reader’s Sign _______________________ Date __________

Q18. Use Kirchhoff’s rules to determine the potential difference between the points A and D when no current flows in
the arm BE. 2
3Ω = R2
F E
D

1V
R1 = 2Ω
R
2Ω

3V
A C
B
6V 4V
OR
State Kirchhoff’s rules and explain on what basis they are justified.
Ans. According to Kirchhoff's junction law at B
i3 = i1 + i2
\ i3 = i1
As i2 = 0 (given)
Applying second law to loop AFEB, of the electronic network shown in the figure.
3Ω
F i3 E i1
i2 D

1V
R1
R
2Ω

3V
A C
i3 B i1
6V 4V

\ i3 × 2 + i3 × 3 + i2 R1 = 1 + 3 + 6
i3 = i1 = 2A
From A to D along AFD
\ VAD = 2i3 – 1 + 3 × i3 = (4 – 1 + 6) V = 9 V
OR
(i) Kirchhoff’s 1st law/junction rule/current rule: It states that the algebraic sum of currents at any junction is
0.
In other words, sum of currents entering a junction is equal to sum of currents leaving the junction.
This law is based on conservation of charge. As the conductor is neutral when current is flowing, so the total amount
of current coming to a junction must be equal to the total current leaving a junction.
(ii) Kirchhoff’s 2nd law/loop rule/voltage rule: It states that around any closed loop, the algebraic sum of change
in potential must be 0. i.e., SDV = 0.
This is based on conservation of energy. As electrostatic force is a conservative force, so the work done by it
along any closed path is zero.
Q19. You are given two converging lenses of focal lengths 1.25 cm and 5 cm to design a compound microscope. If it is
desired to have a magnification of 30, find out the separation between the objective and the eyepiece. 2
OR
A small telescope has an objective lens of focal length 150 cm and eyepiece of focal length 5 cm. What is the
magnifying power of the telescope for viewing distant objects in normal adjustment? If this telescope is used to
view a 100 m tall tower 30 m away, what is the height of the image of the tower formed by the objective lens?

482 Physics–12
D:\EG_Physics-12_(26-06-2022)\Open_Files\Sample Paper_1_(Solved)\Sample Paper_1
\ 27-Jul-2022 Ved_Goswami Proof-4 Reader’s Sign _______________________ Date __________

Ans. Given, f0 = 1.25 cm, fe = 5 cm and m = 30


Let S be tube length (distance i.e. separation between the objective and the eyepiece)
S 25
\ m= ×
f0 fe
(25 cm is a constant and equals to the normal distance of clear vision of the human eye).
S × 25 5S
Hence, 30 = =
1.25 × 5 1.25
\ S = 6 × 1.25 = 7.5 cm
Hence, the tube length = 7.5 cm
OR
Given, f0 = 150 cm, fe = 5 cm, u0 = – 30m = –3000 cm, h0 = 100 m
ve = –D = –25 cm
1 1 1
Using, = – for objective
f0 v0 u0
1 1 1
= –
150 v0 –3000
1 20 – 1 3000
= fi v0 =
v0 3000 19
v0 3000 1 –1
m0 = = × =
u0 19 – 3000 19
Using
1 1 1
= – for eye piece
fe ve ue
1 1 1
= –
5 –25 ue
1 1 1 –1 – 5
fi = – =
ue –25 5 25
–25
ue = cm
6
v –25
me = e = ×6 = 6
ue –25
h hi 1
Total magnification = i = m0 × me ⇒ = − × 6 = – 0.31578
h0 100 19
hi = – 0.31578 × 100 m = – 31.58 m
Thus, the height of the image would be 31.58 m.
Q20. When an electron in hydrogen atom jumps from the excited state to the ground state, how would the de-Broglie
wavelength associated with the electron change? Justify your answer. 2
Ans. In an hydrogen atom, the negatively charged electron revolves in a circular orbit around the heavy positively
charged nucleus. The centripetal force required by the electron is produced by the attractive force exerted by the
nucleus on it. The energy is radiated, when an electron jumps from higher to lower energy orbit and the energy is
absorbed, when jumps from lower to higher energy orbit.
hc
hv = Ef – Ei [Ef > Ei] ⇒ = E f – Ei
λ
It is called Bohr’s frequency condition.
Q21. Calculate the shortest wavelength in the Balmer series of hydrogen atom. In which region (infra-red, visible,
ultraviolet) of hydrogen spectrum does this wavelength lie? 2
Ans. The formula for wavelength (l) by Balmer series is:

Sample Paper–1 (Solved) 483


D:\EG_Physics-12_(26-06-2022)\Open_Files\Sample Paper_1_(Solved)\Sample Paper_1
\ 27-Jul-2022 Ved_Goswami Proof-4 Reader’s Sign _______________________ Date __________

1 JK 1 1 NO
= R KKK 2 – 2 OOO where n = 3, 4, 5... and
λ n P
L2
R = 1.097 × 107 m–1 is the Rydberg constant.
1 JK 1 1 ON
= 1.097 × 107 m–1 × KKK 2 – 2 OOO
λ
L2 3 P
1
= 1.097 × 10 × (0.25 – 0) = 0.2743 × 107
7
λ
l = 3.6456 × 10–7 = 364.56 × 10–9 m = 364.56 nm
This wavelength lies in the visible region of the hydrogen spectrum.
Q22. Calculate the quality factor of a series LCR circuit with L = 2.0 H, C = 2 µF, and R = 10W. Mention the significance
of quality factor in LCR circuit. 2
1 L 1 2
Ans. We have,     = Q = = 100
R C 10 2 × 10 −6
It signifies the sharpness of resonance,
Higher is the value of Q-factor, sharper is the resonance.
Q23. The following table provides the set of values V and I obtained for a given diode. Assuming the characteristics to
be nearly linear, over this range, calculate the forward and reverse bias resistance of the given diode. 2
V I
Forward 2.0 V 60 mA
biasing 2.4 V 80 mA
Reverse 0V 0 µA
biasing –2V – 0.25 µA
∆V
Ans. In forward biasing r =
∆I
DV = 2.4 – 2.0 = 0.4 V
DI = 80 – 60 = 20 mA = 20 × 10–3 A
Forward resistance of diode,
0.4
rfb = = 20W
20 × 10 −3
In reverse biasing: DV = – 2 – 0 = – 2V
DI = – 0.25 – 0 = – 0.25 µA = – 0.25 × 10–6 A
Reverse resistance of diode,
−2
rrb = −6
=8 × 106 W
− 0.25 × 10
Q24. A 15 µF capacitor is connected to a 220 V, 50 Hz source. Find the capacitive reactance and the current (rms and
peak) in the circuit. If the frequency is doubled, what happens to the capacitive reactance and the current?  2
Ans. Vrms = 220V, v = 50 Hz, C = 15 µF = 15 × 10–6 F, XC = ?, Irms = ?, I0 = ?

1 1 1 1 × 106
X
== C = = = 212 ohm
ωC 2πvC 2 × π × 50 × 15 ×10 −6 314 ×15
Vrms 220
  I rms = = = 1.04A
XC 212

  I0 = 2 I rms = 1.414 × 1.04 = 1.47A

484 Physics–12
D:\EG_Physics-12_(26-06-2022)\Open_Files\Sample Paper_1_(Solved)\Sample Paper_1
\ 27-Jul-2022 Ved_Goswami Proof-4 Reader’s Sign _______________________ Date __________

212
When the frequency is doubled, XC will become half of its previous value i.e., = 106 W
2
Q25. A uniform magnetic field B is set up along the positive x-axis. A particle of charge ‘q’ and mass ‘m’ moving
with a velocity v enters the field at the origin in X-Y plane such that it has velocity components both along and
perpendicular to the magnetic field B. Trace, giving reason, the trajectory followed by the particle. Find out the
expression for the distance moved by the particle along the magnetic field in one rotation. 2
OR
Two concentric circular coils X and Y of radius 16 cm and 10 cm respectively lie in the same vertical plane
containing the north-south direction. Coil X has 20 turns and carries a current of 16 A, coil Y has 25 turns and
carries a current of 18A. The sense of the current in X is anticlockwise and in Y is clockwise, for an observer
looking at the coils facing west. Give the magnitude and the direction of the net magnetic field due to the coils at
their centre.
Ans. Field at an axial point of a circular coil:
B= y dB sin θ  ...(1) a x dB cos θ
dB
µ idl θ
Where dB = 0 2 O
θ dB sin θ
4π r
a x P
and sin q = dB cos θ cancels
x × out due to symmetry
On Integrating (1), we get,
µ ia µ ] g
B= 0 3 y dl = 4π0 ia 22πa 2 .3N/2
4π x ^r + a h
2
µ0 ia N

⇒ B=
2 (r 2 + a 2) 3/2
Where N is the number of turns of the circular coil.
If M = iA is the magnetic moment of the coil, then
M = ipa2 for single turn
M = ipa2 N for a coil of N turns
µ 2M
B= 0. 2
4π (r + a 2) 3/2
Q a = 0. For small loop,
µ 2M
B = 0 . 3 Wb/m 2
4π r
This is a similar result as obtained for end on position of electric dipole where
1 2p
E= .
4πε0 r3
where p is electric dipole moment.
OR
For coil X, r = 0.16 m, N = 20, I = 16A
Y

Observer
W E
O

Sample Paper–1 (Solved) 485


D:\EG_Physics-12_(26-06-2022)\Open_Files\Sample Paper_1_(Solved)\Sample Paper_1
\ 27-Jul-2022 Ved_Goswami Proof-4 Reader’s Sign _______________________ Date __________

µ 0 IN 4π × 10 −7 × 16 × 20
Magnetic field at its centre B
Ox = =
2r 2 × 0.16
== 4p × 10−–4
4T
4π ×10 T
As the current is anticlockwise, so field is directed towards east.
For coil Y: r = 0.10 m, I = 18A, N = 25
            µ IN 4π × 10 −7 × 18 × 25
BOY = 0 = = 9π ×10 −4 T
2r 2 × 0.10
Field is directed towards west.
Net field at O is BvO = B
v OX + B
v OY = 5p × 10–4 = 1.6 × 10–3 T towards west.

Section-D
All questions are compulsory. In case of internal choices, attempt any one of them.
Q26. Find the relation between drift velocity and relaxation time of charge carriers in a conductor. A conductor of
length L is connected to a d.c. source of emf ‘E’. If the length of the conductor is tripled by stretching it, keeping
‘E’ constant, explain how its drift velocity would be affected. 3
Ans. The drift velocity of electrons can be formulated as:
qE
vd = at = t
me
where a = average acceleration of the carrier
q = charge of the carrier
me = effective mass of charge carrier
t = carrier lifetime = relaxation time
E = electric field strength
As the drift velocity is directly proportional to the current flowing through the conductor (I = vd en A, i.e., I ∝ vd)
JK V ρl 1 NO
and as the current flowing through the conductor is inversely proportional to its length KK = i.e. I ? OO , so
LI A lP
the drift velocity would be reduced by one third if the length of the conductor is tripled.
Q27. (a) Determine the value of phase difference between the current and the voltage in the given series LCR circuit.
R = 400 Ω

V = V0 sin (1000 t + φ) C = 2µF

L = 100 mH

(b) Calculate the value of the additional capacitor which may be joined suitably to the capacitor C that would make
the power factor of the circuit unity. 3
OR
(i) When an AC source is connected to an ideal inductor show that the average L Lamp
power supplied by the source over a complete cycle is zero.
(ii) A lamp is connected in series with an inductor and an AC source. What
happens to the brightness of the lamp when the key is plugged in and an
iron rod is inserted inside the inductor? Explain.
Ans. (a) V = V0 sin (1000 t + f)
~
Comparing this with
V = V0 sin (w t + f)

486 Physics–12
D:\EG_Physics-12_(26-06-2022)\Open_Files\Sample Paper_1_(Solved)\Sample Paper_1
\ 27-Jul-2022 Ved_Goswami Proof-4 Reader’s Sign _______________________ Date __________

We have,
w = 1000, L = 100 × 10–3 H,
XL = wL = 1000 × 100 × 10–3 = 102 = 100 W
1 1 1
XC = = = × 103 = 500 W
ωC 1000 × 2 × 10 –6 2
Phase difference:
X L – XC
tan f =
R
XC – X L
tan f = , Q here XC > XL
R
We have, R = 400 W
500 – 100 400
tan f = = =1
400 400
tan f = 1
\ tan f = tan 45°
f = 45°
(b) When cos f = 1
f = 0°
\ tan f = 0
XL = XC
1
wL =
ωC
1 1 1

fi C= 2 = 2
= 2 2
ω L (2πf) L 4π f L
1
Then 2 × 10–6 + C1 = 2 2
4π f L
1 1
2 × 10–6 + C1 = 2 2 –3
= –1
4 × (3.14) × (50) × 100 × 10 4 × 9.8596 × 2500 × 10
1 1
= =
4 × 9.8596 × 250 9859.6
1 1 2
C1 = – 2 × 10–6 = –
9859.6 9859.6 106
= 0.000101424 – 0.000002 = 0.00009942 = 99.42 µF
OR

(i) Pinst = Vinst × Iinst
2V0 I0 sin wt cos wt
dW = [V0 coswt. I0 sinwt]dt = 2 dt
V0 I0 T V0 I0 – cos 2wt T
W= y sin 2ωt dt = < F
2 0 2 2w 0
– V0 I0
W= 5cos 2wT – cos 0?
4w
– V0 I0 T
W= [cos2wT – cos 0]
4 # 2π
W
=0
T
Pav = 0

Sample Paper–1 (Solved) 487


D:\EG_Physics-12_(26-06-2022)\Open_Files\Sample Paper_1_(Solved)\Sample Paper_1
\ 27-Jul-2022 Ved_Goswami Proof-4 Reader’s Sign _______________________ Date __________

(ii) when iron rod is inserted then self inductance of the coil increases as
µ0 µ r N 2 A
L=
l
So inductive reactance (wL) increase and the current decreases. The brightness of lamp will decrease.
Q28. (a) Describe briefly three experimentally observed features in the phenomenon of photoelectric effect.
(b) Discuss briefly how wave theory of light cannot explain these features. 3
Ans. (a) (i) The photoelectric effect will not occur when the frequency of the incident light is less than the threshold
frequency. Different materials have different threshold frequencies and most elements have threshold
frequencies in the ultraviolet region of the electromagnetic spectrum.
(ii) The maximum KE of a stream of photo electrons increases linearly with the frequency of the incident light
above the threshold frequency.
(iii) The rate at which photo electrons are emitted from a photosensitive surface is directly proportional to the
intensity of incident light when the frequency is constant.
(b) Classical wave theory cannot explain:
(i) The existence of threshold frequency because it predicts that electrons would absorb enough energy to
escape and there would not be any threshold frequency.
(ii) The almost immediate emission of photo electrons as, according to this theory, electrons require a period
of time before sufficient energy is absorbed by it to escape from the metal; however such a thing does not
happen practically; and
(iii) The independence of KE of photo electrons on intensity and dependence on frequency because it cannot
explain why maximum KE is dependent on frequency and independent of intensity.
Q29. A student wants to use two p-n junction diodes to convert alternating current into direct current. Draw the labelled
circuit diagram she would use and explain how it works.  3
Ans. Circuit diagram to use two p-n junction diodes to convert ac into dc (circuit diagram of full wave rectifier).
D1
ac to be rectified

D1
Output t

D2
RL t

D2 Output t

Working of full wave rectifier – circuit diagram shown in the figure gives rectified output voltage corresponding
to both the positive as well as negative half cycles of ac. So it is known as full wave rectifier. p type semiconductors
of both the junction diodes D1 and D2 are connected to the ends of secondary of the centre tape transformer and n
type semiconductors of both the junction diodes are connected together and rectified output is taken between the
common point of diodes and the midpoint of secondary of transformer. So for a full wave rectifier the secondary
of the transformer is provided with a centre tapping, and due to this transformer is called centre tap transformer. At
a time either D1 or D2 is forward biased, and rectifies only half the cycle, and both do so alternate half cycle. Thus
the output between their common terminals and the centre tap of the transformer becomes a full wave rectified
output, i.e. we get full wave rectified output voltage during both, the positive as well as negative half of the cycle.
Q30. A hydrogen atom initially in the ground level absorbs a photon, which excites it to the n = 4 level. Determine the
wavelength and frequency of photon. 3
OR
The radius of the innermost electron orbit of a hydrogen atom is 5.3 × 10−11 m. What are the radii of the n = 2 and
n = 3 orbits?
Ans. Energy of an electron in nth orbit of H atom
−13.6
En = eV
n2
E1 = – 13.6 eV
488 Physics–12
D:\EG_Physics-12_(26-06-2022)\Open_Files\Sample Paper_1_(Solved)\Sample Paper_1
\ 27-Jul-2022 Ved_Goswami Proof-4 Reader’s Sign _______________________ Date __________

Energy is 4th (n = 4) level


−13.6
R4 = = – 0.85
42
∆E = E4 – E1 or ∆E = – 0.85 – (– 13.6) eV
∆E = – 0.85 + 13.6 = 12.75 eV
hν = 12.75 eV
hν = 12.75 × 1.6 × 10–19 J
12.75 × 1.6 × 10 − 19
ν= = 3.078 × 1015 Hz
6.6 × 10 −34
c 3 × 108
λ= = = 974.4 Å.
ν 3.078 × 1015
OR
r0 = 5.3 × 10−11
m, r = r0 . n2
(i) when n = 2, r = 5.3 × 10−11 × (2)2
or, r = 21.2 × 10−11 m = 2.12 × 10−10 m [Q rn = 0.53 × n2 Å]
(ii) when n = 3, r = 5.3 × 10−11 m × (3)2
= 47.7 × 10−11 m = 4.77 × 10−10 m.

Section-E
All questions are compulsory. In case of internal choices, attempt any one of them.
Q31. (a) Define electric flux. Write its S.I. unit.
“Gauss’s law in electrostatics is true for any closed surface, no matter what its shape or size is”. Justify this
statement with the help of a suitable example.
(b) Use Gauss’s law to prove that electric field inside a uniformly charged spherical shell is zero. 5
OR
(a) Derive the expression for the energy stored in a parallel plate capacitor. Hence obtain the expression for the
energy density of the electric field.
(b) A fully charged parallel plate capacitor is connected across an uncharged identical capacitor. Show that the
energy stored in the combination is less than that stored initially in the single capacitor.
Ans. (a) The electric flux through an area is defined as the electric field multiplied by the area of the surface projected
on a plane, perpendicular to the field. Its S.I. unit is volt-metres (Vm) or Newton metres square per coulomb
(Nm2 C–1). The given statement is justified because while measuring the flux, the surface area is more important
than its volume or its size.
(b) To prove that the electric field inside a uniformly charged spherical shell is zero, we place a single positive
point charge ‘q’ at the centre of an imaginary spherical surface with radius R. The field lines of this point radiate
outside equally in all directions. The magnitude E of the electric field at every point on the surface is given by
1 q
E= × .
4πε0 R 2
At each point on the surface, Ε is ^ to the surface and its magnitude is the same.
Thus, the total electric flux (fE) is the product of their field magnitude E and the A.
1 q 2 q
Hence, fE = EA = (4πR ) =
4πε0 R 2 ε0
If the sphere is uniformly charged, then there is zero charge inside the sphere, according to Gauss’s law. When
q = 0,
0
fE = =0
ε0

Sample Paper–1 (Solved) 489


D:\EG_Physics-12_(26-06-2022)\Open_Files\Sample Paper_1_(Solved)\Sample Paper_1
\ 27-Jul-2022 Ved_Goswami Proof-4 Reader’s Sign _______________________ Date __________

OR
(a) When charging parallel plate capacitor, whose plate area is A and distance between the plate is d. When charge
0+V V
on capacitor is q, potential difference increase 0 to V. Hence average P.D. is = . Total work done, W
2 2
V 1
= q × = qV. Here, work done is equal to energy stored by a capacitor.
2 2
1 1
W = qV = CV 2
2 2
1 2
u CV 1 CV 2 KJ d ON 1 Cd KJ V ON2 1
Energy density = = 2 = KK OO = KK OO = ε E 2 J/m3
V Ad 2 dA L d P 2 A Ld P 2 0
(b) Initially, if we consider a charged capacitor, then its charge would be Q = CV
1 A V B
and energy stored in it is U1= CV2 ...(1)
2 C
Then, this charged capacitor is connected to uncharged capacitor. The charge
flows from first capacitor to the other capacitor until both the capacitor attain the common potential.
So, we have Q = CV1 and Q = CV2 C1
Using law of conservation of charge, we get V1
Q = Q1 + Q2 fi CV = CV1 + CV2
fi V = V 1 + V2 C2
V V2
fi V = V 1 + V1 fi V 1 =
2
1 1 1 JK V NO2 1 JK V NO2
\ Total energy stored, U2 = CV12 + CV22 = C KK OO + C KK OO
2 2 2 L2P 2 L2P
1
fi U2 = CV2...(2)
4
With the help of eqn. (1) and (2), we get U2 < U1
So, energy stored in the combination is less than that stored initially in single capacitor.
Q32. Explain, using a labelled diagram, the principle and working of a moving coil galvanometer. What is the function of
(i) uniform radial magnetic field, (ii) soft iron core? Define the terms (i) current sensitivity and (ii) voltage sensitivity
of a galvanometer. Why does increasing the current sensitivity not necessarily increase voltage sensitivity? 5
OR
(a) Write using Biot-Savart law, the expression for the magnetic field Β due to an element dl carrying current i
at a distance r from it in a vector form.
Hence derive the expression for the magnetic field due to a current carrying loop of radius R at a point P distant
x from its centre along the axis of the loop.
(b) Explain how Biot-Savart law enables one to express the Ampere’s Circuital law in the integral form, viz.
y B . dl = µ0I
Where I is the total current passing through the surface. O
P Q
Ans. The basic principle of a moving coil galvanometer is that when a
current carrying coil is placed in a magnetic field, it experiences
C
a torque. O
N S
When the current I is passed through the coil, the torque R
experienced is given by E
t = NIAB sin q
S R
Where N = no. of turns of the coil, T1 T2
A = area of the coil

490 Physics–12
D:\EG_Physics-12_(26-06-2022)\Open_Files\Sample Paper_1_(Solved)\Sample Paper_1
\ 27-Jul-2022 Ved_Goswami Proof-4 Reader’s Sign _______________________ Date __________

B = magnetic field and


q = angle between normal of coil and magnetic field
(i) The uniform radial magnetic field allows the coil to rotate and
(ii) The soft iron core makes the magnetic field linked with the coil to be radial.
The current sensitivity is defined as the deflection produced in the galvanometer, while passing a current of 1
ampere (1 amp).
JK α NO NBA
Thus, current sensitivity = KK OO =
LIP K
The voltage sensitivity is defined as the deflection produced in the galvanometer when a potential difference of
1V is applied to the coil.
JK α NO NBA
Thus, voltage sensitivity = KK OO =
LVP KR
Where, R is the resistance.
Increasing the current sensitivity does not necessarily increase the voltage sensitivity as there is an increase in the
resistance as well.
OR
(a) Suppose we have a conductor of length l in which current i is flowing. We need dB dB

to calculate the magnetic field at a point P in vacuum. If i dl in one of the P×

infinitely small current element, the magnetic field d Β at point P in given by


r r
θ
idl × r
dΒ µ
r3 i
µ idl × r
dΒ = 0
4π r3
µ0
Where
is a proportional constant.

Suppose there be a circular coil of radius R, carrying dB cos θ dB
dl
a current i. Let P be a point at the axis of the coil at a
distance x from the centre, at which the field is required. R
r
θ
Consider a conducting element dl of the loop. The θ dB sin θ
magnetic field due to dl in given by the Biot-Savart Law x P
µ i | dl × r |
dB = 0 dB
4π r
3
µ idl
⇒ dB = 0 2
4π r
The direction of dB is perpendicular to the plane formed by dl and r. It has an x-component dBx. When the
components perpendicular to the x-axis are summed over, they cancel out and we obtain null result. Thus only
the x-component survives.
So the resultant field Β at P is given by
µ0 i
B= y dB sin θ   ⇒ B=
4π r 2
y dl sin θ
µ0 iR
B=
4π r3
y dl
But y dl = 2pR and r = (R2 + x2)1/2
µ0 2πiR 2

\ B=
4π (R 2 + x 2) 3/2

Sample Paper–1 (Solved) 491


D:\EG_Physics-12_(26-06-2022)\Open_Files\Sample Paper_1_(Solved)\Sample Paper_1
\ 27-Jul-2022 Ved_Goswami Proof-4 Reader’s Sign _______________________ Date __________

If the coil has N turns, then each turn will contribute equally to B. Then
µ0 NiR 2
B=
2 (x 2 + R 2) 3/2
(b) According to Biot-Savart law the line integral of the magnetic field Β around any ‘closed’ path is equal to µ0
times the net current I threading through the area enclosed by the path.

i.e., y B . dl = µ0I
Where µ0 is the permeability of free space. Ampere’s circuital law in electromagnetism is analogous to Gauss’
law in electrostatics.
Q33. (a) Consider two coherent sources S1 and S2 producing monochromatic waves to produce interference pattern. Let
the displacement of the wave produced by S1 be given by y1 = a cos wt and the displacement by S2 be y2 = a cos
(wt + f). Find out the expression for the amplitude of the resultant displacement at a point and show that the
intensity at that point will be I = 4a2 cos2 f/2. Hence establish the conditions for constructive and destructive
interference.
(b) What is the effect on the interference fringes in Young’s double slit experiment when (i) the width of the source
slit is increased. (ii) the monochromatic source is replaced by a source of white light. 5
OR
(a) A ray of light is incident on the face AB of a glass prism ABC (as shown in the figure) and emerges out of the
face AC. Trace the path of the ray. Show that –i + –e = –A + –d. Where d and e denote the angle of deviation
and angle of emergence respectively.
A
Plot a graph showing the variation of the angle of deviation as a function
of angle of incidence. State the condition under which –d is minimum.
(b) A ray of light passing through an equilateral triangular prism from air P
i
undergoes minimum deviation when angle of incidence is 3/4th of the
angle of prism. Calculate the speed of light in the prism.
Ans. (a) Let S be a narrow slit illuminated by a monochromatic source of light,
and S1 and S2 two similar parallel slits very close together and equidistant B C
from S.
Displacement of the wave produced by S1 is given by P
y1 = a cos wt and
the displacement of the wave produced by S2 is given by S1
y2 = a cos (wt + f) Screen
S
The resultant displacement is given by
y = y1 + y2 = a cos wt + a cos (wt + f) S2
⇒ y = 2a cos f/2 cos (wt + f/2)
The amplitude of the resultant displacement is 2a cos f/2. The intensity of light is directly proportional to the
square of amplitude of the wave.
The resultant intensity in given by
I = 4a2 cos2 f/2
For constructive interference the intensity of light at point P is maximum, if
cos f = max = +1 : f = 0, 2p, 4p...
i.e., f = 2np where n = 0, 1, 2, ...
For destructive interference the intensity of light at point P is minimum, if
cos f = –1
\ f = p, 3p, 5p...
or f = (2n – 1)p where n = 1, 2, 3...

492 Physics–12
D:\EG_Physics-12_(26-06-2022)\Open_Files\Sample Paper_1_(Solved)\Sample Paper_1
\ 27-Jul-2022 Ved_Goswami Proof-4 Reader’s Sign _______________________ Date __________

The corresponding path difference between the two waves


λ λ λ
x= φ= (2n – 1) π = (2n – 1)
2π 2π 2
(b) (i) As the width of the slits is increased, the fringe width decreases. It is because,
1

d
(ii) The different colours of white light will produce different interference patterns but the central bright fringes
due to all colours are at the same positions. Therefore, the central bright fringe is white in colour. Since
the wavelength of the blue light is smallest, the fringe close to the either side of the central white fringe is
blue and farthest is red. Beyond a few fringes, no clear fringe pattern is visible.
OR
(a) Let the incident ray meet refracting face AB of the prism at point P. Ray PQ is the refracted ray inside the prism
and ∠d1 and ∠r1 are the angle of the deviation and refraction at interface AB. At interface AC the ray goes out
of the prism. Let e be the angle of emergence. The angle of deviation at point Q is ∠d2 as shown in figure.
Using geometry, we see that at point P,
∠i = ∠d1 + ∠r1 \ ∠d1 = ∠i – ∠r1 A

and at point Q ∠e = ∠d2 + ∠r2


A
∠d2 = ∠e – ∠r2
The total deviation ∠d, suffered by the incident ray is equal to ∠d1
+ ∠d2. Incident ray P δ1 Q e
i
or ∠d = ∠d1 + ∠d2 r 1
r 2 δ2

= (∠i – ∠r 1 ) + (∠e – ∠r 2 )
= (∠i + ∠e) – (∠r1 + ∠r2) ...(i) O

In quadrilateral POQA, the sum of all four angles is 360°.


B C
∠P + ∠O + ∠Q + ∠A = 360° Principal section of equilateral glass prism
as P and Q both are right angles
∠P + ∠Q = 180°
\ ∠O + ∠A = 180° ...(ii)
In triangle POQ
∠O + ∠r1 + ∠r2 = 180° ...(iii)
Comparing equations (ii) and (iii), we have
∠A = ∠r1 + ∠r2
Substituting this value in equation (i)
∠d = ∠i + ∠e – ∠A
∠d + ∠A = ∠i + ∠e...(iv)
So angle of deviation produced by a prism depends upon the angle of incidence, refracting angle of prism, and
the material of the prism.
Y
Angle of deviation

∠i − δ curve
δm

(i = e)

X
10 20 30 40 50 60 70 80
Angle of incidence (i)

Sample Paper–1 (Solved) 493


D:\EG_Physics-12_(26-06-2022)\Open_Files\Sample Paper_1_(Solved)\Sample Paper_1
\ 27-Jul-2022 Ved_Goswami Proof-4 Reader’s Sign _______________________ Date __________

When prism in the position of minimum deviation, then


∠i = ∠e and ∠r1 = ∠r2
By equation (iv) ∠d + ∠A = ∠i + ∠i
(+δ + +A)
∠i =
2
3 3
(b) A = 60°, ∠i = ∠A= × 60°= 45°
4 4
At minimum deviation
∠r1 = ∠r2 = ∠r
∠A
So using ∠r1 + ∠r2 = ∠A, ∠ =
r = 30°
2
sin i sin 45° 1 × 2
m= = = = 2
sin r sin 30° 2
c 3 × 108
m= , 2=
v v
3 × 108
or v= = 2.12 × 108 m/s
2
rrr

494 Physics–12
D:\EG_Physics-12_(26-06-2022)\Open_Files\Sample Paper_2_(Unsolved)\Sample Paper_2_(Unsolved)
\ 27-Jul-2022 Ved_Goswami Proof-4 Reader’s Sign _______________________ Date __________

Sample Paper– 2 (Unsolved)


Time Allowed: 3 hours Maximum Marks: 70
General Instructions: Same as Sample Paper-1

Section-A
All questions are compulsory. In case of internal choices, attempt any one of them.
I. Objective Type Questions (1 Mark)
Q1. The electric field at a point is 1
(a) always continuous. (b) continuous if there is charge at that point.
(c) discontinuous only if there is a negative charge at that point.
(d) discontinuous if there is a charge at that point.
Q2. The self inductance L of a long solenoid of length l and area of cross-section A, with a fixed number of turns N
increases as 1
(a) l and A increases (b) l decreases and A increases
(c) l increases and A decreases (d) Both l and A decreases.
+
Q3. The radius of second orbit of He ion will be 1
(a) 1.058 Å (b) 3.023 Å (c) 2.068 Å (d) 4.058 Å
Q4. The electrical conductivity of semiconductor increases when electromagnetic radiation of wavelength shorter than 2800
nm is incident on it. The band gap in (eV) for the semiconductor is 1
(a) 0.7 eV (b) 0.5 eV (c) 2.5 eV (d) 1.2 eV
II. Very Short Answer Type Questions (1 Mark)
Q5. On which law Kirchhoff’s junction rule is based? 1
OR
What is the basic purpose of using a capacitor?
Q6. If the rate of change of current of 2 ampere/second induces an emf of 40 mV in the solenoid, what is the self-
inductance of this solenoid? 1
OR
If the self-inductance of an iron core inductor increases from 0.01 mH to 10 mH on introducing the iron core into
it, what is the relative permeability of the core material used?
Q7. Must every magnetic configuration have a north pole and a south pole? 1
OR
What is the basic difference between electric and magnetic field lines?
Q8. An alternating current generator has an internal resistance Rg and an internal reactance Xg. It is used to supply
power to a passive load consisting of a resistance Rg and reactance XL. Find the maximum power to be delivered
from the generator to load the value of XL. 1
Q9. A short pulse of white light is incident from air to glass slab at normal incidence. After travelling through the slab,
which colour to be emerge first? 1
OR
When light travel from a rarer to denser medium, the speed decreases. Does this decrease in speed imply a decrease
in the energy carried by the light wave? Justify your answer.
Q10. Find the wavelength of a photon needed to remove a proton from a nucleus which is bound to the nucleus with 1
MeV energy. 1
495
D:\EG_Physics-12_(26-06-2022)\Open_Files\Sample Paper_2_(Unsolved)\Sample Paper_2_(Unsolved)
\ 27-Jul-2022 Ved_Goswami Proof-4 Reader’s Sign _______________________ Date __________

For questions numbers 11, 12, 13 and 14, two statements are given-one labelled Assertion (A) and the other labelled
Reason (R). Select the correct answer to these questions from the codes (a), (b), (c) and (d) as given below.
(a) Both A and R are true and R is the correct explanation of A
(b) Both A and R are true but R is NOT the correct explanation of A
(c) A is true but R is false
(d) A is false and R is also false
3
Q11. Assertion (A): Critical angle for total internal reflection (T.I.R) at glass-water interface is qc. If mg = 2 and mw =
4 π
3 , then angle of incidence (i) of a ray in water, so that its angle of refraction in glass qc, will be 2 . 1
Reason (R): Total internal reflection takes place if the angle of incidence at the denser-rarer interface is more than
the critical angle.
Q12. Assertion (A): In optical fibre, the radius of core is kept small.  1
Reason (R): The smaller radius of the core ensures the fibre should have incident angle more than the critical
angle required for total internal reflection.
Q13. Assertion (A): In the phenomenon of mutual induction, self induction of each of the coil persists. 1
Reason (R): Self-induction arises when strength of current in one coil changes. In mutual induction, current is
changing in both the individual coils.
Q14. Assertion (A): Balmer series lies in the visible region of electromagnetic spectrum. 1
Reason (R): λ = R ; 2 − 2 E , where n = 3, 4, 5,.....
1 1 1

2 n
Section-B
Question 15 and 16 are Case Study based questions and are compulsory. Attempt any 4 sub-parts from each
question. Each sub part carries 1 mark.
1
Q15. Gauss theorem: The total flux through a closed surface, enclosing a volume, in vacuum is, times the net
charge, enclosed by the surface. ε0 4
→ → qenclosed
f= ∫ E . dS =
ε0
s
Gaussian surface: Any closed surface imagined around the charge distribution, so that Gauss theorem can be
conveniently applied to find electric field due to the given charge distribution.
Electric field due to infinitely long straight charged wire of linear charge density λ:
λ
E = , where r is the perpendicular distance of the observation point from the wire.
2πε 0 r
Electric field due to an infinite plane sheet of charge of surface charge density σ.
σ
E=
2ε 0
1. S.I. unit of electric flux is
(a) N2 mC (b) NmC–2 (c) Nm2C–1 (d) Nm–2C
2. Electric flux is a
(a) constant quantity (b) vector quantity (c) scalar quantity (d) None of these
3. Two charges of magnitude –2Q and +Q are located at points (a, 0) and (4a, 0) respectively. What is the electric
flux due to these charges through a sphere of radius ‘3a’ with its centre at origin?
Q – 2Q 3Q – 3Q
(a) (b) (c) (d)
ε0 ε0 ε0 ε0
4. A charge q is placed at the centre of a cube of side l. What is the electric flux passing through each face of the
cube?
q q q q
(a) (b) (c) (d)
5ε 0 9ε 0 6ε 0 ε0

496 Physics–12
D:\EG_Physics-12_(26-06-2022)\Open_Files\Sample Paper_2_(Unsolved)\Sample Paper_2_(Unsolved)
\ 27-Jul-2022 Ved_Goswami Proof-4 Reader’s Sign _______________________ Date __________

5. The figure shows three charges +2q, – q and +3q. Two charges +2q and – q are enclosed within a surface ‘S’. What
is the electric flux due to this configuration through the surface ‘S’?

5q 3q 2q q
(a) (b) (c) (d)
2ε 0 ε0 ε0 ε0
Q16. Diode as a Half-Wave rectifier: Diode conducts corresponding to positive half cycle and does not conduct during
negative half cycle. Hence, ac is converted by diode into unidirectional pulsating dc. This action is known as half-
wave rectification. 4

Diode as a Full-Wave rectifier: In the full-wave rectifier, two p-n junction (Here, D1 and D2) diodes, are used.
Its principle is that junction diode puts very low resistance in forward bias while very high resistance in reverse
bias condition.

Sample Paper–2 (Unsolved) 497


D:\EG_Physics-12_(26-06-2022)\Open_Files\Sample Paper_2_(Unsolved)\Sample Paper_2_(Unsolved)
\ 27-Jul-2022 Ved_Goswami Proof-4 Reader’s Sign _______________________ Date __________

The rectified voltage is in the form of pulses of the shape of half sinusoidals. Although it is unidirectional but not
steady. To get steady dc output from the pulsating voltage, a capacitor is connected across the output terminals,
parallel to load resistance which will filter out the ac ripple and provide a pure dc steady output.

1. The a.c. to be rectified is applied to the


(a) primary of a transformer (b) secondary of a transformer
(c) between primary and secondary (d) anywhere on the diode
2. The secondary coil of the transformer is tapped in the
(a) corner (b) middle (c) upper half (d) lower half
3. During positive half of a.c. cycle diode D1, is in forward biasing and it conducts. At this point the diode D2 will
not conduct because
(a) being in reverse biasing (b) being in forward biasing.
(c) as in high biasing (d) as in low biasing
4. The output of full wave rectifier is
(a) pulsating a.c. (b) pulsating d.c. (c) constant a.c. (d) constant d.c.
5. Pulsations can be minimised by using suitable
(a) filter circuit (b) CL circuit (c) RC circuit (d) CR circuit

Section-C
All questions are compulsory. In case of internal choices, attempt any one of them.
Q17. If the potential difference across a capacitor is doubled, what happens to :
(i) The charge on the capacitor and (ii) The energy stored in the capacitors. 2
OR
Obtain an expression for the equivalent capacitance of a parallel grouping of three capacitors of capacitances C1,
C2 and C3.
Q18. Figure shows a rectangular loop PQRS in which the arm PQ is free to move. A uniform magnetic field acts in the
direction perpendicular to the plane of the loop. Arm PQ is moved with a velocity v towards the arm RS Assuming
that the arms QR, RS and SP have negligible resistances and the moving arm PQ has the resistance r, obtain the
expression for (i) the current in the loop (ii) the force and (iii) the power required to move the arm PQ.  2

OR
Two identical circular loops, P and Q, each of radius r and carrying equal
currents are kept in the parallel planes having a common axis passing through
O. The direction of current in P is clockwise and in Q is anti-clockwise as
seen from O which is equidistant from the loops P and Q. Find the magnitude
of the net magnetic field at O.

498 Physics–12
D:\EG_Physics-12_(26-06-2022)\Open_Files\Sample Paper_2_(Unsolved)\Sample Paper_2_(Unsolved)
\ 27-Jul-2022 Ved_Goswami Proof-4 Reader’s Sign _______________________ Date __________

Q19. Distinguish between n-type and p-type semiconductor. 2


Q20. The wire shown in figure below carries a current of 60 A. Find the magnetic field B at P. 2

Q21. Suggest a suitable method to invert an image


(i) Without change in size.
(ii) Without change in size and without deviation from its original direction of view.
Explain how the same is achieved? 2
OR
An equi-convex lens has refractive index 1.5. Write its focal length in terms of radius of curvature R.
Q22. Find the wavelength of electromagnetic waves of frequency 5 × 1019 Hz in free space. Give its two applications.
 2
Q23. Draw the shape of the refracted/reflected wavefront when a plane wavefront is incident on (i) Prism (ii) Convex
mirror. Give brief explanation for the construction. 2
Q24. Find the ratio of the de-Broglie wavelength, associated with protons, accelerated through a potential of 128 V and
α particles, accelerated through a potential of 64 V. 2
Q25. Draw a plot showing the variation of (i) Electric field (E) and (ii) Electric potential (V) with distance r due to a
point charge Q. 2

Section-D
All questions are compulsory. In case of internal choices, attempt any one of them.
Q26. In a chamber, a uniform magnetic field of 6.5 G (1 G = 10−4 T) is maintained. An electron is shot into the field
with a speed of 4.8 × 106 m s−1 normal to the field. Explain why the path of the electron is a circle. Determine the
radius of the circular orbit. (e = 1.6 × 10−19 C, me = 9.1 × 10−31 kg) 3
OR
(a) A circular coil of 30 turns and radius 8.0 cm, carrying a current of 6.0 A is suspended vertically in a uniform
horizontal magnetic field of magnitude 1.0 T. The field lines make an angle of 60° with the normal to the coil.
Calculate the magnitude of the counter torque that must be applied to prevent the coil from turning.
(b) Would your answer change if the circular coil in (a) were replaced by a planar coil of some irregular shape that
encloses the same area? (All other particulars are also unaltered).
Q27. The binding energy per nucleon of deuteron ( 21 H ) and helium nucleus ( 42 He ) is known to be 1.1 MeV and 7 MeV
respectively. If two deuteron nuclei react to form a single helium nucleus; calculate the energy released. 3
Q28. Draw a plot of potential energy of a pair of nucleons as a function of their separation. Write two important conclusions
which you can draw regarding the nature of nuclear forces. 3
Q29. Group the following six nuclides into three pairs of (i) isotones (ii) isotopes and (iii) isobars. 6C12, 2He3, 80Hg198,
3 197, C14 3
1H , 79Au 6
Q30. (a) Figure shows a cross-section of a ‘light pipe’ made
of a glass fibre of refractive index 1.68. The outer
covering of the pipe is made of a material of refractive
index 1.44. What is the range of the angles of the

Sample Paper–2 (Unsolved) 499


D:\EG_Physics-12_(26-06-2022)\Open_Files\Sample Paper_2_(Unsolved)\Sample Paper_2_(Unsolved)
\ 27-Jul-2022 Ved_Goswami Proof-4 Reader’s Sign _______________________ Date __________

incident rays with the axis of the pipe for which total reflection inside the pipe take place, as shown in the
figure. 3

(b) What is the answer if there is no outer covering of the pipe?


OR
Use Huygen’s principle to verify Snell’s law of refraction.

Section-E
All questions are compulsory. In case of internal choices, attempt any one of them.
Q31. Define relaxation time of the free electrons drifting in a conductor. How is it related to the drift velocity of free
electrons? Use this relation to deduce the expression for the electrical resistivity of the material.  5
OR
State Gauss Law in electrostatics. Use this law to derive an expression for the electric field due to an infinitely
long straight wire of linear charge density l Cm–1.
Q32. (i) (a) Draw a labelled ray diagram to show the formation of image in an astronomical telescope for a distant
object,
(b) Write three distinct advantages of a reflecting type telescope.
(ii) A convex lens of focal length 10 cm is placed coaxially 5 cm away from a concave lens of focal length 10
cm. If an object is placed 30 cm in front of the convex lens, find the position of the final image formed by the
combined system.  5
OR
With the help of ray diagram, show the formation of image of a point object by refraction of light at a spherical
surface separating two media of refractive indices n1 and n2 (n2 > n1) respectively. Using this diagram, derive the
relation.
n2 n1 n2 – n1
– =
v u R
Write the sign conventions used. What happens to the focal length of convex lens when it is immersed in water?
Q33. State the principle of working of a transformer. Can a transformer be used to step up or step down a d c. voltage?
Justify your answer. Mention various energy losses in transformer? 5
OR
Define working, principle and construction of a moving galvanometer with its labelled diagram. How a galvanometer
can be converted into an ammeter and voltmeter.

Answers
1. (d) 2. (b) 3. (a) 4. (b) 5. Conservation of charge
6. 20 mH OR 1000 7. Not necessarily 8. –Xg 9. red
10. 1.2 × 10–3 nm 11. (b) 12. (a) 13. (a) 14. (b)
15. 1. (c) 2. (c) 3. (b) 4. (c) 5. (d)
16. 1. (a) 2. (b) 3. (a) 4. (b) 5. (a)
Blv B2 l 2 v B2 l 2 v 2 µ0 NI
17. (i) doubles (ii) Quadruples 18. (i) ; (ii) ; (iii) OR
r r r (2)3/2 r
20. 1.4 × 10–3 T 21. OR f = R
22. 6 × 10–12 m  24. 2 : 1 26. 4.2 cm OR (a) 3.12 Nm (b) remain unchanged
12 14
27. 23.6 MeV 29. (i) 198
80 Hg
, 197
79 Au
; (ii) 6 C, 6 C ; (iii) 32 He, 13H
30. (a) 0 < i < 60°; (b) i′c = 36.5° 32. (ii) infinity
rrr

500 Physics–12
D:\EG_Physics-12_(26-06-2022)\Open_Files\Sample Paper_3_(Unsolved)\Sample Paper_3_(Unsolved)
\ 27-Jul-2022 Ved_Goswami Proof-4 Reader’s Sign _______________________ Date __________

Sample Paper– 3 (Unsolved)


Time Allowed: 3 hours Maximum Marks: 70
General Instructions: Same as Sample Paper-1

Section-A
All questions are compulsory. In case of internal choices, attempt any one of them.
Q1. The coefficient of mutual inductance, when magnetic flux changes by 2 × 10–2 Wh and current changes by 0.01 A
is 1
(a) 2H (b) 4 H (c) 3 H (d) 28 H
Q2. The self inductance L of a long solenoid of length l and area of cross-section A, with a fixed number of turns N
increases as 1
(a) l and A increases (b) l decreases and A increases
(c) l increases and A decreases (d) Both l and A decreases.
Q3. A positively charged particle is released from rest in a uniform electric field. The electric potential energy of the
charge
(a) remains a constant because the electric field is uniform.
(b) increases because the charge moves along the electric field.
(c) decreases because the charge moves along the electric field.
(d) decreases because the charge moves opposite to the electric field. 1
Q4. If the kinetic energy of a free electron doubles, its de-Broglie wavelength changes by the factor 1
1
(a) 2 (b) (c) 2 (d) 1/2
2
Q5. Write the value of the electric flux through the surfaces (i) to (iv). 1

OR

Figure shows three point charges, +2q, –q and +3q. Two charges + 2q and –q are enclosed within a surface ‘S’.
What is the electric flux due to this configuration through the surface ‘S’?

+2q

–q S
+3q

→ → →
Q6. Use the expression F = q ( v × B) to define the SI unit of magnetic field.  1

501
D:\EG_Physics-12_(26-06-2022)\Open_Files\Sample Paper_3_(Unsolved)\Sample Paper_3_(Unsolved)
\ 27-Jul-2022 Ved_Goswami Proof-4 Reader’s Sign _______________________ Date __________

OR

A plot of magnetic flux (φ) versus current (I) is shown in the figure for two inductors A and B. Which of the two
has larger value of self inductance.
A

φ B

Q7. The refractive index of glass is 1.520 for red light and 1.525 for blue light. Let D1 and D2 be angles of minimum
deviation for red and blue light respectively in a prism of this glass. Then find relation between D1 and D2. 1
OR
A glass lens of refractive index 1.45 disappears when immersed in a liquid. What is the value of refractive index
of the liquid?
Q8. Fusion processes, like combining two deuterons to form a He nucleus are impossible at ordinary temperature and
pressure. Give reason. 1
Q9. Write the term collectively known for the protons and neutrons which are present in the nuclei of atoms. 1
Q10. What happens to the width of depletion layer of a p-n junction when it is
(i) forward biased? (ii) reverse biased?  1
OR
Why cannot we take one slab of p-type semiconductor and physically join it to another slab of n-type semiconductor
to get p-n junction?
For questions numbers 11, 12, 13 and 14, two statements are given-one labelled Assertion (A) and the other labelled
Reason (R). Select the correct answer to these questions from the codes (a), (b), (c) and (d) as given below.
(a) Both A and R are true and R is the correct explanation of A
(b) Both A and R are true but R is NOT the correct explanation of A
(c) A is true but R is false
(d) A is false and R is also false
Q11. Assertion (A): A real object is placed on the axis of a lens so in such a manner that magnification (m) of the image
is + 1.5. Then the lens must be a concave lens.
Reason (R): A concave lens always produces a virtual image of a real object. 1
Q12. Assertion (A): When a source of emf is attached across a conductor, current is immediately established in the
conductor.
Reason (R): Drift velocity (vd) of electrons is so large that electrons exit from one end of conductor can travel to
the other end almost at once. 1
Q13. Assertion (A): A parallel plate capacitor is connected across battery through a key. A dielectric slab of dielectric
constant K is introduced between the plates. The energy which is stored becomes K times.
Reason (R): The surface density of charge on the plate remains constant or unchanged. 1
Q14. Assertion (A): Only a change in magnetic flux (Df) will maintain an induced current in the copper coil.
Reason (R): The presence of large magnetic flux through a copper coil maintains a current in the coil if the circuit
is continuous. 1

Section-B
Question 15 and 16 are Case Study based questions and are compulsory. Attempt any 4 sub parts from each
question. Each sub part carries 1 mark.
Q15. Metre Bridge: It is a device based on wheatstone bridge and can be used to determine the unknown
resistance.

502 Physics–12
D:\EG_Physics-12_(26-06-2022)\Open_Files\Sample Paper_3_(Unsolved)\Sample Paper_3_(Unsolved)
\ 27-Jul-2022 Ved_Goswami Proof-4 Reader’s Sign _______________________ Date __________

It is so called because it consists of 1 m long constantan or manganin wire stretched taut and clamped between
two thick metallic strips as shown in the diagram R is an unknown resistance whose value we can determine using
metre bridge. 4

The jockey is connected to some point D on the wire at a distance l cm from the end A.
The % error in measurement of R can be minimised by adjusting the balance point near the middle of the
bridge.
At null point
R R AD
S = R DC
R kl1
fi  S = k (100 − l1)
l
R = e 1001− l o S
1
It can also be used to calculate the specific resistance or resistivity of the material used for making resistor S. Let
l be the length of the resistor S and A its area of cross section then using
ρl
S = A , we get
SA Sπr 2
r= l = l ,
r = radius of wire S
1. In the metre bridge experiment, balance point was observed at J with AJ = l. Then values of R and X were doubled
and then interchanged. Then new position of balance point would be

(a) (100 – l) from A (b) (100 – l) from B


(c) (l – 100) from A (d) (l – 100) from B
2. If the galvanometer and battery are interchanged at the balanced position, then
the balance point
(a) remains same
(b) get displace to right
(c) get displace to left
(d) direction get reverse

Sample Paper–3 (Unsolved) 503


D:\EG_Physics-12_(26-06-2022)\Open_Files\Sample Paper_3_(Unsolved)\Sample Paper_3_(Unsolved)
\ 27-Jul-2022 Ved_Goswami Proof-4 Reader’s Sign _______________________ Date __________

3. A metre bridge experimental set up is shown in figure, the null point


D is obtained at a distance of 40 cm from end A of the metre bridge
wire. If a resistance of 10W is connected in series with R1, null point
is obtained at AD = 60 cm then the values of R1 and R2 will be
(a) R1 = 4 W, R2 = 5 W
(b) R1 = 6 W, R2 = 4 W
(c) R1 = 8 W, R2 = 16 W
(d) R1 = 8 W, R2 = 12 W
4. In a metre bridge, the null point is found at a distance of 40 cm from A. If
a resistance of 12 W is connected in parallel with S, then null point occurs
at 50.0 cm from A. Ratio of R and S will be
(a) 4 : 5 (b) 2 : 3
(c) 2 : 5 (d) 4 : 3
5. In a metre bridge, the null point is found at a distance of 60 cm from A. If a
resistance of 5 W is connected in series with S, then null point occurs at 50.0
cm from A. Relation between R and S will be
5
(a) R = 2 S
7
(b) R = 2 S
3
(c) R = 2 S
3
(d) S = 2 R
Q16. In resonance, amplitude of a mechanical oscillation becomes maximum when the
frequency of driving force becomes equals to the natural frequency of system.
ʻResonanceʼ is also possible in some specific electric circuits series LCR circuit
is considered important because it can exhibit ʻresonanceʼ which makes it very
useful in designing tuning circuits. We know that peak current (I0) in a series LCR
ac circuit is related to peak volt across the circuit by the relation
V
I0 = Z0

where Z = R 2 + (X L − XC) 2 = R 2 + bωL − ωC l is the impedance of the circuit


1 2

It is clear that impedance depends on frequency, Z = Z(w). Hence, for a given V0,
peak current (I0) will be different for different frequencies. For a certain frequency
w0, impedance Z become minimum so that peak current (amplitude) I0 will have
a maximum value compared to other frequencies. This is called ʻResonanceʼ in
LCR series circuit and w0 in the resonant frequency. We can easily find the value
of frequency w0 for which Z will have a minimum value so that it is also then
possible to determine the maximum value amplitude (peak current). 4
Consider a series LCR circuit connected to an AC source of constant peak voltage
5 50
V0 and variable frequency w0 . Value of L, C and R are given as L = H, C = π mF and R = 400 W for value of
π
w = w1. For another value of w = w2, rms voltage across the resistance is 50 volts.
1. Voltage (rms) across the resistor (VR) is
(a) 80 V (b) 100 V (c) 90 V (d) 110 V
2. Voltage (rms) across the capacitor (VC) is
(a) 12 V (b) 20 V (c) 15 V (d) 40 V
504 Physics–12
D:\EG_Physics-12_(26-06-2022)\Open_Files\Sample Paper_3_(Unsolved)\Sample Paper_3_(Unsolved)
\ 27-Jul-2022 Ved_Goswami Proof-4 Reader’s Sign _______________________ Date __________

3. Voltage (rms) across the inductor (VL) is


(a) 50 V (b) 25 V (c) 100 V (d) 12.5 V
4. Frequency w1 is nearly equal to
(a) 324.2 rad/s (b) 312 rad/s (c) 225 rad/s (d) 625 rad/s
5. Frequency w2 is nearly equal to
(a) 50 rad/s (b) 36 rad/s (c) 73 rad/s (d) 18 rad/s

Section-C
All questions are compulsory. In case of internal choices, attempt any one of them.
Q17. Derive an expression for the magnetic moment (µ) of an electron revolving around the nucleus in terms of its
angular momentum (L) . What is the direction of the magnetic moment of the electron with respect to its angular
momentum? 2
OR
A certain proton moving through a magnetic field region experiences maximum force. When does this
occur?
Q18. What is the ratio of radii of the orbits corresponding to first excited state and ground state in a hydrogen atom?2
OR
The Rydberg constant for hydrogen is 10967700 m−1. Calculate the short and long wavelength limits of Lyman
series.
Q19. State the conditions for the phenomenon of total internal reflection to occur. 2
OR
A convex lens of refractive index 1.5 has a focal length of 18 cm in air. Calculate the change in its focal length
when it is immersed in water of refractive index 4/3.
Q20. Net capacitance of three identical capacitors in series is 1mF. What will be the net capacitance if connected in
parallel? Find the ratio of energy stored in two configurations if they are both connected to same source. 2
125 27
Q21. What is the nuclear radius of Fe , if that of Al is 3.6 fermi. 2
Q22. Define the term atomic mass unit (1 u). Find out its value in kilogram. 2
Q23. Draw a circuit diagram of an illuminated photodiode in reverse bias. How is photodiode used to measure light
intensity? 2
Q24. Arrange the following electromagnetic radiations in ascending order of their frequencies:
(i) Microwave (ii) Radio wave
(iii) X-rays (iv) Gamma rays
Write two uses of any one of these.  2
Q25. An electric lamp having coil of negligible inductance connected in series with a capacitor
and an AC source is glowing with certain brightness. How does the brightness of the lamp
change on reducing the (i) capacitance, and (ii) the frequency? Justify your answer. 2

Section-D
All questions are compulsory. In case of internal choices, attempt any one of them.
Q26. The radii of curvature of the faces of a double convex lens are 10 cm and 15 cm. If focal length of the lens is 12
cm, find the refractive index of the material of the lens. 3
OR
Two lenses of power 10 D and – 5 D are placed in contact.
(i) Calculate the power of lens combination.
(ii) Where should an object be held from the lens, so as to obtain a virtual image of magnification 2?

Sample Paper–3 (Unsolved) 505


D:\EG_Physics-12_(26-06-2022)\Open_Files\Sample Paper_3_(Unsolved)\Sample Paper_3_(Unsolved)
\ 27-Jul-2022 Ved_Goswami Proof-4 Reader’s Sign _______________________ Date __________

Q27. Distinguish between nuclear fission and fusion. Show how in both these processes energy is released. Calculate
the energy release in MeV in the deuterium-tritium fusion reaction: 3
2 3 4 1
1 H + 1 H " 2 He + 0 n
Using the data:
m ^12 H h = 2.014102 u; m ^13 H h = 3.016049 u
m ^24 He h = 4.002603 u; mn = 1.008665 u; 1u = 931.5 MeV/c2
OR
In the deuterium-tritium fusion reaction find the rate at which deuterium and tritium are consumed to produce 1
MW. The Q-value of deuterium-tritium reaction is 17.6 MeV. You can assume that the efficiency is 100%.
Q28. (a) Depict the equipotential surfaces for a system of two identical positive point charges placed at a distance d
apart.
(b) Deduce the expression for the potential energy of a system of two point charges q1 and q2 brought from infinity
to the points r1 and r2 respectively in the presence of external electric field Ε .  3
Q29 Write the principle of working of a ‘Metre bridge’. Describe briefly, with the help of a circuit diagram, how a
‘Metre bridge’ is used to determine the unknown resistance. 3
Q30. (i) How is the focal length of a spherical mirror affected when it is immersed in water?
(ii) A convex lens has 10 cm focal length in air. What is its focal length in water?
(refractive index of air-water = 1.33, refractive index of air-glass = 1.5). 3

Section-E
All questions are compulsory. In case of internal choices, attempt any one of them.
Q31. Describe briefly, with the help of a labelled diagram, the basic elements of an A.C. generator. State its underlying
principle. Show diagrammatically how an alternating emf is generated by a loop of wire rotating in a magnetic
field. Write the expression for the instantaneous value of the emf induced in the rotating loop. 5
OR
A series LCR circuit is connected to an ac source having voltage V = Vm sin wt. Derive the expression for the
instantaneous current ‘i’ and its phase relationship to the applied voltage.
Obtain the condition for resonance to occur. Define ‘power factor’. Sate the conditions under which it is (i) maximum
and (ii) minimum.
Q32. State Huygens principle. Show, with the help of a suitable diagram, how this principle is used to obtain the diffraction
pattern by a single slit.
Draw a plot of intensity distribution and explain clearly why the secondary maxima become weaker with increasing
order (n) of the secondary maxima. 5
OR
Draw a ray diagram to show the working of a compound microscope. Deduce an expression for the total magnification
when the final image is formed at the near point. In a compound microscope, an object is placed at a distance of
1.5 cm from the objective of focal length 1.25 cm. If the eye-piece has a focal length of 5 cm and the final image
is formed at the near point, estimate the magnifying power of the microscope.
Q33. (a) Explain the formation of depletion layer and potential barrier in a p-n junction output.
(b) In the figure given below the input waveform is converted into the output waveform by a device ‘X’. Name
the device and draw its circuit diagram. 5

Device ‘X’

Input Output

506 Physics–12
D:\EG_Physics-12_(26-06-2022)\Open_Files\Sample Paper_3_(Unsolved)\Sample Paper_3_(Unsolved)
\ 27-Jul-2022 Ved_Goswami Proof-4 Reader’s Sign _______________________ Date __________

(c) What is an ideal diode? Draw the output waveform across R for the input waveform given here.

OR
(a) Distinguish between intrinsic and extrinsic semi-conductors on the basis of energy band diagram.
(b) How does the change in temperature affect the behaviour of these materials? Explain briefly. 

Answers
1. (a) 2. (b) 3. (c) 4. (b)
q
5. Same for all the figures OR 6. OR Line-A 7. D1 < D2 OR 1.45
∈°
8. Nuclei are positively charged 9. Nucleons 10. (i) decreases (ii) increases 11. (b)

12. (c) 13. (c) 14. (c)

15. 1. (a) 2. (a) 3. (d) 4. (b) 5. (c)

16. 1. (a) 2. (d) 3. (c) 4. (a) 5. (c)


 
17. direction of µ is of right to L . OR at q = 90°

18. 1 : 4 OR lshort = 911.6 Å, llong = 1215 Å 19. OR 32 cm 20. 1 : 9 21. 6.0 fermi
22. 1.66 × 10–27 kg 24. (i) decreases (ii) decreases
25. Radio wave < Microwaves < X-rays < Gamma rays
26. µ = 1.5 OR (i) 5 D (ii) –10 cm 27. 17.589 MeV OR 1.768 × 10–9 kg/s
30. (i) focal length is not affected (ii) 39.1 cm
31. OR (i) when cos f = 1 (ii) when cos f = 0 32. OR m = –30
33. (b) Full wave rectifier

rrr

Sample Paper–3 (Unsolved) 507

You might also like